๋กœ๊ทธ์ธ


+ -   ๋ถ๋งˆํฌ ์ €์žฅ [ํ˜•์‚ฌ] 1์‹ฌํ•ฉ์˜์‚ฌ๊ฑด

์„œ์šธ์ค‘์•™์ง€๋ฐฉ๋ฒ•์› 2016. 12. 15. ์„ ๊ณ  2016๊ณ ํ•ฉ538, 558(๋ณ‘ํ•ฉ) ํŒ๊ฒฐ

[๊ตญ๊ฐ€๋ณด์•ˆ๋ฒ•์œ„๋ฐ˜(์ฐฌ์–‘ยท๊ณ ๋ฌด๋“ฑ)ยท๊ตญ๊ฐ€๋ณด์•ˆ๋ฒ•์œ„๋ฐ˜(ํšŒํ•ฉยทํ†ต์‹ ๋“ฑ)ยท๊ตญ๊ฐ€๋ณด์•ˆ๋ฒ•์œ„๋ฐ˜(ํŽธ์˜์ œ๊ณต)ยท๊ตญ๊ฐ€๋ณด์•ˆ๋ฒ•์œ„๋ฐ˜(์ž์ง„์ง€์›ยท๊ธˆํ’ˆ์ˆ˜์ˆ˜)][๋ฏธ๊ฐ„ํ–‰] ใ€์ „ ๋ฌธใ€‘ ใ€ํ”ผ ๊ณ  ์ธใ€‘ ํ”ผ๊ณ ์ธ ใ€๊ฒ€ ์‚ฌใ€‘ ๊น€์ฃผํ•„, ์ฐจ์ˆœ๊ธธ(๊ฐ ๊ธฐ์†Œ), ์ฐจ์ˆœ๊ธธ, ์ตœ์ฐฝ๋ฏผ, ์ด์žฌ๋งŒ, ๊ตญ์ƒ์šฐ, ์„œ์„ฑ๋ชฉ(๊ฐ ๊ณตํŒ) ใ€๋ณ€ ํ˜ธ ์ธใ€‘ ๋ฒ•๋ฌด๋ฒ•์ธ ์ƒ๋ก ์™ธ 6์ธ ใ€์ฃผ ๋ฌธใ€‘ ํ”ผ๊ณ ์ธ์„ ์ง•์—ญ 4๋…„ ๋ฐ ์ž๊ฒฉ์ •์ง€ 4๋…„์— ์ฒ˜ํ•œ๋‹ค. [๋ณ„์ง€ 1] ๋ชฐ์ˆ˜ ๋Œ€์ƒ ์••์ˆ˜๋ฌผ ๋ชฉ๋ก ๊ธฐ์žฌ ์••์ˆ˜๋ฌผ๋“ค ์ค‘ ์ˆœ๋ฒˆ 1 ๋‚ด์ง€ 18, 31์„ ๊ฐ ๋ชฐ์ˆ˜ํ•œ๋‹ค. ์ด ์‚ฌ๊ฑด ๊ณต์†Œ์‚ฌ์‹ค ์ค‘ 2013. 11. 2. ์ด์ ํ‘œํ˜„๋ฌผ ์†Œ์ง€๋กœ ์ธํ•œ ๊ตญ๊ฐ€๋ณด์•ˆ๋ฒ•์œ„๋ฐ˜(์ฐฌ์–‘ยท๊ณ ๋ฌด๋“ฑ)์˜ ์ ์€ ๋ฌด์ฃ„. ใ€์ด ์œ ใ€‘ ใ€๋ฒ”์ฃ„์‚ฌ์‹คใ€‘ โ… . 2016๊ณ ํ•ฉ538 1. ํ”ผ๊ณ ์ธ์˜ ๊ฒฝ๋ ฅ ํ”ผ๊ณ ์ธ์€ 1986. 2.๊ฒฝ โ–ณโ–ณ๋Œ€ํ•™๊ต (๋ช…์นญ 23 ์ƒ๋žต)ํ•™๊ณผ๋ฅผ, 1990. 2.๊ฒฝ โ–ณโ–ณ๋Œ€ํ•™๊ต (๋ช…์นญ 24 ์ƒ๋žต)๋Œ€ํ•™์›์„ ์กธ์—…ํ•˜๊ณ , 1992. 3.๊ฒฝ (๋ช…์นญ 25 ์ƒ๋žต)๊ต๋กœ๋ถ€ํ„ฐ ๋ชฉ์‚ฌ ์•ˆ์ˆ˜๋ฅผ ๋ฐ›์€ ์ดํ›„, 1999๋…„๊ฒฝ ์ž์‹ ์ด ์„ค๋ฆฝํ•œ โ€˜(๋ช…์นญ 26 ์ƒ๋žต)โ€™์˜ ๋ชฉ์‚ฌ๋ฅผ ๊ฑฐ์ณ โ€˜(๋ช…์นญ 27 ์ƒ๋žต)โ€™์˜ ๋Œ€์™ธํ˜‘๋ ฅ์œ„์›์žฅ, โ€˜(๋ช…์นญ 28 ์ƒ๋žต)โ€™์˜ ์กฐ์ง์ •์ฑ…๊ตญ์žฅ, โ€˜(๋ช…์นญ 29 ์ƒ๋žต)์—ฐ๊ตฌ์†Œโ€™์˜ ์šด์˜์œ„์›์žฅ, โ€˜(๋ช…์นญ 30 ์ƒ๋žต)์—ฐ๋Œ€โ€™์˜ ์ง‘ํ–‰์œ„์›์žฅ ๋“ฑ์œผ๋กœ ํ™œ๋™ํ•˜์˜€๋‹ค. ํ”ผ๊ณ ์ธ์€ 1989. 9.๊ฒฝ๋ถ€ํ„ฐ ๊ธฐ๋…๊ต์—ฐํ•ฉ์‹ ๋ฌธ โ€˜(๋ช…์นญ 31 ์ƒ๋žต)โ€™ ๊ธฐ์ž๋กœ ํ™œ๋™ํ•˜๋˜ ์ค‘ 1991๋…„๊ฒฝ ๋ฐ˜๊ตญ๊ฐ€๋‹จ์ฒด์ธ โ€˜๋‚จํ•œ์กฐ์„ ๋…ธ๋™๋‹นโ€™ ๊ฐ•์›๋„๋‹น ์ง€๋„์ฑ… ๊ณต์†Œ์™ธ 32์—๊ฒŒ ํฌ์„ญ๋˜์–ด ์ „์œ„์กฐ์ง์ธ โ€˜(๋ช…์นญ 32 ์ƒ๋žต)โ€™์˜ ์–ธ๋ก ๋‹ด๋‹น์œผ๋กœ ํ™œ๋™ํ•œ ์ „๋ ฅ์ด ์žˆ๊ณ , ๊ทธ ์ดํ›„ โ€˜(๋ช…์นญ 33 ์ƒ๋žต)์—ฐ๋Œ€โ€™์˜ ๋Œ€๋ณ€์ธ, โ€˜(๋ช…์นญ 9 ์ƒ๋žต)โ€™์˜ ์ค‘์•™์œ„์› ๋“ฑ์œผ๋กœ ํ™œ๋™ํ•˜์˜€๋‹ค. ๋˜ํ•œ, ํ”ผ๊ณ ์ธ์€ 2008. 9.๊ฒฝ๋ถ€ํ„ฐ 2010. 9.๊ฒฝ๊นŒ์ง€ โ€˜(๋ช…์นญ 34 ์ƒ๋žต)โ€™์˜ ์ค‘์•™๋‹น๊ธฐ์œ„์›์žฅ, 2012. 9.๊ฒฝ๋ถ€ํ„ฐ 2013. 2.๊ฒฝ๊นŒ์ง€ โ€˜(๋ช…์นญ 17 ์ƒ๋žต)๋‹นโ€™์˜ โ—ˆโ—ˆ๊ตฌ์œ„์›ํšŒ ์‚ฌ๋ฌด๊ตญ์žฅ ๋Œ€ํ–‰์œผ๋กœ ํ™œ๋™ํ•˜์˜€๊ณ , ์ตœ๊ทผ โ€˜(๋ช…์นญ 35 ์ƒ๋žต)โ€™์˜ ๋Œ€ํ‘œ, โ€˜(๋ช…์นญ 36 ์ƒ๋žต)โ€™์˜ ์ด์‚ฌ, ์ธํ„ฐ๋„ท์‹ ๋ฌธ โ€˜(๋ช…์นญ 13 ์ƒ๋žต)โ€™์˜ ๋ฐœํ–‰์ธ ๊ฒธ ํŽธ์ง‘๊ตญ์žฅ์œผ๋กœ ํ™œ๋™ํ•˜๊ณ  ์žˆ๋‹ค. 2. ๋ถํ•œ์˜ ๋ฐ˜๊ตญ๊ฐ€๋‹จ์ฒด์„ฑ ๋ถํ•œ๊ณต์‚ฐ์ง‘๋‹จ์€ ์กฐ์„ ๋…ธ๋™๋‹น ๊ทœ์•ฝ ์„œ๋ฌธ์—์„œ โ€œ์กฐ์„ ๋…ธ๋™๋‹น์˜ ๋‹น๋ฉด ๋ชฉ์ ์€ ๊ณตํ™”๊ตญ ๋ถ๋ฐ˜๋ถ€์—์„œ ์‚ฌํšŒ์ฃผ์˜ ๊ฐ•์„ฑ๋Œ€๊ตญ์„ ๊ฑด์„คํ•˜๋ฉฐ ์ „๊ตญ์  ๋ฒ”์œ„์—์„œ ๋ฏผ์กฑํ•ด๋ฐฉ ๋ฏผ์ฃผ์ฃผ์˜ ํ˜๋ช…๊ณผ์—…์„ ์ˆ˜ํ–‰ํ•˜๋Š”๋ฐ ์žˆ์œผ๋ฉฐ, ์ตœ์ข… ๋ชฉ์ ์€ ์˜จ ์‚ฌํšŒ๋ฅผ ๊น€์ผ์„ฑ-๊น€์ •์ผ์ฃผ์˜ํ™”ํ•˜์—ฌ ์ธ๋ฏผ๋Œ€์ค‘์˜ ์ž์ฃผ์„ฑ์„ ์™„์ „ํžˆ ์‹คํ˜„ํ•˜๋Š”๋ฐ ์žˆ๋‹คโ€๊ณ  ๊ทœ์ •ํ•˜๋Š” ๋“ฑ ๊น€์ผ์„ฑ ๋…์žฌ์‚ฌ์ƒ(์ด๋ฅธ๋ฐ” โ€˜์ฃผ์ฒด์‚ฌ์ƒโ€™)์— ๊ธฐ์ดˆํ•œ ํ•œ๋ฐ˜๋„ ์ ํ™”ํ†ต์ผ์„ ๊ธฐ๋ณธ๋ชฉํ‘œ๋กœ ์„ค์ •ํ•˜๊ณ  ์žˆ๋‹ค. ๋˜ํ•œ, ๋ถํ•œ์€ ์ด๋ฅธ๋ฐ” โ€˜๋Œ€๋‚จํ˜๋ช… ์ „๋žตโ€™์— ๋”ฐ๋ผ ๋ฏธ ์ œ๊ตญ์ฃผ์˜์— ์˜ˆ์†๋œ ๋‚จํ•œ ๋ฏผ์ค‘์„ ํ•ด๋ฐฉํ•˜๊ณ  ํ•œ๋ฐ˜๋„์˜ ํ†ต์ผ์„ ์‹คํ˜„ํ•˜๊ธฐ ์œ„ํ•ด ๋‚จํ•œ ๋ฏผ์ค‘์„ ์ฃผ์ฒด์‚ฌ์ƒ์œผ๋กœ ์˜์‹ํ™”ํ•˜์—ฌ โ€˜์ž์ฃผยท๋ฏผ์ฃผยทํ†ต์ผ ํˆฌ์Ÿโ€™์„ ์ „๊ฐœํ•ด์•ผ ํ•œ๋‹ค๊ณ  ์„ ์ „ํ•˜๊ณ  ์žˆ๊ณ , ๊น€์ •์ผ ์„ธ์Šต ์ดํ›„์—๋Š” โ€˜๊ตฐ์‚ฌ์„ ํ–‰์˜ ์›์น™์—์„œ ํ˜๋ช…๊ณผ ๊ฑด์„ค์— ๋‚˜์„œ๋Š” ๋ชจ๋“  ๋ฌธ์ œ๋ฅผ ํ’€์–ด๋‚˜๊ฐ€๋ฉฐ ์ธ๋ฏผ๊ตฐ๋Œ€๋ฅผ ํ˜๋ช…์˜ ๊ธฐ๋‘ฅ์œผ๋กœ ๋‚ด์„ธ์›Œ ์‚ฌํšŒ์ฃผ์˜ ์œ„์—… ์ „๋ฐ˜์„ ๋ฐ€๊ณ  ๋‚˜๊ฐ€๋Š” ์ •์น˜โ€™๋ผ๋Š” ์ด๋ฅธ๋ฐ” โ€˜์„ ๊ตฐ์ •์น˜โ€™๋ฅผ ๋‚ด์„ธ์šฐ๋ฉฐ, ์ด๋ฅผ ํ†ตํ•ด ๊ตฌ์ถ•๋œ ๋ถํ•œ์˜ ๋ง‰๊ฐ•ํ•œ ์ž์œ„์  ๊ตญ๋ฐฉ๋ ฅ์ด ํ•œ๋ฐ˜๋„์˜ ํ‰ํ™”์™€ ์•ˆ์ „์„ ๋‹ด๋ณดํ•˜๊ณ , ๋‚จํ•œ์„ ๊ฐ•์ ํ•˜๊ณ  ์žˆ๋Š” ๋ฏธ๊ตฐ์„ ์ฒ ์ˆ˜์‹œ์ผœ ์ „๊ตญ์  ๋ฒ”์œ„์—์„œ ๋ฏผ์กฑ ์ž์ฃผ๊ถŒ์„ ํ™•๋ฆฝํ•˜์—ฌ ์ž์ฃผํ†ต์ผ ์œ„์—…์„ ์„ฑ์ทจํ•  ์ˆ˜ ์žˆ๊ฒŒ ๋˜์—ˆ๋‹ค๊ณ  ์„ ์ „ยท์„ ๋™ํ•˜๊ณ  ์žˆ๋‹ค. ์ด์— ๋”ฐ๋ผ ๋ถํ•œ์€ 1991. 9. 17. ๋Œ€ํ•œ๋ฏผ๊ตญ๊ณผ ์œ ์—”์— ๋™์‹œ ๊ฐ€์ž…ํ•˜์˜€๊ณ , 1991. 12. 13. ์ด๋ฅธ๋ฐ” ๋‚จ๋ถ๊ณ ์œ„๊ธ‰ํšŒ๋‹ด์—์„œ ๋‚จ๋ถ๊ธฐ๋ณธํ•ฉ์˜์„œ๋ฅผ ์ฑ„ํƒํ•˜์˜€์œผ๋ฉฐ, 2000. 6. 15. ๋ฐ 2007. 10. 4. ๋‘ ์ฐจ๋ก€์— ๊ฑธ์ณ ๋‚จ๋ถ์ •์ƒํšŒ๋‹ด์„ ๊ฐœ์ตœํ•˜๋Š” ๋“ฑ ๋Œ€ํ•œ๋ฏผ๊ตญ๊ณผ ์ •์น˜ยท๊ฒฝ์ œยท์‚ฌํšŒยท๋ฌธํ™” ๋“ฑ ๊ฐ๊ณ„๊ฐ์ธต์—์„œ ๊ต๋ฅ˜์™€ ํ˜‘๋ ฅ์ด ์ด๋ฃจ์–ด์ง€๋Š” ์™€์ค‘์—๋„, 1999. 6. 15. ๋ฐ 2002. 6. 29. ์ œ1์ฐจ, ์ œ2์ฐจ ์—ฐํ‰ํ•ด์ „, 2006. 10. 9. ๋ฐ 2009. 5. 27. ์ œ1ยท2์ฐจ ํ•ต์‹คํ—˜, 2010. 3. 26. ์ฒœ์•ˆํ•จ ํญ์นจ, 2010. 11. 23. ์—ฐํ‰๋„ ํฌ๊ฒฉ ๋“ฑ์˜ ๋ฌด๋ ฅ๋„๋ฐœ์„ ๊ฐํ–‰ํ•˜์˜€๊ณ , ํŠนํžˆ ์ตœ๊ทผ์—๋Š” 2013. 2. 12. ์ œ3์ฐจ ํ•ต์‹คํ—˜, 2013. 3. 5. ์ •์ „ํ˜‘์ • ๋ฐฑ์ง€ํ™” ์„ ์–ธ, 2016. 1. 6. ์ œ4์ฐจ ํ•ต์‹คํ—˜ ๋“ฑ ํ•œ๋ฐ˜๋„ ์ „์Ÿ์œ„ํ—˜์„ ํ•œ์ธต ๊ณ ์กฐ์‹œ์ผœ ๋Œ€ํ•œ๋ฏผ๊ตญ์˜ ์กด๋ฆฝยท์•ˆ์ „์„ ์œ„ํƒœ๋กญ๊ฒŒ ํ•˜๊ณ , 225๊ตญยท์ •์ฐฐ์ด๊ตญยทํ†ต์ผ์ „์„ ๋ถ€ ๋“ฑ ๊ฐ์ข… ๋Œ€๋‚จ๊ณต์ž‘์กฐ์ง์„ ํ†ตํ•ด ๋Œ€ํ•œ๋ฏผ๊ตญ์˜ ๊ธฐ๋ฐ€ ํƒ์ง€, ์ง€ํ•˜๋‹น ์กฐ์ง, ์‚ฌ์ด๋ฒ„ ํ…Œ๋Ÿฌยท์„ ์ „์ „ ๋“ฑ์„ ์ง€์†์ ์œผ๋กœ ๊ฐํ–‰ํ•˜๋Š” ๋“ฑ ๋Œ€ํ•œ๋ฏผ๊ตญ์˜ ์ž์œ ๋ฏผ์ฃผ์  ๊ธฐ๋ณธ์งˆ์„œ๋ฅผ ์ „๋ณตํ•˜๊ณ ์ž ํ•˜๋Š” ์ ํ™”ํ†ต์ผ ๋…ธ์„ ์„ ์œ ์ง€ํ•˜๊ณ  ์žˆ๋‹ค. ํ•œํŽธ, ๋ถํ•œ์€ 2000. 6. 15. ๋ฐ 2007. 10. 4. ๋‘ ์ฐจ๋ก€์— ๊ฑธ์ณ ๋‚จ๋ถ์ •์ƒํšŒ๋‹ด์ด ๊ฐœ์ตœ๋˜๊ณ  ๋‚จ๋ถ๊ณต๋™์„ ์–ธ๋ฌธ์ด ๋ฐœํ‘œ๋˜์ž ์œ„ ์„ ์–ธ์˜ ์ดํ–‰์„ ์ฃผ์žฅํ•˜๋ฉฐ โ€˜์šฐ๋ฆฌ๋ฏผ์กฑ๋ผ๋ฆฌโ€™, โ€˜๋‚ฎ์€ ๋‹จ๊ณ„์˜ ์—ฐ๋ฐฉ์ œโ€™ ๋“ฑ์˜ ์šฉ์–ด๋ฅผ ์•ž์„ธ์›Œ ์™ธ์„ธ์ธ ๋ฏธ๊ตญ์˜ ์ง€๋ฐฐ์™€ ๊ฐ„์„ญ ๋ฐฐ์ œ, ์ฃผํ•œ๋ฏธ๊ตฐ์ฒ ์ˆ˜, ํ•œ๋ฏธ๋™๋งน ํ๊ธฐ, ๋ฐ˜๋ฏธ ๋ฏผ์กฑ๊ณต์กฐ, ๋ฐ˜ํ†ต์ผ ๋ฐ ์‚ฌ๋Œ€๋งค๊ตญ์„ธ๋ ฅ ์ฒ™๊ฒฐ, ๊ตญ๊ฐ€๋ณด์•ˆ๋ฒ• ํ์ง€, ์—ฐ๋ฐฉ์ œ ํ†ต์ผ ๋“ฑ ์†Œ์œ„ ๋ฐ˜๋ฏธ์ž์ฃผํ™”, ๋ฐ˜ํŒŒ์‡ผ๋ฏผ์ฃผํ™”, ์กฐ๊ตญํ†ต์ผ ํˆฌ์Ÿ์„ ๋Š์ž„์—†์ด ์„ ๋™ํ•˜๊ณ  ์žˆ๋‹ค. 3. ์ด์ ํ‘œํ˜„๋ฌผ ์†Œ์ง€ ๊ด€๋ จ ๋ฒ”์ฃ„์‚ฌ์‹ค ๊ฐ€. ๋งˆ์ดํฌ๋กœ ์—์Šค๋””(Micro SD 16G) ํ”ผ๊ณ ์ธ์€ 2015. 11. 13.๊ฒฝ ๊ด‘๋ช…์‹œ (์ฃผ์†Œ 3 ์ƒ๋žต)์— ์žˆ๋Š” ํ”ผ๊ณ ์ธ์˜ ์ฃผ๊ฑฐ์ง€์—์„œ ๋ถํ•œ ๊น€์ผ์„ฑ ํšŒ๊ณ ๋ก ๋“ฑ ๊ฐ์ข… ํŒŒ์ผ๋“ค์„ ์ฒด๊ณ„์ ์œผ๋กœ ์ •๋ฆฌยท์ €์žฅํ•œ ๋งˆ์ดํฌ๋กœ ์—์Šค๋””(16G) ํ˜•์‹์˜ ๋””์ง€ํ„ธ์ €์žฅ๋งค์ฒด๋ฅผ ์ž์‹ ์ด ๊ฐ€์ง€๊ณ  ๋‹ค๋‹ˆ๋˜ ์ง€๊ฐ‘ ์•ˆ์— ๋ณด๊ด€ํ•˜๊ณ  ์žˆ์—ˆ๋‹ค. ์œ„ ์ €์žฅ๋งค์ฒด๋Š” ๊น€์ผ์„ฑ ํšŒ๊ณ ๋ก์ธ โ€˜์„ธ๊ธฐ์™€ ๋”๋ถˆ์–ดโ€™์˜ ๋‚ญ๋… ์Œ์„ฑ ํŒŒ์ผ์ด ์ €์žฅ๋˜์–ด ์žˆ๋Š” ใ€Œ[mp3]ํšŒ๊ณ ใ€ ํด๋”, โ€˜์šฐ๋ฆฌ ์„ธ๋Œ€๊ฐ€ ๊ฒฐ์‚ฐํ•  ๊ฒƒ์ด๋‹คโ€™ ๋“ฑ ๋ถํ•œ์˜ ์ฒด์ œ์ •๋‹น์„ฑ ๋ฐ ๋ฐ˜๋ฏธ์‚ฌ์ƒ์„ ์„ ์ „ํ•˜๋Š” ์ž๋ฃŒ๋“ค์ด ์ €์žฅ๋˜์–ด ์žˆ๋Š” ใ€Œ๊นŒ์น˜ใ€ํด๋”, ใ€Œ๋™์–‘๊ณ ์ „ใ€ํด๋” ๋“ฑ ์ด 3๊ฐœ์˜ ์ƒ์œ„ ํด๋”๋กœ ๊ตฌ๋ถ„๋˜์–ด ์žˆ๋‹ค. ๊ทธ ์ƒ์œ„ ํด๋” ์ค‘ ใ€Œ๋™์–‘๊ณ ์ „ใ€ํด๋”๋Š” ๋‹ค์‹œใ€Œ์‚ผ๊ตญ์ง€ใ€ยทใ€Œ์ˆ˜ํ˜ธ์ง€ใ€๋“ฑ์˜ ํ•˜์œ„ ํด๋”๋กœ ๋‚˜๋ˆ„์–ด์ ธใ€Œ์‚ผ๊ตญ์ง€ใ€ํ•˜์œ„ ํด๋”์—๋Š” โ€˜์‚ผ์ฒœ๋ฆฌ ๊ฐ•์‚ฐโ€™ยทโ€˜์• ๊ตญ์‹œ๋Œ€โ€™ ๋“ฑ ๋ถํ•œ์ฒด์ œ ๋ฐ ๊น€์ผ์„ฑ์„ ์ฐฌ์–‘ํ•˜๋Š” ๋ถํ•œ ์†Œ์„ค์ด ์ €์žฅ๋˜์–ด ์žˆ์œผ๋ฉฐ, ใ€Œ์ˆ˜ํ˜ธ์ง€ใ€ํ•˜์œ„ ํด๋”์—๋Š” ๊น€์ผ์„ฑ์˜ ํ•ญ์ผ์šด๋™์„ ๋ฏธํ™”ยท์ฐฌ์–‘ํ•˜๋Š” ๋‚ด์šฉ์˜ ๋ถํ•œ ์›์ „ โ€˜๋ถ‰์€ ํ•ด๋ฐœ์•„๋ž˜ ํ•ญ์ผํ˜๋ช… 20๋…„(1~5๊ถŒ)โ€™ ๋“ฑ์ด ์ €์žฅ๋˜์–ด ์žˆ๋‹ค. ์œ„ ์ €์žฅ๋งค์ฒด์— ์ˆ˜๋ก๋œ ํŒŒ์ผ๋“ค์˜ ์ฃผ์š” ๋‚ด์šฉ์€ ๋‹ค์Œ๊ณผ ๊ฐ™๋‹ค. ใ€Š[mp3]ํšŒ๊ณ  ํด๋” ๋‚ด โ€˜์„ธ๊ธฐ์™€ ๋”๋ถˆ์–ดโ€™(์Œ์„ฑํŒŒ์ผ)์˜ ์ฃผ์š” ๋‚ด์šฉ ๋“ฑใ€‹ - ๋ถํ•œ ๊น€์ผ์„ฑ์˜ ํ•ญ์ผํˆฌ์Ÿ์„ ์„ ์ „ํ•˜๋Š” ๋“ฑ ๋ถํ•œ์˜ ํ˜๋ช…์  ์ •ํ†ต์„ฑ์„ ๊ฐ•์กฐ, ๋ฏธํ™”ํ•˜๋Š” ๋‚ด์šฉ์˜ ๊น€์ผ์„ฑ ํšŒ๊ณ ๋ก์ธ โ€˜์„ธ๊ธฐ์™€ ๋”๋ถˆ์–ด(1~8๊ถŒ)โ€™์— ๋Œ€ํ•˜์—ฌ, 10~30๋ถ„ ๊ฐ€๋Ÿ‰์˜ ์žฌ์ƒ์‹œ๊ฐ„์œผ๋กœ ๋‚˜๋ˆ„์–ด ๋‚ญ๋… ๋…น์Œ๋œ ์—ฌ๋Ÿฌ ๊ฐœ์˜ ์Œ์„ฑ ํŒŒ์ผ๋“ค๋กœ ๊ตฌ์„ฑ๋˜์–ด ์žˆ๋‹ค. ใ€Š๋™์–‘๊ณ ์ „ ํด๋” ์ค‘ ์‚ผ๊ตญ์ง€ ํด๋” ๋‚ด โ€˜์‚ผ์ฒœ๋ฆฌ ๊ฐ•์‚ฐโ€™์˜ ์ฃผ์š” ๋‚ด์šฉใ€‹ - ํ•ด๋ฐฉ ํ›„ ๊น€์ผ์„ฑ์˜ ์ •์น˜ยท๊ฒฝ์ œยท์‚ฌํšŒ ๋“ฑ ๊ฐ ๋ถ„์•ผ์˜ ํ™œ๋™ ๋…ธ์ • ๋ฐ ๋ถํ•œ ํƒ„์ƒ๊ณผ์ •์„ ์†Œ๊ฐœํ•˜๋ฉด์„œ ๊น€์ผ์„ฑ์˜ ํƒ์›”ํ•œ ์˜๋„๋ ฅ ๋ฐ ํฌ์šฉ๋ ฅ๊ณผ ์ธ๊ฐ„์ ์ธ ๋ฉด์„ ๊ฐ•์กฐ, ์šฐ์ƒํ™”ํ•˜๊ณ  ์ฐฌ์–‘ํ•˜๋Š” ๊ฒƒ์ด ์ฃผ์š” ๋‚ด์šฉ์ด๋‹ค. ใ€Š๋™์–‘๊ณ ์ „ ํด๋” ์ค‘ ์‚ผ๊ตญ์ง€ ํด๋” ๋‚ด โ€˜์• ๊ตญ์‹œ๋Œ€โ€™์˜ ์ฃผ์š” ๋‚ด์šฉใ€‹ - ๋‚จํ•œ์ง€์—ญ์—์„œ ๊น€์ผ์„ฑ์ฃผ์˜ ์ถ”์ข…์ž์˜ ํ™œ๋™์ƒ์„ ๋ฌ˜์‚ฌํ•˜์—ฌ ๋ฐ˜๋ฏธ, ์ฃผํ•œ๋ฏธ๊ตฐ์ฒ ์ˆ˜์™€ ์ธ๋ฏผ๋ฏผ์ฃผ์ฃผ์˜ ํ˜๋ช…์‚ฌ์ƒ์„ ๊ณ ์ทจํ•˜๋ฉฐ, ํ•œ๋ฐ˜๋„์˜ ๋ถํ•œ์‹ ์‚ฌํšŒ์ฃผ์˜ํ™” ์„ ๋™ ๋ฐ ๋ถํ•œ์˜ ์ˆ˜๋ น๊ด€์„ ๋งน๋ชฉ์ ์œผ๋กœ ์˜นํ˜ธํ•˜๋Š” ๋‚ด์šฉ์ด๋‹ค. ใ€Š๋™์–‘๊ณ ์ „ ํด๋” ์ค‘ ์ˆ˜ํ˜ธ์ง€ ํด๋” ๋‚ด โ€˜๋ถ‰์€ ํ•ด๋ฐœ ์•„๋ž˜ ํ•ญ์ผํ˜๋ช… 20๋…„ 1~5โ€™์˜ ์ฃผ์š” ๋‚ด์šฉใ€‹ - 1926. 6.๋ถ€ํ„ฐ 1945. 8. ๊ด‘๋ณต์— ์ด๋ฅด๊ธฐ๊นŒ์ง€ ๊น€์ผ์„ฑ์˜ ํ•ญ์ผํˆฌ์Ÿ์„ ์ฐฌ์–‘ยท๋ฏธํ™”ํ•˜๋ฉด์„œ โ€˜์˜์ƒ๋ถˆ๋ฉธ์˜ ์ฃผ์ฒด์‚ฌ์ƒ์„ ์ฐฝ์‹œํ•˜์‹ฌ์œผ๋กœ์จ ์šฐ๋ฆฌ ํ˜๋ช…์˜ ์ง€๋„์‚ฌ์ƒ์„ ๋งˆ๋ จํ•˜์‹œ์˜€์œผ๋ฉฐ ๊ทธ์— ๊ธฐ์ดˆํ•˜์‹œ์—ฌ ์กฐ์„ ํ˜๋ช…์˜ ์„ฑ๊ฒฉ๊ณผ ์ž„๋ฌด๋ฅผ ๊ณผํ•™์ ์œผ๋กœ ๊ทœ์ •ํ•˜์‹œ๊ณ  ํ•ญ์ผ๋ฌด์žฅํˆฌ์Ÿ ๋กœ์„ ์„ ๋น„๋กฏํ•œ ์กฐ์„ ํ˜๋ช…์˜ ์ฃผ์ฒด์  ๋กœ์„ ๊ณผ ์ „๋žต ์ „์ˆ ์  ๋ฐฉ์นจ๋“ค์„ ๋‚ด๋†“์œผ์‹œ์–ด ์กฐ์„ ํ˜๋ช…์˜ ์ง„๋กœ๋ฅผ ํœ˜ํ™ฉํžˆ ๋ฐํ˜€์ฃผ์‹œ์—ˆ๋‹ค.โ€™, - โ€˜์œ„๋Œ€ํ•œ ์ˆ˜๋ น ๊น€์ผ์„ฑ๋™์ง€๊ป˜์„œ ์กฐ์ง๋ น๋„ํ•˜์‹  ์˜๊ด‘์Šค๋Ÿฌ์šด ํ•ญ์ผํ˜๋ช…ํˆฌ์Ÿ์˜ ๋น›๋‚˜๋Š” ์Šน๋ฆฌ์˜ ๊ธธ ์œ„์—๋Š” ์กฐ๊ตญ์˜ ๋…๋ฆฝ๊ณผ ์ธ๋ฏผ์˜ ์ž์œ ์™€ ํ–‰๋ณต์„ ์œ„ํ•˜์—ฌ ๊ทธ๋ฆฌ๊ณ  ์ฒซ ์‚ฌํšŒ์ฃผ์˜๊ตญ๊ฐ€์ธ ์˜๋ จ์„ ๋ฌด์žฅ์œผ๋กœ ์˜นํ˜ธํ•˜๋ฉฐ ํ˜•์ œ์  ์ค‘๊ตญ์ธ๋ฏผ์˜ ํ•ด๋ฐฉ์„ ์œ„ํ•˜์—ฌ ํ˜๋ฆฐ ์ˆ˜๋งŽ์€ ์กฐ์„ ๊ณต์‚ฐ์ฃผ์˜์ž๋“ค๊ณผ ์• ๊ตญ์  ์ธ๋ฏผ๋“ค์˜ ๊ณ ๊ท€ํ•œ ํ”ผ๊ฐ€ ๋œจ๊ฒ๊ฒŒ ์Šค๋ฉฐ์žˆ๋‹ค. ํ•ญ์ผํ˜๋ช…ํˆฌ์Ÿ์„ ๋น›๋‚˜๋Š” ์Šน๋ฆฌ์—๋กœ ๋ น๋„ํ•˜์‹  ๊ฒฝ์• ํ•˜๋Š” ์ˆ˜๋ น ๊น€์ผ์„ฑ๋™์ง€๊ป˜์„œ๋Š” ์‚ฌ๋ น๋ถ€์„ฑ์›๋“ค์„ ๊ฑฐ๋Š๋ฆฌ์‹œ๊ณ  ๋งˆ์นจ๋‚ด ๋ ฅ์‚ฌ์ ์ธ ์กฐ๊ตญ๊ฐœ์„ ์˜ ๊ธธ์— ์˜ค๋ฅด์‹œ์˜€๋‹ค.โ€™ ๋“ฑ์œผ๋กœ ๊น€์ผ์„ฑ์„ ์ฐฌ์–‘ยท๋ฏธํ™”ํ•˜๋ฉด์„œ ์ฃผ์ฒด์‚ฌ์ƒ ๋ฐ ๋ถํ•œ์˜ ์ˆ˜๋ น๊ด€์„ ๋งน๋ชฉ์ ์œผ๋กœ ์„ ์ „ํ•˜๋Š” ๋‚ด์šฉ์ด๋‹ค. ์ด์™€ ๊ฐ™์ด ์œ„ ๋งˆ์ดํฌ๋กœ ์—์Šค๋””(Micro SD 16G) ํ˜•์‹์˜ ๋””์ง€ํ„ธ์ €์žฅ๋งค์ฒด๋Š” ๊น€์ผ์„ฑ์˜ ์ „ ๊ฐ€๊ณ„๋ฅผ ํ•ญ์ผ ๋…๋ฆฝํˆฌ์‚ฌ๋กœ ๋ฌ˜์‚ฌํ•˜๊ณ  ๊น€์ผ์„ฑ์˜ ํ•ญ์ผ๋ฌด์žฅํˆฌ์Ÿ์„ ๋ฏธํ™”ยท์ฐฌ์–‘ํ•˜๋Š” ๋“ฑ ๊น€์ผ์„ฑ ๊ฐœ์ธ์„ ์šฐ์ƒํ™”ํ•˜๋Š” ์ทจ์ง€์˜ ๋‚ด์šฉ, ์กฐ์„ ๋กœ๋™๋‹น์˜ ์ผ๋‹น๋…์žฌ๋ฅผ ์ •๋‹นํ™”ํ•˜๋ฉด์„œ ๋ถํ•œ์˜ ๋Œ€๋‚จํ˜๋ช…๋…ธ์„ ์„ ์„ ์ „ยท์„ ๋™ํ•˜๋Š” ์ทจ์ง€์˜ ๋‚ด์šฉ, ๋ถํ•œ์‹ ์‚ฌํšŒ์ฃผ์˜๊ฐ€ ์ฃผ์ฒด์‚ฌ์ƒ์œผ๋กœ ๋ฌด์žฅํ•œ ๋ถํ•œ ์ธ๋ฏผ๋“ค์— ์˜ํ•ด ์ ˆ๋Œ€์  ์ง€์ง€๋ฅผ ๋ฐ›๊ณ  ์žˆ๋‹ค๋Š” ์ทจ์ง€๋กœ ๋ถํ•œ ์‚ฌํšŒ์ฃผ์˜์ฒด์ œ์˜ ์šฐ์›”์„ฑ ๋ฐ ํ˜๋ช…์  ์ •๋‹น์„ฑ ๋“ฑ์„ ๊ฐ•์กฐํ•˜๋Š” ์ทจ์ง€์˜ ๋‚ด์šฉ, ๊น€์ผ์„ฑ ์ฃผ์ฒด์‚ฌ์ƒ์˜ ์šฐ์›”์„ฑ์„ ์„ ์ „ํ•˜๋ฉด์„œ ๋ฏธ ์ œ๊ตญ์ฃผ์˜์— ์˜ˆ์†๋œ ๋‚จํ•œ ๋ฏผ์ค‘์„ ํ•ด๋ฐฉํ•˜๊ณ  ํ•œ๋ฐ˜๋„์˜ ํ†ต์ผ์„ ์‹คํ˜„ํ•˜๊ธฐ ์œ„ํ•ด ๋‚จํ•œ ๋ฏผ์ค‘์„ ์ฃผ์ฒด์‚ฌ์ƒ์œผ๋กœ ์˜์‹ํ™”ํ•˜์—ฌ์•ผ ํ•œ๋‹ค๋Š” ์ทจ์ง€์˜ ๋‚ด์šฉ, ์ž๋ณธ์ฃผ์˜๊ฒฝ์ œ์ œ๋„๋Š” ์ธ๋ฏผ๋Œ€์ค‘์„ ์ˆ˜ํƒˆํ•˜๊ธฐ ์œ„ํ•œ ๋„๊ตฌ์— ๋ถˆ๊ณผํ•˜๊ณ  ์ฃผ์ฒด์‚ฌ์ƒ์ด ๊ทผ๊ฐ„์ด ๋œ ๋ถํ•œ์˜ ์‚ฌํšŒ์ฃผ์˜ ์ฒด์ œ๋Š” ๋ชจ๋“  ๊ตฌ์„ฑ์›๋“ค์ด ํ‰๋“ฑํ•˜๊ณ  ํ’์กฑํ•œ ๋ฌผ์งˆ์  ๋ฌธํ™”์ƒํ™œ์ด ๋ณด์žฅ๋˜๋Š” ์‚ฌํšŒ๋ผ๊ณ  ๋ฏธํ™”ํ•˜๋ฉด์„œ ๋ถํ•œ ์‚ฌํšŒ์ฃผ์˜ ์ฒด์ œ์™€ ์ฃผ์ฒด์‚ฌ์ƒ์„ ์ฐฌ์–‘ยท์„ ์ „ํ•˜๋Š” ์ทจ์ง€์˜ ๋‚ด์šฉ, ๊น€์ผ์„ฑยท๊น€์ •์ผ์ฃผ์˜๋ฅผ ์ถ”์ข…ํ•˜๋Š” ๋ถํ•œ ์‚ฌํšŒ์ฃผ์˜์ฒด์ œ์˜ ์šฐ์ˆ˜์„ฑ ๋ฐ ์ฃผ์ฒด์‚ฌ์ƒ๊ณผ ์„ ๊ตฐ์ •์น˜๋ฅผ ์„ ์ „ํ•˜๋Š” ์ทจ์ง€์˜ ๋‚ด์šฉ, ๋ถํ•œ์˜ ๋Œ€๋‚จํ˜๋ช…๋ก ์˜ ์ผํ™˜์œผ๋กœ ์ฃผํ•œ๋ฏธ๊ตฐ์˜ ์ฒ ์ˆ˜๋ฅผ ์ฃผ์žฅํ•˜๊ณ  ๋ถํ•œ์˜ ์—ฐ๋ฐฉ์ œ ํ†ต์ผ๋ฐฉ์‹์˜ ์ •๋‹น์„ฑ ๋“ฑ์„ ์„ ์ „ํ•˜๋Š” ์ทจ์ง€์˜ ๋‚ด์šฉ, ๊น€๋ถ€์ž์— ๋Œ€ํ•œ ์˜์›ํ•œ ์ถฉ์„ฑ์„ ๋งน์„ธํ•˜๋Š” ์ทจ์ง€์˜ ๋‚ด์šฉ ๋“ฑ์— ๊ด€ํ•œ ํŒŒ์ผ ๋“ฑ์„ ์ˆ˜๋กํ•˜๊ณ  ์žˆ๋‹ค. ์ด๋กœ์จ ํ”ผ๊ณ ์ธ์€ ๊ตญ๊ฐ€์˜ ์กด๋ฆฝยท์•ˆ์ „์ด๋‚˜ ์ž์œ ๋ฏผ์ฃผ์  ๊ธฐ๋ณธ์งˆ์„œ๋ฅผ ์œ„ํƒœ๋กญ๊ฒŒ ํ•œ๋‹ค๋Š” ์ •์„ ์•Œ๋ฉด์„œ ๋ฐ˜๊ตญ๊ฐ€๋‹จ์ฒด๋‚˜ ๊ทธ ๊ตฌ์„ฑ์› ๋˜๋Š” ๊ทธ ์ง€๋ น์„ ๋ฐ›์€ ์ž์˜ ํ™œ๋™์„ ์ฐฌ์–‘ยท๊ณ ๋ฌดยท์„ ์ „ ๋˜๋Š” ์ด์— ๋™์กฐํ•  ๋ชฉ์ ์œผ๋กœ ์œ„ ์ด์ ํ‘œํ˜„๋ฌผ์„ ์†Œ์ง€ํ•˜์˜€๋‹ค. ๋‚˜. ์œ ์—์Šค๋น„(USB 8G) ์†Œ์ง€ ํ”ผ๊ณ ์ธ์€ 2015. 11. 13.๊ฒฝ ์„œ์šธ ๋™์ž‘๊ตฌ (์ฃผ์†Œ 2 ์ƒ๋žต) โ–ณโ–ณ๋Œ€ํ•™๊ต ์ฃผ์ฐจ์žฅ์— ์ฃผ์ฐจ๋˜์–ด ์žˆ๋Š” ํ”ผ๊ณ ์ธ ์†Œ์œ ์˜ ์ฝ”๋ž€๋„ ํˆฌ๋ฆฌ์Šค๋ชจ ์Šนํ•ฉ์ฐจ ๋‚ด ์ˆ˜๋‚ฉ๊ณต๊ฐ„์— ๊น€๋ถ€์ž ๋ฐ ๋ถํ•œ์ฒด์ œ๋ฅผ ์ฐฌ์–‘ยท๋ฏธํ™”ํ•˜๊ฑฐ๋‚˜, ์ฃผ์ฒด์‚ฌ์ƒ์— ์ž…๊ฐํ•œ ๋Œ€๋‚จํ˜๋ช…ํˆฌ์Ÿ์„ ์„ ์ „ยท๋™์กฐํ•˜๋Š” ๋“ฑ์˜ ๊ฐ์ข… ํŒŒ์ผ๋“ค์„ ์ฒด๊ณ„์ ์œผ๋กœ ์ •๋ฆฌยท์ €์žฅํ•œ ์œ ์—์Šค๋น„(USB 8G) ๋””์ง€ํ„ธ์ €์žฅ๋งค์ฒด๋ฅผ ๋ณด๊ด€ํ•˜๊ณ  ์žˆ์—ˆ๋‹ค. ์œ„ ์ €์žฅ๋งค์ฒด๋Š” ใ€Œ2015studyใ€ยทใ€ŒBOOKSใ€ยทใ€Œpatriatismใ€ยทใ€Œ๊ต์–‘ใ€ยทใ€Œ์†Œ์„คใ€ยทใ€Œ์ €์ž‘์ง‘ใ€ยทใ€ŒํšŒ๊ณ ๋ก ์„ธ๊ธฐ์™€ ๋”๋ถˆ์–ดใ€ยทใ€ŒํšŒ๊ณ ๋ก ์ „๋ฌธ ๋‚ญ๋… 1-24์žฅ(์ˆœ์„œ์ •๋ฆฌ)ใ€ ๋“ฑ 8๊ฐœ์˜ ์ƒ์œ„ ํด๋”๋กœ ๊ตฌ์„ฑ๋˜์–ด ์žˆ๋‹ค. ์œ„ ์ƒ์œ„ํด๋” ์ค‘ ใ€Œ2015studyใ€ ํด๋”๋Š” ใ€Œ2015์‹ ๋…๊ต์–‘ใ€ยทใ€Œ2015์• ๊ตญ์ฃผ์˜๊ต์–‘ใ€ ๋“ฑ 2๊ฐœ์˜ ํ•˜์œ„ ํด๋”๋กœ ๊ตฌ์„ฑ๋˜์–ด ์žˆ๋Š”๋ฐ, ๊ทธ ํ•˜์œ„ ํด๋” ์ค‘ ใ€Œ2015์‹ ๋…๊ต์–‘ใ€ ํด๋”์—๋Š” โ€˜์ฃฝ์–ด๋„ ํ˜๋ช…์‹ ๋… ๋ฒ„๋ฆฌ์ง€ ๋ง์žโ€™ ๋“ฑ ๋ถํ•œ์ฒด์ œ๋ฅผ ์ฐฌ์–‘ํ•˜๋Š” ์ž๋ฃŒ๋“ค์ด, ใ€Œ2015์• ๊ตญ์ฃผ์˜๊ต์–‘ใ€ ํด๋”์—๋Š” โ€˜๊น€์ •์ผ ์• ๊ตญ์ฃผ์˜ ์š”์•ฝ๋ณธโ€™ ๋“ฑ ๊น€์ •์ผ์„ ๋งน๋ชฉ์ ์œผ๋กœ ์ฐฌ์–‘ํ•˜๋Š” ์ž๋ฃŒ๋“ค์ด ๊ฐ ์ €์žฅ๋˜์–ด ์žˆ๋‹ค. ์œ„ ์ƒ์œ„ํด๋” ์ค‘ ใ€ŒBOOKSใ€ ํด๋”๋Š” ใ€ŒETCใ€ยทใ€Œ์†Œ์„คใ€ยทใ€Œ์—ฐ๊ตฌ์„œ์ ใ€ยทใ€Œ์™ธ๊ตญ์†Œ์„คใ€ ๋“ฑ 4๊ฐœ์˜ ํ•˜์œ„ ํด๋”๋กœ ๊ตฌ์„ฑ๋˜์–ด ์žˆ๋Š”๋ฐ ๊ทธ์ค‘ ใ€ŒETCใ€ ํด๋”๋Š” ใ€Œ๊ธฐํƒ€๋„์„œใ€ยทใ€Œ๋ฌธํ•™์ž‘ํ’ˆใ€ยทใ€Œํ˜๋ช…์†Œ์„คใ€ ๋“ฑ 3๊ฐœ์˜ ์„ธ๋ถ€ ํด๋”๋กœ ๋‚˜๋ˆ„์–ด์ ธ โ€˜๊ฐœ์„ โ€™ ๋“ฑ ๊น€์ผ์„ฑ์„ ์˜์›…์œผ๋กœ ์นญํ•˜๋ฉฐ ๋ฏธํ™”ยท์ฐฌ์–‘ํ•˜๋Š” ์ž๋ฃŒ๋“ค์ด, ใ€Œ์†Œ์„คใ€ ํ•˜์œ„ ํด๋”์—๋Š” ๋ถํ•œ ์†Œ์„ค โ€˜์šฐ๋“ฑ๋ถˆโ€™ ๋“ฑ ๊น€์ผ์„ฑ ํ•ญ์ผ ๋ฌด์žฅํˆฌ์Ÿ ๋ฏธํ™” ๋ฐ ๊น€์ผ์„ฑ์˜ ์‚ฌํšŒ์ฃผ์˜ ํ˜๋ช…์„ ์ฐฌ์–‘ํ•˜๋Š” ๋“ฑ์˜ ์ž๋ฃŒ๋“ค์ด, ใ€Œ์—ฐ๊ตฌ์„œ์ ใ€ ํ•˜์œ„ ํด๋”์—๋Š” โ€˜์šฐ๋ฆฌ์‹ ์‚ฌํšŒ์ฃผ์˜ 100๋ฌธ 100๋‹ตโ€™ ๋“ฑ ๋ถํ•œ์‚ฌํšŒ์ฃผ์˜ ์ฒด์ œ์˜ ์šฐ์›”์„ฑ์„ ์ฐฌ์–‘ํ•˜๋Š” ๋ถํ•œ ์›์ „ ์ž๋ฃŒ ๋“ฑ์ด, ใ€Œ์™ธ๊ตญ์†Œ์„คใ€ ํ•˜์œ„ ํด๋”์—๋Š” ๊ณต์‚ฐ์ฃผ์˜ํ˜๋ช…์„ ์ฐฌ์–‘ํ•˜๋Š” ์ž๋ฃŒ๋“ค์ด ๊ฐ ์ €์žฅ๋˜์–ด ์žˆ๋‹ค. ์œ„ ์ƒ์œ„ํด๋” ์ค‘ ใ€Œpatriatismใ€ ํด๋”์—๋Š” โ€˜๊น€์ •์ผ ์• ๊ตญ์ฃผ์˜์— ๋Œ€ํ•œ ์ด์•ผ๊ธฐโ€™ ๋“ฑ ๊น€์ •์ผ ์• ๊ตญ์‹ฌ๊ณผ ๋ฏผ์กฑ์•  ๋“ฑ์— ๋Œ€ํ•œ ๋งŽ์€ ์ผํ™”๋“ค์„ ํŽธ์ง‘ยท์†Œ๊ฐœํ•˜๋ฉฐ ๊น€์ผ์„ฑยท๊น€์ •์ผ ํ™œ๋™์„ ์ฐฌ์–‘ยท๋ฏธํ™”ํ•˜๋Š” ์ž๋ฃŒ๋“ค์ด ์ €์žฅ๋˜์–ด ์žˆ๋‹ค. ์œ„ ์ƒ์œ„ํด๋” ์ค‘ ใ€Œ๊ต์–‘ใ€ ํด๋”๋Š” 10๊ฐœ์˜ ํ•˜์œ„ ํด๋”๋กœ ๊ตฌ์„ฑ๋˜์–ด ์žˆ๋Š”๋ฐ, ๊ทธ ํ•˜์œ„ ํด๋” ์ค‘ ใ€ŒP๊ต์–‘ใ€ ํด๋”๋Š” ใ€Œ๋…ธ๋ž˜ใ€ยทใ€Œ์• ๊ตญ์ ์ „์œ„๋Œ€ใ€์˜ 2๊ฐœ ์„ธ๋ถ€ ํด๋”๋กœ ๊ตฌ์„ฑ๋˜์–ด โ€˜ํ•œ๊ตญ์ „์œ„์กฐ์ง ์šด๋™์‚ฌโ€™ ๋“ฑ ์ฃผ์ฒด์‚ฌ์ƒ์— ์ž…๊ฐํ•œ ๋Œ€๋‚จํ˜๋ช… ํˆฌ์Ÿ์„ ์„ ๋™ํ•˜๋Š” ์ž๋ฃŒ๋“ค์ด ์ €์žฅ๋˜์–ด ์žˆ๊ณ , ใ€Œ๊ธฐ๋ณธ์Œ์•…ใ€ ํ•˜์œ„ ํด๋”์—๋Š” ๋ถํ•œ ์Œ์•… ๋“ฑ์˜ ํŒŒ์ผ๋“ค๋กœ ๊ตฌ์„ฑ๋˜์–ด โ€˜๊น€์ผ์„ฑ์žฅ๊ตฐ์˜ ๋…ธ๋ž˜โ€™ ๋“ฑ ๊น€์ผ์„ฑ์„ ๋งน๋ชฉ์ ์œผ๋กœ ์ฐฌ์–‘ํ•˜๋Š” ์ž๋ฃŒ๋“ค์ด ์ €์žฅ๋˜์–ด ์žˆ์œผ๋ฉฐ, ใ€Œ๊น€์ผ์„ฑ์ฃผ์˜ ์ด์„œใ€ ํ•˜์œ„ ํด๋”์—๋Š” โ€˜๊น€์ผ์„ฑ์ฃผ์˜ ์ด์„œ 1โ€™ ๋“ฑ ๊น€์ผ์„ฑ์„ ๋งน๋ชฉ์ ์œผ๋กœ ์ฐฌ์–‘ํ•˜๊ณ  ์ฃผ์ฒด์‚ฌ์ƒ์„ ์„ ์ „ยท์ „ํŒŒํ•˜๋Š” ์ž๋ฃŒ๋“ค์ด ์ €์žฅ๋˜์–ด ์žˆ๊ณ , ใ€Œ๋Œ€์žฅใ€ ํ•˜์œ„ ํด๋”์—๋Š” ๊น€์ •์€ ํƒ„์ƒ๊ธฐ๋…์ผ์„ ๋งž์ดํ•˜์—ฌ ๊น€์ •์€์˜ ์œ„๋Œ€์„ฑ์„ ๊ฐ•์กฐํ•œ โ€˜๋ฐฑ๋‘์‚ฐํ˜•์˜ ์œ„์ธ์ด์‹  ์ฒญ๋…„๋Œ€์žฅ ๋™์ง€๋ฅผ ๋ชจ์‹œ์—ฌ ์ฃผ์ฒดํ˜๋ช…์œ„์—…์˜ ๋ฏธ๋ž˜๊ฐ€ ๋์—†์ด ์ฐฝ์ฐฝํ•œ๋ฐ ๋Œ€ํ•˜์—ฌโ€™ ํŒŒ์ผ ๋“ฑ์ด ์ €์žฅ๋˜์–ด ์žˆ์œผ๋ฉฐ, ใ€Œ๋Œ€์ค‘ ์†Œ๋ชจ์ž„ ๊ต์–‘ใ€ ํ•˜์œ„ ํด๋”๋Š” ใ€Œ๋Œ€์ค‘์‚ฌ์—…๋ฐฉ๋ฒ•ใ€ยทใ€Œ์„ ๊ตฐ์‚ฌ์ƒใ€ยทใ€Œ์ฃผ์ฒด์‚ฌํšŒ์ฃผ์˜ใ€ ๋“ฑ์˜ 8๊ฐœ ์„ธ๋ถ€ ํด๋”๋กœ ๊ตฌ์„ฑ๋˜์–ด โ€˜์ฃผ์ฒด์ •์น˜๊ฒฝ์ œํ•™โ€™ ๋“ฑ ๊น€๋ถ€์ž๋ฅผ ์ฐฌ์–‘ํ•˜๊ณ  ๋ถํ•œ์˜ ์ •์น˜๊ฒฝ์ œ์ฒด๊ณ„ ๋“ฑ์„ ๊น€๋ถ€์ž ์ž…์žฅ์—์„œ ์„ค๋ช…ยท์˜นํ˜ธํ•˜๋Š” ์ž๋ฃŒ๋“ค์ด ์ €์žฅ๋˜์–ด ์žˆ๊ณ , ใ€Œ๋Œ€ํ™”ใ€ ํ•˜์œ„ ํด๋”์—๋Š” โ€˜๋‚˜์™€ ์ฃผ์ฒด์‚ฌ์ƒ๊ณผ์˜ ๋Œ€ํ™”โ€™ ๋“ฑ ๊น€์ผ์„ฑยท๊น€์ •์ผ์„ ์ฐฌ์–‘ยท๋ฏธํ™”ํ•˜๋Š” ์ž๋ฃŒ๋“ค์ด ์ €์žฅ๋˜์–ด ์žˆ์œผ๋ฉฐ, ใ€Œ๋„์„œใ€ ํ•˜์œ„ ํด๋”๋Š” ใ€Œ์†Œ์„คใ€ยทใ€Œ์œ„๋Œ€์„ฑใ€์˜ 2๊ฐœ ์„ธ๋ถ€ ํด๋”๋กœ ๊ตฌ์„ฑ๋˜์–ด โ€˜1932๋…„โ€™ ๋“ฑ ๊น€์ผ์„ฑ์˜ ํ•ญ์ผ๋ฌด์žฅํ™œ๋™์„ ์ฐฌ์–‘ยท๋ฏธํ™”ํ•˜๋Š” ์ž๋ฃŒ๋“ค์ด ์ €์žฅ๋˜์–ด ์žˆ๊ณ , ใ€Œ์œ„๋Œ€์„ฑํ•™์Šตใ€ ํ•˜์œ„ ํด๋”๋Š” ใ€Œ์žฅ๊ตฐ๋‹˜์˜ ์œ„๋Œ€์„ฑใ€ยทใ€Œํ˜๋ช…์ผํ™”ใ€ ๋“ฑ 2๊ฐœ์˜ ์„ธ๋ถ€ ํด๋”๋กœ ๊ตฌ์„ฑ๋˜์–ด โ€˜์žฅ๊ตฐ๋‹˜๊ณผ ์‚ฌ์ƒ‰โ€™ ๋“ฑ ๊น€์ •์ผ์„ ๋น„๋ฒ”ํ•œ ์‚ฌ์ƒ‰์„ ํ•˜๋Š” ์œ„๋Œ€ํ•œ ์˜๋„์ž๋ผ ํ‰๊ฐ€ํ•˜๋ฉฐ ์šฐ์ƒํ™”ํ•˜๋Š” ์ž๋ฃŒ๋“ค์ด ์ €์žฅ๋˜์–ด ์žˆ์œผ๋ฉฐ, ใ€Œ์ •์ˆ˜ใ€ ํ•˜์œ„ ํด๋”๋Š” ใ€Œ1๋‹จ๊ณ„ใ€ยทใ€Œ2๋‹จ๊ณ„ใ€ ๋“ฑ 2๊ฐœ์˜ ์„ธ๋ถ€ ํด๋”๋กœ ๊ตฌ์„ฑ๋˜์–ด โ€˜์กฐ์„ ๋กœ๋™๋‹น๋žต์‚ฌโ€™ ๋“ฑ ๊น€์ผ์„ฑ์„ ์ฐฌ์–‘ํ•˜๊ณ  ๋Œ€๋‚จํ˜๋ช…ํˆฌ์Ÿ์„ ์„ ๋™ํ•˜๋Š” ์ž๋ฃŒ๋“ค์ด ์ €์žฅ๋˜์–ด ์žˆ๊ณ , ใ€Œ์กฐ์ง์‚ฌ์—…ใ€ ํด๋”๋Š” ใ€Œ์กฐ์ง์‚ฌ์—…๋ฐฉ๋ฒ•ใ€์˜ 1๊ฐœ ์„ธ๋ถ€ ํด๋”๋กœ ๊ตฌ์„ฑ๋˜์–ด โ€˜์ข…ํŒŒ์ฃผ์˜๋ฅผ ์ฒญ์‚ฐํ•˜๊ณ  ํ˜๋ช…๋Œ€์˜ค์˜ ํ†ต์ผ๋‹จ๊ฒฐ์„ ๊ฐ•ํ™”ํ•˜์žโ€™ ๋“ฑ โ€˜๊น€์ผ์„ฑ ์ €์ž‘์ง‘โ€™์— ์ˆ˜๋ก๋˜์–ด ์žˆ๋Š” ์ž๋ฃŒ๋“ค์ด ์ €์žฅ๋˜์–ด ์žˆ๋‹ค. ์œ„ ์ƒ์œ„ํด๋” ์ค‘ ใ€Œ์†Œ์„คใ€ ํด๋”๋Š” ใ€ŒCONTENTใ€์˜ 1๊ฐœ ํ•˜์œ„ ํด๋”๋กœ ๊ตฌ์„ฑ๋˜์–ด ์žˆ๋Š”๋ฐ ใ€ŒCONTENTใ€ ํด๋”์—๋Š” โ€˜ํ•ด๋ฐฉ์ „์•ผโ€™ ๋“ฑ ๊น€์ผ์„ฑ์˜ ํ•ญ์ผ๋ฌด์žฅํˆฌ์Ÿํ˜๋ช…ํ–‰์ ์„ ์ฐฌ์–‘ยท๋ฏธํ™”ํ•˜๋Š” ์ž๋ฃŒ๋“ค์ด ์ €์žฅ๋˜์–ด ์žˆ๋‹ค. ์œ„ ์ƒ์œ„ํด๋” ์ค‘ ใ€Œ์ €์ž‘์ง‘ใ€ ํด๋”๋Š” ใ€Œ๊น€์ผ์„ฑ ์ €์ž‘์ง‘ใ€ยทใ€Œ๊น€์ผ์„ฑ ์ €์ž‘์ง‘ ์ฃผ์ œ ๋ถ„๋ฅ˜ใ€ยทใ€Œ๊น€์ •์ผ ์ €์ž‘์ง‘ใ€ยทใ€Œ๊น€์ •์ผ ์ €์ž‘์ง‘ ์ฃผ์ œ ๋ถ„๋ฅ˜ใ€์˜ 4๊ฐœ ํ•˜์œ„ ํด๋”๋กœ ๊ตฌ์„ฑ๋˜์–ด ์žˆ๋Š”๋ฐ ๊ทธ์ค‘ ใ€Œ๊น€์ผ์„ฑ ์ €์ž‘์ง‘ใ€ ํ•˜์œ„ ํด๋”๋Š” ใ€Œ๊น€์ผ์„ฑ ์ €์ž‘์ง‘hwpํŒใ€ยทใ€Œ์˜์ƒ-์œ„๋Œ€ํ•œ ์ˆ˜๋ น ๊น€์ผ์„ฑ๋™์ง€๋Š” ์˜์›ํžˆ ์šฐ๋ฆฌ์™€ ํ•จ๊ป˜ ๊ณ„์‹œ๋‹คใ€ ๋“ฑ 2๊ฐœ์˜ ์„ธ๋ถ€ ํด๋”๋กœ ๊ตฌ์„ฑ๋˜์–ด โ€˜์˜์ƒโ€™ ๋“ฑ ๊น€์ผ์„ฑ์„ ์ฐฌ์–‘ยท์„ ์ „ํ•˜๋Š” ์ž๋ฃŒ๋“ค์ด ์ €์žฅ๋˜์–ด ์žˆ๊ณ , ใ€Œ๊น€์ผ์„ฑ ์ €์ž‘์ง‘ ์ฃผ์ œ ๋ถ„๋ฅ˜ใ€ ํ•˜์œ„ ํด๋”๋Š” ใ€Œ1 ์‚ฌ์ƒ์ด๋ก ใ€ยทใ€Œ2 ๋‹น์กฐ์ง ์‚ฌ์—…ใ€ยทใ€Œ3 ๊ตฐ๋Œ€์‚ฌ์—…ใ€ ๋“ฑ 11๊ฐœ์˜ ์„ธ๋ถ€ ํด๋”๋กœ ๊ตฌ์„ฑ๋˜์–ด โ€˜ํ˜„๋Œ€์กฐ์„ ์—ญ์‚ฌโ€™ ๋“ฑ ๊น€์ผ์„ฑ์˜ ํ•ญ์ผ๋ฌด์žฅํ˜๋ช… ํˆฌ์Ÿ์„ ์ฐฌ์–‘ยท๋ฏธํ™”ํ•˜๋Š” ํ•œํŽธ 6ยท25์ „์Ÿ์„ โ€˜์กฐ๊ตญํ•ด๋ฐฉ์ „์Ÿโ€™์œผ๋กœ ๊ทœ์ •ํ•˜๋Š” ๋“ฑ ๊น€์ผ์„ฑ์„ ์ฐฌ์–‘ํ•˜๋Š” ์ž๋ฃŒ๋“ค์ด ์ €์žฅ๋˜์–ด ์žˆ์œผ๋ฉฐ, ใ€Œ๊น€์ •์ผ ์ €์ž‘์ง‘ใ€ ํ•˜์œ„ ํด๋”๋Š” ใ€Œ๊น€์ •์ผ ์ €์ž‘์ง‘hwpํŒใ€ยทใ€Œ๊น€์ •์ผ ์ €์ž‘์ง‘ ์ „์ž์ฑ…ใ€ยทใ€Œ์˜์ƒ-๊น€์ •์ผ์žฅ๊ตฐ์˜ ๋…ธ๋ž˜ใ€ยทใ€Œ์ €์ž‘์ง‘ใ€ ๋“ฑ 4๊ฐœ์˜ ์„ธ๋ถ€ ํด๋”๋กœ ๊ตฌ์„ฑ๋˜์–ด โ€˜๊น€์ •์ผ ์ €์ž‘์ง‘ 1๊ถŒโ€™ ๋“ฑ ๊น€์ •์ผ์„ ์ฐฌ์–‘ยท์„ ์ „ํ•˜๋Š” ์ž๋ฃŒ๋“ค์ด ์ €์žฅ๋˜์–ด ์žˆ๊ณ , ใ€Œ๊น€์ •์ผ์ €์ž‘์ง‘ ์ฃผ์ œ ๋ถ„๋ฅ˜ใ€ ํ•˜์œ„ ํด๋”๋Š” ใ€Œ0 ์ •์„ธใ€ยทใ€Œ1 ์‚ฌ์ƒ์ด๋ก ใ€ยทใ€Œ2 ๋‹น์กฐ์ง์‚ฌ์—…ใ€ยทใ€Œ3 ๊ตฐ๋Œ€์‚ฌ์—…ใ€ ๋“ฑ 10๊ฐœ์˜ ์„ธ๋ถ€ ํด๋”๋กœ ๊ตฌ์„ฑ๋˜์–ด โ€˜์‚ฌ๋žŒ์€ ํ•œ์ƒ์„ ๋น›๋‚˜๊ฒŒ ์‚ด์•„์•ผ ํ•œ๋‹คโ€™ ๋“ฑ์˜ โ€˜๊น€์ •์ผ ์ €์ž‘์ง‘โ€™์— ์ˆ˜๋ก๋˜์–ด ์žˆ๋Š” ์ž๋ฃŒ๋“ค์ด ์ €์žฅ๋˜์–ด ์žˆ๋‹ค. ์œ„ ์ƒ์œ„ํด๋” ์ค‘ ใ€ŒํšŒ๊ณ ๋ก ์„ธ๊ธฐ์™€ ๋”๋ถˆ์–ดใ€ ํด๋”๋Š” ใ€ŒํšŒ๊ณ ๋กCDใ€ยทใ€ŒํšŒ๊ณ ๋ก๊ฐ•์˜ใ€ยทใ€ŒํšŒ๊ณ ๋ก ์„ธ๊ธฐ์™€ ๋”๋ถˆ์–ด๊ฐ•์˜ํŒใ€์˜ 3๊ฐœ ํ•˜์œ„ ํด๋”๋กœ ๊ตฌ์„ฑ๋˜์–ด ์žˆ๋Š”๋ฐ ๊ทธ์ค‘ ใ€ŒํšŒ๊ณ ๋กCDใ€ ํ•˜์œ„ ํด๋”์—๋Š” โ€˜์„ธ๊ธฐ์™€ ๋”๋ถˆ์–ด CD ์‹คํ–‰ํŒŒ์ผโ€™ ๋“ฑ ๊น€์ผ์„ฑ์„ ์ฐฌ์–‘ยท๋ฏธํ™”ํ•˜๋Š” ์ž๋ฃŒ๋“ค์ด ์ €์žฅ๋˜์–ด ์žˆ๊ณ , ใ€ŒํšŒ๊ณ ๋ก๊ฐ•์˜ใ€ ํ•˜์œ„ ํด๋”์—๋Š” โ€˜์„ธ๊ธฐ์™€ ๋”๋ถˆ์–ด ๊ฐ•์˜(1ํšŒ)โ€™ ๋“ฑ ๊น€์ผ์„ฑ์„ ์ฐฌ์–‘ยท๋ฏธํ™”ํ•˜๋Š” ์ž๋ฃŒ๋“ค์ด ์ €์žฅ๋˜์–ด ์žˆ์œผ๋ฉฐ, ใ€ŒํšŒ๊ณ ๋ก ์„ธ๊ธฐ์™€ ๋”๋ถˆ์–ด ๊ฐ•์˜ํŒใ€ ํ•˜์œ„ ํด๋”์—๋Š” โ€˜์„ธ๊ธฐ์™€ ๋”๋ถˆ์–ด ๊ฐ•์ขŒ๋ณธ 1โ€™ ๋“ฑ ๊น€์ผ์„ฑ์„ ์ฐฌ์–‘ยท๋ฏธํ™”ํ•˜๋Š” ์ž๋ฃŒ๋“ค์ด ์ €์žฅ๋˜์–ด ์žˆ๋‹ค. ์œ„ ์ƒ์œ„ํด๋” ์ค‘ ใ€ŒํšŒ๊ณ ๋ก ์ „๋ฌธ ๋‚ญ๋… 1-24์žฅ(์ˆœ์„œ์ •๋ฆฌ)ใ€ ํด๋”์—๋Š” ๊น€์ผ์„ฑ ํšŒ๊ณ ๋ก โ€˜์„ธ๊ธฐ์™€ ๋”๋ถˆ์–ดโ€™ ์ „๋ฌธ ๋‚ญ๋… ํŒŒ์ผ๋“ค๋กœ ๊ตฌ์„ฑ๋˜์–ด โ€˜์„ธ๊ธฐ์™€ ๋”๋ถˆ์–ด 1๊ถŒ 3์žฅ 1์ ˆ(์Œ์„ฑ ํŒŒ์ผ)โ€™ ๋“ฑ ๊น€์ผ์„ฑ์„ ๋ฏธํ™”ยท์ฐฌ์–‘ํ•˜๋Š” ์ž๋ฃŒ๋“ค์ด ์ €์žฅ๋˜์–ด ์žˆ๋‹ค. ์œ„ ์ €์žฅ๋งค์ฒด์— ์ˆ˜๋ก๋œ ํŒŒ์ผ๋“ค์˜ ์ฃผ์š” ๋‚ด์šฉ์€ ๋‹ค์Œ๊ณผ ๊ฐ™๋‹ค. ใ€ŠBOOKS ํด๋” ๋‚ด ETC ํด๋” ์ค‘ โ€˜๊ฐœ์„ โ€™์˜ ์ฃผ์š” ๋‚ด์šฉใ€‹ - โ€˜๊ฐœ์„ โ€™์€ ๊น€์ผ์„ฑ ๊ฐœ์ธ ์šฐ์ƒํ™”๋ฅผ ์œ„ํ•ด ๋ถํ•œ์—์„œ ๋งŒ๋“  ์ด์„œ โ€˜๋ถˆ๋ฉธ์˜ ๋ ฅ์‚ฌโ€™ ์ค‘ ํ•˜๋‚˜๋กœ ํ•ด๋ฐฉ์งํ›„ ๋ถํ•œ์— ์ž…๊ตญํ•œ ๊น€์ผ์„ฑ์„ โ€˜๊ฐœ์„ ์žฅ๊ตฐโ€™์œผ๋กœ ์ด๋ฏธ์ง€ํ™”ํ•˜๊ณ  ์กฐ์„ ๊ณต์‚ฐ๋‹น์˜ 1์ธ์ž๋กœ ๋ถ€๊ฐํ•˜์—ฌ โ€œ๊น€์ผ์„ฑ ์žฅ๊ตฐ๋‹˜์˜ ์กฐ๊ตญ๊ฐœ์„ ์€ ๋‹จ์ˆœํ•œ ๊ฐœ์„ ์˜์›…์˜ ๋‹จ์ˆœํ•œ ์กฐ๊ตญ ๊ท€ํ™˜์ด ์•„๋‹ˆ๋ผ ์กฐ๊ตญ๊ด‘๋ณต์˜ ์„ ๋ฌผ๊นŒ์ง€ ์•ˆ์•„ ์˜ค์‹  ํฌ์„ธ์˜ ์œ„์ธ์˜ ๊ฐœ์„ ์ด์‹œ๋ผ๋Š” ๊ฒƒ์„!โ€ ์ด๋ผ๊ณ  ํ‘œํ˜„ํ•˜๋Š” ๋“ฑ ๊น€์ผ์„ฑ์„ ์˜์›…์œผ๋กœ ์นญํ•˜๋ฉฐ ๋ฏธํ™”ยท์ฐฌ์–‘ํ•˜๋Š” ๋‚ด์šฉ์ด๋‹ค. ใ€ŠBOOKS ํด๋” ๋‚ด ์†Œ์„ค ํด๋” ์ค‘ โ€˜์šฐ๋“ฑ๋ถˆ 1, 2โ€™์˜ ์ฃผ์š” ๋‚ด์šฉใ€‹ - โ€˜์šฐ๋“ฑ๋ถˆโ€™์€ ์ผ์ œ์‹œ๋Œ€ ๋ง์— ๋งŒ์ฃผ์ง€์—ญ์—์„œ ํŽผ์ณ์ง„ ๊น€์ผ์„ฑ์ด ์ฐธ๊ฐ€ํ•˜์˜€๋‹ค๊ณ  ์ฃผ์žฅํ•˜๋Š” ํ•ญ์ผ์œ ๊ฒฉ์ „ ๋“ฑ์„ ๋ฏธํ™”ํ•˜๊ธฐ ์œ„ํ•ด 1941. 2. ๋‘๋งŒ๊ฐ• ์—ฐ์•ˆ์˜ ํ•˜์„ฑํฌ ์ธ๊ทผ ์ˆ˜๋ฆผ ์†์—์„œ ์ •์น˜๊ณต์ž‘์†Œ์กฐ ์ฑ…์ž„์ž, ์ •์น˜๊ณต์ž‘ ์†Œ์กฐ์›, ์ง€ํ•˜์กฐ์ง์› ๋“ฑ์ด ํ•ญ์ผ ์œ ๊ฒฉ์ „์„ ์ „๊ฐœํ•˜๋ฉด์„œ ๊น€์ผ์„ฑ์˜ ์ง€๋„๋ฅผ ๋ฐ›๋“ค์–ด ์‹ ๋…์„ ๊ฐ–๊ณ  ํ•ญ์ผ ๋ฌด์žฅํˆฌ์Ÿ ํ™œ๋™์„ ํŽผ์น˜๋ฉฐ ํ˜๋ช…์„ ์™„์ˆ˜ํ•ด ๋‚˜๊ฐ€๋Š” ๊ณผ์ •์„ ๋ฌ˜์‚ฌํ•˜๋Š” ๋“ฑ ๊น€์ผ์„ฑ ๊ฐœ์ธ ์šฐ์ƒํ™” ๋ฐ ๋ถํ•œ์˜ ์‚ฌํšŒ์ฃผ์˜ ํ˜๋ช…์„ ์ฐฌ์–‘ํ•˜๋Š” ๋‚ด์šฉ์ด๋‹ค. ใ€Špatriatism ํด๋” ๋‚ด โ€˜๊น€์ •์ผ ์• ๊ตญ์ฃผ์˜์— ๋Œ€ํ•œ ์ด์•ผ๊ธฐโ€™์˜ ์ฃผ์š” ๋‚ด์šฉใ€‹ - ๊น€์ •์ผ์˜ ์• ๊ตญ์ฃผ์˜๋ฅผ โ€˜์กฐ๊ตญ๊ณผ ์ธ๋ฏผ์— ๋Œ€ํ•œ ์—ด๋ ฌํ•œ ์‚ฌ๋ž‘์ด๋ฉฐ ์ œ๊ตญ์ฃผ์˜์— ๋งž์„œ ์ฃผ์ฒด์˜ ์‚ฌํšŒ์ฃผ์˜๋ฅผ ๊ฒฐ์‚ฌ์ˆ˜ํ˜ธํ•˜๊ณ  ์กฐ๊ตญํ†ต์ผ์˜ ์„ฑ์Šค๋Ÿฌ์šด ์œ„์—…์„ ๊ธฐ์–ด์ด ์„ฑ์ทจํ•˜๋ฆฌ๋ผ๋Š” ๋ถˆ๊ตด์˜ ์ •์‹ ์ด์ž ์œ„๋Œ€ํ•œ ์ธ๊ฐ„์˜ ์‚ถ์˜ ์„œ์‚ฌ์‹œ๋ผ๊ณ  ์ถ”์•™โ€™ํ•˜๋Š” ๋“ฑ ๊น€์ •์ผ์˜ ์• ๊ตญ์ฃผ์˜์— ๋Œ€ํ•œ ๋งŽ์€ ์ผํ™”๋“ค์„ ํŽธ์ง‘ยท์†Œ๊ฐœํ•˜๋ฉฐ ๊น€์ผ์„ฑยท๊น€์ •์ผ ํ™œ๋™์„ ์ฐฌ์–‘ยท๋ฏธํ™”ํ•˜๋Š” ๋‚ด์šฉ์ด๋‹ค. ใ€Š๊ต์–‘ํด๋” ์ค‘ P๊ต์–‘ ํด๋” ๋‚ด โ€˜ํ•œ๊ตญ์ „์œ„์กฐ์ง ์šด๋™์‚ฌโ€™์˜ ์ฃผ์š” ๋‚ด์šฉใ€‹ - ์ „์œ„์กฐ์ง์€ ์ „์ฒด ๊ทผ๋กœ๋Œ€์ค‘์˜ ์ด์ต์˜ ๋Œ€ํ‘œ์ž์ด๋ฉฐ ๊ทธ๋“ค์„ ์˜์‹ํ™”, ์กฐ์งํ™”ํ•  ์ˆ˜ ์žˆ๋Š” ์ •์น˜ ์‚ฌ์ƒ์ ยท์กฐ์ง์  ๋Šฅ๋ ฅ์˜ ์ฒดํ˜„์ž์ผ๋ฟ๋งŒ ์•„๋‹ˆ๋ผ ์‚ฌํšŒ๋ณ€ํ˜์šด๋™ ์ „๋ฐ˜์„ ํ†ต์ผ์ ์œผ๋กœ ์„ค๊ณ„ํ•˜๊ณ  ์ž‘์ „ํ•˜๋ฉฐ ๊ทธ ์‹คํ˜„์„ ์œ„ํ•œ ํˆฌ์Ÿ์„ ์ •์น˜์ ์œผ๋กœ ์˜๋„ํ•˜๋Š” ๊ธฐ๊ด€์ด๊ธฐ ๋•Œ๋ฌธ์ด๋‹ค. - (๋ช…์นญ 37 ์ƒ๋žต)๋‹น์€ 1985๋…„ ๋ณ€์ฒœ๋˜๋Š” ํ˜„์‹ค๋ฐœ์ „์˜ ์š”๊ตฌ์™€ ๋ฏผ์ค‘์˜ ์ง€ํ–ฅ์„ ๋ฐ˜์˜ํ•ด์„œ ์ž๊ธฐ์˜ ๋ช…์นญ์„ โ—โ—โ—โ—โ—โ—โ—โ—, ์•ฝ์นญ โ€˜โ—โ—โ—โ€™์œผ๋กœ ๊ฐœ์นญํ•˜๊ณ  ๋ฐ˜๋ฏธ์ž์ฃผํ™”์˜ ๊ธฐ์น˜ ๋ฐ‘์— ์ด ๋•…์˜ ์ž์ฃผ์™€ ๋ฏผ์ฃผ, ํ†ต์ผ์„ ์œ„ํ•œ ํˆฌ์Ÿ์—์„œ ์• ๊ตญ์  ์ „์œ„๋Œ€๋กœ์„œ์˜ ์—ญ์‚ฌ์  ์ค‘์ž„์„ ์ˆ˜ํ–‰ํ•  ์ˆ˜ ์žˆ๋Š” ๊ฐ•์ฒ ์˜ ๋Œ€์˜ค๋กœ ์„ฑ์žฅํ•ด ์™”๋‹ค. - โ—โ—โ—์˜ ์ง€๋„๋…ธ์„ ์€ ํฌ๊ฒŒ ์„ธ๊ฐ€์ง€๋กœ ์š”์•ฝํ•  ์ˆ˜ ์žˆ๋Š”๋ฐ, ์ฒซ์งธ ์ง€๋„์ด๋…์—์„œ ์ฃผ์ฒด์‚ฌ์ƒ์„ ๊ฒฌ์ง€ํ•˜๋Š” ๋…ธ์„ , ๋‘˜์งธ ํˆฌ์Ÿ ๋…ธ์„ ์—์„œ ๋ฏผ์กฑํ•ด๋ฐฉ ์šด๋™๋…ธ์„ ์˜ ๊ฒฌ์ง€, ์…‹์งธ ์šด๋™์˜ ๋Œ€์ค‘ํ™” ๋…ธ์„ ์ด๋ฏ€๋กœ 80๋…„๋Œ€ ํ•œ๊ตญ์‚ฌํšŒ ๋ณ€ํ˜์šด๋™์ด ์–ป์–ด๋‚ธ ๊ฐ€์žฅ ํฐ ๊ฒฐ์‹ค์€ ์šฐ๋ฆฌ ์šด๋™์˜ ์ฃผ์ ์ด ๋ฏธ๊ตญ์ž„์„ ๋ช…ํ™•ํžˆ ํ•˜๋ฉด์„œ ๋ฐ˜๋ฏธํˆฌ์Ÿ์„ ์ฃผ๋„๋กœ ํ•˜๋Š” ๋ฏผ์กฑํ•ด๋ฐฉ ์šด๋™์œผ๋กœ์„œ์˜ ์ž๊ธฐ ๊ถค๋„๋ฅผ ์ฐพ์•„๋‚˜๊ฐ„ ๊ฒƒ์ด๋‹ค. - ์ด์™€ ๊ฐ™์€ ๋ฌธ๊ฑด์€ ๋ฐ˜๋ฏธํˆฌ์Ÿ์€ ๋ฏผ์กฑ์ž์ฃผ์ •๊ถŒ ์ˆ˜๋ฆฝ์„ ๊ทผ๋ณธ๊ณผ์ œ๋กœ ํ•˜๊ณ  ์ฃผํ•œ๋ฏธ๊ตฐ์ฒ ์ˆ˜๋ฅผ ๋‹น๋ฉด ์ง€ํ–ฅ์ ์œผ๋กœ ํ•˜์—ฌ ์ „์œ„์กฐ์ง์ด ์ ๊ทน ์„ ๋„ํ•ด ๊ฐ„๋‹ค๋Š” ๋“ฑ์˜ ์ทจ์ง€๋กœ โ€˜โ—โ—โ—โ€™์„ ๋‚จํ•œ์— ์ž์ƒํ•˜๊ณ  ์žˆ๋Š” ์ง€ํ•˜ ์กฐ์ง์ธ ๊ฒƒ์ฒ˜๋Ÿผ ์™œ๊ณก ์ฃผ์žฅํ•˜๋ฉด์„œ ์ฃผ์ฒด์‚ฌ์ƒ์— ์ž…๊ฐํ•œ ๋Œ€๋‚จํ˜๋ช… ํˆฌ์Ÿ์„ ์„ ์ „ยท์„ ๋™ํ•˜๋Š” ๋‚ด์šฉ์ด๋‹ค. ใ€Š๊ต์–‘ ํด๋” ์ค‘ ๋Œ€์ค‘์†Œ๋ชจ์ž„๊ต์–‘ ํด๋” ๋‚ด โ€˜์ฃผ์ฒด์ •์น˜๊ฒฝ์ œํ•™โ€™์˜ ์ฃผ์š” ๋‚ด์šฉใ€‹ - ์œ„๋Œ€ํ•œ ์˜๋„์ž ๊น€์ •์ผ ๋™์ง€๊ป˜์„œ๋Š” ๋‹ค์Œ๊ณผ ๊ฐ™์ด ๊ต์‹œํ•˜์‹œ์—ˆ๋‹ค. โ€œ์ž๋ณธ์ฃผ์˜์‚ฌํšŒ์—์„œ๋Š” ๋ช‡ ๋†ˆ์˜ ์ž๋ณธ๊ฐ€๋“ค์ด ์ƒ์‚ฐ์ˆ˜๋‹จ์„ ๊ฐ€์ง€๊ณ  ์ƒ์‚ฐ๋ฌผ๋„ ๋…์ฐจ์ง€ํ•˜๊ธฐ ๋•Œ๋ฌธ์— ๋…ธ๋™์ž๋“ค์€ ์ƒ์‚ฐ์— ์—ด์„ฑ์„ ๋‚ผ ์ˆ˜ ์—†์œผ๋ฉฐ ๋‹ค๋งŒ ๊ตถ์–ด์ฃฝ์ง€ ์•Š๊ธฐ ์œ„ํ•˜์—ฌ ์ž๊ธฐ์˜ ๋…ธ๋™๋ ฅ์„ ์ž๋ณธ๊ฐ€๋“ค์—๊ฒŒ ํŒ” ๋ฟ์ž…๋‹ˆ๋‹คโ€ - ์œ„๋Œ€ํ•œ ์˜๋„์ž ๊น€์ •์ผ ๋™์ง€๊ป˜์„œ๋Š” ๋‹ค์Œ๊ณผ ๊ฐ™์ด ์ง€์ ํ•˜์‹œ์—ˆ๋‹ค. โ€œ์ง€๋‚œ๋‚ ์˜ ์ œ๊ตญ์ฃผ์˜๋‚˜ ํ˜„๋Œ€์ œ๊ตญ์ฃผ์˜๋‚˜ ๋‹ค ๊ฐ™์ด ๋…์ ์ด ๊ฒฝ์ œ์ƒํ™œ ์ „๋ฐ˜์„ ์ง€๋ฐฐํ•˜๊ณ  ์žˆ์€ ๋…์ ์ž๋ณธ์ฃผ์˜๋ผ๋Š” ์ ์—์„œ๋Š” ์ฐจ์ด๊ฐ€ ์—†์Šต๋‹ˆ๋‹ค. ์ž๋ณธ์ฃผ์˜ ์‚ฌํšŒ์—์„œ ๊ฒฝ์ œ๊ด€๋ฆฌ๋Š” ์ž๋ณธ๊ฐ€๋“ค์˜ ์ด์œค์ถ”๊ตฌ๋ฅผ ์œ„ํ•˜์—ฌ ๊ทผ๋กœ์ž๋“ค์„ ์–ต์••ํ•˜๊ณ  ์ฐฉ์ทจํ•˜๋Š” ๊ธฐ๋Šฅ์œผ๋กœ ๋ฉ๋‹ˆ๋‹ค.โ€ - ์œ„๋Œ€ํ•œ ์˜๋„์ž ๊น€์ •์ผ ๋™์ง€๊ป˜์„œ๋Š” ๋‹ค์Œ๊ณผ ๊ฐ™์ด ์ง€์ ํ•˜์‹œ์—ˆ๋‹ค. โ€œ์ƒ์‚ฐ์ˆ˜๋‹จ์ด ์ธ๋ฏผ์˜ ์†Œ์œ ๋กœ ๋˜์–ด ์žˆ๋Š” ์‚ฌํšŒ์ฃผ์˜์—์„œ๋Š” ์ธ๋ฏผ์˜ ๋Œ€ํ‘œ์ž์ธ ๊ตญ๊ฐ€๊ฐ€ ๋งˆ๋•…ํžˆ ๊ฒฝ์ œ๋ฅผ ํ†ต์ผ์ ์œผ๋กœ ๊ด€๋ฆฌ ์šด์˜ํ•˜์—ฌ์•ผ ํ•ฉ๋‹ˆ๋‹ค.โ€ - ์ด์™€ ๊ฐ™์€ ๋ฌธ๊ฑด์€ ๊น€๋ถ€์ž๋ฅผ ์ฐฌ์–‘ํ•˜๊ณ  ๋ถํ•œ์˜ ์‚ฌํšŒ์ฃผ์˜์  ๊ฒฝ์ œ, ์ฒด์ œ ๋ฐ ์ฃผ์ฒด์‚ฌ์ƒ ๋“ฑ์˜ ์ •๋‹น์„ฑ์„ ์„ ์ „ํ•˜๋ฉฐ ๋ฏธํ™”ยท์ฐฌ์–‘ํ•˜๋Š” ๋‚ด์šฉ์ด๋‹ค. ใ€ŠBOOKS ํด๋” ์ค‘ ์†Œ์„ค ํด๋” ๋‚ด โ€˜1932๋…„โ€™์˜ ์ฃผ์š” ๋‚ด์šฉใ€‹ - ๋ถํ•œ์—์„œ ๊น€์ผ์„ฑ ์šฐ์ƒํ™”๋ฅผ ์œ„ํ•ด ๋งŒ๋“  ์ด์„œ โ€˜๋ถˆ๋ฉธ์˜ ๋ ฅ์‚ฌโ€™ ์ค‘ ํ•˜๋‚˜๋กœ ๊น€์ผ์„ฑ์ด 1932. 4. โ€˜์กฐ์„ ์ธ๋ฏผํ˜๋ช…๊ตฐโ€™์ด๋ž€ ํ•ญ์ผ๋ฌด์žฅ ์œ ๊ฒฉ๋Œ€๋ฅผ ์ฐฝ์„คํ•œ ์ดํ›„ ๋™๋งŒ์ฃผ ์ง€์—ญ์„ ๋ฌด๋Œ€๋กœ ํ•˜์—ฌ ๋‹น์‹œ ํ˜๋ช…์ „์‚ฌ๋“ค์˜ ํ’๋ชจ, ๋ถˆ๊ตด์˜ ๊ธฐ์ƒ, ๋์—†๋Š” ํ—Œ์‹ ์„ฑ ๋“ฑ ํ•ญ์ผ๋ฌด์žฅ์œ ๊ฒฉ ํ™œ๋™์ƒ๊ณผ ์œ ๊ฒฉ๋Œ€์› ๋ฐ ์œ ๊ฒฉ๊ตฌ๋‚ด ์ฃผ๋ฏผ์— ๋Œ€ํ•œ ์ •์น˜๊ตฐ์‚ฌ ํ™œ๋™์„ ์ฐฌ์–‘ยท๋ฏธํ™”ํ•˜๋Š” ๋‚ด์šฉ์ด๋‹ค. ใ€Š๊ต์–‘ ํด๋” ์ค‘ ์ •์ˆ˜ ํด๋” ๋‚ด โ€˜์กฐ์„ ๋กœ๋™๋‹น๋žต์‚ฌโ€™์˜ ์ฃผ์š” ๋‚ด์šฉใ€‹ - ์œ„๋Œ€ํ•œ ์ˆ˜๋ น ๊น€์ผ์„ฑ ๋™์ง€๊ป˜์„œ๋Š” ๊ฒน์Œ“์ธ ๋‚œ๊ด€๊ณผ ์‹œ๋ จ์„ ํ—ค์น˜์‹œ๋ฉด์„œ ํ•ญ์ผ๋ฌด์žฅํˆฌ์Ÿ๊ณผ ์ธ๋ฏผ๋Œ€์ค‘์˜ ๋ฐ˜์ผ๋ฐ˜์ „ํˆฌ์Ÿ์„ ์กฐ์ง๋ น๋„ํ•˜์‹œ์˜€๋‹ค. ์œ„๋Œ€ํ•œ ์ˆ˜๋ น ๊น€์ผ์„ฑ ๋™์ง€๊ป˜์„œ ์กฐ์ง ๋ น๋„ํ•˜์‹  ํ•ญ์ผ๋ฌด์žฅํˆฌ์Ÿ์€ ์ฃผ์ฒด์‚ฌ์ƒ์˜ ํ˜๋ช…์  ๊ธฐ์น˜ ๋ฐ‘์— ์ง„ํ–‰๋œ ์ฒซ ๋ฏผ์กฑํ•ด๋ฐฉ์ „์Ÿ, ํ˜๋ช…์ „์Ÿ์œผ๋กœ์„œ ์กฐ์„ ๊ณต์‚ฐ์ฃผ์˜์ž๋“ค๊ณผ ์กฐ์„ ์ธ๋ฏผ์˜ ํ˜๋ช…์  ๊ธฐ๊ฐœ๋ฅผ ์˜จ ์„ธ์ƒ์— ๋–จ์น˜๊ณ  ์šฐ๋ฆฌ๋ฏผ์กฑ์˜ ์กด์—„๊ณผ ์˜์˜ˆ๋ฅผ ํฌ๊ฒŒ ๋น›๋‚ด์ธ ์ž๋ž‘์ฐฌ ํˆฌ์Ÿ์ด์˜€๋‹ค. - ๋ฏผ์กฑ์˜ ํƒœ์–‘์ด์‹  ์œ„๋Œ€ํ•œ ์ˆ˜๋ น ๊น€์ผ์„ฑ ๋™์ง€์— ๋Œ€ํ•œ ๋‹คํ•จ์—†๋Š” ๊ฒฝ๋ชจ์˜ ์ •์„ ๊นŠ์ด ๊ฐ„์งํ•œ ๋‚จ๋ฐ˜๋ถ€ ์ธ๋ฏผ๋“ค์€ ๋ถ๋ฐ˜๋ถ€์—์„œ์˜ ์‚ฌํšŒ์ฃผ์˜ ๊ฑด์„ค์„ฑ๊ณผ์— ๊ณ ๋ฌด๋˜์–ด ๋ฏธ์ œ๋ฅผ ๋ชฐ์•„๋‚ด๊ณ  ๊ทธ ์•ž์žก์ด๋“ค์˜ ๊ตฐ์‚ฌํŒŒ์‘ˆ๋…์žฌ๋ฅผ ๋’ค์ง‘์–ด์—Ž์œผ๋ฉฐ ์กฐ๊ตญ์˜ ์ž์ฃผ์  ํ‰ํ™”ํ†ต์ผ์„ ์ด๋ฃฉํ•˜๊ธฐ ์œ„ํ•˜์—ฌ ๋”์šฑ ์–ต์„ธ๊ฒŒ ์‹ธ์›Œ๋‚˜๊ฐ”๋‹ค. ์ž์ฃผ, ํ‰ํ™”ํ†ต์ผ, ๋ฏผ์กฑ๋Œ€๋‹จ๊ฒฐ์˜ 3๋Œ€์›์น™์€ ์ „์ฒด ์กฐ์„ ์ธ๋ฏผ์˜ ํ•œ๊ฒฐ๊ฐ™์€ ์˜์‚ฌ์™€ ๋…์›์„ ๋ฐ˜์˜ํ•œ ๋ฏผ์กฑ๊ณต๋™์˜ ํ–‰๋™๊ฐ•๋ น์ด๋‹ค. - ๋กœ๋™์ž, ๋†๋ฏผ, ์ฒญ๋…„ํ•™์ƒ๋“ค์˜ ํˆฌ์Ÿ์— ๋ฐœ๋งž์ถ”์–ด ๋‚จ์กฐ์„ ์˜ ํ•™๊ณ„, ์–ธ๋ก ๊ณ„, ๋ฒ•์กฐ๊ณ„, ์ข…๊ต๊ณ„ ์ธ์‚ฌ๋“ค์„ ๋น„๋กฏํ•˜์—ฌ ์ผ๋ถ€ ์šฐ์ต์ •๊ณ„์ธ์‚ฌ๋“ค ๊นŒ์ง€๋„ ๋ฐ˜ํŒŒ์‘ˆ ๋ฏผ์ฃผํ™”์™€ ์กฐ๊ตญ์˜ ์ž์ฃผ์  ํ†ต์ผ์„ ์œ„ํ•œ ํˆฌ์Ÿ์— ํ•ฉ๋ฅ˜ํ•ด ๋‚˜์„œ ๋‚จ์กฐ์„ ์—์„œ ๋ฐ˜ํŒŒ์‘ˆ ๋ฏผ์ฃผํ™”ํˆฌ์Ÿ ๋Œ€๋ ฌ์ด ๋Š˜์–ด๋‚˜๊ณ  ๋ฏผ์ฃผํ™”์šด๋™์ด ๋ณด๋‹ค ๊ด‘๋ฒ”ํ•œ ๋Œ€์ค‘์  ๊ธฐ๋ฐ˜ ์œ„์—์„œ ์ง„ํ–‰๋˜๊ฒŒ ๋˜์—ˆ์œผ๋ฉฐ ์ด ์šด๋™์˜ ์กฐ์งํ™”๊ณผ์ •์ด ๊ฐ ๊ณ„์ธต์˜ ๊ณต๋™ํ–‰๋™๊ณผ ๋ จ๋Œ€์„ฑ์ด ๋‚ ๋กœ ๊ฐ•ํ™”๋˜๊ฒŒ ๋˜์—ˆ๋‹ค. - ์ด์™€ ๊ฐ™์€ ๋ฌธ๊ฑด์€ ์กฐ์„ ๋…ธ๋™๋‹น์˜ ํ™œ๋™์„ ์„ค๋ช…ํ•˜๋ฉด์„œ ๊น€์ผ์„ฑ์„ ์ฐฌ์–‘ํ•˜๊ณ  ๋ถํ•œ์˜ ๋Œ€๋‚จํ˜๋ช…ํˆฌ์Ÿ ๋“ฑ์„ ์„ ์ „ยท์„ ๋™ํ•˜๋Š” ๋‚ด์šฉ์ด๋‹ค. ใ€Š์ €์ž‘์ง‘ ํด๋” ์ค‘ ๊น€์ผ์„ฑ์ €์ž‘์ง‘์ฃผ์ œ๋ถ„๋ฅ˜ ํด๋” ๋‚ด โ€˜ํ˜„๋Œ€์กฐ์„ ์—ญ์‚ฌโ€™์˜ ์ฃผ์š” ๋‚ด์šฉใ€‹ - ๊น€์ผ์„ฑ์ด ์กฐ์ง ์˜๋„ํ•œ ํ•ญ์ผ๋ฌด์žฅํˆฌ์Ÿ์€ ์ฃผ์ฒด์‚ฌ์ƒ์˜ ํ˜๋ช…์  ๊ธฐ์น˜ ๋ฐ‘์— ์ „๊ฐœ๋œ ๋ฏผ์กฑํ•ด๋ฐฉ์ „์Ÿ, ํ˜๋ช…์ „์Ÿ์œผ๋กœ์„œ ์กฐ์„ ๊ณต์‚ฐ์ฃผ์˜์ž ๋“ค๊ณผ ์กฐ์„ ์ธ๋ฏผ์˜ ํ˜๋ช…์  ๊ธฐ๊ฐœ๋ฅผ ์˜จ ์„ธ์ƒ์— ๋–จ์น˜๊ณ  ์šฐ๋ฆฌ๋ฏผ์กฑ์˜ ์กด์—„๊ณผ ์˜์˜ˆ๋ฅผ ํฌ๊ฒŒ ๋น›๋‚ธ ์ž๋ž‘์ฐฌ ํˆฌ์Ÿ์ด์—ˆ๋‹ค. - ๊น€์ผ์„ฑ์€ ํ•ญ์ผํ˜๋ช… ํˆฌ์Ÿ์‹œ๊ธฐ์— ์ œ์‹œํ•œ ์ฃผ์ฒด์ ์ธ ์ •๊ถŒ ๊ฑด์„ค๋…ธ์„ ๊ณผ ํ•ด๋ฐฉ์ง€๊ตฌ์—์„œ์˜ ์ธ๋ฏผํ˜๋ช… ์ •๋ถ€ ๊ฑด์„ค๊ฒฝํ—˜ ๊ทธ๋ฆฌ๊ณ  ํ•ด๋ฐฉ ํ›„ ์กฐ์„ฑ๋œ ์ •์„ธ๋ฅผ ํ†ต์ฐฐํ•œ๋ฐ ๊ธฐ์ดˆํ•˜์—ฌ ์ •๊ถŒ๋ฌธ์ œ ํ•ด๊ฒฐ์˜ ์˜ณ์€ ๊ธธ์„ ๋ฐํ˜€ ์ฃผ์—ˆ๊ณ  ๋‚จ์กฐ์„  ์ธ๋ฏผ๋“ค์—๊ฒŒ ํฐ ํ˜๋ช…์  ์˜ํ–ฅ์„ ์ฃผ์—ˆ์œผ๋ฉฐ ๊ทธ๋“ค์˜ ํ•ด๋ฐฉํˆฌ์Ÿ์„ ํž˜ ์žˆ๊ฒŒ ๊ณ ๋ฌดยท์ถ”๋™ํ•˜์˜€๋‹ค. - ๋‚จ์กฐ์„ ์„ ๊ตฐ์‚ฌ ๊ธฐ์ง€ํ™”ํ•˜๋Š” ๊ฒƒ์€ ๋ฏธ์ œ๊ฐ€ ๋‚จ์กฐ์„ ์„ ๊ฐ•์ ํ•œ ์ฒซ๋‚ ๋ถ€ํ„ฐ ์ถ”์ง„์‹œ์ผœ ์˜จ ์นจ๋žต์ •์ฑ…์ด์—ˆ๋‹ค. ๋ฏธ์ œ๋Š” โ€˜๋ถ๋ฒŒโ€™๊ณ„ํš์„ ๋งŒ๋“ค์–ด ๋†“๊ณ  ๊ทธ๊ฒƒ์„ ์‹ค์ฒœํ•˜๊ธฐ ์œ„ํ•˜์—ฌ 38๋„์„  ๋ฌด์žฅ์นจ์Šต์‚ฌ๊ฑด์„ ๊ทธ ์–ด๋Š ๋•Œ๋ณด๋‹ค๋„ ๋Œ€๋Œ€์ ์œผ๋กœ ์ผ์œผ์ผฐ๋‹ค. ๋ฏธ์ œ์™€ ์ด์Šน๋งŒ ์ •๊ถŒ์€ 1950. 6. 25. ์กฐ์„ ์ธ๋ฏผ๋ฏผ์ฃผ์ฃผ์˜๊ณตํ™”๊ตญ์„ ๋ฐ˜๋Œ€ํ•˜๋Š” ์นจ๋žต์ „์Ÿ์„ ์ผ์œผ์ผฐ๋‹ค. - ๋ฏธ์ œ๊ตญ์ฃผ์˜์ž๋“ค์€ ์นจ๋žต์ „์Ÿ์„ ํ†ตํ•˜์—ฌ ๋ถ๋ฐ˜๋ถ€ ์ง€์—ญ์„ ๊ฐ•์ ํ•จ์œผ๋กœ์จ ์กฐ์„ ์ธ๋ฏผ๋ฏผ์ฃผ์ฃผ์˜๊ณตํ™”๊ตญ์„ ๋ง์‚ดํ•˜๊ณ  ์ „ ์กฐ์„ ์„ ์ €๋“ค์˜ ์‹๋ฏผ์ง€๋กœ ๋งŒ๋“ค๋ ค๊ณ  ํƒ€์‚ฐํ•˜์˜€๋‹ค. ๋ฏธ์ œ์™€ ๊ทธ ์ฃผ๊ตฌ๋“ค์˜ ๋ฌด๋ ฅ์นจ๊ณต์„ ๋ฐ˜๋Œ€ํ•˜๋Š” ์กฐ์„ ์ธ๋ฏผ์˜ ์ •์˜์˜ ์ „์Ÿ์€ ์กฐ๊ตญ์˜ ์ž์œ ์™€ ๋…๋ฆฝ์„ ์ˆ˜ํ˜ธํ•˜๊ธฐ ์œ„ํ•œ ๋ฏผ์กฑํ•ด๋ฐฉ์ „์Ÿ์ด์—ˆ์œผ๋ฉฐ ์ธ๋ฏผ๋ฏผ์ฃผ์ฃผ์˜์ œ๋„๋ฅผ ์ „๋ณตํ•˜๋ ค๋Š” ์ธ๋ฏผ์˜ ์›์ˆ˜๋“ค์„ ๋ฐ˜๋Œ€ํ•˜๋Š” ๊ณ„๊ธ‰ํˆฌ์Ÿ์ด์—ˆ๋‹ค. - ์กฐ๊ตญํ•ด๋ฐฉ์ „์Ÿ์—์„œ์˜ ์กฐ์„ ์ธ๋ฏผ์˜ ์Šน๋ฆฌ๋Š” ๋ฌด์—‡๋ณด๋‹ค๋„ ๊น€์ผ์„ฑ์˜ ์œ„๋Œ€ํ•œ ์ฃผ์ฒด์‚ฌ์ƒ์˜ ๋น›๋‚˜๋Š” ์Šน๋ฆฌ์˜€๋‹ค. ๊น€์ผ์„ฑ์˜ ์˜๋„ ๋ฐ‘์— 3๋…„๊ฐ„์˜ ์กฐ๊ตญํ•ด๋ฐฉ์ „์Ÿ์—์„œ ๋ฏธ์ œ๋ฅผ ํƒ€์Šนํ•˜๊ณ  ์ •์ „ํ˜‘์ •์„ ์ฒด๊ฒฐํ•จ์œผ๋กœ์จ ์กฐ๊ตญ์˜ ํ†ต์ผ๋ฌธ์ œ๋ฅผ ํ‰ํ™”์ ์œผ๋กœ ํ•ด๊ฒฐํ•  ์ˆ˜ ์žˆ๋Š” ๊ฐ€๋Šฅ์„ฑ์„ ์–ป๊ฒŒ ๋˜์—ˆ๋‹ค. - ์ด์™€ ๊ฐ™์€ ๋ฌธ๊ฑด์€ ๊น€์ผ์„ฑ์˜ ํ•ญ์ผ๋ฌด์žฅํ˜๋ช… ํˆฌ์Ÿ์„ ์ฐฌ์–‘ยท๋ฏธํ™”ํ•˜๋Š” ํ•œํŽธ 6ยท25์ „์Ÿ์„ ๋ฏธ๊ตญ์˜ ๋ถ์นจ์— ๋งž์„  ๊น€์ผ์„ฑ์ด ์ด๋ˆ โ€˜์กฐ๊ตญํ•ด๋ฐฉ์ „์Ÿโ€™์œผ๋กœ ์™œ๊ณกํ•˜๋Š” ๋“ฑ ๊น€์ผ์„ฑ ๊ฐœ์ธ์˜ ์šฐ์ƒํ™”๋ฅผ ์œ„ํ•œ ๋‚ด์šฉ์ด๋‹ค. ใ€Š์ €์ž‘์ง‘ ํด๋” ์ค‘ ๊น€์ •์ผ์ €์ž‘์ง‘ ํด๋” ๋‚ด โ€˜๊น€์ •์ผ ์ €์ž‘์ง‘ 1๊ถŒโ€™์˜ ์ฃผ์š” ๋‚ด์šฉใ€‹ - ์ˆ˜๋ น๋‹˜์˜ ๊ต์‹œ์™€ ๋‹น์˜ ๋ฐฉ์นจ์„ ๊นŠ์ด ์—ฐ๊ตฌํ•˜๊ณ  ์•„๋ž˜์‹ค์ •์„ ์ „๋ฉด์ ์œผ๋กœ ๊ตฌ์ฒด์ ์œผ๋กœ ํŒŒ์•…ํ•˜์—ฌ์•ผ ํ•˜๋ฉฐ ๊ทธ์— ๊ธฐ์ดˆํ•˜์—ฌ ์ง‘์ฒด์ ์ธ ํ˜‘์˜๋ฅผ ๊ฐ•ํ™”ํ•˜์—ฌ์•ผ ํ•œ๋‹ค. - ๋‹น์‚ฌ์—… ๋ฐฉ๋ฒ•๊ณผ ์ž‘ํ’์„ ๋ฐ”๋กœ ์žก์œผ๋ ค๋ฉด ๋ชจ๋“  ์ผ๊ตฐ๋“ค์˜ ์œ„๋Œ€ํ•œ ์ˆ˜๋ น๋‹˜์˜ ์‚ฌ์—…๋ฐฉ๋ฒ•์„ ๋”ฐ๋ผ ๋ฐฐ์›Œ ๋‹น์‚ฌ์—…์„ ์ฒ ์ €ํžˆ ์‚ฌ๋žŒ๊ณผ์˜ ์‚ฌ์—…์œผ๋กœ ์ „ํ™˜ํ•˜์—ฌ์•ผ ํ•œ๋‹ค. - ์ด์™€ ๊ฐ™์€ ๋ฌธ๊ฑด์€ 1964. 4. ~ 1999. 9.๊ฐ„ ๊น€์ •์ผ์˜ 317ํšŒ์— ๊ฑธ์นœ ์–ธ๋™ํ•œ ๋‚ด์šฉ์„ ์ˆ˜๋กํ•œ ๋ถํ•œ ์›์ „์ธ ๊น€์ •์ผ ์ €์ž‘์ง‘์œผ๋กœ, ๊น€์ •์ผ ๊ฐœ์ธ์˜ ์šฐ์ƒํ™”๋ฅผ ์œ„ํ•ด ๊น€์ •์ผ์˜ ์ฃผ์š” ๋‹ดํ™” ๋‚ด์šฉ์„ ๋ฐœ์ทŒยท์ •๋ฆฌํ•˜์—ฌ ์ฐฌ์–‘ยท์„ ์ „ํ•˜๋Š” ๋‚ด์šฉ์ด๋‹ค. ์ด์™€ ๊ฐ™์ด ์œ ์—์Šค๋น„(USB 8G) ํ˜•์‹์˜ ๋””์ง€ํ„ธ์ €์žฅ๋งค์ฒด๋Š” ๊น€์ผ์„ฑ์˜ ์ „ ๊ฐ€๊ณ„๋ฅผ ํ•ญ์ผ ๋…๋ฆฝํˆฌ์‚ฌ๋กœ ๋ฌ˜์‚ฌํ•˜๊ณ  ๊น€์ผ์„ฑ์˜ ํ•ญ์ผ๋ฌด์žฅํˆฌ์Ÿ์„ ๋ฏธํ™”ยท์ฐฌ์–‘ํ•˜๋Š” ๋“ฑ ๊น€์ผ์„ฑ์„ ์šฐ์ƒํ™”ํ•˜๋Š” ์ทจ์ง€์˜ ๋‚ด์šฉ, 6ยท25์ „์Ÿ์„ ๋ฏธ๊ตญ์— ์˜ํ•œ ์นจ๋žต์ „์Ÿ์ธ ์–‘ ์™œ๊ณกํ•˜๋ฉด์„œ ๊น€์ผ์„ฑ์ด ๋ฏธ๊ตญ์˜ ์นจ๋žต์„ ๋ง‰์•„๋‚ธ ์กฐ๊ตญํ•ด๋ฐฉ์ „์Ÿ์ด์—ˆ๋‹ค๋Š” ์ทจ์ง€๋กœ ์„ ์ „ํ•˜๋Š” ๋‚ด์šฉ, ์กฐ์„ ๋กœ๋™๋‹น์˜ ์ผ๋‹น๋…์žฌ๋ฅผ ์ •๋‹นํ™”ํ•˜๋ฉด์„œ ๋Œ€๋‚จํ˜๋ช…๋…ธ์„ ์„ ์„ ์ „ยท์„ ๋™ํ•˜๋Š” ์ทจ์ง€์˜ ๋‚ด์šฉ, ๋ถํ•œ์‹ ์‚ฌํšŒ์ฃผ์˜๊ฐ€ ์ฃผ์ฒด์‚ฌ์ƒ์œผ๋กœ ๋ฌด์žฅํ•œ ๋ถํ•œ ์ธ๋ฏผ๋“ค์— ์˜ํ•ด ์ ˆ๋Œ€์  ์ง€์ง€๋ฅผ ๋ฐ›๊ณ  ์žˆ๋‹ค๋Š” ์ทจ์ง€๋กœ ๋ถํ•œ ์‚ฌํšŒ์ฃผ์˜์ฒด์ œ์˜ ์šฐ์›”์„ฑ ๋ฐ ํ˜๋ช…์  ์ •๋‹น์„ฑ ๋“ฑ์„ ๊ฐ•์กฐํ•˜๋Š” ์ทจ์ง€์˜ ๋‚ด์šฉ, ๊น€์ผ์„ฑ ์ฃผ์ฒด์‚ฌ์ƒ์˜ ์šฐ์›”์„ฑ์„ ๊ฐ•์กฐํ•˜๋ฉด์„œ ๋ฏธ ์ œ๊ตญ์ฃผ์˜์— ์˜ˆ์†๋œ ๋‚จํ•œ ๋ฏผ์ค‘์„ ํ•ด๋ฐฉํ•˜๊ณ  ํ•œ๋ฐ˜๋„์˜ ํ†ต์ผ์„ ์‹คํ˜„ํ•˜๊ธฐ ์œ„ํ•ด ๋‚จํ•œ ๋ฏผ์ค‘์„ ์ฃผ์ฒด์‚ฌ์ƒ์œผ๋กœ ์˜์‹ํ™”ํ•˜์—ฌ์•ผ ํ•œ๋‹ค๋Š” ์ทจ์ง€์˜ ๋‚ด์šฉ, ์ž๋ณธ์ฃผ์˜๊ฒฝ์ œ์ œ๋„๋Š” ์ธ๋ฏผ๋Œ€์ค‘์„ ์ˆ˜ํƒˆํ•˜๊ธฐ ์œ„ํ•œ ๋„๊ตฌ์— ๋ถˆ๊ณผํ•˜๊ณ  ์ฃผ์ฒด์‚ฌ์ƒ์ด ๊ทผ๊ฐ„์ด ๋œ ๋ถํ•œ์˜ ์‚ฌํšŒ์ฃผ์˜ ์ฒด์ œ๋Š” ๋ชจ๋“  ๊ตฌ์„ฑ์›๋“ค์ด ํ‰๋“ฑํ•˜๊ณ  ํ’์กฑํ•œ ๋ฌผ์งˆ์  ๋ฌธํ™”์ƒํ™œ์ด ๋ณด์žฅ๋˜๋Š” ์‚ฌํšŒ๋ผ๊ณ  ๋ฏธํ™”ํ•˜๋ฉด์„œ ๋ถํ•œ ์‚ฌํšŒ์ฃผ์˜ ์ฒด์ œ์™€ ์ฃผ์ฒด์‚ฌ์ƒ์„ ์ฐฌ์–‘ยท์„ ์ „ํ•˜๋Š” ์ทจ์ง€์˜ ๋‚ด์šฉ, ๊น€์ผ์„ฑยท๊น€์ •์ผ์ฃผ์˜๋ฅผ ์ถ”์ข…ํ•˜๋Š” ๋ถํ•œ ์‚ฌํšŒ์ฃผ์˜์ฒด์ œ์˜ ์šฐ์ˆ˜์„ฑ ๋ฐ ์ฃผ์ฒด์‚ฌ์ƒ๊ณผ ์„ ๊ตฐ์ •์น˜๋ฅผ ์„ ์ „ํ•˜๋Š” ์ทจ์ง€์˜ ๋‚ด์šฉ, ๋ถํ•œ์˜ ๋Œ€๋‚จํ˜๋ช…๋ก ์˜ ์ผํ™˜์œผ๋กœ ์ฃผํ•œ๋ฏธ๊ตฐ์˜ ์ฒ ์ˆ˜๋ฅผ ์ฃผ์žฅํ•˜๊ณ  ๋ถํ•œ์˜ ์—ฐ๋ฐฉ์ œ ํ†ต์ผ๋ฐฉ์‹์˜ ์ •๋‹น์„ฑ ๋“ฑ์„ ์„ ์ „ยท์„ ๋™ํ•˜๋Š” ์ทจ์ง€์˜ ๋‚ด์šฉ, ๊น€๋ถ€์ž์— ๋Œ€ํ•œ ์˜์›ํ•œ ์ถฉ์„ฑ์„ ๋งน์„ธํ•˜๋Š” ์ทจ์ง€์˜ ๋‚ด์šฉ์— ๊ด€ํ•œ ๋ฌธ๊ฑดํŒŒ์ผ์„ ์ˆ˜๋กํ•˜๊ณ  ์žˆ๋‹ค. ์ด๋กœ์จ ํ”ผ๊ณ ์ธ์€ ๊ตญ๊ฐ€์˜ ์กด๋ฆฝยท์•ˆ์ „์ด๋‚˜ ์ž์œ ๋ฏผ์ฃผ์  ๊ธฐ๋ณธ์งˆ์„œ๋ฅผ ์œ„ํƒœ๋กญ๊ฒŒ ํ•œ๋‹ค๋Š” ์ •์„ ์•Œ๋ฉด์„œ ๋ฐ˜๊ตญ๊ฐ€๋‹จ์ฒด๋‚˜ ๊ทธ ๊ตฌ์„ฑ์› ๋˜๋Š” ๊ทธ ์ง€๋ น์„ ๋ฐ›์€ ์ž์˜ ํ™œ๋™์„ ์ฐฌ์–‘ยท๊ณ ๋ฌดยท์„ ์ „ ๋˜๋Š” ์ด์— ๋™์กฐํ•  ๋ชฉ์ ์œผ๋กœ ์œ„ ์ด์ ํ‘œํ˜„๋ฌผ์„ ์†Œ์ง€ํ•˜์˜€๋‹ค. ๋‹ค. ์œ ์—์Šค๋น„(USB 32G) ์†Œ์ง€ ํ”ผ๊ณ ์ธ์€ 2015. 11. 13.๊ฒฝ ๊ด‘๋ช…์‹œ (์ฃผ์†Œ 3 ์ƒ๋žต)์— ์žˆ๋Š” ํ”ผ๊ณ ์ธ์˜ ์ฃผ๊ฑฐ์ง€์—์„œ ๊น€๋ถ€์ž ๋ฐ ๋ถํ•œ์ฒด์ œ๋ฅผ ์ฐฌ์–‘ยท๋ฏธํ™”ํ•˜๊ฑฐ๋‚˜, ์ฃผ์ฒด์‚ฌ์ƒ์— ์ž…๊ฐํ•œ ๋Œ€๋‚จํ˜๋ช…ํˆฌ์Ÿ์„ ์„ ์ „ยท๋™์กฐํ•˜๋Š” ๋“ฑ์˜ ๊ฐ์ข… ํŒŒ์ผ๋“ค์„ ์ฒด๊ณ„์ ์œผ๋กœ ์ •๋ฆฌยท์ €์žฅํ•œ ์œ ์—์Šค๋น„(USB 32G) ํ˜•์‹์˜ ๋””์ง€ํ„ธ์ €์žฅ๋งค์ฒด๋ฅผ ๋ณด๊ด€ํ•˜๊ณ  ์žˆ์—ˆ๋‹ค. ์œ„ ์ €์žฅ๋งค์ฒด๋Š” ใ€Œ1 readingsใ€ยทใ€Œ5๋‹จ๊ณ„ํ•™์Šต๊ณผ์ •ใ€ยทใ€Œ10๋‹จ๊ณ„ํ•™์Šตใ€ยทใ€Œgreatnessใ€ยทใ€ŒK1์ €์ž‘์ง‘hwpํŒใ€ยทใ€ŒK2์ €์ž‘์ง‘hwpํŒใ€ยทใ€Œ(๋ช…์นญ 38 ์ƒ๋žต)ใ€ยทใ€Œrojak2012ใ€ยทใ€Œstudy2012ใ€ยทใ€Œstudy2014ใ€ยทใ€Œstudy2015ใ€ยทใ€Œ(๋ช…์นญ 39 ์ƒ๋žต)ใ€ยทใ€ŒYOUTUBEใ€ยทใ€Œ๊ต์–‘ใ€ยทใ€Œ์ €์ž‘์ง‘ใ€ยทใ€Œํ”Œ๋ž˜์‹œ์˜์ƒใ€ ๋“ฑ์˜ ์ƒ์œ„ ํด๋”๋ฅผ ๊ตฌ์„ฑํ•˜์—ฌ ๋ฌธ๊ฑดยท์Œ์„ฑยท๋™์˜์ƒ ๋“ฑ์˜ ํŒŒ์ผ๋“ค์ด ์ €์žฅ๋˜์–ด ์žˆ๋‹ค. ์œ„ ์ƒ์œ„ ํด๋” ์ค‘ ใ€Œ1 readingsใ€ ํด๋”๋Š” ๊น€๋ถ€์ž๋ฅผ ์ฐฌ์–‘ํ•˜๋Š” ์ž๋ฃŒ๋“ค์ด ์ €์žฅ๋œ ใ€Œ๋…ธ์ž‘ใ€ยทใ€Œ์œ„๋Œ€์„ฑใ€๊ณผ ๊ฐ™์€ ํ•˜์œ„ ํด๋”๋กœ ๋‚˜๋ˆ„์–ด์ ธ โ€˜์ƒˆ์‹œ๋Œ€ ์ •์น˜ํ•™์›๋ก โ€™, โ€˜์œ„๋Œ€ํ•œ ์ˆ˜๋ น ๊น€์ผ์„ฑ๋™์ง€ ํ˜๋ช…ํ™œ๋™๋žต๋ ฅโ€™ ๋“ฑ ๊น€๋ถ€์ž ์šฐ์ƒํ™”์™€ ๋ถํ•œ ์ฒด์ œ์˜ ์šฐ์›”์„ฑ์„ ์„ ์ „ํ•˜๋Š” ์ž๋ฃŒ๋“ค์ด ์ €์žฅ๋˜์–ด ์žˆ๋‹ค. ์œ„ ์ƒ์œ„ ํด๋” ์ค‘ ใ€Œ5๋‹จ๊ณ„ํ•™์Šต๊ณผ์ •ใ€ ํด๋”๋Š” ๋‹ค์‹œ 1~5๋‹จ๊ณ„๊นŒ์ง€ ๊ฐ ๋‹จ๊ณ„๋ณ„ ํ•˜์œ„ ํด๋” ๋ฐ ใ€Œ๊ธฐ๋ณธ์Œ์•…ใ€ยทใ€Œํ•„๋…์†Œ์„ค์˜์ƒใ€ ํ•˜์œ„ ํด๋” ๋“ฑ์œผ๋กœ ๊ตฌ์„ฑ๋˜์–ด ์žˆ๋Š”๋ฐ, ใ€Œ1๋‹จ๊ณ„-๋ณ€ํ˜๊ด€ ํšŒ๊ณ ๋กใ€ ํ•˜์œ„ ํด๋”๋Š” ใ€Œ์ฃผ์ฒด์˜ ํ˜๋ช…๊ด€ใ€, ใ€ŒํšŒ๊ณ ๋ก ์„ธ๊ธฐ์™€ ๋”๋ถˆ์–ดใ€ ๋“ฑ 2๊ฐœ์˜ ์„ธ๋ถ€ ํด๋”๋กœ ๊ตฌ์„ฑ๋˜์–ด ๊น€์ผ์„ฑ ํšŒ๊ณ ๋ก โ€˜์„ธ๊ธฐ์™€ ๋”๋ถˆ์–ดโ€™ ๋ฐ โ€˜์ฃผ์ฒด์˜ ์ˆ˜๋ น๊ด€โ€™ ๋“ฑ ๊น€์ผ์„ฑ ์ฃผ์ฒด์‚ฌ์ƒ์„ ์ฐฌ์–‘ํ•˜๋Š” ์ž๋ฃŒ๋“ค์ด ์ €์žฅ๋˜์–ด ์žˆ๊ณ , ใ€Œ2๋‹จ๊ณ„-๋ณ€ํ˜์ด๋ก ์กฐ์ง์‚ฌ์—…ใ€ ํ•˜์œ„ ํด๋”๋Š” ใ€Œ4๋Œ€ ๊ฐœ๋ก ์„œใ€ 1๊ฐœ์˜ ์„ธ๋ถ€ ํด๋”๋กœ ๊ตฌ์„ฑ๋˜์–ด โ€˜์ฃผ์ฒด์˜ ํ•œ๊ตญ์‚ฌํšŒ ๋ณ€ํ˜์šด๋™๋ก โ€™, โ€˜์‚ฌํšŒ์ฃผ์˜์— ๋Œ€ํ•œ ์ฃผ์ฒด์  ๋ฆฌํ•ดโ€™ ๋“ฑ ๋ถํ•œ์˜ ๋Œ€๋‚จ์ „๋žต์ „์ˆ ์„ ์ง€์ง€ํ•˜๊ณ  ๋ถํ•œ์‹ ์‚ฌํšŒ์ฃผ์˜๋ฅผ ์˜นํ˜ธํ•˜๋Š” ๋“ฑ์˜ ์ž๋ฃŒ๋“ค์ด ์ €์žฅ๋˜์–ด ์žˆ์œผ๋ฉฐ, ใ€Œ3๋‹จ๊ณ„-์ฒ ํ•™์ •์น˜๊ฒฝ์ œ๋ณ€ํ˜์‚ฌใ€ ํ•˜์œ„ ํด๋”๋Š” ใ€Œ1 ์ฒ ํ•™ใ€, ใ€Œ2 ์ •์น˜๊ฒฝ์ œํ•™ใ€ 2๊ฐœ์˜ ์„ธ๋ถ€ ํด๋”๋กœ ๊ตฌ์„ฑ๋˜์–ด ๋ถํ•œ์˜ ์ฃผ์ฒด์‚ฌ์ƒ, ๊น€์ผ์„ฑ์ฃผ์˜์˜ ๋“ฑ์„ ์ฐฌ์–‘ํ•˜๋Š” ์ž๋ฃŒ๋“ค์ด ์ €์žฅ๋˜์–ด ์žˆ๊ณ , ใ€Œ4๋‹จ๊ณ„-์„ ๊ตฐ์‚ฌ์ƒใ€ ํ•˜์œ„ ํด๋”์—๋Š” โ€˜์œ„๋Œ€ํ•œ ์„ ๊ตฐ์‹œ๋Œ€โ€™, โ€˜์„ ๊ตฐ์ •์น˜์— ๋Œ€ํ•œ ๋ฆฌํ•ดโ€™ ๋“ฑ ๋ถํ•œ์˜ ์„ ๊ตฐ์‚ฌ์ƒ, ์„ ๊ตฐ์ •์น˜ ๋“ฑ์„ ๋ฏธํ™”ํ•˜๋Š” ์ž๋ฃŒ๊ฐ€ ์ €์žฅ๋˜์–ด ์žˆ์œผ๋ฉฐ, ใ€Œ5๋‹จ๊ณ„-์œ„๋Œ€์„ฑใ€ ํ•˜์œ„ ํด๋”์—๋Š” โ€˜์ ˆ์„ธ์˜ ์• ๊ตญ์ž ๊น€์ •์ผ์žฅ๊ตฐโ€™ ๋“ฑ ๊น€์ •์ผ์„ ์šฐ์ƒํ™”ํ•˜์—ฌ ์ฐฌ์–‘ํ•˜๋Š” ์ž๋ฃŒ๋“ค์ด ์ €์žฅ๋˜์–ด ์žˆ๊ณ , ๋˜ ๋‹ค๋ฅธ ํ•˜์œ„ ํด๋”์ธ ใ€Œ๊ธฐ๋ณธ์Œ์•…ใ€ ํ•˜์œ„ ํด๋”์—๋Š” โ€˜์–ด๋””์— ๊ณ„์‹ญ๋‹ˆ๊นŒ ๊ทธ๋ฆฌ์šด ์žฅ๊ตฐ๋‹˜โ€™ ๋“ฑ ๋ถํ•œ ๋ฐ ๊น€๋ถ€์ž๋ฅผ ์ฐฌ์–‘ํ•˜๋Š” ๋ถํ•œ ์Œ์•… ์ž๋ฃŒ๋“ค์ด ์ €์žฅ๋˜์–ด ์žˆ์œผ๋ฉฐ, ใ€Œํ•„๋…์†Œ์„ค์˜์ƒใ€ ํ•˜์œ„ ํด๋”์—๋Š” โ€˜์‚ผ์ฒœ๋ฆฌ๊ฐ•์‚ฐโ€™, โ€˜์• ๊ตญ์‹œ๋Œ€โ€™ ๋“ฑ ๋ถํ•œ ์ฒด์ œ ๋ฐ ๊น€์ผ์„ฑ์„ ์ฐฌ์–‘ํ•˜๋Š” ์†Œ์„ค์ด ์ €์žฅ๋˜์–ด ์žˆ๋‹ค. ์œ„ ์ƒ์œ„ ํด๋” ์ค‘ ใ€Œ10๋‹จ๊ณ„ํ•™์Šตใ€ ํด๋”๋Š” ๋‹ค์‹œ 1~10๋‹จ๊ณ„๊นŒ์ง€ ๊ฐ ๋‹จ๊ณ„๋ณ„ ํ•˜์œ„ํด๋” ๋ฐ ใ€Œ๋ฌธํ™”์˜ˆ์ˆ ใ€ยทใ€Œ์ €์ž‘์ง‘ใ€ ๋“ฑ์˜ ํ•˜์œ„ ํด๋”๋กœ ๋‚˜๋ˆ„์–ด์ ธ, ใ€Œ1๋‹จ๊ณ„-ํ˜๋ช…์ ํšŒ๊ณ ๋กใ€ ํ•˜์œ„ ํด๋”๋Š” ใ€Œ์‹œ๊ฐ๊ต์ •ใ€, ใ€Œ์ธํ„ฐ๋„ทํšŒ๊ณ ๋ก๊ฐ•์ขŒใ€, ใ€Œ์ฃผ์ฒด์˜ํ˜๋ช…๊ด€ใ€, ใ€ŒํšŒ๊ณ ๋กCDใ€, ใ€ŒํšŒ๊ณ ๋ก ์„ธ๊ธฐ์™€ ๋”๋ถˆ์–ดใ€ ๋“ฑ 5๊ฐœ์˜ ์„ธ๋ถ€ ํด๋”๋กœ ๊ตฌ์„ฑ๋˜์–ด ๊น€์ผ์„ฑ ํšŒ๊ณ ๋ก โ€˜์„ธ๊ธฐ์™€ ๋”๋ถˆ์–ดโ€™ ๋“ฑ ๋ถํ•œ ๋ฐ ๊น€๋ถ€์ž๋ฅผ ์ฐฌ์–‘ยท๋ฏธํ™”ํ•˜๋Š” ์ž๋ฃŒ๋“ค์ด ์ €์žฅ๋˜์–ด ์žˆ๊ณ , ใ€Œ2๋‹จ๊ณ„-๋ณ€ํ˜์ด๋ก 4๋Œ€๊ฐœ๋ก ใ€ ํ•˜์œ„ ํด๋”๋Š” ใ€Œ4๋Œ€ ๊ฐœ๋ก ์„œใ€, ใ€Œ์ •์„ธ๋ถ„์„ใ€, ใ€Œ์กฐ์ง๊ด€์‚ฌ์—…๋ฐฉ๋ฒ•ใ€, ใ€Œํ†ต์ผํ˜๋ช…๋ก ๊ฐ•์ขŒใ€ 4๊ฐœ์˜ ์„ธ๋ถ€ ํด๋”๋กœ ๊ตฌ์„ฑ๋˜์–ด ๊น€์ผ์„ฑ์˜ ํ•ญ์ผ์šด๋™์„ ์ฐฌ์–‘ยท๋ฏธํ™”ํ•˜๋Š” โ€˜ํ˜„๋Œ€์กฐ์„ ์—ญ์‚ฌโ€™ ๋“ฑ์˜ ์ž๋ฃŒ๋“ค์ด ์ €์žฅ๋˜์–ด ์žˆ์œผ๋ฉฐ, ใ€Œ3๋‹จ๊ณ„-์„ ๊ตฐ์‚ฌ์ƒใ€ ํ•˜์œ„ ํด๋”๋Š” ใ€Œ๊น€๋Œ€ํ•™๋ฐฉ์†ก ํŠน๊ฐ•ใ€, ใ€Œ๋…ธ๋ž˜์˜์ƒใ€ ใ€Œ๋…ธ์ž‘ใ€ ใ€Œ๋…ผ๋ฌธใ€ ์„ธ๋ถ€ ํด๋”๋กœ ๊ตฌ์„ฑ๋˜์–ด โ€˜๊น€์ •์ผ์žฅ๊ตฐ ์„ ๊ตฐ์ •์น˜ ๋ฆฌ๋ก โ€™ ๋“ฑ ์„ ๊ตฐ์ •์น˜์˜ ์šฐ์ˆ˜์„ฑ์„ ์ฐฌ์–‘ํ•˜๋Š” ์ž๋ฃŒ๋“ค์ด ์ €์žฅ๋˜์–ด ์žˆ๊ณ , ใ€Œ4๋‹จ๊ณ„-๊น€๋Œ€ํ•™๋ฐฉ์†ก๊ฐ•์ขŒใ€ ํ•˜์œ„ ํด๋”์—๋Š” โ€˜๊น€์ผ์„ฑ์ฃผ์˜๊ฐ•์ขŒโ€™ ๋“ฑ ๊น€๋ถ€์ž ๋ฐ ์ฃผ์ฒด์‚ฌ์ƒ ๋“ฑ์„ ์ฐฌ์–‘ํ•˜๋Š” ๋‚ด์šฉ์˜ ๊ฐ•์˜ ์ž๋ฃŒ๋“ค์ด ์ €์žฅ๋˜์–ด ์žˆ์œผ๋ฉฐ, ใ€Œ5๋‹จ๊ณ„-๊น€์ผ์„ฑ์ฃผ์˜ ์ด์„œใ€ ํ•˜์œ„ ํด๋”์—๋Š” โ€˜์ฃผ์ฒด์‚ฌ์ƒ์ด์„œโ€™(1~10๊ถŒ) ๋ฐ โ€˜๊น€์ผ์„ฑ์ฃผ์˜ ์ด์„œโ€™(1~10๊ถŒ) ๋“ฑ ๊น€์ผ์„ฑ์„ ์ฐฌ์–‘ํ•˜๋ฉฐ ์ฃผ์ฒด์‚ฌ์ƒ ๋ฐ ๊น€์ผ์„ฑ์ฃผ์˜ ๋“ฑ์— ๋Œ€ํ•ด ์„ค๋ช…ํ•˜๊ณ  ์žˆ๋Š” ์ž๋ฃŒ๋“ค์ด ์ €์žฅ๋˜์–ด ์žˆ๊ณ , ใ€Œ6๋‹จ๊ณ„-์œ„๋Œ€์„ฑใ€ ํ•˜์œ„ ํด๋”๋Š” ใ€Œ์†Œ์„คใ€, ใ€Œ์˜๋„์ „๋žตใ€, ใ€Œํ’๋ชจ์ผํ™”ใ€ 3๊ฐœ์˜ ์„ธ๋ถ€ ํด๋”๋กœ ๊ตฌ์„ฑ๋˜์–ด โ€˜๊น€์ •์ผ ์ง€๋„์žโ€™, โ€˜์‚ฌ๋ž‘์˜ ์ •์น˜๊ฐ€ ๊น€์ •์ผ ์žฅ๊ตฐโ€™ ๋“ฑ ๊น€์ •์ผยท์„ ๊ตฐ์ •์น˜๋ฅผ ์ฐฌ์–‘ํ•˜๋Š” ์ž๋ฃŒ๋“ค์ด ์ €์žฅ๋˜์–ด ์žˆ์œผ๋ฉฐ, ใ€Œ7๋‹จ๊ณ„-์ฒ ํ•™์ •์น˜๊ฒฝ์ œ์—ญ์‚ฌใ€ ํ•˜์œ„ ํด๋”๋Š” ใ€Œ์ •์น˜๊ฒฝ์ œํ•™ใ€, ใ€Œ์ฒ ํ•™ใ€, ใ€Œํ†ต์ผํ˜๋ช…์ด๋ก ๋ฐฉ๋ฒ•ใ€ ๋“ฑ์˜ ์„ธ๋ถ€ ํด๋”๋กœ ๊ตฌ์„ฑ๋˜์–ด โ€˜๊น€์ผ์„ฑ์ฃผ์˜ ๋„ํ•ดโ€™ ๋“ฑ ๋ถํ•œ์‹ ์ˆ˜๋ น๊ด€๊ณผ ๋ถํ•œ ์‚ฌํšŒ์ฃผ์˜์ฒด์ œ ์šฐ์›”์„ฑ ๋ฐ ๋ถํ•œ์˜ ํ†ต์ผ๋ก ์„ ์„ ์ „ํ•˜๋Š” ์ž๋ฃŒ๋“ค์ด ์ €์žฅ๋˜์–ด ์žˆ๊ณ , ใ€Œ8๋‹จ๊ณ„-์ฃผ์š”๋…ธ์ž‘๋ชจ์Œใ€ ํ•˜์œ„ ํด๋”์—๋Š” โ€˜์กฐ๊ตญํ†ต์ผ 10๋Œ€ ๋…ธ์ž‘โ€™ ๋“ฑ ๊น€์ผ์„ฑยท๊น€์ •์ผ์˜ ์—ฐ์„ค ๋‚ด์šฉ ๋“ฑ์„ ์ •๋ฆฌํ•œ ๋…ธ์ž‘ ๋ชจ์Œ์ด ์ €์žฅ๋˜์–ด ์žˆ์œผ๋ฉฐ, ใ€Œ9๋‹จ๊ณ„-์ผ๊ตฐ๊ต์–‘1ใ€ ํ•˜์œ„ ํด๋” ๋ฐ ใ€Œ10๋‹จ๊ณ„-์ผ๊ตฐ๊ต์–‘2ใ€ ํ•˜์œ„ ํด๋”์—๋Š” โ€˜ํ•ญ์ผ ์œ ๊ฒฉ๋Œ€์›๋“ค์˜ ์‚ถ๊ณผ ํˆฌ์Ÿ ํšŒ์ƒ๊ธฐ ์ƒยท์ค‘ยทํ•˜โ€™ ๋“ฑ ๊น€์ผ์„ฑ ํ–‰์  ์šฐ์ƒํ™” ๋ฐ ๋ถํ•œ ์กฐ์„ ๋…ธ๋™๋‹น์— ๋Œ€ํ•œ ๊น€์ผ์„ฑ์˜ ๊ฐ์ข… ๋‹ดํ™” ๋“ฑ ๋ถํ•œ ์ฒด์ œ๋ฅผ ๋ฏธํ™”ยท์ฐฌ์–‘ํ•˜๋Š” ์ž๋ฃŒ๋“ค์ด ์ €์žฅ๋˜์–ด ์žˆ๊ณ , ใ€Œ๋ฌธํ™”์˜ˆ์ˆ ใ€ ํ•˜์œ„ ํด๋”๋Š” ใ€Œ์†Œ์„คใ€, ใ€Œ์Œ์•…ใ€ 2๊ฐœ์˜ ์„ธ๋ถ€ ํด๋”๋กœ ๊ตฌ์„ฑ๋˜์–ด โ€˜์••๋ก๊ฐ•โ€™, โ€˜์˜์ƒ, โ€˜๊น€์ •์ผ์žฅ๊ตฐ์˜ ๋…ธ๋ž˜โ€™, โ€˜๋‹ป์€ ์˜ฌ๋ž๋‹คโ€™, โ€˜์ด๋Œ€โ€™, โ€˜๋ ฅ์‚ฌ์˜ ๋Œ€ํ•˜โ€™ ๋“ฑ ๊น€๋ถ€์ž๋ฅผ ์ฐฌ์–‘ยท๋ฏธํ™”ํ•˜๋Š” ๋ถํ•œ ์†Œ์„ค ๋ฐ ์Œ์•…์ž๋ฃŒ ๋“ฑ์ด ์ €์žฅ๋˜์–ด ์žˆ๊ณ , ใ€Œ์ €์ž‘์ง‘ใ€ ํ•˜์œ„ ํด๋”๋Š” ใ€Œ๊น€์ผ์„ฑ ์ €์ž‘์ง‘ ์ฃผ์ œ ๋ถ„๋ฅ˜ใ€, ใ€Œ๊น€์ •์ผ ์ €์ž‘์ง‘ ์ฃผ์ œ ๋ถ„๋ฅ˜ใ€ ๋“ฑ 2๊ฐœ์˜ ์„ธ๋ถ€ ํด๋”๋กœ ๊ตฌ์„ฑ๋˜์–ด ์ฃผ์ œ๋ณ„๋กœ โ€˜๊น€์ผ์„ฑ ์ €์ž‘์ง‘โ€™๊ณผ โ€˜๊น€์ •์ผ ์ €์ž‘์ง‘โ€™ ๋“ฑ์ด ์ฒด๊ณ„์ ์œผ๋กœ ์ €์žฅ๋˜์–ด ์žˆ๋‹ค. ์œ„ ์ƒ์œ„ ํด๋” ์ค‘ ใ€Œgreatnessใ€ ํด๋”๋Š” ใ€Œ์ˆ˜๋ น๋‹˜์˜ ์œ„๋Œ€์„ฑใ€, ใ€Œ์œ„๋Œ€์„ฑใ€, ใ€Œ์žฅ๊ตฐ๋‹˜์˜ ์œ„๋Œ€์„ฑใ€ 3๊ฐœ์˜ ํ•˜์œ„ ํด๋”๋กœ ๊ตฌ์„ฑ๋˜์–ด ๊น€๋ถ€์ž๋ฅผ ์šฐ์ƒํ™”ํ•˜๋ฉฐ ์ฐฌ์–‘ํ•˜๋Š” ๋ถํ•œ์›์ „ ์ž๋ฃŒ ๋“ฑ์ด ์ €์žฅ๋˜์–ด ์žˆ๋‹ค. ์œ„ ์ƒ์œ„ ํด๋” ์ค‘ ใ€ŒK1์ €์ž‘์ง‘hwpํŒใ€ ํด๋”์—๋Š” ๊น€์ผ์„ฑ๊ณผ ๋ถํ•œ์ฒด์ œ๋ฅผ ๋ฏธํ™”, ์ฐฌ์–‘ํ•˜๋Š” โ€˜๊น€์ผ์„ฑ์ €์ž‘์ง‘โ€™(1โˆผ44๊ถŒ)์ด ์ €์žฅ๋˜์–ด ์žˆ๋‹ค. ์œ„ ์ƒ์œ„ ํด๋” ์ค‘ ใ€ŒK2์ €์ž‘์ง‘hwpํŒใ€ ํด๋”์—๋Š” ๊น€์ •์ผ์˜ ์ฃผ์š” ๋‹ดํ™”๋‚ด์šฉ์„ ์ •๋ฆฌํ•œ โ€˜๊น€์ •์ผ์ €์ž‘์ง‘โ€™์ด ์ €์žฅ๋˜์–ด ์žˆ๋‹ค. ์œ„ ์ƒ์œ„ ํด๋” ์ค‘ ใ€Œ(๋ช…์นญ 38 ์ƒ๋žต)ใ€ ํด๋”์—๋Š” ๊น€์ผ์„ฑ์„ ์šฐ์ƒํ™”ํ•˜๋Š” ๋‚ด์šฉ์˜ โ€˜๊น€์ผ์„ฑ์ฃผ์˜โ€™ ํ•™์Šต์„ ์œ„ํ•œ โ€˜๊น€์ผ์„ฑ์ฃผ์˜์ด์„œโ€™(1~10๊ถŒ)๊ฐ€ ์ €์žฅ๋˜์–ด ์žˆ๋‹ค. ์œ„ ์ƒ์œ„ ํด๋” ์ค‘ ใ€Œrojak2012ใ€ ํด๋”์—๋Š” โ€˜์ •์„ธ๊ด€๋ จ ๊ธด๊ธ‰ ๊ต์–‘(Daesa Study) / ์ค‘์š”๋ฐœํ‘œ๋ฌธ ๋ชจ์Œโ€™ ๋“ฑ ๋ถํ•œ์ฒด์ œ๋ฅผ ์ฐฌ์–‘ํ•˜๊ณ  ๋ฐ˜๋ฏธยท๋ฐ˜์ •๋ถ€ ํˆฌ์Ÿ์„ ์„ ์ „ยท์„ ๋™ํ•˜๋Š” ์ž๋ฃŒ๋“ค์ด ์ €์žฅ๋˜์–ด ์žˆ๋‹ค. ์œ„ ์ƒ์œ„ ํด๋” ์ค‘ ใ€Œstudy2012ใ€ ํด๋”์—๋Š” โ€˜๋ฐฑ๋‘์‚ฐ3๋Œ€์žฅ๊ตฐ์˜ ์ˆญ๊ณ ํ•œ ๋‹น์กฐ์ง ๊ด€๋…์„ ๋”ฐ๋ผ ๋ฐฐ์šธ ๋ฐ ๋Œ€ํ•˜์—ฌโ€™์™€ ๊ฐ™์ด ๊น€๋ถ€์ž๋ฅผ ์šฐ์ƒํ™”ํ•˜๊ณ  ์ฐฌ์–‘ํ•˜๋Š” ์ž๋ฃŒ๊ฐ€ ์ €์žฅ๋˜์–ด ์žˆ๋‹ค. ์œ„ ์ƒ์œ„ ํด๋” ์ค‘ ใ€Œstudy2014ใ€ ํด๋”๋Š” ใ€Œ1 ์‹ ๋…„์‚ฌใ€, ใ€Œ2 ๋…ธ์ž‘(2012-2013)ใ€, ใ€Œ3 ์กฐ์ง๊ด€ใ€, ใ€Œ4 ํšŒ๊ณ ๋กใ€, ใ€Œ5 ๊น€์ฃผ์˜๊ฐ•์ขŒใ€ ๋“ฑ 5๊ฐœ์˜ ํ•˜์œ„ ํด๋”๋กœ ๊ตฌ์„ฑ๋˜์–ด ๋ถํ•œ์˜ 2014๋…„ ์‹ ๋…„์‚ฌ ๋“ฑ ๋ถํ•œ ์ฒด์ œ๋ฅผ ์„ ์ „ยท๋ฏธํ™”ํ•˜๋Š” ์ž๋ฃŒ๋“ค์ด ์ €์žฅ๋˜์–ด ์žˆ๋‹ค. ์œ„ ์ƒ์œ„ ํด๋” ์ค‘ ใ€Œstudy2015ใ€ ํด๋”๋Š” ใ€Œ2015์‹ ๋…๊ต์–‘ใ€, ใ€Œ2015์• ๊ตญ์ฃผ์˜๊ต์–‘ใ€ ๋“ฑ 2๊ฐœ์˜ ํ•˜์œ„ ํด๋”๋กœ ๊ตฌ์„ฑ๋˜์–ด โ€˜์กฐ์„ ๋กœ๋™๋‹น ์ค‘์•™์œ„์›ํšŒ, ์กฐ์„ ๋กœ๋™๋‹น ์ค‘์•™๊ตฐ์‚ฌ์œ„์›ํšŒ ๊ณต๋™๊ตฌํ˜ธโ€™ ๋ฐ ๋ถํ•œ ๊น€์ •์€ ์‹œ๋Œ€์˜ ํ˜๋ช…๊ฐ€์š”์ธ โ€˜์ฃฝ์–ด๋„ ํ˜๋ช…์‹ ๋… ๋ฒ„๋ฆฌ์ง€ ๋ง์žโ€™, โ€˜๊ฐ€๋ฆฌ๋ผ ๋ฐฑ๋‘์‚ฐ์œผ๋กœโ€™ ๋“ฑ ๋ถํ•œ ์ฒด์ œ๋ฅผ ์ฐฌ์–‘ํ•˜๋Š” ์ž๋ฃŒ๋“ค์ด ์ €์žฅ๋˜์–ด ์žˆ๋‹ค. ์œ„ ์ƒ์œ„ ํด๋” ์ค‘ ใ€Œ(๋ช…์นญ 39 ์ƒ๋žต)ใ€ ํด๋”๋Š” ใ€Œ์„ ๊ตฐ๊ณผ ์‚ฌํšŒ์ฃผ์˜ ์œ„์—…ใ€, ใ€Œ์„ ๊ตฐ์‚ฌ์ƒ๊ต์–‘์ž๋ฃŒใ€, ใ€Œ์œ„๋Œ€ํ•œ ์žฅ๊ตฐ๋‹˜์— ์˜ํ•œ ์„ ๊ตฐ์‚ฌ์ƒ์˜ ์‹ฌํ™”ใ€, ใ€Œ์ฃผ์ฒด์„ฑ, ๋ฏผ์กฑ์„ฑ์˜ ๊ตฌํ˜„๊ณผ ์„ ๊ตฐ์ •์น˜ใ€ ๋“ฑ 4๊ฐœ์˜ ํ•˜์œ„ ํด๋”๋กœ ๊ตฌ์„ฑ๋˜์–ด ๊น€์ •์ผ๊ณผ ์„ ๊ตฐ์ •์น˜๋ฅผ ๋ฏธํ™”ยท์ฐฌ์–‘ํ•˜๋Š” ์ž๋ฃŒ๋“ค์ด ์ €์žฅ๋˜์–ด ์žˆ๋‹ค. ์œ„ ์ƒ์œ„ ํด๋” ์ค‘ ใ€ŒYOUTUBEใ€ ํด๋”๋Š” ใ€Œ7.27ใ€, ใ€Œํ™”๋ฉด์Œ์•…ใ€ ๋“ฑ 2๊ฐœ์˜ ํ•˜์œ„ ํด๋”๋กœ ๊ตฌ์„ฑ๋˜์–ด ๋ถํ•œ ๋ฐ ๊น€๋ถ€์ž๋ฅผ ์ฐฌ์–‘ํ•˜๋Š” ๊ฐ์ข… ๋ถํ•œ ๋™์˜์ƒ ๋“ฑ์ด ์ €์žฅ๋˜์–ด ์žˆ๋‹ค. ์œ„ ์ƒ์œ„ ํด๋” ์ค‘ ใ€Œ๊ต์–‘ใ€ ํด๋”๋Š” ใ€ŒP๊ต์–‘ใ€, ใ€Œ๊ธฐ๋ณธ์Œ์•…ใ€, ใ€Œ๊น€์ผ์„ฑ์ฃผ์˜์ด์„œใ€, ใ€Œ๋Œ€์žฅใ€, ใ€Œ๋Œ€์ค‘์†Œ๋ชจ์ž„๊ต์–‘ใ€, ใ€Œ๋Œ€ํ•™1ํ•™๋…„ใ€, ใ€Œ๋„์„œใ€, ใ€Œ์œ„๋Œ€์„ฑํ•™์Šตใ€, ใ€Œ์ •์ˆ˜ใ€, ใ€Œ์กฐ์ง์‚ฌ์—…ใ€ ๋“ฑ๊ณผ ๊ฐ™์€ ํ•˜์œ„ ํด๋”๋กœ ๊ตฌ์„ฑ๋˜์–ด ์žˆ๋Š”๋ฐ, ใ€ŒP๊ต์–‘ใ€ ํด๋”๋Š” ใ€Œ๋…ธ๋ž˜ใ€, ใ€Œ์• ๊ตญ์ ์ „์œ„๋Œ€ใ€, ใ€Œ์ธํ„ฐ๋„ท๋‹น๊ฑด์„ค๋ก ํŠน๊ฐ•ใ€ ๋“ฑ 3๊ฐœ์˜ ์„ธ๋ถ€ ํด๋”๋กœ ๊ตฌ์„ฑ๋˜์–ด โ€˜๋‹น์˜ ์œ ์ผ์‚ฌ์ƒ์ฒด๊ณ„ ํ™•๋ฆฝ 10๋Œ€์›์น™โ€™ ๋“ฑ ๋ถํ•œ ์‚ฌํšŒ์ฃผ์˜ ์ฒด์ œ์™€ ์ฃผ์ฒด์‚ฌ์ƒ ๋“ฑ์„ ์ฐฌ์–‘ํ•˜๋Š” ์ž๋ฃŒ๋“ค์ด ์ €์žฅ๋˜์–ด ์žˆ๊ณ , ใ€Œ๊ธฐ๋ณธ์Œ์•…ใ€ ํด๋”์—๋Š” โ€˜๊น€์ผ์„ฑ์žฅ๊ตฐ์˜ ๋…ธ๋ž˜โ€™ ๋“ฑ ๋ถํ•œ ๋ฐ ๊น€๋ถ€์ž๋ฅผ ์ฐฌ์–‘ํ•˜๋Š” ๋ถํ•œ ๋…ธ๋ž˜ ๋ฐ ๋…ธ๋ž˜ ๊ฐ€์‚ฌ ์ž๋ฃŒ๋“ค์ด ์ €์žฅ๋˜์–ด ์žˆ์œผ๋ฉฐ, ใ€Œ๊น€์ผ์„ฑ์ฃผ์˜ ์ด์„œใ€ ํด๋”์—๋Š” ๊น€์ผ์„ฑ์ด ์ฐฝ์‹œํ•œ ๊น€์ผ์„ฑ์ฃผ์˜๋Š” ๊ณต์‚ฐ์ฃผ์˜๋ฏธ๋ž˜๋ฅผ ๋Œ€ํ‘œํ•˜๋Š” ์œ„๋Œ€ํ•œ ํ˜๋ช…์‚ฌ์ƒ์ด๋ผ๋Š” ์ทจ์ง€๋กœ ์ฐฌ์–‘ยท๋ฏธํ™”ํ•˜๋Š” โ€˜๊น€์ผ์„ฑ์ฃผ์˜ ์ด์„œโ€™(1~10๊ถŒ)๊ฐ€ ์ €์žฅ๋˜์–ด ์žˆ๊ณ , ใ€Œ๋Œ€์žฅใ€ ํด๋”์—๋Š” ๊น€์ •์€์ด ํƒœ์–ด๋‚œ ๋‚ ์„ ๊ธฐ๋…ํ•˜๋ฉฐ ๊น€์ •์€์˜ ์œ„๋Œ€์„ฑ์„ ๊ฐ•์กฐํ•˜๋Š” ์Œ์„ฑํŒŒ์ผ ๋“ฑ์ด ์ €์žฅ๋˜์–ด ์žˆ์œผ๋ฉฐ, ใ€Œ๋Œ€์ค‘์†Œ๋ชจ์ž„๊ต์–‘ใ€ ํด๋”๋Š” ใ€Œ๊ธฐ์ดˆ์˜์ƒใ€, ใ€Œ๋Œ€์ค‘์‚ฌ์—…๋ฐฉ๋ฒ•ใ€, ใ€Œ๋ฌธํ™”์˜ˆ์ˆ ใ€, ใ€Œ๋ฐ˜๋ฏธ์ž์ฃผใ€, ใ€Œ๋ณ€ํ˜์ด๋ก -3๋Œ€๊ฐœ๋ก ใ€, ใ€Œ์„ ๊ตฐ์‚ฌ์ƒใ€, ใ€Œ์กฐ๊ตญํ†ต์ผใ€, ใ€Œ์ฃผ์ฒด์‚ฌํšŒ์ฃผ์˜ใ€, ใ€Œํ†ต์ผ์ „์„ ใ€, ใ€ŒํšŒ๊ณ ๋กใ€ ๋“ฑ์˜ ์„ธ๋ถ€ ํด๋”๋กœ ๊ตฌ์„ฑ๋˜์–ด ๋ถํ•œ์˜ ์ฃผ์ฒด์‚ฌ์ƒ, ์„ ๊ตฐ์‚ฌ์ƒ, ๋ฐ˜๋ฏธ์ž์ฃผ, ํ†ต์ผ๋ก  ๋“ฑ ๋ถํ•œ์˜ ์ฃผ์˜ยท์ฃผ์žฅ์„ ์„ ์ „ํ•˜๋Š” ์ž๋ฃŒ๋“ค์ด ์ €์žฅ๋˜์–ด ์žˆ์œผ๋ฉฐ, ใ€Œ๋Œ€ํ•™1ํ•™๋…„ใ€ ํด๋”๋Š” ใ€Œ๊ต์–‘์ปค๋ฆฌ04ใ€, ใ€Œ๋Œ€์ค‘์†Œ๋ชจ์ž„๊ต์–‘ใ€, ใ€Œ์ƒˆ์‹œ๋Œ€ใ€, ใ€Œํ†ต์ผ์—ฌ๋ช…๋ณด๋„๋ชจ์Œใ€ ๋“ฑ์˜ ์„ธ๋ถ€ ํด๋”๋กœ ๊ตฌ์„ฑ๋˜์–ด โ€˜์ฃผ์ฒด์˜ ํ•™์Šต๋ก โ€™, โ€˜์ฃผ์ฒด์˜ ํ˜๋ช…์  ์กฐ์ง๊ด€โ€™ ๋“ฑ ์ฃผ์ฒด์‚ฌ์ƒ์„ ๋น„๋กฏํ•œ ๋ถํ•œ์˜ ๊ฐ์ข… ์ฃผ์˜ยท์ฃผ์žฅ์„ ์„ ์ „ํ•˜๋Š” ์ž๋ฃŒ๋“ค์ด ์ €์žฅ๋˜์–ด ์žˆ๊ณ , ใ€Œ๋„์„œใ€ ํด๋”๋Š” ใ€Œ์†Œ์„คใ€ ๋ฐ ใ€Œ์œ„๋Œ€์„ฑใ€ ๋“ฑ 2๊ฐœ์˜ ์„ธ๋ถ€ ํด๋”๋กœ ๊ตฌ์„ฑ๋˜์–ด ๋ถํ•œ ์†Œ์„ค โ€˜์ด์„œใ€Š๋ถˆ๋ฉธ์˜ ๋ ฅ์‚ฌใ€‹์žฅํŽธ์†Œ์„ค, ํ‰์–‘์€ ์„ ์–ธํ•œ๋‹คโ€™ ๋ฐ โ€˜21์„ธ๊ธฐํƒœ์–‘ ๊น€์ •์ผ์žฅ๊ตฐโ€™ ๋“ฑ ๊น€๋ถ€์ž๋ฅผ ์ฐฌ์–‘ยท๋ฏธํ™”ํ•˜๋Š” ์ž๋ฃŒ๋“ค์ด ์ €์žฅ๋˜์–ด ์žˆ์œผ๋ฉฐ, ใ€Œ์œ„๋Œ€์„ฑํ•™์Šตใ€ ํด๋”๋Š” ใ€Œ์œ„๋Œ€์„ฑ์ž๋ฃŒ ๋ฐ ์„ ๊ตฐ์‚ฌ์ƒ์ž๋ฃŒใ€, ใ€Œ์žฅ๊ตฐ๋‹˜์˜ ์œ„๋Œ€์„ฑใ€, ใ€Œํ˜๋ช…์ผํ™”ใ€ ๋“ฑ 3๊ฐœ์˜ ์„ธ๋ถ€ ํด๋”๋กœ ๊ตฌ์„ฑ๋˜์–ด โ€˜์„ ๊ตฐ์ •์น˜ : ์ฃผ์ฒด์‚ฌํšŒ์ฃผ์˜์˜ ์ƒ๋ช…์„ โ€™ ๋“ฑ ๊น€๋ถ€์ž๋ฅผ ์šฐ์ƒํ™”ํ•˜๊ณ  ์ฐฌ์–‘ํ•˜๋Š” ์ž๋ฃŒ๋“ค์ด ์ €์žฅ๋˜์–ด ์žˆ๊ณ , ใ€Œ์ •์ˆ˜ใ€ํด๋”๋Š” ใ€Œ1๋‹จ๊ณ„ใ€, ใ€Œ2๋‹จ๊ณ„ใ€ ๋“ฑ 2๊ฐœ์˜ ์„ธ๋ถ€ ํด๋”๋กœ ๊ตฌ์„ฑ๋˜์–ด โ€˜ํ•ญ์ผ์œ ๊ฒฉ๋Œ€์›๋“ค์˜ ์‚ถ๊ณผ ํˆฌ์Ÿ ํšŒ์ƒ๊ธฐโ€™(์ƒยท์ค‘ยทํ•˜) ๋ฐ โ€˜์ ˆ์„ธ์˜ ์• ๊ตญ์ž ๊น€์ •์ผ ์žฅ๊ตฐโ€™, โ€˜์ฃผ์ฒด์‚ฌ์ƒ ๋…ธ์ž‘ 60์„ โ€™ ๋“ฑ ๊น€๋ถ€์ž๋ฅผ ์ฐฌ์–‘ยท๋ฏธํ™”ํ•˜๋Š” ์ž๋ฃŒ๋“ค์ด ์ €์žฅ๋˜์–ด ์žˆ์œผ๋ฉฐ, ใ€Œ์กฐ์ง์‚ฌ์—…ใ€ ํด๋”๋Š” ใ€Œ์กฐ์ง์‚ฌ์—…๋ฐฉ๋ฒ•ใ€์ด๋ผ๋Š” 1๊ฐœ์˜ ์„ธ๋ถ€ ํด๋”๋กœ ๊ตฌ์„ฑ๋˜์–ด โ€˜์ข…ํŒŒ์ฃผ์˜๋ฅผ ์ฒญ์‚ฐํ•˜๊ณ  ํ˜๋ช…๋Œ€์˜ค์˜ ํ†ต์ผ๋‹จ๊ฒฐ์„ ๊ฐ•ํ™”ํ•˜์žโ€™ ๋“ฑ โ€˜๊น€์ผ์„ฑ ์ €์ž‘์ง‘โ€™ ๋“ฑ์— ์ˆ˜๋ก๋˜์–ด ์žˆ๋Š” ์ž๋ฃŒ๋“ค์ด ์ €์žฅ๋˜์–ด ์žˆ๋‹ค. ์œ„ ์ƒ์œ„ ํด๋” ์ค‘ ใ€Œ์ €์ž‘์ง‘ใ€ ํด๋”๋Š” ใ€ŒSTUDYใ€, ใ€Œ๊น€์ผ์„ฑ์ €์ž‘์ง‘ใ€ ,ใ€Œ๊น€์ผ์„ฑ์ €์ž‘์ง‘์ฃผ์ œ๋ถ„๋ฅ˜ใ€, ใ€Œ๊น€์ •์ผ์ €์ž‘์ง‘ใ€, ใ€Œ๊น€์ •์ผ์ €์ž‘์ง‘์ฃผ์ œ๋ถ„๋ฅ˜ใ€, ใ€Œ์ €์ž‘์ง‘chmใ€, ใ€Œ์ €์ž‘์ง‘๋ชจ์Œใ€ ๋“ฑ 7๊ฐœ์˜ ํ•˜์œ„ ํด๋”๋กœ ๊ตฌ์„ฑ๋˜์–ด โ€˜๊น€์ผ์„ฑ ์ €์ž‘์ง‘โ€™ยทโ€˜๊น€์ •์ผ ์ €์ž‘์ง‘โ€™ ๋“ฑ์ด ์ฃผ์ œ๋ณ„, hwpํŒ, PDF ํŒŒ์ผ ๋“ฑ ๋‹ค์–‘ํ•œ ํ˜•ํƒœ๋กœ ์ €์žฅ๋˜์–ด ์žˆ๋‹ค. ์œ„ ์ƒ์œ„ ํด๋” ์ค‘ ใ€Œํ”Œ๋ž˜์‹œ์˜์ƒใ€ ํด๋”๋Š” ใ€Œ๊ณตํ™”๊ตญ์ฐฝ๊ฑด55๋Œใ€, ใ€Œ์ˆ˜๋ น๋‹˜์€ ์˜์›ํžˆ ์šฐ๋ฆฌ์™€ ํ•จ๊ป˜ ๊ณ„์‹œ๋‹คใ€ ๋“ฑ 2๊ฐœ์˜ ์„ธ๋ถ€ ํด๋”๋กœ ๊ตฌ์„ฑ๋˜์–ด โ€˜์กฐ์„ ๋กœ๋™๋‹น๋งŒ์„ธโ€™์™€ ๊ฐ™์ด ๋ถํ•œ ๋ฐ ๊น€๋ถ€์ž๋ฅผ ์ฐฌ์–‘ํ•˜๋Š” ๋…ธ๋ž˜ ๋ฐ ์˜์ƒ ํ”Œ๋ž˜์‹œํŒŒ์ผ ๋“ฑ์ด ์ €์žฅ๋˜์–ด ์žˆ๋‹ค. ์œ„ ์ €์žฅ๋งค์ฒด์— ์ˆ˜๋ก๋œ ํŒŒ์ผ๋“ค์˜ ์ฃผ์š” ๋‚ด์šฉ์€ ๋‹ค์Œ๊ณผ ๊ฐ™๋‹ค. ใ€Š1 readings ํด๋” ๋‚ด โ€˜์ƒˆ์‹œ๋Œ€์ •์น˜ํ•™์›๋ก โ€™์˜ ์ฃผ์š” ๋‚ด์šฉใ€‹ - ๋ฏผ์ค‘์€ ๊ณ„๊ธ‰์  ์˜ˆ์†์„ ์ดˆ๋ž˜ํ•˜๋Š” ๋‚ก์€ ์‚ฌํšŒ์ œ๋„๋ฅผ ํƒ€๋„ํ•˜๊ธฐ ์œ„ํ•˜์—ฌ ์ง€๋ฐฐ๊ณ„๊ธ‰์— ๋ฐ˜๋Œ€ํ•˜๋Š” ํˆฌ์Ÿ์„ ์ „๊ฐœํ•˜๊ณ , ๋ฏผ์กฑ์  ์˜ˆ์†์„ ์ดˆ๋ž˜ํ•˜๋Š” ๋‚ก์€ ์‹๋ฏผ์ง€ ์ œ๋„๋ฅผ ํƒ€๋„ํ•˜๊ธฐ ์œ„ํ•˜์—ฌ ์ง€๋ฐฐ๋ฏผ์กฑ์— ๋ฐ˜๋Œ€ํ•˜๋Š” ํˆฌ์Ÿ๋„ ์ง„์ „์‹œ์ผœ์•ผ ํ•œ๋‹ค. - ๋ฐ˜์ œ๋ฐ˜๋ด‰๊ฑด๋ฏผ์ฃผ์ฃผ์˜ ํ˜๋ช…์€ ๋ฏผ์ค‘์„ ์‹๋ฏผ์ง€์  ๋ฐ ๋ด‰๊ฑด์ ์ธ ์–ต์••๊ณผ ์ฐฉ์ทจ๋กœ๋ถ€ํ„ฐ ํ•ด๋ฐฉํ•˜๋Š” ํ˜๋ช…์ด๋ฉฐ, ์‚ฌํšŒ์ฃผ์˜ ํ˜๋ช…์€ ์ž๋ณธ์ฃผ์˜์  ์ฐฉ์ทจ๋ฅผ ๋น„๋กฏํ•œ ๋ชจ๋“  ์ฐฉ์ทจ์™€ ์–ต์••์œผ๋กœ๋ถ€ํ„ฐ ๋ฏผ์ค‘์„ ์ข…๊ตญ์ ์œผ๋กœ ํ•ด๋ฐฉํ•˜๋Š” ๊ฐ€์žฅ ์‹ฌ๊ฐํ•œ ์‚ฌํšŒํ˜๋ช…์ด๋‹ค. ์ด ๋‘๊ฐ€์ง€ ๋‹จ๊ณ„์˜ ํ˜๋ช…์„ ํ†ตํ•˜์—ฌ ๋ฏผ์กฑํ•ด๋ฐฉ์˜ ๊ณผ์ œ์™€ ๊ณ„๊ธ‰ํ•ด๋ฐฉ์˜ ๊ณผ์ œ๊ฐ€ ํ•ด๊ฒฐ๋œ๋‹ค. - ๋ฏผ์ค‘์€ ์˜ค์ง ๋…ธ๋™์ž๊ณ„๊ธ‰ ๋‹น์˜ ์ง€๋„ํ•˜์—์„œ๋งŒ ์ž์ฃผ์ ์ธ ์ •์น˜ํ™œ๋™์„ ์ˆ˜ํ–‰ํ•  ์ˆ˜ ์žˆ๋‹ค. ์‚ฌํšŒ์ฃผ์˜ ์ •์น˜์ œ๋„๋Š” ๋ฏผ์ค‘์ด ์ •๊ถŒ์˜ ์ฃผ์ธ์œผ๋กœ ๋˜๊ณ  ์žˆ์œผ๋ฉฐ ์ •๊ถŒ์ด ๊ทธ๋“ค์„ ์œ„ํ•˜์—ฌ ๋ด‰์‚ฌํ•˜๋Š” ๊ฐ€์žฅ ์„ ์ง„์ ์ธ ์ •์น˜์ œ๋„์ด๋‹ค. - ์ด์™€ ๊ฐ™์€ ๋ฌธ๊ฑด์€ ๋ถํ•œ์‹ ์‚ฌํšŒ์ฃผ์˜์— ์ž…๊ฐํ•˜์—ฌ ๋ถํ•œ์˜ ์ •์น˜์‚ฌํšŒ์ œ๋„ ๋“ฑ์„ ๋ฏธํ™”ยท์ฐฌ์–‘ํ•˜๋Š” ๋“ฑ ๋ถํ•œ ์‚ฌํšŒ์ฃผ์˜์— ๋Œ€ํ•œ ์šฐ์›”์„ฑ์„ ์„ ์ „ํ•˜๋Š” ๋‚ด์šฉ์ด๋‹ค. ใ€Š1 readings ํด๋” ์ค‘ ์œ„๋Œ€์„ฑ ํด๋” ๋‚ด โ€˜์œ„๋Œ€ํ•œ ์ˆ˜๋ น ๊น€์ผ์„ฑ๋™์ง€ ํ˜๋ช…ํ™œ๋™๋žต๋ ฅโ€™์˜ ์ฃผ์š” ๋‚ด์šฉใ€‹ - ๊น€์ผ์„ฑ ์ถœ์ƒ(1912. 4.)๋ถ€ํ„ฐ ์‚ฌ๋ง(1994. 7.)๊นŒ์ง€์˜ ํ™œ๋™ ์ด๋ ฅ์„ ๊ธฐ์ˆ ํ•œ ๋ฌธ๊ฑด์œผ๋กœ ๊น€์ผ์„ฑ์˜ ์ผ์ƒ์— ์žˆ์–ด ๋ชจ๋“  ํ–‰์ ์„ ์šฐ์ƒํ™”ํ•˜๊ณ , ๊น€์ผ์„ฑ ์ƒ์ „์˜ ํ™œ๋™๋“ค๊ณผ ํ•จ๊ป˜ ๊น€์ผ์„ฑ ์ฃผ์ฒด์‚ฌ์ƒ ๋“ฑ์„ ๋งน๋ชฉ์ ์œผ๋กœ ์ฐฌ์–‘ยท๋ฏธํ™”ํ•˜๋Š” ๊ฒƒ์„ ์ฃผ์š” ๋‚ด์šฉ์œผ๋กœ ํ•˜๊ณ  ์žˆ๋‹ค. ใ€Š5๋‹จ๊ณ„ํ•™์Šต๊ณผ์ • ํด๋” ์ค‘ 2๋‹จ๊ณ„-๋ณ€ํ˜์ด๋ก ์กฐ์ง์‚ฌ์—… ํด๋” ๋‚ด โ€˜์ฃผ์ฒด์˜ ํ•œ๊ตญ์‚ฌํšŒ ๋ณ€ํ˜์šด๋™๋ก โ€™์˜ ์ฃผ์š” ๋‚ด์šฉใ€‹ - ํ•œ๊ตญ๋ณ€ํ˜ ์šด๋™์˜ ์ง€๋„์ด๋…, ํ•œ๊ตญ์‚ฌํšŒ์˜ ์„ฑ๊ฒฉ, ๋ณ€ํ˜ ์šด๋™์ด๋ก  ๋ฐ ์ „๋žต์ „์ˆ (3~4์žฅ), ๋Œ€์ค‘์˜์‹ํ™”๋ก  ๋ฐ ๋ฏผ์ค‘์กฐ์งํ™”๋ก (5~6์žฅ) ๋“ฑ 6์žฅ์œผ๋กœ ๊ตฌ์„ฑ๋œ ๋ถํ•œ์˜ ๋Œ€ํ‘œ์  ๋Œ€๋‚จํ˜๋ช… ์ด๋ก ์„œ๋กœ, - ๋ถํ•œ์ด ์ฃผ์žฅํ•˜๋Š” โ€˜๋ฏผ์กฑํ•ด๋ฐฉ ๋ฏผ์ฃผ์ฃผ์˜ ํ˜๋ช…๋ก โ€™์— ์ž…๊ฐํ•˜์—ฌ ๋‚จํ•œ์‚ฌํšŒ๋ฅผ โ€˜๋ฏธ๊ตญ์— ์˜ˆ์†๋œ ์‹๋ฏผ์ง€ ๋ฐ˜(๋ฐ˜)์ž๋ณธ์ฃผ์˜์‚ฌํšŒโ€™๋กœ, ๋‚จํ•œ ํ˜๋ช…์šด๋™์˜ ์„ฑ๊ฒฉ์„ โ€˜๋ฏผ์กฑํ•ด๋ฐฉ ๋ฏผ์ฃผ์ฃผ์˜ํ˜๋ช…โ€™์œผ๋กœ ๊ทœ์ •ํ•˜๊ณ , ์ฃผ์ฒด์‚ฌ์ƒ์„ ์ง€๋„์ด๋…์œผ๋กœ ํ•œ ๋‚จํ•œํ˜๋ช…์˜ ๊ตฌ์ฒด์ ์ธ ์ „์ˆ ๋กœ ํƒ€๋„๋Œ€์ƒ ์„ ์ •ยท์—ญ๋Ÿ‰ํŽธ์„ฑยท์ง€๋„ํ•ต์‹ฌ ๋ฐœ๊ตด์œก์„ฑยทํ†ต์ผ์ „์„ ์ฒด ๊ฑด์„ค ๋“ฑ์„ ์ œ์‹œํ•˜๋ฉด์„œ ๋‚จ๋ถ๊ด€๊ณ„ ๋ฐ ์ •์„ธ๋ณ€ํ™” ์—ฌํ•˜์— ๊ด€๊ณ„์—†์ด ํ˜๋ช…์›์น™ ๊ณ ์ˆ˜๋ฅผ ๊ฐ•์กฐํ•˜๊ณ , ๋ฏผ์ค‘์„ ์ฃผ์ฒด์‚ฌ์ƒ์„ ํ†ตํ•ด ์กฐ์งํ™”ํ•ด์•ผ ํ•œ๋‹ค๋Š” ๋“ฑ์„ ์ฃผ์š” ๋‚ด์šฉ์œผ๋กœ ํ•˜๊ณ  ์žˆ๋‹ค. ใ€Š5๋‹จ๊ณ„ํ•™์Šต๊ณผ์ • ํด๋” ์ค‘ 4๋‹จ๊ณ„-์„ ๊ตฐ์‚ฌ์ƒ ํด๋” ๋‚ด โ€˜์„ ๊ตฐ์ •์น˜์— ๋Œ€ํ•œ ๋ฆฌํ•ดโ€™์˜ ์ฃผ์š” ๋‚ด์šฉใ€‹ - ๊น€์ •์ผ์˜ ์„ ๊ตฐ์ •์น˜์•ผ๋ง๋กœ ์ž์ฃผ์™€ ํ‰ํ™”, ๋ฏผ์กฑ์˜ ํ†ต์ผ๊ณผ ๋ฒˆ์˜์„ ๋‹ด๋ณดํ•ด์ฃผ๋Š” ์• ๊ตญ, ์• ์กฑ, ์• ๋ฏผ์˜ ์ •์น˜์ด๋ฉฐ ์ธ๋ฅ˜์ž์ฃผ์œ„์—…, ์‚ฌํšŒ์ฃผ์˜์œ„์—…์„ ํž˜์žˆ๊ฒŒ ์ถ”๋™ํ•˜๋Š” ๋งŒ๋Šฅ์˜ ๋ณด๊ฒ€์ด๋ผ๋ฉฐ ์ฐฌ์–‘ยท๋ฏธํ™”ํ•˜๋Š” ๋‚ด์šฉ์œผ๋กœ โ€˜์„ ๊ตฐ์ •์น˜๋ฐฉ์‹์˜ ์ถœํ˜„โ€™, โ€˜์„ ๊ตฐ์ •์น˜์˜ ์œ„๋Œ€ํ•œ ์ƒํ™œ๋ ฅโ€™ ๋“ฑ 8๊ฐœ ํ•ญ๋ชฉ์œผ๋กœ ๊ตฌ์„ฑํ•˜์—ฌ ์„ ๊ตฐ์ •์น˜์˜ ์ •๋‹น์„ฑ์„ ์„ ์ „ํ•˜๊ณ  ์žˆ๋Š” ๋ฌธ๊ฑด์ด๋‹ค. ใ€Š5๋‹จ๊ณ„ํ•™์Šต๊ณผ์ • ํด๋” ์ค‘ 5๋‹จ๊ณ„-์œ„๋Œ€์„ฑ ํด๋” ๋‚ด โ€˜์ ˆ์„ธ์˜ ์• ๊ตญ์ž ๊น€์ •์ผ์žฅ๊ตฐโ€™์˜ ์ฃผ์š” ๋‚ด์šฉใ€‹ - ๊น€์ •์ผ์ด ์ฒœ์žฌ์ ์ธ ์˜ˆ์ง€์™€ ๋น„๋ฒ”ํ•œ ์‹ค๋ ฅ, ๋ฌดํ•œ๋Œ€ํ•œ ์• ๊ตญ์˜ ์—ด์ •์œผ๋กœ ์ด๋ฃฉํ•œ ์• ๊ตญ์—…์ ์€ ์—ญ์‚ฌ์— ์ „๋ฌดํ›„๋ฌดํ•œ ๊ฒƒ์ด๋ผ๋Š” ๋“ฑ ๊น€์ •์ผ์„ ์ ˆ์„ธ์˜ ๋ฏผ์กฑ ๊ตฌ์›์ž์ด์ž ์• ๊ตญ์ž๋กœ ์šฐ์ƒํ™”ํ•˜๋ฉฐ ์ฐฌ์–‘ยท๋ฏธํ™”ํ•˜๋Š” ๋‚ด์šฉ์˜ ๋ฌธ๊ฑด์ด๋‹ค. ใ€Š10๋‹จ๊ณ„ํ•™์Šต ํด๋” ์ค‘ 10๋‹จ๊ณ„-์ผ๊ตฐ๊ต์–‘2 ํด๋” ๋‚ด โ€˜ํ•ญ์ผ ์œ ๊ฒฉ๋Œ€์›๋“ค์˜ ์‚ถ๊ณผ ํˆฌ์Ÿ ํšŒ์ƒ๊ธฐ ์ƒยท์ค‘ยทํ•˜โ€™์˜ ์ฃผ์š” ๋‚ด์šฉใ€‹ - โ€˜ํ•ญ์ผ๋นจ์น˜์‚ฐ๋“ค์ด ๊น€์ผ์„ฑ์˜ ๊ฐ€๋ฅด์นจ์œผ๋กœ ๋‹น์˜ ์œ ์ผ ์‚ฌ์ƒ์ฒด๊ณ„๋ฅผ ์ถฉ์‹คํžˆ ๊ฐ–์ถ”๊ณ  ํˆฌ์Ÿํ–ˆ์œผ๋ฉฐ ๊น€์ผ์„ฑ์˜ ํ˜๋ช…์‚ฌ์ƒ๊ณผ ๋กœ์„ ์„ ์ฒ ์ €ํžˆ ๊ด€์ฒ ํ•˜์˜€๋‹คโ€™๋Š” โ€˜๊ณต์†Œ์™ธ 33 ์—ฐ๋Œ€์žฅโ€™์˜ ์ด์•ผ๊ธฐ ๋“ฑ ๊น€์ผ์„ฑ์˜ ํ–‰์ ์„ ์ฐฌ์–‘ยท์„ ์ „ํ•˜๋Š” ๋‚ด์šฉ์œผ๋กœ - โ€˜ํšŒ์ƒ๊ธฐ ์ƒโ€™์—๋Š” โ€˜์ง€๋„์ž์— ๋Œ€ํ•œ ์ถฉ์‹ค์„ฑโ€™, โ€˜๋ฏผ์ค‘๊ณผ์˜ ํ˜ˆ์—ฐ์  ๋ จ๊ณ„โ€™ ์ œํ•˜ ๋ชฉ์ฐจ๋กœ ๊ณต์†Œ์™ธ 34 ๋“ฑ 32๋ช…์˜ ํ•ญ์ผ ๋นจ์น˜์‚ฐ ๋Œ€์›๋“ค์˜ ํšŒ๊ณ ๋‹ด ํ˜•์‹์œผ๋กœ ์ˆ˜๋ก๋˜์–ด ์žˆ๊ณ  - โ€˜ํšŒ์ƒ๊ธฐ ์ค‘โ€™์—๋Š” โ€˜ํ˜๋ช…์  ์‹ ๋…โ€™, โ€˜์กฐ๊ตญ์• โ€™ ์ œํ•˜ ๋ชฉ์ฐจ๋กœ ๊ณต์†Œ์™ธ 35 ๋“ฑ 29๋ช…์˜ ํ•ญ์ผ๋นจ์น˜์‚ฐ ๋Œ€์›๋“ค์˜ ํšŒ๊ณ ๋‹ด ํ˜•์‹์œผ๋กœ ์ˆ˜๋ก๋˜์–ด ์žˆ์œผ๋ฉฐ - โ€˜ํšŒ์ƒ๊ธฐ ํ•˜โ€™์—๋Š” โ€˜ํ˜๋ช…์  ๋™์ง€์• โ€™, โ€˜ํ˜๋ช…์ž„๋ฌด์— ๋Œ€ํ•œ ์ฑ…์ž„์„ฑโ€™ ์ œํ•˜ ๋ชฉ์ฐจ๋กœ ๊ณต์†Œ์™ธ 36 ๋“ฑ 40๋ช…์˜ ํ•ญ์ผ๋นจ์น˜์‚ฐ ๋Œ€์›๋“ค์˜ ํšŒ๊ณ ๋‹ด ํ˜•์‹์œผ๋กœ ์ˆ˜๋ก๋˜์–ด ์žˆ๋‹ค. ใ€Š10๋‹จ๊ณ„ํ•™์Šต ํด๋” ์ค‘ 7๋‹จ๊ณ„-์ฒ ํ•™์ •์น˜๊ฒฝ์ œ์—ญ์‚ฌ ํด๋” ๋‚ด โ€˜๊น€์ผ์„ฑ์ฃผ์˜ ๋„ํ•ดโ€™์˜ ์ฃผ์š” ๋‚ด์šฉใ€‹ - ์œ„๋Œ€ํ•œ ๊น€์ผ์„ฑ์ฃผ์˜๋Š” ๋ฏผ์กฑํ•ด๋ฐฉ, ๊ณ„๊ธ‰ํ•ด๋ฐฉ, ์ธ๊ฐ„ ํ•ด๋ฐฉ์˜ ๋ฆฌ๋ก , ์‚ฌํšŒ๊ฐœ์กฐ, ์ž์—ฐ๊ฐœ์กฐ, ์ธ๊ฐ„๊ฐœ์กฐ๋ฆฌ๋ก ์„ ์ง‘๋Œ€์„ฑํ•œ ์™„๋ฒฝํ•œ ํ˜๋ช…๋ฆฌ๋ก ์ด๊ณ  ํ˜๋ช…์  ๋ น๋„์˜ ๋ณธ์งˆ์€ ์ธ๋ฏผ๋Œ€์ค‘์— ๋Œ€ํ•œ ๋‹น๊ณผ ์ˆ˜๋ น์˜ ๋ น๋„์ด๋‹ค. - ์ˆ˜๋ น์˜ ์œ ์ผ์  ๋ น๋„์ฒด๊ณ„์˜ ๋ณธ์งˆ์˜ ํ•˜๋‚˜๋Š” ์ˆ˜๋ น์˜ ํ˜๋ช…์‚ฌ์ƒ์„ ์œ ์ผํ•œ ์ง€๋„์  ์ง€์นจ์œผ๋กœ ํ•˜์—ฌ ํ˜๋ช…๊ณผ ๊ฑด์„ค์„ ์ˆ˜ํ–‰ํ•˜๋Š” ์ฒด๊ณ„๋ผ๋Š” ๊ฒƒ์ด๋‹ค. - ์ด์™€ ๊ฐ™์€ ๋ฌธ๊ฑด์€ ๊น€์ผ์„ฑ ๊ฐœ์ธ ์šฐ์ƒํ™”๋ฅผ ์œ„ํ•ด ๋ถํ•œ์‹ ์ˆ˜๋ น๊ด€์— ์ž…๊ฐํ•œ ๊น€์ผ์„ฑ์˜ ์—…์  ๋ฏธํ™” ๋ฐ ๋ถํ•œ ์ฒด์ œ ์ฐฌ์–‘ยท์„ ์ „ํ•˜๋Š” ๊ฒƒ์ด ์ฃผ๋œ ๋‚ด์šฉ์ด๋‹ค. ใ€Š10๋‹จ๊ณ„ํ•™์Šต ํด๋” ์ค‘ 5๋‹จ๊ณ„-๊น€์ผ์„ฑ์ฃผ์˜์ด์„œ ํด๋” ๋‚ด โ€˜์ฃผ์ฒด์‚ฌ์ƒ ์ด์„œ 1 ์ฃผ์ฒด์‚ฌ์ƒ์˜ ์ฒ ํ•™์  ์›๋ฆฌโ€™์˜ ์ฃผ์š” ๋‚ด์šฉใ€‹ - ์ฃผ์ฒด์‚ฌ์ƒ์˜ ์ฒ ํ•™์  ์›๋ฆฌ๋ฅผ ์„ค๋ช…ํ•˜๋Š” ๋‚ด์šฉ์œผ๋กœ ๊น€์ผ์„ฑ์ด ์ฐฝ์‹œํ•˜๊ณ  ๊น€์ •์ผ์ด ๋ฐœ์ „์‹œ์ผœ ๋‚˜๊ฐ€๋Š” ๋ถˆ๋ฉธ์˜ ํ˜๋ช…์‚ฌ์ƒ์ธ ์ฃผ์ฒด์‚ฌ์ƒ์„ ๊นŠ์ด ์ฒด๋“ํ•˜๊ณ  ์ฒ ์ €ํžˆ ๊ตฌํ˜„ํ•ด ๋‚˜๊ฐ€๋Š”๋ฐ ๋„์›€์„ ์ฃผ๊ธฐ ์œ„ํ•ด ์ง‘ํ•„๋˜์—ˆ๋‹ค๋Š” ์ทจ์ง€๋กœ ๋ฐœ๊ฐ„ ๋ฐฐ๊ฒฝ์„ ๋ฐํžˆ๋ฉด์„œ - ์ฃผ์ฒด์‚ฌ์ƒ์˜ ์ฐฝ์‹œ ๋ฐ ๊ณ„์Šน๋ฐœ์ „๊ณผ์ •, ์ฃผ์ฒด์‚ฌ์ƒ์˜ ํŠน์ง•๊ณผ ์—ญ์‚ฌ์  ์ง€์œ„, ์ฒ ํ•™์  ์›๋ฆฌ๋กœ์„œ์˜ ๊ธฐ์ดˆ์™€ ๊ทผ๋ณธ๋ฌธ์ œ, ์‚ฌ๋žŒ์ด ์„ธ๊ณ„์™€ ์—ญ์‚ฌ์—์„œ ์ฐจ์ง€ํ•˜๋Š” ์ง€์œ„, ์ฃผ์ฒด์‚ฌ์ƒ์ด ๋ฐํžŒ ์„ธ๊ณ„์— ๋Œ€ํ•œ ๊ด€์ ๊ณผ ์ž…์žฅ ๋“ฑ์ด ๊ธฐ์ˆ ๋œ ๋ถํ•œ ์ฃผ์ฒด์‚ฌ์ƒ์„ ์„ ์ „ํ•˜๋Š” ๋ฌธ๊ฑด์ด๋‹ค. ใ€Š10๋‹จ๊ณ„ํ•™์Šต ํด๋” ์ค‘ 6๋‹จ๊ณ„-์œ„๋Œ€์„ฑ ํด๋” ๋‚ด โ€˜์‚ฌ๋ž‘์˜ ์ •์น˜๊ฐ€ ๊น€์ •์ผ ์žฅ๊ตฐโ€™์˜ ์ฃผ์š” ๋‚ด์šฉใ€‹ - ๊น€์ •์ผ์˜ ํ˜๋ช…์— ๋Œ€ํ•œ ์‚ฌ๋ž‘, ๋™์ง€์— ๋Œ€ํ•œ ์‚ฌ๋ž‘, ๋ฏผ์ค‘์— ๋Œ€ํ•œ ์‚ฌ๋ž‘, ์กฐ๊ตญ์— ๋Œ€ํ•œ ์‚ฌ๋ž‘, ๋ฏผ์กฑ์— ๋Œ€ํ•œ ์‚ฌ๋ž‘ ๋“ฑ์„ ๋ฌ˜์‚ฌํ•˜๋ฉด์„œ, ๊น€์ •์ผ์€ ํ˜„์„ธ๊ฐ€ ๋งž์ดํ•œ ์œ„๋Œ€ํ•œ ์‚ฌ๋ž‘์˜ ์ •์น˜๊ฐ€๋ผ๋Š” ๋“ฑ ๊น€์ •์ผ ๊ฐœ์ธ์„ ์šฐ์ƒํ™”ํ•˜๊ณ  ์ฐฌ์–‘ยท๋ฏธํ™”ํ•˜๋Š” ๊ฒƒ์„ ์ฃผ๋œ ๋‚ด์šฉ์œผ๋กœ ํ•˜๊ณ  ์žˆ๋‹ค. ใ€Š10๋‹จ๊ณ„ํ•™์Šต ํด๋” ์ค‘ ๋ฌธํ™”์˜ˆ์ˆ  ํด๋” ๋‚ด โ€˜์••๋ก๊ฐ•โ€™์˜ ์ฃผ์š” ๋‚ด์šฉใ€‹ - ๊น€์ผ์„ฑ์˜ ์—…์ ์„ ์šฐ์ƒํ™”ํ•œ ๋ถํ•œ ์ด์„œใ€Š๋ถˆ๋ฉธ์˜ ๋ ฅ์‚ฌใ€‹์žฅํŽธ์†Œ์„ค ์ค‘ ํ•˜๋‚˜๋กœ 1936๋…„~1937๋…„๊ฐ„ ์ด๋ฅธ๋ฐ” โ€˜๋ณด์ฒœ๋ณด ์ „ํˆฌโ€™๋ฅผ ์ค‘์‹ฌ์œผ๋กœ ํ•œ ๊น€์ผ์„ฑ์˜ ํ•ญ์ผ๋ฌด์žฅํˆฌ์Ÿ๊ณผ์ •์—์„œ์˜ ์—…์ ๊ณผ ์œ„๋Œ€์„ฑ์„ ๋ฏธํ™”ยท์ฐฌ์–‘ํ•˜์—ฌ ๋ถํ•œ์ฒด์ œ์˜ ์ •๋‹น์„ฑ์„ ์„ ์ „ํ•˜๋Š” ๊ฒƒ์„ ์ฃผ๋œ ๋‚ด์šฉ์œผ๋กœ ํ•˜๊ณ  ์žˆ๋‹ค. ใ€Š10๋‹จ๊ณ„ํ•™์Šต ํด๋” ์ค‘ ๋ฌธํ™”์˜ˆ์ˆ  ํด๋” ๋‚ด โ€˜๋‹ป์€ ์˜ฌ๋ž๋‹คโ€™์˜ ์ฃผ์š” ๋‚ด์šฉใ€‹ - ๊น€์ผ์„ฑ์˜ ์—…์ ์„ ์šฐ์ƒํ™”ํ•œ ๋ถํ•œ ์ด์„œใ€Š๋ถˆ๋ฉธ์˜ ๋ ฅ์‚ฌใ€‹์žฅํŽธ์†Œ์„ค ์ค‘ ํ•˜๋‚˜๋กœ, 1925๋…„~1926๋…„ โ€˜ํƒ€๋„์ œ๊ตญ์ฃผ์˜๋™๋งนโ€™ ๊ฒฐ์„ฑ๊นŒ์ง€๋ฅผ ๊ทธ ๋ฐฐ๊ฒฝ์œผ๋กœ ํ•˜๋ฉด์„œ ๋ชจ๋“  ํ•ญ์ผํˆฌ์Ÿ ์ค‘์‹ฌ์—๋Š” ๊น€์ผ์„ฑ์˜ ์˜๋„๋ ฅ์ด ์žˆ์—ˆ๊ณ , ๊น€์ผ์„ฑ์ด ํ›Œ๋ฅญํ•œ ๋…๋ฆฝ์šด๋™๊ฐ€์˜ ๊ฐ€๊ณ„์—์„œ ํƒœ์–ด๋‚˜ ์–ด๋ฆฐ ๋‚˜์ด์— ์กฐ์„ ๋…๋ฆฝ์˜ ์‚ฌ์ƒ์  ๋งค๋“ญ์„ ์ง€์–ด์ค„ ์ˆ˜ ์žˆ๋Š” ์˜์›…์  ๊ท€์žฌ๋กœ ๋ฌ˜์‚ฌํ•˜๋Š” ๋“ฑ ๊น€์ผ์„ฑ ๊ฐœ์ธ ์šฐ์ƒํ™” ๋ฐ ๋ถํ•œ์‹ ์‚ฌํšŒ์ฃผ์˜์˜ ์ •๋‹น์„ฑ ๋“ฑ์˜ ์„ ์ „์„ ์œ„ํ•ด ๊น€์ผ์„ฑ์˜ ํ•ญ์ผํ˜๋ช…ํˆฌ์Ÿ์„ ์ฐฌ์–‘ยท๋ฏธํ™”ํ•˜๋Š” ๊ฒƒ์„ ์ฃผ์š” ๋‚ด์šฉ์œผ๋กœ ํ•˜๊ณ  ์žˆ๋‹ค. ใ€Š10๋‹จ๊ณ„ํ•™์Šต ํด๋” ์ค‘ ๋ฌธํ™”์˜ˆ์ˆ  ํด๋” ๋‚ด โ€˜์ด๋Œ€โ€™์˜ ์ฃผ์š” ๋‚ด์šฉใ€‹ - ๊น€์ •์ผ์˜ ์—…์ ์„ ์šฐ์ƒํ™”ํ•œ ๋ถํ•œ ์ด์„œ ใ€ˆ๋ถˆ๋ฉธ์˜ ํ–ฅ๋„ใ€‰ ์žฅํŽธ์†Œ์„ค ์ค‘ ํ•˜๋‚˜๋กœ, 1990๋…„๋Œ€ ํ›„๋ฐ˜ ๋ฏธ๊ตญ๊ณผ์˜ ํ•ต ๋Œ€๊ฒฐ ๊ตญ๋ฉด์—์„œ ๊น€์ •์ผ์ด ์„ ๊ตฐ์ •์น˜์˜ ๊ธฐ์น˜๋ฅผ ๋‚ด๊ฑธ๊ณ  ์ „๋ฐฉ์ง€ํœ˜์†Œ์™€ ๊ตฐ๋ถ€๋Œ€๋ฅผ ์ฐพ์•„ ๋ณ‘์‚ฌ๋“ค์˜ ์ƒํ™œ์„ ์‚ดํ”ผ๋ฉด์„œ ๋ถ€๋Œ€์˜ ์ž‘์ „๊ณผ ํ›ˆ๋ จ์„ ํ˜„์ง€์—์„œ ์ง€๋„ํ•˜๋Š” ๊ณผ์ •์„ ๋ฌ˜์‚ฌํ•˜์—ฌ, - ๋ถํ•œ์ด ์‚ฌํšŒ์ฃผ์˜๋ฅผ ์ง€์ผœ๋‚˜๊ฐ€๋Š” ๊ฒƒ์€ ๊น€์ •์ผ์˜ โ€˜์„ ๊ตฐ์ •์น˜โ€™ ๋•๋ถ„์ž„์„ ํ˜•์ƒํ™”ํ•˜์—ฌ ๊น€์ •์ผ ๋ฐ ๊ทธ์˜ ํ†ต์น˜๋ฐฉ์‹์ธ ์„ ๊ตฐ์ •์น˜๋ฅผ ์ฐฌ์–‘ยท๋ฏธํ™”ํ•˜๋ฉด์„œ, ๋‚จํ•œ์˜ ๋ฐ˜์ œ๋ฐ˜๋ฏธ ๊ณต์‚ฐ์ฃผ์˜ํ˜๋ช… ํ†ต์ผ์„ ์„ ์ „ยท์„ ๋™ํ•˜๋Š” ๊ฒƒ์„ ์ฃผ์š” ๋‚ด์šฉ์œผ๋กœ ํ•˜๊ณ  ์žˆ๋‹ค. ใ€Š10๋‹จ๊ณ„ํ•™์Šต ํด๋” ์ค‘ ๋ฌธํ™”์˜ˆ์ˆ  ํด๋” ๋‚ด โ€˜๋ ฅ์‚ฌ์˜ ๋Œ€ํ•˜โ€™์˜ ์ฃผ์š” ๋‚ด์šฉใ€‹ - ๊น€์ •์ผ์˜ ์—…์ ์„ ์šฐ์ƒํ™”ํ•œ ๋ถํ•œ ์ด์„œ ใ€ˆ๋ถˆ๋ฉธ์˜ ํ–ฅ๋„ใ€‰ ์žฅํŽธ์†Œ์„ค ์ค‘ ํ•˜๋‚˜๋กœ 1993๋…„ ํ•œ๋ฏธํ•ฉ๋™๊ตฐ์‚ฌํ›ˆ๋ จ ์‹ค์‹œ์™€ ์ด์— ๋Œ€์‘ํ•œ ๋ถํ•œ์˜ ์ค€์ „์‹œ์ƒํƒœ ์„ ํฌ ๋“ฑ ๋ถํ•œ๊ณผ ๋ฏธ๊ตญ ๊ฐ„์— ๋ฒŒ์–ด์กŒ๋˜ ํ•ต๋Œ€๊ฒฐ์„ ์†Œ์žฌ๋กœ ํ•˜์—ฌ ๋ฏธ๊ตญ์„ ์นจ๋žต์ž๋กœ ๊ทœ์ •ํ•˜๊ณ , ๊น€์ •์ผ์˜ ์ง€๋„๋ ฅ๊ณผ ์œ„๋Œ€์„ฑ์œผ๋กœ ๋ถ์นจ์„ ๊ธฐํšํ•œ ๋ฏธ๊ตญ์„ ๊ตด๋ณต์‹œ์ผฐ๋‹ค๋Š” ์ทจ์ง€๋กœ ๋ถํ•œ์˜ ํ•ต๋ณด์œ  ์ •๋‹น์„ฑ ์˜นํ˜ธ ๋ฐ ๊น€์ •์ผ์˜ ์˜๋„๋ ฅ์„ ๋ฏธํ™”, ์ฐฌ์–‘ํ•˜๋Š” ๊ฒƒ์„ ์ฃผ์š” ๋‚ด์šฉ์œผ๋กœ ํ•˜๊ณ  ์žˆ๋‹ค. ใ€Š10๋‹จ๊ณ„ํ•™์Šต ํด๋” ์ค‘ 2๋‹จ๊ณ„-๋ณ€ํ˜์ด๋ก 4๋Œ€๊ฐœ๋ก  ํด๋” ๋‚ด โ€˜ํ˜„๋Œ€์กฐ์„ ์—ญ์‚ฌโ€™์˜ ์ฃผ์š” ๋‚ด์šฉใ€‹ - ํ•ญ์ผ๋ฌด์žฅํˆฌ์Ÿ์˜ ๋น›๋‚˜๋Š” ์Šน๋ฆฌ๋Š” ์˜ค์ง ํƒ์›”ํ•œ ๊ตฐ์‚ฌ์ „๋žต๊ฐ€์ธ ๊น€์ผ์„ฑ์˜ ํ˜„๋ช…ํ•œ ์˜๋„์— ์˜ํ•˜์—ฌ์„œ๋งŒ ์ด๋ฃฉ๋  ์ˆ˜ ์žˆ์—ˆ๋‹ค. - ๋ฏธ์ œ๋Š” โ€˜๋ถ๋ฒŒโ€™๊ณ„ํš์„ ๋งŒ๋“ค์–ด ๋†“๊ณ  ๊ทธ๊ฒƒ์„ ์‹ค์ฒœํ•˜๊ธฐ ์œ„ํ•˜์—ฌ 38๋„์„  ๋ฌด์žฅ์นจ์Šต์‚ฌ๊ฑด์„ ๊ทธ ์–ด๋Š ๋•Œ๋ณด๋‹ค๋„ ๋Œ€๋Œ€์ ์œผ๋กœ ์ผ์œผ์ผฐ๋‹ค. - ๋ฏธ์ œ์™€ ์ด์Šน๋งŒ ์ •๊ถŒ์€ 1950๋…„ 6์›” 25์ผ ์กฐ์„ ์ธ๋ฏผ ๋ฏผ์ฃผ์ฃผ์˜๊ณตํ™”๊ตญ์„ ๋ฐ˜๋Œ€ํ•˜๋Š” ์นจ๋žต์ „์Ÿ์„ ์ผ์œผ์ผฐ๋‹ค. - ์ด์™€ ๊ฐ™์ด ๊น€์ผ์„ฑ์˜ ํ•ญ์ผ๋ฌด์žฅํ˜๋ช…ํˆฌ์Ÿ์„ ์ฐฌ์–‘ยท๋ฏธํ™”ํ•˜๋Š” ํ•œํŽธ, 6ยท25์ „์Ÿ์„ ๋ฏธ๊ตญ์˜ ์นจ๋žต์— ๋Œ€ํ•œ โ€˜์กฐ๊ตญํ•ด๋ฐฉ์ „์Ÿโ€™์œผ๋กœ ์™œ๊ณกํ•˜๋Š” ๋“ฑ ๊น€์ผ์„ฑ ๊ฐœ์ธ ์šฐ์ƒํ™” ๋ฐ ๋ถํ•œ์ฒด์ œ์˜ ์ •๋‹น์„ฑ์„ ์„ ์ „ํ•˜๊ธฐ ์œ„ํ•ด ๊น€์ผ์„ฑ ํ–‰์ ์˜ ์ฐฌ์–‘ยท๋ฏธํ™” ๋“ฑ์„ ์ฃผ์š” ๋‚ด์šฉ์œผ๋กœ ํ•˜๊ณ  ์žˆ๋‹ค. ใ€Š10๋‹จ๊ณ„ํ•™์Šต ํด๋” ์ค‘ 6๋‹จ๊ณ„-์œ„๋Œ€์„ฑ ํด๋” ๋‚ด โ€˜๊น€์ •์ผ ์ง€๋„์ž ์ œ1๋ถ€โ€™์˜ ์ฃผ์š” ๋‚ด์šฉใ€‹ - ๊น€์ •์ผ์ด ํƒœ์–ด๋‚œ ์ดํ›„ ์–ด๋ฆฐ ์‹œ์ ˆ ์„ฑ์žฅ๊ณผ์ •๊ณผ ๋Œ€ํ•™์‹œ์ ˆ ์ดํ›„ ๋ณด์—ฌ์ค€ ์˜๋„๋ ฅ, ๋ฌธํ™”๋ถ„์•ผ์—์„œ์˜ ์—…์  ๋“ฑ์„ ์ค‘์‹ฌ์œผ๋กœ ๋ฌ˜์‚ฌํ•˜๋ฉด์„œ ๊น€์ •์ผ์˜ ์ง€๋„์ž๋กœ์„œ์˜ ํ’๋ชจ์™€ ๋ชจ์Šต์„ ์ฐฌ์–‘ยท์šฐ์ƒํ™”ํ•˜๋Š” ๋ฌธ๊ฑด์ด๋‹ค. ใ€Š10๋‹จ๊ณ„ํ•™์Šต ํด๋” ์ค‘ 8๋‹จ๊ณ„-์ฃผ์š”๋…ธ์ž‘๋ชจ์Œ ํด๋” ๋‚ด โ€˜์กฐ๊ตญํ†ต์ผ 10๋Œ€ ๋…ธ์ž‘โ€™์˜ ์ฃผ์š” ๋‚ด์šฉใ€‹ - โ‰ช์กฐ๊ตญํ†ต์ผ์„ ์œ„ํ•œ ์ „๋ฏผ์กฑ๋Œ€๋‹จ๊ฒฐ10๋Œ€๊ฐ•๋ น, โ‰ช์กฐ๊ตญํ†ต์ผ3๋Œ€์›์น™์„ ๊ด€์ฒ ํ•˜๊ธฐ ์œ„ํ•˜์—ฌ ๊ฒฌ๊ฒฐํžˆ ํˆฌ์Ÿํ•˜์žโ‰ซ ๋“ฑ์˜ ๋ถํ•œ ์›์ „์œผ๋กœ ๊ตฌ์„ฑ๋˜์–ด ์žˆ๋Š”๋ฐ”, ๋Œ€๋‚จํ˜๋ช…์ „๋žต์— ๋”ฐ๋ฅธ ๋ถํ•œ์‹ ์‚ฌํšŒ์ฃผ์˜์ฒด์ œ ํ†ต์ผ ์„ ์ „ยท์„ ๋™ ๋“ฑ์„ ์ฃผ์š” ๋‚ด์šฉ์œผ๋กœ ํ•˜๊ณ  ์žˆ๋‹ค. ใ€ŠK1์ €์ž‘์ง‘hwpํŒ ํด๋” ๋‚ด โ€˜๊น€์ผ์„ฑ ์ €์ž‘์ง‘ 1~44โ€™ ์ œํ•˜์˜ ์ฃผ์š” ๋‚ด์šฉใ€‹ - 1930. 6. ~ 1994. 7.๊ฐ„ ๊น€์ผ์„ฑ์˜ ์—ฐ์„ค๋ฌธยท๋‹ดํ™”๋ฌธ ๋“ฑ์„ ์—ฐ๋„๋ณ„๋กœ ์ˆ˜๋กํ•œ ์ €์ž‘์ง‘์œผ๋กœ, ๊น€์ผ์„ฑ์˜ ํ•ญ์ผ๋…๋ฆฝ์šด๋™ ์—…์ ์„ ๋ฏธํ™”ยท์„ ์ „ํ•˜๊ณ , ๊น€์ผ์„ฑ๊ณผ ๋ถํ•œ์ฒด์ œ๋ฅผ ๋ฏธํ™”ยท์ฐฌ์–‘ํ•˜๋ฉฐ ๋ถํ•œ์„ ์ด๋ฅธ๋ฐ” ๋ง‘์Šคยท๋ ˆ๋‹Œ์ฃผ์˜๋ฅผ ํ˜„์‹ค์— ๋งž๊ฒŒ ์ ์šฉํ•˜์—ฌ ๊ฑด์„ค๋œ โ€˜์ง„๋ณด์  ๋ฏผ์ฃผ์ฃผ์˜ ์‚ฌํšŒโ€™๋ผ๊ณ  ์„ ์ „ํ•˜๋Š” ํ•œํŽธ, ๊น€์ผ์„ฑ์„ ์กฐ๊ตญํ•ด๋ฐฉ๊ณผ ๋‚จ๋ถํ†ต์ผ์„ ์œ„ํ•ด ์• ์“ฐ๊ณ  ์žˆ๋Š” ์˜์›…์  ์ธ๋ฌผ๋กœ ๋ฌ˜์‚ฌํ•˜๋ฉด์„œ ๊น€์ผ์„ฑ์„ ์ˆ˜๋ น์œผ๋กœ ๋ชจ์‹œ๊ณ  ๊ตฐ์‚ฌยท์ •์น˜ยท๊ฒฝ์ œยท์‚ฌ์ƒ ๋“ฑ ๋ชจ๋“  ๋ฉด์—์„œ ์ฃผ์ฒด์‚ฌ์ƒ์„ ๊ธฐ๋ฐ˜์œผ๋กœ ๊น€์ผ์„ฑ ํ˜๋ช…์ „ํ†ต์„ ํ™•๋ฆฝํ•˜๋ฉฐ ๋Œ€ํ•œ๋ฏผ๊ตญ์„ ๋ถํ•œ์‹ ์‚ฌํšŒ์ฃผ์˜ํ™”ํ•  ๊ฒƒ์„ ์„ ์ „ยท์„ ๋™ํ•˜๋Š” ๋‚ด์šฉ๋“ค๋กœ ๊ตฌ์„ฑ๋˜์–ด ์žˆ๋‹ค. ใ€ŠK2์ €์ž‘์ง‘hwpํŒ ํด๋” ๋‚ด โ€˜๊น€์ •์ผ ์ €์ž‘์ง‘ 1~3, 5~14โ€™ ์ œํ•˜์˜ ์ฃผ์š” ๋‚ด์šฉใ€‹ - 1964. 4. ~ 1999. 9.๊ฐ„ ๊น€์ •์ผ์˜ ์ฃผ์š” ๋‹ดํ™” ๋‚ด์šฉ์„ ๋ฐœ์ทŒยท์ •๋ฆฌํ•œ ๋ฌธ๊ฑด์œผ๋กœ โ€˜์ˆ˜๋ น๋‹˜์˜ ์œ„๋Œ€์„ฑ์„ ๋‚จ์กฐ์„ ์ธ๋ฏผ๋“ค์—๊ฒŒ ๋„๋ฆฌ ์„ ์ „ํ•  ๋ฐ ๋Œ€ํ•˜์—ฌโ€™, โ€˜๋‹น์ƒํ™œ์„ ๊ฐ•ํ™”ํ•˜์—ฌ ๊ฐ„๋ถ€๋“ค์„ ์ฒ ์ €ํžˆ ํ˜๋ช…ํ™”ํ•˜์žโ€˜ ๋“ฑ 317ํšŒ์— ๊ฑธ์นœ ๊น€์ •์ผ ๊ฐœ์ธ ์šฐ์ƒํ™”๋ฅผ ์œ„ํ•ด ๊น€์ •์ผ์˜ ์–ธ๋™ ๋‚ด์šฉ์„ ์ˆ˜๋กํ•˜์—ฌ ์ด๋ฅผ ์ฐฌ์–‘ยท์„ ์ „ํ•˜๋Š” ๋‚ด์šฉ์˜ ๋ฌธ๊ฑด๋“ค๋กœ ๊ตฌ์„ฑ๋˜์–ด ์žˆ๋‹ค. ใ€Š๊ต์–‘ ํด๋” ์ค‘ ์œ„๋Œ€์„ฑํ•™์Šต ํด๋” ๋‚ด โ€˜์„ ๊ตฐ์ •์น˜ : ์ฃผ์ฒด์‚ฌํšŒ์ฃผ์˜ ์ƒ๋ช…์„ โ€™์˜ ์ฃผ์š” ๋‚ด์šฉใ€‹ - ์„ ๊ตฐ์ •์น˜๋ฅผ ํ˜๋ช…์˜ ์ฃผ์ฒด๋ฅผ ํŠผํŠผํžˆ ๋‹ค์ง€๋Š” ํ˜๋ช…์ ์ธ ์ •์น˜๋ฐฉ์‹, ๊ฐ•์„ฑ๋Œ€๊ตญ๊ฑด์„ค์„ ํž˜์žˆ๊ฒŒ ๋‹ค๊ทธ์ณ ๋‚˜๊ฐ€๋Š” ์‚ฌํšŒ์ฃผ์˜ ์ •์น˜๋ฐฉ์‹, ์‚ฌํšŒ์ฃผ์˜ ์กฐ์„ ์˜ ์กด์—„์„ ๋งŒ๋ฐฉ์— ๋–จ์น˜๊ฒŒ ํ•˜๋Š” ์œ„๋ ฅํ•œ ์ •์น˜๋ฐฉ์‹, ์ฃผ์ฒด์‚ฌํšŒ์ฃผ์˜ ์œ„์—…์™„์„ฑ์„ ์œ„ํ•˜์—ฌ ํ•ญ๊ตฌ์ ์œผ๋กœ ํ‹€์–ด์ฅ๊ณ  ๋‚˜๊ฐ€์•ผ ํ•  ์ „๋žต์  ์ •์น˜๋ฐฉ์‹ ๋“ฑ์œผ๋กœ ๋ฌ˜์‚ฌํ•˜๋ฉด์„œ ๊น€์ •์ผ์˜ ์ง€๋„๋ ฅ๊ณผ ์„ ๊ตฐ์ •์น˜์˜ ์ •๋‹น์„ฑ์„ ์„ ์ „ยท์ฐฌ์–‘ํ•˜๋Š” ๊ฒƒ์ด ์ฃผ์š” ๋‚ด์šฉ์ด๋‹ค. ใ€Š๊ต์–‘ ํด๋” ์ค‘ ์ •์ˆ˜ ํด๋” ๋‚ด โ€˜์ฃผ์ฒด์‚ฌ์ƒ ๋…ธ์ž‘ 60์„ โ€™์˜ ์ฃผ์š” ๋‚ด์šฉใ€‹ - 1930. 6. ~ 1998. 4.๊ฐ„ โ‰ช์กฐ์„ ํ˜๋ช…์˜ ์ง„๋กœโ‰ซ, โ‰ช์˜จ ๋ฏผ์กฑ์ด ๋Œ€๋‹จ๊ฒฐํ•˜์—ฌ ์กฐ๊ตญ์˜ ์ž์ฃผ์  ํ‰ํ™”ํ†ต์ผ์„ ์ด๋ฃฉํ•˜์žโ‰ซ ๋“ฑ์˜ ์ฃผ์ฒด์‚ฌ์ƒ ๋…ธ์ž‘ 60๊ฑด์„ ๋ชจ์•„๋†“์€ ๋ฌธ๊ฑด์œผ๋กœ โ€˜์ฃผ์ฒด์‚ฌ์ƒ์€ ์ฃผ์ฒด์‹œ๋Œ€์˜ ์š”๊ตฌ์™€ ํ˜๋ช…ํˆฌ์Ÿ์˜ ๊ฒฝํ—˜์— ๊ธฐ์ดˆํ•˜์—ฌ ์ฐฝ์‹œ๋œ ๋…์ฐฝ์  ํ˜๋ช…์‚ฌ์ƒ์œผ๋กœ ํ˜๋ช…๊ณผ ๊ฑด์„ค์— ๊ฐ€์žฅ ์˜ฌ๋ฐ”๋ฅธ ํ•ด๋‹ต์„ ์ฃผ๋ฏ€๋กœ ๋‹น๊ณผ ์ˆ˜๋ น์— ์ถฉ์„ฑํ•˜๊ณ  ๋ณต์ข…ํ•˜์—ฌ์•ผ ํ•œ๋‹ค.โ€™๋ฉฐ ์ฃผ์ฒด์‚ฌ์ƒ์œผ๋กœ ์‚ฌ์ƒ๋ฌด์žฅํ•˜์—ฌ ๋Œ€ํ•œ๋ฏผ๊ตญ ์ „์—ญ์„ ๊ณต์‚ฐ์ฃผ์˜ํ™” ์‹œ์ผœ์•ผ ํ•œ๋‹ค๋Š” ๊ฒƒ์„ ์ฃผ์š” ๋‚ด์šฉ์œผ๋กœ ํ•˜๊ณ  ์žˆ๋‹ค. ์ด์™€ ๊ฐ™์ด ์œ ์—์Šค๋น„(USB 32G) ํ˜•์‹์˜ ๋””์ €ํ„ธ์ €์žฅ๋งค์ฒด๋Š” ๊น€์ผ์„ฑ์˜ ์ „ ๊ฐ€๊ณ„๋ฅผ ํ•ญ์ผ ๋…๋ฆฝํˆฌ์‚ฌ๋กœ ๋ฌ˜์‚ฌํ•˜๊ณ  ๊น€์ผ์„ฑ์˜ ํ•ญ์ผ๋ฌด์žฅํˆฌ์Ÿ์„ ๋ฏธํ™”ยท์ฐฌ์–‘ํ•˜๋Š” ๋“ฑ ๊น€์ผ์„ฑ์„ ์šฐ์ƒํ™”ํ•˜๋Š” ์ทจ์ง€์˜ ๋‚ด์šฉ, 6ยท25์ „์Ÿ์„ ๋ฏธ๊ตญ์— ์˜ํ•œ ์นจ๋žต์ „์Ÿ์ธ ์–‘ ์™œ๊ณกํ•˜๋ฉด์„œ ๊น€์ผ์„ฑ์ด ๋ฏธ๊ตญ์˜ ์นจ๋žต์„ ๋ง‰์•„๋‚ธ ์กฐ๊ตญํ•ด๋ฐฉ์ „์Ÿ์ด์—ˆ๋‹ค๋Š” ์ทจ์ง€๋กœ ์„ ์ „ํ•˜๋Š” ๋‚ด์šฉ, ์กฐ์„ ๋กœ๋™๋‹น์˜ ์ผ๋‹น๋…์žฌ๋ฅผ ์ •๋‹นํ™”ํ•˜๋ฉด์„œ ๋Œ€๋‚จํ˜๋ช…๋…ธ์„ ์„ ์„ ์ „ยท์„ ๋™ํ•˜๋Š” ์ทจ์ง€์˜ ๋‚ด์šฉ, ๋ถํ•œ์‹ ์‚ฌํšŒ์ฃผ์˜๊ฐ€ ์ฃผ์ฒด์‚ฌ์ƒ์œผ๋กœ ๋ฌด์žฅํ•œ ๋ถํ•œ ์ธ๋ฏผ๋“ค์— ์˜ํ•ด ์ ˆ๋Œ€์  ์ง€์ง€๋ฅผ ๋ฐ›๊ณ  ์žˆ๋‹ค๋Š” ์ทจ์ง€๋กœ ๋ถํ•œ ์‚ฌํšŒ์ฃผ์˜์ฒด์ œ์˜ ์šฐ์›”์„ฑ ๋ฐ ํ˜๋ช…์  ์ •๋‹น์„ฑ ๋“ฑ์„ ๊ฐ•์กฐํ•˜๋Š” ์ทจ์ง€์˜ ๋‚ด์šฉ, ๊น€์ผ์„ฑ ์ฃผ์ฒด์‚ฌ์ƒ์˜ ์šฐ์›”์„ฑ์„ ๊ฐ•์กฐํ•˜๋ฉด์„œ ๋ฏธ ์ œ๊ตญ์ฃผ์˜์— ์˜ˆ์†๋œ ๋‚จํ•œ ๋ฏผ์ค‘์„ ํ•ด๋ฐฉํ•˜๊ณ  ํ•œ๋ฐ˜๋„์˜ ํ†ต์ผ์„ ์‹คํ˜„ํ•˜๊ธฐ ์œ„ํ•ด ๋‚จํ•œ ๋ฏผ์ค‘์„ ์ฃผ์ฒด์‚ฌ์ƒ์œผ๋กœ ์˜์‹ํ™”ํ•˜์—ฌ์•ผ ํ•œ๋‹ค๋Š” ์ทจ์ง€์˜ ๋‚ด์šฉ, ์ž๋ณธ์ฃผ์˜๊ฒฝ์ œ์ œ๋„๋Š” ์ธ๋ฏผ๋Œ€์ค‘์„ ์ˆ˜ํƒˆํ•˜๊ธฐ ์œ„ํ•œ ๋„๊ตฌ์— ๋ถˆ๊ณผํ•˜๊ณ  ์ฃผ์ฒด์‚ฌ์ƒ์ด ๊ทผ๊ฐ„์ด ๋œ ๋ถํ•œ์˜ ์‚ฌํšŒ์ฃผ์˜ ์ฒด์ œ๋Š” ๋ชจ๋“  ๊ตฌ์„ฑ์›๋“ค์ด ํ‰๋“ฑํ•˜๊ณ  ํ’์กฑํ•œ ๋ฌผ์งˆ์  ๋ฌธํ™”์ƒํ™œ์ด ๋ณด์žฅ๋˜๋Š” ์‚ฌํšŒ๋ผ๊ณ  ๋ฏธํ™”ํ•˜๋ฉด์„œ ๋ถํ•œ ์‚ฌํšŒ์ฃผ์˜ ์ฒด์ œ์™€ ์ฃผ์ฒด์‚ฌ์ƒ์„ ์ฐฌ์–‘ยท์„ ์ „ํ•˜๋Š” ์ทจ์ง€์˜ ๋‚ด์šฉ, ๊น€์ผ์„ฑยท๊น€์ •์ผ์ฃผ์˜๋ฅผ ์ถ”์ข…ํ•˜๋Š” ๋ถํ•œ ์‚ฌํšŒ์ฃผ์˜์ฒด์ œ์˜ ์šฐ์ˆ˜์„ฑ ๋ฐ ์ฃผ์ฒด์‚ฌ์ƒ๊ณผ ์„ ๊ตฐ์ •์น˜๋ฅผ ์„ ์ „ํ•˜๋Š” ์ทจ์ง€์˜ ๋‚ด์šฉ, ๋ถํ•œ์˜ ๋Œ€๋‚จํ˜๋ช…๋ก ์˜ ์ผํ™˜์œผ๋กœ ์ฃผํ•œ๋ฏธ๊ตฐ์˜ ์ฒ ์ˆ˜๋ฅผ ์ฃผ์žฅํ•˜๊ณ  ๋ถํ•œ์˜ ์—ฐ๋ฐฉ์ œ ํ†ต์ผ๋ฐฉ์‹์˜ ์ •๋‹น์„ฑ ๋“ฑ์„ ์„ ์ „ยท์„ ๋™ํ•˜๋Š” ์ทจ์ง€์˜ ๋‚ด์šฉ, ๊น€๋ถ€์ž์˜ ํ–‰์ ์„ ์šฐ์ƒํ™”ํ•˜๋ฉด์„œ ์˜์›ํ•œ ์ถฉ์„ฑ์„ ๋งน์„ธํ•˜๋Š” ์ทจ์ง€์˜ ๋‚ด์šฉ ๋“ฑ์— ๊ด€ํ•œ ๊ฐ์ข… ๋ฌธ๊ฑดํŒŒ์ผ ๋“ฑ์„ ์ˆ˜๋กํ•˜๊ณ  ์žˆ๋‹ค. ์ด๋กœ์จ ํ”ผ๊ณ ์ธ์€ ๊ตญ๊ฐ€์˜ ์กด๋ฆฝยท์•ˆ์ „์ด๋‚˜ ์ž์œ ๋ฏผ์ฃผ์  ๊ธฐ๋ณธ์งˆ์„œ๋ฅผ ์œ„ํƒœ๋กญ๊ฒŒ ํ•œ๋‹ค๋Š” ์ •์„ ์•Œ๋ฉด์„œ ๋ฐ˜๊ตญ๊ฐ€๋‹จ์ฒด๋‚˜ ๊ทธ ๊ตฌ์„ฑ์› ๋˜๋Š” ๊ทธ ์ง€๋ น์„ ๋ฐ›์€ ์ž์˜ ํ™œ๋™์„ ์ฐฌ์–‘ยท๊ณ ๋ฌดยท์„ ์ „ ๋˜๋Š” ์ด์— ๋™์กฐํ•  ๋ชฉ์ ์œผ๋กœ ์œ„ ์ด์ ํ‘œํ˜„๋ฌผ์„ ์†Œ์ง€ํ•˜์˜€๋‹ค. ๋ผ. ํ•˜๋“œ๋””์Šคํฌ(HDD 40G) ์†Œ์ง€ ํ”ผ๊ณ ์ธ์€ 2015. 11. 13.๊ฒฝ ๊ด‘๋ช…์‹œ (์ฃผ์†Œ 3 ์ƒ๋žต)์— ์žˆ๋Š” ํ”ผ๊ณ ์ธ์˜ ์ฃผ๊ฑฐ์ง€์—์„œ ๊น€๋ถ€์ž ๋ฐ ๋ถํ•œ์ฒด์ œ๋ฅผ ์ฐฌ์–‘ยท๋ฏธํ™”ํ•˜๊ฑฐ๋‚˜, ์ฃผ์ฒด์‚ฌ์ƒ์— ์ž…๊ฐํ•œ ๋Œ€๋‚จํ˜๋ช…ํˆฌ์Ÿ์„ ์„ ์ „ยท๋™์กฐํ•˜๋Š” ๋“ฑ์˜ ๊ฐ์ข… ํŒŒ์ผ๋“ค์„ ์ฒด๊ณ„์ ์œผ๋กœ ์ •๋ฆฌยท์ €์žฅํ•œ ํ•˜๋“œ๋””์Šคํฌ(HDD 40G) ํ˜•์‹์˜ ๋””์ง€ํ„ธ์ €์žฅ๋งค์ฒด๋ฅผ ๋ณด๊ด€ํ•˜๊ณ  ์žˆ์—ˆ๋‹ค. ์œ„ ์ €์žฅ๋งค์ฒด์—๋Š” ใ€Œ์ •์„ธ๋ถ„์„์ž๋ฃŒใ€ยทใ€Œ์ดˆ๋ฒŒ์ž๋ฃŒใ€ยทใ€Œ(๋ช…์นญ 5 ์ƒ๋žต)์—ฐ๊ตฌ์†Œใ€ ๋“ฑ์˜ ์ƒ์œ„ ํด๋”๋กœ ๊ตฌ์„ฑ๋˜์–ด ๋ฌธ๊ฑดยท์Œ์„ฑยท๋™์˜์ƒ ๋“ฑ ๊ฐ์ข… ํŒŒ์ผ๋“ค์ด ๋ณด๊ด€๋˜์–ด ์žˆ๋‹ค. ์œ„ ์ƒ์œ„ํด๋” ์ค‘ ใ€Œ์ •์„ธ๋ถ„์„์ž๋ฃŒใ€ ํด๋”๋Š” ใ€Œ๋…ธ์„ ๊ณผ ์ •์ฑ…ใ€, ใ€Œ๋ฐฐ์งฑใ€, ใ€Œ์ „๋žต์ „์ˆ ์›๋ก ใ€ ๋“ฑ 3๊ฐœ์˜ ํ•˜์œ„ ํด๋”๋กœ ๋‚˜๋ˆ„์–ด์ ธ โ€˜์„ ์ „์„ ๋™์›๋ก โ€™, โ€˜2007๋…„ ๊ณต๋™์‚ฌ์„ค ํ•™์Šต์ž๋ฃŒโ€™, โ€˜๋…ธ์„ ๊ณผ ์ •์ฑ… 1ํ˜ธโ€™ ๋“ฑ ๊น€์ผ์„ฑยท๊น€์ •์ผ ๋ถ€์ž์™€ ์„ ๊ตฐ์ •์น˜๋ฅผ ์ฐฌ์–‘ยท๋ฏธํ™”ํ•˜๋Š” ์ž๋ฃŒ๋“ค์ด ์ €์žฅ๋˜์–ด ์žˆ๋‹ค. ์œ„ ์ƒ์œ„ ํด๋” ์ค‘ ใ€Œ์ดˆ๋ฒŒ์ž๋ฃŒใ€ ํด๋”๋Š” ใ€Œ[mp3]๋ฐฉ์†ก๊ฐ•์ขŒใ€, ใ€Œ์„ ๊ตฐ์ •์น˜์ž๋ฃŒใ€, ใ€Œ์œ„๋Œ€์„ฑ์ž๋ฃŒใ€, ใ€Œํ†ต์ผ(๋ถํ•œ)๊ด€๋ จ์ž๋ฃŒใ€ ๋“ฑ์˜ ํ•˜์œ„ ํด๋”๋กœ ๋‚˜๋ˆ„์–ด์ ธ ใ€Œ[mp3]๋ฐฉ์†ก๊ฐ•์ขŒใ€ ํ•˜์œ„ ํด๋”์—๋Š” ๊น€์ผ์„ฑยท๊น€์ •์ผ ๋ถ€์ž๋ฅผ ์šฐ์ƒํ™”ํ•˜์—ฌ ์ถ”์•™ํ•˜๊ณ  ๋ถํ•œ ์‚ฌํšŒ์ฃผ์˜ ์ฒด์ œ๋ฅผ ๋ฏธํ™”ยท์ฐฌ์–‘ํ•˜๋Š” ์Œ์„ฑ ๊ฐ•์˜ ์ž๋ฃŒ๋“ค์ด ์ €์žฅ๋˜์–ด ์žˆ๊ณ , ใ€Œ์„ ๊ตฐ์ •์น˜์ž๋ฃŒใ€ ํ•˜์œ„ ํด๋”์—๋Š” โ€˜๋ฏผ์กฑ๊ณผ ์„ ๊ตฐ์ •์น˜โ€™, โ€˜์„ ๊ตฐ๊ณผ ๋ฏผ์กฑ์˜ ์šด๋ช…โ€™, โ€˜๊น€์ •์ผ์žฅ๊ตฐ์˜ ํ†ต์ผ ์ „๋žตโ€™, โ€˜๊น€์ •์ผ์žฅ๊ตฐ ์„ ๊ตฐ์ •์น˜๋ฆฌ๋ก โ€™, โ€˜์„ ๊ตฐ์ •์น˜: ์ฃผ์ฒด์‚ฌํšŒ์ฃผ์˜์˜ ์ƒ๋ช…์„ โ€™, โ€˜์œ„๋Œ€ํ•œ ์˜๋„์ž ๊น€์ •์ผ๋™์ง€๊ป˜์„œ ๋ฐํžˆ์‹  ์„ ๊ตฐํ˜๋ช… ๋ น๋„โ€™, โ€˜์„ธ์ƒ์—์„œ ์ œ์ผ ๊ฐ•ํ•˜์‹  ๋ถ„โ€™ ๋“ฑ ์„ ๊ตฐ์ •์น˜๋ฅผ ์ฐฌ์–‘ํ•˜๋Š” ์ฑ…์ž, ๋…ผ๋ฌธ๋“ค์ด ์ €์žฅ๋˜์–ด ์žˆ์œผ๋ฉฐ, ใ€Œ์œ„๋Œ€์„ฑ์ž๋ฃŒใ€ ํ•˜์œ„ ํด๋”์—๋Š” โ€˜๊น€์ •์ผ ์žฅ๊ตฐ๋‹˜์€ ์–ด๋–ค ๋ถ„์ด์‹ ๊ฐ€โ€™, โ€˜์ ˆ์„ธ์˜ ์• ๊ตญ์ž ๊น€์ •์ผ ์žฅ๊ตฐโ€™, โ€˜์œ„์ธ์˜ ๋งค๋ ฅโ€™ ๋“ฑ์„ ๋น„๋กฏํ•œ ๊น€์ผ์„ฑ์˜ ํ•ญ์ผ๋ฌด์žฅํˆฌ์Ÿ์„ ์ฐฌ์–‘ยท๋ฏธํ™”ํ•˜๊ณ  ๊น€์ •์ผ์˜ ์—…์  ๋“ฑ์„ ๋ฏธํ™”ํ•˜์—ฌ ๊ฐœ์ธ ์šฐ์ƒํ™”ํ•˜๋Š” ์ž๋ฃŒ๋“ค์ด ์ €์žฅ๋˜์–ด ์žˆ๊ณ , ใ€Œํ†ต์ผ(๋ถํ•œ)๊ด€๋ จ์ž๋ฃŒใ€ ํ•˜์œ„ ํด๋”๋Š” ใ€ŒํšŒ๊ณ ๋ก ์„ธ๊ธฐ์™€ ๋”๋ถˆ์–ดใ€, ใ€ŒํšŒ์ƒ๊ธฐ ๊ฐœ์ •ํŒใ€ ๋“ฑ 2๊ฐœ์˜ ์„ธ๋ถ€ ํด๋”๋กœ ๊ตฌ์„ฑ๋˜์–ด ๊น€์ผ์„ฑ ํšŒ๊ณ ๋ก โ€˜์„ธ๊ธฐ์™€ ๋”๋ถˆ์–ด(1๊ถŒ~8๊ถŒ)โ€™ ์ฑ…์ž์™€ ๊ด€๋ จํ•œ ๊ฐ•์˜ ์ž๋ฃŒ, ๊น€์ผ์„ฑ ํ•ญ์ผ๋ฌด์žฅํˆฌ์Ÿ์„ ์ฐฌ์–‘ยท๋ฏธํ™”ํ•˜๋Š” ์ž๋ฃŒ๋“ค์ด ์ €์žฅ๋˜์–ด ์žˆ๋‹ค. ์œ„ ์ƒ์œ„ ํด๋” ์ค‘ ใ€Œ(๋ช…์นญ 5 ์ƒ๋žต)์—ฐ๊ตฌ์†Œใ€ ํด๋”๋Š” ใ€Œmiscใ€, ใ€Œresearchใ€์˜ 2๊ฐœ ํ•˜์œ„ ํด๋”๋กœ ๊ตฌ์„ฑ๋˜์–ด โ€˜(๋ช…์นญ 5 ์ƒ๋žต)์—ฐ๊ตฌ์†Œโ€™์˜ ์†Œ์žฅ ๊ณต์†Œ์™ธ 22๊ฐ€ ์ž‘์„ฑํ•œ โ€˜(์„œ์ ๋ช… 1 ์ƒ๋žต)โ€™, โ€˜(์„œ์ ๋ช… 2 ์ƒ๋žต)โ€™ ๋“ฑ ๋ถํ•œ ํ•ต๋ณด์œ ๋ฅผ ์˜นํ˜ธํ•˜๊ณ  ๋ถํ•œ ์‚ฌํšŒ์ฃผ์˜์ฒด์ œ์™€ ๋ถํ•œ์˜ ์—ฐ๋ฐฉ์ œ ํ†ต์ผ๋ฐฉ์•ˆ์„ ์ง€์ง€ํ•˜๋Š” ์ž๋ฃŒ๋“ค์ด ์ €์žฅ๋˜์–ด ์žˆ๋‹ค. ์œ„ ์ €์žฅ๋งค์ฒด์— ์ˆ˜๋ก๋œ ํŒŒ์ผ๋“ค์˜ ์ฃผ์š” ๋‚ด์šฉ์€ ๋‹ค์Œ๊ณผ ๊ฐ™๋‹ค. ใ€Š์ •์„ธ๋ถ„์„์ž๋ฃŒ ํด๋” ์ค‘ ๋…ธ์„ ๊ณผ์ •์ฑ… ํด๋” ๋‚ด โ€˜๋…ธ์„ ๊ณผ ์ •์ฑ… 1ํ˜ธโ€™์˜ ์ฃผ์š” ๋‚ด์šฉใ€‹ - ๋กœ๋™์‹ ๋ฌธ, ๊ตฌ๊ตญ์ „์„ , ์กฐ์„ ์‹ ๋ณด, ๋ฐ˜์ œ๋ฏผ์ „ ๋“ฑ์˜ ๋ถํ•œ์–ธ๋ก ๋งค์ฒด ๋ณด๋„๋ฌธํ—Œ ๋ฐ ๊น€์ •์ผ ๋…ธ์ž‘ ์ผ๋ถ€๋ฅผ ๋ฐœ์ทŒํ•˜์—ฌ ์ฃผ์ œ๋ณ„(์„ ๊ตฐ ์žฅ์ •์˜ ๊ธธ, ๋…ธ์ž‘, ์ •์„ธ)๋กœ ํŽธ์ง‘ํ•œ ๊ฐ„ํ–‰๋ฌผ๋กœ ์„ ๊ตฐ์ •์น˜ ๋ฏธํ™”ํ•˜๋ฉฐ ๊น€์ผ์„ฑยท๊น€์ •์ผ์„ ์ฐฌ์–‘ํ•˜๊ณ , ๋ฐ˜๋ฏธยท๋ฐ˜์ •๋ถ€ ํˆฌ์Ÿ์„ ์„ ์ „ยท์„ ๋™ํ•˜๋Š” ๋‚ด์šฉ์ด๋‹ค. ใ€Š์ดˆ๋ฒŒ์ž๋ฃŒ ํด๋” ์ค‘ ์œ„๋Œ€์„ฑ์ž๋ฃŒ ํด๋” ๋‚ด โ€˜๊น€์ •์ผ ์žฅ๊ตฐ๋‹˜์€ ์–ด๋–ค ๋ถ„์ด์‹ ๊ฐ€โ€™์˜ ์ฃผ์š” ๋‚ด์šฉใ€‹ - ๊น€์ •์ผ์˜ ์˜๋„์ž๋กœ์„œ์˜ ์˜์ˆ˜์ˆญ๋ฐฐ์™€ ์ •์น˜๋ น๋ฅœ, ์—ด์˜์ธ์œผ๋กœ์„œ์˜ ์›…๋น„, ์ •์˜์ธ์œผ๋กœ์„œ์˜ ์„ฑ์ •, ์œ„์ธ์œผ๋กœ์„œ์˜ ์ทจํ–ฅ, ์ฐธ์ธ๊ฐ„์œผ๋กœ์„œ์˜ ์ธ์ƒํ‘œ์ฒ™, ๋ฏผ์ค‘์ •์น˜๊ฐ€๋กœ์„œ์˜ ์ขŒ์šฐ๋ช… ๋“ฑ์œผ๋กœ ๋‚˜๋ˆ„์–ด ๊ตฌ์„ฑ๋œ ๋ฌธ๊ฑด์œผ๋กœ ๊น€์ •์ผ์˜ ์—…์  ๋“ฑ์„ ์„ ์ „, ์šฐ์ƒํ™”ํ•˜๋Š” ๋‚ด์šฉ์ด๋‹ค. ใ€Š์ดˆ๋ฒŒ์ž๋ฃŒ ํด๋” ์ค‘ ์œ„๋Œ€์„ฑ์ž๋ฃŒ ํด๋” ๋‚ด โ€˜์ ˆ์„ธ์˜ ์• ๊ตญ์ž ๊น€์ •์ผ ์žฅ๊ตฐโ€™ ์ œํ•˜์˜ ์ฃผ์š” ๋‚ด์šฉใ€‹ - ๊น€์ •์ผ์˜ ์ฒœ์žฌ์ ์ธ ์˜ˆ์ง€์™€ ๋น„๋ฒ”ํ•œ ์‹ค๋ ฅ, ๋ฌดํ•œ๋Œ€ํ•œ ์• ๊ตญ์˜ ์—ด์ •์œผ๋กœ ์ด๋ฃฉํ•œ ์• ๊ตญ์—…์ ์€ ์—ญ์‚ฌ์— ์ „๋ฌดํ›„๋ฌดํ•œ ๊ฒƒ์ด๋ผ๋Š” ๋“ฑ ๊น€์ •์ผ์„ ์ ˆ์„ธ์˜ ์• ๊ตญ์ž๋กœ ์นญํ•˜๋ฉฐ, ๋ฏผ์กฑ์˜ ์œ„์ธ์œผ๋กœ์„œ ์šฐ์ƒํ™”ํ•˜์—ฌ ์ฐฌ์–‘ยท๋ฏธํ™”ํ•˜๋Š” ๋‚ด์šฉ์ด๋‹ค. ใ€Š์ดˆ๋ฒŒ์ž๋ฃŒ ํด๋” ์ค‘ ์œ„๋Œ€์„ฑ์ž๋ฃŒ ํด๋” ๋‚ด โ€˜์œ„์ธ์˜ ๋งค๋ ฅโ€™์˜ ์ฃผ์š” ๋‚ด์šฉใ€‹ - ๊น€์ •์ผ์˜ ํ•œ์—†์ด ์•„๋ฆ„๋‹ต๊ณ  ์ˆญ๊ณ ํ•œ ์ธ๊ฐ„์ ์ธ ๋งค๋ ฅ์„ ์†Œ๊ฐœํ•˜๋ฉด์„œ ๊น€์ •์ผ์˜ ํ˜๋ช…๊ฐ€์  ์ด๋ ฅ๊ณผ ์‚ฌ์ƒยท์ •์‹ ์  ๋งค๋ ฅ, ์„ฑํ’ˆ๊ณผ ํ™œ๋™๋ฐฉ์‹ ๋“ฑ์„ ๋ฏธํ™”ํ•˜๋ฉฐ ๊น€์ •์ผ์„ ์šฐ์ƒํ™”ํ•˜์—ฌ ์ฐฌ์–‘ํ•˜๋Š” ๋‚ด์šฉ์ด๋‹ค. ใ€Š์ดˆ๋ฒŒ์ž๋ฃŒ ํด๋” ์ค‘ ์„ ๊ตฐ์ •์น˜์ž๋ฃŒ ํด๋” ๋‚ด โ€˜๋ฏผ์กฑ๊ณผ ์„ ๊ตฐ์ •์น˜โ€™์˜ ์ฃผ์š” ๋‚ด์šฉใ€‹ - ์„ ๊ตฐ์ •์น˜์˜ ์ •์˜์™€ ๋‹น์œ„์„ฑ์„ ์—ญ์„คํ•˜๋ฉฐ ๋ฏผ์กฑ์˜ ์ž์ฃผ์„ฑ๊ณผ ํ†ต์ผ์กฐ๊ตญ๊ฑด์„ค ๋ฐ”ํƒ•์˜ ์ด๋…์ด ๋œ๋‹ค๊ณ  ์ฃผ์žฅํ•˜๋ฉด์„œ ์ฃผ์ฒด์‚ฌ์ƒ ๋ฐ ์„ ๊ตฐ์ •์น˜๋ฅผ ์„ ์ „ยท๋ฏธํ™”ํ•˜๋Š” ๊ฒƒ์ด ๋ฌธ๊ฑด์˜ ์ฃผ์š” ๋‚ด์šฉ์ด๋‹ค. ใ€Š์ดˆ๋ฒŒ์ž๋ฃŒ ํด๋” ์ค‘ ์„ ๊ตฐ์ •์น˜์ž๋ฃŒ ํด๋” ๋‚ด โ€˜๊น€์ •์ผ์žฅ๊ตฐ์˜ ํ†ต์ผ์ „๋žตโ€™์˜ ์ฃผ์š” ๋‚ด์šฉใ€‹ - ๊น€์ •์ผ์˜ ํ†ต์ผ์ „๋žต๊ณผ ๊ตฐ์‚ฌ์ „๋žต์„ ์„ค๋ช…ํ•˜๊ณ , ๋ถ์กฐ์„ ์ด ๋ฏธ๋ณธํ† ์— ๋Œ€ํ•œ ๋Œ€๋Ÿ‰๋ณด๋ณต๊ณต๊ฒฉ๋Šฅ๋ ฅ์„ ๋ณด์œ ํ•˜๊ฒŒ ๋œ ๋ฐฐ๊ฒฝ์„ ๊น€์ •์ผ์˜ ์กฐ์„ ํ†ต์ผ์ „๋žต๊ด€์ ์—์„œ ์„ ๊ตฐ์ •์น˜ ๋ฐ ๊น€์ •์ผ์˜ ํ–‰์ ์„ ๋ฏธํ™”ยท์ฐฌ์–‘ํ•˜๋Š” ๋“ฑ ๋ถํ•œ์˜ ๋Œ€๋‚จํ†ต์ผ๋…ธ์„  ๋“ฑ์„ ์„ ์ „ยท์„ ๋™ํ•˜๋Š” ๋‚ด์šฉ์˜ ๋ฌธ๊ฑด์ด๋‹ค. ใ€Š์ดˆ๋ฒŒ์ž๋ฃŒ ํด๋” ์ค‘ ์„ ๊ตฐ์ •์น˜์ž๋ฃŒ ํด๋” ๋‚ด โ€˜๊น€์ •์ผ์žฅ๊ตฐ ์„ ๊ตฐ์ •์น˜๋ฆฌ๋ก โ€™์˜ ์ฃผ์š” ๋‚ด์šฉใ€‹ - ๊น€์ •์ผ์ด ๋ฐํžŒ ์„ ๊ตฐ์ •์น˜์— ๊ด€ํ•œ ์‚ฌ์ƒ๋ฆฌ๋ก ๋“ค์„ ์ฒด๊ณ„ํ™”ํ•˜์—ฌ ์„ค๋ช…ํ•˜๋ฉด์„œ ๊น€์ •์ผ์ด ์ฃผ๋„ํ•˜๋Š” ์„ ๊ตฐ์ •์น˜์˜ ์ •๋‹น์„ฑ ๋“ฑ์„ ์„ ์ „ํ•˜๋Š” ๋“ฑ ๊น€์ •์ผ ๊ฐœ์ธ์„ ์šฐ์ƒํ™”ํ•˜์—ฌ ์ฐฌ์–‘ํ•˜๋Š” ๋‚ด์šฉ์˜ ๋ฌธ๊ฑด์ด๋‹ค. ใ€Š์ดˆ๋ฒŒ์ž๋ฃŒ ํด๋” ์ค‘ ์„ ๊ตฐ์ •์น˜์ž๋ฃŒ ํด๋” ๋‚ด โ€˜์„ ๊ตฐ์ •์น˜: ์ฃผ์ฒด์‚ฌํšŒ์ฃผ์˜ ์ƒ๋ช…์„ โ€™์˜ ์ฃผ์š” ๋‚ด์šฉใ€‹ - ์„ ๊ตฐ์ •์น˜๋ฅผ ํ˜๋ช…์˜ ์ฃผ์ฒด๋ฅผ ํŠผํŠผํžˆ ๋‹ค์ง€๋Š” ํ˜๋ช…์ ์ธ ์ •์น˜๋ฐฉ์‹, ๊ฐ•์„ฑ๋Œ€๊ตญ๊ฑด์„ค์„ ํž˜ ์žˆ๊ฒŒ ๋‹ค๊ทธ์ณ ๋‚˜๊ฐ€๋Š” ์‚ฌํšŒ์ฃผ์˜ ์ •์น˜๋ฐฉ์‹, ์‚ฌํšŒ์ฃผ์˜ ์กฐ์„ ์˜ ์กด์—„์„ ๋งŒ๋ฐฉ์— ๋–จ์น˜๊ฒŒ ํ•˜๋Š” ์œ„๋ ฅํ•œ ์ •์น˜๋ฐฉ์‹, ์ฃผ์ฒด์‚ฌํšŒ์ฃผ์˜ ์œ„์—… ์™„์„ฑ์„ ์œ„ํ•˜์—ฌ ํ•ญ๊ตฌ์ ์œผ๋กœ ํ‹€์–ด์ฅ๊ณ  ๋‚˜๊ฐ€์•ผ ํ•  ์ „๋žต์  ์ •์น˜๋ฐฉ์‹ ๋“ฑ์œผ๋กœ ๋ฌ˜์‚ฌํ•˜๋ฉด์„œ ๊น€์ •์ผ์˜ ์ง€๋„๋ ฅ๊ณผ ์„ ๊ตฐ์ •์น˜์˜ ์ •๋‹น์„ฑ์„ ์„ ์ „ยท์ฐฌ์–‘ํ•˜๋Š” ๋‚ด์šฉ์ด๋‹ค. ใ€Š์ดˆ๋ฒŒ์ž๋ฃŒ ํด๋” ์ค‘ ์„ ๊ตฐ์ •์น˜์ž๋ฃŒ ํด๋” ๋‚ด โ€˜์œ„๋Œ€ํ•œ ์˜๋„์ž ๊น€์ •์ผ๋™์ง€๊ป˜์„œ ๋ฐํžˆ์‹  ์„ ๊ตฐํ˜๋ช… ๋ น๋„์— ๊ด€ํ•œ ๋…์ฐฝ์  ์‚ฌ์ƒโ€™์˜ ์ฃผ์š” ๋‚ด์šฉใ€‹ - ์„ ๊ตฐํ˜๋ช… ๋ น๋„์‚ฌ์ƒ์€ ๊น€์ผ์„ฑ์ด ํ•ญ์ผํ˜๋ช… ํˆฌ์Ÿ๊ณผ์ •์—์„œ ์ด๋ฃฌ ์„ ๊ตฐ์‚ฌ์ƒ์„ ๊น€์ •์ผ์ด ์˜ค๋Š˜์˜ ํ˜„์‹ค์  ์กฐ๊ฑด์— ๋งž๊ฒŒ ์™„์„ฑํ•œ ์ •์น˜๋ฐฉ์‹์ด๋ผ๊ณ  ๋ฌ˜์‚ฌํ•˜๋ฉด์„œ ๊น€์ผ์„ฑยท๊น€์ •์ผ์„ ์ฐฌ์–‘ยท๋ฏธํ™”ํ•˜๊ณ  ์„ ๊ตฐ์ •์น˜์˜ ์ •๋‹น์„ฑ ๋“ฑ์„ ์„ ์ „ํ•˜๋Š” ๋‚ด์šฉ์ด๋‹ค. ใ€Š์ดˆ๋ฒŒ์ž๋ฃŒ ํด๋” ์ค‘ ์„ ๊ตฐ์ •์น˜์ž๋ฃŒ ํด๋” ๋‚ด โ€˜์„ธ์ƒ์—์„œ ์ œ์ผ ๊ฐ•ํ•˜์‹  ๋ถ„โ€™์˜ ์ฃผ์š” ๋‚ด์šฉใ€‹ - ๊น€์ •์ผ์„ ๋ฏผ์กฑ์˜ ํƒœ์–‘์œผ๋กœ ๋ฌ˜์‚ฌํ•˜๋ฉด์„œ ๋น„๋ฒ”ํ•œ ๊ด€์ฐฐ๋ ฅยทํ†ต์ฐฐ๋ ฅยท๋ถ„์„๋ ฅยท์ง๊ด€๋ ฅ๊ณผ ์ง€๋žตยท์šฉ๊ธฐยท๋‹ด๋ ฅ ๋“ฑ์„ ๊ฐ–์ถ˜ ์œ„๋Œ€ํ•œ ์˜๋„์ž๋ผ๊ณ  ์šฐ์ƒํ™” ์‹œ์ผœ ๋ฏธํ™”ยท์ฐฌ์–‘ํ•˜๋Š” ๋‚ด์šฉ์ด๋‹ค. ใ€Š์ดˆ๋ฒŒ์ž๋ฃŒ ํด๋” ์ค‘ ํ†ต์ผ(๋ถํ•œ)๊ด€๋ จ์ž๋ฃŒ ํด๋” ๋‚ด โ€˜ํ•ญ์ผ ๋นจ์ฐŒ์‚ฐ ์ฐธ๊ฐ€์ž๋“ค์˜ ํšŒ์ƒ๊ธฐ 4โ€™์˜ ์ฃผ์š” ๋‚ด์šฉใ€‹ - ์†Œ์œ„ ํ•ญ์ผ ๋นจ์ฐŒ์‚ฐ ์ถœ์‹ ์ž๋“ค์˜ ๊ณผ๊ฑฐ ํšŒ์ƒ ๋‚ด์šฉ์„ ์‹œ๋ฆฌ์ฆˆ๋ฌผ๋กœ ์—ฎ์€ ๊ฒƒ์œผ๋กœ ๊น€์ผ์„ฑ์˜ ํ•ญ์ผ ๋นจ์ฐŒ์‚ฐ์€ ์ ๊ณผ ์‹ธ์šฐ๋Š” ์ „์‚ฌ๋ฟ๋งŒ ์•„๋‹ˆ๋ผ ์ธ๋ฏผ๋“ค์„ ๊ณต์‚ฐ์ฃผ์˜์‚ฌ์ƒ์œผ๋กœ ๊ต์–‘ยท์„ ๋™ํ•˜๋Š” ์„ ์ „์ž๊ฐ€ ๋˜์–ด์•ผ ํ•˜๋ฉฐ, ํ•ญ์ผ ๋นจ์ฐŒ์‚ฐ ์—ญ๋Ÿ‰์„ ๊ฐ•ํ™”ํ•˜์—ฌ ์กฐ์„ ์ธ๋ฏผํ˜๋ช…๊ตฐ์œผ๋กœ ๋ฐœ์ „์‹œ์ผœ, ๊ถ๊ทน์ ์œผ๋กœ ํ•œ๋ฐ˜๋„ ์ „์ฒด๋ฅผ ๊ณต์‚ฐํ™”์‹œ์ผœ์•ผ ํ•œ๋‹ค๊ณ  ์„ ์ „ยท์„ ๋™ํ•˜๋Š” ๋‚ด์šฉ์ด๋‹ค. ใ€Š์ดˆ๋ฒŒ์ž๋ฃŒ ํด๋” ์ค‘ ํ†ต์ผ(๋ถํ•œ)๊ด€๋ จ์ž๋ฃŒ ํด๋” ๋‚ด โ€˜์„ธ๊ธฐ์™€ ๋”๋ถˆ์–ด 1โ€™ ๋ฐ โ€˜์„ธ๊ธฐ์™€ ๋”๋ถˆ์–ด ๊ฐ•์ขŒ๋ณธ 1โ€™์˜ ์ฃผ์š” ๋‚ด์šฉใ€‹ - โ€˜์„ธ๊ธฐ์™€ ๋”๋ถˆ์–ดโ€™ ์ œ1๊ถŒ์€ ๊น€์ผ์„ฑ์˜ ์œ ์†Œ๋…„๊ธฐ(1912. 4.~1930. 5.)๋ฅผ ๋‹ค๋ฃฌ ๊ฒƒ์œผ๋กœ, ๊น€์ผ์„ฑ์˜ ๊ฐ€๋ฌธ๊ณผ ์ถœ์ƒ๊ณผ์ •, โ€˜ํƒ€๋„์ œ๊ตญ์ฃผ์˜๋™๋งนโ€™ ๊ฒฐ์„ฑ ๊ณผ์ • ๋ฐ ๊ธธ๋ฆผ์—์„œ์˜ ์ฒญ๋…„์‹œ์ ˆ ๋“ฑ ๊น€์ผ์„ฑ์ด ์ฃผ์žฅํ•˜๋Š” ํ•ญ์ผํˆฌ์Ÿ ํ™œ๋™ ๋“ฑ์„ ๋ฏธํ™”ํ•œ ๊ฒƒ์ด ์ฃผ๋œ ๋‚ด์šฉ์ด๊ณ , โ€˜์„ธ๊ธฐ์™€ ๋”๋ถˆ์–ด ๊ฐ•์ขŒ๋ณธ 1โ€™์€ ์œ„ โ€˜์„ธ๊ธฐ์™€ ๋”๋ถˆ์–ดโ€™ 1๊ถŒ ์ค‘ ์ œ1ยท2์žฅ๋งŒ ๋ฐœ์ทŒยทํŽธ์ง‘ํ•œ ๊ฐ•์ขŒ๋ณธ์ด๋‹ค. ์ด์™€ ๊ฐ™์€ ํŒŒ์ผ๋“ค์ด ์ €์žฅ๋œ ํ•˜๋“œ๋””์Šคํฌ(HDD 40G) ์ž๋ฃŒ์ง‘์€ ๊น€์ผ์„ฑ์˜ ์ „ ๊ฐ€๊ณ„๋ฅผ ํ•ญ์ผ ๋…๋ฆฝํˆฌ์‚ฌ๋กœ ๋ฌ˜์‚ฌํ•˜๊ณ  ๊น€์ผ์„ฑ์˜ ํ•ญ์ผ๋ฌด์žฅํˆฌ์Ÿ์„ ๋ฏธํ™”ยท์ฐฌ์–‘ํ•˜๋Š” ๋“ฑ ๊น€์ผ์„ฑ์„ ์šฐ์ƒํ™”ํ•˜๋Š” ์ทจ์ง€์˜ ๋‚ด์šฉ, ์กฐ์„ ๋กœ๋™๋‹น์˜ ์ผ๋‹น๋…์žฌ๋ฅผ ์ •๋‹นํ™”ํ•˜๋ฉด์„œ ๋Œ€๋‚จํ˜๋ช…๋…ธ์„ ์„ ์„ ์ „ยท์„ ๋™ํ•˜๋Š” ์ทจ์ง€์˜ ๋‚ด์šฉ, ๋ถํ•œ์‹ ์‚ฌํšŒ์ฃผ์˜๊ฐ€ ์ฃผ์ฒด์‚ฌ์ƒ์œผ๋กœ ๋ฌด์žฅํ•œ ๋ถํ•œ ์ธ๋ฏผ๋“ค์— ์˜ํ•ด ์ ˆ๋Œ€์  ์ง€์ง€๋ฅผ ๋ฐ›๊ณ  ์žˆ๋‹ค๋Š” ์ทจ์ง€๋กœ ๋ถํ•œ ์‚ฌํšŒ์ฃผ์˜์ฒด์ œ์˜ ์šฐ์›”์„ฑ ๋ฐ ํ˜๋ช…์  ์ •๋‹น์„ฑ ๋“ฑ์„ ๊ฐ•์กฐํ•˜๋Š” ์ทจ์ง€์˜ ๋‚ด์šฉ, ๊น€์ผ์„ฑ ์ฃผ์ฒด์‚ฌ์ƒ์˜ ์šฐ์›”์„ฑ์„ ๊ฐ•์กฐํ•˜๋ฉด์„œ ๋ฏธ ์ œ๊ตญ์ฃผ์˜์— ์˜ˆ์†๋œ ๋‚จํ•œ ๋ฏผ์ค‘์„ ํ•ด๋ฐฉํ•˜๊ณ  ํ•œ๋ฐ˜๋„์˜ ํ†ต์ผ์„ ์‹คํ˜„ํ•˜๊ธฐ ์œ„ํ•ด ๋‚จํ•œ ๋ฏผ์ค‘์„ ์ฃผ์ฒด์‚ฌ์ƒ์œผ๋กœ ์˜์‹ํ™”ํ•˜์—ฌ์•ผ ํ•œ๋‹ค๋Š” ์ทจ์ง€์˜ ๋‚ด์šฉ, ์ž๋ณธ์ฃผ์˜๊ฒฝ์ œ์ œ๋„๋Š” ์ธ๋ฏผ๋Œ€์ค‘์„ ์ˆ˜ํƒˆํ•˜๊ธฐ ์œ„ํ•œ ๋„๊ตฌ์— ๋ถˆ๊ณผํ•˜๊ณ  ์ฃผ์ฒด์‚ฌ์ƒ์ด ๊ทผ๊ฐ„์ด ๋œ ๋ถํ•œ์˜ ์‚ฌํšŒ์ฃผ์˜ ์ฒด์ œ๋Š” ๋ชจ๋“  ๊ตฌ์„ฑ์›๋“ค์ด ํ‰๋“ฑํ•˜๊ณ  ํ’์กฑํ•œ ๋ฌผ์งˆ์  ๋ฌธํ™”์ƒํ™œ์ด ๋ณด์žฅ๋˜๋Š” ์‚ฌํšŒ๋ผ๊ณ  ๋ฏธํ™”ํ•˜๋ฉด์„œ ๋ถํ•œ ์‚ฌํšŒ์ฃผ์˜ ์ฒด์ œ์™€ ์ฃผ์ฒด์‚ฌ์ƒ์„ ์ฐฌ์–‘ยท์„ ์ „ํ•˜๋Š” ์ทจ์ง€์˜ ๋‚ด์šฉ, ๊น€์ผ์„ฑยท๊น€์ •์ผ์ฃผ์˜๋ฅผ ์ถ”์ข…ํ•˜๋Š” ๋ถํ•œ ์‚ฌํšŒ์ฃผ์˜์ฒด์ œ์˜ ์šฐ์ˆ˜์„ฑ ๋ฐ ์ฃผ์ฒด์‚ฌ์ƒ๊ณผ ์„ ๊ตฐ์ •์น˜๋ฅผ ์„ ์ „ํ•˜๋Š” ์ทจ์ง€์˜ ๋‚ด์šฉ, ๋ถํ•œ์˜ ๋Œ€๋‚จํ˜๋ช…๋ก ์˜ ์ผํ™˜์œผ๋กœ ์ฃผํ•œ๋ฏธ๊ตฐ์˜ ์ฒ ์ˆ˜๋ฅผ ์ฃผ์žฅํ•˜๊ณ  ๋ถํ•œ์˜ ์—ฐ๋ฐฉ์ œ ํ†ต์ผ๋ฐฉ์‹์˜ ์ •๋‹น์„ฑ ๋“ฑ์„ ์„ ์ „ยท์„ ๋™ํ•˜๋Š” ์ทจ์ง€์˜ ๋‚ด์šฉ, ๊น€๋ถ€์ž์— ๋Œ€ํ•œ ์˜์›ํ•œ ์ถฉ์„ฑ์„ ๋งน์„ธํ•˜๋Š” ์ทจ์ง€์˜ ๋‚ด์šฉ ๋“ฑ์— ๊ด€ํ•œ ๊ฐ์ข… ๋ฌธ๊ฑดํŒŒ์ผ ๋“ฑ์„ ์ˆ˜๋กํ•˜๊ณ  ์žˆ๋‹ค. ์ด๋กœ์จ ํ”ผ๊ณ ์ธ์€ ๊ตญ๊ฐ€์˜ ์กด๋ฆฝยท์•ˆ์ „์ด๋‚˜ ์ž์œ ๋ฏผ์ฃผ์  ๊ธฐ๋ณธ์งˆ์„œ๋ฅผ ์œ„ํƒœ๋กญ๊ฒŒ ํ•œ๋‹ค๋Š” ์ •์„ ์•Œ๋ฉด์„œ ๋ฐ˜๊ตญ๊ฐ€๋‹จ์ฒด๋‚˜ ๊ทธ ๊ตฌ์„ฑ์› ๋˜๋Š” ๊ทธ ์ง€๋ น์„ ๋ฐ›์€ ์ž์˜ ํ™œ๋™์„ ์ฐฌ์–‘ยท๊ณ ๋ฌดยท์„ ์ „ ๋˜๋Š” ์ด์— ๋™์กฐํ•  ๋ชฉ์ ์œผ๋กœ ์œ„ ์ด์ ํ‘œํ˜„๋ฌผ์„ ์†Œ์ง€ํ•˜์˜€๋‹ค. ๋งˆ. ์™ธ์žฅ ํ•˜๋“œ๋””์Šคํฌ(HD 500G) ์†Œ์ง€ ํ”ผ๊ณ ์ธ์€ 2015. 11. 13.๊ฒฝ ๊ด‘๋ช…์‹œ (์ฃผ์†Œ 3 ์ƒ๋žต)์— ์žˆ๋Š” ํ”ผ๊ณ ์ธ์˜ ์ฃผ๊ฑฐ์ง€ ์ง€ํ•˜ ์ฃผ์ฐจ์žฅ์— ์„ธ์›Œ ๋‘” ํ”ผ๊ณ ์ธ ์†Œ์œ ์˜ ์˜คํ† ๋ฐ”์ด ์•ˆ์žฅ ๋ฐ‘ ๋ณด๊ด€ํ•จ์— ๊น€๋ถ€์ž ๋ฐ ๋ถํ•œ์ฒด์ œ๋ฅผ ์ฐฌ์–‘ยท๋ฏธํ™”ํ•˜๊ฑฐ๋‚˜, ์ฃผ์ฒด์‚ฌ์ƒ์— ์ž…๊ฐํ•œ ๋Œ€๋‚จํ˜๋ช…ํˆฌ์Ÿ์„ ์„ ์ „ยท๋™์กฐํ•˜๋Š” ๋“ฑ์˜ ๊ฐ์ข… ํŒŒ์ผ๋“ค์„ ์ฒด๊ณ„์ ์œผ๋กœ ์ •๋ฆฌยท์ €์žฅํ•œ ์™ธ์žฅ ํ•˜๋“œ๋””์Šคํฌ(HD 500G) ํ˜•์‹์˜ ๋””์ง€ํ„ธ์ €์žฅ๋งค์ฒด๋ฅผ ๋ณด๊ด€ํ•˜๊ณ  ์žˆ์—ˆ๋‹ค. ์œ„ ์ €์žฅ๋งค์ฒด๋Š” ใ€Œ1 ํ•™์Šตใ€ยทใ€Œ2 ์ž๋ฃŒ๋ชจ์Œใ€ยทใ€Œ3 ์˜ํ™”ใ€ยทใ€Œ[mp3]ใ€ ๋“ฑ 4๊ฐœ์˜ ์ƒ์œ„ ํด๋”๋กœ ๊ตฌ์„ฑํ•˜์—ฌ ํŒŒ์ผ๋“ค์ด ๋ณด๊ด€๋˜์–ด ์žˆ๋‹ค. ์œ„ ์ƒ์œ„ํด๋” ์ค‘ ใ€Œ1 ํ•™์Šตใ€ ํด๋”๋Š” ใ€Œ10๋‹จ๊ณ„ ํ•™์Šตใ€ยทใ€Œ5๋‹จ๊ณ„ํ•™์Šต๊ณผ์ •ใ€ยทใ€Œ๊ต์–‘ใ€ ๋“ฑ 3๊ฐœ์˜ ํ•˜์œ„ ํด๋”๋กœ ๊ตฌ์„ฑ๋˜์–ด ์žˆ๋Š”๋ฐ, ํ•˜์œ„ ํด๋” ์ค‘ ใ€Œ10๋‹จ๊ณ„ ํ•™์Šตใ€ ํด๋”๋Š” ๋‹จ๊ณ„๋ณ„๋กœ ใ€Œ1๋‹จ๊ณ„-ํ˜๋ช…๊ด€ํšŒ๊ณ ๋กใ€ยทใ€Œ2๋‹จ๊ณ„-๋ณ€ํ˜์ด๋ก  4๋Œ€ ๊ฐœ๋ก ใ€ยทใ€Œ3๋‹จ๊ณ„-์„ ๊ตฐ์‚ฌ์ƒใ€ยทใ€Œ4๋‹จ๊ณ„-๊น€๋Œ€ํ•™๋ฐฉ์†ก๊ฐ•์ขŒใ€ยทใ€Œ5๋‹จ๊ณ„-๊น€์ผ์„ฑ์ฃผ์˜ ์ด์„œใ€ยทใ€Œ6๋‹จ๊ณ„-์œ„๋Œ€์„ฑใ€ยทใ€Œ7๋‹จ๊ณ„-์ฒ ํ•™์ •์น˜๊ฒฝ์ œ์—ญ์‚ฌใ€ยทใ€Œ8๋‹จ๊ณ„-์ฃผ์š”๋…ธ์ž‘๋ชจ์Œใ€ยทใ€Œ9๋‹จ๊ณ„-์ผ๊ตฐ๊ต์–‘1ใ€ยทใ€Œ10๋‹จ๊ณ„-์ผ๊ตฐ๊ต์–‘2ใ€๋“ฑ 12๊ฐœ์˜ ์„ธ๋ถ€ ํด๋”๋กœ ๋‚˜๋ˆ„์–ด์ ธ โ€˜ํ•œ๊ตญ์‚ฌํšŒ ์„ฑ๊ฒฉ๋…ผ์˜์˜ ์žฌ์กฐ๋ช…โ€™ ๋“ฑ ์ฃผ์ฒด์‚ฌ์ƒ์— ์ž…๊ฐํ•œ ๋Œ€๋‚จํ˜๋ช…ํˆฌ์Ÿ์„ ์„ ๋™ํ•˜๋Š” ์ž๋ฃŒ๋“ค์ด ์ €์žฅ๋˜์–ด ์žˆ๊ณ , ใ€Œ5๋‹จ๊ณ„ํ•™์Šต๊ณผ์ •ใ€ ํ•˜์œ„ ํด๋”๋Š” ๋‹จ๊ณ„๋ณ„๋กœ ใ€Œ1๋‹จ๊ณ„-๋ณ€ํ˜๊ด€ ํšŒ๊ณ ๋กใ€ยทใ€Œ2๋‹จ๊ณ„-๋ณ€ํ˜์ด๋ก ์กฐ์ง์‚ฌ์—…ใ€ยทใ€Œ3๋‹จ๊ณ„-์ฒ ํ•™์ •์น˜ ๊ฒฝ์ œ๋ณ€ํ˜์‚ฌใ€ยทใ€Œ4๋‹จ๊ณ„-์„ ๊ตฐ์‚ฌ์ƒใ€ ใ€Œ5๋‹จ๊ณ„-์œ„๋Œ€์„ฑใ€ ๋“ฑ์˜ 7๊ฐœ ์„ธ๋ถ€ ํด๋”๋กœ ๊ตฌ์„ฑ๋˜์–ด โ€˜์• ๊ตญ์‹œ๋Œ€โ€™ ๋“ฑ ๋ถํ•œ ๋Œ€๋‚จํ˜๋ช…๋ก ์— ๋™์กฐยท์„ ์ „ํ•˜๋Š” ์ž๋ฃŒ๋“ค์ด ์ €์žฅ๋˜์–ด ์žˆ์œผ๋ฉฐ, ใ€Œ๊ต์–‘ใ€ ํ•˜์œ„ ํด๋”๋Š” ใ€Œ14์ž๋ฃŒใ€ยทใ€Œ๊ฐ„๋ถ€ใ€ยทใ€Œ๊น€์ผ์„ฑ์ฃผ์˜ ์ด์„œใ€ยทใ€Œ๋Œ€์ค‘ ์†Œ๋ชจ์ž„ ๊ต์–‘ใ€ยทใ€Œ์„ ๊ตฐ์‚ฌ์ƒ๊ด€๋ จ์ž๋ฃŒใ€ ๋“ฑ 33๊ฐœ์˜ ์„ธ๋ถ€ ํด๋”๋กœ ๊ตฌ์„ฑ๋˜์–ด โ€˜์กฐ์„ ๋…ธ๋™๋‹น ์ค‘์•™์œ„ ์ •์น˜๊ตญ ํ™•๋Œ€ํšŒ์˜์— ๊ด€ํ•œ ๋ณด๋„(2013. 12. 9.์ž ์กฐ์„ ์ค‘์•™ํ†ต์‹ )โ€™, โ€˜๊ฒฝ์• ํ•˜๋Š” ๊น€์ •์€ ๋™์ง€์˜ ๋‘๋ฆฌ์— ์ฒ ํ†ต ๊ฐ™์ด ๋ญ‰์ณ ์ฃผ์ฒดํ˜๋ช…์˜ ํ•œ๊ธธ๋กœ ์–ต์„ธ๊ฒŒ ๋‚˜์•„๊ฐ€์žโ€™(2013. 12. 10.์ž ๋…ธ๋™์‹ ๋ฌธ), โ€˜์ฒœ๋งŒ๊ตฐ๋ฏผ์˜ ์น˜์†Ÿ๋Š” ๋ถ„๋…ธ์˜ ํญ๋ฐœ, ๋งŒ๊ณ ์—ญ์  ๋‹จํ˜ธํžˆ ์ฒ˜๋‹จโ€™(2013. 12. 13.์ž ๋…ธ๋™์‹ ๋ฌธ), โ€˜์กฐ์„ ๋กœ๋™๋‹น์€ ์ผ์‹ฌ๋‹จ๊ฒฐ์˜ ์œ„๋ ฅ์œผ๋กœ ์ „์ง„ํ•˜๋Š” ๋ถˆํŒจ์˜ ํ˜๋ช…์ ๋‹น์ด๋‹ค.โ€™(2013. 12. 13.์ž ๋…ธ๋™์‹ ๋ฌธ), โ€˜์กฐ์ง๊ด€โ€™, โ€˜์กฐ์ง์˜ ์•ˆ์ „๋ณด์œ„ ๋Œ€์ฑ…์„ ์ฒ ์ €ํžˆ ์„ธ์šธ๋ฐ ๋Œ€ํ•˜์—ฌโ€™, โ€˜๋ฐฑ๋‘์‚ฐ 3๋Œ€ ์žฅ๊ตฐ์˜ ์ˆญ๊ณ ํ•œ ๋‹น์กฐ์ง ๊ด€๋…์„ ๋”ฐ๋ผ ๋ฐฐ์šธ ๋ฐ ๋Œ€ํ•˜์—ฌโ€™, โ€˜๋ฏผ์กฑ๊ณผ ์ฒ ํ•™โ€™ ๋“ฑ ์ฃผ์ฒด์‚ฌ์ƒ์„ ์ฐฌ์–‘ํ•˜๋Š” ์ž๋ฃŒ๋“ค์ด ์ €์žฅ๋˜์–ด ์žˆ๋‹ค. ์œ„ ์ƒ์œ„ ํด๋” ์ค‘ ใ€Œ2 ์ž๋ฃŒ๋ชจ์Œใ€ ํด๋”๋Š” ใ€Œ1 study1ใ€ยทใ€Œ16Gใ€ยทใ€Œ2 study2ใ€ยทใ€Œ3 ์„ธ๊ธฐ์™€ ๋”๋ถˆ์–ดใ€ยทใ€Œ4 ์ธํ„ฐ๋„ท ์ฃผ์ฒด์‚ฌ์ƒ๊ฐ•์ขŒใ€ยทใ€Œ5 ์„ ๊ตฐ์‚ฌ์ƒใ€ยทใ€Œ6 ์ €์ž‘์ง‘ใ€ยทใ€Œ7 ๋ฌธํ™”์˜ˆ์ˆ ใ€ยทใ€Œ8 ์ •์„ธ๋™ํ–ฅใ€ยทใ€Œ9 ์œ„๋Œ€์„ฑใ€ยทใ€ŒBookใ€ ๋“ฑ 20๊ฐœ์˜ ํ•˜์œ„ ํด๋”๋กœ ๊ตฌ์„ฑ๋˜์–ด ์žˆ๋Š”๋ฐ, ใ€Œ1 study1ใ€ ํ•˜์œ„ ํด๋”๋Š” ใ€Œ๊ฐ•์ขŒใ€ยทใ€Œ๊ตฌ๊ตญ์˜์†Œ๋ฆฌใ€ยทใ€Œ๊น€์ผ์„ฑ์ €์ž‘์„ ์ง‘ใ€ยทใ€Œ์ฒ ํ•™ ์ •์น˜๊ฒฝ์ œํ•™ใ€ยทใ€Œํ†ต์ผ์—ฌ๋ช…ใ€ยทใ€ŒํšŒ๊ณ ๋ก๊ฐœ์ •ํŒใ€๋“ฑ 8๊ฐœ์˜ ์„ธ๋ถ€ ํด๋”๋กœ ๋‚˜๋ˆ„์–ด์ ธ โ€˜์ •์น˜์‚ฌ์ „1 ์œ„๋Œ€ํ•œ ์ˆ˜๋ น ๊น€์ผ์„ฑ๋™์ง€~๋ฒจ์ง€๋„โ€™ ๋“ฑ ๊น€์ผ์„ฑยท๊น€์ •์ผ ๋ถ€์ž๋ฅผ ์ฐฌ์–‘ํ•˜๋Š” ์ž๋ฃŒ๋“ค์ด ์ €์žฅ๋˜์–ด ์žˆ๊ณ , ใ€Œ16Gใ€ ํ•˜์œ„ ํด๋”๋Š” ใ€ŒK1์ €์ž‘์ง‘hwpํŒใ€ยทใ€ŒK2์ €์ž‘์ง‘hwpํŒใ€ยทใ€Œ์ €์ž‘์ง‘ใ€ยทใ€ŒํšŒ๊ณ ๋ก์ „๋ฌธ๋‚ญ๋…1-22์žฅ(์ˆœ์„œ์ •๋ฆฌ)ใ€ยทใ€Œ14์ž๋ฃŒใ€๋“ฑ์˜ 12๊ฐœ ์„ธ๋ถ€ ํด๋”๋กœ ๊ตฌ์„ฑ๋˜์–ด โ€˜2013๋…„ ํ•˜๋ฐ˜๊ธฐ 11-12์›” ๊ต์–‘โ€™ , โ€˜์œ„๋Œ€ํ•œ ๊น€์ผ์„ฑ-๊น€์ •์ผ์ฃผ์˜ ๋งŒ์„ธ!โ€™, โ€˜์ „๋‹น๊ณผ ์˜จ ์‚ฌํšŒ๋ฅผ ๊น€์ผ์„ฑ-๊น€์ •์ผ์ฃผ์˜ํ™”ํ•˜์žโ€™, โ€˜์‚ฌํšŒ์ฃผ์˜์— ๋Œ€ํ•œ ์ฃผ์ฒด์  ๋ฆฌํ•ดโ€™ ๋“ฑ ๋ถํ•œ์ฒด์ œ์™€ ์ฃผ์ฒด์‚ฌ์ƒ์„ ์˜นํ˜ธยท์ฐฌ์–‘ํ•˜๋Š” ์ž๋ฃŒ๋“ค์ด ์ €์žฅ๋˜์–ด ์žˆ์œผ๋ฉฐ, ใ€Œ2 study2ใ€ ํ•˜์œ„ ํด๋”๋Š” ใ€Œ๊น€์ผ์„ฑ์ฃผ์˜,์ฃผ์ฒด์ฒ ํ•™ใ€ยทใ€Œ๋‹น์‚ฌ์ƒ์ •์ฑ…ใ€ยทใ€Œ๊น€์‚ฌ์ „ใ€ยทใ€Œ์ฃผ์ฒด์‚ฌ์ƒ์ด์„œใ€ ๋“ฑ 8๊ฐœ์˜ ์„ธ๋ถ€ ํด๋”๋กœ ๊ตฌ์„ฑ๋˜์–ด โ€˜๊น€์ผ์„ฑ์ฃผ์˜ ๋„ํ•ดโ€™ ๋“ฑ ๋ถํ•œ ์ฃผ์ฒด์‚ฌ์ƒ์„ ์ฐฌ์–‘ํ•˜๋Š” ์ž๋ฃŒ๋“ค์ด ์ €์žฅ๋˜์–ด ์žˆ๊ณ , ใ€Œ3 ์„ธ๊ธฐ์™€ ๋”๋ถˆ์–ดใ€ ํ•˜์œ„ ํด๋”๋Š” ใ€Œ1 ํšŒ๊ณ ๋ก๊ฐ•์ขŒใ€ยทใ€Œ2 ์•ฝ๋ ฅ ๋ฐ ์ผํ™”ใ€ยทใ€Œ3 ์‚ฌ์ƒ๊ณผ ์ด๋ก ใ€ ยทใ€Œ4 ํŒ๋ณธ๋“คใ€ยทใ€Œ5 ๋ฌธํ•™์˜ˆ์ˆ ใ€ยทใ€ŒํšŒ๊ณ ๋กCDใ€ ๋“ฑ 9๊ฐœ์˜ ์„ธ๋ถ€ ํด๋”๋กœ ๊ตฌ์„ฑ๋˜์–ด โ€˜์„ธ๊ธฐ์™€ ๋”๋ถˆ์–ด 1โ€™ ๋“ฑ ๊น€์ผ์„ฑ์„ ์ฐฌ์–‘ํ•˜๋Š” ์ž๋ฃŒ๋“ค์ด ์ €์žฅ๋˜์–ด ์žˆ์œผ๋ฉฐ, ใ€Œ4 ์ธํ„ฐ๋„ท์ฃผ์ฒด์‚ฌ์ƒ๊ฐ•์ขŒใ€ ํ•˜์œ„ ํด๋”๋Š” ใ€Œ๊ธฐ๋ณธ๋ฌธํ™”์˜ˆ์ˆ ใ€ยทใ€Œ์‹ฌํ™”ํ•™์Šต ๋ฐ ์ •์„ธ๋ถ„์„ใ€ยทใ€Œ์ธํ„ฐ๋„ท ์ฃผ์ฒด์‚ฌ์ƒ ๊ฐ•์ขŒ ํ•ต์‹ฌ์š”์•ฝํ•™์Šตใ€ ๋“ฑ 10๊ฐœ์˜ ์„ธ๋ถ€ ํด๋”๋กœ ๊ตฌ์„ฑ๋˜์–ด โ€˜์ •ํ†ต๊ณผ ๊ณ„์Šน ์œ„๋Œ€ํ•œ ์ธ๊ฐ„, ์ƒˆ๋กœ์šด ๋ฌธ๋ช…์„ ์œ„ํ•˜์—ฌโ€™ ๋“ฑ ๊น€์ผ์„ฑ ์ผ๊ฐ€๋ฅผ ์ฐฌ์–‘ยท๋ฏธํ™” ๋ฐ ๋Œ€๋‚จํ˜๋ช…ํˆฌ์Ÿ์„ ์„ ๋™ํ•˜๋Š” ์ž๋ฃŒ๋“ค์ด ์ €์žฅ๋˜์–ด ์žˆ๊ณ , ใ€Œ5 ์„ ๊ตฐ์‚ฌ์ƒใ€ ํ•˜์œ„ ํด๋”๋Š” ใ€Œ์„ ๊ตฐ๋ฌธํ™”์˜ˆ์ˆ ใ€ยทใ€Œ์„ ๊ตฐ๋ฏผ์กฑใ€ยทใ€Œ์„ ๊ตฐ์‚ฌ์ƒํ•™์Šต1ใ€ ๋“ฑ 7๊ฐœ์˜ ์„ธ๋ถ€ ํด๋”๋กœ ๊ตฌ์„ฑ๋˜์–ด โ€˜์„ ๊ตฐ์ •์น˜: ์ฃผ์ฒด์‚ฌํšŒ์ฃผ์˜ ์ƒ๋ช…์„ โ€™ ๋“ฑ ๊น€์ •์ผ์˜ ์ง€๋„๋ ฅ๊ณผ ์„ ๊ตฐ์ •์น˜๋ฅผ ์ฐฌ์–‘ํ•˜๋Š” ์ž๋ฃŒ๋“ค์ด ์ €์žฅ๋˜์–ด ์žˆ์œผ๋ฉฐ, ใ€Œ6 ์ €์ž‘์ง‘ใ€ ํ•˜์œ„ ํด๋”๋Š” ใ€Œ๊น€์ผ์„ฑ์ €์ž‘์ง‘ใ€ยทใ€Œ๊น€์ผ์„ฑ์ €์ž‘์ง‘์ฃผ์ œ๋ถ„๋ฅ˜ใ€ยทใ€Œ๊น€์ •์ผ์ €์ž‘์ง‘ใ€ยทใ€Œ๊น€์ •์ผ์ €์ž‘์ง‘์ฃผ์ œ๋ถ„๋ฅ˜ใ€ยทใ€Œ์ €์ž‘์ง‘๋ชจ์Œใ€์˜ 5๊ฐœ ์„ธ๋ถ€ ํด๋”๋กœ ๊ตฌ์„ฑ๋˜์–ด โ€˜๊น€์ผ์„ฑ ์ €์ž‘์ง‘ ์ œ1๊ถŒโ€™ ๋“ฑ ๊น€์ผ์„ฑ์„ ์ฐฌ์–‘ํ•˜๋Š” ์ž๋ฃŒ๋“ค์ด ์ €์žฅ๋˜์–ด ์žˆ๊ณ , ใ€Œ7 ๋ฌธํ™”์˜ˆ์ˆ ใ€ ํ•˜์œ„ ํด๋”๋Š” ใ€Œ์†Œ์„คใ€ยทใ€Œ10๋Œ€ ํ•„์ˆ˜์Œ์•…ใ€ยทใ€Œ์‹œ์ง‘ใ€๋“ฑ 12๊ฐœ์˜ ์„ธ๋ถ€ ํด๋”๋กœ ๊ตฌ์„ฑ๋˜์–ด โ€˜๋ ฅ์‚ฌ์˜ ๋Œ€ํ•˜โ€™ ๋“ฑ ๊น€์ •์ผ์„ ์šฐ์ƒํ™”ํ•˜๊ณ  ์ฐฌ์–‘ยท๋ฏธํ™”ํ•˜๋Š” ์ž๋ฃŒ๋“ค์ด ์ €์žฅ๋˜์–ด ์žˆ์œผ๋ฉฐ, ใ€Œ8 ์ •์„ธ๋™ํ–ฅใ€ ํ•˜์œ„ ํด๋”๋Š” ใ€Œ์ •์„ธ2003ใ€ยทใ€Œ์ •์„ธ2004ใ€ยทใ€Œ์ •์„ธ2005ใ€ยทใ€Œ์ •์„ธํ‰๊ฐ€ใ€ยทใ€Œ(๋ช…์นญ 40 ์ƒ๋žต)์—ฐ๊ตฌ์†Œ์ •์„ธ์—ฐ๊ตฌใ€ ๋“ฑ 5๊ฐœ์˜ ์„ธ๋ถ€ ํด๋”๋กœ ๊ตฌ์„ฑ๋˜์–ด โ€˜(์กฐ์„ )๋ฐ˜๋„์˜ ์šด๋ช…์„ ๊ฒฐ์ •์ง€์„ ๋‘ ๊ฐœ์˜ ๊ตฐ์‚ฌ์ „๋žต ์ œ1๋ถ€ : ๋ฏธ๊ตญ์˜ ์„ ์ œ๊ณต๊ฒฉ์ „๋žต๊ณผ โ€˜์ž‘์ „๊ณ„ํš5027โ€™ ๋“ฑ ๋ถํ•œ์˜ ์ฃผ์˜ยท์ฃผ์žฅ์— ๋™์กฐํ•˜๋Š” ๊ฐ์ข… ์ž๋ฃŒ๋“ค์ด ์ €์žฅ๋˜์–ด ์žˆ๊ณ , ใ€Œ9 ์œ„๋Œ€์„ฑใ€ ํ•˜์œ„ ํด๋”๋Š” ใ€Œ-๊น€์ผ์„ฑ์ฃผ์„-ใ€ยทใ€Œ๊น€์ •์ˆ™์—ฌ์‚ฌใ€ยทใ€Œ๊น€์ •์ผ์œ„์›์žฅใ€ ๋“ฑ 10๊ฐœ์˜ ์„ธ๋ถ€ ํด๋”๋กœ ๊ตฌ์„ฑ๋˜์–ด โ€˜21์„ธ๊ธฐ ๊น€์ •์ผ์‹œ๋Œ€(21์„ธ๊ธฐ ํ–ฅ๋„์ด๋…, ๊น€์ผ์„ฑ-๊น€์ •์ผ์ฃผ์˜)โ€™ ๋“ฑ ๊น€์ •์ผ์„ ์ฐฌ์–‘ยท๋ฏธํ™”ํ•˜๋Š” ์ž๋ฃŒ๋“ค์ด ์ €์žฅ๋˜์–ด ์žˆ์œผ๋ฉฐ, ใ€ŒBookใ€ ํ•˜์œ„ ํด๋”๋Š” ใ€ŒML์ฃผ์˜์›์ „PDFํŒใ€ยทใ€Œ์ €์ž‘์ง‘ใ€ยทใ€Œ์ „์žBOOKใ€๋“ฑ์˜ 7๊ฐœ ์„ธ๋ถ€ ํด๋”๋กœ ๊ตฌ์„ฑ๋˜์–ด โ€˜ํ”ผ๋ฐ”๋‹คโ€™ ๋“ฑ ๋ถํ•œ์˜ ๊ณต์‚ฐ์ฃผ์˜ ํ˜๋ช…ํˆฌ์Ÿ์„ ์„ ๋™ํ•˜๋Š” ์ž๋ฃŒ๋“ค์ด ์ €์žฅ๋˜์–ด ์žˆ๊ณ , ใ€ŒCD์ž๋ฃŒใ€ ํ•˜์œ„ ํด๋”๋Š” ใ€Œcd_1ใ€ยทใ€Œcd_2ใ€ยทใ€ŒํšŒ๊ณ ๋กCDใ€ ๋“ฑ 5๊ฐœ์˜ ์„ธ๋ถ€ ํด๋”๋กœ ๊ตฌ์„ฑ๋˜์–ด โ€˜์ •์น˜๊ฒฝ์ œํ•™๊ฐœ๋ก  ์ฃผ์ฒด์˜ ์ •์น˜๊ฒฝ์ œํ•™โ€™ ๋“ฑ ํ•œ๊ตญ์„ ๋ฏธ๊ตญ์˜ ์‹๋ฏผ์ง€๋กœ ๊ทœ์ •ํ•˜๋ฉด์„œ ๋Œ€๋‚จ๊ณต์‚ฐํ™”๋ฅผ ์„ ๋™ํ•˜๋Š” ์ž๋ฃŒ๋“ค์ด ์ €์žฅ๋˜์–ด ์žˆ์œผ๋ฉฐ, ใ€ŒMusicใ€ ํ•˜์œ„ ํด๋”๋Š” ใ€Œ๊ฐ€๊ทนใ€ยทใ€Œ๋ช…๊ณก2ใ€ยทใ€Œ์กฐ์„ ๋ช…๊ณก์ „์ง‘ใ€ ๋“ฑ์˜ 6๊ฐœ ์„ธ๋ถ€ ํด๋”๋กœ ๊ตฌ์„ฑ๋˜์–ด โ€˜๋‹น์‹ ์ด ์—†์œผ๋ฉด ์กฐ๊ตญ๋„ ์—†๋‹ค(์Œ์„ฑํŒŒ์ผ)โ€™ ๋“ฑ ๊น€์ •์ผ์„ ์ฐฌ์–‘ํ•˜๋Š” ์ž๋ฃŒ๋“ค์ด ์ €์žฅ๋˜์–ด ์žˆ๊ณ , ใ€ŒPaekDusanใ€ ํ•˜์œ„ ํด๋”๋Š” ใ€Œalbumใ€ยทใ€Œsongsanใ€ ๋“ฑ 2๊ฐœ์˜ ์„ธ๋ถ€ ํด๋”๋กœ ๊ตฌ์„ฑ๋˜์–ด โ€˜๊น€์ผ์„ฑ ์žฅ๊ตฐ์˜ ๋…ธ๋ž˜(์Œ์„ฑํŒŒ์ผ)โ€™ ๋“ฑ ๊น€์ผ์„ฑ์„ ์ฐฌ์–‘ํ•˜๋Š” ์ž๋ฃŒ๋“ค์ด ์ €์žฅ๋˜์–ด ์žˆ์œผ๋ฉฐ, ใ€Œ๊ฐœ์ •์ž‘์—…ใ€ ํ•˜์œ„ ํด๋”๋Š” ใ€Œ1๋‹จ๊ณ„-ํ˜๋ช…๊ด€ํšŒ๊ณ ๋กใ€ยทใ€Œ2๋‹จ๊ณ„-๋ณ€ํ˜์ด๋ก 4๋Œ€๊ฐœ๋ก ใ€ยทใ€Œ3๋‹จ๊ณ„-์„ ๊ตฐ์‚ฌ์ƒใ€ ๋“ฑ 13๊ฐœ์˜ ์„ธ๋ถ€ ํด๋”๋กœ ๊ตฌ์„ฑ๋˜์–ด โ€˜์˜์ƒ(์ด์„œ โ€˜๋ถˆ๋ฉธ์˜ ํ–ฅ๋„โ€™ ์žฅํŽธ์†Œ์„ค)โ€™ ๋“ฑ ๊น€์ผ์„ฑยท๊น€์ •์ผ์„ ์ฐฌ์–‘ยท๋ฏธํ™”ํ•˜๋Š” ์ž๋ฃŒ๋“ค์ด ์ €์žฅ๋˜์–ด ์žˆ๊ณ , ใ€Œ๊ต์–‘์ปค๋ฆฌ04ใ€ ํ•˜์œ„ ํด๋”๋Š” ใ€Œ1 ๊ธฐ๋ณธํ•™์Šต๊ต์–‘ใ€ยทใ€Œ2 ์‹ฌํ™”ํ•™์Šต๊ต์–‘ใ€ยทใ€Œ3 ๊ณ ๊ธ‰ํ•™์Šต๊ต์–‘ใ€ ๋“ฑ 3๊ฐœ์˜ ์„ธ๋ถ€ ํด๋”๋กœ ๊ตฌ์„ฑ๋˜์–ด โ€˜์ฒ ์˜ ์‹ ๋…(์žฅํŽธ์†Œ์„ค)โ€™ ๋“ฑ ๊น€์ผ์„ฑ์˜ ํ˜๋ช…์‚ฌ์ƒ์„ ์ฐฌ์–‘ํ•˜๊ณ  ์ฃผ์ฒด์‚ฌ์ƒ์— ๋”ฐ๋ฅธ ํ˜๋ช…์  ์ƒํ™œ์„ ์„ ๋™ํ•˜๋Š” ์ž๋ฃŒ๋“ค์ด ์ €์žฅ๋˜์–ด ์žˆ์œผ๋ฉฐ, ใ€Œ์ •์„ธ๋ถ„์„ใ€ ํ•˜์œ„ ํด๋”๋Š” ใ€Œ์ •์„ธ๋™ํ–ฅใ€ยทใ€Œ์ •์„ธ๋ถ„์„์ž๋ฃŒใ€ยทใ€Œ์ฃผ์ œ๋ณ„์ž๋ฃŒใ€ ๋“ฑ 4๊ฐœ์˜ ํ•˜์œ„ ํด๋”๋กœ ๊ตฌ์„ฑ๋˜์–ด โ€˜๊น€์ผ์„ฑ์ฃผ์„๊ณผ ๋ฏผ์กฑ๋Œ€๋‹จ๊ฒฐโ€™ ๋“ฑ ๊น€์ผ์„ฑ์„ ์ฐฌ์–‘ยท๋ฏธํ™”ํ•˜๋Š” ์ž๋ฃŒ๋“ค์ด ์ €์žฅ๋˜์–ด ์žˆ๊ณ , ใ€Œ์ฃผ์ œ๋ณ„์ž๋ฃŒใ€ ํ•˜์œ„ ํด๋”๋Š” ใ€Œ์ฃผ์ฒด์‚ฌํšŒ์ฃผ์˜ใ€ยทใ€Œํ†ต์ผ์ „์„ ์ด๋ก ์ „๋žตใ€ยทใ€Œ์กฐ๊ตญํ†ต์ผ์ด๋ก ใ€ ๋“ฑ 6๊ฐœ์˜ ์„ธ๋ถ€ ํด๋”๋กœ ๊ตฌ์„ฑ๋˜์–ด โ€˜๋”๋””๊ฐ€๋„ ์‚ฌ๋žŒ์ƒ๊ฐ ํ•˜์ง€์š”โ€™ ๋“ฑ ์ฃผ์ฒด์‚ฌ์ƒ๊ณผ ๊น€๋ถ€์ž์˜ ์ •์น˜, ์˜ˆ์ˆ , ๊ฑด์„ค ๋“ฑ ๋‹ค์–‘ํ•œ ๋ถ„์•ผ์— ๋Œ€ํ•œ ์žฌ๋Šฅ์„ ์ฐฌ์–‘ํ•˜๋Š” ์ž๋ฃŒ๋“ค์ด ์ €์žฅ๋˜์–ด ์žˆ์œผ๋ฉฐ, ใ€Œ์ดˆ๋ฒŒ์ž๋ฃŒใ€ ํ•˜์œ„ ํด๋”๋Š” ใ€Œ(๋ช…์นญ 41 ์ƒ๋žต)ใ€ยทใ€Œ๊น€์ •์ผ์œ„์ธ์ƒใ€ยทใ€Œ(๋ช…์นญ 5 ์ƒ๋žต)์—ฐ๊ตฌ์†Œใ€ ๋“ฑ 25๊ฐœ์˜ ์„ธ๋ถ€ ํด๋”๋กœ ๊ตฌ์„ฑ๋˜์–ด โ€˜๋ฌธํ•™์˜ˆ์ˆ ์ž‘ํ’ˆ์˜ ์ข…์žโ€™ ๋“ฑ ๋ฌธํ•™์˜ˆ์ˆ ์ž‘ํ’ˆ์„ ์กฐ์„ ๋กœ๋™๋‹น์˜ ๋…ธ์„ ๊ณผ ์ •์ฑ…์„ ๊ตฌํ˜„ํ•˜๊ณ  ์ˆ˜๋‹จ์œผ๋กœ ์„ค๋ช…ํ•˜๋Š” ์ž๋ฃŒ ๋“ฑ์ด ์ €์žฅ๋˜์–ด ์žˆ๊ณ , ใ€Œํ™”๋ฉด์Œ์•…ใ€ ํ•˜์œ„ ํด๋”์—๋Š” ๋ถํ•œ๋…ธ๋ž˜๋ฅผ ํ™”๋ฉด์Œ์•…์œผ๋กœ ์ œ์ž‘ํ•œ ๋™์˜์ƒ ํŒŒ์ผ๋“ค๋กœ โ€˜์‚ฌํšŒ์ฃผ์˜๋Š” ์šฐ๋ฆฌ๊ฑฐ์•ผโ€™ ๋“ฑ ๋ถํ•œ์‹ ์‚ฌํšŒ์ฃผ์˜๋ฅผ ์ฐฌ์–‘ํ•˜๋Š” ์ž๋ฃŒ๋“ค์ด ์ €์žฅ๋˜์–ด ์žˆ๋‹ค. ์œ„ ์ƒ์œ„ ํด๋” ์ค‘ ใ€Œ3 ์˜ํ™”ใ€ ํด๋”๋Š” ์˜ํ™” ์ œ๋ชฉ๋ณ„ ํ•˜์œ„ ํด๋”๋กœ ๊ตฌ์„ฑ๋˜์–ด โ€˜์›”๋ฏธ๋„โ€™, โ€˜์‹ฌ์žฅ์— ๋‚จ๋Š” ์‚ฌ๋žŒโ€™ ๋“ฑ ๊น€์ผ์„ฑ์— ๋Œ€ํ•œ ์ ˆ๋Œ€์  ์ถฉ์„ฑ์„ ๊ฐ•์กฐํ•˜๊ณ  ๋ถํ•œ์‹ โ€˜๊ฒฐ์‚ฌ๊ด€์ฒ  ์ •์‹ โ€™ ๋“ฑ์„ ์„ ์ „ํ•˜๋Š” ์ž๋ฃŒ๋“ค์ด ์ €์žฅ๋˜์–ด ์žˆ๋‹ค. ์œ„ ์ƒ์œ„ ํด๋” ์ค‘ ใ€Œmp3ใ€ ํด๋”๋Š” ใ€Œ[mp3]์กฐ์ธ๊ตฐ๊ฐ€ใ€ยทใ€ŒํšŒ๊ณ ๋ก์ „๋ฌธ๋‚ญ๋…1-24์žฅ(์ˆœ์„œ์ •๋ฆฌ)ใ€ยทใ€Œ[mp3]ํšŒ๊ณ ๋กใ€ ๋“ฑ 6๊ฐœ์˜ ํ•˜์œ„ ํด๋”๋กœ ๊ตฌ์„ฑ๋˜์–ด ์žˆ๋Š”๋ฐ, ใ€Œ[mp3]์กฐ์ธ๊ตฐ๊ฐ€ใ€ ํ•˜์œ„ ํด๋”์—๋Š” โ€˜์ฒœ๋งŒ์ด ์ดํญํƒ„ ๋˜๋ฆฌ๋ผ(๋ถํ•œ ๋…ธ๋ž˜ ํŒŒ์ผ)โ€™ ๋“ฑ ์„ ๊ตฐ์ •์น˜๋ฅผ ์ •๋‹นํ™”ํ•˜๊ณ  ๋ฏธํ™”ยท์ฐฌ์–‘ํ•˜๋Š” ์ž๋ฃŒ๋“ค์ด ์ €์žฅ๋˜์–ด ์žˆ๊ณ , ใ€ŒํšŒ๊ณ ๋ก์ „๋ฌธ๋‚ญ๋…1-24์žฅ(์ˆœ์„œ์ •๋ฆฌ)ใ€ ํ•˜์œ„ ํด๋”์—๋Š” โ€˜์„ธ๊ธฐ์™€ ๋”๋ถˆ์–ด 1๊ถŒ 3์žฅ 1์ ˆ(์Œ์„ฑ ํŒŒ์ผ)โ€™ ๋“ฑ ๊น€์ผ์„ฑ์„ ์šฐ์ƒํ™”ํ•˜๊ณ  ๋ฏธํ™”ยท์ฐฌ์–‘ํ•˜๋Š” ์ž๋ฃŒ๋“ค์ด ์ €์žฅ๋˜์–ด ์žˆ์œผ๋ฉฐ, ใ€Œ[mp3]๋ฐฉ์†ก๊ฐ•์ขŒใ€ ํ•˜์œ„ ํด๋”์—๋Š” โ€˜ํ•ญ์ผ ์—ฌ์„ฑ์˜์›… ๊น€์ •์ˆ™ ๋™์ง€ ํ˜๋ช…์—ญ์‚ฌ ๊ณผ๋ชฉ์— ๋Œ€ํ•œ ๊ฐ•์˜โ€™ ๋“ฑ ๊น€์ผ์„ฑ ๊ฐ€๊ณ„ ์ธ๋ฌผ ๋“ฑ์„ ์ฐฌ์–‘ยท๋ฏธํ™”ํ•˜๋Š” ์ž๋ฃŒ๋“ค์ด ์ €์žฅ๋˜์–ด ์žˆ๊ณ , ใ€Œ[mp3]ํšŒ๊ณ ๋กใ€ยทใ€Œ[mp3]ํšŒ๊ณ ๋ก์ „๋ฌธ๋‚ญ๋…1-11์žฅใ€ยทใ€Œ[mp3]ํšŒ๊ณ ๋ก์ „๋ฌธ๋‚ญ๋…1-24์žฅใ€ ๋“ฑ ๊ฐ ํ•˜์œ„ ํด๋”์—๋Š” โ€˜์„ธ๊ธฐ์™€ ๋”๋ถˆ์–ด(์Œ์„ฑํŒŒ์ผ)โ€™ ๋“ฑ ๊น€์ผ์„ฑ์„ ์šฐ์ƒํ™”ํ•˜์—ฌ ๋ฏธํ™”ยท์ฐฌ์–‘ํ•˜๋Š” ์ž๋ฃŒ๋“ค์ด ์ €์žฅ๋˜์–ด ์žˆ๋‹ค. ์œ„ ์ €์žฅ๋งค์ฒด์— ์ˆ˜๋ก๋œ ํŒŒ์ผ๋“ค์˜ ์ฃผ์š” ๋‚ด์šฉ์€ ๋‹ค์Œ๊ณผ ๊ฐ™๋‹ค. ใ€Š1 ํ•™์Šต ํด๋” ์ค‘ 10๋‹จ๊ณ„ํ•™์Šต ํด๋” ๋‚ด โ€˜ํ•œ๊ตญ์‚ฌํšŒ ์„ฑ๊ฒฉ ๋…ผ์˜์˜ ์žฌ์กฐ๋ช…โ€™์˜ ์ฃผ์š” ๋‚ด์šฉใ€‹ - ํ•œ๊ตญ์€ ๋Œ€๋ฏธ๊ตฐ์‚ฌ์˜ˆ์†์€ ๋ฏธ๊ตญ์ด ๊ตฐ์‚ฌ์ ์œผ๋กœ ์นจํˆฌํ•ด ์žˆ๋Š” ๊ทธ ์–ด๋Š ๋‚˜๋ผ์—๋„ ๋น„๊ธธ ์ˆ˜ ์—†์ด ์‹ฌํ™”๋œ ์ƒํƒœ์— ์žˆ๋‹ค. ํ•œ๊ตญ์‚ฌํšŒ์˜ ์‹๋ฏผ์ง€์„ฑ์€ ๊ตฐ์‚ฌ์  ์˜ˆ์†์„ฑ์„ ๋ฐฐ๊ฒฝ์œผ๋กœ ํ•˜์—ฌ ์ •์น˜์ฒด์ œ๊ฐ€ ๋ฏธ์ œ์—๊ฒŒ ์™„์ „ํžˆ ์˜ˆ์†ํ™”๋˜์–ด ์žˆ๋‹ค๋Š” ์‚ฌ์‹ค์— ๊ทผ๊ฑฐํ•˜๊ณ  ์žˆ๋‹ค. - ํ•œ๊ตญ์‚ฌํšŒ์˜ ํƒˆ ์‹๋ฏผ์ง€ํ™”๋Š” ์ฃผํ•œ๋ฏธ๊ตฐ๊ณผ ํ•ต๋ฌด๊ธฐ๋ฅผ ๋น„๋กฏํ•œ ๊ตฐ์‚ฌ์žฅ๋น„์˜ ์™„์ „ ์ฒ ์ˆ˜, ๊ตฐํ†ต์ˆ˜๊ถŒ์˜ ์‹ค์งˆ์  ์™„์ „ํ•œ ๋ฐ˜ํ™˜, ์ผ์ฒด ๋ถˆํ‰๋“ฑํ•œ ์กฐ์•ฝ๊ณผ ํ˜‘์•ฝ์˜ ์™„์ „ ํ๊ธฐ์™€ ํ•จ๊ป˜ ๋ฏธ์ œ๊ตญ์ฃผ์˜์— ์˜ํ•˜์—ฌ ๊ตฌ์ถ•๋œ ์‹๋ฏผ์ง€ ์ง€๋ฐฐ๋ฅผ ์œ„ํ•œ ์ •์น˜ยท๊ฒฝ์ œ๊ตฌ์กฐ๊ฐ€ ์ฒ ํ๋˜์–ด์•ผ ์‹คํ˜„๋  ์ˆ˜ ์žˆ๋Š” ๊ฒƒ์ด๋‹ค. - ์ฃผ์ฒด์‚ฌ์ƒ์ด ํ•œ๊ตญ๋ณ€ํ˜์šด๋™๋Œ€์˜ค์—์„œ ํ™•๊ณ ํ•œ ์ง€๋„์‚ฌ์ƒ์œผ๋กœ ๋ฟŒ๋ฆฌ๋‚ด๋ฆฌ๊ณ  ๋ฐ˜๋ฏธ์ž์ฃผํ™”์šด๋™์„ ์ถ•์œผ๋กœ ๋ฐ˜ํŒŒ์‘ˆ ๋ฏผ์ฃผํ™”์™€ ์กฐ๊ตญํ†ต์ผ์ด‰์ง„์šด๋™์ด ๊ธ‰๊ฒฉํžˆ ๊ณ ์–‘๋˜๊ณ  ์žˆ๋Š” ๊ฒƒ๊ณผ ๋•Œ๋ฅผ ๊ฐ™์ด ํ•˜์—ฌ์•ผ ํ•  ๊ฒƒ์ด๋‹ค. - ์œ„ ๋ฌธ๊ฑด์€ ํ•œ๊ตญ์„ ๋ฏธ๊ตญ์˜ ์‹๋ฏผ์ง€๋กœ ์™œ๊ณก ๊ทœ์ •ํ•˜๊ณ , ๊น€์ผ์„ฑ ์ฃผ์ฒด์‚ฌ์ƒ์— ์ž…๊ฐํ•œ ๋Œ€๋‚จํ˜๋ช…ํˆฌ์Ÿ ๋“ฑ์— ๋Œ€ํ•˜์—ฌ ์„ ์ „ยท์„ ๋™ํ•˜๋Š” ๊ฒƒ์„ ์ฃผ์š” ๋‚ด์šฉ์œผ๋กœ ํ•˜๊ณ  ์žˆ๋‹ค. ใ€Š1 ํ•™์Šต ํด๋” ์ค‘ ๊ต์–‘ ํด๋” ๋‚ด โ€˜๋ฐฑ๋‘์‚ฐ 3๋Œ€ ์žฅ๊ตฐ์˜ ์ˆญ๊ณ ํ•œ ๋‹น์กฐ์ง ๊ด€๋…์„ ๋”ฐ๋ผ ๋ฐฐ์šธ ๋ฐ ๋Œ€ํ•˜์—ฌโ€™์˜ ์ฃผ์š” ๋‚ด์šฉใ€‹ - โ€˜1. ์กฐ์ง์„ ๊ท€์ค‘ํžˆ ์—ฌ๊ธฐ๊ณ  ์กด์—„์žˆ๊ฒŒ ๋Œ€ํ•˜๋Š” ๋น›๋‚˜๋Š” ๋ชจ๋ฒ”์„ ๋”ฐ๋ผ ๋ฐฐ์›Œ์•ผ ํ•œ๋‹ค.โ€™, โ€˜2. ๋ชจ๋“  ๊ฒƒ์„ ๋‹น์กฐ์ง์— ์ฒ ์ €ํžˆ ์˜๊ฑฐํ•ด์„œ ์ง„ํ–‰ํ•˜๋Š” ์ˆญ๊ณ ํ•œ ํ’๋ชจ๋ฅผ ๋ณธ๋ฐ›์•„์•ผ ํ•œ๋‹ค.โ€™, โ€˜3. ๋‹น์กฐ์ง ๊ทœ์œจ์„ ์–ด๊น€์—†์ด ์ง€ํ‚ค๋Š” ์ž๊ฐ์ ์ธ ๋‹น์ƒํ™œ ๊ธฐํ’์„ ๋”ฐ๋ผ ๋ฐฐ์›Œ์•ผ ํ•œ๋‹ค.โ€™ ๋“ฑ์˜ ์ˆœ์„œ๋กœ ๋ฐฑ๋‘์‚ฐ 3๋Œ€ ์žฅ๊ตฐ(๊น€์ผ์„ฑ, ๊น€์ •์ˆ™, ๊น€์ •์ผ)์ด ๋ณด์—ฌ์ค€ ์ˆญ๊ณ ํ•œ ๋‹น์กฐ์ง ๊ด€๋…์„ ๋ฐฐ์›Œ ํ˜๋ช…์  ๋‹น์ƒํ™œ ๊ธฐํ’์„ ํ™•๋ฆฝํ•˜๊ณ , ๊น€์ •์€ ๋™์ง€๊ป˜ ์ถฉ์‹คํ•œ ๋‹น์›์ด ๋˜์ž๋Š” ์ทจ์ง€์˜ ๋‚ด์šฉ์ด๋‹ค. ใ€Š1 ํ•™์Šต ํด๋” ์ค‘ ๊ต์–‘ ํด๋” ๋‚ด โ€˜๋ฏผ์กฑ๊ณผ ์ฒ ํ•™โ€™์˜ ์ฃผ์š” ๋‚ด์šฉใ€‹ - ์ˆ˜๋ น, ๋‹น, ๋ฏผ์ค‘์˜ ํ†ต์ผ์ฒด๋กœ์„œ์˜ ์—ญ์‚ฌ์˜ ์ž์ฃผ์  ์ฃผ์ฒด์— ๊ด€ํ•œ ์ˆ˜๋ น๊ด€์˜ ๊ทผ๋ณธ์›๋ฆฌ๋Š” ์ฃผ์ฒด์˜ ์ฒ ํ•™์  ์›๋ฆฌ์— ๊ธฐ์ดˆํ•˜๊ณ  ์žˆ๋‹ค. ์ฃผ์ฒด์ฒ ํ•™์€ ์‚ฌ๋žŒ์ด ๋ชจ๋“  ๊ฒƒ์˜ ์ฃผ์ธ์ด๋ฉฐ ๋ชจ๋“  ๊ฒƒ์„ ๊ฒฐ์ •ํ•œ๋‹ค๋Š” ์ฒ ํ•™์  ์›๋ฆฌ์— ๊ธฐ์ดˆํ•ด์„œ ์ „๊ฐœ๋˜๊ณ  ์ „์ผ์ ์œผ๋กœ ์ฒด๊ณ„ํ™”๋œ๋‹ค. - ๊น€์ผ์„ฑ์ฃผ์„๊ป˜์„œ๋Š” ์ผ์ฐ ๊ทธ ๋ˆ„๊ตฌ๋„ ์ง€๋‹Œ ์  ์—†๋˜ ๋น„๋ฒ”ํ•œ ์˜ˆ์ง€์™€ ํƒ์›”ํ•œ ์˜๋„๋ ฅ ๊ณ ๋งคํ•œ ๋•์„ฑ์„ ํ•œ ๋ชธ์— ์ฒดํ˜„ํ•˜์‹œ๊ณ  ์‹ฌ์˜คํ•œ ์‚ฌํšŒ๋ณ€ํ˜์šด๋™์ด๋ก ๊ณผ ๊ฑฐ์ฐฝํ•œ ์‹ค์ฒœ์œผ๋กœ ํ˜„๋Œ€ ์—ญ์‚ฌ๋ฅผ ์ƒˆ๋กญ๊ฒŒ ๊ฐœ์ฒ™ํ•˜์‹œ๊ณ  ๋น›๋‚ด์‹  ๋ฏผ์ค‘์˜ ๊ฐ€์žฅ ์œ„๋Œ€ํ•œ ์ˆ˜๋ น์ด์‹œ๋ฉฐ ๋ฏผ์ค‘์˜ ์ž์ฃผ์œ„์—…์— ๋Œ€ํ•œ ๋์—†๋Š” ํ—Œ์‹ ์„ฑ๊ณผ ๋ฏผ์ค‘์— ๋Œ€ํ•œ ๋œจ๊ฑฐ์šด ์‚ฌ๋ž‘์œผ๋กœ ์‹ค์ฒœํˆฌ์Ÿ์˜ ์ „ ๋…ธ์ •์„ ์ˆ˜๋†“์•„ ์˜ค์‹  ๋ฏผ์ค‘์˜ ์ž์• ๋กœ์šด ์–ด๋ฒ„์ด์ด์‹œ๋‹ค. - ๊น€์ •์ผ ๋น„์„œ๊ป˜์„œ๋Š” ๊น€์ผ์„ฑ์ฃผ์„๊ป˜์„œ ๊ฐœ์ฒ™ํ•˜์‹  ๋ฏผ์ค‘์˜ ์ž์ฃผ ์œ„์—…์„ ๋Œ€๋ฅผ ์ด์–ด ๋๊นŒ์ง€ ์™„์„ฑํ•ด ๋‚˜๊ฐ€๋Š” ๊ฒƒ์„ ํ•„์ƒ์˜ ๊ณผ์ œ๋กœ ์‚ผ์œผ์‹œ๊ณ  ๋ถˆ๋ฉธ์˜ ์ฃผ์ฒด์‚ฌ์ƒ์„ ์ „๋ฉด์ ์œผ๋กœ ์ฒด๋“ํ•˜์‹œ๊ณ  ์‹ฌํ™” ๋ฐœ์ „์‹œ์ผœ ๋‚˜๊ฐ€์‹œ๋Š” ์ถฉ์‹ค์„ฑ์˜ ๊ท€๊ฐ์ด์‹œ๊ณ  ์‚ฌ์ƒ์ด๋ก ์˜ ์˜์žฌ์ด์‹œ๋‹ค. - ์œ„ ๋ฌธ๊ฑด์€ ๊น€์ผ์„ฑ ์ฃผ์ฒด์‚ฌ์ƒ์˜ ์ฒ ํ•™์  ์„ธ๊ณ„๊ด€, ์ฃผ์ฒด์‚ฌ๊ด€, ์ฃผ์ฒด์˜ ์ธ์ƒ๊ด€, ์ฃผ์ฒด์˜ ์ˆ˜๋ น๊ด€์„ ์„ค๋ช…ํ•˜๋ฉด์„œ ์ฃผ์ฒด์‚ฌ์ƒ์„ ํ†ตํ•ด ๊น€์ผ์„ฑยท๊น€์ •์ผ์„ ์šฐ์ƒํ™”ํ•˜๊ณ  ์ฐฌ์–‘ยท๋ฏธํ™”ํ•˜๋Š” ๊ฒƒ์„ ์ฃผ์š” ๋‚ด์šฉ์œผ๋กœ ํ•˜๊ณ  ์žˆ๋‹ค. ใ€Š2 ์ž๋ฃŒ๋ชจ์Œ ํด๋” ๋‚ด 1 study1 ํด๋” ์ค‘ โ€˜์ •์น˜์‚ฌ์ „1 ์œ„๋Œ€ํ•œ ์ˆ˜๋ น ๊น€์ผ์„ฑ๋™์ง€~๋ฒจ์ง€๋„โ€™์˜ ์ฃผ์š” ๋‚ด์šฉใ€‹ - ์œ„๋Œ€ํ•œ ์ˆ˜๋ น ๊น€์ผ์„ฑ ๋™์ง€๊ป˜์„œ๋Š” ์ธ๋ฅ˜์‚ฌ์ƒ์‚ฌ์—์„œ ๊ฐ€์žฅ ๋†’๊ณ  ๋น›๋‚˜๋Š” ์ž๋ฆฌ๋ฅผ ์ฐจ์ง€ํ•˜๋Š” ์˜์ƒ๋ถˆ๋ฉธ์˜ ์ฃผ์ฒด์‚ฌ์ƒ์„ ์ฐฝ์‹œํ•˜์‹œ์—ฌ ์ธ๋ฅ˜ ๋ ฅ์‚ฌ ๋ฐœ์ „์˜ ์ƒˆ๋กœ์šด ์‹œ๋Œ€, ์ฃผ์ฒด์‹œ๋Œ€๋ฅผ ๊ฐœ์ฒ™ํ•˜์‹œ์˜€์œผ๋ฉฐ ์ฃผ์ฒด์‚ฌ์ƒ์˜ ํ˜๋ช…์  ๊ธฐ์น˜๋ฅผ ๋†’์ด ๋“œ์‹œ๊ณ  ํ˜๋ช…๊ณผ ๊ฑด์„ค์„ ๋Š์ž„์—†๋Š” ์Šน๋ฆฌ๋กœ ์ด๋„์‹œ์–ด ์šฐ๋ฆฌ ์กฐ๊ตญ๊ณผ ๋ฏผ์กฑ์˜ ๋ ฅ์‚ฌ์  ์ง€์œ„๋ฅผ ๊ทผ๋ณธ์ ์œผ๋กœ ์ „๋ณ€์‹œํ‚ค๊ณ  ๊ตญ์ œ๊ณต์‚ฐ์ฃผ์˜์šด๋™๊ณผ ์„ธ๊ณ„ํ˜๋ช…๋ฐœ์ „์— ํŠน์ถœํ•œ ๊ณตํ—Œ์„ ํ•˜์‹œ์˜€์œผ๋ฉฐ, ๋กœ๋™ ๊ณ„๊ธ‰์˜ ํ˜๋ช…์œ„์—…์— ๋Œ€ํ•œ ๋์—†๋Š” ํ—Œ์‹ ์„ฑ๊ณผ ์ธ๋ฏผ์— ๋Œ€ํ•œ ๋œจ๊ฑฐ์šด ์‚ฌ๋ž‘์œผ๋กœ ๊ธฐ๋‚˜๊ธด ํ˜๋ช…ํˆฌ์Ÿ์˜ ์ „ ๋กœ์ •์„ ์ฐฌ๋ž€ํžˆ ์ˆ˜๋†“์•„ ์˜ค์‹œ์˜€๋‹ค. - ์œ„๋Œ€ํ•œ ์ˆ˜๋ น ๊น€์ผ์„ฑ ๋™์ง€์˜ ๋ น๋„ ๋ฐ‘์— ์กฐ์„ ๊ณต์‚ฐ์ฃผ์˜์ž๋“ค๊ณผ ์• ๊ตญ์  ์ธ๋ฏผ๋“ค์€ ์˜์›…์ ์ธ ํ•ญ์ผ๋ฌด์žฅํˆฌ์Ÿ์„ ์Šน๋ฆฌ์ ์œผ๋กœ ์กฐ์ง ์ „๊ฐœํ•˜์—ฌ ์˜๋ จ๊ตฐ๋Œ€์™€ ํ•จ๊ป˜ ์ผ์ œ ์นจ๋žต์ž๋“ค์„ ๊ฒฉ๋ฉธ ์†Œํƒ•ํ•จ์œผ๋กœ์จ ๋งˆ์นจ๋‚ด ์กฐ๊ตญ๊ด‘๋ณต์˜ ์„ฑ์Šค๋Ÿฌ์šด ์œ„์—…์„ ๋น›๋‚˜๊ฒŒ ์ˆ˜ํ–‰ํ•˜์˜€๋‹ค. - ์˜ค๋ž˜์ „๋ถ€ํ„ฐ ์นจ๋žต์ „์Ÿ์ค€๋น„๋ฅผ ๋‹ค๊ทธ์ณ ์˜ค๋˜ ๋ฏธ์ œ๊ตญ์ฃผ์˜์ž๋“ค๊ณผ ๊ทธ ์•ž์žก์ด๋“ค์€ ์กฐ๊ตญ์˜ ํ‰ํ™”์  ํ†ต์ผ์„ ์‹คํ˜„ํ•˜๊ธฐ ์œ„ํ•œ ๊ณตํ™”๊ตญ์ •๋ถ€์˜ ์ •๋‹นํ•œ ๋ฐฉ์•ˆ๋“ค์„ ๋‹ค ๊ฑฐ๋ถ€ํ•˜๊ณ  1950๋…„ 6์›” 25์ผ ๋“œ๋””์–ด ๋ถˆ์˜์— ๋ถ๋ฐ˜๋ถ€์— ๋Œ€ํ•œ ๋ฌด๋ ฅ์นจ๊ณต์„ ๊ฐํ–‰ํ•˜์˜€๋‹ค. - ๋ฏธ์ œ ์นจ๋žต์ž๋“ค์„ ๋ฐ˜๋Œ€ํ•˜๋Š” ์šฐ๋ฆฌ ์ธ๋ฏผ์˜ ์ •์˜์˜ ์กฐ๊ตญํ•ด๋ฐฉ์ „์Ÿ์€ ์ž์ฃผ์ ์ธ ์ธ๋ฏผ๊ณผ ์ œ๊ตญ์ฃผ์˜ ์นจ๋žต์ž, ํ˜๋ช…๊ตฐ๋Œ€์™€ ๋ฐ˜ํ˜๋ช…์  ์นจ๋žต๊ตฐ๋Œ€๊ฐ„์˜ ์ฒจ์˜ˆํ•œ ์ •์น˜๊ตฐ์‚ฌ์  ๋Œ€๊ฒฐ์ด์˜€์œผ๋ฉฐ ์šฐ๋ฆฌ ์ธ๋ฏผ์—๊ฒŒ ์žˆ์–ด์„œ ๋˜๋‹ค์‹œ ์ƒ์‚ฌ๋ฅผ ํŒ๊ฐ€๋ฆฌํ•˜๋Š” ์ค€์—„ํ•œ ์‹œ๋ จ์ด์—ˆ๋‹ค. - ์œ„ ๋ฌธ๊ฑด์€ ๊น€์ผ์„ฑ์˜ ํ•ญ์ผ๋ฌด์žฅํˆฌ์Ÿ์˜ ์—…์ ์„ ๋ฏธํ™”ํ•˜๊ณ , ๊น€์ผ์„ฑ์ด ์ผ์œผํ‚จ 6.25์ „์Ÿ์„ ๋ฏธ๊ตญ์˜ ์นจ๋žต์— ๋Œ€ํ•œ ๊น€์ผ์„ฑ์˜ ์กฐ๊ตญํ•ด๋ฐฉ์ „์Ÿ์œผ๋กœ ์™œ๊ณก ๋ฌ˜์‚ฌํ•˜๋Š” ๋“ฑ ๊น€์ผ์„ฑยท๊น€์ •์ผ์„ ์šฐ์ƒํ™”ํ•˜์—ฌ ๋ถํ•œ์ฒด์ œ์˜ ์ •๋‹น์„ฑ ๋“ฑ์„ ์„ ์ „ํ•˜๋Š” ๊ฒƒ์ด ์ฃผ์š” ๋‚ด์šฉ์ด๋‹ค. ใ€Š2 ์ž๋ฃŒ๋ชจ์Œ ํด๋” ์ค‘ 16G ํด๋” ๋‚ด โ€˜์‚ฌํšŒ์ฃผ์˜์— ๋Œ€ํ•œ ์ฃผ์ฒด์  ๋ฆฌํ•ดโ€™์˜ ์ฃผ์š” ๋‚ด์šฉใ€‹ - ์ธ๋ฅ˜์˜ ์ด์ƒ์ธ ์‚ฌํšŒ์ฃผ์˜๋ฅผ ์™„์„ฑํ•˜๊ธฐ ์œ„ํ•ด์„œ๋Š” ์ฃผ์ฒด์‚ฌ์ƒ๋งŒ์ด ๊ทธ ๋ฐฉ๋„์ด๋ฉฐ ์„ธ๊ณ„์˜ ์ด๋ชฉ์„ ์ง‘์ค‘์‹œํ‚ค๊ณ  ์žˆ๋Š” ๊ณตํ™”๊ตญ์˜ ์‚ฌํšŒ์ฃผ์˜๋Š” ์ฃผ์ฒด์˜ ์‚ฌํšŒ์ฃผ์˜ ์ด๋ก ์— ๊ธฐ์ดˆํ•˜์—ฌ ๊ฑด์„ค๋œ ์ธ๋ฏผ๋Œ€์ค‘ ์ค‘์‹ฌ์˜ ์‚ฌํšŒ์ฃผ์˜๋ผ๋Š” ์ทจ์ง€๋กœ ๋ถํ•œ์ฒด์ œ์™€ ์ฃผ์ฒด์‚ฌ์ƒ์„ ์˜นํ˜ธยท์ฐฌ์–‘ํ•˜๋Š” ๋‚ด์šฉ์ด๋‹ค. ใ€Š2 ์ž๋ฃŒ๋ชจ์Œ ํด๋” ์ค‘ 3 ์„ธ๊ธฐ์™€ ๋”๋ถˆ์–ด ํด๋” ๋‚ด โ€˜๊น€์ผ์„ฑ ๋™์ง€ ํšŒ๊ณ ๋ก ์„ธ๊ธฐ์™€ ๋”๋ถˆ์–ด 1~8โ€™์˜ ์ฃผ์š” ๋‚ด์šฉใ€‹ - ์ œ1๊ถŒ์€ ๊น€์ผ์„ฑ์˜ ์œ ์†Œ๋…„๊ธฐ(1912. 4. ~ 1930. 5.)๋ฅผ ๋‹ค๋ฃฌ ๊ฒƒ์œผ๋กœ, ๊น€์ผ์„ฑ ๊ฐ€๋ฌธ๊ณผ ์ถœ์ƒ๊ณผ์ •, โ€˜ํƒ€๋„์ œ๊ตญ์ฃผ์˜๋™๋งนโ€™ ๊ฒฐ์„ฑ๊ณผ์ • ๋ฐ ๊ธธ๋ฆผ์—์„œ์˜ ์ฒญ๋…„์‹œ์ ˆ ๋“ฑ์ด ์ˆ˜๋ก๋˜์–ด ์žˆ๊ณ , - ์ œ2๊ถŒ์€ ๊น€์ผ์„ฑ์˜ 20๋Œ€ ์ „ํ›„(1930. 5. ~ 1933. 2.)๋ฅผ ๋‹ค๋ฃฌ ๊ฒƒ์œผ๋กœ, ๊น€์ผ์„ฑ์˜ ์นด๋ฅœํšŒ์˜์™€ ์กฐ์„ ํ˜๋ช…๊ตฐ ํ™œ๋™, โ€˜9.18์‚ฌ๊ฑดโ€™ ๋“ฑ ์ฒญ๋…„ ๊ณต์‚ฐ์ฃผ์˜์ž๋กœ์„œ์˜ ์ดˆ๊ธฐ ํ™œ๋™์ƒ ๋“ฑ์ด ์ˆ˜๋ก๋˜์–ด ์žˆ์œผ๋ฉฐ, - ์ œ3๊ถŒ์€ ๊น€์ผ์„ฑ์˜ 20๋Œ€ ์ดˆ๋ฐ˜(1933. 2. ~ 1935. 2.)์„ ๋‹ค๋ฃฌ ๊ฒƒ์œผ๋กœ, ๋™๋…„ํ˜„์„ฑ ์ „ํˆฌ์™€ ๋งˆ์ดŒ์ž‘์ „, 1์ฐจ ๋ถ๋งŒ(๋ถ๋งŒ)์›์ •๊ณผ ์กฐ์„ ํ˜๋ช…๊ตฐ ์ฐฝ์„ค ๋“ฑ ๋งŒ์ฃผ์ง€์—ญ์—์„œ์˜ ํ•ญ์ผ๋ฌด์žฅํˆฌ์Ÿ ๊ณผ์ • ๋“ฑ์ด ์ˆ˜๋ก๋˜์–ด ์žˆ๊ณ , - ์ œ4๊ถŒ์€ ๊น€์ผ์„ฑ์˜ 20๋Œ€ ์ „๋ฐ˜(1935. 2. ~ 1936. 5.)์„ ๋‹ค๋ฃฌ ๊ฒƒ์œผ๋กœ, ๊ณต์†Œ์™ธ 37, ๊ณต์†Œ์™ธ 38 ๋“ฑ ์ธ๋ฏผํ˜๋ช…๊ตฐ ๊ฐ„๋ถ€๋“ค๊ณผ ํ•จ๊ป˜ ๊ฐ„๋„ํ† ๋ฒŒ, 2์ฐจ ๋ถ๋งŒ ์›์ • ๋“ฑ ๋™๋งŒ์—์„œ์˜ ํ•ญ์ผํˆฌ์Ÿ๊ณผ์ • ๋ฐ ์กฐ๊ตญ๊ด‘๋ณตํšŒ ๊ฒฐ์„ฑ ๋“ฑ์ด ์ˆ˜๋ก๋˜์–ด ์žˆ์œผ๋ฉฐ, - ์ œ5๊ถŒ์€ ๊น€์ผ์„ฑ์˜ 20๋Œ€ ์ค‘๋ฐ˜(1936. 5. ~ 1937. 3.)์„ ๋‹ค๋ฃฌ ๊ฒƒ์œผ๋กœ, ๋ถ๊ฐ„๋„์—์„œ ๋ฐฑ๋‘์‚ฐ ๋ฐ€์˜์œผ๋กœ ๊ทผ๊ฑฐ์ง€๋ฅผ ์˜ฎ๊ธฐ๋Š” ๊ณผ์ •, ์ผ๋ณธ ๊ด€๋™๊ตฐ ํ† ๋ฒŒ๋Œ€์™€์˜ ํ•ญ์ผํˆฌ์Ÿ๊ณผ์ • ๋“ฑ์ด ์ˆ˜๋ก๋˜์–ด ์žˆ๊ณ , - ์ œ6๊ถŒ์€ ๊น€์ผ์„ฑ์˜ 20๋Œ€ ์ค‘๋ฐ˜(1937. 3. ~ 11.)์„ ๋‹ค๋ฃฌ ๊ฒƒ์œผ๋กœ, ์กฐ๊ตญ๊ด‘๋ณตํšŒ ์กฐ์ง๋ง ํ™•๋Œ€, ๋ฌด์†ก์œผ๋กœ์˜ ํ–‰๊ตฐ, ๋ณด์ฒœ๋ณด ์ „ํˆฌ์˜ ์Šน๋ฆฌ์™€ ํ˜œ์‚ฐ์‚ฌ๊ฑด ๋“ฑ์ด ์ˆ˜๋ก๋˜์–ด ์žˆ์œผ๋ฉฐ, - ์ œ7๊ถŒ์€ ๊น€์ผ์„ฑ์˜ 20๋Œ€ ํ›„๋ฐ˜(1937. 11. ~ 1940. 3.)์„ ๋‹ค๋ฃฌ ๊ฒƒ์œผ๋กœ, ์ค‘๊ตญ๋™๋ถ ์ง€๋ฐฉ์˜ โ€˜๋งˆ๋‹น๊ฑฐ์šฐโ€™ ๋ฐ€์˜์—์„œ์˜ ์ƒํ™œ๊ณผ โ€˜๊ณ ๋‚œ์˜ ํ–‰๊ตฐโ€™ ๊ณผ์ • ๋ฐ ๋ฐฑ๋‘์‚ฐ ์ผ๋Œ€์—์„œ ์ผ์ œ์˜ ๋Œ€ํ† ๋ฒŒ๊ตฐ์„ ๋ฌผ๋ฆฌ์นœ ๊ณผ์ • ๋“ฑ์ด ์ˆ˜๋ก๋˜์–ด ์žˆ๊ณ , - ์ œ8๊ถŒ์€ ๊น€์ผ์„ฑ์˜ 30๋Œ€ ์ „ํ›„(1940. 3. ~ 1945. 10.)๋ฅผ ๋‹ค๋ฃฌ ๊ฒƒ์œผ๋กœ, ์†Œ๋ จยท์ค‘๊ตญ๊ณผ ๊ตญ์ œ์—ฐํ•ฉ๊ตฐ ์กฐ์ง ๋ฐ ๋™๋ถํ•ญ์ผ์—ฐ๊ตฐ๊ณผ์˜ ๋™๋งน ๋“ฑ 8.15 ํ•ด๋ฐฉ๊นŒ์ง€์˜ ํ•ญ์ผ๋ฌด์žฅํˆฌ์Ÿ ๋“ฑ์ด ์ˆ˜๋ก๋˜์–ด ์žˆ๋Š”๋ฐ”, - ์ด์™€ ๊ฐ™์ด ์œ„ ๋ฌธ๊ฑด์€ ๊น€์ผ์„ฑ์˜ ์ผ๋Œ€๊ธฐ๋ฅผ ๊ธฐ์ˆ ํ•œ ํšŒ๊ณ ๋ก ํ˜•์‹์œผ๋กœ ๊น€์ผ์„ฑ์˜ ํ•ญ์ผ๋ฌด์žฅํˆฌ์Ÿ ๋“ฑ์„ ์ฐฌ์–‘ยท๋ฏธํ™”ํ•˜๋ฉฐ ๊น€์ผ์„ฑ ์šฐ์ƒํ™”, ๋ถํ•œ์ฒด์ œ์˜ ์ •๋‹น์„ฑ ์„ ์ „, ๋Œ€๋‚จํ˜๋ช…ํˆฌ์Ÿ์ „๋žต ์„ ๋™ ๋“ฑ์„ ์ฃผ์š” ๋‚ด์šฉ์œผ๋กœ ํ•˜๊ณ  ์žˆ๋‹ค. ใ€Š2 ์ž๋ฃŒ๋ชจ์Œ ํด๋” ์ค‘ 4 ์ธํ„ฐ๋„ท์ฃผ์ฒด์‚ฌ์ƒ๊ฐ•์ขŒ ํด๋” ๋‚ด โ€˜์ •ํ†ต๊ณผ ๊ณ„์Šน ์œ„๋Œ€ํ•œ ์ธ๊ฐ„, ์ƒˆ๋กœ์šด ๋ฌธ๋ช…์„ ์œ„ํ•˜์—ฌโ€™์˜ ์ฃผ์š” ๋‚ด์šฉใ€‹ - ์กฐ์„ ์ธ๋ฏผํ˜๋ช…๊ตฐ ์‚ฌ๋ น๊ด€์ด์—ˆ๋˜ ๊น€์ผ์„ฑ ์žฅ๊ตฐ๊ณผ ์—ญ์‹œ ๊ฐ™์€ ๋ถ€๋Œ€์˜ ์œ ๊ฒฉ๋Œ€์›์ด๋˜ ๊น€์ •์ˆ™ ์—ฌ์‚ฌ๋ฅผ ๋ถ€๋ชจ๋กœ ํ•˜์—ฌ ํƒœ์–ด๋‚œ ์•„๊ธฐ๋Š” ๋‚œ์„ธ๋ฅผ ์˜ฌ๋ฐ”๋กœ ๋น„์ถฐ์ฃผ๋Š” ํƒœ์–‘์ด ๋˜๋ผ๋Š” ๋œป์—์„œ ๊น€์ •์ผ(๊น€์ •์ผ)๋กœ ์ด๋ฆ„ ์ง€์–ด์กŒ๋‹ค. - ๋ฏธ๊ตญ์ƒ์„  ์ œ๋„ˆ๋Ÿด์…”๋จผํ˜ธ๊ฐ€ ๋Œ€๋™๊ฐ•์œผ๋กœ ์˜ฌ๋ผ์™€ ๋Œ€ํฌ๋ฅผ ์˜๋ฉฐ ํ–‰ํŒจ๋ฅผ ๋ถ€๋ ธ์„ ๋‹น์‹œ ๊น€์ฃผ์„์˜ ์ฆ์กฐ๋ถ€์ธ ๊น€์‘์šฐ๊ฐ€ ๋†๋ฏผ๋“ค์„ ๋ชจ์•„์„œ ์‹ธ์› ๋‹ค๊ณ  ํ•œ๋‹ค. ๊น€์ฃผ์„์˜ ์•„๋ฒ„์ง€์ธ ๊น€ํ˜•์ง ์„ ์ƒ์ด ์ฒ ์ €ํ•œ ๋ฏผ์กฑ์ฃผ์˜์ž๋กœ ์ฒญ๋…„์‹œ์ ˆ๋ถ€ํ„ฐ ๋…๋ฆฝ์šด๋™์— ๋‚˜์„ฐ๋˜ ๊ฒƒ์€ ๋‹น์—ฐํ•œ ์ผ์ด๋ผ ํ•  ๊ฒƒ์ด๋‹ค. - ์กฐ๊ตญํ†ต์ผ์„ ์œ„ํ•œ ํˆฌ์Ÿ์˜ ์ „๋ฉด์— ๋‚˜์„  ๊ฐ€์žฅ ์ค‘์š”ํ•œ ๊ณผ์—…์€ ๊ณ ๋ ค๋ฏผ์ฃผ์—ฐ๋ฐฉ๊ณตํ™”๊ตญ ์ฐฝ๋ฆฝ๋ฐฉ์•ˆ์„ ์‹คํ˜„ํ•˜๋Š” ๊ฒƒ์ด์—ˆ๋‹ค. ๊น€์ผ์„ฑ ์ฃผ์„์ด ์ฒœ๋ช…ํ•œ ๊ณ ๋ ค ๋ฏผ์ฃผ์—ฐ๋ฐฉ๊ณตํ™”๊ตญ ์ฐฝ๋ฆฝ ๋ฐฉ์•ˆ์€ ์ž์ฃผ, ํ‰ํ™”ํ†ต์ผ, ๋ฏผ์กฑ ๋Œ€๋‹จ๊ฒฐ์˜ 3๋Œ€์›์น™์— ๊ธฐ์ดˆํ•˜์—ฌ ๋‚˜๋ผ์˜ ํ˜„์‹ค์  ์กฐ๊ฑด์— ๋งž๊ฒŒ ์กฐ๊ตญํ†ต์ผ์„ ์‹คํ˜„ํ•  ์ˆ˜ ์žˆ๋Š” ๋ฐฉ๋„๋ฅผ ๋ฐํžŒ ๊ณต๋ช…์ •๋Œ€ํ•œ ํ†ต์ผํ—Œ์žฅ์ด๋‹ค. - ์ด์™€ ๊ฐ™์ด ์œ„ ๋ฌธ๊ฑด์€ ๊น€์ผ์„ฑยท๊น€์ •์ผ ์ผ๊ฐ€๋ฅผ ์šฐ์ƒํ™”ํ•˜์—ฌ ์ฐฌ์–‘ยท๋ฏธํ™”ํ•˜๊ณ , ๋ถํ•œ์˜ ๋Œ€๋‚จํ˜๋ช…ํˆฌ์Ÿ์— ๋Œ€ํ•˜์—ฌ ์„ ์ „ยท์„ ๋™ํ•˜๋Š” ๊ฒƒ์„ ์ฃผ์š” ๋‚ด์šฉ์œผ๋กœ ํ•˜๊ณ  ์žˆ๋‹ค. ใ€Š2 ์ž๋ฃŒ๋ชจ์Œ ํด๋” ์ค‘ 9 ์œ„๋Œ€์„ฑ ํด๋” ๋‚ด โ€˜21์„ธ๊ธฐ ๊น€์ •์ผ์‹œ๋Œ€ (21์„ธ๊ธฐ ํ–ฅ๋„์ด๋…, ๊น€์ผ์„ฑ-๊น€์ •์ผ์ฃผ์˜)โ€™์˜ ์ฃผ์š” ๋‚ด์šฉใ€‹ - 20์„ธ๊ธฐ๋Š” ๊น€์ผ์„ฑ์ฃผ์„์˜ ์˜๋„ ๋ฐ‘์— ๊น€์ผ์„ฑ์ฃผ์˜๋ฅผ ๊ตฌํ˜„ํ•˜์—ฌ ์ธ๋ฅ˜๊ฐ€ ์ˆ˜์ฒœ๋…„ ๋™์•ˆ ์ด๋ฃฉํ•  ์ˆ˜ ์—†์—ˆ๋˜ ์„ธ๊ธฐ์  ๋ณ€ํ˜์„ ์ด๋ฃฉํ•œ ์˜๊ด‘์Šค๋Ÿฌ์šด ๊น€์ผ์„ฑ์‹œ๋Œ€์ด๋‹ค. - 21์„ธ๊ธฐ๋Š” ์ธ๋ฅ˜์˜ ์˜์›ํ•œ ์ง€๋„์‚ฌ์ƒ์ธ ๊น€์ผ์„ฑ์ฃผ์˜์— ์˜ํ•ด ์ง€๋„๋˜๋Š” ๊น€์ผ์„ฑ์‹œ๋Œ€๋ฐœ์ „์˜ ์ƒˆ๋กœ์šด ๋†’์€ ๋‹จ๊ณ„์ด๋‹ค. - ๊ฒฝ์• ํ•˜๋Š” ๊น€์ผ์„ฑ์ฃผ์„๊ป˜์„œ ๊น€์ผ์„ฑ์ฃผ์˜๋ฅผ ์ฐฝ์‹œํ•˜์‹œ๊ณ  ์šฐ๋ฆฌ ์‹œ๋Œ€๋ฅผ ์˜๋„ํ•˜์‹œ๋ฉด์„œ ํ˜„์‹œ๋Œ€์™€ ์˜์›ํ•œ ๋ฏธ๋ž˜์‹œ๋Œ€์ธ ๊น€์ผ์„ฑ์‹œ๋Œ€๊ฐ€ ํŽผ์ณ์ง€๊ฒŒ ๋˜์—ˆ๋‹ค - ๊น€์ •์ผ ์˜๋„์ž๊ป˜์„œ ๊น€์ผ์„ฑ์ฃผ์˜๋ฅผ ์ƒˆ๋กœ์šด ๋†’์€ ๋‹จ๊ณ„๋กœ ๋ฐœ์ „์‹œ์ผœ ๊น€์ผ์„ฑ-๊น€์ •์ผ์ฃผ์˜๋ฅผ ์ง€์นจ์œผ๋กœ ํ•˜์—ฌ ์‹œ๋Œ€๋ฅผ ์˜๋„ํ•˜์‹œ๋ฉด์„œ๋ถ€ํ„ฐ ๊น€์ผ์„ฑ์‹œ๋Œ€ ๋ฐœ์ „์˜ ์ƒˆ ๋‹จ๊ณ„๊ฐ€ ํŽผ์ณ์ง€๊ฒŒ ๋˜์—ˆ๋‹ค. - ์ด์™€ ๊ฐ™์€ ๋ฌธ๊ฑด์€ ๊น€์ผ์„ฑยท๊น€์ •์ผ์ฃผ์˜์˜ ์šฐ์ˆ˜์„ฑ์„ ์„ ์ „ํ•˜์—ฌ ๊น€๋ถ€์ž ์„ธ์Šต์˜ ์ •๋‹น์„ฑ, ๋ถํ•œ์ฒด์ œ์˜ ์šฐ์›”์„ฑ ๋“ฑ์„ ์„ ์ „ยท์ฐฌ์–‘ํ•˜๋Š” ๊ฒƒ์ด ์ฃผ์š” ๋‚ด์šฉ์œผ๋กœ ๋˜์–ด ์žˆ๋‹ค. ใ€Š2 ์ž๋ฃŒ๋ชจ์Œ ํด๋” ์ค‘ CD์ž๋ฃŒ ํด๋” ๋‚ด โ€˜์ •์น˜๊ฒฝ์ œํ•™๊ฐœ๋ก  ์ฃผ์ฒด์˜ ์ •์น˜๊ฒฝ์ œํ•™โ€™์˜ ์ฃผ์š” ๋‚ด์šฉใ€‹ - ์˜จ ์‚ฌํšŒ๋ฅผ ์ฃผ์ฒด์‚ฌ์ƒํ™”ํ•œ๋‹ค๋Š” ๊ฒƒ์€ ๊ณตํ™”๊ตญ ์ •๋ถ€์˜ ์ตœ์ข… ๋ชฉ์ ์ด๋‹ค. ์˜จ ์‚ฌํšŒ๋ฅผ ์ฃผ์ฒด์‚ฌ์ƒํ™”ํ•˜๊ธฐ ์œ„ํ•œ ํˆฌ์Ÿ์€ ๋ณธ์งˆ์— ์žˆ์–ด์„œ ์ธ๋ฏผ๋Œ€์ค‘์˜ ์ž์ฃผ์„ฑ์„ ์‹คํ˜„ํ•˜๊ธฐ ์œ„ํ•œ ํˆฌ์Ÿ์œผ๋กœ ์ธ๋ฏผ๋Œ€์ค‘์€ ํ˜๋ช…๊ณผ ๊ฑด์„ค์˜ ์ฃผ์ธ์ด๋ฉฐ ์ง์ ‘์  ๋‹ด๋‹น์ž์ด๋‹ค. - ์ฃผ์ฒด์˜ ์ •์น˜๊ฒฝ์ œํ•™์€ ์ด๋ถ ๋•…์— ๋ชจ๋“  ๊ฒƒ์ด ๊ทผ๋กœ๋ฏผ์ค‘์„ ์œ„ํ•ด ๋ณต๋ฌดํ•˜๋Š” ๊ฐ€์žฅ ์„ ์ง„์ ์ธ ์‚ฌํšŒ์ฃผ์˜ ์ œ๋„์™€ ์ž๋ฆฝ์  ๋ฏผ์กฑ๊ฒฝ์ œ๋ฅผ ๊ฐ€์ง„ ๊ฐ•๋ ฅํ•œ ์ž์ฃผ ๋…๋ฆฝ๊ตญ๊ฐ€๋ฅผ ์ผ๋– ์„ธ์šธ ์ˆ˜ ์žˆ๊ฒŒ ํ•œ ๊ณผํ•™์  ์ง€์นจ์ด๋ฉฐ ํ˜๋ช…์˜ ์ „๊ตญ์  ์Šน๋ฆฌ์™€ ์„ธ๊ณ„ํ˜๋ช…์˜ ์™„์ˆ˜๋ฅผ ์œ„ํ•œ ์œ„๋ ฅํ•œ ์‚ฌ์ƒ ์ด๋ก ์  ๋ฌด๊ธฐ์ด๋‹ค. - ์ฐฉ์ทจ์ž์ง‘๋‹จ๋“ค์€ ๋” ๋งŽ์€ ์ด์œค์„ ์–ป๊ธฐ ์œ„ํ•ด ์„œ๋กœ ์•„๊ท€๋‹คํˆผ์„ ํ•˜๋ฉด์„œ๋„ ๋…ธ๋™์ž๋“ค์„ ์–ต์••ํ•˜๊ณ  ์ฐฉ์ทจ ํ•˜๋Š”๋ฐ์„œ๋Š” ์ดํ•ด๊ด€๊ณ„๋ฅผ ๊ฐ™์ดํ•˜๋ฉฐ ํ•˜๋‚˜์˜ ์„ธ๋ ฅ์œผ๋กœ ๊ทœํ•ฉ๋œ๋‹ค. ๊ทธ๋Ÿฌ๋ฏ€๋กœ ๋…ธ๋™์ž๋“ค์€ ์ž๊ธฐ๋ฅผ ๊ณ ์šฉํ•œ ์ž๋ณธ๊ฐ€๋ฟ ์•„๋‹ˆ๋ผ ์ž๋ณธ๊ฐ€๊ณ„๊ธ‰ ์ „์ฒด๋ฅผ ๋ฐ˜๋Œ€ํ•˜์—ฌ ํˆฌ์Ÿํ•จ์œผ๋กœ์จ๋งŒ ์ž๋ณธ์˜ ์ฒ ์‡„๋ฅผ ๋Š์–ด๋ฒ„๋ฆด ์ˆ˜ ์žˆ๋‹ค. - ํ•œ๊ตญ์‚ฌํšŒ์˜ ์ •์น˜์  ์‹ค๊ถŒ์€ ์‚ฌ์‹ค์ƒ ๋ฏธ์ œ๊ตญ์ฃผ์˜์ž๋“ค์˜ ์†์— ์ฅ์–ด์ ธ ์žˆ์œผ๋ฉฐ, ๊ทธ๋“ค์˜ ์†Œ์œ„ ์›์กฐ๋Š” ํ•œ๊ตญ๊ฒฝ์ œ์˜ ์ž์ฃผ์  ๋ฐœ์ „์„ ์ €ํ•ดํ•˜๊ณ  ์ €๋“ค์˜ ๊ฒฝ์ œ์  ๊ตฐ์‚ฌ์  ๋ถ€์†๋ฌผ๋กœ ๋งŒ๋“ค๊ธฐ ์œ„ํ•œ ์ˆ˜๋‹จ์— ์ง€๋‚˜์ง€ ์•Š๋Š”๋‹ค. - ํ•˜๋ฃจ๋นจ๋ฆฌ ์ด๋‚จ์‚ฌํšŒ์˜ ๋ฏผ์ฃผํ™”์™€ ์กฐ๊ตญํ†ต์ผ์„ ์„ฑ์ทจํ•˜๋ฉฐ ๋‚˜์•„๊ฐ€์‚ฌ ํ†ต์ผ๋œ ์‚ผ์ฒœ๋ฆฌ์— ์ด๋ถ ํ˜•์ œ๋“ค๊ณผ ํž˜์„ ํ•ฉ์ณ ๊ณต์‚ฐ์ฃผ์˜์ด์ƒ์‚ฌํšŒ, ์ฃผ์ฒด์˜ ์ง€์ƒ๋‚™์›์„ ์ผ๋– ์„ธ์›Œ์•ผ ํ•œ๋‹ค. ์ด๋‚จ ๋ฏผ์ค‘์€ ๊ตญํ† ๋ถ„๋‹จ์˜ ๋น„์šด๊ณผ ๋ฐ˜๋™์  ์‹๋ฏผ์ง€์‚ฌํšŒ์ฒด์ œ์˜ ์ •์น˜๊ฒฝ์ œ์  ๊ตฌ์† ๋ฐ‘์—์„œ ์„ธ์›”์„ ๋ณด๋‚ด๊ณ  ์žˆ๋‹ค. - ์ด์™€ ๊ฐ™์€ ๋ฌธ๊ฑด์€ ๋‚จํ•œ์„ ๋ฏธ๊ตญ์˜ ์‹๋ฏผ์ง€๋กœ ์™œ๊ณก ๊ทœ์ •ํ•˜๊ณ  ๋ถํ•œ์˜ ์‚ฌํšŒยท๊ฒฝ์ œ์ฒด๊ณ„๋ฅผ ์ฃผ์ฒด์‚ฌ์ƒ์— ์˜ํ•ด ์‹คํ˜„๋œ ์ด์ƒ์ ์ธ ์‚ฌํšŒ์ฒด์ œ์ธ ์–‘ ์„ ์ „ํ•˜๋ฉด์„œ ๋Œ€๋‚จํ˜๋ช…์ „๋žต์— ๋”ฐ๋ฅธ ํ•œ๋ฐ˜๋„์˜ ๊ณต์‚ฐํ™”๋ฅผ ์‹คํ˜„ํ•˜๊ธฐ ์œ„ํ•ด ์„ ๋™ยท์„ ์ „ํ•˜๋Š” ๋‚ด์šฉ์œผ๋กœ ๊ตฌ์„ฑ๋˜์–ด ์žˆ๋‹ค. ใ€Š2 ์ž๋ฃŒ๋ชจ์Œ ํด๋” ์ค‘ ์ •์„ธ๋ถ„์„ ํด๋” ๋‚ด โ€˜๊น€์ผ์„ฑ์ฃผ์„๊ณผ ๋ฏผ์กฑ๋Œ€๋‹จ๊ฒฐโ€™์˜ ์ฃผ์š” ๋‚ด์šฉใ€‹ - ๊ฒฝ์• ํ•˜๋Š” ์ˆ˜๋ น ๊น€์ผ์„ฑ์ฃผ์„์„ ๋ฏผ์กฑ์˜ ์ˆ˜์œ„์— ๋†’์ด ๋ชจ์‹ฌ์œผ๋กœ์จ ๋งˆ์นจ๋‚ด ์šฐ๋ฆฌ ๋ฏผ์กฑ์‚ฌ์—์„œ๋Š” ๋ฏผ์กฑ๋Œ€๋‹จ๊ฒฐ์˜ ์œ„๋Œ€ํ•œ ๊ฒฝ๋ฅœ์ด ํŽผ์ณ์งˆ ์น ์น ์ด ๋“œ๋ฆฌ์šด ๋น„์šด์„ ๊ฐ€์‹œ๊ณ  ์šด๋ช…์„ ๊ฐœ์ฒ™ํ•  ๋ฏผ์กฑ์˜ ํ™œ๋กœ๊ฐ€ ๋ฐํ˜€์กŒ์œผ๋ฉฐ ์ด๋•Œ๋ถ€ํ„ฐ ์˜จ ๊ฒจ๋ ˆ๋Š” ๋ฏผ์กฑ๋Œ€๋‹จ๊ฒฐ์˜ ์œ„๋Œ€ํ•œ ์ค‘์‹ฌ์„ ๊ฐ€์ง€๊ฒŒ ๋˜์—ˆ๋‹ค. - ์šฐ๋ฆฌ ๋ฏผ์กฑ์ด ์ „๋ก€์—†์ด ๋น„๊ทน์— ์ž ๊ฒจ์žˆ๋˜ 20์„ธ๊ธฐ ์ดˆ์—ฝ์œผ๋กœ๋ถ€ํ„ฐ ์˜ค๋Š˜์— ์ด๋ฅด๋Š” ๊ฒฝ์• ํ•˜๋Š” ๊น€์ผ์„ฑ์ฃผ์„์˜ ํ•œ์ƒ์€ ๋ฏผ์กฑ๋ฌธ์ œ ํ•ด๊ฒฐ์— ์ „๋ ฅ์„ ๋‹คํ•ด ์˜ค์‹  ์˜๊ด‘์˜ ํ•œ์ƒ์ด๊ณ  ๋ฏผ์กฑ๋Œ€๋‹จํ•ฉ์— ํ—Œ์‹ ํ•ด ์˜ค์‹  ๋ถˆ๋ฉธ์˜ ๋ ฅ์‚ฌ์ด๋‹ค. ใ€Š2 ์ž๋ฃŒ๋ชจ์Œ ํด๋” ์ค‘ Music ํด๋” ๋‚ด โ€˜๋‹น์‹ ์ด ์—†์œผ๋ฉด ์กฐ๊ตญ๋„ ์—†๋‹คโ€™์˜ ์ฃผ์š” ๋‚ด์šฉใ€‹ - ๊น€์ •์ผ์„ ์šฐ์ƒํ™”ํ•˜์—ฌ ๋งน๋ชฉ์ ์œผ๋กœ ์ฐฌ์–‘ยท๋ฏธํ™”ํ•˜๋Š” ๋‚ด์šฉ์˜ ๋ถํ•œ ๋…ธ๋ž˜๋กœ, ๊ทธ ๊ฐ€์‚ฌ ๋‚ด์šฉ์€ โ€˜1. ์‚ฌ๋‚˜์šด ํญํ’๋„ ์ณ๋ชฐ์•„๋‚ด๊ณ  / ์‹ ๋…์„ ์•ˆ๊ฒจ์ค€ ๊น€์ •์ผ๋™์ง€ / ๋‹น์‹ ์ด ์—†์œผ๋ฉด ์šฐ๋ฆฌ๋„ ์—†๊ณ  / ๋‹น์‹ ์ด ์—†์œผ๋ฉด ์กฐ๊ตญ๋„ ์—†๋‹ค. 2. ๋ฏธ๋ž˜๋„ ํฌ๋ง๋„ ๋‹ค ๋งก์•„์ฃผ๋Š” / ๋ฏผ์กฑ์˜ ์šด๋ช…์ธ ๊น€์ •์ผ๋™์ง€ / ๋‹น์‹ ์ด ์—†์œผ๋ฉด ์šฐ๋ฆฌ๋„ ์—†๊ณ  / ๋‹น์‹ ์ด ์—†์œผ๋ฉด ์กฐ๊ตญ๋„ ์—†๋‹ค. 3. ์„ธ์ƒ์ด ์—ด๋ฐฑ๋ฒˆ ๋ณ€ํ•œ๋‹ค ํ•ด๋„ / ์ธ๋ฏผ์€ ๋ฏฟ๋Š”๋‹ค ๊น€์ •์ผ๋™์ง€ / ๋‹น์‹ ์ด ์—†์œผ๋ฉด ์šฐ๋ฆฌ๋„ ์—†๊ณ  / ๋‹น์‹ ์ด ์—†์œผ๋ฉด ์กฐ๊ตญ๋„ ์—†๋‹คโ€™๋ผ๊ณ  ๊ตฌ์„ฑ๋˜์–ด ์žˆ๋‹ค. ใ€Š2 ์ž๋ฃŒ๋ชจ์Œ ํด๋” ์ค‘ PaekDusan ํด๋” ๋‚ด โ€˜๊น€์ผ์„ฑ ์žฅ๊ตฐ์˜ ๋…ธ๋ž˜โ€™์˜ ์ฃผ์š” ๋‚ด์šฉใ€‹ - ๊น€์ผ์„ฑ์„ ๋งน๋ชฉ์ ์œผ๋กœ ์ฐฌ์–‘ํ•˜๋Š” ๋‚ด์šฉ์˜ ๋ถํ•œ ๋…ธ๋ž˜๋กœ, ๊ทธ ๊ฐ€์‚ฌ ๋‚ด์šฉ์€ โ€˜1. ์žฅ๋ฐฑ์‚ฐ ์ค„๊ธฐ์ค„๊ธฐ ํ”ผ์–ด๋ฆฐ ์ž์šฑ / ์••๋ก๊ฐ• ๊ตฝ์ด๊ตฝ์ด ํ”ผ์–ด๋ฆฐ ์ž์šฑ / ์˜ค๋Š˜๋„ ์ž์œ ์กฐ์„  ๊ฝƒ๋‹ค๋ฐœ ์šฐ์— / ๋ ฅ๋ ฅํžˆ ๋น„์ถฐ์ฃผ๋Š” ๊ฑฐ๋ฃฉํ•œ ์ž์šฑ / ์•„ ๊ทธ ์ด๋ฆ„๋„ ๊ทธ๋ฆฌ์šด ์šฐ๋ฆฌ์˜ ์žฅ๊ตฐ / ์•„ ๊ทธ ์ด๋ฆ„๋„ ๋น›๋‚˜๋Š” ๊น€์ผ์„ฑ ์žฅ๊ตฐ / 2. ๋งŒ์ฃผ๋ฒŒ ๋ˆˆ๋ฐ”๋žŒ์•„ ์ด์•ผ๊ธฐํ•˜๋ผ / ๋ฐ€๋ฆผ์˜ ๊ธด๊ธด ๋ฐค์•„ ์ด์•ผ๊ธฐํ•˜๋ผ / ๋งŒ๊ณ ์˜ ๋นจ์น˜์‚ฐ์ด ๋ˆ„๊ตฌ์ธ๊ฐ€๋ฅผ / ์ ˆ์„ธ์˜ ์• ๊ตญ์ž๊ฐ€ ๋ˆ„๊ตฌ์ธ๊ฐ€๋ฅผ / ์•„ ๊ทธ ์ด๋ฆ„๋„ ๊ทธ๋ฆฌ์šด ์šฐ๋ฆฌ์˜ ์žฅ๊ตฐ / ์•„ ๊ทธ ์ด๋ฆ„๋„ ๋น›๋‚˜๋Š” ๊น€์ผ์„ฑ ์žฅ๊ตฐ / 3. ๋กœ๋™์ž ๋Œ€์ค‘์—๊ฒŒ ํ•ด๋ฐฉ์˜ ์€์ธ / ๋ฏผ์ฃผ์˜ ์ƒˆ์กฐ์„ ์— ์œ„๋Œ€ํ•œ ํƒœ์–‘ / ์ด์‹ญ๊ฐœ ์ •๊ฐ•์œ„์— ๋ชจ๋‘ ๋‹ค ๋ญ‰์ณ / ๋ถ์กฐ์„  ๋ฐฉ๋ฐฉ๊ณก๊ณก ์ƒˆ๋ด„์ด ์˜ค๋‹ค / ์•„ ๊ทธ ์ด๋ฆ„๋„ ๊ทธ๋ฆฌ์šด ์šฐ๋ฆฌ์˜ ์žฅ๊ตฐ / ์•„ ๊ทธ ์ด๋ฆ„๋„ ๋น›๋‚˜๋Š” ๊น€์ผ์„ฑ ์žฅ๊ตฐโ€™์ด๋ผ๊ณ  ๊ตฌ์„ฑ๋˜์–ด ์žˆ๋‹ค. ใ€Š2 ์ž๋ฃŒ๋ชจ์Œ ํด๋” ์ค‘ ๊ฐœ์ •์ž‘์—… ํด๋” ๋‚ด โ€˜์˜์ƒโ€™์˜ ์ฃผ์š” ๋‚ด์šฉใ€‹ - 1997๋…„ ๋ถํ•œ์—์„œ ๊น€์ผ์„ฑ ์šฐ์ƒํ™”๋ฅผ ์œ„ํ•ด ๋งŒ๋“  ์ด์„œ โ€˜๋ถˆ๋ฉธ์˜ ๋ ฅ์‚ฌโ€™ ์†Œ์„ค ์ค‘ ํ•˜๋‚˜๋กœ ๊น€์ผ์„ฑ์ด ์‚ฌ๋งํ•œ 1994๋…„๋„์˜ ๊ทธ์˜ ํ–‰์ ์„ ๋ฏธํ™”ํ•œ ๊ฒƒ์œผ๋กœ ๊น€์ผ์„ฑ์€ ์‹ ๋…„์‚ฌ๋ฅผ ๋งˆ์นœ ๊ทธ๊ธธ๋กœ ํ˜‘๋™๋†์žฅ์„ ๋ฐฉ๋ฌธํ–ˆ๊ณ  ๋‹จ๊ตฐ๋ฆ‰ ๊ฐœ๊ฑด ๊ณต์‚ฌ์žฅ์„ ์‹œ์ฐฐํ•˜๋Š”๊ฐ€ ํ•˜๋ฉด ๋ฐค์ด๋ฉด ๋ฐค๋งˆ๋‹ค ์ƒˆ๋ฒฝ 3์‹œ๊ฐ€ ๋˜๋„๋ก ์—…๋ฌด๋ฅผ ๋ณด๊ณ  ์ •๋‹น, ์‚ฌํšŒ๋‹จ์ฒด ์ผ๊พผํ˜‘์˜ํšŒ, ์†Œ๋…„๋‹จ ๋Œ€ํšŒ ๋“ฑ์— ์ฐธ์„ํ•˜์—ฌ ์ด๋“ค์„ ์นœํžˆ ์ง€๋„ํ•˜์˜€๊ณ  ์ด๋Ÿฌํ•œ ๊น€์ผ์„ฑ์˜ ํƒ์›”ํ•œ ์‚ฌ์ƒ๊ณผ ์œ„๋Œ€ํ•œ ๋ฏธ๋•์„ ๊ทธ๋Œ€๋กœ ์ด์–ด๋ฐ›์€ ๊น€์ •์ผ์ด ์žˆ์–ด ๋ฏผ์กฑ์˜ ์•ž๋‚ ์€ ์ฐฝ์ฐฝํ•˜๋‹คโ€™ ๋“ฑ์œผ๋กœ ๊น€์ผ์„ฑยท๊น€์ •์ผ์„ ์šฐ์ƒํ™”ํ•˜์—ฌ ์ฐฌ์–‘ยท๋ฏธํ™”ํ•˜๋Š” ๋‚ด์šฉ์ด๋‹ค. ใ€Š3 ์˜ํ™” ํด๋” ์ค‘ ์›”๋ฏธ๋„ ํด๋” ๋‚ด โ€˜์›”๋ฏธ๋„โ€™์˜ ์ฃผ์š” ๋‚ด์šฉใ€‹ - 1950๋…„ ๋ถํ•œ๊ตฐ ํ•ด์•ˆํฌ๋ณ‘ ์ค‘๋Œ€ ๋ณ‘์‚ฌ๋“ค์ด ํ•œยท๋ฏธ๊ตญ๊ตฐ์˜ ์ธ์ฒœ์ƒ๋ฅ™ ์ž‘์ „์— ๋งž์„œ ๋ถํ•œ๊ตฐ์€ ๋ถˆ๊ณผ 4๋ฌธ์˜ ํฌ๋ฅผ ๊ฐ€์ง€๊ณ  5๋งŒ ๋ช…์˜ ๋ณ‘๋ ฅ ๋ฐ ์ˆ˜๋ฐฑ ์ฒ™์˜ ๊ตฐํ•จ ๋“ฑ์„ ๋™์›ํ•œ ๋ฏธ๊ตฐ์— ๋Œ€ํ•ด 3์ผ๊ฐ„ ๊ฒฐ์‚ฌ์ „์„ ๋ฒŒ์ด๋Š” ๊ฐ€์šด๋ฐ ์ฃผ์ธ๊ณต ๋ฆฌํƒœ์šด์€ ํฌ๋ณ‘ ์ฐธ๋ชจ์žฅ์œผ๋กœ ์Šน์ง„๋˜์—ˆ์Œ์—๋„ ์ด๋ฅผ ์ž์ง„ ํฌ๊ธฐํ•˜๊ณ  ์ง„์ง€๋ฅผ ์‚ฌ์ˆ˜ํ•  ๊ฒƒ์„ ์ฒœ๋ช…ํ•˜๊ณ , - ์ด์— ๋ถํ•œ๊ตฐ ๋ณ‘์‚ฌ๋“ค์€ ์œ„ํƒœ๋กœ์šด ์ƒํ™ฉ์—์„œ๋„ โ€œ๊น€์ผ์„ฑ ์žฅ๊ตฐ ๋งŒ์„ธ! ์กฐ๊ตญ์ด์—ฌ! ๋ฒˆ์˜ํ•˜๋ผ!โ€๋Š” ๋“ฑ ๊น€์ผ์„ฑ ์žฅ๊ตฐ์—๊ฒŒ ํŽธ์ง€๋ฅผ ์“ฐ๋ฉฐ ์ถฉ์„ฑ์„ ๋‹ค์งํ•˜๊ณ  ์ „์‚ฌํ•œ๋‹ค๋Š” ๊ฒƒ์œผ๋กœ ์†Œ์œ„ โ€˜ํ˜๋ช…์  ๋™์ง€์• โ€™์™€ ๊น€์ผ์„ฑ์— ๋Œ€ํ•œ ๋งน๋ชฉ์ ์ธ ์ถฉ์„ฑ์„ ๊ฐ•์กฐํ•˜์—ฌ ๊น€์ผ์„ฑ์— ๋Œ€ํ•œ ์ถฉ์„ฑ์‹ฌ์„ ๋งน๋ชฉ์ ์œผ๋กœ ๊ฐ•์กฐํ•˜๋Š” ๋‚ด์šฉ์˜ ์˜ํ™”์ด๋‹ค. ใ€Š3 ์˜ํ™” ํด๋” ์ค‘ ์‹ฌ์žฅ์— ๋‚จ๋Š” ์‚ฌ๋žŒ ํด๋” ๋‚ด โ€˜์‹ฌ์žฅ์— ๋‚จ๋Š” ์‚ฌ๋žŒโ€™์˜ ์ฃผ์š” ๋‚ด์šฉใ€‹ - ์˜ํ™” โ€˜์‹ฌ์žฅ์— ๋‚จ๋Š” ์‚ฌ๋žŒโ€™ 1๋ถ€ โ€˜์–ธ์•ฝโ€™์—์„œ๋Š” โ€˜๋ช…๋™ํƒ€์ด์–ด๊ณต์žฅโ€™ ์ดˆ๊ธ‰๋‹น๋น„์„œ์ธ ์ฃผ์ธ๊ณต์„ ์ทจ์žฌํ•˜๋Ÿฌ ์˜จ ์—ฌ์„ฑ์ด ์ฃผ๋ณ€ ์‚ฌ๋žŒ๋“ค๋กœ๋ถ€ํ„ฐ ์ฃผ์ธ๊ณต์˜ ํ™œ๋™์„ ๋“ฃ๋Š” ํ˜•์‹์„ ํ†ตํ•ด ๊ณต์žฅ์ดˆ๊ธ‰๋‹น์ดํšŒ ๊ฒฐ์ •์„œ๊ฐ€ ์ฑ„ํƒ๋˜๊ธฐ๊นŒ์ง€์˜ ๊ณผ์ •๊ณผ ์ง‘ํ–‰๊ณผ์ •์„ ๋ณด์—ฌ์ฃผ๊ณ , 2๋ถ€ โ€˜์ƒ๋ช…โ€™์—์„œ๋Š” ๋‹น๋น„์„œ๊ฐ€ ๊ฐ€์ •์˜ ์šฐํ™˜๋„ ๊ฐ์ถฐ๊ฐ€๋ฉฐ 1๋ถ€์—์„œ ์ฑ„ํƒ๋œ ๋‹น๊ฒฐ์ •์‚ฌ์—…์„ ์ƒ๋ช…์„ ๊ฑธ๊ณ  ์ง€์ผœ๋‚˜๊ฐ€ ํƒ€์ด์–ด ์ƒ์‚ฐ์„ ์ •์ƒํ™”์‹œํ‚จ๋‹ค๋Š” ๋‚ด์šฉ์œผ๋กœ - ํ†ต์ผ์ „์„ ์ „์ˆ ์— ๋”ฐ๋ฅธ ์‚ฌ๋žŒ ์กฐ์ง์‚ฌ์—…์„ ๊ฐ•์กฐํ•˜๋ฉด์„œ, โ€˜๋…ธ๋™๋‹น์˜ ๊ฒฐ์ •์€ ์ •์น˜์  ์ƒ๋ช…์„ ๊ฑธ๊ณ  ์‹ ๋…๊ณผ ์˜์ง€๋กœ ์ง€์ผœ๋‚˜๊ฐ€์•ผ ํ•œ๋‹คโ€™๋ผ๋ฉฐ ๋ถํ•œ์‹ โ€˜๊ฒฐ์‚ฌ๊ด€์ฒ  ์ •์‹ โ€™ ๋“ฑ์„ ์„ ์ „ํ•˜๊ณ  ์žˆ๋‹ค. ใ€Š[mp3] ํด๋” ์ค‘ [mp3]์กฐ์ธ๊ตฐ๊ฐ€ ํด๋” ๋‚ด โ€˜์ฒœ๋งŒ์ด ์ดํญํƒ„ ๋˜๋ฆฌ๋ผโ€™์˜ ์ฃผ์š” ๋‚ด์šฉใ€‹ - ๋ถํ•œ์˜ ์„ ๊ตฐ์ •์น˜๋ฅผ ๋ฏธํ™”ยท์ฐฌ์–‘ํ•˜๋Š” ๋ถํ•œ ๋…ธ๋ž˜๋กœ ๊ทธ ๊ฐ€์‚ฌ ๋‚ด์šฉ์€ โ€˜1. ๊ทธ ์–ด๋–ค ๋ด‰์‡„๋„ ์••๋ ฅ๋„ ์šฐ๋ฆฌ๋ฅผ ๋†€๋ž˜์ง€ ๋ชปํ•˜๋ฆฌ / ๊ฐ•์ฒ ์˜ ๋ น์žฅ์ด ๊ณ„์‹œ์—ฌ ๋ฐฐ์‹ฌ ๋“ ๋“ ํ•˜๋‹ค / ์šฐ๋ฆฐ ๋นˆ๋ง ์•ˆ ํ•œ๋‹ค ์šฐ๋ฆฐ ๋นˆ๋ง ์•ˆ ํ•œ๋‹ค / ์ฒœ๋งŒ์ด ์ดํญํƒ„ ๋˜๋ฆฌ๋ผ ์กฐ๊ตญ๊ณผ ์ธ๋ฏผ ์œ„ํ•˜์—ฌ - 2. ๊ทธ ์–ด๋–ค ๋ด‰์‡„๋„ ์••๋ ฅ๋„ ์šฐ๋ฆฌ๋ฅผ ๋†€๋ž˜์ง€ ๋ชปํ•˜๋ฆฌ / ๋‹จ๊ฒฐ๋œ ์ธ๋ฏผ์ด ์žˆ๋Š” ํ•œ ์Šน๋ฆฌ๋งŒ ์žˆ๋‹ค / ์šฐ๋ฆฐ ๋นˆ๋ง ์•ˆ ํ•œ๋‹ค / ์šฐ๋ฆฐ ๋นˆ๋ง ์•ˆ ํ•œ๋‹ค / ์ฒœ๋งŒ์ด ์ดํญํƒ„ ๋˜๋ฆฌ๋ผ / ์กฐ๊ตญ๊ณผ ์ธ๋ฏผ ์œ„ํ•˜์—ฌ - 3. ๊ทธ ์–ด๋–ค ๋ด‰์‡„๋„ ์••๋ ฅ๋„ ์šฐ๋ฆฌ๋ฅผ ๋†€๋ž˜์ง€ ๋ชปํ•˜๋ฆฌ / ๊ณต๊ฒฉ๊ณผ ๋ฐฉ์–ด๋ฅผ ๋‹ค ๊ฐ–์ถ˜ ๊ฐ•๊ตฐ์ด ์žˆ๋‹ค / ์šฐ๋ฆฐ ๋นˆ๋ง ์•ˆ ํ•œ๋‹ค / ์šฐ๋ฆฐ ๋นˆ๋ง ์•ˆ ํ•œ๋‹ค / ์ฒœ๋งŒ์ด ์ดํญํƒ„ ๋˜๋ฆฌ๋ผ / ์กฐ๊ตญ๊ณผ ์ธ๋ฏผ ์œ„ํ•˜์—ฌ / ์ฒœ๋งŒ์ด ์ดํญํƒ„ ๋˜๋ฆฌ๋ผ / ์ดํญํƒ„ ๋˜๋ฆฌ๋ผโ€˜๋ผ๊ณ  ๊ตฌ์„ฑ๋˜์–ด ์žˆ๋‹ค. ์ด์™€ ๊ฐ™์€ ํ•˜๋“œ๋””์Šคํฌ(HD 500G) ํ˜•์‹์˜ ๋””์ง€ํ„ธ์ €์žฅ๋งค์ฒด๋Š” ๊น€์ผ์„ฑ์˜ ์ „ ๊ฐ€๊ณ„๋ฅผ ํ•ญ์ผ ๋…๋ฆฝํˆฌ์‚ฌ๋กœ ๋ฌ˜์‚ฌํ•˜๊ณ  ๊น€์ผ์„ฑ์˜ ํ•ญ์ผ๋ฌด์žฅํˆฌ์Ÿ์„ ๋ฏธํ™”ยท์ฐฌ์–‘ํ•˜๋Š” ๋“ฑ ๊น€์ผ์„ฑ ๊ฐœ์ธ๊ณผ ๊ฐ€๊ณ„๋ฅผ ์šฐ์ƒํ™”ํ•˜๋Š” ์ทจ์ง€์˜ ๋‚ด์šฉ, 6ยท25์ „์Ÿ์„ ๋ฏธ๊ตญ์— ์˜ํ•œ ์นจ๋žต์ „์Ÿ์ธ ์–‘ ์™œ๊ณกํ•˜๋ฉด์„œ ๊น€์ผ์„ฑ์ด ๋ฏธ๊ตญ์˜ ์นจ๋žต์„ ๋ง‰์•„๋‚ธ ์กฐ๊ตญํ•ด๋ฐฉ์ „์Ÿ์ด์—ˆ๋‹ค๋Š” ์ทจ์ง€๋กœ ์„ ์ „ํ•˜๋Š” ๋‚ด์šฉ, ์กฐ์„ ๋กœ๋™๋‹น์˜ ์ผ๋‹น๋…์žฌ๋ฅผ ์ •๋‹นํ™”ํ•˜๋ฉด์„œ ๋Œ€๋‚จํ˜๋ช…๋…ธ์„ ์„ ์„ ์ „ยท์„ ๋™ํ•˜๋Š” ์ทจ์ง€์˜ ๋‚ด์šฉ, ๋ถํ•œ์‹ ์‚ฌํšŒ์ฃผ์˜๊ฐ€ ์ฃผ์ฒด์‚ฌ์ƒ์œผ๋กœ ๋ฌด์žฅํ•œ ๋ถํ•œ ์ธ๋ฏผ๋“ค์— ์˜ํ•ด ์ ˆ๋Œ€์  ์ง€์ง€๋ฅผ ๋ฐ›๊ณ  ์žˆ๋‹ค๋Š” ์ทจ์ง€๋กœ ๋ถํ•œ ์‚ฌํšŒ์ฃผ์˜์ฒด์ œ์˜ ์šฐ์›”์„ฑ ๋ฐ ํ˜๋ช…์  ์ •๋‹น์„ฑ ๋“ฑ์„ ๊ฐ•์กฐํ•˜๋Š” ์ทจ์ง€์˜ ๋‚ด์šฉ, ๊น€์ผ์„ฑ ์ฃผ์ฒด์‚ฌ์ƒ์˜ ์šฐ์›”์„ฑ์„ ๊ฐ•์กฐํ•˜๋ฉด์„œ ๋ฏธ ์ œ๊ตญ์ฃผ์˜์— ์˜ˆ์†๋œ ๋‚จํ•œ ๋ฏผ์ค‘์„ ํ•ด๋ฐฉํ•˜๊ณ  ํ•œ๋ฐ˜๋„์˜ ํ†ต์ผ์„ ์‹คํ˜„ํ•˜๊ธฐ ์œ„ํ•ด ๋‚จํ•œ ๋ฏผ์ค‘์„ ์ฃผ์ฒด์‚ฌ์ƒ์œผ๋กœ ์˜์‹ํ™”ํ•˜์—ฌ์•ผ ํ•œ๋‹ค๋Š” ์ทจ์ง€์˜ ๋‚ด์šฉ, ์ž๋ณธ์ฃผ์˜๊ฒฝ์ œ์ œ๋„๋Š” ์ธ๋ฏผ๋Œ€์ค‘์„ ์ˆ˜ํƒˆํ•˜๊ธฐ ์œ„ํ•œ ๋„๊ตฌ์— ๋ถˆ๊ณผํ•˜๊ณ  ์ฃผ์ฒด์‚ฌ์ƒ์ด ๊ทผ๊ฐ„์ด ๋œ ๋ถํ•œ์˜ ์‚ฌํšŒ์ฃผ์˜ ์ฒด์ œ๋Š” ๋ชจ๋“  ๊ตฌ์„ฑ์›๋“ค์ด ํ‰๋“ฑํ•˜๊ณ  ํ’์กฑํ•œ ๋ฌผ์งˆ์  ๋ฌธํ™”์ƒํ™œ์ด ๋ณด์žฅ๋˜๋Š” ์‚ฌํšŒ๋ผ๊ณ  ๋ฏธํ™”ํ•˜๋ฉด์„œ ๋ถํ•œ ์‚ฌํšŒ์ฃผ์˜ ์ฒด์ œ์™€ ์ฃผ์ฒด์‚ฌ์ƒ์„ ์ฐฌ์–‘ยท์„ ์ „ํ•˜๋Š” ์ทจ์ง€์˜ ๋‚ด์šฉ, ๊น€์ผ์„ฑยท๊น€์ •์ผ์ฃผ์˜๋ฅผ ์ถ”์ข…ํ•˜๋Š” ๋ถํ•œ ์‚ฌํšŒ์ฃผ์˜์ฒด์ œ์˜ ์šฐ์ˆ˜์„ฑ ๋ฐ ์ฃผ์ฒด์‚ฌ์ƒ๊ณผ ์„ ๊ตฐ์ •์น˜๋ฅผ ์„ ์ „ํ•˜๋Š” ์ทจ์ง€์˜ ๋‚ด์šฉ, ๋ถํ•œ์˜ ๋Œ€๋‚จํ˜๋ช…๋ก ์˜ ์ผํ™˜์œผ๋กœ ์ฃผํ•œ๋ฏธ๊ตฐ์˜ ์ฒ ์ˆ˜๋ฅผ ์ฃผ์žฅํ•˜๊ณ  ๋ถํ•œ์˜ ์—ฐ๋ฐฉ์ œ ํ†ต์ผ๋ฐฉ์‹์˜ ์ •๋‹น์„ฑ ๋“ฑ์„ ์„ ์ „ยท์„ ๋™ํ•˜๋Š” ์ทจ์ง€์˜ ๋‚ด์šฉ, ๊น€๋ถ€์ž์— ๋Œ€ํ•œ ์˜์›ํ•œ ์ถฉ์„ฑ์„ ๋งน์„ธํ•˜๋Š” ์ทจ์ง€์˜ ๋‚ด์šฉ์— ๊ด€ํ•œ ๊ฐ์ข… ๋ฌธ๊ฑด ๋ฐ ๋™์˜์ƒํŒŒ์ผ ๋“ฑ์„ ์ˆ˜๋กํ•˜๊ณ  ์žˆ๋‹ค. ์ด๋กœ์จ ํ”ผ๊ณ ์ธ์€ ๊ตญ๊ฐ€์˜ ์กด๋ฆฝยท์•ˆ์ „์ด๋‚˜ ์ž์œ ๋ฏผ์ฃผ์  ๊ธฐ๋ณธ์งˆ์„œ๋ฅผ ์œ„ํƒœ๋กญ๊ฒŒ ํ•œ๋‹ค๋Š” ์ •์„ ์•Œ๋ฉด์„œ ๋ฐ˜๊ตญ๊ฐ€๋‹จ์ฒด๋‚˜ ๊ทธ ๊ตฌ์„ฑ์› ๋˜๋Š” ๊ทธ ์ง€๋ น์„ ๋ฐ›์€ ์ž์˜ ํ™œ๋™์„ ์ฐฌ์–‘ยท๊ณ ๋ฌดยท์„ ์ „ ๋˜๋Š” ์ด์— ๋™์กฐํ•  ๋ชฉ์ ์œผ๋กœ ์œ„ ์ด์ ํ‘œํ˜„๋ฌผ์„ ์†Œ์ง€ํ•˜์˜€๋‹ค. โ…ก. 2016๊ณ ํ•ฉ558 [๋ชจ๋‘์‚ฌ์‹ค] ํ”ผ๊ณ ์ธ์€ 2014. 9. 26. ์„œ์šธ์ค‘์•™์ง€๋ฐฉ๋ฒ•์›์—์„œ ์ผ๋ฐ˜๊ตํ†ต๋ฐฉํ•ด, ์ง‘ํšŒ๋ฐ์‹œ์œ„์—๊ด€ํ•œ๋ฒ•๋ฅ ์œ„๋ฐ˜์ฃ„ ๋“ฑ์œผ๋กœ ๋ฒŒ๊ธˆ 500,000์›์„ ์„ ๊ณ ๋ฐ›์€ ์ „๋ ฅ์ด ์žˆ๋‹ค. ํ”ผ๊ณ ์ธ์˜ ๊ฒฝ๋ ฅ์€ ์œ„ 2016๊ณ ํ•ฉ538 ๋ฒ”์ฃ„์‚ฌ์‹ค์—์„œ ๊ธฐ์žฌํ•œ ๋ฐ”์™€ ๊ฐ™๋‹ค. [๋ฒ”์ฃ„์‚ฌ์‹ค] 1. ๋ถํ•œ๊ณผ โ€˜225๊ตญโ€™์˜ ์‹ค์ฒด ๊ฐ€. ๋ถํ•œ์˜ ๋ฐ˜๊ตญ๊ฐ€๋‹จ์ฒด์„ฑ ๋ถํ•œ์€ ๋Œ€ํ•œ๋ฏผ๊ตญ ์ •๋ถ€๋ฅผ ์ฐธ์นญํ•˜๊ณ  ๊ตญ๊ฐ€๋ฅผ ๋ณ€๋ž€ํ•  ๋ชฉ์ ์œผ๋กœ ํ™œ๋™ํ•˜๊ณ  ์žˆ๋Š” ๋ฐ˜๊ตญ๊ฐ€๋‹จ์ฒด๋กœ์„œ ํ•œ๋ฐ˜๋„ ์ ํ™”ํ†ต์ผ์„ ๊ธฐ๋ณธ๋ชฉํ‘œ๋กœ ์„ค์ •ํ•˜๊ณ , ์กฐ์„ ๋…ธ๋™๋‹น ๊ทœ์•ฝ ์„œ๋ฌธ์—์„œ โ€œ์กฐ์„ ๋…ธ๋™๋‹น์˜ ๋‹น๋ฉด ๋ชฉ์ ์€ ๊ณตํ™”๊ตญ ๋ถ๋ฐ˜๋ถ€์—์„œ ์‚ฌํšŒ์ฃผ์˜ ๊ฐ•์„ฑ๋Œ€๊ตญ์„ ๊ฑด์„คํ•˜๋ฉฐ ์ „๊ตญ์  ๋ฒ”์œ„์—์„œ ๋ฏผ์กฑํ•ด๋ฐฉ ๋ฏผ์ฃผ์ฃผ์˜ ํ˜๋ช…๊ณผ์—…์„ ์ˆ˜ํ–‰ํ•˜๋Š”๋ฐ ์žˆ์œผ๋ฉฐ, ์ตœ์ข… ๋ชฉ์ ์€ ์˜จ ์‚ฌํšŒ๋ฅผ ๊น€์ผ์„ฑ-๊น€์ •์ผ์ฃผ์˜ํ™”ํ•˜์—ฌ ์ธ๋ฏผ๋Œ€์ค‘์˜ ์ž์ฃผ์„ฑ์„ ์™„์ „ํžˆ ์‹คํ˜„ํ•˜๋Š”๋ฐ ์žˆ๋‹คโ€๋ผ๊ณ  ๊ทœ์ •ํ•˜๋Š” ๋“ฑ ๋งˆ๋ฅดํฌ์Šคยท๋ ˆ๋‹Œ์ฃผ์˜์˜ ๋ณ€ํ˜•์ธ โ€˜๊น€์ผ์„ฑ ๋…์žฌ์‚ฌ์ƒโ€™(์ฃผ์ฒด์‚ฌ์ƒ)์— ์ž…๊ฐํ•˜์—ฌ ๋ณ€์ฆ๋ฒ•์  ์œ ๋ฌผ๋ก ์— ๋”ฐ๋ฅธ ์—ญ์‚ฌํ•ด์„๊ณผ ๊ณ„๊ธ‰ํˆฌ์Ÿ์˜ ๊ด€์ ์—์„œ ํ•œ๊ตญ์˜ ์—ญ์‚ฌ๋ฅผ ์ง€๋ฐฐ๊ณ„๊ธ‰์— ๋Œ€ํ•œ ํ”ผ์ง€๋ฐฐ๊ณ„๊ธ‰์˜ ๊ณ„๊ธ‰ํˆฌ์Ÿ์œผ๋กœ ๊ทœ์ •ํ•˜๋Š” ํ•œํŽธ, ๋‚จํ•œ์‚ฌํšŒ๋Š” ๋ฏธ๊ตญ์˜ ์ œ๊ตญ์ฃผ์˜์  ๊ฐ•์  ํ•˜์— ๋ฏธ๊ตญ์ด ๋‚ด์„ธ์šด ํŒŒ์‡ผ์ •๊ถŒ์„ ํ†ตํ•˜์—ฌ ์ง€๋ฐฐ๋˜๊ณ  ์žˆ๋Š” ์‹๋ฏผ์ง€๋กœ์„œ ๋ชจ๋“  ์ธ๋ฏผ์ด ์ˆ˜ํƒˆ๋‹นํ•˜๊ณ  ์žˆ๋‹ค๊ณ  ์ฃผ์žฅํ•˜๊ณ  ์žˆ๋‹ค. ๋˜ํ•œ, ๋ถํ•œ์€ ๋‚จํ•œ์˜ ์–ต์••๋ฐ›๋Š” ๋ฏผ์ค‘์„ ํ•ด๋ฐฉํ•˜๊ณ  ํ•œ๋ฐ˜๋„ ํ‰ํ™”ํ†ต์ผ์„ ์ด๋ฃฉํ•˜๊ธฐ ์œ„ํ•ด์„œ๋Š” ์†Œ์œ„ โ€˜๋ฏผ์กฑํ•ด๋ฐฉ๋ฏผ์ค‘๋ฏผ์ฃผ์ฃผ์˜ํ˜๋ช…(NLPDR)โ€™ ์‚ฌ์—…์„ ์™„์ˆ˜ํ•˜์—ฌ์•ผ ํ•˜๋ฉฐ, ์ด๋ฅผ ์œ„ํ•ด์„œ๋Š” ๋…ธ๋™์ž, ๋†๋ฏผ, ๋„์‹œ๋นˆ๋ฏผ, ์ฒญ๋…„ํ•™์ƒ, ์ง„๋ณด์  ์ง€์‹์ธ ๋“ฑ ๋ฏธ์ œ์™€ ํŒŒ์‡ผ๊ถŒ๋ ฅ์— ์˜ํ•˜์—ฌ ์–ต์••๋ฐ›๊ณ  ์žˆ๋Š” ๋ชจ๋“  ๋ฏผ์ค‘์„ ๋ฏผ์กฑ์• , ๋ฏผ์ฃผ์ฃผ์˜, ๊ณ„๊ธ‰๊ต์–‘ ๋“ฑ์œผ๋กœ ์˜์‹ํ™”ํ•˜๊ณ , ์ง€๋„ํ•ต์‹ฌ์„ ์œก์„ฑยท๋ฐœ๊ตดํ•˜์—ฌ ์‚ฌํšŒ์ •์น˜์  ์ƒ๋ช…์ฒด์˜ ๋‡Œ์ˆ˜์ธ ์ˆ˜๋ น์˜ ์˜๋„ ์•„๋ž˜ ํ˜๋ช…์  ์˜๋ฆฌ์™€ ๋™์ง€์• ๋กœ ๊ฒฐํ•ฉ๋œ ๋ฏผ์กฑ๋ฏผ์ฃผํ†ต์ผ์ „์„ ์ฒด๋ฅผ ๊ฒฐ์„ฑํ•œ ๋‹ค์Œ, ํ•ฉ๋ฒ•ยท๋ฐ˜ํ•ฉ๋ฒ•ยท๋น„ํ•ฉ๋ฒ•์ ์ธ ๋ชจ๋“  ๋ฐฉ๋ฒ•์„ ๋™์›ํ•˜์—ฌ ๋ฏธ๊ตฐ์ฒ ์ˆ˜ ๋“ฑ ๋ฏธ์ œ ํƒ€๋„๋ฅผ ์œ„ํ•œ โ€˜๋ฐ˜๋ฏธ์ž์ฃผํ™” ํˆฌ์Ÿโ€™, ํŒŒ์‡ผ๊ถŒ๋ ฅ๊ณผ ๊ทธ๋“ค์˜ ๋ฏผ์ค‘ ์ง€๋ฐฐ๋„๊ตฌ์ธ ๊ตญ๊ฐ€๋ณด์•ˆ๋ฒ• ๋“ฑ ๊ฐ์ข… ์•…๋ฒ•์„ ์ฒ ํํ•˜๋Š” โ€˜๋ฐ˜ํŒŒ์‡ผ ๋ฏผ์ฃผํ™” ํˆฌ์Ÿโ€™์„ ์ „๊ฐœํ•จ์œผ๋กœ์จ ๋ฏธ์ œ ๋ฐ ํŒŒ์‡ผ๊ถŒ๋ ฅ์„ ํƒ€๋„ํ•œ ํ›„ ๋‚จํ•œ ๋‚ด์— ์†Œ์œ„ โ€˜์ž์ฃผ์ ์ธ ๋ฏผ์ฃผ์ •๋ถ€โ€™๋ฅผ ์ˆ˜๋ฆฝํ•˜์—ฌ ๋ฏผ์ค‘์„ ํ•ด๋ฐฉํ•˜๊ณ , ์ตœ์ข…์ ์œผ๋กœ ๋ถํ•œ์˜ ์—ฐ๋ฐฉ์ œ ํ†ต์ผ๋ก ์— ๋”ฐ๋ผ ์†Œ์œ„ โ€˜์ž์ฃผ์ ์ธ ํ‰ํ™”ํ†ต์ผโ€™์„ ์ด๋ฃฉํ•˜์—ฌ์•ผ ํ•œ๋‹ค๊ณ  ๋Š์ž„์—†์ด ์„ ์ „ยท์„ ๋™ํ•˜๊ณ  ์žˆ๋‹ค. ์•„์šธ๋Ÿฌ, ๋ถํ•œ์€ ๋‚จ๋ถํ•œ ํ†ต์ผ๋ฐฉ์•ˆ์œผ๋กœ 1๋ฏผ์กฑ 1๊ตญ๊ฐ€ 2์ฒด์ œ 2์ •๋ถ€์˜ ์†Œ์œ„ โ€˜๊ณ ๋ ค์—ฐ๋ฐฉ์ œ ํ†ต์ผ๋ฐฉ์•ˆโ€™์„ ๋‚ด์„ธ์›Œ ๊ทธ ์„ ๊ฒฐ ์กฐ๊ฑด์œผ๋กœ ๊ตญ๊ฐ€๋ณด์•ˆ๋ฒ• ํ์ง€, ํ‰ํ™”ํ˜‘์ • ์ฒด๊ฒฐ ๋ฐ ์ฃผํ•œ๋ฏธ๊ตฐ ์ฒ ์ˆ˜ ๋“ฑ์„ ๋‚ด์„ธ์šฐ๊ณ  ์žˆ์„ ๋ฟ ์•„๋‹ˆ๋ผ, ์ œ3๊ตญ์˜ ๊ณต์ž‘๊ฑฐ์  ๋ฐ ํ•ด์™ธ ๋ฐ˜ํ•œ๊ต๋ฏผ๋‹จ์ฒด๋ฅผ ์ „์œ„์กฐ์ง์œผ๋กœ ํ•˜์—ฌ ์œ„์žฅ ํ‰ํ™”๊ณต์„ธ๋ฅผ ์ „๊ฐœํ•˜๋Š” ๋™์‹œ์— ๊ตญ๋‚ด ๋ฐ˜์ •๋ถ€ ์ธ์‚ฌ ๋ฐ ์šด๋™๊ถŒ ํ•™์ƒ๋“ค์„ ์ค‘์‹ฌ์œผ๋กœ ๋‚จํ•œ ๋‚ด โ€˜์ง€ํ•˜๋‹นโ€™์„ ๊ตฌ์ถ•ํ•˜๊ณ ์ž ์ง€์†์ ์œผ๋กœ ํš์ฑ…ํ•˜๊ณ  ์žˆ๋‹ค. ์ด์™€ ๊ฐ™์ด, ๋ถํ•œ์€ ํ•œ๋ฐ˜๋„ ์ ํ™”ํ†ต์ผ์„ ๊ธฐ๋ณธ๋ชฉํ‘œ๋กœ ํ•˜๊ณ  ์ฃผ์ฒด์‚ฌ์ƒ ๋ฐ ๋ฏผ์กฑํ•ด๋ฐฉ๋ฏผ์ค‘๋ฏผ์ฃผ์ฃผ์˜ํ˜๋ช…๋ก ์— ์ž…๊ฐํ•˜์—ฌ ๋Œ€๋‚จํ˜๋ช…์˜ ๊ตฌ์ฒด์ ์ธ ๊ฒฝ๋กœ๋ฅผ โ€˜์ฃผํ•œ๋ฏธ๊ตฐ ์ฒ ์ˆ˜, ๊ตญ๊ฐ€๋ณด์•ˆ๋ฒ• ํ์ง€โ†’์ž์ฃผ์  ๋ฏผ์ฃผ์ •๋ถ€ ์ˆ˜๋ฆฝโ†’์—ฐ๋ฐฉ์ œ ํ†ต์ผโ†’ํ•œ๋ฐ˜๋„ ์ ํ™”ํ†ต์ผโ€™๋กœ ์„ค์ •ํ•˜๊ณ , ์ด๋ฅผ ์‹คํ˜„์‹œํ‚ค๊ธฐ ์œ„ํ•ด ์ง€์†์ ์œผ๋กœ ์„ ์ „ยท์„ ๋™ํ•˜๊ณ  ์žˆ๋Š” ๋ฐ˜๊ตญ๊ฐ€๋‹จ์ฒด์ด๋‹ค. ๋‚˜. ๋ถํ•œ ๋Œ€๋‚จ๊ณต์ž‘์กฐ์ง โ€˜225๊ตญโ€™์˜ ์‹ค์ฒด ๋ถํ•œ ๋Œ€๋‚จ๊ณต์ž‘์กฐ์ง ์ค‘ ํ•˜๋‚˜์ธ โ€˜225๊ตญโ€™์€ 1962. 4. ๋‚จํ•œ ๋‚ด ์ง€ํ•˜๋‹น ๊ตฌ์ถ•์„ ๋ชฉ์ ์œผ๋กœ โ€˜๋Œ€๋‚จ์‚ฌ์—…์ด๊ตญโ€™์ด ์‹ ์„ค๋˜์–ด ๋Œ€๋‚จ๊ณต์ž‘๋ถ€์„œ์ธ โ€˜์—ฐ๋ฝ๋ถ€โ€™์™€ โ€˜๋ฌธํ™”๋ถ€โ€™๋ฅผ ๊ด€์žฅํ•˜๋‹ค 1974๋…„ ์ด๋ฅผ ํ†ตํ•ฉํ•˜์—ฌ ์ฐฝ์„ค๋œ โ€˜๋ฌธํ™”์—ฐ๋ฝ๋ถ€โ€™๋ฅผ ๋ชจํƒœ๋กœ ํ•˜์—ฌ, โ€˜๋Œ€๋‚จ์—ฐ๋ฝ๋ถ€โ€™, โ€˜์‚ฌํšŒ๋ฌธํ™”๋ถ€โ€™, โ€˜๋Œ€์™ธ์—ฐ๋ฝ๋ถ€โ€™๋กœ ๊ทธ ๋ช…์นญ์ด ๋ณ€๊ฒฝ๋˜์—ˆ๋‹ค๊ฐ€ ํ˜„์žฌ ๋…ธ๋™๋‹น ์‚ฐํ•˜ โ€˜ํ†ต์ผ์ „์„ ๋ถ€โ€™ ์†Œ์† โ€˜225๊ตญโ€™์œผ๋กœ ํ†ต์นญ๋˜๊ณ  ์žˆ๋‹ค. โ€˜225๊ตญโ€™์€ ํŠน์ˆ˜๊ต์œก์„ ํ†ตํ•˜์—ฌ ์–‘์„ฑํ•œ ๋Œ€๋‚จ๊ณต์ž‘์›์„ ์งํŒŒ ๋˜๋Š” ์šฐํšŒ ์นจํˆฌ์‹œ์ผœ ๋Œ€ํ•œ๋ฏผ๊ตญ ์ •๊ณ„ยท๊ตฐ๋ถ€ยท์‚ฌํšŒยท๋ฌธํ™”ยท์ข…๊ต๊ณ„ ๋ฐ ์‹œ๋ฏผ๋‹จ์ฒด ๋“ฑ ๊ฐ๊ณ„๊ฐ์ธต์˜ ์ธ์‚ฌ๋ฅผ ํฌ์„ญํ•˜์—ฌ ์กฐ์„ ๋…ธ๋™๋‹น์˜ ์ง€๋ น์— ๋”ฐ๋ฅด๋Š” ์ง€ํ•˜๋‹น ์กฐ์ง์„ ๊ฒฐ์„ฑํ•˜์—ฌ, ์ด๋ฅธ๋ฐ” โ€˜๋‚จ์กฐ์„  ํ˜๋ช…์˜ ๊ฒฐ์ •์  ์‹œ๊ธฐโ€™์— ์ง€ํ•˜๋‹น ์กฐ์ง์„ ํ˜๋ช…๋งค๊ฐœ์ฒด๋กœ ํ™œ์šฉํ•˜์—ฌ ๋Œ€ํ•œ๋ฏผ๊ตญ ์ฒด์ œ ์ „๋ณต์„ ๋ชฉํ‘œ๋กœ ํ™œ๋™ํ•˜๋ฉด์„œ ๊ตญ๊ฐ€๊ธฐ๋ฐ€์˜ ํƒ์ง€ยท์ˆ˜์ง‘, ๋ถํ•œ์ฒด์ œ์˜ ์šฐ์›”์„ฑ๊ณผ ๊น€์ผ์„ฑ ์ผ๊ฐ€ ์œ„๋Œ€์„ฑ ์„ ์ „ ๋ฐ ์š”์ธ์•”์‚ดยทํ…Œ๋Ÿฌ ๋“ฑ ๋Œ€๋‚จ๊ณต์ž‘ ์ž„๋ฌด๋ฅผ ์ˆ˜ํ–‰ํ•˜๊ณ  ์žˆ๋‹ค. 2. ํšŒํ•ฉยทํ†ต์‹ , ๊ธˆํ’ˆ์ˆ˜์ˆ˜, ํŽธ์˜์ œ๊ณต์˜ ์  ๊ฐ€. 2011. 4. 21.๊ฒฝ ํšŒํ•ฉ ํ”ผ๊ณ ์ธ์€ ๋ถํ•œ โ€˜225๊ตญโ€™ ์†Œ์† ๊ณต์ž‘์›์„ ๋งŒ๋‚˜๊ธฐ ์œ„ํ•˜์—ฌ, 2011. 4. 19.๊ฒฝ ์ธ์ฒœ๊ตญ์ œ๊ณตํ•ญ์—์„œ ์ค‘๊ตญ (๋ช…์นญ 42 ์ƒ๋žต)ํ•ญ๊ณต (ํŽธ๋ช… 1 ์ƒ๋žต)ํŽธ์„ ์ด์šฉ ์ค‘๊ตญ ๋Œ€๋ จ์œผ๋กœ ์ถœ๊ตญํ•˜์—ฌ, ๊ฐ™์€ ๋‚  12:05๊ฒฝ(์ค‘๊ตญ ํ˜„์ง€ ์‹œ๊ฐ) ์ค‘๊ตญ ๋Œ€๋ จ๊ณตํ•ญ์— ๋„์ฐฉํ•˜์˜€๋‹ค. ํ”ผ๊ณ ์ธ์€ 2011. 4. 21. 15:00๊ฒฝ ์ค‘๊ตญ ๋Œ€๋ จ์— ์žˆ๋Š” โ€˜์˜ฌ๋ฆผํ”ฝ๊ด‘์žฅโ€™์˜ ์˜ฌ๋ฆผํ”ฝ๊ธฐ๋…ํƒ‘ ์•ž์—์„œ, โ€˜225๊ตญโ€™ ์†Œ์† ๊ณต์ž‘์› ๊ณต์†Œ์™ธ 39๋ฅผ ๋งŒ๋‚˜ ๊ทธ์˜ ์•ˆ๋‚ด์— ๋”ฐ๋ผ ์˜ฌ๋ฆผํ”ฝ๊ด‘์žฅ ์ง€ํ•˜์ƒ๊ฐ€ ์ž…๊ตฌ๋กœ ์ด๋™ํ•˜์—ฌ ๊ทธ๊ณณ์—์„œ โ€˜225๊ตญโ€™ ์†Œ์† ๊ณต์ž‘์› โ˜†์ง€๋„์›(๊ฐ€๋ช… : โ˜†โ˜†โ˜†, โ˜†โ–’โ–’, โ˜†โˆˆโˆˆ)์„ ๋งŒ๋‚˜, ํ•จ๊ป˜ ํƒ์‹œ๋กœ ์ค‘์‚ฐ๊ด‘์žฅ ์ธ๊ทผ์— ์žˆ๋Š” โ€˜(๋ช…์นญ 43 ์ƒ๋žต)์ฃผ์ โ€™์œผ๋กœ ์ด๋™ํ•œ ํ›„, ํ”ผ๊ณ ์ธ์€ ๊ณต์†Œ์™ธ 39์™€ ํ•จ๊ป˜ โ€˜(๋ช…์นญ 43 ์ƒ๋žต)์ฃผ์ โ€™ ์•ˆ์œผ๋กœ ๋“ค์–ด๊ฐ€ ๋Œ€ํ™”๋ฅผ ๋‚˜๋ˆ„์—ˆ๊ณ , โ˜†์ง€๋„์›์€ ์ฃผ๋ณ€์—์„œ ๋Œ€๊ธฐํ•˜์˜€๋‹ค. ๊ณ„์†ํ•˜์—ฌ, ํ”ผ๊ณ ์ธ์€ ๊ฐ™์€ ๋‚  16:10๊ฒฝ ๊ณต์†Œ์™ธ 39์™€ ํ•จ๊ป˜ โ€˜(๋ช…์นญ 43 ์ƒ๋žต)์ฃผ์ โ€™์—์„œ ๋‚˜์™€ ๋ฐ–์—์„œ ๋Œ€๊ธฐํ•˜๊ณ  ์žˆ๋˜ โ˜†์ง€๋„์›๊ณผ ํ•จ๊ป˜ ์œ„ ์ค‘์‚ฐ๊ด‘์žฅ ์ธ๊ทผ์— ์žˆ๋Š” โ€˜(๋ช…์นญ 44 ์ƒ๋žต)โ€™ 2์ธต์œผ๋กœ ์ด๋™ํ•œ ํ›„ ๊ฐ™์€ ๋‚  23:55๊ฒฝ๊นŒ์ง€ โ˜†์ง€๋„์› ๋ฐ ๊ณต์†Œ์™ธ 39์™€ ํ•จ๊ป˜ ๋‚จํ•œ ๋‚ด ์ •์„ธ ๋“ฑ์— ๊ด€ํ•˜์—ฌ ์ด์•ผ๊ธฐ๋ฅผ ๋‚˜๋ˆ„์—ˆ๋‹ค. ์ดํ›„ ํ”ผ๊ณ ์ธ์€ 2011. 4. 23.๊ฒฝ ์ค‘๊ตญ ๋Œ€๋ จ๊ณตํ•ญ์—์„œ (ํŽธ๋ช… 2 ์ƒ๋žต)ํŽธ์„ ์ด์šฉํ•˜์—ฌ ์ธ์ฒœ๊ตญ์ œ๊ณตํ•ญ์œผ๋กœ ์ž…๊ตญํ•˜์˜€๋‹ค. ์ด๋กœ์จ ํ”ผ๊ณ ์ธ์€ ๊ตญ๊ฐ€์˜ ์กด๋ฆฝยท์•ˆ์ „์ด๋‚˜ ์ž์œ ๋ฏผ์ฃผ์  ๊ธฐ๋ณธ์งˆ์„œ๋ฅผ ์œ„ํƒœ๋กญ๊ฒŒ ํ•œ๋‹ค๋Š” ์ •์„ ์•Œ๋ฉด์„œ ๋ฐ˜๊ตญ๊ฐ€๋‹จ์ฒด ๊ตฌ์„ฑ์› ๋˜๋Š” ๊ทธ ์ง€๋ น์„ ๋ฐ›์€ ์ž์™€ ํšŒํ•ฉํ•˜์˜€๋‹ค. ๋‚˜. 2011. 11.๊ฒฝ ํ†ต์‹ ์—ฐ๋ฝ ํ”ผ๊ณ ์ธ์€ 2011. 11.๊ฒฝ ๋ถˆ์ƒ์˜ ์žฅ์†Œ์—์„œ ์„ฑ๋ช…์„ ์•Œ ์ˆ˜ ์—†๋Š” ์ž๋กœ๋ถ€ํ„ฐ ๋ถํ•œ โ€˜225๊ตญโ€™ ์†Œ์† ๊ณต์ž‘์›์ด ๋ณด๋‚ด๋Š” ์•”ํ˜ธํ™” ๋ฐ ๋ณตํ˜ธํ™” ๋„๊ตฌ์ธ ์Šคํ…Œ๊ฐ€๋…ธ๊ทธ๋ผํ”ผ ํ”„๋กœ๊ทธ๋žจ ๋ฐ ์Šคํ…Œ๊ฐ€๋…ธ๊ทธ๋ผํ”ผ๋กœ ์•”ํ˜ธํ™”๋œ โ€˜info.docxโ€™ ํŒŒ์ผ์„ ์ „๋‹ฌ๋ฐ›์•˜๋‹ค. ์œ„ ์Šคํ…Œ๊ฐ€๋…ธ๊ทธ๋ผํ”ผ ํ”„๋กœ๊ทธ๋žจ์€, ๋ฌธ์ž๋ฅผ ์ˆซ์ž๋กœ ์ž๋™ ๋ณ€ํ™˜์‹œ์ผœ์ฃผ๋Š” ๋‚œ์ˆ˜๋ณ€ํ™˜ ํ”„๋กœ๊ทธ๋žจ๊ณผ ๊ทธ ๊ฒฐ๊ณผ๋ฅผ ์ปค๋ฒ„ํŒŒ์ผ๋กœ ์œ„์žฅํ•ด์ฃผ๋Š” ํ”„๋กœ๊ทธ๋žจ์ด๊ณ , ์Šคํ…Œ๊ฐ€๋…ธ๊ทธ๋ผํ”ผ๋กœ ์•”ํ˜ธํ™”๋œ โ€˜๊ตญ์™ธ(๊ตญ๋‚ด)์—ฌํ–‰์‹ ์ฒญ ๋ฐ ๊ณ„์•ฝ์„œโ€™๋ผ๋Š” ์ œ๋ชฉ์˜ โ€˜info.docxโ€™ ํŒŒ์ผ์—๋Š” ๊ตฌ์ฒด์ ์ธ ํ”„๋กœ๊ทธ๋žจ ์‚ฌ์šฉ์„ค๋ช…์„œ, ์ด๋ฉ”์ผ์ฃผ์†Œ ๊ต์ฒด์ด์šฉ๊ณผ ๊ด€๋ จ๋œ ์•ฝ์†์‚ฌํ•ญ ๋“ฑ์ด ๊ธฐ์žฌ๋˜์–ด ์žˆ์—ˆ๋Š”๋ฐ, ๊ทธ ๊ตฌ์ฒด์ ์ธ ๋‚ด์šฉ์€ ๋‹ค์Œ๊ณผ ๊ฐ™๋‹ค.
1. 1์ฐจ ์ž‘์—…(1๋ฒˆ ํ”„๋กœ๊ทธ๋žจ ์‚ฌ์šฉ์„ค๋ช…์„œ)
- ํ”„๋กœ๊ทธ๋žจ์ด์šฉ์„ ์œ„ํ•œ ์‚ฌ์ „์ค€๋น„
๋ณด๋‚ด๋ ค๋Š” ๋ฌธ์„œ๋ฅผ ๋งŒ๋“ค๋˜ ์šฉ๋Ÿ‰์ด 5K๊ฐ€ ๋„˜์ง€ ๋ง์•„์•ผ ํ•œ๋‹ค.
๋งŒ์•ฝ 5K๊ฐ€ ๋„˜๋Š” ๊ฒฝ์šฐ 1๋ฒˆ ์ž‘์—…์ด ๋๋‚œ ๋‹ค์Œ ์••์ถ•ํ”„๋กœ๊ทธ๋žจ์œผ๋กœ ์••์ถ•ํ•˜์—ฌ 5K(ZIP)์ดํ•˜ ๋˜๊ฒŒ ํ•  ๊ฒƒ
- ๊ธฐ๋™๋ฐฉ๋ฒ•
SISVGADRIVER - AGPPACK ํด๋”์—์„œ README.TXTํŒŒ์ผ์„ ํŽผ์นœ๋‹ค.
๋ฉ”๋ชจ์žฅ์— ํŽผ์ณ์ง„ ๋‚ด์šฉ์—์„œ ใ€ŠTHIS PACKAGE INCLUDE FOUR PARTS.ใ€‹๋ฌธ์ž์—ด์„ ๋ณต์‚ฌํ•œ๋‹ค.
๋‹ค์Œ SETUP.EXE ํŒŒ์ผ์„ ๊ธฐ๋™ํ•œ๋‹ค.
ใ€Š์ž ์‹œ๋งŒ ๊ธฐ๋‹ค๋ ค ์ฃผ์‹ญ์‹œ์š”ใ€‹๋ผ๋Š” ๋ฌธ์ž์—ด์ด ํ™”๋ฉด์— ๋‚˜์˜ค๋ฉด ์„ฑ๊ณต์ ์œผ๋กœ ๊ธฐ๋™๋œ ๊ฒƒ์„ ์˜๋ฏธํ•œ๋‹ค.
๋ฐœ์‹ 
OUT๋ฅผ ์„ ํƒํ•˜๊ณ  ID์— ์‹œ์ž‘๋ฒˆํ˜ธ๋ฅผ ๋„ฃ์–ด์ค€๋‹ค.
๋‹ค์Œ BROWS ๋‹จ์ถ”๋ฅผ ๋ˆŒ๋Ÿฌ ๊ธฐ์•ˆ๋ฌธํŒŒ์ผ์„ ์ฝ์–ด๋“ค์ด๊ณ  START ๋‹จ์ถ”๋ฅผ ๋ˆ„๋ฅธ๋‹ค. ๊ทธ๋Ÿฌ๋ฉด ๋ฐœ์‹ ์ด ์ง„ํ–‰๋˜๊ณ  ์ž‘์—…์ด ๋๋‚œ๋‹ค.
ํ”„๋กœ๊ทธ๋žจ์˜ ์•„๋ž˜ ๋ถ€๋ถ„์— ๋‹ค์Œ๋ฒˆ ์‚ฌ์šฉํ•  ์‹œ์ž‘๋ฒˆํ˜ธ๊ฐ€ ๋‚˜ํƒ€๋‚œ๋‹ค.(์˜ˆ 82๊ฐ€ ๋‚˜์™”์œผ๋ฉด 82๋ฒˆ ์ค„ ๊ฒƒ)
ํŒŒ์ผ์€ ๊ธฐ์•ˆ๋ฌธ์ด ์žˆ๋˜ ํด๋”์— RESUIT.TXT๋กœ ์ƒ์„ฑ๋œ๋‹ค.
IN์„ ์„ ํƒํ•˜๊ณ  BROWS ๋‹จ์ถ”๋ฅผ ๋ˆŒ๋Ÿฌ ์ž‘์—…ํ•œ ํŒŒ์ผ์„ ์ฝ์–ด ๋“ค์ธ๋‹ค. ๋‹ค์Œ START ๋‹จ์ถ”๋ฅผ ๋ˆ„๋ฅธ๋‹ค.
์ฃผ์˜์ 
๋ฒˆํ˜ธ๋ฅผ ์ค‘๋ณต์‚ฌ์šฉํ•˜์ง€ ๋ง๋ฉฐ
์ค‘์š”ํ•œ ๋‚ด์šฉ(์ด๋ฆ„, ์—ฐ๋ น, ๋‹จ์ฒด์ด๋ฆ„, ์ง€์—ญ, ์ง€๋ช… ๋“ฑ๊ณผ ๊ฐ™์€ ๋‹จ์–ด๋Š” ๊บพ์‡ ๊ด„ํ˜ธ๋กœ ๋ฌถ์–ด 2์ค‘์œผ๋กœ ํ•  ์ˆ˜ ์žˆ๋‹ค.
2. 2์ฐจ ์ž‘์—…(2๋ฒˆ ํ”„๋กœ๊ทธ๋žจ ์‚ฌ์šฉ๋ฐฉ๋ฒ•์„œ)
ํ”„๋กœ๊ทธ๋žจ๊ธฐ๋™์„ ์œ„ํ•œ ์‚ฌ์ „์ค€๋น„
- 1๋ฒˆ ํ”„๋กœ๊ทธ๋žจ์˜ ์ž‘์—…์—์„œ ์–ป์–ด์ง„ 5K ๋„˜์ง€ ์•Š๋Š” ํ…์ŠคํŠธ๋ฌธ์„œ
- ์ด๋ฏธ ์ƒํ˜ธ๊ฐ„์— ์•ฝ์†ํ•˜์—ฌ ๊ฐ€์ง€๊ณ  ์žˆ๋Š” ๋ผ๋ฃจ(RAR)ํŒŒ์ผ
- ์•ฝ์†ํ•œ ๋น„๋ฐ€๋ฒˆํ˜ธ ๋ฐ ์‹œ์ž‘๋ฒˆํ˜ธ
(๋ณธ์‚ฌ๋Š” ๋ณต์žก์„ฑ์„ ํ”ผํ•˜๊ธฐ ์œ„ํ•˜์—ฌ A์˜ ์ƒ๋…„(1963)์„ ๋น„๋ฐ€๋ฒˆํ˜ธ๋กœ ์ •ํ•จ)
- ์›Œ๋“œ ๋„ํฌํ™”์ผ ์ค€๋น„
๋„ํฌํŒŒ์ผ์„ ์ค€๋น„ํ•  ๋•Œ WORD OPTION(์˜ต์…˜) - SAVE - ๋งˆ์ง€๋ง‰ ์„ธ ์นธ์„ ๋ชจ๋‘ ์ฒดํฌํ•ด ์ค€๋‹ค.
๋„ํฌํŒŒ์ผ์„œ์ฒด๋Š” WINDOWS ํ‘œ์ค€์„œ์ฒด๋Š” ์“ฐ์ง€ ๋ง๊ณ  ใ€Šํ•œ์ปด๋ฐ”ํƒ•ใ€‹, ใ€Šํ•œ์ปด๋‹์Œใ€‹์ด ๋˜๊ฒŒ ํ•˜์—ฌ์•ผ ํ•จ
์ตœ์ข…์ ์œผ๋กœ ์ž‘์—…์ด ๋๋‚˜๋ฉด ํ† ํฌํŒŒ์ผ์ด ์žˆ๋˜ ์œ„์น˜์— ๋ฐ‘ ๋‹ค์‹œ ( ) ํ‘œ์‹œ๊ฐ€ ๋ถ™์€ ํŒŒ์ผ์ด ์ƒˆ๋กœ ์ƒ๊ฒจ๋‚˜๋Š”๋ฐ ๋ฐ”๋กœ ์ด ํŒŒ์ผ์„ ๋ณด๋‚ด์ฃผ์–ด์•ผ ํ•œ๋‹ค.
ํ”„๋กœ๊ทธ๋žจ์˜ ๊ธฐ๋™
๋ณด๋ผ๋ฏธ๊ณ ๊ฐ๊ด€๋ฆฌ ๋‹ค๋ถˆํด๋ฆญ - ์‚ฌ์šฉ์„ค๋ช…์„œ, TXT ๋‹ค๋ถˆํด๋ฆญ - ๊ณ ๊ฐ๊ด€๋ฆฌ ํŒ๋งค๊ด€๋ฆฌ๋ถ€ํ„ฐ ํ•™๊ต์กธ์—… ๋“ฑ)์—์„œ ๊ณต๋ฐฑ๊ฑด๊นŒ์ง€ ์„ ํƒํ•˜๊ณ  ๋ณต์‚ฌํ•œ ๋‹ค์Œ AUTORUN.EXE๋ฅผ ๋‹ค๋ถˆํด๋ฆญํ•œ๋‹ค.
์—ฌ๊ธฐ์„œ ๋ณด๋ผ๋ฏธ๊ณ ๊ฐ๊ด€๋ฆฌ๋ฅผ ๋‹ค์‹œ ์ฐพ์•„ SETUP.IMX๋ฅผ ๋‹ค๋ถˆํด๋ฆญํ•˜๋ฉด ํ”„๋กœ๊ทธ๋žจ์ฐฝ์ด ๋œฌ๋‹ค.
(ํ”„๋กœ๊ทธ๋žจ ์ฐฝ์—์„œ ์ฒซ ๋ณด๋‚ด๊ธฐ, ๋‘ ๋ฒˆ์งธ ๋ฐ›๊ธฐ, ์นธ์€ ์‚ฌ์šฉ๋ฒˆํ˜ธ, ์นธ์€ ์ƒํ˜ธ ์•ฝ์†๋œ ๋น„๋ฐ€๋ฒˆํ˜ธ ๋„ฃ๋Š” ์นธ์ž„, ๊ทธ ์•„๋ž˜ ์ฒซ ๋„ค๋ชจ ์นธ์€ 1์ฐจ ์ž‘์—…ํ•œ ํŒŒ์ผ, ๋‘˜์งธ ์นธ์€ ์ƒํ˜ธ ๊ฐ€์ง€๊ณ  ์žˆ๋Š” RARํŒŒ์ผ, ์„ธ ๋ฒˆ์งธ ์นธ์€ DOCX๋ฅผ ๋„ฃ๋Š” ์นธ์ž„)
ํ•ด๋‹น ์ฐฝ์— ์š”๊ตฌํ•˜๋Š” ๊ฒƒ๋“ค์„ ๋„ฃ๊ณ  START๋ฅผ ํด๋ฆญํ•˜๋ฉด ์„ฑ๊ณต์ด๋ผ๋Š” ๋ฉ”์„ธ์ง€๊ฐ€ ๋‚˜์˜ด.
์ด์ƒ์˜ ์‚ฌ์šฉ์„ค๋ช…์„œ๋Š” ์ตœ๋Œ€ ๊ทน๋น„์ž๋ฃŒ์ด๋ฏ€๋กœ A๊ฐ€ ์™„์ „ํžˆ ์ˆ™์ง€ํ•œ ๋‹ค์Œ์—๋Š” ์‚ญ์ œํ•˜์—ฌ์•ผ ํ•˜๋ฉฐ ๋‚ด์šฉ์„ ๋‹ด์•˜๋˜ USB๋Š” ๋ฌผ๋ฆฌ์ ์œผ๋กœ ํŒŒ๊ดดํ•˜์—ฌ์•ผ ํ•  ๊ฒƒ์ž„.
3. ์ด๋ฉ”์ผ์ฃผ์†Œ ๊ต์ฒด์ด์šฉ๊ณผ ๊ด€๋ จํ•œ ์•ฝ์†์‚ฌํ•ญ
๋ณด์•ˆ๊ณผ ์•ˆ์ „์„ ์œ„ํ•ด ๋ณธ์‚ฌ์™€ ์ง€์‚ฌ๊ฐ„์˜ ์ด๋ฉ”์ผ์ฃผ์†Œ๋ฅผ ๋ถ„๊ธฐ๋งˆ๋‹ค ๋ฐ”๊พธ์–ด ์‚ฌ์šฉํ•˜๊ฒ ์Œ.
๋ถ„๊ธฐ๋งˆ๋‹ค ์ด์šฉํ•  ์ด๋ฉ”์ผ์ฃผ์†Œ๋ฅผ ๊ทธ๋•Œ๊ทธ๋•Œ ๋ณด๋‚ด์ฃผ์–ด ์ด์šฉํ•˜๋ฉด ์ข‹๊ฒ ์œผ๋‚˜ ์ฃผ์†Œ๋ฅผ ์ธํ„ฐ๋„ท์ƒ์œผ๋กœ ์ฃผ๊ณ ๋ฐ›๋Š” ๊ฒƒ์€ ์œ„ํ—˜ํ•˜๋‹ค๊ณ  ํŒ๋‹จ๋˜์—ฌ ์ธํŽธ์œผ๋กœ ๋ณด๋ƒ„.
๋ถ„๊ธฐ๋ณ„๋กœ ๊ต์ฒด์‚ฌ์šฉ ํ•  ์ด๋ฉ”์ผ์ฃผ์†Œ
- 2011๋…„ 12์›”
์ •์ƒ์„  ID (์˜๋ฌธ ID 3 ์ƒ๋žต)@โ—Žโ—Žโ—Žโ—Žโ—Ž.CN, P (์˜๋ฌธ ๋น„๋ฐ€๋ฒˆํ˜ธ 3 ์ƒ๋žต)
[ID (ํ•œ๊ธ€ ID 3 ์ƒ๋žต)@โ—Žโ—Žโ—Žโ—Žโ—Ž.CN P (ํ•œ๊ธ€ ๋น„๋ฐ€๋ฒˆํ˜ธ 3 ์ƒ๋žต)]
์˜ˆ๋น„์„  ID (์˜๋ฌธ ID 3 ์ƒ๋žต)@โ–ฝโ–ฝโ–ฝโ–ฝ.COM P (์˜๋ฌธ ๋น„๋ฐ€๋ฒˆํ˜ธ 3 ์ƒ๋žต)
[ID (ํ•œ๊ธ€ ID 3 ์ƒ๋žต)@โ–ฝโ–ฝโ–ฝโ–ฝ.COM. P (ํ•œ๊ธ€ ๋น„๋ฐ€๋ฒˆํ˜ธ 3 ์ƒ๋žต)]
- 2012๋…„ 1์›”-3์›”๊นŒ์ง€
์ •์ƒ์„  ID (์˜๋ฌธ ID 4 ์ƒ๋žต)@โ—Žโ—Žโ—Žโ—Žโ—Ž.CN, P (์˜๋ฌธ ๋น„๋ฐ€๋ฒˆํ˜ธ 4 ์ƒ๋žต)
[ID (ํ•œ๊ธ€ ID 4 ์ƒ๋žต)@โ—Žโ—Žโ—Žโ—Žโ—Ž.CN P (ํ•œ๊ธ€ ๋น„๋ฐ€๋ฒˆํ˜ธ 4 ์ƒ๋žต)]
์˜ˆ๋น„์„  ID (์˜๋ฌธ ID 4 ์ƒ๋žต)@โ–ฝโ–ฝโ–ฝโ–ฝ.COM P (์˜๋ฌธ ๋น„๋ฐ€๋ฒˆํ˜ธ 4 ์ƒ๋žต)
[ID (ํ•œ๊ธ€ ID 4 ์ƒ๋žต)@โ–ฝโ–ฝโ–ฝโ–ฝ.COM. P (ํ•œ๊ธ€ ๋น„๋ฐ€๋ฒˆํ˜ธ 4 ์ƒ๋žต)]
- 2012๋…„ 4์›”-6์›”๊นŒ์ง€
์ •์ƒ์„  ID (์˜๋ฌธ ID 5 ์ƒ๋žต)@โ—Žโ—Žโ—Žโ—Žโ—Ž.CN, P (์˜๋ฌธ ๋น„๋ฐ€๋ฒˆํ˜ธ 5 ์ƒ๋žต)
[ID (ํ•œ๊ธ€ ID 5 ์ƒ๋žต)@โ—Žโ—Žโ—Žโ—Žโ—Ž.CN P (ํ•œ๊ธ€ ๋น„๋ฐ€๋ฒˆํ˜ธ 5 ์ƒ๋žต)]
์˜ˆ๋น„์„  ID (์˜๋ฌธ ID 5 ์ƒ๋žต)@โ–ฝโ–ฝโ–ฝโ–ฝ.COM P (์˜๋ฌธ ๋น„๋ฐ€๋ฒˆํ˜ธ 5 ์ƒ๋žต)
[ID (ํ•œ๊ธ€ ID 5 ์ƒ๋žต)@โ–ฝโ–ฝโ–ฝโ–ฝ.COM. P (ํ•œ๊ธ€ ๋น„๋ฐ€๋ฒˆํ˜ธ 5 ์ƒ๋žต)]
- 7์›”-9์›”๊นŒ์ง€
์ •์ƒ์„  ID (์˜๋ฌธ ID 6 ์ƒ๋žต)@โ—Žโ—Žโ—Žโ—Žโ—Ž.CN, P (์˜๋ฌธ ๋น„๋ฐ€๋ฒˆํ˜ธ 6 ์ƒ๋žต)
[ID (ํ•œ๊ธ€ ID 6 ์ƒ๋žต)@โ—Žโ—Žโ—Žโ—Žโ—Ž.CN P (ํ•œ๊ธ€ ๋น„๋ฐ€๋ฒˆํ˜ธ 6 ์ƒ๋žต)]
์˜ˆ๋น„์„  ID (์˜๋ฌธ ID 6 ์ƒ๋žต)@โ–ฝโ–ฝโ–ฝโ–ฝ.COM P (์˜๋ฌธ ๋น„๋ฐ€๋ฒˆํ˜ธ 6 ์ƒ๋žต)
[ID (ํ•œ๊ธ€ ID 6 ์ƒ๋žต)@โ–ฝโ–ฝโ–ฝโ–ฝ.COM. P (ํ•œ๊ธ€ ๋น„๋ฐ€๋ฒˆํ˜ธ 6 ์ƒ๋žต)]
- 10์›”-12์›”๊นŒ์ง€
์ •์ƒ์„  ID (์˜๋ฌธ ID 1 ์ƒ๋žต)@โ—Žโ—Žโ—Žโ—Žโ—Ž.CN, P (์˜๋ฌธ ๋น„๋ฐ€๋ฒˆํ˜ธ 1 ์ƒ๋žต)
[ID (ํ•œ๊ธ€ ID 1 ์ƒ๋žต)@โ—Žโ—Žโ—Žโ—Žโ—Ž.CN P (ํ•œ๊ธ€ ๋น„๋ฐ€๋ฒˆํ˜ธ 1 ์ƒ๋žต)]
์˜ˆ๋น„์„  ID (์˜๋ฌธ ID 1 ์ƒ๋žต)@โ–ฝโ–ฝโ–ฝโ–ฝ.COM P (์˜๋ฌธ ๋น„๋ฐ€๋ฒˆํ˜ธ 1 ์ƒ๋žต)
[ID (ํ•œ๊ธ€ ID 1 ์ƒ๋žต)@โ–ฝโ–ฝโ–ฝโ–ฝ.COM. P (ํ•œ๊ธ€ ๋น„๋ฐ€๋ฒˆํ˜ธ 1 ์ƒ๋žต)]
๋งŒ์•ฝ ๋ถ„๊ธฐ๋งˆ๋‹ค ๊ต์ฒด์‚ฌ์šฉํ•  ๋•Œ ํ•ด๋‹น ์ด๋ฉ”์ผ์ฃผ์†Œ๋“ค์˜ ์‚ฌ์šฉ์ด ๋ถˆ๊ฐ€๋Šฅํ•œ ๊ฒฝ์šฐ ์ด๋ฏธ ์‚ฌ์šฉํ•˜๊ณ  ์žˆ๋˜ ์ฃผ์†Œ๋กœ ์ œ๊ธฐ๋˜๋Š” ๋ฌธ์ œ๋“ค์„ ์ƒํ˜ธ ํ†ต์ง€ํ•ด์ฃผ์–ด ํ˜‘์˜๋Œ€์ฑ…ํ•ด ๋‚˜๊ฐ€๋„๋ก ํ•œ๋‹ค.
์ถ”์‹ 
์‹ ๋…„ ๋ฉ”์„ธ์ง€๋ฅผ 12์›” 12์ผ์— ๋ณด๋‚ด์ฃผ๊ธฐ ๋ฐ”๋žŒ.
์ด๋กœ์จ ํ”ผ๊ณ ์ธ์€ ๊ตญ๊ฐ€์˜ ์กด๋ฆฝยท์•ˆ์ „์ด๋‚˜ ์ž์œ ๋ฏผ์ฃผ์  ๊ธฐ๋ณธ์งˆ์„œ๋ฅผ ์œ„ํƒœ๋กญ๊ฒŒ ํ•œ๋‹ค๋Š” ์ •์„ ์•Œ๋ฉด์„œ ๋ฐ˜๊ตญ๊ฐ€๋‹จ์ฒด ๊ตฌ์„ฑ์› ๋˜๋Š” ๊ทธ ์ง€๋ น์„ ๋ฐ›์€ ์ž์™€ ํ†ต์‹  ๊ธฐํƒ€์˜ ๋ฐฉ๋ฒ•์œผ๋กœ ์—ฐ๋ฝํ•˜์˜€๋‹ค. ๋‹ค. 2011. 11.โˆผ12.๊ฒฝ ํ†ต์‹ ์—ฐ๋ฝ ๋ฐ ํŽธ์˜์ œ๊ณต ํ”ผ๊ณ ์ธ์€ 2011. 11. 29.๊ฒฝ ๋ถˆ์ƒ์˜ ์žฅ์†Œ์—์„œ โ€œ์—ฐ๋ฝ๋™์ง€ ๋ฌด์‚ฌ๊ท€ํ™˜ ์‚ฌ์‹ค, ์•”ํ˜ธํ”„๋กœ๊ทธ๋žจ ์‚ฌ์šฉ๋ฐฉ๋ฒ• ์ˆ™์ง€ ์‚ฌ์‹ค, (๋ช…์นญ 6 ์ƒ๋žต) ์—ฐ๊ตฌ์†Œ์กฐ์›์— ๋Œ€ํ•œ ๊ตญ์ •์› ์••์ˆ˜์ˆ˜์ƒ‰ ์ƒํ™ฉ, ์‹œ๋ฏผ๋‹จ์ฒด ์นจํˆฌํ˜„ํ™ฉ ๋ฐ ์กฐ์ง์‚ฌ์—… ๋ฐฉํ–ฅโ€ ๋“ฑ์ด ๊ธฐ์žฌ๋œ ๋Œ€๋ถ๋ณด๊ณ ๋ฌธ์„ ์ž‘์„ฑํ•˜์—ฌ ์ด๋ฅผ โ€˜๊ณ ๋‚œ์˜ ์ฃผ๊ฐ„ ์„ค๊ตโ€™๋กœ ์‹œ์ž‘๋˜๋Š” ํ•œ๊ธ€ํŒŒ์ผ๋กœ ์œ„์žฅํ•˜์—ฌ ์Šคํ…Œ๊ฐ€๋…ธ๊ทธ๋ผํ”ผ๋กœ ์•”ํ˜ธํ™”ํ•œ ํŒŒ์ผ โ€˜to you.docxโ€™์„ ์ƒ์„ฑํ•œ ๋‹ค์Œ, ๊ทธ ๋ฌด๋ ต ๋ถˆ์ƒ์˜ ๋ฐฉ๋ฒ•์œผ๋กœ ์„ฑ๋ช…์„ ์•Œ ์ˆ˜ ์—†๋Š” โ€˜225๊ตญโ€™ ์†Œ์† ๊ณต์ž‘์›์—๊ฒŒ ์ „๋‹ฌํ•˜์˜€๋Š”๋ฐ, ๊ทธ ๊ตฌ์ฒด์ ์ธ ๋‚ด์šฉ์€ ๋‹ค์Œ๊ณผ ๊ฐ™๋‹ค.
๋ณด๊ณ 
1. ์—ฐ๋ฝ๋™์ง€ ๋ฌด์‚ฌ๊ท€ํ™˜ํ–ˆ์Šต๋‹ˆ๋‹ค.
2. ์•”ํ˜ธํ”„๋กœ๊ทธ๋žจ ์‚ฌ์šฉ ๋ฐฉ๋ฒ•์„ ์ˆ™์ง€ํ–ˆ์Šต๋‹ˆ๋‹ค. ์ด ๋ณด๊ณ ๋ฅผ ๋ฐ›๋Š” ๊ฒƒ์ด ์ฆ๊ฑฐ๋กœ ๋ฉ๋‹ˆ๋‹ค.
3. 12์ผ ์‹ ๋…„๋ณด๊ณ  ๋“œ๋ฆฌ๊ฒ ์Šต๋‹ˆ๋‹ค.
4. ๊ฐ์ฒด๋Š” ์—†๋‹ค๋Š” ์›๋ฆฌ ๋”ฐ๋ผ ๋ณด์•ˆ์ฒด๊ณ„๋ฅผ ๊ณ„์† ๊ฐ•ํ™”ํ•˜๊ณ  ๋ฐœ์ „์‹œํ‚ค๊ณ  ์žˆ์Šต๋‹ˆ๋‹ค.
5. ์ตœ๊ทผ ์ „๋žต์ง€์—ญ์˜ ๊ต์–‘์กฐ์ง๋Œ€์ƒ[(๋ช…์นญ 6 ์ƒ๋žต) ์—ฐ๊ตฌ์†Œ์กฐ์›]์— ๋Œ€ํ•ด ๊ตญ์ •์›์ด ์••์ˆ˜์ˆ˜์ƒ‰์„ ํ–ˆ์Šต๋‹ˆ๋‹ค. ์—ฐํ–‰์€ ํ•˜์ง€ ์•Š๊ณ  ์••์ˆ˜์ˆ˜์ƒ‰๋งŒ ํ•œ ๊ฒƒ์ž…๋‹ˆ๋‹ค. ์ง์ ‘์ ์œผ๋กœ ์กฐ์ง์„ ๊ฒจ๋ƒฅํ•˜๊ณ  ์žˆ๋‹ค๊ณ  ํŒ๋‹จ๋˜์ง€๋Š” ์•Š์ง€๋งŒ ๋ณด์œ„๊ฐ•ํ™”์ฐจ์›์—์„œ ์ „๋žต๋™์ง€์™€ ํšŒํ•ฉ์„ ์ผ๋‹จ ์ค‘์ง€ํ•˜๊ณ  ์˜ˆ์˜์ฃผ์‹œํ•˜๊ณ  ์žˆ์Šต๋‹ˆ๋‹ค. โ™กโ™กโ™กโ™ก (๋ช…์นญ 7 ์ƒ๋žต)์—ฐ๋Œ€๋ฅผ ํ–ฅํ•˜๋Š” ๊ฒƒ์ด ์•„๋‹Œ๊ฐ€ ์ƒ๊ฐ๋˜์ง€๋งŒ ์ „๋žต์ง€์—ญ์˜ ์ค‘์š”์„ฑ์— ๋น„์ถ”์–ด ์ฃผ์˜ ๋˜ ์ฃผ์˜ํ•˜๊ณ  ์žˆ์Šต๋‹ˆ๋‹ค. ํ–ฅํ›„ ์ƒํ™ฉ๋ณด๊ณ  ๋“œ๋ฆฌ๊ฒ ์Šต๋‹ˆ๋‹ค.
6. ์‹œ๋ฏผ๋‹จ์ฒด์— ํฌ์น˜๋œ โ—๋™์ง€์™€์˜ ํšŒํ•ฉ์„ ํ–ˆ์Šต๋‹ˆ๋‹ค. (๋ช…์นญ 8 ์ƒ๋žต)์—ฐ๋Œ€์— ๊นŠ์ˆ™์ด ์นจํˆฌํ•ด ์ฃผ์š” ์‹œ๋ฏผ๋‹จ์ฒด๋ฅผ ๋Œ€์ƒ์œผ๋กœ ๊พธ์ค€ํžˆ ์‚ฌ์—…ํ•˜๊ณ  ์žˆ์œผ๋ฉฐ ๋ฐ•์‚ฌํ•™์œ„๋ฅผ ๋ฐ›์•„ ์ „๋ฌธ๊ฐ€๋กœ ์ž๋ฆฌ ์žก์œผ๋ ค ํ•˜๊ณ  ์žˆ์Šต๋‹ˆ๋‹ค. ์ตœ๊ทผ (๋ช…์นญ 9 ์ƒ๋žต)์—๋„ ๊ฐ€์ž…ํ–ˆ๊ณ  (๋ช…์นญ 8 ์ƒ๋žต)์—ฐ๋Œ€ ๋‚ด๋ถ€์—์„œ ์ Š์€ ํ™œ๋™๊ฐ€๋“ค์„ ๋Œ€์ƒ์œผ๋กœ ์กฐ์ง์‚ฌ์—…์„ ํ•˜๊ธฐ๋กœ ๋ฐฉํ–ฅ์„ ์žก์•˜์Šต๋‹ˆ๋‹ค. ์ •๊ธฐ์  ํšŒํ•ฉ ์ผ์ž์™€ ๋ฐฉ๋ฒ•์„ ์ •ํ–ˆ์Šต๋‹ˆ๋‹ค. ์ด์ƒ ๋ณด์ถฉ๋ณด๊ณ  ๋“œ๋ฆฝ๋‹ˆ๋‹ค.
7. ์ •์„ธ๋Š” ์—„ํ˜นํ•˜๊ณ  ํƒ„์••์€ ๊ณ„์†๋˜์ง€๋งŒ ์กฐ์ง์€ ์œ„๋Œ€ํ•œ ์žฅ๊ตฐ๋‹˜๊ณผ ์กด๊ฒฝํ•˜๋Š” ๋Œ€์žฅ๋‹˜์„ ๊ตณ๊ฒŒ ๋ฏฟ๊ณ  ์ ˆ๋Œ€์  ์ถฉ์„ฑ์„ ๋‹คํ•˜๊ธฐ ์œ„ํ•ด ์ตœ์„  ๋˜ ์ตœ์„ ์„ ๋‹คํ•˜๊ณ  ์žˆ์Šต๋‹ˆ๋‹ค.
2012. 12. 1.
์ด์ƒ ๋ณด๊ณ  ๋.
์ด๋กœ์จ ํ”ผ๊ณ ์ธ์€ ๊ตญ๊ฐ€์˜ ์กด๋ฆฝยท์•ˆ์ „์ด๋‚˜ ์ž์œ ๋ฏผ์ฃผ์  ๊ธฐ๋ณธ์งˆ์„œ๋ฅผ ์œ„ํƒœ๋กญ๊ฒŒ ํ•œ๋‹ค๋Š” ์ •์„ ์•Œ๋ฉด์„œ ๋ฐ˜๊ตญ๊ฐ€๋‹จ์ฒด ๊ตฌ์„ฑ์› ๋˜๋Š” ๊ทธ ์ง€๋ น์„ ๋ฐ›์€ ์ž์™€ ํ†ต์‹  ๊ธฐํƒ€์˜ ๋ฐฉ๋ฒ•์œผ๋กœ ์—ฐ๋ฝํ•˜๊ณ , โ€˜225๊ตญโ€™ ์†Œ์† ๊ณต์ž‘์›์—๊ฒŒ ํŽธ์˜๋ฅผ ์ œ๊ณตํ•˜์˜€๋‹ค. ๋ผ. 2012. 5. 31.๊ฒฝ ํšŒํ•ฉ ํ”ผ๊ณ ์ธ์€ ๋ถํ•œ โ€˜225๊ตญโ€™ ์†Œ์† ๊ณต์ž‘์›์„ ๋งŒ๋‚˜๊ธฐ ์œ„ํ•˜์—ฌ, 2012. 5. 29.๊ฒฝ ์ธ์ฒœ๊ตญ์ œ๊ณตํ•ญ์—์„œ (๋ช…์นญ 45 ์ƒ๋žต)ํ•ญ๊ณต (ํŽธ๋ช… 3 ์ƒ๋žต)ํŽธ์„ ์ด์šฉ ๋ฒ ํŠธ๋‚จ ํ˜ธ์น˜๋ฏผ์œผ๋กœ ์ถœ๊ตญํ•˜์—ฌ, ๊ฐ™์€ ๋‚  13:10๊ฒฝ(๋ฒ ํŠธ๋‚จ ํ˜„์ง€ ์‹œ๊ฐ) ๋ฒ ํŠธ๋‚จ โ€˜ํƒ„์†๋ˆ„ํŠธ๊ตญ์ œ๊ณตํ•ญโ€™์— ๋„์ฐฉํ•˜์˜€๋‹ค. ํ”ผ๊ณ ์ธ์€ 2012. 5. 31. 09:00๊ฒฝ ๋ฒ ํŠธ๋‚จ ํ˜ธ์น˜๋ฏผ์— ์žˆ๋Š” โ€˜๋…ธํ‹€๋‹ด์„ฑ๋‹นโ€™์˜ ์„ฑ๋ชจ๋งˆ๋ฆฌ์•„์ƒ ์•ž์—์„œ, โ€˜225๊ตญโ€™ ์†Œ์† ๊ณต์ž‘์› ๊ณต์†Œ์™ธ 39๋ฅผ ๋งŒ๋‚˜ ํ•จ๊ป˜ ์ธ๊ทผ์— ์žˆ๋Š” โ€˜(๋ช…์นญ 46 ์ƒ๋žต) ์ปคํ”ผ์ˆโ€™์œผ๋กœ ์ด๋™ํ•˜์—ฌ ๊ทธ๊ณณ์—์„œ ๊ธฐ๋‹ค๋ฆฌ๊ณ  ์žˆ๋˜ โ€˜225๊ตญโ€™ ์†Œ์† ๊ณต์ž‘์› โ˜†์ง€๋„์›์„ ๋งŒ๋‚ฌ๋‹ค. ํ”ผ๊ณ ์ธ์€ ๊ฐ™์€ ๋‚  09:28๊ฒฝ โ€˜(๋ช…์นญ 46 ์ƒ๋žต) ์ปคํ”ผ์ˆโ€™์—์„œ ๋จผ์ € ๋‚˜์™€ ํƒ์‹œ๋กœ โ€˜๋ฐํƒ๊ฑฐ๋ฆฌโ€™์— ์žˆ๋Š” โ€˜(๋ช…์นญ 47 ์ƒ๋žต) ํ˜ธํ…”โ€™๋กœ ์ด๋™ํ•˜์—ฌ ๋ฏธ์ƒ์˜ ํ˜ธ์‹ค๋กœ ๋“ค์–ด๊ฐ”๊ณ , ๊ฐ™์€ ๋‚  10:05๊ฒฝ ์œ„ ๋ฏธ์ƒ์˜ ํ˜ธ์‹ค๋กœ ๋’ค๋”ฐ๋ผ ๋“ค์–ด๊ฐ„ โ˜†์ง€๋„์›๊ณผ ๊ณต์†Œ์™ธ 39๋ฅผ ๋งŒ๋‚˜ ๊ทธ ๋ฌด๋ ต๋ถ€ํ„ฐ ๊ฐ™์€ ๋‚  19:00๊ฒฝ๊นŒ์ง€ ๋‚จํ•œ ๋‚ด ์ •์„ธ ๋“ฑ์— ๊ด€ํ•˜์—ฌ ์ด์•ผ๊ธฐ๋ฅผ ๋‚˜๋ˆ„์—ˆ๋‹ค. ๊ณ„์†ํ•˜์—ฌ, ํ”ผ๊ณ ์ธ์€ โ˜†์ง€๋„์› ๋ฐ ๊ณต์†Œ์™ธ 39์™€ ํ•จ๊ป˜ โ€˜(๋ช…์นญ 47 ์ƒ๋žต) ํ˜ธํ…”โ€™์„ ๋‚˜์™€ ํƒ์‹œ๋กœ ํ˜ธ์น˜๋ฏผ ๋™์ฝ”์ด์— ์žˆ๋Š” ํ•œ๊ตญ์‹๋‹น โ€˜(๋ช…์นญ 48 ์ƒ๋žต)โ€™์œผ๋กœ ์ด๋™ํ•˜์—ฌ, ๊ฐ™์€ ๋‚  20:00๊ฒฝ๋ถ€ํ„ฐ ๊ฐ™์€ ๋‚  21:00๊ฒฝ๊นŒ์ง€ ํ•จ๊ป˜ ์‹์‚ฌ๋ฅผ ํ•˜๋ฉด์„œ ๋‚จํ•œ ๋‚ด ์ •์„ธ ๋“ฑ์— ๊ด€ํ•˜์—ฌ ์ด์•ผ๊ธฐ๋ฅผ ๋‚˜๋ˆ„์—ˆ๋‹ค. ์ดํ›„ ํ”ผ๊ณ ์ธ์€ 2012. 6. 2.๊ฒฝ ๋ฒ ํŠธ๋‚จ โ€˜ํƒ„์†๋ˆ„ํŠธ๊ตญ์ œ๊ณตํ•ญโ€™์—์„œ (ํŽธ๋ช… 4 ์ƒ๋žต)์„ ์ด์šฉํ•˜์—ฌ ๊ฐ™์€ ๋‹ฌ 3.๊ฒฝ ์ธ์ฒœ๊ตญ์ œ๊ณตํ•ญ์œผ๋กœ ์ž…๊ตญํ•˜์˜€๋‹ค. ์ด๋กœ์จ ํ”ผ๊ณ ์ธ์€ ๊ตญ๊ฐ€์˜ ์กด๋ฆฝยท์•ˆ์ „์ด๋‚˜ ์ž์œ ๋ฏผ์ฃผ์  ๊ธฐ๋ณธ์งˆ์„œ๋ฅผ ์œ„ํƒœ๋กญ๊ฒŒ ํ•œ๋‹ค๋Š” ์ •์„ ์•Œ๋ฉด์„œ ๋ฐ˜๊ตญ๊ฐ€๋‹จ์ฒด ๊ตฌ์„ฑ์› ๋˜๋Š” ๊ทธ ์ง€๋ น์„ ๋ฐ›์€ ์ž์™€ ํšŒํ•ฉํ•˜์˜€๋‹ค. ๋งˆ. 2013. 7. 7.๊ฒฝ ํ†ต์‹ ์—ฐ๋ฝ ๋ฐ ํŽธ์˜์ œ๊ณต ํ”ผ๊ณ ์ธ์€ 2013. 7.๊ฒฝ ๋ถˆ์ƒ์˜ ์žฅ์†Œ์—์„œ โ€œ225๊ตญ๊ณผ ์ด๋ฉ”์ผ ํ†ต์‹ ๋ฐฉ๋ฒ• ์ œ์•ˆ, ๊ตญ์ •์› ๋Œ€์„ ๋น„๋ฆฌ์‚ฌ๊ฑด๊ณผ ๊ด€๋ จํ•˜์—ฌ ๋Œ€์‘์‚ฌ์—… ์ถ”์ง„ ์ƒํ™ฉ, ์กฐ์ง ๋ชจ์ž„์˜ ์ง„ํ–‰์ƒํ™ฉ, ๋ฏผ์‹ฌ๋™ํ–ฅ ๋ณด๊ณ ์™€ ๊ด€๋ จ๋œ ๊ฐ์˜คโ€ ๋“ฑ์ด ๊ธฐ์žฌ๋œ ๋Œ€๋ถ๋ณด๊ณ ๋ฌธ์„ ์ž‘์„ฑํ•˜์—ฌ ์ด๋ฅผ โ€˜๊ณ ๋‚œ์ฃผ๊ฐ„์„ค๊ตโ€™๋กœ ์‹œ์ž‘๋˜๋Š” ํ•œ๊ธ€ํŒŒ์ผ๋กœ ์œ„์žฅํ•˜์—ฌ ์Šคํ…Œ๊ฐ€๋…ธ๊ทธ๋ผํ”ผ๋กœ ์•”ํ˜ธํ™”ํ•œ ํŒŒ์ผ โ€˜toyou7-7.docxโ€™์„ ์ƒ์„ฑํ•˜์˜€๋‹ค. ์ดํ›„, ํ”ผ๊ณ ์ธ์€ 2013. 7. 7.๊ฒฝ ๋ถˆ์ƒ์˜ ์žฅ์†Œ์—์„œ ์ค‘๊ตญ ์ธํ„ฐ๋„ท ํฌํƒˆ์—…์ฒด โ€˜โ–ฝโ–ฝโ–ฝโ–ฝ.COMโ€™์— ์ ‘์†ํ•˜์—ฌ ๋ถํ•œ โ€˜225๊ตญโ€™ ์†Œ์† ๊ณต์ž‘์›๊ณผ ๊ณต๋™์œผ๋กœ ์‚ฌ์šฉํ•˜๋Š” ID์ธ โ€˜(์˜๋ฌธ ID 1 ์ƒ๋žต)โ€™๋กœ ๋กœ๊ทธ์ธํ•œ ๋‹ค์Œ, ๋ฏธ๋ฆฌ ์Šคํ…Œ๊ฐ€๋…ธ๊ทธ๋ผํ”ผ๋กœ ์•”ํ˜ธํ™”ํ•ด ๋†“์€ ์œ„ โ€˜toyou7-7.docxโ€™ ํŒŒ์ผ์„ (์˜๋ฌธ ID 1 ์ƒ๋žต)@โ–ฝโ–ฝโ–ฝโ–ฝ.COM๊ณผ (์˜๋ฌธ ID 1 ์ƒ๋žต)@โ—Žโ—Žโ—Žโ—Žโ—Ž.com์„ ์ˆ˜์‹ ์ธ์œผ๋กœ ์ง€์ •ํ•˜์—ฌ ๋ฐœ์†กํ•˜์˜€๋Š”๋ฐ, ๊ทธ ๊ตฌ์ฒด์ ์ธ ๋‚ด์šฉ์€ ๋‹ค์Œ๊ณผ ๊ฐ™๋‹ค.
ํ˜๋ช…์ ์ธ์‚ฌ๋ฅผ ๋“œ๋ฆฝ๋‹ˆ๋‹ค.
- ์ด๋ฉ”์ผ์„ ํ™•์ธํ–ˆ์Šต๋‹ˆ๋‹ค.
- ์ •์ƒ์„ ๊ณผ ์˜ˆ๋น„์„ ์ด ์ •ํ™•ํ•˜๊ฒŒ ์ ‘์† ๊ฐ€๋™๋˜์—ˆ์Šต๋‹ˆ๋‹ค.
- ๋”ฐ๋ผ์„œ ์˜ค๋Š˜ 7์›”7์ผ ์ดํ›„๋กœ ์ƒˆ๋กœ์šด ์ด๋ฉ”์ผ์„ ์ •์ƒ์„ ๊ณผ ์˜ˆ๋น„์„ ์œผ๋กœ ์“ธ ๊ฒƒ์„ ์ œ์•ˆํ•ฉ๋‹ˆ๋‹ค.
- ์ง€๊ธˆ๊นŒ์ง€ ์‚ฌ์šฉํ–ˆ๋˜ (ํ•œ๊ธ€ ID 1 ์ƒ๋žต)@โ—Žโ—Žโ—Žโ—Žโ—Ž.CN ๋ฉ”์ผ์€ ๋น„์ƒ์„ ์œผ๋กœ ์‚ฌ์šฉํ•  ๊ฒƒ์„ ์ œ์•ˆํ•ฉ๋‹ˆ๋‹ค.
- ๋น„์ƒ์„ ์ด๋ž€ ์—ฐ๋ฝ์ฑ…์—๊ฒŒ๋งŒ ์•Œ๋ ค์ฃผ๋Š” ๋ฉ”์ผ๋กœ ์กฐ์ง ํŒŒ๊ดด ๋“ฑ ๋น„์ƒ์˜ ์ƒํ™ฉ์—์„œ ์‚ฌ์šฉํ•˜๋Š” ๋ฉ”์ผ์ž…๋‹ˆ๋‹ค.
- ์กฐ์ง์€ ๊ตญ์ •์› ๋Œ€์„ ๋น„๋ฆฌ์‚ฌ๊ฑด์— ๊ธฐ๋…๊ต๊ณ„ ์ „์ฒด์˜ ๋Œ€์ฑ…๊ธฐ๊ตฌ์— ์ฐธ์—ฌํ•˜๋Š” ๋“ฑ ๋Œ€์‘์‚ฌ์—…์„ ํ•˜๊ณ  ์žˆ์œผ๋ฉฐ 7.27 ๊ตญ์ œ ํ‰ํ™”๋Œ€ํšŒ๋ฅผ ์œ„ํ•ด ์ „๊ตฌ๊ฐ„ ๊ฑท๊ธฐ ๋Œ€์› ์ค‘ ๋ชฉ์‚ฌ ์„ฑ์›์„ ํŒŒ๊ฒฌํ•ด ์ฃผ๋™์ ์œผ๋กœ ์ฐธ์—ฌํ•˜๊ณ  ์žˆ์Šต๋‹ˆ๋‹ค.
- ์›”๋ก€ ๋ชจ์ž„๋„ ์ •์ƒ์ ์œผ๋กœ ์ง„ํ–‰ํ•˜๊ณ  ์žˆ์Šต๋‹ˆ๋‹ค.
- ๋ฏผ์‹ฌ๋™ํ–ฅ ๋ณด๊ณ ์™€ ๊ด€๋ จํ•˜์—ฌ ์ข€๋” ๊ฒฝ๊ฐ์‹ฌ์„ ๊ฐ–๊ณ  ๋ณด๊ณ ํ•˜๋„๋ก ํ•˜๊ฒ ์Šต๋‹ˆ๋‹ค. ์ด์ƒ ๋.
์ด๋กœ์จ ํ”ผ๊ณ ์ธ์€ ๊ตญ๊ฐ€์˜ ์กด๋ฆฝยท์•ˆ์ „์ด๋‚˜ ์ž์œ ๋ฏผ์ฃผ์  ๊ธฐ๋ณธ์งˆ์„œ๋ฅผ ์œ„ํƒœ๋กญ๊ฒŒ ํ•œ๋‹ค๋Š” ์ •์„ ์•Œ๋ฉด์„œ ๋ฐ˜๊ตญ๊ฐ€๋‹จ์ฒด ๊ตฌ์„ฑ์› ๋˜๋Š” ๊ทธ ์ง€๋ น์„ ๋ฐ›์€ ์ž์™€ ํ†ต์‹ ์—ฐ๋ฝํ•˜๊ณ , โ€˜225๊ตญโ€™ ์†Œ์† ๊ณต์ž‘์›์—๊ฒŒ ํŽธ์˜๋ฅผ ์ œ๊ณตํ•˜์˜€๋‹ค. ๋ฐ”. ๊ณต์†Œ์™ธ 6๊ณผ ๊ณต๋ชจํ•˜์—ฌ, 2015. 4. 5.๊ฒฝ ํšŒํ•ฉ ๋ฐ ๊ธˆํ’ˆ์ˆ˜์ˆ˜ ํ”ผ๊ณ ์ธ์€ 2015. 4. 5.๊ฒฝ ๋ง๋ ˆ์ด์‹œ์•„ ์ฟ ์•Œ๋ผ๋ฃธํ‘ธ๋ฅด์—์„œ ๋ถํ•œ โ€˜225๊ตญโ€™ ์†Œ์† ๊ณต์ž‘์›๊ณผ์˜ ๋งŒ๋‚จ์„ ์•ฝ์ •ํ•˜๋ฉด์„œ, ๊ณต์†Œ์™ธ 6์„ ๋ณด๋‚ด๊ธฐ๋กœ ํ•˜์˜€๋‹ค. ํ”ผ๊ณ ์ธ์€ ๋ฏธ๋ฆฌ ๊ณต์†Œ์™ธ 6๊ณผ์˜ ๋งŒ๋‚จ์„ ํ†ตํ•ด โ€˜225๊ตญโ€™ ์†Œ์† ๊ณต์ž‘์›๊ณผ์˜ ํšŒํ•ฉ์„ ์ค€๋น„ํ•˜์˜€๋Š”๋ฐ, 2015. 3. 8. 19:20๊ฒฝ๋ถ€ํ„ฐ ๊ฐ™์€ ๋‚  21:50๊ฒฝ๊นŒ์ง€ ์„œ์šธ ์†กํŒŒ๊ตฌ ์ž ์‹ค์—ญ ์ธ๊ทผ์— ์žˆ๋Š” โ€˜(๋ช…์นญ 49 ์ƒ๋žต) ํ˜ธํ”„์ง‘โ€™์—์„œ ๊ณต์†Œ์™ธ 6์„ ๋งŒ๋‚˜ ๋ง๋ ˆ์ด์‹œ์•„ ์ถœ๊ตญ์ค€๋น„ ๋ฐ ๊ณต์ž‘์›๊ณผ์˜ ๋Œ€ํ™”๋‚ด์šฉ ๋“ฑ์„ ๋…ผ์˜ํ•˜์˜€๊ณ , 2015. 3. 30. 21:30๊ฒฝ๋ถ€ํ„ฐ ๊ฐ™์€ ๋‚  23:20๊ฒฝ๊นŒ์ง€ ์„œ์šธ ์ข…๋กœ๊ตฌ ์ข…๊ฐ์—ญ ์ธ๊ทผ์— ์žˆ๋Š” โ€˜(๋ช…์นญ 50 ์ƒ๋žต) ์ปคํ”ผ์ˆโ€™์—์„œ ๊ณต์†Œ์™ธ 6์„ ๋งŒ๋‚˜ ๊ณต์ž‘์›๊ณผ์˜ ์ ‘์„  ์ผ์ •, ์ฃผ์š” ์งˆ๋ฌธ๋‚ด์šฉ ๋ฐ ๋‹ต๋ณ€ ์š”๋ น, ์ถœ์ž…๊ตญ ์‹œ ๊ณตํ•ญ์„ธ๊ด€์˜ ๊ฒ€์ƒ‰๊ณผ์ • ๋ฐ ์ˆ˜์ˆ˜ํ•œ ๊ธˆํ’ˆ์ด ๋‹จ์†๋  ๊ฒฝ์šฐ์˜ ๋Œ€๋น„์ฑ… ๋“ฑ์— ๊ด€ํ•˜์—ฌ ๋…ผ์˜ํ•˜์˜€๋‹ค. ๊ทธ ์ดํ›„, ๊ณต์†Œ์™ธ 6์€ ๋ถํ•œ โ€˜225๊ตญโ€™ ์†Œ์† ๊ณต์ž‘์›์„ ๋งŒ๋‚˜๊ธฐ ์œ„ํ•˜์—ฌ, 2015. 4. 3.๊ฒฝ ์ธ์ฒœ๊ตญ์ œ๊ณตํ•ญ์—์„œ (ํŽธ๋ช… 5 ์ƒ๋žต)ํŽธ์„ ์ด์šฉ ๋ง๋ ˆ์ด์‹œ์•„ ์ฟ ์•Œ๋ผ๋ฃธํ‘ธ๋ฅด๋กœ ์ถœ๊ตญํ•˜์—ฌ, ๊ฐ™์€ ๋‚  17:21๊ฒฝ(๋ง๋ ˆ์ด์‹œ์•„ ํ˜„์ง€ ์‹œ๊ฐ) ์ฟ ์•Œ๋ผ๋ฃธํ‘ธ๋ฅด๊ตญ์ œ๊ณตํ•ญ์— ๋„์ฐฉํ•˜์˜€๋‹ค. ๊ณต์†Œ์™ธ 6์€ 2015. 4. 5. 17:20๊ฒฝ ์ฟ ์•Œ๋ผ๋ฃธํ‘ธ๋ฅด โ€˜๋ฉ”๋ฅด๋ฐ์นดโ€™ ๊ด‘์žฅ์˜ ๊ตญ๊ธฐ๋ด‰ ์•ž์—์„œ, โ€˜225๊ตญโ€™ ์†Œ์† ๊ณต์ž‘์› ๊ณต์†Œ์™ธ 39๋ฅผ ๋งŒ๋‚˜ ๊ทธ์˜ ์‹ ํ˜ธ์— ๋”ฐ๋ผ ๊ด‘์žฅ๊ณผ ์—ฐ๊ฒฐ๋œ ์ง€ํ•˜ํ†ต๋กœ๋กœ ์ด๋™ํ•˜์—ฌ ๊ณต์†Œ์™ธ 39๋กœ๋ถ€ํ„ฐ 2์ฐจ ์ ‘์„ ์žฅ์†Œ๋ฅผ ์ „๋‹ฌ๋ฐ›๊ณ  ํ—ค์–ด์ง„ ํ›„, ๊ณต์†Œ์™ธ 6์€ โ€˜๋ฉ”๋ฅด๋ฐ์นดโ€™ ๊ด‘์žฅ์—์„œ ํƒ์‹œ๋กœ ์ฟ ์•Œ๋ผ๋ฃธํ‘ธ๋ฅด ์ปจ๋ฒค์…˜์„ผํ„ฐ ์ธ๊ทผ โ€˜(๋ช…์นญ 51 ์ƒ๋žต) ํ˜ธํ…”โ€™ ์•ž์œผ๋กœ ์ด๋™ํ•˜์—ฌ ๋Œ€๊ธฐํ•˜๋‹ค ๊ฐ™์€ ๋‚  18:23๊ฒฝ ๋„์ฐฉํ•œ โ€˜225๊ตญโ€™ ์†Œ์† ๊ณต์ž‘์› ๊ณต์†Œ์™ธ 40(๊ฐ€๋ช… : โ–ทโ–ทโ–ท, โ–ทโ™คโ™ค), ๊ณต์†Œ์™ธ 39 ๋“ฑ์˜ ์‹ ํ˜ธ์— ๋”ฐ๋ผ ์ฃผ๋ณ€์„ ๊ฐ์‹œํ•˜๋ฉด์„œ ์œ„ โ€˜(๋ช…์นญ 51 ์ƒ๋žต) ํ˜ธํ…”โ€™์—์„œ ์•ฝ 200m ๋–จ์–ด์ง„ โ€˜(๋ช…์นญ 52 ์ƒ๋žต) ํ˜ธํ…”โ€™๋กœ ์ด๋™ํ•œ ํ›„, ํ˜ธํ…” ๋‚ด ๋˜๋Š” ์ธ๊ทผ ๋ถˆ์ƒ์˜ ์žฅ์†Œ์—์„œ ๊ณต์†Œ์™ธ 40 ๋ฐ ๊ณต์†Œ์™ธ 39๋ฅผ ๋งŒ๋‚˜ ํ•จ๊ป˜ ๋‚จํ•œ ๋‚ด ์ •์„ธ ๋“ฑ์— ๊ด€ํ•˜์—ฌ ์ด์•ผ๊ธฐ๋ฅผ ๋‚˜๋ˆ„์—ˆ๋‹ค. ๋˜ํ•œ, ๊ณต์†Œ์™ธ 6์€ ์œ„ ๊ณต์†Œ์™ธ 40, ๊ณต์†Œ์™ธ 39์™€์˜ ํšŒํ•ฉ ๊ณผ์ •์—์„œ ๊ทธ๋“ค๋กœ๋ถ€ํ„ฐ ํ™œ๋™๋น„ ๋“ฑ ๋ช…๋ชฉ์œผ๋กœ ๋ฏธํ™” 18,900๋‹ฌ๋Ÿฌ ์ƒ๋‹น์„ ๊ต๋ถ€๋ฐ›์•˜๋‹ค. ์ดํ›„ ๊ณต์†Œ์™ธ 6์€ 2015. 4. 6.๊ฒฝ ์ฟ ์•Œ๋ผ๋ฃธํ‘ธ๋ฅด๊ตญ์ œ๊ณตํ•ญ์—์„œ (ํŽธ๋ช… 6 ์ƒ๋žต)ํŽธ์„ ์ด์šฉํ•˜์—ฌ ๊ฐ™์€ ๋‹ฌ 7.๊ฒฝ ์ธ์ฒœ๊ตญ์ œ๊ณตํ•ญ์œผ๋กœ ์ž…๊ตญํ•˜์˜€๋‹ค. ์ด๋กœ์จ ํ”ผ๊ณ ์ธ์€ ๊ณต์†Œ์™ธ 6๊ณผ ๊ณต๋ชจํ•˜์—ฌ, ๊ตญ๊ฐ€์˜ ์กด๋ฆฝยท์•ˆ์ „์ด๋‚˜ ์ž์œ ๋ฏผ์ฃผ์  ๊ธฐ๋ณธ์งˆ์„œ๋ฅผ ์œ„ํƒœ๋กญ๊ฒŒ ํ•œ๋‹ค๋Š” ์ •์„ ์•Œ๋ฉด์„œ ๋ฐ˜๊ตญ๊ฐ€๋‹จ์ฒด ๊ตฌ์„ฑ์› ๋˜๋Š” ๊ทธ ์ง€๋ น์„ ๋ฐ›์€ ์ž์™€ ํšŒํ•ฉํ•˜๊ณ , โ€˜225๊ตญโ€™ ์†Œ์† ๊ณต์ž‘์›์œผ๋กœ๋ถ€ํ„ฐ ๊ธˆํ’ˆ์„ ์ˆ˜์ˆ˜ํ•˜์˜€๋‹ค. ์‚ฌ. 2015. 11. 12.๊ฒฝ ํ†ต์‹ ์—ฐ๋ฝ ๋ฐ ํŽธ์˜์ œ๊ณต ํ”ผ๊ณ ์ธ์€ 2015. 11. 12.๊ฒฝ ํ”ผ๊ณ ์ธ์˜ ์ฃผ๊ฑฐ์ง€์—์„œ ๋…ธํŠธ๋ถ์„ ์ด์šฉํ•˜์—ฌ โ€œ2015 ์ดํ™”๋ณด๊ณ ์„œโ€๊ฐ€ ๊ธฐ์žฌ๋œ ๋Œ€๋ถ๋ณด๊ณ ๋ฌธ์„ ์ž‘์„ฑํ•˜์—ฌ ์ด๋ฅผ โ€˜์ด์Šค๋ผ์—˜ ๋ฐฑ์„ฑ๋“ค์—๊ฒŒ์„œ ํ•˜๋‚˜๋‹˜์˜ ์–ธ์•ฝ๊ถค๋ฅผ ๋นผ์•—์„โ€™๋กœ ์‹œ์ž‘๋˜๋Š” ํ•œ๊ธ€ํŒŒ์ผ์„ ์œ„์žฅํ•˜์—ฌ ์Šคํ…Œ๊ฐ€๋…ธ๊ทธ๋ผํ”ผ๋กœ ์•”ํ˜ธํ™”ํ•œ ํŒŒ์ผ โ€˜toyou11-12.docxโ€™์„ ์ƒ์„ฑํ•œ ๋‹ค์Œ ์ด๋ฅผ Micro SD์นด๋“œ์— ์ €์žฅํ•˜์˜€๋‹ค. ์ดํ›„, ํ”ผ๊ณ ์ธ์€ 2015. 11. 12. 17:23๊ฒฝ ์„œ์šธ ๋™์ž‘๊ตฌ (์ฃผ์†Œ 2 ์ƒ๋žต)์— ์žˆ๋Š” โ–ณโ–ณ๋Œ€ํ•™๊ต ์ œ1์ข…ํ•ฉ๊ด€ 1์ธต ๋กœ๋น„์—์„œ, ๊ทธ๊ณณ์— ์„ค์น˜๋œ ์ธํ„ฐ๋„ท PC๋ฅผ ์ด์šฉํ•˜์—ฌ ์ค‘๊ตญ ํฌํ„ธ์‚ฌ์ดํŠธ(www.โ–ฒโ–ฒโ–ฒโ–ฒ.net)์— ์ ‘์†ํ•˜์—ฌ ๋ถํ•œ โ€˜225๊ตญโ€™ ์†Œ์† ๊ณต์ž‘์›๊ณผ ๊ณต๋™์œผ๋กœ ์‚ฌ์šฉํ•˜๋Š” ID์ธ โ€˜(์˜๋ฌธ ID 2 ์ƒ๋žต)โ€™์œผ๋กœ ๋กœ๊ทธ์ธํ•œ ๋‹ค์Œ, ๋ฏธ๋ฆฌ ์Šคํ…Œ๊ฐ€๋…ธ๊ทธ๋ผํ”ผ๋กœ ์•”ํ˜ธํ™”ํ•˜์—ฌ Micro SD์นด๋“œ์— ์ €์žฅํ•ด ๋†“์€ ์œ„ โ€˜toyou11-12.docxโ€™ ํŒŒ์ผ์„ ์ˆ˜์‹ ์ž๋ฅผ โ€˜(์˜๋ฌธ ID 2 ์ƒ๋žต)โ€™์œผ๋กœ ์ง€์ •ํ•˜์—ฌ ๋ฐœ์†กํ•˜์˜€๋Š”๋ฐ, ๊ทธ ๊ตฌ์ฒด์ ์ธ ๋‚ด์šฉ์€ ๋‹ค์Œ๊ณผ ๊ฐ™๋‹ค.
1. ํ˜๋ช…์  ์ธ์‚ฌ๋ฅผ ๋“œ๋ฆฝ๋‹ˆ๋‹ค.
2. 2015๋…„ ์ดํ™” ๋ณด๊ณ ๋ฅผ ๋“œ๋ฆฝ๋‹ˆ๋‹ค.
1) ์ด์  ์ดํ™”
- 2015๋…„์€ ์ด์ ์œผ๋กœ ๋‹น์ฐฝ๊ฑด 70๋Œ์„ ๋งž์œผ๋ฉฐ ์กฐ์ง์„ ์žฌ์ •๋น„ํ•˜๊ณ  ์ž๊ธฐ ์ž„๋ฌด๋ฅผ ์ˆ˜ํ–‰ํ•˜๊ธฐ ์œ„ํ•œ ์—ญ๋Ÿ‰ ๊ฐ•ํ™”์˜ ํ•œ ํ•ด์˜€์Šต๋‹ˆ๋‹ค.
- ์ข…๊ต๋ถ€๋ฌธ์—์„œ ๋Œ€์ค‘์  ์„ ์ „์—ญ๋Ÿ‰ ๊ฐ•ํ™”์™€ ์„ ์ง„์  ํˆฌ์Ÿ์—ญ๋Ÿ‰ ๊ฐ•ํ™”๋ฅผ ์ด๋ฃจ์–ด๋‚ธ ์„ฑ๊ณผ๋ฅผ ๊ฑฐ๋‘์—ˆ์Šต๋‹ˆ๋‹ค.
- ๊ฒฝ๊ธฐ๋ถ๋ถ€์ง€์—ญ๊ณผ ๊ฒฝ๊ธฐ๋‚จ๋ถ€์ง€์—ญ์—์„œ ์ง€๋„๋ถ€ ์—ญ๋Ÿ‰์„ ๊ฐ•ํ™”ํ•ด ํ–ฅํ›„ ์กฐ์ง์  ๋Œ€์ค‘์  ์ฃผ์ฒด ๊ฐ•ํ™”๋ฅผ ์ด๋ฃฐ ์ˆ˜ ์žˆ๋Š” ์„ฑ๊ณผ๋ฅผ ๊ฑฐ๋‘์—ˆ์Šต๋‹ˆ๋‹ค.
- ์„œ์šธ์ง€์—ญ๊ณผ ์ „๋ถ์ง€์—ญ์—์„œ ์กฐ์ง์„ ์žฌ์ •๋น„ํ•˜๊ณ  ํ–ฅํ›„ ํ™œ๋™์˜ ๋ฐฉํ–ฅ์„ฑ์„ ํ™•๋ฆฝํ•œ ํ•œํ•ด์˜€์Šต๋‹ˆ๋‹ค.
- (๋ช…์นญ 10 ์ƒ๋žต)์—ฐ๋Œ€์˜ ๋†์„ฑ๊ณผ ํˆฌ์Ÿ์„ ์ง€์› ํ˜‘์กฐํ•˜๋ฉฐ ์ข…๊ต๋ถ€๋ฌธ์—์„œ ์—ญํ• ์„ ๋‹คํ•œ ์„ฑ๊ณผ๋ฅผ ๊ฑฐ๋‘” ํ•œ ํ•ด์˜€์Šต๋‹ˆ๋‹ค.
- ๊ณต์†Œ์™ธ 24 ๊ด€๋ จ ์Ÿ์ ์„ ๋Œ€์ค‘์ ์œผ๋กœ ์ œ๊ธฐํ•˜๊ณ  ์ด ๋ฌธ์ œ๋ฅผ ์ฃผ๋„ํ•ด ๊ฐ„ ํ™œ๋™์„ฑ๊ณผ๋ฅผ ์ด๋ฃจ์—ˆ์Šต๋‹ˆ๋‹ค.
- ๋ฐ˜์ „ํ‰ํ™” ํˆฌ์Ÿ์—์„œ ์ข…๊ต๋ถ€๋ฌธ ์กฐ์ง์œผ๋กœ์„œ ์ž๊ธฐ ์—ญํ• ์„ ๋‹คํ•œ ํ•œ ํ•ด์˜€์Šต๋‹ˆ๋‹ค.
- ๋ฏผ์‹ฌ๋™ํ–ฅ์„ ์ฃผ๊ธฐ์ ์œผ๋กœ ๋ณด๊ณ ํ•˜์—ฌ ๋ณธ๋ถ€์˜ ์ •์ฑ… ํŒ๋‹จ์— ๋ณต๋ฌดํ•œ ํ•œํ•ด์˜€์Šต๋‹ˆ๋‹ค. ์ข…๊ต์ธ, ์‹œ๋ฏผ๋Œ€์ค‘, ์šด๋™๊ถŒ, ์ •์น˜๊ถŒ, ์–ธ๋ก  ๋™ํ–ฅ ๋“ฑ์„ ๋ถ„์„, ๋ณด๊ณ ํ–ˆ์Šต๋‹ˆ๋‹ค.
- ๋ฏผ์ƒ, ๋ฏผ์ฃผ ๋ฏผ๊ถŒ, ์„ธ์›”ํ˜ธ ๋“ฑ ํ˜„์•ˆ๊ณผ ๋Œ€์ค‘ ํˆฌ์Ÿ์—์„œ ์ž๊ธฐ ์—ญํ• ์„ ํ•œ ํ•œํ•ด์˜€์Šต๋‹ˆ๋‹ค.
- ์ง„๋ณด์ง„์˜ ๋‹จ๊ฒฐ๊ณผ ํ˜์‹ , ์—ฐ๋Œ€์™€ ํ†ตํ•ฉ์„ ์œ„ํ•œ ๊ธธ์—์„œ ์ตœ์„ ์„ ๋‹คํ•ด ์ž๊ธฐ ์—ญํ• ์„ ํ•œ ํ•œํ•ด์˜€์Šต๋‹ˆ๋‹ค.
- ๋ฐ˜๋ฉด ํ•ต์‹ฌ ์กฐ์ง์„ฑ์› ํ™•๋Œ€์™€ ์ง€์—ญ์ง€๋„๋ถ€ ์™„์„ฑ์˜ ๊ณผ์ œ๋ฅผ ์ด๋ฃจ๋Š” ๋ฐ๋Š” ํ•œ๊ณ„๋ฅผ ๋ณด์˜€์Šต๋‹ˆ๋‹ค.
- ์ข€ ๋” ํญ๋„“์€ ๋Œ€์ค‘์„ ๊ต์–‘ํ•˜๊ณ  ํˆฌ์Ÿ ์ฃผ์ฒด๋กœ ๋‚ด์„ธ์šฐ๋Š” ๋ฐ๋Š” ์—ญ๋Ÿ‰์˜ ํ•œ๊ณ„๋ฅผ ๋ณด์˜€์Šต๋‹ˆ๋‹ค.
- ์ด์ ์œผ๋กœ ์ •์„ธ๋ณ€ํ™”์˜ ์‹ฌํ™”์™€ ์—ญ๋Ÿ‰์˜ ํ•œ๊ณ„ ์†์—์„œ๋„ ์ตœ์„ ์„ ๋‹คํ•ด ์›์ˆ˜๋‹˜์˜ ์˜๋„์™€ ๋‹น์˜ ์ž„๋ฌด๋ฅผ ์ˆ˜ํ–‰ํ•˜๊ธฐ ์œ„ํ•ด ์ ˆ์น˜๋ถ€์‹ฌ์˜ ๋…ธ๋ ฅ์„ ๋‹คํ•œ ํ•œํ•ด์˜€์Šต๋‹ˆ๋‹ค.
2) ๊ฐ ์ง€์—ญ ๋ถ€๋ฌธ ๋“ฑ ์ดํ™”
- ์ข…๊ต๋ถ€๋ฌธ
1) ์ข…๊ต๋ถ€๋ฌธ์€ ์„ ์ง„์  ๋Œ€์ค‘๋“ค์ด ์‹ค์ฒœ์„ ์œ„ํ•ด ์กฐ์งํ•œ (๋ช…์นญ 11 ์ƒ๋žต) ์กฐ์ง์ด ๊ฐ€์žฅ ํฐ ์„ฑ๊ณผ์ž„. ์•ฝ 20๋ช…์˜ ์‹ค์ฒœ์  ๊ธฐ๋…๊ต๋ชฉ์‚ฌ๋“ค์ด ์„ ๋„์ ์œผ๋กœ ํˆฌ์Ÿ์— ๋‚˜์„œ๋Š” ๊ฒƒ์ด ์กฐ์ง์˜ ๋ชฉ์ ์ž„. ํ–ฅํ›„ ๋ฐ˜๋ฏธ, ๋ฐ˜์ „, ๋ฏผ์ƒ ํˆฌ์Ÿ์— ์•ž์žฅ์„œ ์‹ธ์šฐ๊ฒŒ ๋  ๊ฒƒ์ž„. ๊ธฐ์กด ๋‹จ์ฒด์™€ ์›๋งŒํ•œ ๊ด€๊ณ„์™€ ๋‹จ๊ฒฐํˆฌ์Ÿ์„ ์œ„ํ•ด ์ง€๋„์˜ ์ดˆ์ ์„ ๋งž์ถ”๊ณ  ์žˆ์Œ.
2) ๊ณต์†Œ์™ธ 24์”จ๋ฅผ ์กฐ์ง์—์„œ ๊ด€๋ฆฌํ•˜๋ฉฐ ์†กํ™˜ ํˆฌ์Ÿ์„ ๋ฒŒ์ด๊ณ  ์žˆ์Œ. ๋ณธ์‚ฌ์™€์˜ ์œ ๊ธฐ์  ํ˜‘์กฐ๋ฅผ ์š”๋งํ•จ. ํ˜„์žฌ โ– โ– ๋Œ€์‚ฌ๊ด€์œผ๋กœ ๋›ฐ์–ด๋“ค์–ด ์ถ”๋ฐฉ๋‹นํ•˜๋Š” ์‹์œผ๋กœ ์ถ”๋ฐฉ๋ผ ๋‚˜๊ฐˆ ๊ฒƒ์„ ๊ณต์†Œ์™ธ 24์”จ๋Š” ์ƒ๊ฐํ•˜๋Š”๋ฐ, ์ด๋Š” (๋ช…์นญ 12 ์ƒ๋žต)์‹ ๋ฌธ โ—†๋ชจ ๊ธฐ์ž์˜ ๋ฐœ์ƒ์ž„. ์กฐ์ง์€ ์ด๋ฅผ ๋ฐ˜๋Œ€ํ•˜๋ฉฐ ๋Œ€์ค‘์ ์œผ๋กœ ์ค‘์žฅ๊ธฐ์ ์œผ๋กœ ๋ฌธ์ œ๋ฅผ ํ’€ ๊ฒƒ์„ ์„ค๋“ํ•˜๊ณ  ์žˆ์Œ.
3) ์ข…๊ต๋ถ€๋ฌธ์—์„œ๋Š” ์ƒ๋ฐ˜๊ธฐ์— (๋ช…์นญ 10 ์ƒ๋žต)์—ฐ๋Œ€์— ๋Œ€ํ•œ ์ง€์›์‚ฌ์—…์„ ์ค‘์‹ฌ์œผ๋กœ ์‚ฌ์—…ํ–ˆ์œผ๋ฉฐ ์ด๋Š” ์„ฑ๊ณผ์ ์ด์—ˆ๋‹ค๊ณ  ํ‰๊ฐ€ํ•จ.
4) 1๋…„์—ฌ ๋™์•ˆ ์ด์ฑ…์€ ์ž๊ธฐ ๊ต๋‹จ์ธ ์ธก์— (๋ช…์นญ 13 ์ƒ๋žต)๋ผ๋Š” ์ธํ„ฐ๋„ท์‹ ๋ฌธ์„ ์‹œ์ž‘ํ•ด ํ˜„์žฌ ์ƒ๋‹นํ•œ ์„ฑ๊ณผ๋ฅผ ๊ฑฐ๋‘๊ณ  ์ž๋ฆฌ์žก๊ณ  ์žˆ์Œ. ๊ทนํžˆ ๋ณด์ˆ˜์ ์ธ ์ข…๊ต์ง€๋„์ž๋“ค์„ ๋Œ€์ƒ์œผ๋กœ ๊ต์–‘๊ณผ ์„ ์ „์˜ ํ†ต๋กœ๊ฐ€ ๋งŒ๋“ค์–ด์กŒ์œผ๋ฉฐ, ๊ทธ๋“ค์˜ ๋ฏผ์‹ฌ๋™ํ–ฅ๊ณผ ์–ธ๋ก ์„ ํ†ตํ•œ ์ •๋ณด ์ˆ˜์ง‘์— ์œ ๋ฆฌํ•œ ๊ตญ๋ฉด์„ ์ฐฝ์ถœํ–ˆ์Œ.
5) ์„ ์ „์ฑ…์€ ๊ด€์•…์ง€์—ญ์—์„œ ํ˜์‹ ์ ์ธ ์ง„๋ณด์„ธ๋ ฅ์„ ๋ชจ์•„ (๋ช…์นญ 14 ์ƒ๋žต)์ด๋ผ๋Š” ๋ฐ˜ํ•ฉ๋ฒ• ๊ตฐ์ค‘ํ•ต์‹ฌ ๋ชจ์ž„์„ ํ˜•์„ฑํ–ˆ์œผ๋ฉฐ, ์ด ์†์—์„œ โ–ผ๋ชจ์”จ๋ฅผ ์ธ์ž…ํ•ด ๊ต์–‘์ค‘์ž„. ํ–ฅํ›„ ์ง€์—ญ๋Œ€์ค‘์‚ฌ์—…์„ ์ง„ํ–‰์ฝ”์žํ•จ.
- ์‹œ๋ฏผ๋ถ€๋ฌธ
ํ˜„์žฌ ์ฃผ์ฒด๊ฐ€ ๋งˆ๋ จ๋˜์ง€ ์•Š๊ณ  ์žˆ์Œ. ๋‹ค์‹œ ํ•œ๋ฒˆ ๊ตฌ์กฐ์ง์˜ ๊ณต์†Œ์™ธ 25๋ฅผ ๋Œ€์ƒ์œผ๋กœ ์‹œ๋ฏผ๋ถ€๋ฌธ์— ๋Œ€ํ•œ ์‚ฌ์—… ์š”๊ตฌ๋ฅผ ํ•ด๋ณด๊ฒ ์Œ. ๋˜ํ•œ ๋Œ€์ƒ์œผ๋กœ ๊ณต์†Œ์™ธ 26์„ ์ถ”๊ฐ€ํ•˜๊ฒ ์Œ.
- ์„œ์šธ์ง€์—ญ
1) ์„œ์šธ์ง€์—ญ์€ ์ง€๋‚œํ•ด์˜ ํ˜ผ๋ž€๊ณผ ๋ถ€์ง„์„ ๊ทน๋ณตํ•˜๊ณ  ์˜ฌํ•ด๋ถ€ํ„ฐ ์‚ฌ์ƒ์šด๋™์„ ๊ธฐ์น˜๋กœ ํ•˜๋Š” ๋…ธ๋™์šด๋™์˜ ํ˜์‹ ๊ณผ ์ง€์—ญ์šด๋™์˜ ๋‹จํ•ฉ์„ ์œ„ํ•ด ๋…ธ๋ ฅํ–ˆ์Œ.
2) ๋…ธ๋™์šด๋™์—์„œ ์‚ฌ์ƒ์ ์œผ๋กœ ํ”๋“ค๋ฆฌ๊ณ  ํ˜ผ๋ž€์Šค๋Ÿฌ์šด ํ˜„์‹ค์„ ๊ทน๋ณตํ•˜๊ธฐ ์œ„ํ•œ ๊ต์–‘๊ณผ ์˜ํ–ฅ๋ ฅ ํ™•๋Œ€๋ฅผ ๊พ€ํ•˜๊ณ  ๊ทธ ์ผ์„ ์ง€์—ญ์—์„œ ๋ชจ๋ฒ”์„ ์„ธ์šฐ๊ณ  ์ „๊ตญ์ง€์—ญ์„ ์—ฐ๊ฒฐํ•˜๋Š” ๋ฐฉํ–ฅ์—์„œ ์ง€ํ–ฅํ•ด๊ฐ€๊ณ  ์žˆ์Œ.
3) ์„œ์šธ์ฑ…์ž„์ž๊ฐ€ (๋ช…์นญ 7 ์ƒ๋žต)์—ฐ๋Œ€๋ผ๋Š” ํ™œ๋™๊ฐ€ ์กฐ์ง์˜ ๊ฐ„๋ถ€๊ฐ€ ๋˜์—ˆ์œผ๋ฉฐ ์ด๋ฅผ ํ†ตํ•ด ๋…ธ๋™์šด๋™ ํ˜์‹ ์„ ์ถ”๊ตฌํ•˜๊ณ  ์žˆ์Œ.
4) ์ง€์—ญ์—์„œ๋Š” (๋ช…์นญ 15 ์ƒ๋žต)๋ชจ์ž„ ์ด๋ผ๋Š” ์ง€์—ญํ•ต์‹ฌ์˜ ํ•™์Šต๋ชจ์ž„์„ 3๋…„์งธ ์œ ์ง€ํ•˜๊ณ  ์žˆ์œผ๋ฉฐ, ์ด๋ฅผ ํ†ตํ•ด ์‹ค์ฒœ์„ฑ๊ณผ ํ˜„์žฅ์„ฑ, ์—ฐ๋Œ€์„ฑ์„ ๋†’์—ฌ๊ฐ€๊ณ  ์žˆ์Œ.
5) ์„œ์šธ ์ฑ…์ž„์ž๊ฐ€ ๊ทธ๋™์•ˆ ์กฐ์ง์„  ๋ฐ–์— ์žˆ๋˜ ๊ณต์†Œ์™ธ 27 ์—ฐ๋ฝ์› ๋™์ง€์— ๋Œ€ํ•œ ์กฐ์ง๊ด€๋ฆฌ๋ฅผ ํ•˜๊ณ  ๊ต์–‘๊ณผ ์กฐ์งํ™” ์‚ฌ์—…์„ ํ•˜๊ณ  ์žˆ์Œ.
6) ์„œ์šธ์ฑ…์ž„์ž์˜ ๊ณผ๋„ํ•œ ๋Œ€์ค‘์กฐ์ง ์—…๋ฌด๋กœ ์ธํ•ด ๋™์ง€๋“ค๊ณผ์˜ ์—ฐ๊ฒฐ์ด ์›๋งŒํžˆ ๋ณด์žฅ๋˜์ง€ ์•Š๋Š” ์ ์ด ๋ฌธ์ œ์ ์ด์ง€๋งŒ, ์ถฉ์‹ค์„ฑ์„ ๋ฐ”ํƒ•์œผ๋กœ ํ•œ ์„œ์šธ ์ฑ…์ž„์ž์˜ ํ™œ๋™์€ ๋งค์šฐ ์ข‹์Œ.
- ๊ฒฝ๊ธฐ๋ถ๋ถ€์ง€์—ญ
1) ๊ฒฝ๋ถ์ง€์—ญ์€ ์˜ฌํ•ด ๋ณธ์‚ฌ์™€์˜ ์ ‘์ด‰์„ ํ†ตํ•ด ์‚ฌ์—…๊ณผ ์‚ฌ์ƒ์— ๋”์šฑ ํฐ ๋ฐœ์ „์„ ๊ฐ€์ ธ์˜ค๊ณ  ์žˆ์Œ.
2) ์ดˆ๊ธฐ ์ž๊ธˆ ์ „๋‹ฌ๋กœ ์ธํ•œ ์–ด๋ ค์›€์€ ์™„์ „ํžˆ ๊ทน๋ณต๋˜์—ˆ์œผ๋ฉฐ, ๋ชจ๋“  ํ™œ๋™์„ ์ •์ƒํ™”ํ–ˆ์Œ. ์ž๊ธˆ๋Œ€์ฑ…์œผ๋กœ ๋‚ด๋…„๋ถ€ํ„ฐ๋Š” ์ผ์ •์ด ์ •ํ•ด์ง€๋ฉด 2์ธ ์ด์ƒ์ด ๊ฐ๊ธฐ ๋‹ค๋ฅธ ๋‚  ์ถœ๊ตญ ์ž…๊ตญํ•ด ๋ณธ์‚ฌ ๋ฉด๋‹ด ํ›„ ๋งŒ๋‚˜ ์ƒํ˜ธ 1๋งŒ์”ฉ ๋‚˜๋ˆ„๋Š” ๋ฐฉ๋ฒ•์œผ๋กœ ์•ˆ์ „์„ ๋ณด์žฅํ•˜๋ คํ•จ. ์ตœ๊ทผ ์ž์œ ์—ฌํ–‰ ํ™œ์„ฑํ™”๋กœ ์ ์€ ๋น„์šฉ์œผ๋กœ ํ•ด์™ธ์—ฌํ–‰์ด ๊ฐ€๋Šฅํ•ด์ง. ๋น„์šฉ๋ณด๋‹ค ์•ˆ์ „ ์šฐ์„  ์›์น™์œผ๋กœ ํ•˜๊ฒ ์Œ.
3) ๊ฒฝ๋ถ์ง€์—ญ์ฑ…์ž„์ž๋Š” โ™กโ™กโ™กโ™ก ํ˜„์ง€๋„๋ถ€์™€ ๊ธด๋ฐ€ํ•œ ๊ด€๊ณ„๊ฐ€ ๊ตฌ์ถ•๋˜์–ด โ™กโ™กโ™กโ™ก ์ƒ์ธต์— ๋Œ€ํ•œ ์ •์ฑ…์ง€๋„ ์‚ฌ์—…๊ณผ ์„ค๋“์„ ํ•˜๊ณ  ์žˆ์Œ. ์ขŒํŒŒ์  ์ง€๋„๋ถ€์ด์ง€๋งŒ, ํˆฌ์Ÿ๊ณผ ์ธ์‹์—์„œ ์ขŒ์šฐํŽธํ–ฅ์ด ์‹ฌํ•ด ์ด๋ฅผ ์˜ณ์„ ๋ฐฉํ–ฅ์œผ๋กœ ์ด๋„๋Š” ์ •์น˜์‚ฌ์—…์„ ๋ณ‘ํ–‰ํ•จ.
4) ๊ฒฝ๋ถ์ง€์—ญ์ด ์ฃผ ๋™์ง€์— ๋Œ€ํ•œ ์ž…๋‹น ์‚ฌ์—…์ด ์„ฑ๊ณผ์ ์œผ๋กœ ๊ฒฐ์†๋จ. ์ •์น˜์  ์กฐ์ง์ , ์‚ฌ์ƒ์  ์ •์„œ์ ์œผ๋กœ ๋งค์šฐ ๋†’์€ ๊ฒฐ์˜๋ฅผ ํ‘œ๋ช…ํ•จ.
5) ๊ฒฝ๋ถ์ง€์—ญ์˜ ๋Œ€์ค‘์  ์กฐ์ง์  ๋ฐœ์ „์„ ์œ„ํ•ด ๋…ธ๋ ฅํ•˜๊ณ  ์žˆ์Œ.
6) ์‚ฌ์—…์ค‘์ธ (๋ช…์นญ 16 ์ƒ๋žต)๋…ธ์กฐ๋Š” ํ†ตํ•ฉ์„ ํ†ตํ•ด 2๋งŒ ์กฐํ•ฉ์›์œผ๋กœ ๋ฐœ์ „๋œ ๋…ธ์กฐ๋ฅผ ์ „๋งํ•˜๊ณ  ์žˆ์Œ.
- ๊ฒฝ๊ธฐ๋‚จ๋ถ€์ง€์—ญ
1) ์ง€์—ญ์ฑ…์ž„์ž๋Š” ๋…ธ์ˆ™์„ฑ์„ ๋”ํ•˜๊ณ  ์žˆ์œผ๋ฉฐ ์ง€ํ•˜๋‹น ์‚ฌ์—…์˜ ๋ฌ˜๋ฆฌ๋ฅผ ์•Œ์•„๊ฐ€๋ฉฐ ์‚ฌ์—…์— ์ง„์ „์ด ์ผ์–ด๋‚˜๊ณ  ์žˆ์Œ.
2) ์ง€์—ญ์˜ ๊ตฐ์ค‘ํ•ต์‹ฌ๊ณผ ํ™œ๋™๊ฐ€๋ฅผ ๋Œ€์ƒ์œผ๋กœ ํ•™์Šต์†Œ์กฐ ํ˜•์„ฑํ‚ค๋กœ ํ–ˆ์Œ.
3) ์˜ค์‚ฐ ํ‰ํƒ์ง€์—ญ ๋Œ€์ค‘์‚ฌ์—…๊ณผ ์—ฐ๊ณ„๋ฅผ ๊ฐ–๊ณ  ์ง€์†์ ์œผ๋กœ ํ™œ๋™ํ•จ.[โ—€์ฐจ, ์˜ค์‚ฐ์ง€์—ญํ™œ๋™, ๊ตฌ (๋ช…์นญ 17 ์ƒ๋žต)๋‹น ์‚ฌ๋žŒ๋“ค]
4) ์ƒ๋ฐ˜๊ธฐ (๋ช…์นญ 10 ์ƒ๋žต)์—ฐ๋Œ€์— ๋Œ€ํ•œ ์ง€์› ์‚ฌ์—…์„ ์ „๊ฐœํ•จ. ์ตœ๊ทผ ๋“ค์–ด (๋ช…์นญ 10 ์ƒ๋žต)์—ฐ๋Œ€์— ๋Œ€ํ•œ ํƒ„์••์ด ์˜ˆ๊ฒฌ๋˜๋ฉด์„œ ๋ณด์œ„๋ฅผ ์œ„ํ•ด ๊ฑฐ๋ฆฌ๋ฅผ ๋‘๊ณ  ์žˆ์Œ.
5) ๋‚ด๋…„ ๊ฒฝ๋‚จ์ง€์—ญ์ฑ…๋™์ง€์— ๋Œ€ํ•œ ๋ณธ์‚ฌ ๋ฉด๋‹ด์„ ๊ณ„ํšํ•˜๊ณ  ์žˆ์Œ. ๋น„๋ก ๋ณด์•ˆ๊ด€์ฐฐ์ค‘์ด์ง€๋งŒ, ๊ฐ€์กฑ์—ฌํ–‰(๋ถ€์ธ๊ณผ ๋”ธ)์„ ๋นŒ๋ฏธ๋กœ ๋น„์ž๋ฅผ ๋งŒ๋“ค๊ณ , ๋ฒ ํŠธ๋‚จ์œผ๋กœ ์—ฌํ–‰ํ•ด ํ•˜๋ฃจ์ •๋„ ๊ฐ€์กฑ๊ณผ ๋–จ์–ด์ ธ ๋ณธ์‚ฌ์™€ ๋ฉด๋‹ดํ•˜๋Š” ๊ฒƒ์ž„. ์ด์ œ๋Š” ๋ฉด๋‹ดํ•ด๋„ ์ข‹์€ ์กฐ๊ฑด์ด ๋งˆ๋ จ๋๋‹ค๊ณ  ํŒ๋‹จ๋จ. ๋ฌด๋ฆฌํ•˜์ง€ ์•Š๊ฒŒ ๊ตฌ์ฒด ์ƒํ™ฉ ๋ณด๋ฉด์„œ ๋ณธ์‚ฌ์˜ ์˜๊ฒฌ์„ ๋“ฃ๊ณ  ์ตœ์ข… ํŒ๋‹จํ•˜๊ฒ ์Œ. ํ•„์š”ํ•˜๋‹ค๋ฉด ์ด์ฑ…์ด ๋ณด์ด์ง€ ์•Š๊ฒŒ ์—ญํ• ํ•˜๊ฒ ์Œ.
- ์ „๋ถ์ง€์—ญ
1) ํ•˜๋ฐ˜๊ธฐ ๋“ค์–ด ์ „๋ถ์ง€์—ญ์ฑ…์ž„์ž์™€ ํšŒํ•ฉ์ด ์›๋งŒํžˆ ์ด๋ฃจ์–ด์ง€์ง€ ๋ชปํ•จ.
2) ์ง€์—ญ์„ ํ†ตํ•ด ๋“ค์€ ๋ฐ”๋กœ๋Š” ์ •์ƒ์ ์ธ ๋Œ€์ค‘ํ™œ๋™์„ ์ „๊ฐœํ•˜๊ณ  ์žˆ์Œ. (๋ช…์นญ 18 ์ƒ๋žต)์‚ฌ์—…, (๋ช…์นญ 19 ์ƒ๋žต) ์‚ฌ์—… ๋“ฑ ์ •์ƒ์ ์œผ๋กœ ์ง„ํ–‰์ค‘์ž„.
3) ์ƒ๋ฐ˜๊ธฐ ํšŒํ•ฉ์‹œ ์ „๋ถ์ง€์—ญ์ฑ…์ž„์ž๋Š” ํ‰ํ™”ํ†ต์ผ๊ณผ ์—ฐ๋ฐฉ์ œ์— ๋Œ€ํ•œ ์ดํ•ด์—์„œ ์ผ๋ฉด์  ์ดํ•ด๋ฅผ ๋“œ๋Ÿฌ๋ƒ„. ์ฆ‰ ๋งˆ์น˜ ์กฐ๊ตญํ†ต์ผ๋Œ€์ „์„ ๋ฌด๋ ฅ ํก์ˆ˜๋กœ ์ƒ๊ฐํ•˜๋Š” ํŽธํ–ฅ์žˆ์Œ. ๋Œ€๋ถ€๋ถ„ ํ™œ๋™๊ฐ€๋“ค์ด ์ด์ ์„ ์ดํ•ดํ•˜์ง€ ๋ชปํ•ด ์ •์„ธ๊ฐ€ ๊ทน์ ์œผ๋กœ ํ๋ฅผ๋•Œ ๋‚จ์ชฝ ํ™œ๋™๊ฐ€๋“ค ์†์—์„œ๋Š” ์ฃผ์ถคํ•˜๋Š” ํ˜„์ƒ์ด ๋‚˜ํƒ€๋‚จ. ์ด์— ๋Œ€ํ•œ ๊ต์–‘๋ฌผ์ด ํ•„์š”ํ•จ. ์ฆ‰ ์ „์Ÿ์€ ๋ฐ˜ํ†ต์ผ์„ธ๋ ฅ๊ณผ์˜ ํˆฌ์Ÿ์ด ๋ณธ์งˆ์ด์š” ์ด๋‚จ์„ ์‚ฌํšŒ์ฃผ์˜ํ™”ํ•˜๋Š” ๊ฒƒ์ด ์•„๋‹Œ ์—ฐ๋ฐฉ์ œ๋กœ ํ†ต์ผํ•˜๊ธฐ ์œ„ํ•œ ๊ฒƒ์ด๋ฏ€๋กœ ๊ทธ๊ฒƒ์€ ๋ฐ˜ํ†ต์ผ์„ธ๋ ฅ์„ ๋ฐฐ์ œํ•œ, ๋‚จ๋ถ์ •์น˜ํ˜‘์ƒ์— ์˜ํ•œ ํ‰ํ™”์ ์ธ ํ†ต์ผ์ด๋ผ๋Š” ์ ์„ ๊ฐ•์กฐํ•˜๋Š” ๊ต์–‘ ์ž๋ฃŒ๊ฐ€ ํ•„์š”ํ•จ.
4) ์ง€์—ญ์ฑ… ๋™์ง€์˜ ์ถฉ์‹ค์„ฑ๊ณผ ์›์น™์  ์ž์„ธ๋Š” ์‹ ๋ขฐํ•  ์ˆ˜ ์žˆ์Œ. ์—ฐ๋ง์— ํšŒํ•ฉ์„ ํšŒ๋ณตํ•ด ์ •์ƒ์ ์œผ๋กœ ์‚ฌ์—…ํ•˜๊ฒ ์Œ.
- ๊ธฐํƒ€
1) ์ „๋ถ โ–ถ์„ ์ƒ์€ ์ •๊ธฐ์ ์œผ๋กœ ํšŒํ•ฉ์„ ํ•˜๋ฉฐ ๊ณ ๋ฏผ๊ณผ ์˜๊ฒฌ์„ ๋งŒ์ด ๋‚˜๋ˆ”. ์กฐ์ง๊ด€๋ฆฌ์™€ ์ •์„ธ, ์‚ฌ์ƒ ํ๋ฆ„์— ํฐ ๋„์›€์„ ์ฃผ๊ณ  ์žˆ์Œ.
2) ์ „๋ถ โ™ ๋ชฉ์‚ฌ๋„ ๋น„๋ก ์ธ๊ฐ„๊ด€๊ณ„ ์ฐจ์›์—์„œ ๋งŒ๋‚˜์ง€๋งŒ ์†”์งํ•œ ์ด์•ผ๊ธฐ๋ฅผ ๋‚˜๋ˆ„๋ฉฐ ํ˜‘์กฐํ•ด ์ฃผ๊ณ  ์žˆ์Œ.
3) ์‹ ๋ฌธ์‚ฌ์—…๊ณผ ์—ฐ๊ตฌ์†Œ์‚ฌ์—…, ์šฐ๋ฆฌ์กฐ์ง์‚ฌ์—… ๋“ฑ ์ข…ํ•ฉ์  ๋ชฉ์ ์„ ์œ„ํ•ด 9์ธ์Šน ์ฐจ๋Ÿ‰์„ ๊ตฌ์ž…ํ–ˆ์Œ. 5๋…„(60๊ฐœ์›”) ํ• ๋ถ€๋กœ ์ฐจ๋Ÿ‰๊ฐ€๊ฒฉ์€ 3๋งŒ๋ถˆ ์ •๋„ํ•จ. 5๋…„ ํ• ๋ถ€์ด๋ฏ€๋กœ ํฐ ๋ถ€๋‹ด ์—†์Œ. ์‹ ๋ฌธ์‚ฌ์˜ ์ˆ˜์ž…์ผ๋ถ€๊ฐ€ ๋“ค์–ด๊ฐ€๋‚˜ ๋ณธ์‚ฌ์˜ ์ง€์›์„ ์—ผ๋‘์— ๋‘” ๊ฒƒ์ž„. ์˜ฌํ•ด ์ˆ˜์ค€์—์„œ ์ž๊ธˆ ์ง€์›์„ ์š”์ฒญํ•จ. ํ–ฅํ›„ ์‹ ๋ฌธ ํ†ตํ•ด ์‹ ๋ถ„์•ˆ์ „๊ณผ ์ •๋ณด์ˆ˜์ง‘, ํ•ด์™ธ์—ฌํ–‰, ๋Œ€์ค‘๊ต์–‘ ๋“ฑ ๋‹ค์–‘ํ•œ ์ด์ต์„ ๋ณผ ์ˆ˜ ์žˆ์Œ.
4) ์ง€๋‚œ๋ฒˆ ์•”ํ˜ธํ”„๋กœ๊ทธ๋žจ ์ˆซ์ž๊ฐ€ 46๋ฒˆ์ด๋ผ๊ณ  ํ–ˆ์œผ๋‚˜ ๋‹ค์‹œ ํ™•์ธํ•ด ๋ณด๋‹ˆ 5๊ฐœ์˜ ์—ฌ์œ ๊ฐ€ ๋” ์žˆ์—ˆ์Œ. ํ˜„์žฌ 4245๋ฒˆ, 4750๋ฒˆ๊นŒ์ง€ ์‚ฌ์šฉ๊ฐ€๋Šฅํ•œ ๊ฒƒ์œผ๋กœ ํŒŒ์•…๋จ. ๋ณธ์‚ฌ์˜ ํ™•์ธ์„ ์š”๋งํ•จ. ์ด๋ฒˆ์—๋Š” 47๋ฒˆ์„ ์‚ฌ์šฉํ•˜๊ฒ ์Œ.
5) ์‹ ๋…„์ถ•ํ•˜๋ฌธ๊ณผ ์›์ˆ˜๋‹˜ ํƒ„์ƒ ์ถ•ํ•˜๋ฌธ์„ 12์›” ์ดˆ์— ์˜ฌ๋ฆฌ๊ฒ ์Œ.
6) ๋‹น์ฐฝ๊ฑด 70๋Œ์˜ ์›์ˆ˜๋‹˜ ์—ฐ์„ค๊ณผ ์—ด๋ณ‘์‹์€ ์กฐ์ง๊ณผ ์„ฑ์›์„ ํ•œ์—†์ด ๊ณ ๋ฌดํ•˜๊ณ  ์žˆ์œผ๋ฉฐ, ๋‚ด๋…„ 7์ฐจ ๋‹น๋Œ€ํšŒ ๋ฐœํ‘œ๋Š” ๋‹ค๊ฐ€์˜ค๋Š” ํ†ต์ผ์กฐ๊ตญ์˜ ๋‚ ์„ ํ™•์‹ ํ•˜๊ณ  ์žˆ์Šต๋‹ˆ๋‹ค. ์กฐ์ง์€ ๋‹ค๊ฐ€์˜ค๋Š” ํ†ต์ผ์กฐ๊ตญ์˜ ์ •์„ธ์— ๋”์šฑ ๋ณต๋ฌดํ•˜๊ฒ ์œผ๋ฉฐ, ์ด๋‚จ ๋ณ€ํ˜์šด๋™์˜ ์•™์–‘์„ ์œ„ํ•ด ๋”์šฑ ๊นŠ์ด ํ—Œ์‹  ๋ณต๋ฌดํ•˜๊ฒ ์Šต๋‹ˆ๋‹ค.
์ด์ƒ ๋.
2015๋…„ 11์›” 12์ผ
์„œ์šธ. โ™ฅโ™ฅโ™ฅโ™ฅโ™ฅํšŒ
์ด๋กœ์จ ํ”ผ๊ณ ์ธ์€ ๊ตญ๊ฐ€์˜ ์กด๋ฆฝยท์•ˆ์ „์ด๋‚˜ ์ž์œ ๋ฏผ์ฃผ์  ๊ธฐ๋ณธ์งˆ์„œ๋ฅผ ์œ„ํƒœ๋กญ๊ฒŒ ํ•œ๋‹ค๋Š” ์ •์„ ์•Œ๋ฉด์„œ ๋ฐ˜๊ตญ๊ฐ€๋‹จ์ฒด ๊ตฌ์„ฑ์› ๋˜๋Š” ๊ทธ ์ง€๋ น์„ ๋ฐ›์€ ์ž์™€ ํ†ต์‹ ์—ฐ๋ฝํ•˜๊ณ , โ€˜225๊ตญโ€™ ์†Œ์† ๊ณต์ž‘์›์—๊ฒŒ ํŽธ์˜๋ฅผ ์ œ๊ณตํ•˜์˜€๋‹ค. 3. ์ด์ ๋™์กฐ ๋ฐ ์ด์ ํ‘œํ˜„๋ฌผ ์†Œ์ง€์˜ ์  ๊ฐ€. 2013. 11. 4. ๊ณต์†Œ์™ธ 4์™€ ์‚ฌ์ƒํ•™์Šต(์ด์ ๋™์กฐ) ํ”ผ๊ณ ์ธ์€ 2013. 11. 4.๊ฒฝ ์„œ์šธ ์ข…๋กœ๊ตฌ (์ฃผ์†Œ 4 ์ƒ๋žต)์— ์žˆ๋Š” (๋ช…์นญ 53 ์ƒ๋žต)ํšŒ๊ด€ ๋‚ด (๋ช…์นญ 29 ์ƒ๋žต)์—ฐ๊ตฌ์†Œ์—์„œ, ๊ณต์†Œ์™ธ 4๋ฅผ ๋งŒ๋‚˜ ๋‚จํ•œ ๋ณ€ํ˜์šด๋™๊ณผ ํ†ต์ผ์šด๋™์˜ ๋ชฉํ‘œ ๋ฐ โ€™๋ฏผ์กฑํ•ด๋ฐฉ๋ฏผ์ฃผ์ฃผ์˜ํ˜๋ช…๋ก โ€˜ ๋“ฑ์— ๊ด€ํ•˜์—ฌ ํ† ๋ก ํ•˜๋Š” ๋“ฑ ์‚ฌ์ƒํ•™์Šต์„ ์ง„ํ–‰ํ•˜์˜€๋Š”๋ฐ, ๊ทธ ์ฃผ์š” ๋Œ€ํ™”๋‚ด์šฉ์€ ๋‹ค์Œ๊ณผ ๊ฐ™๋‹ค. ใ€Žใ€Š2013. 11. 4. ํ”ผ๊ณ ์ธ๊ณผ ๊ณต์†Œ์™ธ 4 ๊ฐ„์˜ ์ฃผ์š” ๋Œ€ํ™”๋‚ด์šฉใ€‹ (ํ”ผ๊ณ ์ธ) : ์•„ ์ด๋ฒˆ์— ๋ณด๋‹ˆ๊นŒ, ๋ฟŒ๋ฝ์น˜ ๊ทธ๋†ˆ ์žˆ์ž–์•„. ๊ณต์†Œ์™ธ 41 ์‚ฌ๊ฑด (ํ”ผ๊ณ ์ธ) : 2010๋…„๋ถ€ํ„ฐ ๋ฟŒ๋ฝ์น˜ ํ™œ๋™ํ–ˆ๋”๋ผ๊ณ  (ํ”ผ๊ณ ์ธ) : ๋…น์ŒํŒŒ์ผ์ด 70 ๋ช‡ ๊ฐœ๋ž˜, ๊ฐ์ฒญ์˜์žฅ์„ ๋ฐ›๊ณ ๋‚˜์„œ ํ•œ ํŒŒ์ผ์€ 40 ๋ช‡ ๊ฐœ์ด๊ณ , ๊ฐ์ฒญ์˜์žฅ์„ ๋ฐ›๊ธฐ ์ „์— ํ•œ ํŒŒ์ผ์€ 30 ๋ช‡ ๊ฐœ๋ž˜ (๊ณต์†Œ์™ธ 4) : ์›๋ž˜ ๋ฟŒ๋ฝ์น˜๊ฐ€ ๋ช‡ ๋…„์”ฉ ์šธ๊ถˆ๋จน๋Š” ๊ฒŒ ๋ฟŒ๋ฝ์น˜์ฃ . (ํ”ผ๊ณ ์ธ) : ์กฐ์ง์—์„œ ๊ด€๋ฆฌํ•˜๊ณ  ์ฑ…์ž„์ง€๋Š” ์‚ฌ๋žŒ ๋Š๋‚Œ์€ ์žˆ์„ ๊ฑฐ๋ž€ ๋ง์•ผ ๋Š๋‚Œ์ด, ๋ฌธ์ œ๋Š” ์‚ฌ์ƒ์ด์•ผ ์‚ฌ์ƒ, ๋ญ”๊ฐ€ ๊ทธ๊ฒƒ์— ์ด๋ ‡๊ฒŒ ๋…น์ด ์Šจ๋‹ค๊ณ  ํ• ๊นŒ (ํ”ผ๊ณ ์ธ) : ๊ทธ๋ฆฌ๊ณ  ์˜ค๋žœ ๊ธฐ๊ฐ„ ๋™์•ˆ์„ ์‚ฌ๋žŒ์„ ๋‹ค ์•Œ์•„์˜จ ์‚ฌ๋žŒ๋“ค ์•„๋ƒ, ๊ทธ๋Ÿฐ๋ฐ ๊ทธ๊ฒŒ ์ดํ•ดํ•  ์ˆ˜ ์žˆ๊ฑฐ๋‚˜ ๋ญ ๋™์ง€๋ฅผ ๋ฏฟ์ง€ ์•Š๋Š” ๊ฒƒ์ด ์•„๋‹ˆ๋ผ ๋ฏฟ์œผ๋ฉด์„œ ๊ฒ€์—ดํ•˜๋Š” ๊ฒƒ์ด๋ผ๊ณ  ํ•˜๋Š” ๊ฑฐ๊ฑฐ๋“ , ๋ฏฟ์Œ์€ ๋ฏฟ์Œ์ด๊ณ  ๊ฒ€์—ด์€ ๊ฒ€์—ด์ด์•ผ. ๋ถˆ์‹œ์— ์šฐ๋ฆฌ๋„ ๊ฒ€์—ดํ•˜๋Š” ๊ฑฐ์ง€ (ํ”ผ๊ณ ์ธ) : ๊ทธ ์ค„๊ธฐ์—์„œ ์„œ๋กœ ์„ž์ง€ ๋ง๊ณ  ์šฐ๋ฆฌ๊ฐ€ ์ข€ ๋” ์ง„๋ณด์ ์œผ๋กœ ์–˜๊ธฐํ•  ๋•Œ๋Š” ๊ทธ๊ฑด ๋ณ€ํ˜์šด๋™ ์–˜๊ธฐ๋ž€ ๋ง์•ผ, ๋ฐ˜๋ฐ•ํ•  ์ด์œ ๊ฐ€ ์—†์ง€ ๊ทธ๋ž˜์„œ ๊ทธ๋Ÿฐ ํˆฌ์Ÿ์€ ํ•˜๋Š” ๊ทธ๊ฑด ๋ณ€ํ˜์šด๋™ ์–˜๊ธฐ๋ž€ ๋ง์•ผ ์‘, ๊ทธ๋ฆฌ๊ณ  ํ‰ํ™”์ ์œผ๋กœ ์ด๋Ÿฐ ๊ฑฐ๋Š” ํ†ต์ผ์šด๋™ ์–˜๊ธฐ๋ž€ ๋ง์•ผ, ํ•˜๋‚˜์˜ ์ฃผ์ฒด๊ฐ€ ๋‘ ๊ฐœ์˜ ์–ด๋–ป๊ฒŒ ๋ณด๋ฉด ์—ฐ๊ด€๋ผ ์žˆ๋Š” ๋ถ€๋ถ„์ด๊ณ , ํ†ต์ผ์šด๋™์ด ๋ณ€ํ˜์šด๋™์˜ --- ํ•˜๊ณ  ๋ณ€ํ˜์šด๋™์˜ ๋‹จ์ดˆ๋Š” ํ†ต์ผ์šด๋™์˜ --- ํ•˜๊ณ  ์ด๋Ÿฐ๋‹ค๋ฉด์€ ์šฐ๋ฆฌ๊ฐ€ ์ด ๋‘˜์„ ์ž˜ ๋ด์•ผ์ง€ ๋˜๋Š” ๊ฒƒ์ด์ง€, ๊ทธ๋Ÿฌ๋‹ˆ๊นŒ ๋ญ ์ € ๊ฒจ๋ ˆํ•˜๋‚˜๋Š” ์š”์ฆ˜ ํ†ต์ผ์šด๋™์˜ ๋ฒ”์ฃผ์— ์žˆ๋Š” ๊ฑฐ ์•„๋ƒ (๊ณต์†Œ์™ธ 4) : ๊ทผ๋ฐ ๊ทธ๊ฑฐ๋ฅผ ๊ทธ๋ ‡๊ฒŒ ์ดํ•ดํ•˜๊ธฐ ๋ณด๋‹ค๋Š” ์˜คํžˆ๋ ค ์ €๋Š” ์šด๋™์ด, ๋ณ€ํ˜์šด๋™์ด๋ผ๊ณ  ํ•˜๋‚˜๊ฐ€ ์•„๋‹ˆ๊ณ , ๊ทธ ์†์—์„œ ํ†ต์ผ์šด๋™๊ณผ ๋…ธ๋™์šด๋™๊ณผ ์ด๋ ‡๊ฒŒ ๋ณ€ํ•˜๋Š” ๊ทธ๋ ‡๊ฒŒ ์ดํ•ด๋ฅผ ํ•ด์•ผ ๋˜์ง€ ์•Š๊ฒ ๋‚˜ (ํ”ผ๊ณ ์ธ) : ํ†ต์ผ์šด๋™์„ ๋ณ€ํ˜์šด๋™์˜, ๋ณ€ํ˜์šด๋™๊ณผ ์—ฐ๊ด€ ์žˆ๊ธฐ๋Š” ํ•˜์ง€๋งŒ ํ†ต์ผ์šด๋™์˜ ์›์ธ์€ ๋ฏผ์กฑํ•ด๋ฐฉ์— ์žˆ๊ณ , ๋ณ€ํ˜์šด๋™์˜ ์›์ธ์€ ์ด๊ฑฐ์•ผ ๊ณ„๊ธ‰ํˆฌ์Ÿ์— ์žˆ๋‹ค. ์„œ๋กœ๊ฐ„์˜ ์›๋ฆฌ๊ฐ€ ๋‹ฌ๋ผ. ์›๋ฆฌ๊ฐ€ ๋‹ฌ๋ผ ์ด๊ฒƒ์ด, ์ด๊ฒƒ์„ ์„ž์–ด ๋ฒ„๋ฆฌ๋ฉด ๋ณ€ํ˜์šด๋™์€, ์šด๋™์ด๋ผ๋Š” ๊ฒƒ์„ ๋ณ€ํ˜์šด๋™๊ณผ ํ†ต์ผ์šด๋™ ์ด๊ฑด ์ž˜๋ชป๋œ ๊ทœ์ •์ด์•ผ. ๋…ธ๋™์šด๋™, ๋ฌด์Šจ ์šด๋™, ๋ฌด์Šจ ๋†๋ฏผ์šด๋™, ์ฒญ๋…„์šด๋™, ํ•™์ƒ์šด๋™ ๊ทธ๋Ÿฐ ์šด๋™์ด ํ†ต์ผ์šด๋™์„ (ํ”ผ๊ณ ์ธ) : ์ง„๋ณด์šด๋™์˜ ๋…ผ๋ฆฌ์™€ ์›๋ฆฌ๋Œ€๋กœ ํ†ต์ผ์šด๋™์„ ํ•  ์ˆ˜ ์—†๋Š” ๊ฑฐ์•ผ. ๊ทธ๋ƒฅ ๋ฏผ์กฑ๋Œ€๋‹จ๊ฒฐ, ์„œ๋กœ ์›๋ฆฌ๊ฐ€ ๋‹ฌ๋ผ. ์šฐ๋ฆฌ๊ฐ€ ์ด๊ฒƒ, ์ด๊ฑด ์ด๋ก ์˜ ์•„์ฃผ ๊ธฐ๋ณธ์ธ๋ฐ ๋ฏผ์กฑํ•ด๋ฐฉ๋ฏผ์ฃผ์ฃผ์˜ ๋ผ๋Š” ์šด๋™์˜ ๊ธฐ๋ณธ์ด์•ผ. ๊ทธ๋ž˜์„œ ํ•ญ์ƒ ํ•œ๊ตญ์‚ฌํšŒ๋ณ€ํ˜์šด๋™๋ก ๊ณผ ์ด๋‚จ์‚ฌํšŒ๋ณ€ํ˜์šด๋™๊ณผ ์ •์น˜๋ณ€ํ˜๋ก  ๋˜ ์„ธ๊ณ„์‚ฌ๋ก  ์ด๋ ‡๊ฒŒ ๊ฐ€๋Š” ๊ฑฐ์•ผ (๊ณต์†Œ์™ธ 4) : ๊ทธ ์ง€์ ์ด ํ—ท๊ฐˆ๋ ค ๊ฐ€์ง€๊ตฌ, ๊ทธ๊ฑฐ์— ๋Œ€ํ•œ ์ž๋ฃŒ๋‚˜ ์ฑ…์ž๋Š” ํ˜•๋‹˜ ์–ด๋–ค ์ฑ…์„ ๋ด์š”? (ํ”ผ๊ณ ์ธ) : ๋ฏผ์กฑํ•ด๋ฐฉ๋ฏผ์ฃผ์ฃผ์˜ํ˜๋ช…๋ก ! (ํ”ผ๊ณ ์ธ) : ์šฐ๋ฆฌ๋Š” ์šฐ๋ฆฌ๊ฐ€ ํŽผ์น˜๋Š” ๋ณ€ํ˜์šด๋™๊ณผ ํ†ต์ผ์šด๋™์„ ๋™์‹œ์— ํ•˜๋Š” ์‚ฌ์—…์ด์ง€ (ํ”ผ๊ณ ์ธ) : ๋ชฉํ‘œ๋„ ๋ชฉํ‘œ๊ฐ€ ๋‹ค๋ฅธ ๊ฒŒ, ๋ณ€ํ˜์šด๋™์˜ ๋ชฉํ‘œ๋Š” ์ •๊ถŒ์Ÿ์ทจ์•ผ, ํ†ต์ผ์šด๋™์˜ ๋ชฉํ‘œ๋Š” ๋ฏผ์กฑํ†ต์ผ์ •๊ถŒ์ด ๋ชฉํ‘œ์•ผ. ๊ทธ๋Ÿฌ๋ฉด ๋ฏผ์กฑํ†ต์ผ์ •๊ถŒ๊ณผ ์ด ๋ณ€ํ˜์šด๋™์—์„œ ๋งŒ๋“  ์ •๊ถŒ, ๊ทธ๊ฑฐ๋Š” ๋‚จ์ชฝ์—๋Š” ์ž์ฃผ์  ๋ฏผ์ฃผ์ •๋ถ€ ์ˆ˜๋ฆฝ, ์ž์ฃผ์  ๋ฏผ์ฃผ์ •๋ถ€ ์ˆ˜๋ฆฝ์ด๊ณ  ์ž์ฃผ์  ๋ฏผ์ฃผ์ •๋ถ€๊ฐ€ ๋ฏผ์กฑํ†ต์ผ ์ •๋ถ€์˜ ์ง€์—ญ์  ์ •๊ถŒ์ด ๋˜๋Š” ๊ฑฐ์•ผ ์—ฐ๋ฐฉ์ œ ํ•˜์—์„œ, ๊ทธ๋ฆฌ๊ณ  ์ „๊ตญ์ ์ธ ์ •๊ถŒ์€ ๋ฏผ์กฑํ†ต์ผ์ •๊ถŒ์ด ๋˜๋Š” ๊ฑฐ์•ผ. ๋ฏผ์กฑํ†ต์ผ์ •๊ถŒ๊ณผ ์ง€์—ญ์— ์žˆ๋Š” ์ž์ฃผ์  ๋ฏผ์ฃผ์ •๊ถŒ์€ ์„œ๋กœ ๋‹ค๋ฅธ ๋ณ€ํ˜๊ณผ์ œ๋ฅผ ๊ฐ€์ง€๊ณ  ์žˆ๋Š” ๊ฑฐ์•ผ (ํ”ผ๊ณ ์ธ) : ์•ž์œผ๋กœ ๋‹จ๊ณ„์ ์ธ ๋ณ€ํ˜์šด๋™์„ ๊ฑฐ์ณ์„œ ์ž๋ณธ์ฃผ์˜ ๋‹จ๊ณ„๋ฅผ ์ฒ˜์Œ์—๋Š” ์ง„๋ณด์  ๋ณ€ํ˜์ด๋ผ๊ณ  ๊ทธ๋Ÿฌ์ง€, ์ง„๋ณด์ ์ธ --- ํ•˜๊ณ  ๊ทธ๋ ‡๊ฒŒ ํ•˜๊ณ ์„œ ๊ทธ ๋’ค์— ์‚ฌํšŒ์ฃผ์˜๋กœ ๋„˜์–ด๊ฐ€๋Š” ๊ฑฐ๊ณ  ๊ทธ๋ž˜์„œ ์‚ฌํšŒ์ฃผ์˜ ์™„์ „์Šน๋ฆฌ๋ฅผ ํ–ฅํ•ด์„œ ์ด๋ฏธ ์‚ฌํšŒ์ฃผ์˜๊ฐ€ ๋์œผ๋‹ˆ๊นŒ ๊ทธ๋ž˜์„œ ๊ฐ€๋Š” ๊ฑธ๋กœ ์ด๊ฒƒ์„ ์ „์ฒด์ ์œผ๋กœ ์ „์ฒด ๋ฐ›๋Š” ์ชฝ์—์„œ ํ•˜๋Š” ๊ฒƒ์€ ๋ฏผ์กฑํ†ต์ผ์ •๋ถ€, ๊ทธ๋ฆฌ๊ณ  ๋ถ์— ์ •๋ถ€๋„ ์žˆ๋Š” ๊ฑฐ์•ผ ํ•˜์œ„์ ์œผ๋กœ ๋‚จ์— ์ •๋ถ€๋„ ์žˆ๊ณ  ์ž์ฃผ๋ฏผ์ฃผ์ •๋ถ€, ๋ฏผ์กฑํ†ต์ผ์ •๋ถ€๋„, ๊ทธ๋Ÿฌ๊ธฐ ๋•Œ๋ฌธ์— ์„œ๋กœ ๊ฐ„์— ์ž„๋ฌด์™€ ์—ญํ• ์ด ๊ฐ™์€ ๊ฑฐ์•ผ (๊ณต์†Œ์™ธ 4) : ์‘, ๊ทธ๋Ÿฌ๋‹ˆ๊น ์˜ˆ๋ฅผ ๋“ค์–ด์„œ (ํ”ผ๊ณ ์ธ) : ๊ทธ๋Ÿฐ, ๊ทธ๋Ÿฌ๋‹ˆ๊นŒ ๋ฏผ์กฑํ†ต์ผ์ •๊ถŒ์˜ ์ž์ฃผ์ ๋ฏผ์ฃผ์ •๊ถŒ์ด ํ•˜๋‚˜์˜ ๊ตฌ์„ฑ๋ถ€๋ถ„์ด๊ธฐ๋Š” ํ•˜์ง€๋งŒ์€ ์ž๊ธฐ์˜ ๋‹น๋ฉด ๊ณผ์ œ๋Š” ์ž์ฃผ์ •๋ถ€์ˆ˜๋ฆฝ ์„ธ์šฐ๊ณ  ์ง„๋ณด์  ๋ฏผ์ฃผ๋ณ€ํ˜์„ ํ•˜๋Š” ๊ฒƒ์ด ์ด๋‚จ์˜ ํ‰์†Œ ์‚ฌํšŒ๋ณ€ํ˜์šด๋™์ด์ง€ (๊ณต์†Œ์™ธ 4) : ์˜ค์ผ€์ด (ํ”ผ๊ณ ์ธ) : ๊ทธ๋Ÿฌ๊ธฐ ๋•Œ๋ฌธ์— ๊ทธ๋Ÿฐ ๋ถ€๋ถ„๋“ค์„ ํ˜น๊ฐ„ ์ด๊ฒŒ ์–ด๋ ค์šด๊ฑด ์•„๋‹Œ๋ฐ ์ด๋Ÿฐ ๋…ผ์˜๋ฅผ ๊ทธ๋•Œ๊ทธ๋•Œ ๋–ผ๋‹ค๋†“๋Š”, ๊ทธ๋ฆฌ๊ณ  ์ด์ œ ์ด ๋ชจ๋“  ๊ฒƒ์„ ์ „์ฒด๋กœ ํ•˜๋Š” ๊ฒƒ์ด ๋ฐ”๋กœ ์ „๊ตญ์ ์ธ ๋‹น๊ณผ ์ „๊ตญ์ „์ธ ํ†ต์ผ์šด๋™ ๋‹น์ด ํ†ต๊ณผ์ฒ˜๋Ÿผ ์ง„ํ–‰ํ•˜๋Š” ๊ฑฐ์ง€ (๊ณต์†Œ์™ธ 4) : ํ•œ๊ตญ์‚ฌ์˜ ๋ณ€ํ˜์šด๋™ ์žˆ์œผ๋‹ˆ๊นŒ ์ €๋„ ์ฑ… ์ œ๋ชฉ์€ ๋ณธ ๊ฒƒ ๊ฐ™์•„์š” (ํ”ผ๊ณ ์ธ) : ๊ทธ๋ž˜์„œ ์ด์ œ ์ฒซ์งธ ๊ณ„์† ๋…ผ์˜ํ•˜๋Š” ๊ฒƒ์€ ์šฐ๋ฆฌ๋Š” ๊ทธ๋Ÿฐ ์ผ์„ ํ•˜๋Š” ๊ฒƒ์ธ๋ฐ, ๊ฑฐ๊ธฐ๋Š” ์ด์ œ ๋Œ€๋ถ์ง€์›์„ ํ†ต์ผ์šด๋™ ํ•˜๋Š”๋ฐ ๊ธฐ์—ฌํ•˜๊ณ  ์ด๋Ÿฌ๊ณ , ๊ต์œกํ•  ๋•Œ ์˜์‹ํ™”๋˜๊ณ  ์žˆ๋Š” ์‚ฌ๋žŒ๋“ค์ด ๊ทธ๋ฆฌ๊ณ  ๋‚˜์ค‘์— ์ž์ฃผ์ ์œผ๋กœ ์ง„๋ณด์—ฐ๋Œ€ ํ•˜๋Š”๋ฐ ํ•œ ์ฃผ์ฒด๊ฐ€ ๋” ๋˜๋„๋ก ๋Œ€์ค‘์„ ๊ทธ๋ ‡๊ฒŒ ๊ทธ๋Ÿฐ ์‹์œผ๋กœ ํ•˜๋Š” ๊ฒƒ๋„ ํ•˜๋Š” ๊ฒƒ์ด์ง€ (๊ณต์†Œ์™ธ 4) : ๊ทธ๊ฑธ ์ฃผ๊ด€์ฃผ์˜์  ํƒœ๋„, --- ๊ณต์†Œ์™ธ 42, ์ฃผ๊ด€์  ์ถœ์„ธ์ฃผ์˜๋ผ๊ณ  ํ•ด์•ผ ํ•˜๋‚˜ ํ•˜์—ฌ๊ฐ„ ๋ญ ๊ณต์†Œ์™ธ 42 ์–˜๊ธฐ... ๊ณต์†Œ์™ธ 43 ์„ ์ƒ๋‹˜์ด ๊ทธ๋Ÿฐ ์ธ์‹์˜ ํ‰๊ฐ€๋ฅผ ์•„ ๊ทธ ์–˜๊ธฐ, ์•„ ๊ณต์†Œ์™ธ 42 ์–˜๊ธฐ, ๊ทผ๋ฐ ์ฃผ๊ด€์  ์ถœ์„ธ์ฃผ์˜์ž์ž–์•„์š”. (๊ณต์†Œ์™ธ 4) : ๊น€์ผ์„ฑ ์ˆ˜๋ น์„ ๋งŒ๋‚˜๊ณ  ์™€์„œ๋„, ์˜คํžˆ๋ ค ๊ฑฐ๊ธฐ์„œ ์ด์ œ ๋งŒ๋‚˜๊ฐ–๊ณ  ์˜คํžˆ๋ ค ๊ฑฐ๊ธฐ์„œ ์ด์ œ ์ด๋ ‡๊ฒŒ ์„ ๋ณ„์ด ๋๋‹ค ๊ทธ๋ ‡๊ฒŒ, ์•„ ์ง„์งœ๋กœ --- ๊ฒฐ๊ณผ๋ฉด์—์„œ์š”. ์ˆ˜๋ น๋‹˜์ด ๋ณด์‹œ๋ฉด ๋Œ€๋‹จํ•œ ์ˆ˜์žฌ๋กœ... ๋™๋„ค ํ• ์•„๋ฒ„์ง€ ๊ฐ™์ดโ€˜๋ฐฅ ๋จน์—ˆ์–ดโ€™์•ผ (ํ”ผ๊ณ ์ธ) : ๊ทธ๋ž˜์„œ ์ธ์ œ ์•ž์œผ๋กœ ์กฐ๊ธˆ์”ฉ ์กฐ๊ธˆ์”ฉ ์ฑ…์ž„์ง€๊ณ  ๋‚˜๊ฐ€๋ฉด์„œ ์‹ค์ œ ๊ฒฝํ—˜์„ ์ฑ„์›Œ๋ผ, ๋‚ด๋ฉด์ ์œผ๋กœ ๊ด€๋ฆฌ, ๊ทธ ์ฒด๊ณ„ ์†์—์„œ ์žˆ๋Š” ๊ฑฐ ์•„๋‹ˆ์•ผ. ๋‚˜๋„ ์‚ฌ์‹ค์€ ๋‚˜๋Š” 10๋…„ ๋ผ๊ฐ€์ž–์•„, ์ง€๊ธˆ๋„ ์–ด๋ ค์›Œ, ๊ทผ๋ฐ ํ•œ ๋งˆ๋””๋„ ๋ชปํ•ด, ๊ทผ๋ฐ ํ•  ์–˜๊ธฐ๋Š” ๋‹คํ•ด ํ•  ์–˜๊ธฐ๋Š” ๋‹ค ํ•˜๋Š”๋ฐ, ์–ด๋–ค ๋•Œ๋Š” ์ง์ ‘ ํ•˜๊ธฐ๋„ ํ•˜๊ณ , ์–ด์จŒ๋“  ๋‚˜๋กœ์„œ ํ•  ๊ฐ์ •์ด๋ผ๋“ ๊ฐ€ ๊ทธ ๋‚ด์šฉ์ ์ธ ๊ฒƒ๋“ค์„ ๋‹ค ํ•˜๊ธด ํ•˜๋Š”๋ฐ ๊ทธ๋ž˜๋„ ์–ด๋ ค์›Œ, ๊ทธ๋ž˜๋„ ์–ด๋ ค์›Œ, (ํ”ผ๊ณ ์ธ) : ๋‚˜๋Š” ๊ธฐ๋ณธ์ ์ธ ์ž…์žฅ์€ ์—์ด์ „ํŠธ๋ผ๊ณ  ์ƒ๊ฐํ•ด, ์ง€์‚ฌ์˜ ์—์ด์ „ํŠธ์ง€ ๋‚ด๊ฐ€ ๊ฒฐ์ •ํ•  ๊ถŒํ•œ์„ ๊ฐ€์ง€๊ณ , ์ „๊ถŒ์„ ๊ฐ€์ง€๊ณ , ๋ฌผ๋ก  ๊ทธ๋ ‡๊ฒŒ ํ•˜๋Š” ์‚ฌ๋žŒ๋„ ์žˆ๋”๋ผ๊ตฌ, ์ „๊ถŒ๊นŒ์ง€๋Š” ์•„๋‹ˆ์ง€๋งŒ ๋‚˜๋ฆ„ ์ „๊ถŒ์— ๊ฐ€๊นŒ์šด ๊ถŒํ•œ์„ ๊ฐ€์ง€๊ณ  ํ•˜๋Š” ์‚ฌ๋žŒ๋„ ์žˆ์–ด (๊ณต์†Œ์™ธ 4) : ๊ทผ๋ฐ ๊ทธ์ชฝ์—์„œ ์ด์ œ ๋„ˆ๋Š” ์‹ค๋ฌด์ž์ง€ ์ง€๋„์ƒ์œ„ ๊ทธ๊ฑด ์•„๋‹ˆ๋‹ค ๊ฑฐ๊ธฐ์— ์ด๋ ‡๊ฒŒ ๋˜๋Š” ๊ฒ๋‹ˆ๊นŒ? ์ด๋ ‡๊ฒŒ ํ•˜๊ธฐ ์œ„ํ•œ, ์‰ฝ๊ฒŒ ์–˜๊ธฐํ•˜๋ฉด ๊ทธ๊ฑฐ๋ฅผ ์—ฐ๋ฝ ๋‹ด๋‹นํ•˜๋Š” ์–ด๋–ค ์ฒญ์™€๋Œ€ ๋น„์„œ๊ด€์ด์ง€, ๋„ˆ๋Š” ์ฒญ์™€๋Œ€ ๋น„์„œ๊ด€์ด์ง€ ๋‚˜๋งŒ ์ฒœ์ƒ ์ฒญ์™€๋Œ€ ๊ฐ€์„œ ๋ณด๊ณ ๋ฅผ ํ•ด์•ผ ๋˜๋Š”๋ฐ, ๋‹น์—ฐํžˆ ๋น„์„œ๊ด€์ด์ง€ (ํ”ผ๊ณ ์ธ) : ์‚ฌ์‹ค ์šฐ๋ฆฌ๋Š” ๋ผ›์†๊นŒ์ง€ NL์ด์ž–์•„, ๋Œ€ํ•™ ๋‚˜๋Š” ์ด์ œ NL๋กœ ๊ทธ๋•Œ๋Š” NL๋กœ ์‹œ์ž‘ํ–ˆ์ž–์•„. ๋‚˜๋Š” 82๋…„๋„ โ–ณโ–ณ๋Œ€ ๋“ค์–ด๊ฐ€๋Š” ๊ณผ์ •์—์„œ ๋‚˜๋Š” ์–ด๋””๋ถ€ํ„ฐ ์‹œ์ž‘ํ–ˆ๋ƒ๋ฉด ์‹œ๋ฏผ๋ฏผ์ฃผ์ฃผ์˜, CDF, ์”จํ‹ฐ์ฆŒ ๋ฐ๋ชจํฌ๋ผ์‹œ, ๋‚˜๋Š” ๋‚ด๊ฐ€ ์™œ ์šด๋™์„ ํ–ˆ๋ƒ๋ฉด ๊ด‘์ฃผ๋ฏผ์ฃผํ™”์šด๋™ ์žˆ์—ˆ์ง€, ๋‘˜์งธ๋Š” ๋ฏผ์ฃผํ™”๋ฅผ ํ•ด์•ผ ๋œ๋‹ค. ๋Œ€ํ†ต๋ น, ์ด๊ฑธ๋กœ๋ถ€ํ„ฐ ์ถœ๋ฐœ์„ ํ•œ๊ฑฐ์•ผ. ๊ทผ๋ฐ ๊ทธ๋Ÿฌ๋‹ค ๋ณด๋‹ˆ๊นŒ ํ•™์ƒ์šด๋™ํ™” ๋˜๊ณ , --- ์กฐ์ง์—์„œ๋‚˜ ๊ทธ ๋•Œ โ–ณโ–ณ๋Œ€์—์„œ ์„œํด ๋™์•„๋ฆฌ ์ƒ๊ธด๊ฑฐ์•ผ. ๊ณต์†Œ์™ธ 44๊ฐ€ ๋งŒ๋‚œ๊ฑฐ์•ผ. ๋‚˜๋Š” ๊ทธ ๋Œ€์ƒ์ด ์•„๋‹ˆ์—ˆ์–ด, ์‹ ํ•™ํ•™๊ต ๋‹ค๋…”๊ณ  --- ์—๋‹ค๊ฐ€ ๋‹ค๋ฅธ --- ๊ฐ‘์ž๊ธฐ ๋‚˜ํƒ€๋‚œ ์‚ฌ๋žŒ์ด๋ผ ์ƒ๊ฐ๋˜๋‹ˆ๊นŒ ํ•™์ƒ์šด๋™์—, ๊ทธ๋ฆฌ๊ณ  ๋Œ€ํ•™ ์กธ์—…ํ•˜๊ณ  ๋‚˜์„œ ๋‚ด๊ฐ€ โ–ณ๋Œ€๋กœ ๋ฐ”๋กœ ๊ฐ€์•ผ๋˜๋Š”๋ฐ 1๋…„ ๊ฟ‡์—ˆ์–ด, ์‹ ํ•™์‹œํ—˜์„ ์•ˆ๋ณด๊ณ  ๋‚ด๊ฐ€ ์‚ฌํšŒ๊ณผํ•™ ๊ณต๋ถ€๋ฅผ ํ•œ ๊ฑฐ์•ผ. ๊ทธ๋ž˜๊ฐ€์ง€๊ณ  ์‚ฌํšŒ๊ณผํ•™ ๊ณต๋ถ€๋ฅผ 1๋…„ ๋™์•ˆ ํ•˜๋ฉด์„œ ๊ทธ๋•Œ ์ด์ œ NLPDR, (๋ช…์นญ 54 ์ƒ๋žต)๋Œ€ํ•™๊ต์— ๊ฐ•์‚ฌ๋กœ... NLPDR ์ฃผ์‚ฌ --- ๋‚ด ์ฃผ์‚ฌ ์ž…๋ฌธ๊นŒ์ง€ ๋”ฑ ๊ฐ•์˜๋ฅผ ๊ทธ๊ฑฐ๋ฅผ ํ•œ 3๊ฐœ์›” ํ•™์Šต์„ ๋ฐ›์•˜์–ด, ๊ทธ๊ฑฐ๋ฅผ 3๊ฐœ์›” ๋™์•ˆ ์ซ™ ๋ฐ›๊ณ ์„œ ์ตœ๊ณ ์˜ ์šฐ๋ฆฌํ•™๊ต๋กœ์„œ๋Š” ์ตœ๊ณ ์˜ NLPDR ์ด๋ก ๊ฐ€๊ฐ€ ๋์ง€... ๊ทธ๋Ÿฌ๊ณ  ์ด์ œ โ–ณ๋Œ€๋กœ ๊ฐ”๋Š”๋ฐ, ๊ทธ๋Ÿฌ๋‹ค๊ฐ€ ์ฒ˜์Œ์— PD์—์„œ ๊ทธ ๋‹ค์Œ์— ๊ฐ€์„œ ์ด์ œ NL์ชฝ์— ๊ด€์‹ฌ์„ ๊ฐ–๊ณ  ๊ทธ๋ž˜์„œ ์ด์ œ ์ฑ… ์‚ฌ๊ฐ€์ง€๊ณ , ๋‚ด๊ฐ€ ๋‚ด ์„œ์žฌ์— ๊ฐ€๋ฉด ์›์ „๋“ค์ด ๋‹ค ์žˆ๊ณ  (ํ”ผ๊ณ ์ธ) : ๊ทธ๋ž˜๊ฐ€์ง€๊ณ  NL์ชฝ์—์„œ ๋‚˜๋Š” ์ •ํ†ต ๋งˆ๋ฅดํฌ์Šค์ฃผ์˜ --- ์ด๋Ÿฐ ์ƒ๊ฐ์„ ๊ฐ–๊ฒŒ ๋œ ๊ฑฐ์•ผ. ๊ทธ ๋‹ค์Œ์— ์ด์ œ NLPDR์ด์•ผ. ์ด๊ฒŒ ๋งž๋Š” ๊ฑฐ์•ผ, ๊ทธ ๋‹ค์Œ์— ์ฃผ์‚ฌ๋Š” ๋ˆ„๊ฐ€ ๊ฐ€๋ฅด์ณ ์ค€ ๊ฒƒ๋„ ์—†์–ด... ๋ˆ„๊ฐ€ ํŠน๋ณ„ํžˆ ๋ญ ๊ต์œก์„ ํ•ด์คฌ๋‹ค ์ด๋Ÿฐ ๊ฑฐ๋Š” ์—†์–ด, ๋ถ€์ง„ํ•œ ๊ทธ ์‚ฌ๊ฑด ๋•Œ ํ•œ 3~4๊ฐœ์›” ์ •๋„ ์กฐ์ง์ƒํ™œ์„ ํ•ด๋ดค์ง€... ๊ทธ๋Ÿฌ๋‹ˆ๊นŒ ๋ˆ„๊ฐ€ NLPDR ์ดํ›„์—๋Š” ๋‚˜๋„ ์ด๋ก  ์„ ์ƒ์ด ์—†์—ˆ์–ด. ์ง€๊ธˆ๋„ ๊ทธ๋ž˜, ์ง€๊ธˆ๋„ ์ด๋ก  ์„ ์ƒ์€ ๊ทธ๋Ÿฌ๋‹ˆ๊นŒ ๋›ฐ์–ด๋‚œ ์‚ฌ๋žŒ๋“ค์€ ๋งŽ์ด ์žˆ๋Š”๋ฐ ๋‚ด๊ฐ€ ์•„ ์ด๋Ÿฐ ์„ ์ƒํ•œํ…Œ ๋ฐฐ์šด ์‚ฌ๋žŒ์€ ์—†๋Š” ๊ฑฐ์•ผใ€ ์ด์™€ ๊ฐ™์ด ํ”ผ๊ณ ์ธ์€ ๊ณต์†Œ์™ธ 4๋ฅผ ๋งŒ๋‚˜ โ€˜RO์‚ฌ๊ฑดโ€™์„ ๊ณ„๊ธฐ๋กœ ๋ณ€์ ˆ์ž ์ƒ‰์ถœ ๋“ฑ ์กฐ์ง ๋ณด์œ„์˜ ํ•„์š”์„ฑ ๋“ฑ์„ ์–ธ๊ธ‰ํ•˜๊ณ , ๊ณต์†Œ์™ธ 4์™€ ํ•จ๊ป˜ โ€˜๋ณ€ํ˜์šด๋™๊ณผ ํ†ต์ผ์šด๋™โ€™, โ€˜๋ฏผ์กฑํ•ด๋ฐฉ๋ฏผ์ฃผ์ฃผ์˜ํ˜๋ช…๋ก โ€™ ๋“ฑ์— ๊ด€ํ•˜์—ฌ ํ† ๋ก ํ•˜๋ฉด์„œ, โ€œ๋ณ€ํ˜์šด๋™์˜ ์›์ธ์€ ๊ณ„๊ธ‰ํˆฌ์Ÿ์— ์žˆ๊ณ , ํ†ต์ผ์šด๋™์˜ ์›์ธ์€ ๋ฏผ์กฑํ•ด๋ฐฉ์— ์žˆ๋‹ค. ๋ณ€ํ˜์šด๋™์˜ ๋ชฉํ‘œ๋Š” ์ •๊ถŒ์Ÿ์ทจ์ด๊ณ , ํ†ต์ผ์šด๋™์˜ ๋ชฉํ‘œ๋Š” ๋ฏผ์กฑํ†ต์ผ์ •๊ถŒ ์ˆ˜๋ฆฝ์ด๋‹ค. ๋ฏผ์กฑํ†ต์ผ์ •๊ถŒ๊ณผ ๋ณ€ํ˜์šด๋™์ด ๋งŒ๋“  ์ •๊ถŒ์ด ์—ฐ๋ฐฉ์ œ ํ•˜์—์„œ ๋‚จ์ชฝ์—๋Š” ์ž์ฃผ์ ๋ฏผ์ฃผ์ •๋ถ€ ์ˆ˜๋ฆฝ์ด๋‹คโ€๋ผ๋Š” ์ทจ์ง€๋กœ ์‚ฌ์ƒํ•™์Šต์„ ์ง„ํ–‰ํ•˜์˜€๋‹ค. ์ด๋กœ์จ ํ”ผ๊ณ ์ธ์€ ๊ณต์†Œ์™ธ 4์™€ ๊ณต๋ชจํ•˜์—ฌ, ๊ตญ๊ฐ€์˜ ์กด๋ฆฝยท์•ˆ์ „์ด๋‚˜ ์ž์œ ๋ฏผ์ฃผ์  ๊ธฐ๋ณธ์งˆ์„œ๋ฅผ ์œ„ํƒœ๋กญ๊ฒŒ ํ•œ๋‹ค๋Š” ์ •์„ ์•Œ๋ฉด์„œ ๋ฐ˜๊ตญ๊ฐ€๋‹จ์ฒด๋‚˜ ๊ทธ ๊ตฌ์„ฑ์› ๋˜๋Š” ๊ทธ ์ง€๋ น์„ ๋ฐ›์€ ์ž์˜ ํ™œ๋™์„ ์ฐฌ์–‘ยท๊ณ ๋ฌดยท์„ ์ „ ๋˜๋Š” ์ด์— ๋™์กฐํ•˜์˜€๋‹ค. ๋‚˜. 2013. 12. 14. ๊ณต์†Œ์™ธ 5์™€ ์‚ฌ์ƒํ•™์Šต(์ด์ ๋™์กฐ, ์ด์ ํ‘œํ˜„๋ฌผ ์†Œ์ง€) ํ”ผ๊ณ ์ธ์€ 2013. 12. 14. 16:25๊ฒฝ๋ถ€ํ„ฐ ๊ฐ™์€ ๋‚  18:15๊ฒฝ๊นŒ์ง€ ์ˆ˜์›์‹œ์— ์žˆ๋Š” ์„ธ๋ฅ˜์—ญ ์ธ๊ทผ (๋ช…์นญ 3 ์ƒ๋žต) ์ปคํ”ผ์ˆ์—์„œ ๊ณต์†Œ์™ธ 5๋ฅผ ๋งŒ๋‚˜ ํ”ผ๊ณ ์ธ์˜ ํƒœ๋ธ”๋ฆฟ PC์— ์ €์žฅ๋˜์–ด ์žˆ๋˜ โ€˜์กฐ์„ ๋…ธ๋™๋‹น ์ค‘์•™์œ„ ์ •์น˜๊ตญ ํ™•๋Œ€ํšŒ์˜์— ๊ด€ํ•œ ๋ณด๋„โ€™๋ผ๋Š” ์ œ๋ชฉ์˜ 2013. 12. 9.์ž ์กฐ์„ ์ค‘์•™ํ†ต์‹  ๊ธฐ์‚ฌ, โ€˜๊ฒฝ์• ํ•˜๋Š” ๊น€์ •์€ ๋™์ง€์˜ ๋‘๋ฆฌ์— ์ฒ ํ†ต๊ฐ™์ด ๋ญ‰์ณ ์ฃผ์ฒดํ˜๋ช…์˜ ํ•œ๊ธธ๋กœ ์–ต์„ธ๊ฒŒ ๋‚˜์•„๊ฐ€์žโ€™๋ผ๋Š” ์ œ๋ชฉ์˜ 2013. 12. 10.์ž ๋…ธ๋™์‹ ๋ฌธ ๊ธฐ์‚ฌ, โ€˜์ฒœ๋งŒ๊ตฐ๋ฏผ์˜ ์น˜์†Ÿ๋Š” ๋ถ„๋…ธ์˜ ํญ๋ฐœ. ๋งŒ๊ณ ์—ญ์  ๋‹จํ˜ธํžˆ ์ฒ˜๋‹จโ€™์ด๋ผ๋Š” ์ œ๋ชฉ์˜ 2013. 12. 13.์ž ๋…ธ๋™์‹ ๋ฌธ ๊ธฐ์‚ฌ, โ€˜์กฐ์„ ๋กœ๋™๋‹น์€ ์ผ์‹ฌ๋‹จ๊ฒฐ์˜ ์œ„๋ ฅ์œผ๋กœ ์ „์ง„ํ•˜๋Š” ๋ถˆํŒจ์˜ ํ˜๋ช…์ ๋‹น์ด๋‹คโ€™๋ผ๋Š” ์ œ๋ชฉ์˜ 2013. 12. 13.์ž ๋…ธ๋™์‹ ๋ฌธ ๊ธฐ์‚ฌ๋ฅผ ๊ณต์†Œ์™ธ 5์—๊ฒŒ ์—ด๋žŒ์‹œ์ผฐ๋Š”๋ฐ, ๊ทธ ์ฃผ์š”๋‚ด์šฉ์€ ๋‹ค์Œ๊ณผ ๊ฐ™๋‹ค. ใ€Žใ€Šโ€˜์กฐ์„ ๋…ธ๋™๋‹น ์ค‘์•™์œ„ ์ •์น˜๊ตญ ํ™•๋Œ€ํšŒ์˜์— ๋Œ€ํ•œ ๋ณด๋„โ€™์ œํ•˜์˜ ๊ธฐ์‚ฌ ์ฃผ์š”๋‚ด์šฉใ€‹ - ๊ณต์†Œ์™ธ 45์™€ ๊ทธ ์ถ”์ข…์ž๋“ค์€ ์šฐ๋ฆฌ๋‹น์˜ ์กฐ์ง์  ์˜์‚ฌ์ธ ๋‹น์˜ ๋กœ์„ ๊ณผ ์ •์ฑ…์„ ์ง„์‹ฌ์œผ๋กœ ๋ฐ›์•„๋“ค์ด์ง€ ์•Š๊ณ  ๊ทธ ์ง‘ํ–‰์„ ์˜์‹์ ์œผ๋กœ ํƒœ๊ณตํ•˜๊ณ  ์™ธ๊ณก์ง‘ํ–‰ํ•˜์˜€์œผ๋ฉฐ ๋‹น์˜ ๋ฐฉ์นจ์„ ๊ณต๊ณต์—ฐํžˆ ๋’ค์ง‘์–ด์—Ž๋˜ ๋‚˜๋จธ์ง€ ์กฐ์„ ์ธ๋ฏผ๊ตฐ ์ตœ๊ณ ์‚ฌ๋ น๊ด€ ๋ช…๋ น์— ๋ถˆ๋ณตํ•˜๋Š” ๋ฐ˜ํ˜๋ช…์ ์ธ ํ–‰์œ„๋ฅผ ์„œ์Šด์—†์ด ๊ฐํ–‰ํ•˜์˜€๋‹ค. - ํ† ๋ก ์ž๋“ค์€ ํ•œ๊ฒฐ๊ฐ™์ด ๊ณต์†Œ์™ธ 45์ผ๋‹น์ด ๊ฐํ–‰ํ•œ ๋ฐ˜๋‹น๋ฐ˜ํ˜๋ช…์ ์ข…ํŒŒํ–‰์œ„๋ฅผ ๊ฐ•ํ•˜๊ฒŒ ๋น„ํŒํ•˜์˜€์œผ๋ฉฐ ๊ฒฝ์• ํ•˜๋Š” ๊น€์ •์€ ๋™์ง€์˜ ์‚ฌ์ƒ๊ณผ ๋ น๋„๋ฅผ ์ถฉ์งํ•˜๊ฒŒ ๋ฐ›๋“ค๋ฉฐ ๋‹น์ค‘์•™์„ ์ •์น˜์‚ฌ์ƒ์ ์œผ๋กœ ๋ชฉ์ˆจ์œผ๋กœ ๊ฒฐ์‚ฌ ์˜น์œ„ํ•ด๋‚˜๊ฐˆ ๊ตณ์€ ๊ฒฐ์˜๋ฅผ ํ‘œ๋ช…ํ•˜์˜€๋‹ค. ใ€Šโ€˜๊ฒฝ์• ํ•˜๋Š” ๊น€์ •์€ ๋™์ง€์˜ ๋‘๋ฆฌ์— ์ฒ ํ†ต๊ฐ™์ด ๋ญ‰์ณ ์ฃผ์ฒดํ˜๋ช…์˜ ํ•œ๊ธธ๋กœ ์–ต์„ธ๊ฒŒ ๋‚˜์•„๊ฐ€์žโ€™ ์ œํ•˜์˜ ๊ธฐ์‚ฌ ์ฃผ์š”๋‚ด์šฉใ€‹ - ๊ฒฝ์• ํ•˜๋Š” ์›์ˆ˜๋‹˜์˜ ์ง€๋„๋ฐ‘์— ์ง„ํ–‰๋œ ์กฐ์„ ๋กœ๋™๋‹น ์ค‘์•™์œ„์›ํšŒ ์ •์น˜๊ตญ ํ™•๋Œ€ํšŒ์˜์˜ ๊ฒฐ์ •์„ ์ „ํญ์ ์œผ๋กœ ์ง€์ง€์ฐฌ๋™ํ•˜๋ฉฐ ์œ„๋Œ€ํ•œ ๊น€์ผ์„ฑ-๊น€์ •์ผ์ฃผ์˜๊ธฐ์น˜๋ฅผ ๋†’์ด ๋“ค๊ณ  ๋‹น์ค‘์•™์˜ ๋ น๋„๋”ฐ๋ผ ์ตœํ›„์˜ ์Šน๋ฆฌ๋ฅผ ํ–ฅํ•˜์—ฌ ํž˜์ฐจ๊ฒŒ ๋‚˜์•„๊ฐ€๋ ค๋Š” ์šฐ๋ฆฌ ๊ตฐ๋Œ€์™€ ์ธ๋ฏผ์˜ ์‹ ๋…๊ณผ ์˜์ง€๋Š” ํ™•๊ณ ๋ถ€๋™ํ•˜๋‹ค. - ์ด๋ฒˆ์— ๋‹น์ค‘์•™์œ„์›ํšŒ ์ •์น˜๊ตญ ํ™•๋Œ€ํšŒ์˜๋ฅผ ํ†ตํ•˜์—ฌ ์šฐ๋ฆฌ ๋‹น๋Œ€๋ ฌ์— ์šฐ์—ฐํžˆ ๋ผ์—ฌ๋“  ๋ถˆ์ˆœ๋ถ„์ž, ํ˜„๋Œ€ํŒ ์ข…ํŒŒ๋“ค์ด ์ ๋ฐœ์ˆ™์ฒญ๋จ์œผ๋กœ์จ ์šฐ๋ฆฌ๋‹น์ด ๊ฒฝ์• ํ•˜๋Š” ์›์ˆ˜๋‹˜์„ ์ค‘์‹ฌ์œผ๋กœ ํ•˜๋Š” ์‚ฌ์ƒ์  ์ˆœ๊ฒฐ์ฒด, ์กฐ์ง์  ์ „์ผ์ฒด๋กœ ๋”์šฑ ๊ฐ•ํ™”๋˜๊ฒŒ ๋˜์—ˆ๋‹ค. ใ€Šโ€˜์ฒœ๋งŒ๊ตฐ๋ฏผ์˜ ์น˜์†Ÿ๋Š” ๋ถ„๋…ธ์˜ ํญ๋ฐœ, ๋งŒ๊ณ ์—ญ์  ๋‹จํ˜ธํžˆ ์ฒ˜๋‹จโ€™์ œํ•˜์˜ ๊ธฐ์‚ฌ ์ฃผ์š”๋‚ด์šฉใ€‹ - ํ”ผ์†Œ์ž ๊ณต์†Œ์™ธ 45๋Š” ์šฐ๋ฆฌ๋‹น๊ณผ ๊ตญ๊ฐ€์˜ ์ง€๋„๋ถ€์™€ ์‚ฌํšŒ์ฃผ์˜์ œ๋„๋ฅผ ์ „๋ณตํ•  ๋ชฉ์  ๋ฐ‘์— ๋ฐ˜๋‹น๋ฐ˜ํ˜๋ช…์ข…ํŒŒํ–‰์œ„๋ฅผ ๊ฐํ–‰ํ•˜๊ณ  ์กฐ๊ตญ์„ ๋ฐ˜์—ญํ•œ ์ฒœํ•˜์˜ ๋งŒ๊ณ ์—ญ์ ์ด๋‹ค. - ๊ณต์†Œ์™ธ 45 ๋†ˆ์€ ์‹ฌ๋ฆฌ๊ณผ์ •์— ใ€ˆ๋‚˜๋Š” ๊ตฐ๋Œ€์™€ ์ธ๋ฏผ์ด ํ˜„์žฌ ๋‚˜๋ผ์˜ ๊ฒฝ์ œ์‹คํƒœ์™€ ์ธ๋ฏผ์ƒํ™œ์ด ํŒŒ๊ตญ์ ์œผ๋กœ ๋ฒˆ์ ธ์ง€๋Š”๋ฐ๋„ ๋ถˆ๊ตฌํ•˜๊ณ  ํ˜„์ •๊ถŒ์ด ์•„๋ฌด๋Ÿฐ ๋Œ€์ฑ…๋„ ์„ธ์šฐ์ง€ ๋ชปํ•œ๋‹ค๋Š” ๋ถˆ๋งŒ์„ ํ’ˆ๊ฒŒ ํ•˜๋ ค๊ณ  ์‹œ๋„ํ•˜์˜€๋‹คใ€‰๊ณ  ํ•˜๋ฉด์„œ ์ •๋ณ€์˜ ๋Œ€์ƒ์ด ๋ฐ”๋กœ ใ€ˆ์ตœ๊ณ  ๋ น๋„์ž๋™์ง€์ด๋‹คใ€‰๊ณ  ๋งŒ๊ณ ์—ญ์ ์˜ ์ถ”์•…ํ•œ ๋ณธ์‹ฌ์„ ๊ทธ๋Œ€๋กœ ๋“œ๋Ÿฌ๋‚ด๋†“์•˜๋‹ค. ใ€Šโ€˜์กฐ์„ ๋กœ๋™๋‹น์€ ์ผ์‹ฌ๋‹จ๊ฒฐ์˜ ์œ„๋ ฅ์œผ๋กœ ์ „์ง„ํ•˜๋Š” ๋ถˆํŒจ์˜ ํ˜๋ช…์  ๋‹น์ด๋‹คโ€™์ œํ•˜์˜ ๊ธฐ์‚ฌ ์ฃผ์š”๋‚ด์šฉใ€‹ - ์กฐ์„ ๋กœ๋™๋‹น์˜ ๋ ฅ์‚ฌ๋Š” ํ•˜๋‚˜์˜ ์‚ฌ์ƒ, ํ•˜๋‚˜์˜ ์ค‘์‹ฌ์— ๊ธฐ์ดˆํ•œ ํ˜๋ช…์  ๋‹จ๊ฒฐ, ์ผ์‹ฌ๋‹จ๊ฒฐ์˜ ์œ„๋ ฅ์œผ๋กœ ๋ฐฑ์ „๋ฐฑ์Šน์„ ํŽผ์ณ์˜จ ์„ฑ์Šค๋Ÿฌ์šด ๋ ฅ์‚ฌ์ด๋‹ค. ์œ„๋Œ€ํ•œ ์ˆ˜๋ น, ์œ„๋Œ€ํ•œ ๋ น๋„์ž์˜ ์œ ์ผ์‚ฌ์ƒ์ฒด๊ณ„, ์œ ์ผ์  ๋ น๋„์ฒด๊ณ„์˜ ํ™•๊ณ ๋ถ€๋™์„ฑ, ๋ฐ”๋กœ ์—ฌ๊ธฐ์— ์šฐ๋ฆฌ ์ผ์‹œ๋‹จ๊ฒฐ์˜ ๊ณต๊ณ ์„ฑ์ด ์žˆ๊ณ  ์กฐ์„ ๋กœ๋™๋‹น์˜ ํ•„์Šน๋ถˆํŒจ์„ฑ์ด ์žˆ๋‹ค. - ๋กœ๋™๊ณ„๊ธ‰์˜ ๋‹น์ด ํ˜๋ช…ํˆฌ์Ÿ๊ณผ ๊ฑด์„ค์‚ฌ์—…์„ ์ด๋Œ์–ด๊ฐ€๋Š” ๊ณผ์ •์— ์ฒซ์งธ๋„ ๋‘˜์งธ๋„ ๋†“์น˜์ง€ ๋ง์•„์•ผ ํ•  ๊ฒƒ์ด ์žˆ๋‹ค. ๊ทธ๊ฒƒ์€ ํ˜๋ช…์˜ ์ค€์—„ํ•œ ์‹œ๊ธฐ์— ๋ น๋„์˜ ๊ณ„์Šน์˜ ์‹œ๊ธฐ์— ๋จธ๋ฆฌ๋ฅผ ์ณ๋“œ๋Š” ์ข…ํŒŒ๋ถ„์ž๋“ค์„ ์ ๋ฐœ์ˆ™์ฒญํ•˜๋Š” ๊ฒƒ์ด๋‹ค. ์ข…ํŒŒ๋ฅผ ์ œ๊ฑฐํ•˜์—ฌ์•ผ ๋‹น์ด ๊ฐ•ํ™”๋˜๊ณ  ์ผ์‹ฌ๋‹จ๊ฒฐ์ด ๊ณต๊ณ ํ•ด์ง€๋ฉฐ ํ˜๋ช…์ด ์Šน๋ฆฌ์ ์œผ๋กœ ์ „์ง„ํ•˜๊ฒŒ ๋œ๋‹ค๋Š” ๊ฒƒ์€ ๋ ฅ์‚ฌ์˜ ์ฒ ๋ฆฌ์ด๋‹ค.ใ€ ์ด์™€ ๊ฐ™์ด ์œ„ ๊ธฐ์‚ฌ๋“ค์€ ๊น€์ •์€์ด ๊ณต์†Œ์™ธ 45๋ฅผ ๋ฐ˜๋‹น๋ฐ˜ํ˜๋ช… ์ข…ํŒŒํ–‰์œ„์ž๋กœ ์ˆ™์ฒญํ•œ ๊ฒƒ์„ ์ •๋‹นํ™”ํ•˜๋ฉด์„œ, ๊น€์ผ์„ฑ-๊น€์ •์ผ์ฃผ์˜ ๊ฐ•ํ™” ๋ฐ ๊น€์ •์€์„ ์ค‘์‹ฌ์œผ๋กœ ํ•œ ์œ ์ผ์  ์˜๋„์ฒด๊ณ„ ํ™•๋ฆฝ์„ ์ฃผ์žฅํ•˜๋Š” ๋‚ด์šฉ๋“ค์ด๋‹ค. ํ”ผ๊ณ ์ธ์€ ๊ณต์†Œ์™ธ 5์—๊ฒŒ ์œ„ ๊ธฐ์‚ฌ๋“ค์„ ์—ด๋žŒํ•˜๊ฒŒ ํ•œ ํ›„, ๊ณต์†Œ์™ธ 5์™€ ํ•จ๊ป˜ ์œ„ ๊ธฐ์‚ฌ์— ๊ด€ํ•ด ํ† ๋ก ํ•˜์˜€๋Š”๋ฐ, ๊ทธ ์ฃผ์š” ๋Œ€ํ™”๋‚ด์šฉ์€ ๋‹ค์Œ๊ณผ ๊ฐ™๋‹ค. ใ€Žใ€Š2013. 12. 14. ํ”ผ๊ณ ์ธ๊ณผ ๊ณต์†Œ์™ธ 5 ๊ฐ„์˜ ์ฃผ์š” ๋Œ€ํ™”๋‚ด์šฉใ€‹ (ํ”ผ๊ณ ์ธ) : ์œ ์‹ฌ ๋นผ๊ณ  ์™”์–ด (๊ณต์†Œ์™ธ 5) : ์œ ์‹ฌ์ด ๋ญ์—์š”? (ํ”ผ๊ณ ์ธ) : ์ฝ์–ด๋ณด์‹œ๊ณ  (ํ”ผ๊ณ ์ธ) : ์–ด๋– ์…จ์–ด์š”? ๊ณต์†Œ์™ธ 45 ์‚ฌ๊ฑด (๊ณต์†Œ์™ธ 5) : ์ข€ ์ถฉ๊ฒฉ์ ์ด์—ˆ์–ด์š”. (ํ”ผ๊ณ ์ธ) : ์›์ธ์ด ์–ด๋””์— ์žˆ๋Š” ๊ฑฐ ๊ฐ™์•„์š”? ์ด๋ฒˆ ์ผ์ด ๋ฐœ์ƒํ•œ (๊ณต์†Œ์™ธ 5) : ๋‚ด๋ถ€์ , ์™ธ๋ถ€์  ์š”์ธ์ด ๋‹ค ์ž‘์šฉํ•œ ๊ฑฐ ๊ฐ™์€๋ฐ์š”. ๋ฏธ๊ตญ์˜ ์ œ๊ตญ์ฃผ์˜ ์„ธ๋ ฅ์ด ์ •์น˜๊ตฐ์‚ฌ์  ๊ณต์„ธ๋งŒ ์•„๋‹ˆ๋ผ ๋‚ด๋ถ€์ ์ธ ๊ทธ๋Ÿฐ ๋งˆ์ˆ˜๊ฐ€ ๋ฏธ์น˜๋Š” ์˜ํ–ฅ์ด ์—ฐ๊ณ„๋ถ€๋ถ„์ด ์žˆ๋Š” ๊ฒƒ ๊ฐ™๊ธฐ๋„ ํ•˜๊ณ , ๋˜ ๊ถŒ๋ ฅ ๋ฐ•ํƒˆํ•˜๋ ค๋Š” ์„ธ๋ ฅ๋“ค์ด ์กด์žฌํ•˜๋Š” ๊ฒƒ๋„ ์žˆ๊ณ . ๊ทธ๋ž˜์„œ ์ „ํ™˜์˜ ์‹œ๊ธฐ์—๋Š” ์ข€ ์กฐ์‹ฌํ•ด์•ผ ๋  ๊ฑฐ ๊ฐ™๋‹ค. ๊ถŒ๋ ฅ ์ „ํ™˜์˜ ์‹œ๊ธฐ์—๋Š” ๊ทธ๋Ÿฐ ์ผ์ด ํ„ฐ์ง€๋Š” ๊ฒƒ ๊ฐ™๋‹ค. ๊ทธ๋Ÿฐ ์ƒ๊ฐ์ด ๋“ญ๋‹ˆ๋‹ค. (ํ”ผ๊ณ ์ธ) : ์˜๋„์˜ ๊ณ„์Šน์˜ ์‹œ๊ธฐ์— ์ง€๊ธˆ ๊ฐ€์žฅ ํ•ต์‹ฌ์ ์ธ ๋ฌธ์ œ๋Š” ์œ ์ผ์˜๋„์ฒด๊ณ„๋‹ค. ์œ ์ผ์˜๋„์ฒด๊ณ„๋ฅผ ๊ตฌ์ถ•์„ ํ•ด์•ผ ๋˜๋Š”๋ฐ, ๊ทธ๊ฑฐ๋ฅผ ๊ฐ€์žฅ ๊ฑฐ๋ถ€ํ•˜๊ณ  ๋‚˜์„  ๊ฒƒ์ด ๊ณต์†Œ์™ธ 45 ์ผ๊ฐ€์˜€๋‹ค. ์œ ์ผ์˜๋„์ฒด๊ณ„๊ฐ€ ๊ฐ–์ถฐ์ง€์ง€ ๋ชปํ•˜๋„๋ก, ์ž๊ธฐ์˜ ๊ตญ๊ฐ€๊ถŒ๋ ฅ์„ (ยท) ์™ธ๋ถ€์ ์ธ ์ „๋žต์ ์ธ (ยท) ๋‚จ์ชฝ์˜ ๊ธฐ๋‹ค๋ฆฌ๋Š” ์ •์ฑ…์ด๋ผ๋˜๊ฐ€ ์ด๋Ÿฐ ๊ฒƒ์— ํŽธ์Šน์„ ํ•ด์„œ ๋‚ด๋ถ€์ ์œผ๋กœ ์†Œ์š”๋ฅผ ์ผ์œผํ‚ค๊ฑฐ๋‚˜ ์ด๋ ‡๊ฒŒ ํ•˜๋ฉด ์ •๊ถŒ์„ ์žก์„ ๊ฒƒ์ด๋ผ๊ณ  ์ƒ๊ฐ์„ ํ•˜๊ณ  ๊ทธ๋Ÿฐ ํ–‰์œ„๋ฅผ ํ•˜์˜€๋‹จ ๋ง์ด์ฃ . ๊ทธ๊ฒƒ์ด ์ด๋ฒˆ์— ๋“œ๋Ÿฌ๋‚œ ๊ฒƒ์ด๊ณ , ๊ฒฐ๊ตญ ์œ ์ผ์˜๋„์ฒด๊ณ„๋ฅผ ์„ธ์šฐ๋Š” ์ด๋Ÿฐ ๊ณผ์ •์—์„œ ์ ๋ฐœ๋œ ๊ฑฐ์ฃ . (๊ณต์†Œ์™ธ 5) : ๋‹น์—์„œ์˜ ์ผ์‹ฌ๋‹จ๊ฒฐ ๊ทธ๊ฑฐ๋ฅผ ๋นผ๋ฒ„๋ฆฌ๋ฉด ๋‹น์ž์ฒด๊ฐ€ ๋…ธ๋™ ๋Œ€์˜ค์˜ ๋‹น์œผ๋กœ ๋ณผ ์ˆ˜ ์—†๊ณ , ๊ทธ๋ƒฅ ๋ฌด์žฅํ•ด์ œ๋ผ๋Š” ๊ฑฐ (๊ณต์†Œ์™ธ 5) : ์—ฌ๊ธฐ ๋‚จํ•œ์˜ ๋ฐ•๊ทผํ˜œ ์ •๊ถŒ์ด ํƒ€๋„๋œ๋‹ค๊ณ  ํ•ด์„œ ๊ทธ๊ฒƒ ์ž์ฒด๊ฐ€ ํ”๋“ค๋ฆฌ๋Š” ๊ฒƒ ์•„๋‹ˆ์ž–์•„์š”? ์ฒด์ œ์  ๋ฌธ์ œ๊ธฐ ๋•Œ๋ฌธ์— ์—„์ฒญ๋‚œ ์ผ์ด ๋ฒŒ์–ด์ง€๋Š” ๊ฑฐ์ฃ , ๋ฌด์‹œ๋ฌด์‹œํ•œ ์ผ์ด ๋ฒŒ์–ด์ง„ ๊ฑฐ์ฃ . (ํ”ผ๊ณ ์ธ) : ์šฐ๋ฆฌ๊ฐ€ ๊ฐ–๊ณ  ์žˆ๋Š” ์šฐ๋ฆฌ ์‚ฌ์ƒ์—์„œ ๊ฐ€์žฅ ํฐ ๋‹จ์–ด๊ฐ€ ๋ญ๊ฒ ์Šต๋‹ˆ๊นŒ? ์œ ์ผ์˜๋„๋ฅผ ๊ฑฐ๋ถ€ํ•˜๊ณ  ๋‚˜์„œ๋ฉด์„œ, ์œ ์ผ์˜๋„๋ฅผ ๊ฑฐ๋ถ€ํ•˜๋ฉด์„œ ๋’ค์—Ž๊ณ  ์ •๊ถŒ์„ ์ฐจ์ง€ํ•˜๊ฒ ๋‹ค ํ•˜๋Š” ๊ทธ๋Ÿฐ ๊ฒƒ์ด์ฃ . (๊ณต์†Œ์™ธ 5) : ์ผ๋ฐ˜ ๋ฒ”์ฃ„ํ–‰์œ„ํ•˜๊ณ ๋Š” ์ฐจ์›์ด ํ‹€๋ ค์š”. ์ ๋“ค์—๊ฒŒ ์กฐ๊ตญ๊ณผ ๋ฏผ์กฑ์„ ๋‹ค ๋ฐ”์น˜๋ ค๋Š” ํ–‰์œ„ (ํ”ผ๊ณ ์ธ) : ๋งž์Šต๋‹ˆ๋‹ค. (ยท) ์œ ์ผ์˜๋„์ฒด๊ณ„๋ฅผ ๊ตณ๊ฒŒ ์„ธ์›Œ์„œ ๋ชจ๋“  ์ผ์— ์žˆ์–ด์„œ ์ง„ํ–‰์„ ํ•ด๋‚˜๊ฐ€์•ผ ๋˜๊ฒ ๊ณ , ์ด๋‚จ์— ์žˆ๋Š” ์šฐ๋ฆฌ๋„, ๊ทธ๋ž˜์„œ ์ด๋ฒˆ ์‚ฌ๊ฑด์— ์ข…ํŒŒ์‚ฌ๊ฑด์— (ยท) ์ •์น˜ ํ™•๋Œ€ํšŒ์˜ ๊ฒฐ์˜์„œ, ํŠน๋ณ„์  ํŒ๊ฒฐ๋ฌธ, ๋…ธ๋™์‹ ๋ฌธ ์‚ฌ์„ค์— ๋‚˜ํƒ€๋‚œ ์—ฌ๋Ÿฌ ๊ฐ€์ง€ ์ด๋Ÿฐ ๋ถ€๋ถ„๋“ค (ยท) ๋ฐฉ๊ธˆ ์ค‘์•™์œ„์›ํšŒ ์ข…ํŒŒ๋ฌธ์ œ๋กœ (ยท) ๋ชจ์—ฌ์„œ ํŠน๋ณ„๊ฒฐ์˜๋ฅผ ํ–ˆ์Šต๋‹ˆ๋‹ค. ํŠน๋ณ„๊ฒฐ์˜๋ฌธ์˜ ํ˜•ํƒœ๋กœ ์ „ํŒŒํ•ด ๋Œ๋ฆฌ๊ธฐ๋กœ, 3๊ฐ€์ง€ ๋‚ด์šฉ์œผ๋กœ ๊ฒฐ์˜๋ฅผ ํ–ˆ๋Š”๋ฐ (ยท) ์ค‘์•™์œ„์ •์น˜๊ตญ ํ™•๋Œ€ํšŒ์˜์˜ ๊ฒฐ๊ณผ(ยท) ๋ฌด์กฐ๊ฑด ์ ‘์ˆ˜ํ•˜๊ณ  ์ „ํญ ์ง€์ง€ํ•œ๋‹ค. ์ „์ ์œผ๋กœ ์ง€์ง€ํ•œ๋‹ค๋ผ๊ณ  ํ•˜๋Š” ๊ฒƒ. ๋‘ ๋ฒˆ์งธ๊ฐ€ ์œ ์ผ์˜๋„์ฒด๊ณ„๋ฅผ ๋ˆˆ๋™์ž์™€ ๊ฐ™์ด ์ง€ํ‚ค๊ณ  ์šฐ๋ฆฌ ์กฐ์ง์—์„œ ๊ตฌํ˜„ํ•˜๊ณ  ์‹คํ˜„ํ•˜๊ธฐ ์œ„ํ•ด์„œ ๋…ธ๋ ฅํ•œ๋‹ค. ๊ฐ„๋‹จํ•˜๊ฒŒ๋Š” ํˆฌ์Ÿํ•œ๋‹ค๋ผ๊ณ  ํ–ˆ์ฃ . ์—ฌํŠผ ๋ฐฉ๋ฒ•์ ์œผ๋กœ ์‚ฌ์ƒ ๊ต์–‘, ์œ ์ผ ์‚ฌ์ƒ์„ ์กฐ์ง์— ๋Œ€ํ•œ ์‚ฌ์ƒ์„ ๋” ๊ฐ•ํ™”ํ•˜์ž๋ผ๊ณ  ํ•˜๋Š”. ๋˜, ์œ ์ผ ์˜๋„๊ฐ€ ๊ด€์ฒ ๋˜๋Š” ์กฐ์ง์ƒํ™œ์„ ๋”์šฑ๋” ๊ฐ•ํ™”ํ•œ๋‹ค. ์œ ์ผ์‚ฌ์ƒ์ฒด๊ณ„์˜ ์œ ์ผ์˜๋„์ฒด๊ณ„๋ฅผ ์‚ด์•„์žˆ๊ณ  ์ˆจ ์‰ฌ๊ธฐ ์œ„ํ•ด์„œ ํˆฌ์Ÿํ•œ๋‹ค๋ผ๊ณ  ํ•˜๋Š” ๊ฒฐ์˜๊ณ , ์„ธ ๋ฒˆ์งธ๋กœ ์ด ์œ ์ผ์˜๋„, ์›์ˆ˜๋‹˜์˜ ์œ ์ผ์˜๋„์™€ ์ด๋‚จ์˜ ์‚ฌํšŒ๋ณ€ํ˜์šด๋™๊ณผ ์กฐ๊ตญํ†ต์ผ์šด๋™์ด ๊ตฌํ˜„๋  ์ˆ˜ ์žˆ๋„๋ก ์ˆ˜ํ–‰ํ•œ๋‹ค. ์ด ์„ธ ๊ฐ€์ง€ ๋‚ด์šฉ์„, ํŠน๋ณ„๊ฒฐ์˜๋ฅผ ์ „์ฒด ์„ฑ์›๋“ค์—๊ฒŒ ํ†ต๋ณดํ•  ์ˆ˜ ์žˆ๋„๋ก (ยท) ๋™์ง€์—๊ฒŒ ํ†ต๋ณดํ•  ์ˆ˜ ์žˆ๋„๋ก ํ•ด ์ฃผ์‹ญ์‹œ์˜ค. (ํ”ผ๊ณ ์ธ) : ์•„์‹œ๊ฒ ์ฃ ? ์œ ์ผ์˜๋„์™€ ์šฐ๋ฆฌ ๋ณ€ํ˜์šด๋™๊ณผ ์กฐ๊ตญํ†ต์ผ์šด๋™์ด ์‹คํ˜„, ๊ตฌํ˜„, ์‹คํ˜„๋  ์ˆ˜ ์žˆ๋„๋ก ํˆฌ์Ÿํ•œ๋‹ค. ์ด ์„ธ ๊ฐ€์ง€, ๊ทธ๋Ÿฌ๊ธฐ ์œ„ํ•œ ๋ฐฉ๋„๊ฐ€ ์œ ์ผ์‚ฌ์ƒ์ฒด๊ณ„์™€ ์‚ฌ์ƒํ•™์Šต๊ณผ ์กฐ์ง์ƒํ™œ ์ด ์‹ค์ฒœ ์„ธ ๊ฐ€์ง€์ฃ . ๊ทธ๊ฑฐ๋Š” ์ด์ œ๋Š” ๊ฒฐ์˜ํ•œ๋‹ค๊ธฐ๋ณด๋‹ค ๊ทธ ๋‚ด์šฉ์— ํฌํ•จ๋˜์–ด ์žˆ์ฃ  ๊ทธ๋Ÿฌ๊ธฐ ์œ„ํ•œ ๋ฐฉ๋„๋Š” ์‚ฌ์ƒํ•™์Šต๊ณผ ์กฐ์ง์ƒํ™œ๊ณผ ์‹ค์ฒœํˆฌ์Ÿ. ์ง€๊ธˆ ์ด์ œ ์กฐ์ง์ ์œผ๋กœ ์ „ํŒŒํ•˜์‹ญ์‹œ์˜ค. (๊ณต์†Œ์™ธ 5) : ๋„ค ๋„ค. (ํ”ผ๊ณ ์ธ) : ๊ทธ๋Ÿฐ ๊ฒฐ์˜๋ฌธ์„ ๋ณธ์‚ฌ์— ์ด์•ผ๊ธฐ๋ฅผ ํ–ˆ๋‹ค. ์—ฌ๊ธฐ์— ๋Œ€ํ•ด์„œ ๋‹ค๋ฅธ ์˜๊ฒฌ, ๋ฐ˜๋Œ€ํ•˜๋Š” ์˜๊ฒฌ์ด๋ผ๋“ ์ง€ ๊ทธ๊ฒƒ์ด ์žˆ์œผ๋ฉด ํ—ˆ์‹ฌํƒ„ํšŒํ•˜๊ฒŒ ์ด์•ผ๊ธฐ ํ•˜๋„๋ก ํ•˜๊ณ  ๊ทธ๋ ‡์ง€ ์•Š์œผ๋ฉด ์ ‘์ˆ˜ํ•˜๊ณ  (---) ์ผ์‹ฌ๋‹จ๊ฒฐํ•ด์„œ ํˆฌ์Ÿํ•˜๋‚˜๊ฐ€ (---) ๋งจ ๋’ค์— ๋ณด๋ฉด ์‚ฌ์„ค์—์„œ ์ž˜ ํ•ด์„คํ•ด ์ฃผ๊ณ  ์žˆ๋Š”๋ฐ, ๋งจ ๋’ค์— ์ด์ œ ๋ณ„ํ‘œ (ยท), ์ „ํ™˜ ์ ๊ธฐ, ์กฐ์ง๋‹น์˜ ๊ฐ•ํ™” ๋ฐœ์ „, ์กฐ์ง๊ณ„ํš ๊ฐ•ํ™” ๋ฐœ์ „, (---) ์ฒซ ๋ฒˆ์งธ๊ฐ€ (---) ๊ฐ„๋ถ€๋“ค์ด (---) ์‚ฌํšŒ์ฃผ์˜ ๊ตญ๊ฐ€ ๊ฑด์„ค์—์„œ ๋Š์ž„์—†๋Š” ๋น„์•ฝ๊ณผ ํ˜์‹ ์„ ์ง€์ผœ๋‚˜๊ฐ€์•ผ ํ•œ๋‹ค. ํ•œ๊ตญ ์‚ฌํšŒ๋ณ€ํ˜์šด๋™๊ณผ ์ด๋‚จ์˜ ์‚ฌํšŒ์ฃผ์˜ ์šด๋™๊ณผ ์กฐ๊ตญ ํ†ต์ผ์šด๋™์€ ๋Š์ž„์—†๋Š” ๋น„์•ฝ๊ณผ ํ˜์‹ ์„ ์ง€์ผœ๋‚˜๊ฐ€์•ผ ํ•œ๋‹ค. ์š”๋ ‡๊ฒŒ ์ฝ์„ ์ˆ˜ ์žˆ์ฃ . (---) ์‚ฌ์ƒ๊ต์–‘์„ (---) ์ด๊ฑด ์šฐ๋ฆฌ ์กฐ์ง์€ ์„ฑ์›๋“ค๊ณผ ๋Œ€์ค‘์— ๋Œ€ํ•ด์„œ (---) ๋ฌผ๋ก  ์‚ฌ์ƒ๊ต์–‘ ํ•  ์ˆ˜ ์—†์ง€๋งŒ, ์–ด์จŒ๋“  ์ด๋Ÿฐ ์ €๊ธฐ์— ๋Œ€ํ•ด์„œ ๊ฐ€๊นŒ์šด ์‚ฌ๋žŒ๋“ค ํ•ด์„ค์„ ํ•ด์ฃผ๊ณ , (---) ์ž๊ธฐ์˜ ์ƒํ™œ๊ณผ ์ž๊ธฐ์˜ ๊ฒฐ์ • ์ง€์‹œ, ์กฐ์ง์˜ ๊ฒฐ์ • ์ง€์‹œ๋ฅผ ์ฒ ์ €ํžˆ ์ง‘ํ–‰ํ•˜๋Š” (ยท) ๋†’์€ ์กฐ์ง๊ด€์„ (ยท) ๊ฐ„๋ถ€๋“ค์ด ๋†’์€ ์กฐ์ง๊ด€์„ ๊ฐ–๊ณ  ๊ฒฐ์ •, ์ง€์‹œ ์ฒ ์ €ํžˆ ์ง€์‹œํ•˜๊ณ  (---) ์•„์‹œ๊ฒ ์ฃ ? (๊ณต์†Œ์™ธ 5) : ๋„ค, ์‚ฌ์„ค์€ ์ž์ฃผ๋ฏผ๋ณด๋‚˜ ์–ด๋””์—์„œ ์ ‘ํ•  ์ˆ˜ ์žˆ๋‚˜์š”? (ํ”ผ๊ณ ์ธ) : ์‚ฌ์„ค์€ ์ ‘ํ•  ์ˆ˜ ์—†๋Š”๋ฐ, ์ œ๊ฐ€ ๊ฐ€๋ฅด์ณ ๋“œ๋ฆด๊ป˜์š”. (ยท) ์™ธ์šธ ์ˆ˜ ์žˆ๋‚˜? ์˜์–ด๋กœ ๋˜๊ฐ€์ง€๊ณ , ํ”„๋ก์‹œ๋ผ๊ณ , ํ”„๋ก์‹œ ์„œ๋ฒ„ ์•„์‹œ์ฃ ? โ–กโ–กโ–ก์— ๋“ค์–ด๊ฐ€์‹œ๋ฉด, (ํ”ผ๊ณ ์ธ) : ํƒœ๋ธ”๋ฆฟ์œผ๋กœ, ํƒœ๋ธ”๋ฆฟ ์“ฐ์„ธ์š”? ํƒœ๋ธ”๋ฆฟ์œผ๋กœ ๋ณด์…”๋„ ๋˜๊ณ , ์ €๋„ ๊ฐ€๋” ์ด๋ ‡๊ฒŒ ๋ณด๊ฑฐ๋“ ์š”. ๊ทธ๋ฆฌ๊ณ  ๋ณด๊ธฐ๋งŒ ํ•˜๊ณ  ์ €์žฅ์€ ํ•˜์ง€ ๋งˆ์„ธ์š”. ๋‚ด์šฉ ์ €์žฅ์€, ์ด๋ ‡๊ฒŒ ๋ณด๋ฉด ๋˜๋‹ˆ๊น, ์ €์žฅ์€ ์ด์ œ ์ œ๊ฐ€ ๊ต์–‘์ž๋ฃŒ๋กœ ๋งŒ๋“ค๊ธฐ ์œ„ํ•ด์„œ ์ €๋Š” ๋ถˆ๊ฐ€ํ”ผํ•˜๊ฒŒ ์ €์žฅ์„ ํ•˜๋Š”๋ฐ ์ €์žฅ๋„ usb์—๋‹ค ํ•˜๋‹ˆ๊นŒ. (ํ”ผ๊ณ ์ธ) : ์˜ค๋Š˜์€ ์ด๋Ÿฐ ๊ฑฐ ํ† ๋ก ํ•˜๋Š๋ผ๊ณ  ์กฐ๊ธˆ ์ง€์ฒดํ•˜๊ณ  ๊ทธ๋žฌ๋”๋‹ˆ๋งŒ ๊ธด์žฅ๋œ ์‹œ๊ฐ„์ด ํ˜๋Ÿฌ๊ฐ€๋‹ˆ๊น ๋งฅ์ด ๋งŽ์ด ๋น ์ง€๋„ค์š”. ์šฐ๋ฆฌ ์žฅ๊ตฐ๋‹˜๊ป˜์„œ ๋ง์”€ํ•˜์‹  ๊ฒƒ์ด (---) ๊น€์ •์€ ๋™์ง€๋ฅผ ์ค‘์‹ฌ์œผ๋กœ ๋‹จ๊ฒฐํ•˜๊ณ  ๋˜ ๋‹จ๊ฒฐํ•˜์—ฌ ๋ฐฑ๋‘์—์„œ ๊ฐœ์ฒ™ํ•œ ํ˜๋ช…์˜ ์ฒซ๊ฑธ์Œ์„ ๋๊นŒ์ง€ ์ด์–ด๋‚˜๊ฐ€์•ผ ํ•œ๋‹ค๋Š” ๋ง์”€์ด ์žˆ์ž–์•„์š”. ๊ตฌํ˜ธ๋กœ ์šฐ๋ฆฌ๋Š” ๊น€์ผ์„ฑ ๊น€์ •์ผ ๊น€์ •์€ ์™ธ์—๋Š” (ยท) ๊ทธ๋ฆฌ๊ณ  ๊น€์ผ์„ฑ ๊น€์ •์ผ ๊น€์ •์€ ์™ธ์—๋Š” ๊ทธ ๋ˆ„๊ตฌ๋„ ์•Œ์ง€ ๋ชปํ•œ๋‹ค. ์š”๊ฑฐ๋ฅผ (---) ๊น€์ผ์„ฑ ๊น€์ •์ผ ๊น€์ •์€ ์™ธ์—๋Š” ๊ทธ ๋ˆ„๊ตฌ๋„ ์•Œ์ง€ ๋ชปํ•œ๋‹ค. ์„ธ ๊ฐ€์ง€ ํŠน๋ณ„ ๊ฒฐ์˜, ๋งˆ์ง€๋ง‰์œผ๋กœ ํ•˜์…จ๋˜ ๋ง์”€์€ ๊น€์ •์€ ๋™์ง€์ฃผ์˜์— ๋‹จ๊ฒฐํ•˜๊ณ  ๋‹จ๊ฒฐํ•˜๊ณ  ๋˜ ๋‹จ๊ฒฐํ•˜๋ผ. ๋ฐฑ๋‘์—์„œ ๊ฐœ์ฒ™ํ•œ ์ฃผ์ฒดํ˜๋ช…์˜ ์ฒซ๊ฑธ์Œ์„ ์ด์–ด๋‚˜๊ฐ€์•ผ ๋  ๊ฒƒ์ด๋‹ค. ์„ธ ๊ฐ€์ง€ ํŠน๋ณ„ ๊ฒฐ์˜, (ยท) ์•ˆ ์จ๋„ ๊ผญ ๋จธ๋ฆฌ ์†์— ๊ผญ ๋ช…์‹ฌ์„ (ยท) ๊ณต์†Œ์™ธ 45๊ฐ€ ๊ทธ๋ ‡๊ฒŒ ๋œ ๊ฒƒ์€ ์ฐธ ์•ˆ๋œ ์ผ์ด์ง€๋งŒ, ๊ฒฐ์ฝ” ์šฉ์„œํ•  ์ˆ˜ ์—†๋Š” ํ–‰์œ„๋ฅผ ํ•ด์„œ ๊ทธ๊ฒƒ์— ๋Œ€ํ•œ ๋‹น์—ฐํ•œ ์ฒ˜์‚ฌ๋ฅผ ๋ฐ›์€ ๊ฑฐ์ฃ .ใ€ ์ด์™€ ๊ฐ™์ด ํ”ผ๊ณ ์ธ์€ ๊ณต์†Œ์™ธ 5๋ฅผ ๋งŒ๋‚˜ ํ•™์Šต ๋ชจ์ž„์— ํœด๋Œ€ํฐ ์œ ์‹ฌ ์นฉ์„ ์ง‘์— ๋‘๊ณ  ์˜ฌ ๊ฒƒ๊ณผ ํ”„๋ก์‹œ ์„œ๋ฒ„๋กœ ๋ถํ•œ์‚ฌ์ดํŠธ์— ์šฐํšŒ ์ ‘์†ํ•˜๋Š” ๋ฐฉ๋ฒ•์„ ๊ต์œก์‹œํ‚ค๊ณ , ๊ณต์†Œ์™ธ 5์—๊ฒŒ ์œ„ ๊ธฐ์‚ฌ๋“ค์„ ์—ด๋žŒํ•˜๊ฒŒ ํ•œ ํ›„, ๊ณต์†Œ์™ธ 45 ์ฒ˜ํ˜•์˜ ์›์ธ๊ณผ ๊น€์ •์€ ์œ ์ผ์˜๋„์ฒด๊ณ„ ๊ตฌ์ถ• ๋“ฑ์— ๊ด€ํ•˜์—ฌ ํ† ๋ก ํ•˜๋ฉด์„œ, ์ค‘์•™์œ„ ์ •์น˜๊ตญ ํ™•๋Œ€ํšŒ์˜์˜ ๊ฒฐ๊ณผ๋ฅผ ์ „ํญ ์ง€์ง€ํ•˜๊ณ , ๊น€์ •์€ ์œ ์ผ์˜๋„์ฒด๊ณ„, ์ด๋‚จ์˜ ์‚ฌํšŒ๋ณ€ํ˜์šด๋™๊ณผ ์กฐ๊ตญํ†ต์ผ์šด๋™์ด ๊ตฌํ˜„๋  ์ˆ˜ ์žˆ๋„๋ก ์ ๊ทน ์ˆ˜ํ–‰ํ•  ๊ฒƒ์„ ๋‹ค์งํ•˜๋Š” ๋“ฑ ์‚ฌ์ƒํ•™์Šต์„ ํ•˜์˜€๋‹ค. ์ด๋กœ์จ ํ”ผ๊ณ ์ธ์€ ๊ตญ๊ฐ€์˜ ์กด๋ฆฝยท์•ˆ์ „์ด๋‚˜ ์ž์œ ๋ฏผ์ฃผ์  ๊ธฐ๋ณธ์งˆ์„œ๋ฅผ ์œ„ํƒœ๋กญ๊ฒŒ ํ•œ๋‹ค๋Š” ์ •์„ ์•Œ๋ฉด์„œ, ๋ฐ˜๊ตญ๊ฐ€๋‹จ์ฒด๋‚˜ ๊ทธ ๊ตฌ์„ฑ์› ๋˜๋Š” ๊ทธ ์ง€๋ น์„ ๋ฐ›์€ ์ž์˜ ํ™œ๋™์„ ์ฐฌ์–‘ยท๊ณ ๋ฌดยท์„ ์ „ํ•  ๋ชฉ์ ์œผ๋กœ ์ด์ ํ‘œํ˜„๋ฌผ์„ ์†Œ์ง€ํ•˜๊ณ , ๊ณต์†Œ์™ธ 5์™€ ๊ณต๋ชจํ•˜์—ฌ ๋ฐ˜๊ตญ๊ฐ€๋‹จ์ฒด๋‚˜ ๊ทธ ๊ตฌ์„ฑ์› ๋˜๋Š” ๊ทธ ์ง€๋ น์„ ๋ฐ›์€ ์ž์˜ ํ™œ๋™์— ๋™์กฐํ•˜์˜€๋‹ค. ๋‹ค. 2014. 1. 4. ๊ณต์†Œ์™ธ 5์™€ ์‚ฌ์ƒํ•™์Šต(์ด์ ๋™์กฐ, ์ด์ ํ‘œํ˜„๋ฌผ ์†Œ์ง€) ํ”ผ๊ณ ์ธ์€ 2014. 1. 4. 16:10๊ฒฝ๋ถ€ํ„ฐ ๊ฐ™์€ ๋‚  17:40๊ฒฝ๊นŒ์ง€ ์ˆ˜์›์‹œ์— ์žˆ๋Š” ์„ธ๋ฅ˜์—ญ ์ธ๊ทผ (๋ช…์นญ 3 ์ƒ๋žต) ์ปคํ”ผ์ˆ์—์„œ ๊ณต์†Œ์™ธ 5์™€ ๋งŒ๋‚˜ ํ”ผ๊ณ ์ธ์˜ ํƒœ๋ธ”๋ฆฟ PC์— ์ €์žฅ๋˜์–ด ์žˆ๋˜ โ€˜2014๋…„ ๊น€์ •์€ ์‹ ๋…„์‚ฌโ€™, โ€˜์ผ์‹ฌ๋‹จ๊ฒฐ์˜ ์œ„๋ ฅ ๋–จ์น˜๋ฉฐ ์กฐ๊ตญ์ด์—ฌ ์•ž์œผ๋กœ!โ€™๋ผ๋Š” ์ œ๋ชฉ์˜ 2014. 1. 4.์ž ๋…ธ๋™์‹ ๋ฌธ ๊ธฐ์‚ฌ, โ€˜์Šน๋ฆฌ๋ฅผ ํ–ฅํ•˜์—ฌ ์ง„๊ตฐ ๋˜ ์ง„๊ตฐโ€™์ด๋ผ๋Š” ์ œ๋ชฉ์˜ 2014. 1. 4.์ž ๋…ธ๋™์‹ ๋ฌธ ๊ธฐ์‚ฌ๋ฅผ ๊ณต์†Œ์™ธ 5์—๊ฒŒ ์—ด๋žŒ์‹œ์ผฐ๋Š”๋ฐ, ๊ทธ ์ฃผ์š”๋‚ด์šฉ์€ ๋‹ค์Œ๊ณผ ๊ฐ™๋‹ค. ใ€Žใ€Šโ€˜2014๋…„ ๊น€์ •์€ ์‹ ๋…„์‚ฌโ€™ ์ฃผ์š”๋‚ด์šฉใ€‹ - ๋ฏธ๊ตญ๊ณผ ๋‚จ์กฐ์„ ํ˜ธ์ „๊ด‘๋“ค์€ ์กฐ์„ ๋ฐ˜๋„์™€ ์ฃผ๋ณ€์— ํ•ต์ „์Ÿ์žฅ๋น„๋“ค์„ ๋Œ€๋Œ€์ ์œผ๋กœ ๋Œ์–ด๋“ค์—ฌ ๋ถ์นจํ•ต์ „์Ÿ ์—ฐ์Šต์„ ๊ด‘๋ž€์ ์œผ๋กœ ๋ฒŒ๋ฆฌ๊ณ  ์žˆ์œผ๋ฉฐ ์ด๋กœ ํ•˜์—ฌ ์‚ฌ์†Œํ•œ ์šฐ๋ฐœ์ ์ธ ๊ตฐ์‚ฌ์  ์ถฉ๋Œ๋„ ์ „๋ฉด์ „์Ÿ์œผ๋กœ ๋ฒˆ์งˆ ์ˆ˜ ์žˆ๋Š” ์œ„ํ—˜ํ•œ ์ •์„ธ๊ฐ€ ์กฐ์„ฑ๋˜๊ณ  ์žˆ์Šต๋‹ˆ๋‹ค. - ๋ฐฑํ•ด๋ฌด์ตํ•œ ๋น„๋ฐฉ์ค‘์ƒ์„ ๋๋‚ผ ๋•Œ๊ฐ€ ๋˜์—ˆ์œผ๋ฉฐ ํ™”ํ•ด์™€ ๋‹จํ•ฉ์— ์ €ํ•ด๋ฅผ ์ฃผ๋Š” ์ผ์„ ๋” ์ด์ƒ ํ•ด์„œ๋Š” ์•ˆ ๋  ๊ฒƒ์ž…๋‹ˆ๋‹ค. ๋‚จ์กฐ์„  ๋‹น๊ตญ์€ ๋ฌด๋ชจํ•œ ๋™์กฑ๋Œ€๊ฒฐ๊ณผ ใ€ˆ์ข…๋ถใ€‰ ์†Œ๋™์„ ๋ฒŒ๋ฆฌ์ง€ ๋ง์•„์•ผ ํ•˜๋ฉฐ ์ž์ฃผ์™€ ๋ฏผ์ฃผ, ์กฐ๊ตญํ†ต์ผ์„ ์š”๊ตฌํ•˜๋Š” ๊ฒจ๋ ˆ์˜ ๋ชฉ์†Œ๋ฆฌ์— ๊ท€๋ฅผ ๊ธฐ์šธ์ด๊ณ  ๋ถ๋‚จ๊ด€๊ณ„ ๊ฐœ์„ ์—๋กœ ๋‚˜์™€์•ผ ํ•ฉ๋‹ˆ๋‹ค. ใ€Šโ€˜์ผ์‹ฌ๋‹จ๊ฒฐ์˜ ์œ„๋ ฅ ๋–จ์น˜๋ฉฐ ์กฐ๊ตญ์ด์—ฌ ์•ž์œผ๋กœ!โ€™ ์ œํ•˜์˜ ๊ธฐ์‚ฌ ์ฃผ์š”๋‚ด์šฉใ€‹ - ์šฐ๋ฆฌ์˜ ์ตœ๊ฐ•์˜ ๋ฌด๊ธฐ, ์‚ฌํšŒ์ฃผ์˜ ๊ฐ•์„ฑ๊ตญ๊ฐ€๊ฑด์„ค์˜ ์œ„๋ ฅํ•œ ์ถ”์ง„๋ ฅ์ธ ์ผ์‹ฌ๋‹จ๊ฒฐ! ์ •๋…• ๊ทธ๊ฒƒ์€ ์–ต๋งŒ๊ธˆ์—๋„ ๋น„๊ธธ ์ˆ˜ ์—†๋Š” ์šฐ๋ฆฌ์˜ ๊ฐ€์žฅ ํฐ ์žฌ๋ถ€์ด๋ฉฐ ์ ˆ์„ธ์˜ ์œ„์ธ์„ ๋†’์ด ๋ชจ์‹  ์„ ๊ตฐ์กฐ์„ ์˜ ์ž๋ž‘์Šค๋Ÿฌ์šด ํž˜์ด๋‹ค. - ์„ธ์ƒ์—๋Š” ๋‚˜๋ผ๋„ ๋งŽ๊ณ  ์ •์น˜๊ฐ€๋“ค๋„ ๋งŽ์ง€๋งŒ ์šฐ๋ฆฌ์˜ ๋Œ€์›์ˆ˜๋‹˜๋“ค๊ณผ ๊ฐ™์ด, ์šฐ๋ฆฌ ์›์ˆ˜๋‹˜์ฒ˜๋Ÿผ ๋Š˜ ์ธ๋ฏผ์„ ์ฐพ์•„๊ฐ€๊ณ  ์ธ๋ฏผ์˜ ๋ชฉ์†Œ๋ฆฌ์— ๊ท€๋ฅผ ๊ธฐ์šธ์ด๋ฉฐ ์ธ๋ฏผ๊ณผ ๋‹จ ํ•œ์น˜์˜ ๊ฐ„๊ฒฉ๋„ ์—†๋Š” ๊ทธ๋Ÿฐ ์œ„์ธ๋“ค์€ ์ผ์ฐ์ด ์—†์—ˆ๋‹ค. ใ€Šโ€˜์Šน๋ฆฌ๋ฅผ ํ–ฅํ•˜์—ฌ ์ง„๊ตฐ ๋˜ ์ง„๊ตฐ(์ •๋ก )โ€™ ์ œํ•˜์˜ ๊ธฐ์‚ฌ ์ฃผ์š”๋‚ด์šฉใ€‹ - ๊ฒฝ์• ํ•˜๋Š” ๊น€์ •์€ ๋™์ง€๊ป˜์„œ๋Š” ๋‹ค์Œ๊ณผ ๊ฐ™์ด ๋ง์”€ํ•˜์‹œ์˜€๋‹ค. ใ€ˆ์˜ฌํ•ด ์šฐ๋ฆฌ์˜ ํˆฌ์Ÿ์€ ์ธ๋ฏผ์˜ ์•„๋ฆ„๋‹ค์šด ๋ฆฌ์ƒ๊ณผ ๊ฟˆ์„ ์•ž๋‹น๊ฒจ ์‹คํ˜„ํ•˜๊ธฐ ์œ„ํ•œ ๋ณด๋žŒ์ฐฌ ํˆฌ์Ÿ์ด๋ฉฐ ์˜๊ด‘์Šค๋Ÿฌ์šด ์กฐ์„ ๋กœ๋™๋‹น ์ฐฝ๊ฑด 70๋์„ ๋น›๋‚˜๊ฒŒ ์žฅ์‹ํ•  ๋Œ€์ถ•์ „์žฅ๊ณผ ์ž‡๋‹ฟ์•„ ์žˆ๋Š” ์Šน๋ฆฌ์ž์˜ ์ง„๊ตฐ์ž…๋‹ˆ๋‹ค. ์šฐ๋ฆฌ๋Š” ํฌ๋ง์ฐฌ ์ƒˆํ•ด์— ์Šน๋ฆฌ์— ๋Œ€ํ•œ ํ™•์‹ ๊ณผ ์—ด์ •์— ๋„˜์ณ ์˜์›…์  ์ง„๊ตฐ์„ ๋‹ค๊ทธ์นจ์œผ๋กœ์จ ํ˜๋ช…๊ณผ ๊ฑด์„ค์—์„œ ์ผ๋Œ€ ๋ฒˆ์˜๊ธฐ๋ฅผ ์—ด์–ด ๋†“์•„์•ผ ํ•ฉ๋‹ˆ๋‹ค.ใ€‰ - ์ด ๋•…์— ์ƒ์„ ๋‘” ๋ชจ๋“  ์‚ฌ๋žŒ๋“ค์ด ๊ฒฝ์• ํ•˜๋Š” ์›์ˆ˜๋‹˜์˜ ์• ๊ตญ์˜ ๋ณดํญ์— ์‹ฌ์žฅ์˜ ๋ฐ•๋™์„ ๋งž์ถ”์–ด ์ฒœ๊ธˆ๊ฐ™์ด ๊ท€์ค‘ํ•œ ์˜ค๋Š˜์˜ ํ•˜๋ฃจํ•˜๋ฃจ๋ฅผ ๊ธฐ์ ๊ณผ ์œ„ํ›ˆ์œผ๋กœ ๋น›๋‚ด์ผ ๋•Œ ์šฐ๋ฆฌ์˜ ๋ชจ๋“  ์ดˆ์†Œ์™€ ์ผํ„ฐ๋“ค์ด ๋ชฉํ‘œ๋ฅผ ๋†’์ด ์„ธ์šฐ๊ณ  ์ „์ง„ ๋˜ ์ „์ง„ํ•˜๋Š” ์• ๊ตญ์ž ์ง‘๋‹จ์ด ๋˜๊ณ  ํ˜์‹ ์ž์ง‘๋‹จ์ด ๋  ๋•Œ ์‚ฌ๋ž‘ํ•˜๋Š” ์–ด๋จธ๋‹ˆ ์šฐ๋ฆฌ ์กฐ๊ตญ์€ ์˜จ ์„ธ์ƒ์ด ๋ถ€๋Ÿฌ์›Œ ๋ฐ”๋ผ๋ณด๋Š” ์ฃผ์ฒด์˜ ์‚ฌํšŒ์ฃผ์˜ ๊ฐ•์„ฑ๊ตญ๊ฐ€๋กœ ๋ˆˆ๋ถ€์‹œ๊ฒŒ ์†Ÿ๊ตฌ์ณ ์˜ค๋ฅผ ๊ฒƒ์ด๋‹ค.ใ€ ์ด์™€ ๊ฐ™์ด ์œ„ ๊ธฐ์‚ฌ๋“ค์€ ๊น€์ •์€์˜ ์ง€๋„๋ ฅ์„ ๋ฏธํ™”ํ•˜๋ฉด์„œ ๋ถํ•œ์ฒด์ œ๋ฅผ ์‚ฌํšŒ์ฃผ์˜ ๊ฐ•์„ฑ๋Œ€๊ตญ์ด๋ผ๊ณ  ์ฐฌ์–‘ํ•˜๋Š” ๋‚ด์šฉ๋“ค์ด๋‹ค. ํ”ผ๊ณ ์ธ์€ ๊ณต์†Œ์™ธ 5์—๊ฒŒ ์œ„ ๊ธฐ์‚ฌ๋“ค์„ ์—ด๋žŒํ•˜๊ฒŒ ํ•œ ํ›„, ๊ณต์†Œ์™ธ 5์™€ ํ•จ๊ป˜ ์œ„ ๊ธฐ์‚ฌ์— ๊ด€ํ•ด ํ† ๋ก ํ•˜์˜€๋Š”๋ฐ, ๊ทธ ์ฃผ์š” ๋Œ€ํ™”๋‚ด์šฉ์€ ๋‹ค์Œ๊ณผ ๊ฐ™๋‹ค. ใ€Žใ€Š2014. 1. 4. ํ”ผ๊ณ ์ธ๊ณผ ๊ณต์†Œ์™ธ 5 ๊ฐ„์˜ ์ฃผ์š” ๋Œ€ํ™”๋‚ด์šฉใ€‹ (ํ”ผ๊ณ ์ธ) : ์‹ ๋…„์‚ฌ๋Š” ๋ณด์…จ๋‚˜์š”? (๊ณต์†Œ์™ธ 5) : ์˜ˆ, ์‹ ๋…„์‚ฌ๋Š” ๋ดค๋Š”๋ฐ (ํ”ผ๊ณ ์ธ) : ๋ณด์‹œ๊ณ  ๋“ค์œผ์‹œ๋ฉด์„œ ๋Š๋‚Œ์€ ์–ด๋– ์…จ์–ด์š”? (๊ณต์†Œ์™ธ 5) : ๊ตฌ์ฒด์ ์ธ ์‚ฌํšŒ์ ์ธ ์›๋ฌด๊ฐ€, ๋ชจ์Šต์ด ๊ทธ๋ ค์ง„ ๊ฒƒ ๊ฐ™๋‹ค. (ํ”ผ๊ณ ์ธ) : ์ด์ œ๋Š” ๋ญ ๊ณผ๊ฑฐ์˜ ๊ฒฝ์ œ๋‚œ์ด๋‚˜ ์–ด๋–ค ์ด๋Ÿฐ ๋ถ€๋ถ„๋“ค์„ ์ด๊ฒจ๋‚˜๊ฐ€์ž ์ด๊ฒƒ์ด ์•„๋‹ˆ๋ผ ์ด์   ๊ฑด์„คํ•˜๊ณ  ์ฐฝ์กฐํ•˜๊ณ  ์‚ฌํšŒ์  ๊ณต์‚ฐ๊ตญ๊ฐ€, ์‚ฌํšŒ์  ๋ฌธ๋ช…๊ตญ์„ ๊ฑด์„คํ•˜์ž๋Š” ๋น„์•ฝ์˜ ํ•œํ•ด๋กœ ์‚ผ์ž. ๊ทธ๋ž˜์„œ ๊ตฌํ˜ธ๋„ ๋น„์•ฝ์ด์ž–์•„์š”. (ํ”ผ๊ณ ์ธ) : ์šฐ๋ฆฌ๋ฏผ์กฑ๋ผ๋ฆฌ ์กฐ๊ตญํ†ต์ผ 3๋Œ€์›์น™๊ณผ 6.15, 10.4์„ ์–ธ ์›์น™ ์•„๋ž˜์„œ ์šฐ๋ฆฌ๋ฏผ์กฑ๋ผ๋ฆฌ ์ž…์žฅ์„ ๊ฐ€์ง€๊ณ  ๋‚จ๋ถ๊ด€๊ณ„๋ฅผ ๊ฐœ์„ ํ•ด ๋‚˜๊ฐ€์ž. (ํ”ผ๊ณ ์ธ) : ์ด์ ์œผ๋กœ๋Š” ๋ญ ์ด๋ ‡๊ฒŒ ํŠน๋ณ„ํžˆ ๋“œ๋Ÿฌ๋‚œ ๊ฑด ์—†์ง€๋งŒ, ํŠน๋ณ„ํžˆ ๋ญ˜ ์–ด๋–ป๊ฒŒ ํ•˜์ž ๋ญ ์ด๊ฒƒ์ด ์•„๋‹ˆ๋ผ ๊ทธ๊ฒƒ์€ ์•„๋‹ˆ์ง€๋งŒ, ์–ด์จŒ๋“  ์ „์ฒด์ ์ธ ์ฃผ์ฒด๊ฐ€ ๊ฐ•ํ™”๋œ ๋ฐ์—์„œ ๋‚˜์˜ค๋Š” ์ž์‹ ๊ฐ๊ณผ ํ˜„ ์ƒํ™ฉ์— ๋Œ€ํ•œ ์–ด๋–ค ๊ด€๋ฆฌ์™€ ๋˜ ์•ž์œผ๋กœ ์ƒํ™ฉ์„ ํ•ด๋‚˜๊ฐ€๋Š” ๋ถ€๋ถ„์— ๋Œ€ํ•ด์„œ ์ž์‹ ๊ฐ๊ณผ ์•ˆ์ •๊ฐ์ด ์žˆ๋Š” ๊ทธ๋Ÿฐ ์‹ ๋…„์‚ฌ ๊ฐ™์•„์š”. (๊ณต์†Œ์™ธ 5) : ๋‹ค๋ฅธ ํ•ด๋ณด๋‹ค (ยท) ์ด์ œ ํ•ต, ํ•ต. ๊ฒฝ์ œ๊ฑด์„ค, ๊ฒฝ์ œ, ํ•ต๊ณผ ๋ณ‘์ง„๋…ธ์„ ์ด ์ด์ œ ๋‹ค์ ธ์ง„ ๊ฒƒ ๊ฐ™๊ณ  ์ข€ ๋ฐ˜๋ฐœ์ด ์žˆ์ง€๋งŒ, ์ข€ ์•„์‰ฌ์›€์ด ๋‚จ๋Š” ๊ฒƒ์ด ์ข€ ๋นจ๋ฆฌ๋นจ๋ฆฌ ๋๋ƒˆ์œผ๋ฉด ์ข‹๊ฒ ๋Š”๋ฐ, ์ข€ ๋Šฆ์ถฐ์ง€๋Š” ๊ฒƒ ๊ฐ™๋‹ค๋Š” ์ƒ๊ฐ์ด (ํ”ผ๊ณ ์ธ) : ์ €๋Š” ์ด์ œ ์–ด๋–ป๊ฒŒ ๋ณด๋ฉด ์ด๋‚จ์˜ ์‚ฌํšŒ๋ณ€ํ˜ ๋ถ€๋ถ„์ด๊ธฐ ๋•Œ๋ฌธ์— ์‹ ๋…„์‚ฌ์—์„œ ๋ถ์ด ๋‹ค๋ฃฐ ์ˆ˜ ์—†๋Š” ๋ถ€๋ถ„์ด ์žˆ์ž–์•„์š”. ์‹ ๋…„์‚ฌ์—์„œ ๋ถ์ด ๋ญ ์ง์ ‘์ ์œผ๋กœ ๋‚จ์ชฝ์˜ ์ •์น˜๋ฌธ์ œ. ๋ถ์—์„œ ํ•  ์ˆ˜ ์žˆ๋Š” ๊ฒƒ์€ โ€˜์ข…๋ถ๋ชฐ์ด ํ•˜์ง€ ๋งˆ๋ผโ€™โ€˜๊ด€๊ณ„ ๊ฐœ์„ ํ•ด๋‚˜๊ฐ€์žโ€™์ด๋Ÿฐ ์ •๋„์ง€, ์ด๋‚จ์˜ ์‚ฌํšŒ๋ณ€ํ˜์šด๋™์€ ์ด๋‚จ์˜ ๋ณ€ํ˜์šด๋™์€ ๋˜ ๋‹ค๋ฅธ ์ฐจ์›์—์„œ ์šฐ๋ฆฌ๊ฐ€ ์ƒ๊ฐ์„ ํ•˜๊ณ , ์ด๋Ÿฐ ๊ฒƒ์€ ๋…ผ์™ธ์ , ๊ณต๊ฐœ์ ์œผ๋กœ ํ•˜๋Š” ๊ฒƒ์ด๋‹ˆ๊นŒ. ๋‚ด์ ์œผ๋กœ ํ•˜๋Š” ์ด ๋ณ€ํ˜์šด๋™์—๋„ ๋ณด๋ฉด ๋˜ ๋‹ค๋ฅธ ๊ฒƒ์ด์ฃ . ๋‚จ์กฐ์„  ํ˜๋ช… ๊ฐ™์€ ๊ฒƒ์„ ์ง์ ‘ ์–ธ๊ธ‰ํ•  ์ˆœ ์—†๋Š” ๊ฒƒ์ด๊ณ . (ํ”ผ๊ณ ์ธ) : ์ด๋‚จ์— ์šฐ๋ฆฌ๋Š” ํ•œํŽธ์—์„œ๋Š” ์ž๊ธฐ (ยท) ๊ตณ์„ธ๊ฒŒ ๊ทธ๋Ÿฌ๋‹ˆ๊นŒ ์ผ์ข…์˜ ์ž๋ ฅ๊ฐฑ์ƒ ์œ„์— ์ž๊ธฐํ† ๋Œ€๋ฅผ ๊ฐ•ํ™”ํ•˜๊ณ  ๋•Œ๊ฐ€๋˜๋ฉด ๊ณต๊ฒฉ์— ๋‚˜์„œ์„œ ๊ณต๊ฒฉ์˜ ์—ญ๋Ÿ‰์„ ๊ฐ•ํ™”ํ•˜๋Š”, ์ด ๋‘˜์„ ๋™์‹œ์ ์œผ๋กœ ์šฐ๋ฆฌ๋Š” ๋Œ€๋น„ํ•ด์•ผ ๋œ๋‹ค. (ํ”ผ๊ณ ์ธ) : ๊ทธ๊ฑฐ๋ฅผ ๋ช…์‹œ์ ์œผ๋กœ ์šฐ๋ฆฌ๊ฐ€ ๋ง์„ ํ•˜์ง€ ์•Š์„ ๋ฟ์ด์ง€ ์ •์„ธํ˜•๊ตญ์—์„œ๋Š” ๋ฐ•๊ทผํ˜œ๊ฐ€ ์Šค์Šค๋กœ ๊ตญ๊ฐ€์— ๋Œ€ํ•ด์„œ, ํŠนํžˆ ์ด์ œ ๋ถ€์ •์„ ๊ฑฐ๋ฌธ์ œ๊ฐ€ ๊ฑธ๋ ค ์žˆ๊ธฐ ๋•Œ๋ฌธ์— ์ •๊ถŒํ‡ด์ง„์ด๊ฒ ์ฃ . ํ‡ด์ง„์ด ์„ค์‚ฌ ์•ˆ ๋˜์—ˆ๋‹ค ํ•˜๋”๋ผ๋„ ์ด์ œ ์‚ฌ์‹ค์ƒ์€ ์ •๊ถŒ์˜ ์‹ค์„ธ์—์„œ๋Š” ์•„์›ƒ์ด ๋˜๋Š”, ๋งˆ์น˜ 80๋…„๋Œ€ ์ „๋‘ํ™˜ ๋•Œ์ฒ˜๋Ÿผ (ยท) ์˜จ ๊ตญ๋ฏผ๋“ค์ด ๋‹ค ๋ฐ˜๋…์žฌํˆฌ์Ÿ์˜ ๋งค๊ฐœ์ฒด์ด๋‹ค. ์ด์ œ๋Š” ๋ฐ˜๋Œ€ํˆฌ์Ÿ, ๋ฐ˜๋ฐ•๊ทผํ˜œ ํˆฌ์Ÿ์— ์˜จ ๊ตญ๋ฏผ์ด ๋‹ค ๋‚˜์„œ์„œ ๋‚จ๋ถ๊ด€๊ณ„๋ฅผ ๊ฐœ์„ ํ•˜๊ณ  ํ•œ๋ฐ˜๋„ ํ‰ํ™”ํ˜‘์ •์— (ยท) ํ•˜๋Š” (---) ํ˜„์‹คํ™” ๋  ์ˆ˜ ์žˆ๋Š” ๊ฒƒ์ด์ฃ . ๊ทธ๋Ÿฐ ์ชฝ์œผ๋กœ ๋‚˜๊ฐ€๋Š” ์ž์‹ ๊ฐ (ยท) ์•ˆ์ •๊ฐ ๊ทธ๋ฆฌ๊ณ  ๊ด€๋ฆฌ์™€ ๋งŒ์•ฝ์˜ ์ •์„ธ์— ๋Œ€ํ•œ ๊ฒฝ๊ณ ์™€ ๋Œ€๋น„. ์ด๋Ÿฐ ๋ถ€๋ถ„๋“ค๊นŒ์ง€ ๋“œ๋Ÿฌ๋‚œ ํ•˜๋‚˜์˜ ์‹ ๋…„์‚ฌ๊ฐ€ ์•„๋‹ˆ๊ฒ ๋Š”๊ฐ€. (ํ”ผ๊ณ ์ธ) : ์ง€๊ธˆ ๋ญ ์‹ ๋…„์‚ฌ๋„ ๋ณด์‹œ๊ณ , ์ผ๋ฐ˜์ ์ธ ์˜ฌํ•ด ์ „์ฒด์˜ ์šฐ๋ฆฌ ๋Œ€์˜ค ์šฐ๋ฆฌ ์กฐ์ง์˜ ๊ธฐ์กฐ๋ฅผ ์ฒซ์งธ๋Š” ์ง€๋‚œ๋ฒˆ์— ๊ฒฐ์˜ํ•œ ๊ทธ๋Œ€๋กœ ์šฐ๋ฆฌ ๋Œ€์˜ค์— ์œ ์ผ์˜๋„ ์ฒด๊ณ„๋ฅผ ์„ธ์›Œ๋‚˜๊ฐ€์•ผ๊ฒ ๋‹ค. ๊ทธ๋Ÿฌ๊ธฐ ์œ„ํ•ด์„œ๋Š” ๋ช‡ ๊ฐ€์ง€๋ฅผ ์ข€ ํ•ด์•ผ ๋˜๊ฒ ๋‹ค. ์ฒซ ๋ฒˆ์งธ๋กœ๋Š” ์‚ฌ์ƒํ•™์Šต์„ ๊ฐ•ํ™”ํ•ด์•ผ ๋˜๊ฒ ๋‹ค. ๋‘ ๋ฒˆ์งธ๊ฐ€ ์ด์ œ ๊น€์ฃผ์˜ ํ•™์Šต, ๊น€์ผ์„ฑ ๊น€์ •์ผ (---) ๊น€์ฃผ์˜ ํ•™์Šต, ์„ธ ๋ฒˆ์งธ๊ฐ€ ์‚ฌ์ƒํ•™์Šต๊ณผ ๊ด€๋ จํ•ด์„œ๋Š” ์›์ˆ˜๋‹˜์˜ ๋…ธ์ž‘. 100์ฃผ๋…„ ์—ด๋ณ‘์‹ ๋•Œ ํ–ˆ๋˜ ์—ฐ์„ค๋ฌธ์ด๋ผ๋“ ๊ฐ€ ๋‹ดํ™” ๋…ธ์ž‘, ๋…ผ๋ฌธ ์ด๋Ÿฐ ๊ฒŒ ์žˆ์–ด์š”. ๊ทธ๊ฑฐ๋Š” ์ œ๊ฐ€ ๋งŒ๋“ค์–ด์˜ฌ๊ฒŒ์š”. ํ•œ๊ธ€๋กœ๋‹ค๊ฐ€ (๊ณต์†Œ์™ธ 5) : USB๋ฅผ ๊ฐ€์ ธ์˜ฌ๊นŒ์š”? (ํ”ผ๊ณ ์ธ) : ์˜ˆ. ๊ฐ€์ ธ์˜ค์‹œ๋ฉด, ์ œ๊ฐ€ ๋‹ค ์ค€๋น„ํ•ด์„œ ๋ณต์‚ฌํ•ด ๋“œ๋ฆด๊ฒŒ์š”. ์ œ๊ฐ€ ๋‹ค ์ค€๋น„ํ•ด์„œ ๋ณต์‚ฌํ•ด ๋“œ๋ฆด๊ฒŒ์š”. ์˜ฌํ•ด ์œ ์ผ์˜๋„ ์ฒด๊ณ„์˜ ๋ฐฉํ–ฅ ์‚ฌ์ƒํ•™์Šต (ยท) ํšŒํ•ฉํ•  ๋•Œ๋งˆ๋‹ค ์‚ฌ์ƒํ•™์Šต ์ง„ํ–‰ ์ •ํ˜•์— ๋Œ€ํ•ด์„œ ์ดํ™”ํ•˜๊ณ  ๋˜ ๊ทธ์— ๋Œ€ํ•ด์„œ (ยท) ์ดํ™”ํ•˜๊ณ , ํ•™์Šต๋‚ด์šฉ๋“ค๋กœ ํ† ๋ก ํ•˜๊ณ  ์–ด๋–ค ๋‚ด์šฉ์ด ์ข‹์•˜๋‹ค๋ผ๊ณ  ํ•˜๋Š” ๊ฒƒ๊นŒ์ง€. ์‚ฌ์ƒํ•™์Šต ์ดํ™”๋ฅผ ๊พธ์ค€ํ•˜๊ฒŒ ํšŒํ•ฉ ๋•Œ๋งˆ๋‹ค ํ•ด์•ผ ๋˜๊ฒ ๋‹ค ํ•˜๋Š” ๋ถ€๋ถ„์ด๊ณ . ๋‘ ๋ฒˆ์งธ๊ฐ€ ์ด์ œ ์กฐ์ง์ƒํ™œ์˜ ๊ฐ•ํ™”, ๊ณผ์—…๊ณผ ์ƒํ™œ์— ๋Œ€ํ•œ ์ดํ™”, ์„ธ ๋ฒˆ์งธ๋Š” ๋ณด์•ˆ๋ฌธ์ œ์ฃ ? ๋ณด์•ˆ๋ฌธ์ œ. ํšŒํ•ฉํ•  ๋•Œ์˜ ๋ณด์•ˆ, ์ž์œ ๊ณต๊ฐ„์—์„œ์˜ ๋ณด์•ˆ, ๋ญ ๊ทธ ๋ฐ–์˜ ์—ฌ๋Ÿฌ ์ฃผ๋ณ€์— ๋Œ€ํ•œ ๋ณด์•ˆใ€ ์ด์™€ ๊ฐ™์ด ํ”ผ๊ณ ์ธ์€ ๊ณต์†Œ์™ธ 5์—๊ฒŒ ์œ„ ๊ธฐ์‚ฌ๋“ค์„ ์—ด๋žŒํ•˜๊ฒŒ ํ•œ ํ›„, 2014๋…„ ๊น€์ •์€ ์‹ ๋…„์‚ฌ๋ฅผ ์ฝ์–ด๋ณธ ๋Š๋‚Œ์— ๊ด€ํ•˜์—ฌ, โ€œ์กฐ๊ตญํ†ต์ผ 3๋Œ€์›์น™๊ณผ 6ยท15, 10ยท4์„ ์–ธ ์›์น™ ์•„๋ž˜ ์šฐ๋ฆฌ๋ฏผ์กฑ๋ผ๋ฆฌ ์ž…์žฅ์„ ๊ฐ€์ง€๊ณ  ๋‚จ๋ถ๊ด€๊ณ„๋ฅผ ๊ฐœ์„ ํ•ด ๋‚˜๊ฐ€์ž๋Š” ๋‚ด์šฉ์œผ๋กœ์„œ ์ž์‹ฌ๊ฐ๊ณผ ์•ˆ์ •๊ฐ์ด ์žˆ๋Š” ์‹ ๋…„์‚ฌโ€๋ผ๊ณ  ํ‰๊ฐ€ํ•˜๊ณ , ์‹ ๋…„์‚ฌ์— ๋ถ€ํฅํ•˜๊ธฐ ์œ„ํ•ด ์œ ์ผ์˜๋„์ฒด๊ณ„๋ฅผ ์„ธ์›Œ๋‚˜๊ฐ€์•ผ ํ•˜๊ณ , ์ด๋ฅผ ์œ„ํ•˜์—ฌ ์‚ฌ์ƒํ•™์Šต์„ ๊ฐ•ํ™”ํ•˜๊ณ , ๊น€์ผ์„ฑ-๊น€์ •์ผ์ฃผ์˜๋ฅผ ๊ฐ•ํ™”ํ•˜๋ฉฐ, ๋ณด์•ˆ์„ ๊ฐ•ํ™”ํ•˜์—ฌ ์กฐ์ง์ƒํ™œ์„ ๊ฐ•ํ™”ํ•˜์ž๊ณ  ๋‹ค์งํ•˜๋Š” ๋“ฑ ์‚ฌ์ƒํ•™์Šต์„ ํ•˜์˜€๋‹ค. ์ด๋กœ์จ ํ”ผ๊ณ ์ธ์€ ๊ตญ๊ฐ€์˜ ์กด๋ฆฝยท์•ˆ์ „์ด๋‚˜ ์ž์œ ๋ฏผ์ฃผ์  ๊ธฐ๋ณธ์งˆ์„œ๋ฅผ ์œ„ํƒœ๋กญ๊ฒŒ ํ•œ๋‹ค๋Š” ์ •์„ ์•Œ๋ฉด์„œ, ๋ฐ˜๊ตญ๊ฐ€๋‹จ์ฒด๋‚˜ ๊ทธ ๊ตฌ์„ฑ์› ๋˜๋Š” ๊ทธ ์ง€๋ น์„ ๋ฐ›์€ ์ž์˜ ํ™œ๋™์„ ์ฐฌ์–‘ยท๊ณ ๋ฌดยท์„ ์ „ํ•  ๋ชฉ์ ์œผ๋กœ ์ด์ ํ‘œํ˜„๋ฌผ์„ ์†Œ์ง€ํ•˜๊ณ , ๊ณต์†Œ์™ธ 5์™€ ๊ณต๋ชจํ•˜์—ฌ ๋ฐ˜๊ตญ๊ฐ€๋‹จ์ฒด๋‚˜ ๊ทธ ๊ตฌ์„ฑ์› ๋˜๋Š” ๊ทธ ์ง€๋ น์„ ๋ฐ›์€ ์ž์˜ ํ™œ๋™์— ๋™์กฐํ•˜์˜€๋‹ค. ๋ผ. 2014. 2. 3. ๊ณต์†Œ์™ธ 4์™€ ์‚ฌ์ƒํ•™์Šต(์ด์ ๋™์กฐ) ํ”ผ๊ณ ์ธ์€ 2014. 2. 3. 15:50๊ฒฝ๋ถ€ํ„ฐ ๊ฐ™์€ ๋‚  21:10๊ฒฝ๊นŒ์ง€ ์„œ์šธ ์ข…๋กœ๊ตฌ์— ์žˆ๋Š” (๋ช…์นญ 53 ์ƒ๋žต)ํšŒ๊ด€ ๋‚ด (๋ช…์นญ 29 ์ƒ๋žต)์—ฐ๊ตฌ์†Œ์—์„œ ๊ณต์†Œ์™ธ 4์™€ ํ•จ๊ป˜ โ€˜์šฐ๋ฆฌ๋ฏผ์กฑ๋ผ๋ฆฌ ๋‹จํ•ฉ๋œ ํž˜์œผ๋กœ ๋ถ๋‚จ๊ด€๊ณ„๊ฐœ์„ ์˜ ํ™œ๋กœ๋ฅผ ์—ด์–ด๋‚˜๊ฐ€์ž ๋‚จ์กฐ์„  ๋‹น๊ตญ์— ๋ณด๋‚ด๋Š” ์ค‘๋Œ€์ œ์•ˆโ€™์ด๋ผ๋Š” ์ œ๋ชฉ์˜ 2014. 1. 16.์ž ๋ถํ•œ ๊ตญ๋ฐฉ์œ„์›ํšŒ๊ฐ€ ๋ฐœํ‘œํ•œ ์„ฑ๋ช…์„œ๋ฅผ ์ฝ์–ด๋ณด์•˜๋Š”๋ฐ, ๊ทธ ์ฃผ์š”๋‚ด์šฉ์€ ๋‹ค์Œ๊ณผ ๊ฐ™๋‹ค. ใ€Žใ€Š๋ถํ•œ ๊ตญ๋ฐฉ์œ„์›ํšŒ ์„ฑ๋ช…์„œ ์ฃผ์š”๋‚ด์šฉใ€‹ - ์ด ๋•…์— ์ดˆ๋ž˜ํ•  ํ•ต ์žฌ๋‚œ์„ ๋ง‰๊ธฐ ์œ„ํ•œ ํ˜„์‹ค์ ์ธ ์กฐ์น˜๋„ ํ˜ธ์ƒ ์ทจํ•ด๋‚˜๊ฐˆ ๊ฒƒ์„ ์ œ์•ˆํ•œ๋‹ค. ์กฐ์„ ๋ฐ˜๋„๋น„ํ•ตํ™”๋Š” ๋ฏผ์กฑ๊ณต๋™์˜ ๋ชฉํ‘œ์ด๋‹ค. ๋”ฐ๋ผ์„œ ์กฐ์„ ๋ฐ˜๋„๋น„ํ•ตํ™”๋ฅผ ์‹คํ˜„ํ•˜๋ ค๋Š” ๊ฒƒ์€ ์šฐ๋ฆฌ ๊ตฐ๋Œ€์™€ ์ธ๋ฏผ์˜ ๋ณ€ํ•จ์—†๋Š” ์˜์ง€์ด๋‹ค. ์šฐ๋ฆฌ๊ฐ€ ๋ณด์œ ํ•œ ํ•ต๋ฌด๋ ฅ๊ณผ ๋ณ‘์ง„๋กœ์„ ์— ๋Œ€ํ•˜์—ฌ ๋งํ•œ๋‹ค๋ฉด ๊ทธ๊ฒƒ์€ ์šฐ๋ฆฌ ๋ฏผ์กฑ๋ชจ๋‘์— ๋Œ€ํ•œ ๋ฏธ๊ตญ์˜ ํ•ต์œ„ํ˜‘๊ณผ ๊ณต๊ฐˆ์„ ์ข…์‹์‹œํ‚ค๊ณ  ์กฐ์„ ๋ฐ˜๋„์˜ ๋น„ํ•ตํ™”๋Š” ๋ฌผ๋ก  ์„ธ๊ณ„์˜ ๋น„ํ•ตํ™”๊นŒ์ง€ ๋‚ด๋‹ค๋ณธ ๋ฏผ์กฑ๊ณต๋™์˜ ๋ณด๊ฒ€์ด๋ฉฐ ๊ฐ€์žฅ ์ •๋‹นํ•œ ์ž์œ„์ ์ธ ์„ ํƒ์ด๋‹ค. ์šฐ๋ฆฌ ํ•ต ๋ฌด๋ ฅ์€ ์ฒ ๋‘์ฒ ๋ฏธ ๋ฏธ๊ตญ์˜ ํ•ต์œ„ํ˜‘์„ ์–ต์ œํ•˜๊ธฐ ์œ„ํ•œ ์ˆ˜๋‹จ์ด์ง€ ๊ฒฐ์ฝ” ๋™์กฑ์„ ๊ณต๊ฐˆํ•˜๊ณ  ํ•ด์น˜๊ธฐ ์œ„ํ•œ ์ˆ˜๋‹จ์€ ์•„๋‹ˆ๋‹ค. ์šฐ๋ฆฌ๋Š” ์ด ๊ธฐํšŒ์— ๋‚จ์กฐ์„ ๋‹น๊ตญ์ด ๋” ์ด์ƒ ๋ฏธ๊ตญ์˜ ์œ„ํ—˜์ฒœ๋งŒํ•œ ํ•ตํƒ€๊ฒฉ ์ˆ˜๋‹จ๋“ค์„ ๋‚จ์กฐ์„ ๊ณผ ๊ทธ ์ฃผ๋ณ€์ง€์—ญ์— ๋Œ์–ด๋“ค์ด๋Š” ๋ฌด๋ชจํ•œ ํ–‰์œ„์— ๋งค๋‹ฌ๋ฆฌ์ง€ ๋ง๋ฐ ๋Œ€ํ•˜์—ฌ ์ •์ค‘ํžˆ ์ œ์•ˆํ•œ๋‹ค. - ์ด์ƒ๊ณผ ๊ฐ™์€ ์šฐ๋ฆฌ์˜ ์ค‘๋Œ€์ œ์•ˆ์—๋Š” ์™ธ์„ธ๋ฅผ ๋ฐฐ๊ฒฉํ•˜๊ณ  ์šฐ๋ฆฌ๋ฏผ์กฑ๋ผ๋ฆฌ์˜ ๋ฆฝ์žฅ์—์„œ ๋‚˜๋ผ์˜ ์ž์ฃผํ†ต์ผ๊ณผ ํ‰ํ™”๋ฒˆ์˜์„ ์•ž๋‹น๊ฒจ์˜ค๋ ค๋Š” ์˜จ ๊ฒจ๋ ˆ์˜ ์ง€ํ–ฅ๊ณผ ์š”๊ตฌ๊ฐ€ ๊ทธ๋Œ€๋กœ ๋‹ด๊ฒจ์ ธ ์žˆ๋‹ค. ์ด ์ค‘๋Œ€์ œ์•ˆ์ด ์‹คํ˜„๋˜๋ฉด ํฉ์–ด์ง„ ๊ฐ€์กฑ, ์นœ์ฒ™์ƒ๋ด‰์„ ๋น„๋กฏํ•˜์—ฌ ๋ถ๋‚จ๊ด€๊ณ„์—์„œ ์ œ๊ธฐ๋˜๋Š” ํฌ๊ณ  ์ž‘์€ ๋ชจ๋“  ๋ฌธ์ œ๋“ค์ด ๋‹ค ํ’€๋ฆฌ๊ฒŒ ๋  ๊ฒƒ์ด๋‹ค. ์šฐ๋ฆฌ ๋ฏผ์กฑ๋ผ๋ฆฌ์˜ ๋‹จํ•ฉ๋œ ํž˜์œผ๋กœ ๋ถ๋‚จ๊ด€๊ณ„๊ฐœ์„ ์˜ ํ™œ๋กœ๋ฅผ ์—ด์–ด๋‚˜๊ฐ€์ž๋Š” ๊ฒƒ์ด ์šฐ๋ฆฌ ๊ตฐ๋Œ€์™€ ์ธ๋ฏผ์˜ ํ•œ๊ฒฐ๊ฐ™์€ ์š”๊ตฌ์ด๋‹ค. - ๋ถ๋‚จ๊ด€๊ณ„๊ฐœ์„ ์˜ ๋ถ„์œ„๊ธฐ๋ฅผ ๋งˆ๋ จํ•  ๋ฐ ๋Œ€ํ•œ ๋œจ๊ฑฐ์šด ํ˜ธ์†Œ์— ๋ถ€์‘ํ•˜์—ฌ ์‹ค์ฒœ์ ์ธ ์กฐ์น˜๋ถ€ํ„ฐ ์ทจํ•  ๊ฒƒ์„ ์ œ์•ˆํ•œ๋‹ค. - ์ด๋กœ๋ถ€ํ„ฐ ์šฐ๋ฆฌ๋Š” ์˜ค๋Š” 1์›” 30์ผ๋ถ€ํ„ฐ ์„ค ๋ช…์ ˆ์„ ๊ณ„๊ธฐ๋กœ ์„œ๋กœ๋ฅผ ์ž๊ทนํ•˜๊ณ  ๋น„๋ฐฉ์ค‘์ƒํ•˜๋Š” ๋ชจ๋“  ํ–‰์œ„๋ถ€ํ„ฐ ์ „๋ฉด์ค‘์ง€ํ•˜๋Š” ์‹ค์ œ์ ์ธ ์กฐ์น˜๋ฅผ ์ทจํ•˜์ž๋Š” ๊ฒƒ์„ ๋‚จ์กฐ์„ ๋‹น๊ตญ์— ์ •์‹์œผ๋กœ ์ œ์˜ํ•œ๋‹ค.ใ€ ํ”ผ๊ณ ์ธ์€ ๊ณต์†Œ์™ธ 4์™€ ํ•จ๊ป˜ ์œ„ ์„ฑ๋ช…์„œ๋ฅผ ์ฝ๊ณ  ์œ„ ์„ฑ๋ช…์„œ์— ๊ด€ํ•ด ํ† ๋ก ํ•˜์˜€๋Š”๋ฐ, ๊ทธ ์ฃผ์š” ๋Œ€ํ™”๋‚ด์šฉ์€ ๋‹ค์Œ๊ณผ ๊ฐ™๋‹ค. ใ€Žใ€Š2014. 2. 3. ํ”ผ๊ณ ์ธ๊ณผ ๊ณต์†Œ์™ธ 4 ๊ฐ„์˜ ์ฃผ์š” ๋Œ€ํ™”๋‚ด์šฉใ€‹ (ํ”ผ๊ณ ์ธ) : ์กฐ์ง๊ด€ ํด๋”๋Š” ์–ด๋–ป๊ฒŒ ํ•˜๋‚˜? (๊ณต์†Œ์™ธ 4) : ์•„๋‡จ, ์•„์ง ๋ชปํ–ˆ์–ด์š” (ํ”ผ๊ณ ์ธ) : ์กฐ์ง๊ด€ ํด๋”, ์กฐ์ง๊ด€์ด ์ œ์ผ ์ค‘์š”ํ•ด, ์กฐ์ง์˜ ์ตœ์šฐ์„  ์‚ฌ์—…์ด๋‹ˆ๊นŒ, ํšŒ๊ณ ๋ก์„ ์ด๋ ‡๊ฒŒ ๋ณด๋ฉด ์žˆ์ž–์•„ ์™œ, ํšŒ๊ณ ๋ก์— ๋ณด๋ฉด ์˜คํžˆ๋ ค ๊น€์ฃผ์˜ ํ•™์Šต์„ ๋ฌด์กฐ๊ฑด์ ์œผ๋กœ ์ง„ํ–‰ํ•ด์•ผ ํ•œ๋‹ค. (ํ”ผ๊ณ ์ธ) : ์ฒซ ๋ฒˆ์งธ๋Š” ์ฐธ์—ฌ์—์„œ ์‚ฌ๋ก€๊ฐ€ ์„ธ ๊ฐ€์ง€๊ฐ€ ์žˆ๋Š”๋ฐ, ์–ด์ฐจํ”ผ ์กฐ์ง์ด ๊ฐ€์ง€๊ณ  ์žˆ๋Š” ์กฐ๊ธˆ ์ด๋ ‡๊ฒŒ ์‘ ๋”ฐ๋กœ ์žˆ์ž–์•„, ์กฐ์ง์ด๋ผ๊ณ  ํ•˜๋Š” ๊ฒƒ์€ ์‚ฌํšŒ์ •์น˜์ƒ๋ช…๊ณผ ํ•จ๊ป˜ ์ค‘์ถ”๋‹ค. ์ •์น˜์  ์ƒ๋ช…, ์‚ฌํšŒ, ์‚ฌํšŒ์— ์žˆ์–ด์„œ ์ •์น˜์ ์ธ ์—ญ์‚ฌ์ ์ธ ์ƒ๋ช…์ด ์ค‘์ถ”๋‹ค. ๋‘ ๋ฒˆ์งธ๋กœ๋Š” ๊ทผ๋ณธ์ ์ธ ๊ณผ์—…์ˆ˜ํ–‰์ด๋‚˜ ์ƒํ™œ์ด๋‚˜ ๋ชจ๋“  ๊ฒƒ์„ ์กฐ์ง์— ์ฒ ์ €ํžˆ ์œ„ํ•ด์„œ ์ง„ํ–‰ํ•ด์•ผ ํ•œ๋‹ค. ์ƒํ™ฉ์ดˆ๊ธฐ์— ์ง„ํ–‰ํ•˜๊ฒ ์ง€๋งŒ ์ง„๋กœ๋ฌธ์ œ ์ค€๋น„ํ•˜๊ณ , ์„œ๋กœ ๋ง์ด์ง€๋งŒ 1๋…„ 6๊ฐœ์›” ๋™์•ˆ์—, ์‚ฌ์‚ฌ๊ตฌ๋Š” 15๋…„์ด์ž–์•„, ๊ด€๊ณ„์™€ ์ •, ์ •์ด ์ด์ œ ์žˆ๋Š”๋ฐ, ์†”์งํžˆ ์ฒ ์ €ํ•˜๊ฒŒ ์‚ฌ์‹ค์€ ์‚ฌ๋žŒ๋„ ์˜ˆ๋ฅผ ๋“ค์–ด์„œ ์ด์ œ --- ์„ธ ๋ฒˆ์งธ๊ฐ€ ๊ทœ์œจ์„ ์ง€์ผœ๋ผ ์ค‘์š”ํ•œ ์ดํ™”๋ฅผ ์ž˜ํ•˜๊ณ  ๊ณผ์˜ค๋ฅผ ๋˜‘๋˜‘ํžˆ ๋‚ด์„ธ์šฐ๋Š” ๊ฒƒ์ด๊ณ , ์‚ฌ์—… ์ดํ™”๋ฅผ ์ž˜ํ•˜๊ณ , ์ผ์— ๋Œ€ํ•œ ๊ณผ์˜ค๋ฅผ ๋˜‘๋˜‘ํžˆ ๋‚ด์„ธ์šฐ๊ฑฐ๋‚˜ ์‚ฌ์—…ํ•˜๋Š” ๊ฑด ์ž๋ž‘์Šค๋Ÿฌ์šด ๊ฒƒ์ด๊ณ , ์ž˜๋ชปํ•˜๋ฉด ํž˜๋“  ๊ฒƒ์ด๋‹ค. ์ด๋ ‡๊ฒŒ ํ•  ์ˆ˜ ์žˆ์œผ๋ฉฐ, ์ œ2์˜ ์„ธ์ƒ์„ ๋ณด๋Š” ๋Œ€์ฑ…์„ ์„ธ์šธ ์ˆ˜ ์žˆ์Šต๋‹ˆ๋‹ค. ์ดํ™”๋ฅผ ๋˜‘๋˜‘ํžˆ ํ•ด์•ผ ๋‚˜๋Š” ์ž˜๋œ ๊ฒƒ๊ณผ ์ž˜๋ชป๋œ ๊ฒƒ, ๊ทธ๋ฆฌ๊ณ  --- ๊ฒƒ์„ ๋ณด๋ฉด์„œ ์‚ฌ์—…์„ ์œ„ํ•ด์„œ ๋Œ€์ฑ…์„ ๋งŒ๋“œ๋Š” ๊ฒƒ์ž…๋‹ˆ๋‹ค. ๊ทธ ๋Œ€์ฑ…์„ ์„ธ์šด ๊ฒƒ์ด ์‹ ๋…„์‚ฌ๊ฐ€ ๋•๋‹ด๊ณผ ํ•จ๊ป˜ ํ•˜๋Š” ๊ฒƒ์ด์•ผ ๋ง๋กœ ์ •์น˜ํŠน๋ณด๋ฅผ ๋‘๊ณ  ์‹ค์ฒœ์•„๋ž˜ ๊ตฌ์ฒด์ ์œผ๋กœ ์‚ฌ์—…ํ•  ๋ฐ ๋Œ€ํ•œ ๊ฒƒ์ด ์ด์ œ ์ผ๊พผ๋“ค ์ฆ‰ ๊ฐ„๋ถ€๋“ค์˜ ์ƒ๊ฐ์ด์š” ๋ฏฟ์Œ์ธ ๊ฒƒ์ธ๋ฐ, ์šฐ๋ฆฌ ์‹ค์ •์— ์˜ํ•ด์„œ ์šฐ๋ฆฌ ์ตœ๊ณ ์‚ฌ๋ น๊ด€ ๋ฐฉ์นจ์„ ๋”ฐ๋ฅด๋Š” ๊ฒƒ์„ ๊ทธ๊ฒŒ ์ด์ œ ์กฐ์ง์‚ฌ์—…์ธ ๊ฑฐ์ง€ (๊ณต์†Œ์™ธ 4) : ๊ทธ๋ ‡์ฃ , ์กฐ์ง์‚ฌ์—…์ด๋ผ๊ณ  ํ•˜๋Š” ๊ฒƒ์ด (ํ”ผ๊ณ ์ธ) : ์•ž์œผ๋กœ ๋‚˜๋จธ์ง€ ๋ฌธ์ œ๋“ค์€ ์ด๊ณณ์— ์ด๋ ‡๊ฒŒ ์กฐ์งํ™”, ์—ฌ์„ฑ ์žฌ์ •๊ฐ€๋“ค์— ๋Œ€ํ•ด์„œ๋Š”, ์„ธํฌ๋น„์„œ๋“ค์— ๋Œ€ํ•ด์„œ๋Š” ๋ณด์•ˆ๋ฌธ์ œ ํ–ˆ์œผ๋ฉด ์ข‹๊ฒ ๋‹ค. (ํ”ผ๊ณ ์ธ) : ๊ทœ์•ฝ ์„œ๋ฌธ ์žˆ์ž–์•„, ํด๋”์— ๋‹ค ์žˆ์–ด (๊ณต์†Œ์™ธ 4) : ์˜ˆ, ์„ธํฌ ์„œ๋ฌธ ๊ฐœ์ • ์žˆ๊ณ , ๊ทœ์•ฝ ์„œ๋ฌธ ์žˆ๊ณ  (ํ”ผ๊ณ ์ธ) : ์„ธํฌ ๋น„์„œ๋„ ์žˆ์ž–์•„ (๊ณต์†Œ์™ธ 4) : ์—†์–ด์š” (ํ”ผ๊ณ ์ธ) : ์„ธํฌ ์„œ๋ฌธ ๋„ฃ์–ด์คฌ์ž–์•„, ๊ทœ์•ฝ ์„œ๋ฌธ ๊ทธ ์•ˆ์—, ์„ธํฌ ๋น„์„œ๋ฅผ ์œ„ํ•˜์—ฌ (๊ณต์†Œ์™ธ 4) : ์ด๊ฑฐ ๋ณด์•ˆ์ด์ฃ ? ๊ทผ๋ฐ ์ด๋ ‡๊ฒŒ ์žˆ์œผ๋ฉด, ๋‹ค ์ง€์› ์–ด์š” ์ด์ œ (ํ”ผ๊ณ ์ธ) : ๋ณด์•ˆ์ด ๋ˆ„์„ค์ด ์ข€ ์•ˆ๋˜๊ฒŒ ํ•ด์•ผ์ง€ (ํ”ผ๊ณ ์ธ) : ์ด ๋•…์— ์ดˆ๋ž˜ํ•  ํ•ต ์žฌ๋‚œ์„ ๋ง‰๊ธฐ ์œ„ํ•œ ํ˜„์‹ค์ ์ธ ์กฐ์น˜๋„ ํ˜ธ์ƒ ์ทจํ•ด๋‚˜๊ฐˆ ๊ฒƒ์ด๋‹ค. ์ทจํ•ด๋‚˜๊ฐˆ ๊ฒƒ์„ ์ œ์•ˆํ•œ๋‹ค. ์šฐ๋ฆฌ๋Š”, ๋”ฐ๋ผ์„œ ๋น„ํ•ตํ™”๋ฅผ ์‹คํ˜„ํ•˜๋Š” ๊ฒƒ์€ ๋ณ€ํ•จ์—†๋Š” ์˜์ง€๋‹ค. ์šฐ๋ฆฌ๊ฐ€ ๋ณ‘์ง„๋…ธ์„ ์— --- ๊ฑฐ๊ธฐ์— ๋Œ€ํ•œ ๋ฏธ๊ตญ์˜ ํ•ต ์œ„ํ˜‘๊ณผ ๊ด€๋ จํ•ด ๊ฐ€์ง€๊ตฌ ๋น„ํ•ตํ™”๋ผ๋Š” ์„ธ๊ณ„์ ์ธ ๊ฒƒ์ด๋‹ค. ์šฐ๋ฆฌ ๋ฏผ์กฑ์€ ๋ฏธ๊ตญ์˜ ํ•ต ์œ„ํ˜‘์„ ์–ต์ œํ•˜๊ธฐ ์œ„ํ•œ ์ˆ˜๋‹จ์ด์ง€ ๋™์กฑ์„ ํ•ด์น˜๋Š” ์ˆ˜๋‹จ์€, ๋‚˜๋Š” ์ด ๊ธฐํšŒ์— ๊ทธ์ชฝ์—์„œ --- ๋ฐฉ์นจ ์•„๋ž˜ --- ์ฃผ๋ณ€์ง€์—ญ์„ ๋ฌด๋ชจํ•œ --- ๋งŒ๋“ค๊ธฐ ์œ„ํ•ด ์ •์ค‘ํžˆ ์ œ์•ˆํ•œ๋‹ค. ์™ธ์„ธ์— ์šฐ๋ฆฌ๋‚˜๋ผ๋ฅผ ์ง€ํ‚จ๋‹ค๋“ ์ง€ ์œ„ํ•œ ์ด์ค‘์ ์ธ ๋ฐ”ํƒ•์˜ ๊ฒฐ๊ณผ์ด๋‹ค ์ƒˆ๋กญ๊ฒŒ ์ฃผ์žฅํ•˜๋Š”, ์ด์ƒ์œผ๋กœ์„œ ์šฐ๋ฆฌ ์ค‘๋Œ€์ œ์•ˆ์—๋Š” ์™ธ์„ธ๋ฅผ ๋ฐฐ๊ฒฉํ•˜๊ณ  ๋ฏผ์กฑ์˜ ์ž…์žฅ์—์„œ ์ž์ฃผ --- ์ด ์ค‘๋Œ€์ œ์•ˆ์ด ์‹คํ˜„๋˜๋ฉด ๊ฐ€์กฑ ์นœ์ฒ™ ์ƒ๋ด‰๊ณผ ๊ฐ™์€ ํฌ๊ณ  ์ž‘์€ ๋ฌธ์ œ๋“ค์ด ๋‹ค ํ’€๋ฆฌ๊ฒŒ ๋  ๊ฒƒ์ด๋‹ค. ๋‹จํ•ฉ๋œ ์šฐ๋ฆฌ๋ฏผ์กฑ๋ผ๋ฆฌ --- ์ด๋‚จ ์ง€์—ญ์—์„œ ํ™”์š”์ผ ๋‚  ๋„˜์–ด๊ฐ€๋Š” ๊ฒƒ์ด ์šฐ๋ฆฌ์— ๋Œ€ํ•œ ์ค‘๋Œ€์ œ์•ˆ --- ๋ถ์ชฝ์—์„œ --- ์ฒซ ๋ฒˆ์งธ๊ฐ€, ๋น„๋ฐฉ์ค‘์ƒ ํšŒ์˜๋ฅผ ์ „๋ฉด ์ค‘์ง€ํ•˜์ž ์ด ์†Œ๋ฆฌ๊ฑฐ๋“  ๋‘ ๋ฒˆ์งธ๊ฐ€, ๊ตฐ์‚ฌ ์ €์ด‰ํ–‰์œ„, ๊ตฐ์‚ฌํ›ˆ๋ จ์„ ์ค‘์ง€ํ•˜๋Š” ์‹ค์ œ์ ์ธ ์กฐ์น˜๋ฅผ ์ทจํ•˜์ž. ์„ธ ๋ฒˆ์งธ๊ฐ€ ๋น„ํ•ตํ™”๋Š” ์™ธ์„ธ์— ํ•ต์„ ์šฐ๋ฆฌ๊ฐ€ ์ธ์ œ ๋Œ์–ด๋“ค์ด์ง€ ๋ง์ž. ๋ฏธ๊ตญ์˜ ์œ„ํ—˜์ฒœ๋งŒํ•œ ํ•ต ๊ด€๋ จ --- ์ค‘๋Œ€์ œ์˜๋ฅผ --- ๋ง๋ฐ ๋Œ€ํ•œ ์ •์ค‘ํžˆ ์ œ์•ˆ์„ ํ•˜๊ณ , ํ•ต์„ ๋Œ์–ด๋“ค์ด์ง€ ๋ง์ž.ใ€ ์ด์™€ ๊ฐ™์ด ํ”ผ๊ณ ์ธ์€ ๊ณต์†Œ์™ธ 4์—๊ฒŒ ์กฐ์ง๊ด€์˜ ์ค‘์š”์„ฑ์„ ๊ฐ•์กฐํ•˜๊ณ , ์กฐ์ง๋ณด์™„ ๊ด€๋ จ ์‚ฌ์ƒํ•™์Šต์„ ์ง„ํ–‰ํ•˜๋ฉด์„œ โ€œ์ตœ๊ณ ์‚ฌ๋ น๊ด€ ๋ฐฉ์นจ์„ ๋”ฐ๋ฅด์ž, ์œ ์ผ์˜๋„์ฒด๊ณ„ ๊ฐ•ํ™”ํ•˜์žโ€๋ผ๊ณ  ๊ต์œกํ•˜๊ณ , ๋ถํ•œ ๊ตญ๋ฐฉ์œ„์›ํšŒ ์„ฑ๋ช…์„œ์˜ ์ฃผ์š” ๋‚ด์šฉ์„ ์ฝ์–ด๊ฐ€๋ฉฐ ๊ทธ ์˜๋ฏธ๋ฅผ ์„ค๋ช…ํ•˜๋Š” ๋“ฑ ํ•ต๋ฌด๊ธฐ ๊ฐœ๋ฐœ์— ๋Œ€ํ•œ ๋ถํ•œ์˜ ์ž…์žฅ์„ ์˜นํ˜ธํ•˜๋Š” ์‚ฌ์ƒํ•™์Šต์„ ํ•˜์˜€๋‹ค. ์ด๋กœ์จ ํ”ผ๊ณ ์ธ์€ ๊ณต์†Œ์™ธ 4์™€ ๊ณต๋ชจํ•˜์—ฌ, ๊ตญ๊ฐ€์˜ ์กด๋ฆฝยท์•ˆ์ „์ด๋‚˜ ์ž์œ ๋ฏผ์ฃผ์  ๊ธฐ๋ณธ์งˆ์„œ๋ฅผ ์œ„ํƒœ๋กญ๊ฒŒ ํ•œ๋‹ค๋Š” ์ •์„ ์•Œ๋ฉด์„œ ๋ฐ˜๊ตญ๊ฐ€๋‹จ์ฒด๋‚˜ ๊ทธ ๊ตฌ์„ฑ์› ๋˜๋Š” ๊ทธ ์ง€๋ น์„ ๋ฐ›์€ ์ž์˜ ํ™œ๋™์„ ์ฐฌ์–‘ยท๊ณ ๋ฌดยท์„ ์ „ ๋˜๋Š” ์ด์— ๋™์กฐํ•˜์˜€๋‹ค. ๋งˆ. 2014. 4. 28. ๊ณต์†Œ์™ธ 4์™€ ์‚ฌ์ƒํ•™์Šต(์ด์ ๋™์กฐ) ํ”ผ๊ณ ์ธ์€ 2014. 4. 28. 17:18๊ฒฝ๋ถ€ํ„ฐ ๊ฐ™์€ ๋‚  21:10๊ฒฝ๊นŒ์ง€ ์„œ์šธ ์ข…๋กœ๊ตฌ์— ์žˆ๋Š” (๋ช…์นญ 53 ์ƒ๋žต)ํšŒ๊ด€ ๋‚ด (๋ช…์นญ 29 ์ƒ๋žต)์—ฐ๊ตฌ์†Œ์—์„œ ๊ณต์†Œ์™ธ 4์™€ ํ•จ๊ป˜ โ€˜์šฐ๋ฆฌ์˜ ์กด์—„๊ณผ ์ฒด์ œ, ๋ณ‘์ง„๋กœ์„ ์— ๊ฐํžˆ ๋„์ „ํ•˜๋Š” ์ž๋“ค์„ ์ ˆ๋Œ€๋กœ ์šฉ๋‚ฉ์ง€ ์•Š์„ ๊ฒƒ์ด๋‹คโ€™๋ผ๋Š” ์ œ๋ชฉ์˜ 2014. 4. 27.์ž ๋ถํ•œ ์กฐ๊ตญํ‰ํ™”ํ†ต์ผ์œ„์›ํšŒ๊ฐ€ ๋ฐœํ‘œํ•œ ์„ฑ๋ช…์„œ๋ฅผ ์ฝ์–ด๋ณด์•˜๋Š”๋ฐ, ๊ทธ ์ฃผ์š”๋‚ด์šฉ์€ ๋‹ค์Œ๊ณผ ๊ฐ™๋‹ค. ใ€Žใ€Šโ€˜์šฐ๋ฆฌ์˜ ์กด์—„๊ณผ ์ฒด์ œ, ๋ณ‘์ง„๋กœ์„ ์— ๊ฐํžˆ ๋„์ „ํ•˜๋Š” ์ž๋“ค์„ ์ ˆ๋Œ€๋กœ ์šฉ๋‚ฉ์ง€ ์•Š์„ ๊ฒƒ์ด๋‹คโ€™ ์ œํ•˜์˜ ์„ฑ๋ช…์„œ ์ฃผ์š”๋‚ด์šฉใ€‹ - ์ž๊ธฐ์˜ ์‹ ๋ขฐํ”„๋กœ์„ธ์Šค๋ผ๋Š” ๊ฒƒ์ด ์ด๋ช…๋ฐ• ์—ญ๋„์˜ ๊ฒƒ๊ณผ ๊ฐ™์€ ๋Œ€๊ฒฐ์ •์ฑ…์ด๋ผ๋Š” ๊ฒƒ์„ ์ ๋‚˜๋ผํ•˜๊ฒŒ ๋‚ด๋ณด์˜€์œผ๋ฉฐ ๋ถ๋‚จ๊ด€๊ณ„๊ฐœ์„ ์€ ๊ผฌ๋ฌผ๋งŒ์น˜๋„ ์•ˆ์ค‘์— ์—†๊ณ  ์šฐ๋ฆฌ์™€ ๋๊นŒ์ง€ ๋Œ€๊ฒฐํ•˜๋ฉด์„œ ์ •์„ธ๋ฅผ ํŒŒ๊ตญ์—๋กœ ๋ชฐ์•„๊ฐ€๊ฒ ๋‹ค๋Š” ๊ฒƒ์„ ์˜จ ์„ธ์ƒ์— ์„ ํฌํ•˜์˜€๋‹ค. - ๊ทธ๋Š” ๋ฏธ๊ตญ ํ•ต์— ๋Œ€ํ•ด์„œ๋Š” ํ•œ๋งˆ๋””๋„ ๋ชปํ•˜๊ณ  ๋™์กฑ์˜ ํ•ต์€ ํ”ผ๋ˆˆ์ด ๋˜์–ด ๊ฑธ๊ณ  ๋“ค๋ฉด์„œ ์ƒ์ „์˜ ์นจ๋žต์  ์š”๊ตฌ์— ๋ชจ๋“  ๊ฒƒ์„ ์„ฌ๊ฒจ ๋ฐ”์นจ์œผ๋กœ์จ ๊ทน์•…ํ•œ ์‚ฌ๋Œ€๋งค๊ตญ๋…ธ, ์ถ”์•…ํ•œ ๋ฏธ๊ตญ์œ„์•ˆ๋ถ€, ๋”๋Ÿฌ์šด ๋ฏผ์กฑ๋ฐ˜์—ญ๋งค์Œ๋ถ€๋กœ์„œ์˜ ๋ชฐ๊ณจ์„ ์—ฌ์ง€์—†์ด ๋“œ๋Ÿฌ๋ƒˆ๋‹ค. - ์ด๋ฒˆ ์˜ค๋ฐ”๋งˆ์˜ ๋‚จ์กฐ์„ ํ–‰๊ฐ ๊ฒฐ๊ณผ๋Š” ๋ฐฑ๋…„์ˆ™์  ๋ฏธ๊ตญ๊ณผ๋Š” ๋ง์ด ์•„๋‹ˆ๋ผ ์˜ค์ง ํž˜์œผ๋กœ๋งŒ ๋งž์„œ์•ผ ํ•˜๋ฉฐ ์ „๋ฉด ํ•ต๋Œ€๊ฒฐ์ „์— ์˜ํ•œ ์ตœํ›„์˜ ๊ฒฐ์‚ฐ ๋ฐ–์— ์—†๋‹ค๋Š” ์šฐ๋ฆฌ์˜ ํŒ๋‹จ๊ณผ ๊ฐ์˜ค๊ฐ€ ๋ฐฑ๋ฒˆ ์˜ณ์•˜์œผ๋ฉฐ ์šฐ๋ฆฌ๊ฐ€ ์„ ํƒํ•˜๊ณ  ์ฒœ๋ช…ํ•œ ๊ธธ๋กœ ๊ณ„์† ๋‚˜๊ฐ€์•ผ ํ•˜๊ฒ ๋‹ค๋Š” ์˜์ง€์™€ ๊ฒฐ์‹ฌ์„ ๋”์šฑ ํ™•๊ณ ํžˆ ํ•ด์ค„ ๋ฟ์ด๋‹ค. - ๋ฐ•๊ทผํ˜œ๋Š” ์ƒ์ „์ธ ์˜ค๋ฐ”๋งˆ ๋ณด๋‹ค๋„ ์šฐ๋ฆฌ๋ฅผ ๋” ์•…๋ž„ํ•˜๊ฒŒ ํ—๋œฏ์Œ์œผ๋กœ์จ ์—ญ๋Œ€ ๊ดด๋ขฐํ†ต์น˜๋ฐฐ๋ฅผ ๋Šฅ๊ฐ€ํ•˜๋Š” ๋™์กฑ๋Œ€๊ฒฐ๊ด‘์‹ ์ž, ํŠน๋“ฑ์นœ๋ฏธ์ฃผ๊ตฌ๋ผ๋Š” ๊ฒƒ์„ ๋งŒ์ฒœํ•˜์— ๊ณผ์‹œํ•˜์˜€๋‹ค. - ๋ฐ•๊ทผํ˜œ๋Š” ์ด๋ฒˆ์— ์šฐ๋ฆฌ๊ฐ€ ์ค€ ๊ธฐํšŒ๋ฅผ ์Šค์Šค๋กœ ์ €๋ฒ„๋ฆฌ๊ณ  ๋ฐ˜ํ†ต์ผ์˜ ๊ธธ, ๋ฐ˜ ํ‰ํ™”์˜ ๊ธธ, ๋Œ€๊ฒฐ๊ณผ ์ „์Ÿ์˜ ๊ธธ์„ ํƒํ•œ ๊ฒƒ์œผ๋กœ ํ•˜์—ฌ ๊ทธ ๋Œ€๊ฐ€๋ฅผ ๋‹จ๋‹จํžˆ ์น˜๋ฅด๊ฒŒ ๋  ๊ฒƒ์ด๋‹ค.ใ€ ํ”ผ๊ณ ์ธ์€ ๊ณต์†Œ์™ธ 4์™€ ํ•จ๊ป˜ ์œ„ ์„ฑ๋ช…์„œ๋ฅผ ์ฝ๊ณ  ์œ„ ์„ฑ๋ช…์„œ์— ๊ด€ํ•ด ํ† ๋ก ํ•˜์˜€๋Š”๋ฐ, ๊ทธ ์ฃผ์š” ๋Œ€ํ™”๋‚ด์šฉ์€ ๋‹ค์Œ๊ณผ ๊ฐ™๋‹ค. ใ€Žใ€Š2014. 4. 28. ํ”ผ๊ณ ์ธ๊ณผ ๊ณต์†Œ์™ธ 4 ๊ฐ„์˜ ์ฃผ์š” ๋Œ€ํ™”๋‚ด์šฉใ€‹ (ํ”ผ๊ณ ์ธ) : ๋”ด ๊ฒŒ ์•„๋‹ˆ๋ผ ์˜ค๋Š˜ ์–ด์ €๊ป˜ ์กฐํ‰ํ†ต์—์„œ, ๋ฐ•๊ทผํ˜œ ๊ด€๋ จํ•ด์„œ ๋‚จ๊ธด ๊ธ€ ์žˆ์ž–์•„์š”, ๊ด€๋ จํ•ด ํ‘œํ˜„์ด ๋” ๊ต‰์žฅํžˆ --- ์ด๊ฑด ์ „๋ฌธ์ด์ง€ ์ •์น˜๋ฉด์—์„œ (ํ”ผ๊ณ ์ธ) : ์ •์ƒํšŒ๋‹ด๊ณผ ๊ณต๋™๊ธฐ์žํšŒ๊ฒฌ์ด๋ผ๋Š” ๋ฐ์„œ ๋Š๋‹ท์—†์ด ๋…„์€ ์˜ค๋ฐ”๋งˆ์™€ ์–ต์ง€๋กœ ์ž… ๋งž์ถ”๊ณ  --- ๋ณ‘์ง„๋กœ์„ ์€ ์‹คํŒจํ•  ์ˆ˜๋ฐ–์— ์—†์„ ๊ฒƒ์ด๋ผ๋Š๋‹ˆ, 4์ฐจ ํ•ต ์‹œํ—˜์„ ํ•˜๋ฉด 6์žํšŒ๋‹ด์ด ๋ฌผ๊ฑฐํ’ˆ ๋  ๊ฒƒ์ด๋ผ๋Š๋‹ˆ, ๋ถ์˜ ์ƒˆ๋กœ์šด ํ˜•ํƒœ์˜ ๋„๋ฐœ์€ ์ƒˆ๋กœ์šด ๊ฐ•๋„์˜ ๊ตญ์ œ์  ์••๋ฐ•์„ ๊ฐ€์ ธ์˜ฌ ๊ฒƒ์ด๋ผ๋Š๋‹ˆ ํ•˜๊ณ  ์ž…์— ๊ฐœ๊ฑฐํ’ˆ์„ ๋ฌผ๊ณ  ์ง€๊ป„์—ฌ๋Œ”๋‹ค. ์ž๊ธฐ์˜ ์‹ ๋ขฐํ”„๋กœ์„ธ์Šค๋ผ๋Š” ๊ฒƒ์ด ์ด๋ช…๋ฐ• ์—ญ๋„์˜ ๊ฒƒ๊ณผ ๊ฐ™์€ ๋Œ€๊ฒฐ์ •์ฑ…์ด๋ผ๋Š” ๊ฒƒ์„ ์ ๋‚˜๋ผํ•˜๊ฒŒ ๋‚ด๋ณด์˜€์œผ๋ฉฐ ๋ถ๋‚จ๊ด€๊ณ„๊ฐœ์„ ์€ ๊ผฌ๋ฌผ๋งŒ์น˜๋„ ์•ˆ์ค‘์— ์—†๊ณ  ์šฐ๋ฆฌ์™€ ๋๊นŒ์ง€ ๋Œ€๊ฒฐํ•˜๋ฉด์„œ ์ •์„ธ๋ฅผ ํŒŒ๊ตญ์—๋กœ ๋ชฐ์•„๊ฐ€๊ฒ ๋‹ค๋Š” ๊ฒƒ์„ ์˜จ ์„ธ์ƒ์— ์„ ํฌํ•˜์˜€๋‹ค. ๊ทธ๋Š” ๋ฏธ๊ตญ ํ•ต์— ๋Œ€ํ•ด์„œ๋Š” ํ•œ๋งˆ๋””๋„ ๋ชปํ•˜๊ณ  ๋™์กฑ์˜ ํ•ต์€ ํ”ผ๋ˆˆ์ด ๋˜์—ฌ ๊ฑธ๊ณ  ๋“ค๋ฉด์„œ ์ƒ์ „์˜ ์นจ๋žต์  ์š”๊ตฌ์— ๋ชจ๋“  ๊ฒƒ์„ ์„ฌ๊ฒจ ๋ฐ”์นจ์œผ๋กœ์จ ๊ทน์•…ํ•œ ์‚ฌ๋Œ€๋งค๊ตญ๋…ธ, ์ถ”์•…ํ•œ ๋ฏธ๊ตญ์œ„์•ˆ๋ถ€, ๋”๋Ÿฌ์šด ๋ฏผ์กฑ๋ฐ˜์—ญ๋งค์Œ๋ถ€๋กœ์„œ์˜ ๋ชฐ๊ณจ์„ ์—ฌ์ง€์—†์ด ๋“œ๋Ÿฌ๋ƒˆ๋‹ค. ์˜ค๋ฐ”๋งˆ๋Š” ์˜จ ๋‚จ์กฐ์„  ๋•…์ด ์—ฌ๊ฐ์„  ์นจ๋ชฐ์‚ฌ๊ณ ๋กœ ๋Œ€๋‚œ๋ฆฌ๊ฐ€ ๋‚œ ์ดˆ์ƒ์ง‘์— ๊ธฐ์—ฌ ๋“ค์–ด ๋ฏผ์‹ฌ์˜ ์˜ˆ๋ฆฌํ•œ ๋ˆˆ์ดˆ๋ฆฌ ์•ž์— ๋งˆ์Œ๋Œ€๋กœ ์›ƒ์ง€๋„ ๋จน์ง€๋„ ๋ชปํ•˜๊ณ  ์ˆจ์–ด ๋‹ค๋‹ˆ๋‹ค์‹œํ”ผ ํ•˜๋ฉด์„œ ์œ ์‹ ๋…์žฌ์ž์˜ ๋”ธ์ด ๋ฏธ๊ตญ์˜ ์ „์Ÿ์ˆ˜์ฒญ์„ ๋” ์ž˜ ๋“ค๋„๋ก ํ—ˆ์šฐ๋Œ€์— ์–ด์šธ๋ฆฌ์ง€ ์•Š๊ฒŒ ๋…„์˜ ๋“ฑ์„ ๋‘๋“œ๋ ค์ฃผ๋ฉฐ ์ฐฝํ”ผํ•˜๊ฒŒ ๋†€์•˜๋‹ค. ์ด๋ฒˆ ์˜ค๋ฐ”๋งˆ์˜ ๋‚จ์กฐ์„ ํ–‰๊ฐ ๊ฒฐ๊ณผ๋Š” ๋ฐฑ๋…„์ˆ™์  ๋ฏธ๊ตญ๊ณผ๋Š” ๋ง์ด ์•„๋‹ˆ๋ผ ์˜ค์ง ํž˜์œผ๋กœ๋งŒ ๋งž์„œ์•ผ ํ•˜๋ฉฐ ์ „๋ฉด ํ•ต๋Œ€๊ฒฐ์ „์— ์˜ํ•œ ์ตœํ›„์˜ ๊ฒฐ์‚ฐ๋ฐ–์— ์—†๋‹ค๋Š” ์šฐ๋ฆฌ์˜ ํŒ๋‹จ๊ณผ ๊ฐ์˜ค๊ฐ€ ๋ฐฑ๋ฒˆ ์˜ณ์•˜์œผ๋ฉฐ ์šฐ๋ฆฌ๊ฐ€ ์„ ํƒํ•˜๊ณ  ์ฒœ๋ช…ํ•œ ๊ธธ๋กœ ๊ณ„์† ๋‚˜๊ฐ€์•ผ ํ•˜๊ฒ ๋‹ค๋Š” ์˜์ง€์™€ ๊ฒฐ์‹ฌ์„ ๋”์šฑ ํ™•๊ณ ํžˆ ํ•ด์ค„ ๋ฟ์ด๋‹ค ๊ทธ๋Š” ์ƒ์ „์ธ ์˜ค๋ฐ”๋งˆ ๋ณด๋‹ค๋„ ์šฐ๋ฆฌ๋ฅผ ๋” ์•…๋ž„ํ•˜๊ฒŒ ํ—๋œฏ์Œ์œผ๋กœ์จ ์—ญ๋Œ€ ๊ดด๋ขฐํ†ต์น˜๋ฐฐ๋ฅผ ๋Šฅ๊ฐ€ํ•˜๋Š” ๋™์กฑ๋Œ€๊ฒฐ๊ด‘์‹ ์ž, ํŠน๋“ฑ ์นœ๋ฏธ์ฃผ๊ตฌ๋ผ๋Š” ๊ฒƒ์„ ๋งŒ์ฒœํ•˜์— ๊ณผ์‹œํ•˜์˜€๋‹ค. ๋ฐ•๊ทผํ˜œ๋Š” ์ด๋ฒˆ์— ์šฐ๋ฆฌ๊ฐ€ ์ค€ ๊ธฐํšŒ๋ฅผ ์Šค์Šค๋กœ ์ €๋ฒ„๋ฆฌ๊ณ  ๋ฐ˜ํ†ต์ผ์˜ ๊ธธ, ๋ฐ˜ ํ‰ํ™”์˜ ๊ธธ, ๋Œ€๊ฒฐ๊ณผ ์ „์Ÿ์˜ ๊ธธ์„ ํƒํ•œ ๊ฒƒ์œผ๋กœ ํ•˜์—ฌ ๊ทธ ๋Œ€๊ฐ€๋ฅผ ๋‹จ๋‹จํžˆ ์น˜๋ฅด๊ฒŒ ๋  ๊ฒƒ์ด๋‹ค ์šฐ๋ฆฌ์˜ ์กด์—„๊ณผ ์ฒด์ œ๋ฅผ ํ•จ๋ถ€๋กœ ๊ฑด๋“œ๋ฆฐ๋ฐ ์ด๋Ÿฐ ๋ถ์ธก์˜ --- ์กฐ๊ธˆ, ๋Œ€๋ณ€์ธ ์„ฑ๋ช…, ๊ทธ๋•Œ๋Š” ๋Œ€๋ถ ์ง€๋‚œ 4์›” 23์ผ๋‚  ๊ณต๊ฐœ ์งˆ๋ฌธ์žฅ ์คฌ์ž–์•„, ์ฃผ๋ฉด์„œ 10๊ฐ€์ง€, 10๊ฐ€์ง€ ์งˆ๋ฌธ์—์„œ ํ†ต์ผ์„ ๋ฐ”๋ผ๋Š” ๊ธฐ๋ฏธ๊ฐ€ ์—†์œผ๋‹ˆ๊นŒ ์ฒด์ œ ๋ณ€๊ฒฝ์ด๋‚˜ ์ฒด์ œ๋ณ€๊ฒฝ ํ•˜๋Š”๋ฐ ๊ทธ๊ฒƒ์„ ํ•˜๊ณ  (ํ”ผ๊ณ ์ธ) : ์šฐ๋ฆฌ๊ฐ€ ๋ฏธ๊ตญ์ด ๊ทธ๋™์•ˆ ๋ถ์„ ์ž๊ทนํ•˜๋ฉด์€ ์ด์ œ ๊ตฐ์‚ฌํ›ˆ๋ จ์„ ํ†ตํ•ด์„œ ํ‰์–‘์„ ์ ๋ นํ•˜๊ฒ ๋‹ค๋Š”, ๊ทธ๋ฆฌ๊ณ  ํŠน๋ณ„ํžˆ ์ผˆ๋ฆฌ๊ฐ€, ์ผ€๋ผ, ์ผˆ๋ฆฌ ๊ตญ๋ฌด์žฅ๊ด€์ด ๋ถ์„ ํžˆํ‹€๋Ÿฌ์— ๋‚˜์น˜์— ๋น„์œ ํ•˜๋Š”๊ฑฐ ์•„๋ƒ (๊ณต์†Œ์™ธ 4) : ์š”๊ฑฐ ํ•œ๋ฒˆ ์ฝ์–ด ๋ณด๊ณ  --- ํ•ด์ฃผ์‹œ์ฃ  (ํ”ผ๊ณ ์ธ) : ์š”์•ฝ์„ ํ•ด์•ผ์ง€, ๋ณธ๋ก  ๋งˆ์น˜๊ณ  ์–ด๋–ป๊ฒŒ ํ•ด ์š”์•ฝ์„ ํ•ด์•ผ์ง€ (๊ณต์†Œ์™ธ 4) : ๋ฏธ๊ตญ์— ์žˆ์–ด์„œ์˜ 3๊ฐ€์ง€ ๊ตฌ์ฒด์ ์œผ๋กœ ๋ณด๋ฉด ์ ๋Œ€์  ์ •์ฑ…! ์—ฌ๊ธฐ์— ์žˆ์ž–์•„์š”. ๋Œ€๋ถ์ „์Ÿ์—ฐ์Šต ํŠนํžˆ, ๋Œ€๋ถ์ธ๊ถŒ๋ฌธ์ œ ๊ทธ๋ฆฌ๊ณ  ๋ฏธ๊ตญ ๊ฒฝ์ œ ์œ ์น˜ ์ด 3๊ฐ€์ง€๋ฅผ ํ–ˆ๋Š”๋ฐ ๋ฏธ๊ตญ์€ ์ด 3๊ฐ€์ง€๋ฅผ ์ „ํ˜€ ๋ณด์™„ํ•ด์•ผํ•  ๊ทธ๋ ‡์ง€ ์•Š๊ฑฐ๋‚˜ ๋ณด์™„ํ•ด์•ผ ๋  ์ž…์žฅ์ด ์•„๋‹ˆ๋‹ˆ ๋ถํ•œ์—์„œ๋Š”, ๋ถํ•œ์—์„œ๋Š” ๊ทธ๊ฑฐ๋ฅผ ๋ถ์ชฝ์—์„œ ๋ถ์ชฝ์—์„œ๋Š” ๊ทธ๊ฑฐ์— ๋Œ€ํ•œ ๋Œ€๋น„ ๋ณดํ˜ธ๋ฌธ์ œ๋ฅผ ์•ˆ์ทจํ•˜๋Š” ๊ฑธ๋กœ ์šฐ๋ฆฌ๋Š” ์ธ์ •ํ•˜๋Š” ๊ฑฐ๋‹ค ๊ทธ๋Ÿฌ๋‹ˆ๊น ๋ณดํ˜ธ๋ฌธ์ œ ํ•œ ๊ฑฐ๋‹ค. ๊ทผ๋ฐ ์šฐ๋ฆฌ๊ฐ€ โ—‹โ—‹ํ• ๊ฑฐ๋ƒ ๋ผ๊ณ  ํ•˜๋Š” โ—‹โ—‹์†์—์„œ ์€ํ•˜3ํ˜ธ ๋ณด๋‹ค ๋” ํฐ ์šฐ๋ฆฌ๋„ ์žˆ๋‹ค. ์ด์ œ ์šด๋ฐ˜๋ถ€ํ„ฐ ๋‚ ์”จ๋„ ์ข‹์€๋ฐ ๋ฉ€๋ฆฌ ๊ฐˆ ์ˆ˜ ์žˆ๋‚˜ ๋ผ๊ณ  ํ•˜๋Š” ๋จผ ๋ฐœ์น˜๋กœ, ๊ทธ๋ฆฌ๊ณ  ์ด๋ฒˆ ํ•ต์‹คํ—˜ ๊ฐ™์€ ๊ฒฝ์šฐ๋Š” 2๋‹จ๊ณ„๋กœ ํ•ต์‹คํ—˜์ธ๋ฐ, ๊ณ„๊ธฐ๊ฐ€ ์žˆ๋‹ค๋ผ๊ณ  ํ•˜๋Š” ๊ฒƒ์„ ๊ทœ์ •๋Œ€๋กœ ํ•˜๋Š”, ๋˜ ๋™์‹œ, ๋™์‹œ์— ํ•ต์‹คํ—˜ ํ•  ๊ฒƒ์ด๋‹ค.ใ€ ์ด์™€ ๊ฐ™์ด ํ”ผ๊ณ ์ธ์€ ๊ณต์†Œ์™ธ 4์™€ ํ•จ๊ป˜ ๋Œ€ํ†ต๋ น์„ ์›์ƒ‰์ ์œผ๋กœ ๋ชจ์š•ํ•˜๊ณ  ๋น„๋ฐฉํ•˜๋Š” ๋‚ด์šฉ์ด ๋‹ด๊ธด ์กฐ๊ตญํ‰ํ™”ํ†ต์ผ์œ„์›ํšŒ์˜ ์„ฑ๋ช…์„œ๋ฅผ ์ฝ์–ด๊ฐ€๋ฉด์„œ ๊ทธ ์˜๋ฏธ๋ฅผ ์„ค๋ช…ํ•˜๊ณ , ์ •๋ถ€์˜ ๋Œ€๋ถ์ •์ฑ…์„ ๋น„๋‚œํ•˜๋ฉด์„œ ๋ถํ•œ์˜ ํ•ต๋ณด์œ , ํ•ต๊ฐœ๋ฐœ์„ ์˜นํ˜ธํ•˜๋Š” ๋“ฑ ์‚ฌ์ƒํ•™์Šต์„ ํ•˜์˜€๋‹ค. ์ด๋กœ์จ ํ”ผ๊ณ ์ธ์€ ๊ณต์†Œ์™ธ 4์™€ ๊ณต๋ชจํ•˜์—ฌ, ๊ตญ๊ฐ€์˜ ์กด๋ฆฝยท์•ˆ์ „์ด๋‚˜ ์ž์œ ๋ฏผ์ฃผ์  ๊ธฐ๋ณธ์งˆ์„œ๋ฅผ ์œ„ํƒœ๋กญ๊ฒŒ ํ•œ๋‹ค๋Š” ์ •์„ ์•Œ๋ฉด์„œ ๋ฐ˜๊ตญ๊ฐ€๋‹จ์ฒด๋‚˜ ๊ทธ ๊ตฌ์„ฑ์› ๋˜๋Š” ๊ทธ ์ง€๋ น์„ ๋ฐ›์€ ์ž์˜ ํ™œ๋™์„ ์ฐฌ์–‘ยท๊ณ ๋ฌดยท์„ ์ „ ๋˜๋Š” ์ด์— ๋™์กฐํ•˜์˜€๋‹ค. ๋ฐ”. 2014. 10. 5. ๊ณต์†Œ์™ธ 6๊ณผ ์‚ฌ์ƒํ•™์Šต(์ด์ ํ‘œํ˜„๋ฌผ ์†Œ์ง€) ํ”ผ๊ณ ์ธ์€ 2014. 10. 5. 19:10๊ฒฝ๋ถ€ํ„ฐ ๊ฐ™์€ ๋‚  21:10๊ฒฝ๊นŒ์ง€ ์„œ์šธ ์†กํŒŒ๊ตฌ์— ์žˆ๋Š” ์ž ์‹ค์—ญ ์ธ๊ทผ (๋ช…์นญ 49 ์ƒ๋žต) ํ˜ธํ”„์ง‘์—์„œ ๊ณต์†Œ์™ธ 6๊ณผ ๋งŒ๋‚˜ ํ”ผ๊ณ ์ธ์˜ ํƒœ๋ธ”๋ฆฟ PC์— ์ €์žฅ๋˜์–ด ์žˆ๋˜ โ€˜๋ฐฑ๋‘๋ น์žฅ๋”ฐ๋ผ ์ตœํ›„์Šน๋ฆฌ์˜ ์ถ•ํฌ์„ฑ ํ„ฐ์ณ์˜ฌ๋ฆฌ๋ฆฌ๋ผโ€™๋ผ๋Š” ์ œ๋ชฉ์˜ 2014. 10. 3.์ž ๋…ธ๋™์‹ ๋ฌธ ๊ธฐ์‚ฌ๋ฅผ ์—ด๋žŒ์‹œ์ผฐ๋Š”๋ฐ, ๊ทธ ์ฃผ์š”๋‚ด์šฉ์€ ๋‹ค์Œ๊ณผ ๊ฐ™๋‹ค. ใ€Žใ€Š๋ฐฑ๋‘๋ น์žฅ๋”ฐ๋ผ ์ตœํ›„์Šน๋ฆฌ์˜ ์ถ•ํฌ์„ฑ ํ„ฐ์ณ์˜ฌ๋ฆฌ๋ฆฌ๋ผใ€‹ ์ œํ•˜์˜ ๊ธฐ์‚ฌ ์ฃผ์š”๋‚ด์šฉใ€‹ - ๋‹น์ฐฝ๊ฑด๊ธฐ๋…์ผ์ด ์•„๋กœ์ƒˆ๊ฒจ์ง„ 10์›”์˜ ํ•˜๋Š˜๊ฐ€์— ์˜จ ๋‚˜๋ผ๋ฅผ ๋“๊ฒŒ ํ•˜๋Š” ์Šน๋ฆฌ์˜ ๋ขฐ์„ฑ์ด ํ„ฐ์ ธ์˜ฌ๋ž๋‹ค. - ๊ฒฝ์• ํ•˜๋Š” ์›์ˆ˜๋‹˜๊ป˜์„œ ์•ˆ๊ฒจ์ฃผ์‹  ๋ฌด๋น„์˜ ๋‹ด๋ ฅ๊ณผ ๋ฐฐ์งฑ์„ ์•ˆ๊ณ  ๊ฒฐ์Šน๊ฒฝ๊ธฐ์— ๋‚˜์„  ์šฐ๋ฆฌ์˜ ๋ฏธ๋”์šด ๋…€์ž์ถ•๊ตฌ์„ ์ˆ˜๋“ค์€ ๊ฒฝ๊ธฐ์†Œ์‹์œผ๋กœ ์ž  ๋ชป ๋“ค๋˜ ์šฐ๋ฆฌ ์ธ๋ฏผ์—๊ฒŒ ์Šน๋ฆฌ์˜ ํ™˜ํฌ์™€ ๊ฒฉ์ •์„ ์•ˆ๊ฒจ์ฃผ์—ˆ๋‹ค. - ์˜ค๋Š˜์˜ ์Šน๋ฆฌ๋Š” ์„ธ๊ธฐ๋ฅผ ์ด์–ด ๋ฐฑ์Šน๋งŒ์„ ๋–จ์ณ์˜จ ์šฐ๋ฆฌ ์กฐ๊ตญ์˜ ์Šน๋ฆฌ์˜ ์ „ํ†ต๊ณผ ์˜ค์ง ์Šน๋ฆฌ๋งŒ์„ ์œ„ํ•ด ์‹ธ์šฐ๋Š” ์ฒœ๋งŒ๊ตฐ๋ฏผ์˜ ์‹ ๋…๊ณผ ์˜์ง€์— ๋Œ€ํ•œ ๋‚จ๊น€ ์—†๋Š” ์‹œ์œ„์ด๋‹ค. ์Šน๋ฆฌ์˜ ์†Œ์‹์œผ๋กœ ์ด์•ผ๊ธฐ๊ฝƒ์„ ํ”ผ์šฐ๋Š” ์ดˆ์†Œ์™€ ์ผํ„ฐ๋งˆ๋‹ค์— ๋…ธ๋ž˜ใ€Š์Šน๋ฆฌ๋Š” ๋Œ€๋ฅผ ์ด์–ดใ€‹๊ฐ€ ์šธ๋ ค ํผ์ง€๊ณ  ์žˆ๋‹ค. - ๊ฒฝ์• ํ•˜๋Š” ๊น€์ •์€ ๋™์ง€๊ป˜์„œ๋Š” ๋‹ค์Œ๊ณผ ๊ฐ™์ด ๋ง์”€ํ•˜์‹œ์˜€๋‹ค. โ€œ์œ„๋Œ€ํ•œ ๊น€์ผ์„ฑ๋™์ง€์™€ ๊น€์ •์ผ ๋™์ง€์˜ ๋ถˆ๋ฉธ์˜ ํƒœ์–‘๊ธฐ๋ฅผ ๋†’์ด ํœ˜๋‚ ๋ฆฌ๋ฉฐ ๋‚˜์•„๊ฐ€๋Š” ์šฐ๋ฆฌ์˜ ์•ž๊ธธ์—๋Š” ์˜ค์ง ์Šน๋ฆฌ์™€ ์˜๊ด‘๋งŒ์ด ์žˆ์„ ๊ฒƒ์ž…๋‹ˆ๋‹ค.โ€ใ€ ์ด์™€ ๊ฐ™์ด ์œ„ ๊ธฐ์‚ฌ๋Š” 2014 ์ธ์ฒœ ์•„์‹œ์•ˆ๊ฒŒ์ž„ ์—ฌ์ž์ถ•๊ตฌํŒ€ ์šฐ์Šน์œผ๋กœ ๋ถํ•œ ์ „์—ญ์— ๊น€์ •์€์„ ์ฐฌ์–‘ํ•˜๋Š” ๋…ธ๋ž˜๊ฐ€ ์šธ๋ ค ํผ์ง„๋‹ค๋Š” ๋‚ด์šฉ์œผ๋กœ ๊น€์ •์€์˜ ์ง€๋„๋ ฅ์„ ์ฐฌ์–‘ํ•˜๊ณ  ์„ ์ „ํ•˜๋Š” ๋‚ด์šฉ์ด๋‹ค. ์ด๋กœ์จ ํ”ผ๊ณ ์ธ์€ ๊ตญ๊ฐ€์˜ ์กด๋ฆฝยท์•ˆ์ „์ด๋‚˜ ์ž์œ ๋ฏผ์ฃผ์  ๊ธฐ๋ณธ์งˆ์„œ๋ฅผ ์œ„ํƒœ๋กญ๊ฒŒ ํ•œ๋‹ค๋Š” ์ •์„ ์•Œ๋ฉด์„œ ๋ฐ˜๊ตญ๊ฐ€๋‹จ์ฒด๋‚˜ ๊ทธ ๊ตฌ์„ฑ์› ๋˜๋Š” ๊ทธ ์ง€๋ น์„ ๋ฐ›์€ ์ž์˜ ํ™œ๋™์„ ์ฐฌ์–‘ยท๊ณ ๋ฌดยท์„ ์ „ ๋˜๋Š” ์ด์— ๋™์กฐํ•  ๋ชฉ์ ์œผ๋กœ ์ด์ ํ‘œํ˜„๋ฌผ์„ ์†Œ์ง€ํ•˜์˜€๋‹ค. ์‚ฌ. ๊ธฐํƒ€ ์ด์ ํ‘œํ˜„๋ฌผ ์†Œ์ง€ 1) 2010. 7. 21. ํ”ผ๊ณ ์ธ ๋ช…์˜๋กœ ๊ฐœ์„คํ•œ โ€˜โ—โ—โ—โ—โ—โ€™์— ์ด์ ํ‘œํ˜„๋ฌผ ์†Œ์ง€ ํ”ผ๊ณ ์ธ์€ 2010. 7. 21.๊ฒฝ ํ”ผ๊ณ ์ธ ๋ช…์˜๋กœ ๊ฐœ์„คํ•œ โ€˜โ—โ—โ—โ—โ—โ€™[ID โ€˜(์˜๋ฌธ ID 7 ์ƒ๋žต)โ€™]์— โ€˜์กฐ์„ ๋ฏผ์ฃผ์ฃผ์˜์ธ๋ฏผ๊ณตํ™”๊ตญ์€ ๋ถˆํŒจ์˜ ์œ„๋ ฅ์„ ์ง€๋‹Œ ์ฃผ์ฒด์˜ ์‚ฌํšŒ์ฃผ์˜๊ตญ๊ฐ€์ด๋‹คโ€™๋ผ๋Š” ์ œ๋ชฉ์˜ ๋ฌธ๊ฑด์„ ํŒŒ์ผ ํ˜•ํƒœ๋กœ ๋ณด๊ด€ํ•˜๊ณ  ์žˆ์—ˆ๋Š”๋ฐ, ๊ทธ ์ฃผ์š”๋‚ด์šฉ์€ ๋‹ค์Œ๊ณผ ๊ฐ™๋‹ค. ใ€Žใ€Šโ€˜์กฐ์„ ๋ฏผ์ฃผ์ฃผ์˜์ธ๋ฏผ๊ณตํ™”๊ตญ์€ ๋ถˆํŒจ์˜ ์œ„๋ ฅ์„ ์ง€๋‹Œ ์ฃผ์ฒด์˜ ์‚ฌํšŒ์ฃผ์˜๊ตญ๊ฐ€์ด๋‹คโ€™ ์ œํ•˜์˜ ๋ฌธ๊ฑด ์ฃผ์š”๋‚ด์šฉใ€‹ - ์œ„๋Œ€ํ•œ ์ˆ˜๋ น ๊น€์ผ์„ฑ๋™์ง€๋Š” ์šฐ๋ฆฌ ๊ณตํ™”๊ตญ์˜ ์ฐฝ๊ฑด์ž์ด์‹œ๋ฉฐ ์‚ฌํšŒ์ฃผ์˜์กฐ์„ ์˜ ์‹œ์กฐ์ด์‹œ๊ณ  ์˜์›ํ•œ ๊ตญ๊ฐ€์ฃผ์„์ด์‹ญ๋‹ˆ๋‹ค. ์œ„๋Œ€ํ•œ ์ˆ˜๋ น๋‹˜์˜ ๋ น๋„ ๋ฐ‘์— ๊ณตํ™”๊ตญ์ด ์ฐฝ๊ฑด๋จ์œผ๋กœ์จ ์šฐ๋ฆฌ ์กฐ๊ตญ๊ณผ ์ธ๋ฏผ์˜ ์šด๋ช…๊ฐœ์ฒ™์—์„œ ๊ทผ๋ณธ์ ์ธ ์ „ํ™˜์„ ๊ฐ€์ ธ์˜ค๊ฒŒ ๋˜์˜€์Šต๋‹ˆ๋‹ค. - ์œ„๋Œ€ํ•œ ์ˆ˜๋ น๋‹˜๊ป˜์„œ ์กฐ๊ตญ๊ณผ ์ธ๋ฏผ์„ ์œ„ํ•˜์—ฌ ์Œ“์•„์˜ฌ๋ฆฌ์‹  ๋ถˆ๋ฉธ์˜ ์—…์ ์€ ์šฐ๋ฆฌ ์กฐ๊ตญ๊ณผ ํ˜๋ช…์˜ ๋งŒ๋…„์ดˆ์„์ด๋ฉฐ ์‚ฌํšŒ์ฃผ์˜๊ฐ•์„ฑ๋Œ€๊ตญ๊ฑด์„ค๊ณผ ์ฃผ์ฒดํ˜๋ช…์œ„์—…์˜ ๊ณ„์Šน์™„์„ฑ์„ ์œ„ํ•œ ํ™•๊ณ ํ•œ ํ† ๋Œ€๋กœ ๋ฉ๋‹ˆ๋‹ค. - ์šฐ๋ฆฌ ๊ณตํ™”๊ตญ์€ ์œ„๋Œ€ํ•œ ์ˆ˜๋ น๋‹˜๊ป˜์„œ ์ฐฝ์‹œํ•˜์‹  ์ฃผ์ฒด์‚ฌ์ƒ์„ ์ง€๋„์‚ฌ์ƒ์œผ๋กœ ํ•˜๊ณ  ์šฐ๋ฆฌ ๋‹น์˜ ๋ น๋„ ๋ฐ‘์— ์ฃผ์ฒด์‚ฌ์ƒ์„ ๊ตญ๊ฐ€๊ฑด์„ค๊ณผ ๊ตญ๊ฐ€ํ™œ๋™์— ์ „๋ฉด์ ์œผ๋กœ ๊ตฌํ˜„ํ•ด๋‚˜๊ฐ€๊ธฐ์— ๋ถˆํŒจ์˜ ์œ„๋ ฅ์„ ์ง€๋‹Œ ์ฃผ์ฒด์˜ ์‚ฌํšŒ์ฃผ์˜๊ฐ•๊ตญ์œผ๋กœ ์ผ๋– ์„œ๊ฒŒ ๋˜์˜€์Šต๋‹ˆ๋‹ค. - ์˜์ƒ๋ถˆ๋ฉธ์˜ ์ฃผ์ฒด์‚ฌ์ƒ์€ ์‚ฌ๋žŒ์ค‘์‹ฌ์˜ ์„ธ๊ณ„๊ด€์ด๊ณ  ์ž์ฃผ์˜ ์‚ฌ์ƒ์ด๋ฉฐ ์ธ๋ฏผ๋Œ€์ค‘์˜ ์ž์ฃผ์„ฑ, ๋‚˜๋ผ์™€ ๋ฏผ์กฑ์˜ ์ž์ฃผ์„ฑ์„ ์˜นํ˜ธํ•˜๊ณ  ์‹คํ˜„ํ•˜๋Š” ๊ธธ์„ ๊ณผํ•™์ ์œผ๋กœ ๋ฐํ˜€์ฃผ๊ณ  ์žˆ๋Š” ์šฐ๋ฆฌ ์‹œ๋Œ€์˜ ์œ„๋Œ€ํ•œ ์ง€๋„์‚ฌ์ƒ์ž…๋‹ˆ๋‹ค. - ์ฃผ์ฒด์‚ฌ์ƒ์˜ ๊ธฐ์น˜ ๋ฐ‘์— ๊ฐ•ํ™”๋ฐœ์ „ ๋œ ์šฐ๋ฆฌ ๊ณตํ™”๊ตญ์€ ๊ณ„๊ธ‰์ ์›์น™, ํ˜๋ช…์ ์›์น™์„ ์ฒ ์ €ํžˆ ๊ตฌํ˜„ํ•œ ์ธ๋ฏผ๋Œ€์ค‘ ์ค‘์‹ฌ์˜ ์‚ฌํšŒ์ฃผ์˜๋‚˜๋ผ์ธ ๋™์‹œ์— ์ฃผ์ฒด์„ฑ๊ณผ ๋ฏผ์กฑ์„ฑ์ด ๊ฐ•ํ•œ ์ž์ฃผ, ์ž๋ฆฝ, ์ž์œ„์˜ ์‚ฌํšŒ์ฃผ์˜๊ตญ๊ฐ€๋กœ, ์˜จ ๋ฏผ์กฑ์˜ ์ฐธ๋‹ค์šด ์กฐ๊ตญ์œผ๋กœ ์ž๋ž‘๋–จ์น˜๊ฒŒ ๋˜์˜€์Šต๋‹ˆ๋‹ค... ์šฐ๋ฆฌ ๊ณตํ™”๊ตญ์€ ์„ ๊ตฐ์œผ๋กœ ์œ„์šฉ๋–จ์น˜๋Š” ๋ถˆํŒจ์˜ ์‚ฌํšŒ์ฃผ์˜๊ฐ•๊ตญ์ž…๋‹ˆ๋‹ค. - ์œ„๋Œ€ํ•œ ์ˆ˜๋ น๋‹˜๊ณผ ์šฐ๋ฆฌ ๋‹น์˜ ํ˜๋ช…๋ ฅ์‚ฌ๋Š” ์„ ๊ตฐํ˜๋ช…๋ น๋„์˜ ๋ ฅ์‚ฌ์ด๋ฉฐ ์šฐ๋ฆฌ ์กฐ๊ตญ๊ณผ ํ˜๋ช…์€ ์„ ๊ตฐ์˜ ๊ธฐ์น˜ ๋ฐ‘์— ๋ถˆํŒจ์˜ ์œ„๋ ฅ์„ ๋–จ์น˜๋ฉฐ ์Šน๋ฆฌ์™€ ์˜๊ด‘์˜ ๊ธธ์„ ์—ด์–ด์™”์Šต๋‹ˆ๋‹ค. ์šฐ๋ฆฌ ์ˆ˜๋ น๋‹˜๊ป˜์„œ๋Š” ์„ ๊ตฐ์œผ๋กœ ์กฐ๊ตญ์„ ํ•ด๋ฐฉํ•˜์‹œ๊ณ  - ์šฐ๋ฆฌ์˜ ์„ ๊ตฐ์ •์น˜๋Š” ์ฃผ์ฒด์‚ฌ์ƒ์— ๋ฟŒ๋ฆฌ๋ฅผ ๋‘๊ณ  ์ฃผ์ฒด์˜ ํ˜๋ช…์›๋ฆฌ์™€ ์ „๋žต์ „์ˆ , ์ฃผ์ฒด์˜ ๋ น๋„๋ฐฉ๋ฒ•๊ณผ ๋ น๋„์˜ˆ์ˆ ์„ ์ „๋ฉด์ ์œผ๋กœ ๊ตฌํ˜„ํ•˜๊ณ  ์ง‘๋Œ€์„ฑํ•˜๊ณ  ์žˆ๋Š” ๊ฒƒ์œผ๋กœ ํ•˜์—ฌ ์šฐ๋ฆฌ ์กฐ๊ตญ๊ณผ ํ˜๋ช…์˜ ๋ฐฑ์ „๋ฐฑ์Šน์˜ ๊ธฐ์น˜๋กœ, ํ˜๋ช…๊ณผ ๊ฑด์„ค์˜ ๋งŒ๋Šฅ์˜ ๋ณด๊ฒ€์œผ๋กœ ๋˜๊ณ  ์žˆ์Šต๋‹ˆ๋‹ค. - ์šฐ๋ฆฌ๋Š” ์„ ๊ตฐํ˜๋ช…์ด์ง„๊ตฐ์„ ํž˜์žˆ๊ฒŒ ๋‹ค๊ทธ์ณ ์ด ๋•…์œ„์— ์‚ฌํšŒ์ฃผ์˜๊ฐ•์„ฑ๋Œ€๊ตญ์„ ์ผ๋– ์„ธ์›€์œผ๋กœ์จ ์ˆ˜๋ น๋‹˜์˜ ์œ„์—…, ์ฃผ์ฒด์˜ ์‚ฌํšŒ์ฃผ์˜์œ„์—…์ˆ˜ํ–‰์—์„œ ๊ฒฐ์ •์ ์ธ ์Šน๋ฆฌ๋ฅผ ์ด๋ฃฉํ•˜์—ฌ์•ผ ํ•ฉ๋‹ˆ๋‹ค. - ์šฐ๋ฆฌ ๊ณตํ™”๊ตญ์€ ์ฃผ์ฒดํ˜๋ช…์˜ ์œ„๋Œ€ํ•œ ์ „์ทจ๋ฌผ์ด๋ฉฐ ๊ณตํ™”๊ตญ์ •๊ถŒ์€ ํ˜๋ช…๊ณผ ๊ฑด์„ค์˜ ๊ฐ•๋ ฅํ•œ ๋ฌด๊ธฐ์ž…๋‹ˆ๋‹ค. ๊ณตํ™”๊ตญ์ •๊ถŒ์€ ์šฐ๋ฆฌ ๋‹น์˜ ์„ ๊ตฐ๋ น๋„ ๋ฐ‘์— ํ˜๋ช…๊ณผ ๊ฑด์„ค์„ ํž˜์žˆ๊ฒŒ ๋‹ค๊ทธ์ณ ์ˆ˜๋ น๋‹˜์˜ ์œ„์—…, ์ฃผ์ฒดํ˜๋ช…์œ„์—…์„ ๋๊นŒ์ง€ ์™„์„ฑํ•˜์—ฌ์•ผ ํ•ฉ๋‹ˆ๋‹ค.ใ€ ์œ„์™€ ๊ฐ™์ด ์œ„ ๋ฌธ๊ฑด์€ 2008. 9. 5. ์กฐ์„ ์‹ ๋ณด์— ๊ฒŒ์žฌ๋œ ๊น€์ •์ผ์˜ ๋…ธ์ž‘์œผ๋กœ์„œ, ๊น€์ผ์„ฑ ์ฃผ์ฒด์‚ฌ์ƒ์„ ์ธ๋ฏผ๋Œ€์ค‘๊ณผ ๋ฏผ์กฑ์˜ ์ž์ฃผ์„ฑ์„ ์˜นํ˜ธํ•˜๊ณ  ์‹คํ˜„ํ•˜๋Š” ์œ„๋Œ€ํ•œ ์ง€๋„์‚ฌ์ƒ์ด๋ผ๊ณ  ๋ฏธํ™”ํ•˜๊ณ , ์„ ๊ตฐ์ •์น˜๋ฅผ ํ˜๋ช…๊ณผ ๊ฑด์„ค์˜ ๋งŒ๋Šฅ ๋ณด๊ฒ€์ด๋ผ๊ณ  ์ฐฌ์–‘ํ•˜๋ฉด์„œ ๋ถํ•œ์„ ์‚ฌํšŒ์ฃผ์˜ ๊ฐ•์„ฑ๋Œ€๊ตญ์ด๋ผ๊ณ  ์„ ์ „ํ•˜๋Š” ๋‚ด์šฉ์ด๋‹ค. ์ด๋กœ์จ ํ”ผ๊ณ ์ธ์€ ๊ตญ๊ฐ€์˜ ์กด๋ฆฝยท์•ˆ์ „์ด๋‚˜ ์ž์œ ๋ฏผ์ฃผ์  ๊ธฐ๋ณธ์งˆ์„œ๋ฅผ ์œ„ํƒœ๋กญ๊ฒŒ ํ•œ๋‹ค๋Š” ์ •์„ ์•Œ๋ฉด์„œ ๋ฐ˜๊ตญ๊ฐ€๋‹จ์ฒด๋‚˜ ๊ทธ ๊ตฌ์„ฑ์› ๋˜๋Š” ๊ทธ ์ง€๋ น์„ ๋ฐ›์€ ์ž์˜ ํ™œ๋™์„ ์ฐฌ์–‘ยท๊ณ ๋ฌดยท์„ ์ „ ๋˜๋Š” ์ด์— ๋™์กฐํ•  ๋ชฉ์ ์œผ๋กœ ์ด์ ํ‘œํ˜„๋ฌผ์„ ์†Œ์ง€ํ•˜์˜€๋‹ค. 2) 2015. 11. 13. ํ”ผ๊ณ ์ธ์˜ ์ฃผ๊ฑฐ์ง€์— ์ด์ ํ‘œํ˜„๋ฌผ ์†Œ์ง€ ํ”ผ๊ณ ์ธ์€ 2015. 11. 13.๊ฒฝ ํ”ผ๊ณ ์ธ์˜ ์ฃผ๊ฑฐ์ง€์— โ€˜21์„ธ๊ธฐ ์‚ฌํšŒ๋ณ€ํ˜์šด๋™๊ณผ ์ง„๋ณด์  ์ •๊ถŒ๊ต์ฒด : ๋ฐ˜์„ฑ๊ณผ ์ „๋งโ€™์ด๋ผ๋Š” ์ œ๋ชฉ์˜ ์ฑ…์ž๋ฅผ ๋ณด๊ด€ํ•˜๊ณ  ์žˆ์—ˆ๋Š”๋ฐ, ๊ทธ ์ฃผ์š”๋‚ด์šฉ์€ ๋‹ค์Œ๊ณผ ๊ฐ™๋‹ค. ใ€Žใ€Šโ€˜21์„ธ๊ธฐ ์‚ฌํšŒ๋ณ€ํ˜์šด๋™๊ณผ ์ง„๋ณด์  ์ •๊ถŒ๊ต์ฒด : ๋ฐ˜์„ฑ๊ณผ ์ „๋งโ€™ ์ œํ•˜์˜ ์ฑ…์ž ์ฃผ์š”๋‚ด์šฉใ€‹ - ๋ถ(์กฐ์„ )๊ณผ ๊พธ๋ฐ”๋Š” ์‚ฌํšŒ์ฃผ์˜๊ณ„ํš๊ฒฝ์ œ์™€ ์‚ฌํšŒ์ฃผ์˜์ž๋ฆฝ๊ฒฝ์ œ๋ฅผ ๊ณ ์ˆ˜ํ•˜๋ฉด์„œ ์ œ๊ตญ์ฃผ์˜์„ธ๊ณ„์ฒด์ œ์— ๋งž์„œ ์‹ธ์šฐ๋Š” ์‚ฌํšŒ์ฃผ์˜๋ฐ˜์ œํˆฌ์Ÿ์„ ๋ฒŒ์ด๊ณ  ์žˆ๋‹ค. ํŠนํžˆ ๊ทธ ๋‘ ๋‚˜๋ผ๊ฐ€ ๋ฐ˜์‚ฌํšŒ์ฃผ์˜์ ๋Œ€์ •์ฑ…์— ๋งž์„œ ์‹ธ์šฐ๊ธฐ ์œ„ํ•ด ํ˜•์„ฑํ•œ ๋ฐ˜์ œ๊ตฐ์‚ฌ์ „์„ ์ด ๋‹๋ณด์ธ๋‹ค. - ์•„์‹œ์•„์•„ํ”„๋ฆฌ์นด, ๋ผํ‹ด์•„๋ฉ”๋ฆฌ์นด ๊ณณ๊ณณ์— ํ˜•์„ฑ๋œ ์—ฌ๋Ÿฌ ํ˜•ํƒœ์˜ ๋ฐ˜์ œ์ „์„ ๋“ค ๊ฐ€์šด๋ฐ์„œ ๋‹๋ณด์ด๋Š” ๊ฒƒ์€ ํ•œ(์กฐ์„ )๋ฐ˜๋„์˜ ์ „๋žต์  ์œ„์ƒ์ด๋‹ค. - ์ œ๊ตญ์ฃผ์˜์ ๋ น๊ตฐ๊ณผ ๋Œ€์น˜ํ•œ ๋ถ์ธก์— ํ˜•์„ฑ๋œ ๋ฐ˜์ œ๊ตฐ์‚ฌ์ „์„ ์€ ๋ฐ˜์‚ฌํšŒ์ฃผ์˜์ ๋Œ€์ •์ฑ…์„ ๋ฌด๋ ฅํ™”ํ•˜๋Š” ์‚ฌํšŒ์ฃผ์˜๋ฐ˜์ œํˆฌ์Ÿ์˜ ์ค‘์‹ฌ์ถ•์ด๋‹ค. - ํ˜„ ์‹œ๊ธฐ ๋ฐ˜์ œํˆฌ์Ÿ์˜ ์ „๋žต๋ชฉํ‘œ๋Š”, โ€˜ํ•œ๋ฏธ๋™๋งน๊ด€๊ณ„โ€™์˜ ๋‹จ์ ˆ, ์‹ ์‹๋ฏผ์ฃผ์˜์ฒด์ œ์˜ ํ•ด์ฒด, ๊ทธ๋ฆฌ๊ณ  ์ œ๊ตญ์ฃผ์˜์„ธ๊ณ„์ฒด์ œ์—์„œ์˜ ์ดํƒˆ์ด๋‹ค. ๋ฐ˜์ œํˆฌ์Ÿ์˜ ๊ธฐ๋ณธ๋ฐฉํ–ฅ์€ ์ฃผํ•œ๋ฏธ๊ตญ๊ตฐ์‚ฌ๋ น๋ถ€ ํ•ด์ฒด์™€ ์ „๋ฉด์ฒ ๊ตฐ, ์ฃผํ•œ๋ฏธ๊ตญ๋Œ€์‚ฌ๊ด€์˜ ๋‚ด์ •๊ฐ„์„ญ๊ณผ ์ •์น˜๊ณต์ž‘ ๋ฐ˜๋Œ€, ๋ฏธ๊ตญ๊ณ„ํˆฌ๊ธฐ์ž๋ณธ์˜ ์ˆ˜ํƒˆ์ €์ง€์™€ ํ•œ๋ฏธ์ž์œ ๋ฌด์—ญํ˜‘์ • ๋ฐ˜๋Œ€์ด๋‹ค. - ์‹ ์‹๋ฏผ์ฃผ์˜ ์˜ˆ์†์ •๊ถŒ์„ ํ‡ด์ถœ์‹œํ‚ค๋Š” ์ง„๋ณด์  ์ •๊ถŒ๊ต์ฒด๋Š”, ๊ตญ๊ฐ€์˜ˆ์†๊ด€๊ณ„์˜ ์ค‘์‹ฌ์ถ•์„ ๋ถ€๋Ÿฌ๋œจ๋ฆฌ๋Š” ๋ฐ˜์ œํˆฌ์Ÿ์ด๋ฉฐ ๋™์‹œ์— ๊ทธ ์ •๊ถŒ์˜ ๋น„ํ˜ธ๋ฅผ ๋ฐ›์œผ๋ฉด์„œ ๊ทธ ์ •๊ถŒ๊ณผ ๊ฒฐํƒํ•œ ์‹ ์‹๋ฏผ์ฃผ์˜๊ธฐ์ƒ์ž๋ณธ์˜ ๊ณ„๊ธ‰์ฐฉ์ทจ๊ด€๊ณ„๋ฅผ ์—†์• ๋Š” ๊ณ„๊ธ‰ํˆฌ์Ÿ์ด๋‹ค. - ๋‚จ(ํ•œ๊ตญ)์˜ ์ž๋ณธ์ฃผ์˜๊ณต์—…ํ™”๊ฐ€ ์ œ๊ตญ์ฃผ์˜๋…์ ์ž๋ณธ์—๊ฒŒ ๋นŒ๋ถ™์–ด ์„ฑ์žฅํ•ด์˜จ ์‹ ์‹๋ฏผ์ฃผ์˜ ๊ธฐ์ƒ์ž๋ณธ์— ์˜ํ•ด ์ถ”์ง„๋˜์–ด ์™”๊ณ , ์ œ๊ตญ์ฃผ์˜์„ธ๊ณ„์ฒด์ œ์˜ ์ƒ์ธต์ค‘์•™์ธ ๋ฏธ๊ตญ์ด ์ง์ ‘ ์œก์„ฑํ•œ ์‹ ์‹๋ฏผ์ฃผ์˜ ์˜ˆ์†์ •๊ถŒ์ด ๋‚จ(ํ•œ๊ตญ)์˜ ์ž๋ณธ์ฃผ์˜๊ณต์—…ํ™”๋ฅผ ์ด๋Œ์–ด์™”๋‹ค๋Š” ์ ์ด๋‹ค. - ๊ทธ๊ฒƒ์€ ์‹ ์‹๋ฏผ์ฃผ์˜ ์˜ˆ์†์ •๊ถŒ์˜ ๊ฐ•์••์  ๊ฐœ๋ฐœ๋…์žฌ์™€ ์‹ ์‹๋ฏผ์ฃผ์˜๊ธฐ์ƒ์ž๋ณธ์˜ ๋ณ€ํƒœ์  ์ž๋ณธ์ถ•์ ์œผ๋กœ ์‹คํ˜„๋œ ์‹ ์‹๋ฏผ์ฃผ์˜๊ณต์—…ํ™”์ด๋‹ค. - ๋Œ€์„ ์ด๋‚˜ ์ด์„ ์—์„œ ๋…ธ๋™๊ณ„๊ธ‰๊ณผ ๊ทผ๋กœ๋Œ€์ค‘์ด ์ˆ˜๊ตฌ์šฐํŒŒ์ •๋‹น์— ํˆฌํ‘œํ•˜์—ฌ ์ž์‹ ์˜ ๊ณ„๊ธ‰์  ์ฒ˜์ง€์™€ ์ƒ๋ฐ˜๋˜๋Š” ์ •์น˜์  ์„ ํƒ์„ ๋˜ํ’€์ดํ•˜๋Š” ๊ฒƒ์€ ๊ฐ€์žฅ ๋‹๋ณด์ด๋Š” ์ผํƒˆ์˜์‹ํ™”ํ˜„์ƒ์ด๋‹ค. - ์ด๋…์ฃผ์ž…์žฅ์น˜๋ฅผ ํ†ตํ•˜์—ฌ ์ด๋ฏธ ์ผํƒˆ์˜์‹ํ™” ๋œ ๋…ธ๋™๊ณ„๊ธ‰๊ณผ ๊ทผ๋กœ๋Œ€์ค‘์„ ๊ณ„๊ธ‰์˜์‹ํ™”, ์ง„๋ณด์˜์‹ํ™”, ๋ฐ˜์ œ์˜์‹ํ™”ํ•˜๋Š” ๊ณผ์—…์€ ๋ฐ˜๋“œ์‹œ ๋ฏผ์ฃผ๋…ธ์กฐ์šด๋™๊ณผ ๊ฐ๊ณ„๊ฐ์ธต ๋Œ€์ค‘์šด๋™์„ ํ†ตํ•ด์„œ ์‹คํ˜„๋˜์–ด์•ผ ํ•œ๋‹ค. - ํ˜„์žฅ์ •์น˜ํ™œ๋™์„ ๊ฒฐ์ •์ ์œผ๋กœ ๊ฐ•ํ™”, ๋ฐœ์ „์‹œ์ผœ์•ผ ํ•œ๋‹ค. ํ˜„์ •ํ™œ๋™์˜ ๊ธฐ๋ณธ์€ ์˜์‹ํ™”๊ต์œก์ด๋‹ค. ํ˜„์ •ํ™œ๋™์€ ํ˜„์žฅ์˜ ๊ตฌ์ฒด์ ์ธ ์กฐ๊ฑด์— ๋”ฐ๋ผ ์ˆ˜์—†์ด ๋‹ค์–‘ํ•œ ๋ฐฉ์‹์œผ๋กœ ์ „๊ฐœ๋˜๋Š”๋ฐ, ์„ ๋™์„ ์ „ํ•™๊ต, ๋…ธ๋™์ž์ •์น˜ํ•™๊ต, ๋†๋ฏผ์ •์น˜ํ•™๊ต, ํ™œ๋™๊ฐ€์œก์„ฑ, ์ „์ˆ ํˆฌ์Ÿ์กฐ์ง ๋“ฑ์ด ํฌํ•จ๋  ๊ฒƒ์ด๋‹ค. - 20๋ช… ์ •๋„๋กœ ์ด๋ฃจ์–ด์ง„ ๋…ธ์กฐํ•™์Šต๋ฐ˜, ๋งˆ์„๋ถ„ํšŒ, ์ง€์—ญ์ฃผ๋ฏผํ˜‘์˜ํšŒ, ๋‹น์›ํ˜‘์˜ํšŒ, ์ฒญ๋…„๋ถ„ํšŒ, ๋Œ€ํ•™์ƒ ๋™์•„๋ฆฌ ๋“ฑ์„ 1๋งŒ๊ฐœ ์กฐ์งํ•  ํ•„์š”๊ฐ€ ์žˆ๋‹ค. ๋‚จ์ธก์˜ ์ธ๊ตฌ์ˆ˜๋กœ ๋ณด๋‚˜ ์‚ฌํšŒ๋ณ€ํ˜์šด๋™์˜ ๋ฐœ์ „๋‹จ๊ณ„๋กœ ๋ณด๋‚˜, ์˜ค๋Š˜ ์‚ฌํšŒ๋ณ€ํ˜์šด๋™์— ์ œ๊ธฐ๋œ ์ผ์ฐจ์  ๊ณผ์ œ๋Š”, ์กฐ์งํ™”๋˜๊ณ  ์˜์‹ํ™”๋œ 20๋งŒ ๋ช…์˜ ํˆฌ์Ÿ๋Œ€์˜ค์™€ 1๋งŒ ๋ช…์˜ 30-40๋Œ€ ํ˜„์žฅ์ •์น˜ ํ™œ๋™๊ฐ€๋ฅผ ์งง์€ ๊ธฐ๊ฐ„์— ์ง‘์ค‘์ ์œผ๋กœ ์œก์„ฑํ•˜๋Š” ๊ฒƒ์ด๋‹ค. - ์šฐ๋ฆฌ ์‹์˜ ์ง„๋ณด์  ์ •๊ถŒ๊ต์ฒด๊ฒฝ๋กœ์— ๋Œ€์ค‘ํ•ญ์Ÿ์ด ๋“ค์–ด๊ฐ€๋Š” ๊นŒ๋‹ญ์€, ๋Œ€์ค‘ํ•ญ์Ÿ์ด๋ผ๋Š” ๊ฐ€์žฅ ๊ฐ•๋ ฅํ•œ ์ถฉ๊ฒฉ์„ ๊ฐ€ํ•˜๋ฉด์„œ ์ •๊ถŒ๊ตฌ์กฐ๋ฅผ ์‹ ์†ํ•˜๊ณ  ๊ฒฐ์ •์ ์œผ๋กœ ํƒ€๊ฒฉํ•ด์•ผ ๋‚จ(ํ•œ๊ตญ)์— ์„ธ์›Œ์ง„ ๊ฒฌ๊ณ ํ•œ ์‹ ์‹๋ฏผ์ฃผ์˜์ฒด์ œ๋ฅผ ํ•ด์ฒดํ•  ์ˆ˜ ์žˆ๊ธฐ ๋•Œ๋ฌธ์ด๋‹ค. ์šฐ๋ฆฌ์‹์˜ ์ง„๋ณด์  ์ •๊ถŒ๊ต์ฒด๊ฒฝ๋กœ์—์„œ ์ผ์–ด๋‚  ๋Œ€์ค‘ํ•ญ์Ÿ์€, 1960๋…„ 4์›”, 1980๋…„ 5์›”, 1987๋…„ 6์›”์— ์ผ์–ด๋‚ฌ๋˜ ๋ฏธ์™„์„ฑ ๋Œ€์ค‘ํ•ญ์Ÿ์˜ ์—ญ์‚ฌ์  ๊ณผ์—…์„ ์™„์ˆ˜ํ•˜๋Š” ์™„์„ฑ๋œ ๋Œ€์ค‘ํ•ญ์Ÿ์œผ๋กœ ๋  ๊ฒƒ์ด๋‹ค.ใ€ ์ด์™€ ๊ฐ™์ด ์œ„ ์ฑ…์ž๋Š” 2008. 2. 16. โ€˜(๋ช…์นญ 40 ์ƒ๋žต)์—ฐ๊ตฌ์†Œโ€™๊ฐ€ ์ฃผ์ตœํ•œ โ€˜์ฝ”๋ฆฌ์•„๋ฐ˜๋„ ๋Œ€๋ณ€ํ˜๊ธฐโ€™ ํ† ๋ก ํšŒ์—์„œ (๋ช…์นญ 5 ์ƒ๋žต)์—ฐ๊ตฌ์†Œ ์†Œ์žฅ ๊ณต์†Œ์™ธ 22๊ฐ€ ๋ฐœํ‘œํ•œ ๋ฐœ์ œ๋ฌธ์œผ๋กœ์„œ, ๋‚จํ•œ์„ ๋ฏธ๊ตญ์˜ ์‹ ์‹๋ฏผ์ฃผ์˜ ์˜ˆ์†์ •๊ถŒ์œผ๋กœ ๊ทœ์ •ํ•˜๊ณ , ๋ถํ•œ์˜ ๋ฐ˜๋ฏธ๋…ธ์„ ์„ ์ฐฌ์–‘ํ•˜๋ฉด์„œ ๋…ธ๋™๊ณ„๊ธ‰์„ ์ฃผ์ถ•์œผ๋กœ ๋ฐ˜๋ฏธยท๋ฐ˜์ •๋ถ€ํˆฌ์Ÿ์„ ๋ฒŒ์—ฌ ๋‚จํ•œ ๋‚ด์—์„œ โ€˜์ง„๋ณด์  ๋ฏผ์ฃผ์ฃผ์˜โ€™๋ฅผ ์‹คํ˜„ํ•˜๊ณ  โ€˜๋‚จํ•œ์˜ ์ฒด์ œ๋ณ€ํ˜โ€™์„ ์œ„ํ•œ ํ˜„์žฅ์ •์น˜ ํ™œ๋™๊ฐ€๋ฅผ ์ง‘์ค‘ ์œก์„ฑํ•˜์—ฌ ํ˜„์žฅ ํ™œ๋™์„ ๊ฐ•ํ™”ํ•ด ๋‚˜๊ฐ€์ž๊ณ  ์„ ๋™ํ•˜๋Š” ๋‚ด์šฉ์ด๋‹ค. ์ด๋กœ์จ ํ”ผ๊ณ ์ธ์€ ๊ตญ๊ฐ€์˜ ์กด๋ฆฝยท์•ˆ์ „์ด๋‚˜ ์ž์œ ๋ฏผ์ฃผ์  ๊ธฐ๋ณธ์งˆ์„œ๋ฅผ ์œ„ํƒœ๋กญ๊ฒŒ ํ•œ๋‹ค๋Š” ์ •์„ ์•Œ๋ฉด์„œ ๋ฐ˜๊ตญ๊ฐ€๋‹จ์ฒด๋‚˜ ๊ทธ ๊ตฌ์„ฑ์› ๋˜๋Š” ๊ทธ ์ง€๋ น์„ ๋ฐ›์€ ์ž์˜ ํ™œ๋™์„ ์ฐฌ์–‘ยท๊ณ ๋ฌดยท์„ ์ „ ๋˜๋Š” ์ด์— ๋™์กฐํ•  ๋ชฉ์ ์œผ๋กœ ์ด์ ํ‘œํ˜„๋ฌผ์„ ์†Œ์ง€ํ•˜์˜€๋‹ค. ใ€์ฆ๊ฑฐ์˜ ์š”์ง€ใ€‘ [2016๊ณ ํ•ฉ538] 1. ์ฆ์ธ ๊ณต์†Œ์™ธ 10, ๊ณต์†Œ์™ธ 7, ๊ณต์†Œ์™ธ 46์˜ ๊ฐ ๋ฒ•์ •์ง„์ˆ  1. ์„œ์šธ์ค‘์•™์ง€๋ฐฉ๋ฒ•์› 2015๊ณ ๋‹จ7843 ์‚ฌ๊ฑด์˜ ์ œ5ํšŒ ๊ณตํŒ์กฐ์„œ ์ค‘ ์ฆ์ธ ๊ณต์†Œ์™ธ 47์˜ ์ง„์ˆ ๊ธฐ์žฌ 1. ์ด ๋ฒ•์›์˜ ์ฆ๊ฑฐ๋ชฉ๋ก ์ˆœ๋ฒˆ 132 ๋‚ด์ง€ 201์— ๋Œ€ํ•œ ๊ฐ ๊ฒ€์ฆ๊ฒฐ๊ณผ 1. ์ด์ ํ‘œํ˜„๋ฌผ ๋ถ„์„ ๋ณด๊ณ ์„œ, ๋งค์ฒด๋ณ„ ์ด์ ํ‘œํ˜„๋ฌผ ๋ถ„์„๋ณด๊ณ ์„œ(์ฆ์ œ122-1ํ˜ธ), ๋งค์ฒด๋ณ„ ์ด์ ํ‘œํ˜„๋ฌผ ๋ถ„์„๋ณด๊ณ ์„œ(์ฆ์ œ128-1ํ˜ธ), ๋งค์ฒด๋ณ„ ์ด์ ํ‘œํ˜„๋ฌผ ๋ถ„์„๋ณด๊ณ ์„œ(์ฆ์ œ129-1ํ˜ธ), ๋งค์ฒด๋ณ„ ์ด์ ํ‘œํ˜„๋ฌผ ๋ถ„์„๋ณด๊ณ ์„œ(์ฆ์ œ131-1ํ˜ธ), ๋งค์ฒด๋ณ„ ์ด์ ํ‘œํ˜„๋ฌผ ๋ถ„์„๋ณด๊ณ ์„œ(์ฆ์ œ132-1ํ˜ธ) 1. ์ˆ˜์‚ฌ๋ณด๊ณ ์„œ[์ฆ์ œ132ํ˜ธ HDD(์‚ผ์„ฑ์™ธ์žฅํ•˜๋“œ) ๋‚ด ์ €์žฅ๋˜์–ด ์žˆ๋Š” ๋ฐฑ๋‘์‚ฐ 3๋Œ€ ์žฅ๊ตฐ์˜ ์ˆญ๊ณ ํ•œ ๋‹น์กฐ์ง ๊ด€๋…์„ ๋”ฐ๋ผ ๋ฐฐ์šธ ๋ฐ ๋Œ€ํ•˜์—ฌ ์ด์ ํ‘œํ˜„๋ฌผ ์ฒจ๋ถ€ ๋ณด๊ณ ], ๋ฐฑ๋‘์‚ฐ 3๋Œ€ ์žฅ๊ตฐ์˜ ์ˆญ๊ณ ํ•œ ๋‹น์กฐ์ง ๊ด€๋…์„ ๋ฐฐ์šธ ๋ฐ ๋Œ€ํ•˜์—ฌ ์‚ฌ๋ณธ ์ถœ๋ ฅ๋ฌผ 1. EnCase์—์„œ ์ถ”์ถœํ•œ ํ”ผ๊ณ ์ธ ์ „์ž์ •๋ณด(ํŒŒ์ผ)๋ชฉ๋ก ์ถœ๋ ฅ๋ณธ, ํ”ผ๊ณ ์ธ ์ด์ ํ‘œํ˜„๋ฌผ ํŒŒ์ผ ๋ถ„์„ ์—‘์…€ ์ถœ๋ ฅ๋ณธ, ํ”ผ๊ณ ์ธ ์••์ถ•ํŒŒ์ผ์— ๋Œ€ํ•œ ์ด์ ํ‘œํ˜„๋ฌผ ๋ถ„์„ ์—‘์…€ ์ถœ๋ ฅ๋ณธ 1. ์ฆ์ œ122-1ํ˜ธ ์ €์žฅ โ€˜์ด์„œใ€Š๋ถˆ๋ฉธ์˜ ๋ ฅ์‚ฌใ€‹์žฅํŽธ์†Œ์„ค ์‚ผ์ฒœ๋ฆฌ ๊ฐ•์‚ฐโ€™(ํŒŒ์ผ๋ช… : ์‚ผ์ฒœ๋ฆฌ๊ฐ•์‚ฐ. HWP) ์ œํ•˜ ๋ฌธ๊ฑด ์ผ๋ถ€ ์ถœ๋ ฅ๋ฌผ, ์ฆ์ œ122-1ํ˜ธ ์ €์žฅ โ€˜์• ๊ตญ์‹œ๋Œ€โ€™(ํŒŒ์ผ๋ช… : ์• ๊ตญ์‹œ๋Œ€.hwp) ์ œํ•˜ ๋ฌธ๊ฑด ์ผ๋ถ€ ์ถœ๋ ฅ๋ฌผ, ์ฆ์ œ122-1ํ˜ธ ์ €์žฅ โ€˜๋ถ‰์€ํ•ด๋ฐœ์•„๋ž˜ํ•ญ์ผํ˜๋ช…20๋…„1โ€™(ํŒŒ์ผ๋ช… : โ€˜๋ถ‰์€ํ•ด๋ฐœ์•„๋ž˜ํ•ญ์ผํ˜๋ช…20๋…„1.hwp)โ€™ ์ œํ•˜ ๋ฌธ๊ฑด ์ผ๋ถ€ ์ถœ๋ ฅ๋ฌผ, ์ฆ์ œ122-1ํ˜ธ ์ €์žฅ โ€˜๋ถ‰์€ํ•ด๋ฐœ์•„๋ž˜ํ•ญ์ผํ˜๋ช…20๋…„2โ€™(ํŒŒ์ผ๋ช… : ๋ถ‰์€ํ•ด๋ฐœ์•„๋ž˜ํ•ญ์ผํ˜๋ช…20๋…„2.hwp) ์ œํ•˜ ๋ฌธ๊ฑด ์ผ๋ถ€ ์ถœ๋ ฅ๋ฌผ, ์ฆ์ œ122-1ํ˜ธ ์ €์žฅ โ€˜๋ถ‰์€ํ•ด๋ฐœ์•„๋ž˜ํ•ญ์ผํ˜๋ช…20๋…„3โ€™(ํŒŒ์ผ๋ช… : ๋ถ‰์€ํ•ด๋ฐœ์•„๋ž˜ํ•ญ์ผํ˜๋ช…20๋…„3.hwp) ์ œํ•˜ ๋ฌธ๊ฑด ์ผ๋ถ€ ์ถœ๋ ฅ๋ฌผ, ์ฆ์ œ122-1ํ˜ธ ์ €์žฅ โ€˜๋ถ‰์€ํ•ด๋ฐœ์•„๋ž˜ํ•ญ์ผํ˜๋ช…20๋…„4โ€™(ํŒŒ์ผ๋ช… : ๋ถ‰์€ํ•ด๋ฐœ์•„๋ž˜ํ•ญ์ผํ˜๋ช…20๋…„4.hwp) ์ œํ•˜ ๋ฌธ๊ฑด ์ผ๋ถ€ ์ถœ๋ ฅ๋ฌผ, ์ฆ์ œ122-1ํ˜ธ ์ €์žฅ โ€˜๋ถ‰์€ํ•ด๋ฐœ์•„๋ž˜ํ•ญ์ผํ˜๋ช…20๋…„5โ€™(ํŒŒ์ผ๋ช… : ๋ถ‰์€ํ•ด๋ฐœ์•„๋ž˜ํ•ญ์ผํ˜๋ช…20๋…„5.hwp) ์ œํ•˜ ๋ฌธ๊ฑด ์ผ๋ถ€ ์ถœ๋ ฅ๋ฌผ, ์ฆ์ œ122-1ํ˜ธ ์ €์žฅ โ€˜์„ธ๊ธฐ์™€ ๋”๋ถˆ์–ดโ€™ ๋‚ญ๋… ์Œ์„ฑํŒŒ์ผ ๋‚ด์šฉ ์ถœ๋ ฅ๋ฌผ, ์ฆ์ œ128-1ํ˜ธ ์ €์žฅ โ€˜ํ‰์–‘์€ ์„ ์–ธํ•œ๋‹ค 1,2โ€™(ํŒŒ์ผ๋ช… : 9.HWP, 10.HWP) ์ œํ•˜ ๋ฌธ๊ฑด ์ผ๋ถ€ ์ถœ๋ ฅ๋ฌผ, ์ฆ์ œ128-1ํ˜ธ ์ €์žฅ โ€˜๊น€์ •์ผ์• ๊ตญ์ฃผ์˜์— ๋Œ€ํ•œ ์ด์•ผ๊ธฐโ€™(ํŒŒ์ผ๋ช… : 1.HWP) ์ œํ•˜ ๋ฌธ๊ฑด ์ผ๋ถ€ ์ถœ๋ ฅ๋ฌผ, ์ฆ์ œ128-1ํ˜ธ ์ €์žฅ โ€˜ํ•œ๊ตญ์ „์œ„์กฐ์ง ์šด๋™์‚ฌโ€™(ํŒŒ์ผ๋ช… : ํ•œ๊ตญ์ „์œ„์กฐ์ง์šด๋™์‚ฌ.HWP) ์ œํ•˜ ๋ฌธ๊ฑด ์ผ๋ถ€ ์ถœ๋ ฅ๋ฌผ, ์ฆ์ œ128-1ํ˜ธ ์ €์žฅ โ€˜ํ•ด๋ฐฉ์ „์•ผ'(ํŒŒ์ผ๋ช… : 201501062109067_HBSTUZIQ.hwp) ์ œํ•˜ ๋ฌธ๊ฑด ์ผ๋ถ€ ์ถœ๋ ฅ๋ฌผ, ์ฆ์ œ128-1ํ˜ธ ์ €์žฅ โ€˜๊น€์ •์ผ ์ €์ž‘์ง‘ 1๊ถŒโ€™(ํŒŒ์ผ๋ช… : ๊น€์ •์ผ์ €์ž‘์ง‘01.hwp) ์ œํ•˜ ๋ฌธ๊ฑด ์ผ๋ถ€ ์ถœ๋ ฅ๋ฌผ, ์ฆ์ œ128-1ํ˜ธ ์ €์žฅ โ€˜์„ธ๊ธฐ์™€๋”๋ถˆ์–ด ๊ฐ•์ขŒ๋ณธ 1โ€™(ํŒŒ์ผ๋ช… : ์„ธ๊ธฐ์™€๋”๋ถˆ์–ด๊ฐ•์ขŒ๋ณธ1.hwp) ์ œํ•˜ ๋ฌธ๊ฑด ์ผ๋ถ€ ์ถœ๋ ฅ๋ฌผ, ์ฆ์ œ128-1ํ˜ธ โ€˜์กฐ์„ ๋กœ๋™๋‹น๋žต์‚ฌโ€™(ํŒŒ์ผ๋ช… : ์กฐ์„ ๋กœ๋™๋‹น๋žต์‚ฌ.HWP) ์ œํ•˜ ๋ฌธ๊ฑด ์ผ๋ถ€ ์ถœ๋ ฅ๋ฌผ, ์ฆ์ œ128-1ํ˜ธ ์ €์žฅ โ€˜๊ฐœ์„ โ€™(ํŒŒ์ผ๋ช… : ์ด์„œใ€Š๋ถˆ๋ฉธ์˜๋ ฅ์‚ฌใ€‹- ๊ฐœ์„ .pdf) ์ œํ•˜ ๋ฌธ๊ฑด ์ผ๋ถ€ ์ถœ๋ ฅ๋ฌผ, ์ฆ์ œ128-1ํ˜ธ ์ €์žฅ โ€˜์šฐ๋“ฑ๋ถˆ 1,2โ€™(ํŒŒ์ผ๋ช… : 3.HWP, 4.HWP) ์ œํ•˜ ๋ฌธ๊ฑด ์ผ๋ถ€ ์ถœ๋ ฅ๋ฌผ, ์ฆ์ œ128-1ํ˜ธ ์ €์žฅ โ€˜๋ ฅ์‚ฌ์‚ฌ์ „โ€™(ํŒŒ์ผ๋ช… : ์—ญ์‚ฌ์‚ฌ์ „.hwp) ์ œํ•˜ ๋ฌธ๊ฑด ์ผ๋ถ€ ์ถœ๋ ฅ๋ฌผ, ์ฆ์ œ128-1ํ˜ธ ์ €์žฅ โ€˜ํ˜„๋Œ€์กฐ์„ ์—ญ์‚ฌโ€™(ํŒŒ์ผ๋ช… : ํ˜„๋Œ€์กฐ์„ ์—ญ์‚ฌ.hwp) ์ œํ•˜ ๋ฌธ๊ฑด ์ผ๋ถ€ ์ถœ๋ ฅ๋ฌผ, ์ฆ์ œ 128-1ํ˜ธ ์ €์žฅ โ€˜์„ธ๊ณ„๋ฅผ ๋งคํ˜น์‹œํ‚ค๋Š” ๊น€์ •์ผ ์ •์น˜'(ํŒŒ์ผ๋ช… : 12.HWP) ์ œํ•˜ ๋ฌธ๊ฑด ์ผ๋ถ€ ์ถœ๋ ฅ๋ฌผ, ์ฆ์ œ128-1ํ˜ธ ์ €์žฅ โ€˜1932๋…„โ€™(ํŒŒ์ผ๋ช… : 24.HWP) ์ œํ•˜ ๋ฌธ๊ฑด ์ผ๋ถ€ ์ถœ๋ ฅ๋ฌผ, ์ฆ์ œ128-1ํ˜ธ ์ €์žฅ โ€˜์ฃผ์ฒด์„ฑ๊ณผ ๋ฏผ์กฑ์„ฑ์„ ์ง€ํ‚ค์‹œ๋Š” ๊น€์ •์ผ ์žฅ๊ตฐโ€™(ํŒŒ์ผ๋ช… : 35.HWP) ์ œํ•˜ ๋ฌธ๊ฑด ์ผ๋ถ€ ์ถœ๋ ฅ๋ฌผ, ์ฆ์ œ128-1ํ˜ธ ์ €์žฅ โ€˜์ฃผ์ฒด์ •์น˜๊ฒฝ์ œํ•™โ€™(ํŒŒ์ผ๋ช… : ์ฃผ์ฒด์˜์ •์น˜๊ฒฝ์ œํ•™.hwp) ์ œํ•˜ ๋ฌธ๊ฑด ์ผ๋ถ€ ์ถœ๋ ฅ๋ฌผ, ์ฆ์ œ128-1ํ˜ธ ์ €์žฅ โ€˜๊ฒฝ์ œ์‚ฌ์ „1 ์œ„๋Œ€ํ•œ ์ˆ˜๋ น ๊น€์ผ์„ฑ๋™์ง€~์‚ฌํšŒ์ด์ƒ์‚ฐ์•กโ€™(ํŒŒ์ผ๋ช… : ๊ฒฝ์ œ์‚ฌ์ „1.hwp) ์ œํ•˜ ๋ฌธ๊ฑด ์ผ๋ถ€ ์ถœ๋ ฅ๋ฌผ, ์ฆ์ œ128-1ํ˜ธ ์ €์žฅ โ€˜์‚ผ์ฒœ๋ฆฌ ๊ฐ•์‚ฐโ€™(ํŒŒ์ผ๋ช… : ์‚ผ์ฒœ๋ฆฌ๊ฐ•์‚ฐ.HWP) ์ œํ•˜ ๋ฌธ๊ฑด ์ผ๋ถ€ ์ถœ๋ ฅ๋ฌผ, ์ฆ์ œ128-1ํ˜ธ ์ €์žฅ โ€˜์„ธ๊ธฐ์™€ ๋”๋ถˆ์–ด 1๊ถŒ 3์žฅ 1์ ˆโ€™(ํŒŒ์ผ๋ช… : huigo-3-1-01.mp3) ์ œํ•˜ ์Œ์„ฑํŒŒ์ผ ๋‚ด์šฉ ์ถœ๋ ฅ๋ฌผ, ์ฆ์ œ129-1ํ˜ธ ์ €์žฅ โ€˜๊น€์ผ์„ฑ ์ €์ž‘์ง‘โ€™ 1~44๊ถŒ(ํŒŒ์ผ๋ช… : ๊น€์ผ์„ฑ์ €์ž‘์ง‘01.hwp ~ ๊น€์ผ์„ฑ์ €์ž‘์ง‘44.hwp) ์ œํ•˜ ๋ฌธ๊ฑด ์ผ๋ถ€ ์ถœ๋ ฅ๋ฌผ, ์ฆ์ œ129-1ํ˜ธ ์ €์žฅ โ€˜๊น€์ •์ผ ์ €์ž‘์ง‘โ€™ 1~3๊ถŒ, 5~14๊ถŒ(ํŒŒ์ผ๋ช… : ๊น€์ •์ผ์ €์ž‘์ง‘01.hwp ~ ๊น€์ •์ผ์ €์ž‘์ง‘3.hwp, ๊น€์ •์ผ์ €์ž‘์ง‘05.hwp ~ ๊น€์ •์ผ์ €์ž‘์ง‘14.hwp) ์ œํ•˜ ๋ฌธ๊ฑด ์ผ๋ถ€ ์ถœ๋ ฅ๋ฌผ, ์ฆ์ œ129-1ํ˜ธ ์ €์žฅ โ€˜์ƒˆ์‹œ๋Œ€์ •์น˜ํ•™์›๋ก โ€™(ํŒŒ์ผ๋ช… : ์ƒˆ์‹œ๋Œ€์ •์น˜ํ•™์›๋ก .hwp) ์ œํ•˜ ๋ฌธ๊ฑด ์ผ๋ถ€ ์ถœ๋ ฅ๋ฌผ, ์ฆ์ œ129-1ํ˜ธ ์ €์žฅ โ€˜์„ ๊ตฐ์ •์น˜ : ์ฃผ์ฒด์‚ฌํšŒ์ฃผ์˜ ์ƒ๋ช…์„ โ€™(ํŒŒ์ผ๋ช… : ์„ ๊ตฐ์ •์น˜_์ฃผ์ฒด์‚ฌํšŒ์ฃผ์˜์ƒ๋ช…์„ .hwp) ์ œํ•˜ ๋ฌธ๊ฑด ์ผ๋ถ€ ์ถœ๋ ฅ๋ฌผ, ์ฆ์ œ129-1ํ˜ธ ์ €์žฅ โ€˜ํ•ญ์ผ์œ ๊ฒฉ๋Œ€์›๋“ค์˜ ์‚ถ๊ณผ ํˆฌ์Ÿ ํšŒ์ƒ๊ธฐ (์ƒ)โ€™(ํŒŒ์ผ๋ช… : ํšŒ์ƒ๊ธฐ(์ƒํŽธ).hwp) ์ œํ•˜ ๋ฌธ๊ฑด ์ผ๋ถ€ ์ถœ๋ ฅ๋ฌผ, ์ฆ์ œ129-1ํ˜ธ ์ €์žฅ โ€˜ํ•ญ์ผ์œ ๊ฒฉ๋Œ€์›๋“ค์˜ ์‚ถ๊ณผ ํˆฌ์Ÿ ํšŒ์ƒ๊ธฐ (์ค‘)โ€™(ํŒŒ์ผ๋ช… : ํšŒ์ƒ๊ธฐ(์ค‘ํŽธ).HWP) ์ œํ•˜ ๋ฌธ๊ฑด ์ผ๋ถ€ ์ถœ๋ ฅ๋ฌผ, ์ฆ์ œ129-1ํ˜ธ ์ €์žฅ โ€˜ํ•ญ์ผ์œ ๊ฒฉ๋Œ€์›๋“ค์˜ ์‚ถ๊ณผ ํˆฌ์Ÿ ํšŒ์ƒ๊ธฐ(ํ•˜)โ€™(ํŒŒ์ผ๋ช… : ํšŒ์ƒ๊ธฐ(ํ•˜ํŽธ).HWP) ์ œํ•˜ ๋ฌธ๊ฑด ์ผ๋ถ€ ์ถœ๋ ฅ๋ฌผ, ์ฆ์ œ129-1ํ˜ธ ์ €์žฅ โ€˜์ฃผ์ฒด์˜ ํ•œ๊ตญ์‚ฌํšŒ ๋ณ€ํ˜์šด๋™๋ก โ€™(ํŒŒ์ผ๋ช… : 1-1 ์ฃผ์ฒด์˜ํ•œ๊ตญ์‚ฌํšŒ๋ณ€ํ˜์šด๋™๋ก .HWP) ์ œํ•˜ ๋ฌธ๊ฑด ์ผ๋ถ€ ์ถœ๋ ฅ๋ฌผ, ์ฆ์ œ129-1ํ˜ธ ์ €์žฅ โ€˜๊น€์ผ์„ฑ์ฃผ์˜ ๋„ํ•ดโ€™(ํŒŒ์ผ๋ช… : ๊น€์ผ์„ฑ์ฃผ์˜๋„ํ•ด.HWP) ์ œํ•˜ ๋ฌธ๊ฑด ์ผ๋ถ€ ์ถœ๋ ฅ๋ฌผ, ์ฆ์ œ129-1ํ˜ธ ์ €์žฅ โ€˜์ฃผ์ฒด์‚ฌ์ƒ์ด์„œ1โ€™(ํŒŒ์ผ๋ช… : ์ฃผ์ฒด์‚ฌ์ƒ์ด์„œ_01.hwp) ์ œํ•˜ ๋ฌธ๊ฑด ์ผ๋ถ€ ์ถœ๋ ฅ๋ฌผ, ์ฆ์ œ129-1ํ˜ธ ์ €์žฅ โ€˜์ฃผ์ฒด์‚ฌ์ƒ ๋…ธ์ž‘ 60์„ (30/30)โ€™(ํŒŒ์ผ๋ช… : ๋…ธ์ž‘60์„ .HWP) ์ œํ•˜ ๋ฌธ๊ฑด ์ผ๋ถ€ ์ถœ๋ ฅ๋ฌผ, ์ฆ์ œ129-1ํ˜ธ ์ €์žฅ โ€˜์‚ฌ๋ž‘์˜ ์ •์น˜๊ฐ€ ๊น€์ •์ผ ์žฅ๊ตฐโ€™(ํŒŒ์ผ๋ช… : ์‚ฌ๋ž‘์˜์ •์น˜๊ฐ€๊น€์ •์ผ์žฅ๊ตฐ.HWP) ์ œํ•˜ ๋ฌธ๊ฑด ์ผ๋ถ€ ์ถœ๋ ฅ๋ฌผ, ์ฆ์ œ129-1ํ˜ธ ์ €์žฅ โ€˜ํ•œ(์กฐ์„ )๋ฐ˜๋„์˜ ์šด๋ช…์„ ๊ฒฐ์ •์ง€์„ ๋‘ ๊ฐœ์˜ ๊ตฐ์‚ฌ์ „๋žต ์ œ1๋ถ€ : ๋ฏธ๊ตญ์˜ ์„ ์ œ๊ณต๊ฒฉ์ „๋žต๊ณผ โ€˜์ž‘์ „๊ณ„ํš5027โ€™โ€™(ํŒŒ์ผ๋ช… : ํ•œ๋ฐ˜๋„์˜์šด๋ช…์„๊ฒฐ์ •์ง€์„๋‘๊ฐœ์˜๊ตฐ์‚ฌ์ „๋žต1.hwp) ์ œํ•˜ ๋ฌธ๊ฑด ์ผ๋ถ€ ์ถœ๋ ฅ๋ฌผ, ์ฆ์ œ129-1ํ˜ธ ์ €์žฅ โ€˜์••๋ก๊ฐ•โ€™(ํŒŒ์ผ๋ช… : ์••๋ก๊ฐ•.HWP) ์ œํ•˜ ๋ฌธ๊ฑด ์ผ๋ถ€ ์ถœ๋ ฅ๋ฌผ, ์ฆ์ œ129-1ํ˜ธ ์ €์žฅ โ€˜๋‹ป์€ ์˜ฌ๋ž๋‹คโ€™(ํŒŒ์ผ๋ช… : 01๋‹ป์€์˜ฌ๋ž๋‹ค.HWP) ์ œํ•˜ ๋ฌธ๊ฑด ์ผ๋ถ€ ์ถœ๋ ฅ๋ฌผ, ์ฆ์ œ129-1ํ˜ธ ์ €์žฅ โ€˜์˜์ƒโ€™(ํŒŒ์ผ๋ช… : ์˜์ƒ.HWP) ์ œํ•˜ ๋ฌธ๊ฑด ์ผ๋ถ€ ์ถœ๋ ฅ๋ฌผ, ์ฆ์ œ129-1ํ˜ธ ์ €์žฅ โ€˜์ด๋Œ€โ€™(ํŒŒ์ผ๋ช… : ์„ ๊ตฐ์†Œ์„ค์ด๋Œ€.HWP) ์ œํ•˜ ๋ฌธ๊ฑด ์ผ๋ถ€ ์ถœ๋ ฅ๋ฌผ, ์ฆ์ œ129-1ํ˜ธ ์ €์žฅ โ€˜๋ ฅ์‚ฌ์˜ ๋Œ€ํ•˜โ€™(ํŒŒ์ผ๋ช… : ์—ญ์‚ฌ์˜๋Œ€ํ•˜.HWP) ์ œํ•˜ ๋ฌธ๊ฑด ์ผ๋ถ€ ์ถœ๋ ฅ๋ฌผ, ์ฆ์ œ 129-1ํ˜ธ ์ €์žฅ โ€˜ํ˜„๋Œ€์กฐ์„ ์—ญ์‚ฌโ€™(ํŒŒ์ผ๋ช… : ํ˜„๋Œ€์กฐ์„ ์—ญ์‚ฌ.hwp) ์ œํ•˜ ๋ฌธ๊ฑด ์ผ๋ถ€ ์ถœ๋ ฅ๋ฌผ, ์ฆ์ œ 129-1ํ˜ธ ์ €์žฅ โ€˜์กฐ๊ตญํ†ต์ผ 10๋Œ€ ๋…ธ์ž‘โ€™(ํŒŒ์ผ๋ช… : ์กฐ๊ตญํ†ต์ผ10๋Œ€๋…ธ์ž‘.hwp) ์ œํ•˜ ๋ฌธ๊ฑด ์ผ๋ถ€ ์ถœ๋ ฅ๋ฌผ, ์ฆ์ œ129-1ํ˜ธ ์ €์žฅ โ€˜๊น€์ •์ผ ์ง€๋„์žโ€™ ์ œ1๋ถ€(ํŒŒ์ผ๋ช… : ๊น€์ •์ผ์ง€๋„์ž1.hwp) ์ œํ•˜ ๋ฌธ๊ฑด ์ผ๋ถ€ ์ถœ๋ ฅ๋ฌผ, ์ฆ์ œ129-1ํ˜ธ ์ €์žฅ โ€˜์„ ๊ตฐ์ •์น˜ ๋…ผ๋ฌธ 10์„ โ€™(ํŒŒ์ผ๋ช… : ์„ ๊ตฐ์ •์น˜๋ก ๋ฌธ10์„ .hwp) ์ œํ•˜ ๋ฌธ๊ฑด ์ผ๋ถ€ ์ถœ๋ ฅ๋ฌผ, ์ฆ์ œ129-1ํ˜ธ ์ €์žฅ โ€˜์„ ๊ตฐ์ •์น˜ ๊ด€๋ จ ๋กœ๋™์‹ ๋ฌธ ์ •๋ก ์ง‘โ€™(ํŒŒ์ผ๋ช… : ์„ ๊ตฐ์ •์น˜ ๊ด€๋ จ ๋กœ๋™์‹ ๋ฌธ์ •๋ก  ๋ชจ์Œ์ง‘.hwp) ์ œํ•˜ ๋ฌธ๊ฑด ์ผ๋ถ€ ์ถœ๋ ฅ๋ฌผ, ์ฆ์ œ129-1ํ˜ธ ์ €์žฅ โ€˜์ ˆ์„ธ์˜ ์• ๊ตญ์ž ๊น€์ •์ผ ์žฅ๊ตฐโ€™(ํŒŒ์ผ๋ช… : 1 ์ ˆ์„ธ์˜์• ๊ตญ์ž๊น€์ •์ผ์žฅ๊ตฐ.hwp) ์ œํ•˜ ๋ฌธ๊ฑด ์ผ๋ถ€ ์ถœ๋ ฅ๋ฌผ, ์ฆ์ œ129-1ํ˜ธ ์ €์žฅ โ€˜์œ„๋Œ€ํ•œ ์ˆ˜๋ น ๊น€์ผ์„ฑ๋™์ง€ ํ˜๋ช…ํ™œ๋™๋žต๋ ฅโ€™(ํŒŒ์ผ๋ช… : 23.hwp) ์ œํ•˜ ๋ฌธ๊ฑด ์ผ๋ถ€ ์ถœ๋ ฅ๋ฌผ, ์ฆ์ œ129-1ํ˜ธ ์ €์žฅ โ€˜์„ ๊ตฐ์ •์น˜์— ๋Œ€ํ•œ ๋ฆฌํ•ดโ€™(ํŒŒ์ผ๋ช… : 1 ์„ ๊ตฐ์ •์น˜์— ๋Œ€ํ•œ ๋ฆฌํ•ด.hwp) ์ œํ•˜ ๋ฌธ๊ฑด ์ผ๋ถ€ ์ถœ๋ ฅ๋ฌผ, ์ฆ์ œ129-1ํ˜ธ ์ €์žฅ โ€˜๊น€์ •์ผ์žฅ๊ตฐ์˜ ๋…ธ๋ž˜โ€™(ํŒŒ์ผ๋ช… : ๊น€์ •์ผ์žฅ๊ตฐ์˜ ๋…ธ๋ž˜.mp3) ์ œํ•˜ ์Œ์„ฑํŒŒ์ผ ๋‚ด์šฉ ์ถœ๋ ฅ๋ฌผ, ์ฆ์ œ129-1ํ˜ธ ์ €์žฅ โ€˜์–ด๋””์—๊ณ„์‹ญ๋‹ˆ๊นŒ ๊ทธ๋ฆฌ์šด์žฅ๊ตฐ๋‹˜โ€™(ํŒŒ์ผ๋ช… : ์–ด๋””์—๊ณ„์‹ญ๋‹ˆ๊นŒ๊ทธ๋ฆฌ์šด์žฅ๊ตฐ๋‹˜.mp3) ์ œํ•˜ ์Œ์„ฑํŒŒ์ผ ๋‚ด์šฉ ์ถœ๋ ฅ๋ฌผ, ์ฆ์ œ131-1ํ˜ธ ์ €์žฅ โ€˜์ ˆ์„ธ ์œ„์ธ์˜ ๋ช…์–ธโ€™(ํŒŒ์ผ๋ช… : Kim-JI-G_myung.hwp) ์ œํ•˜ ๋ฌธ๊ฑด ์ผ๋ถ€ ์ถœ๋ ฅ๋ฌผ, ์ฆ์ œ131-1ํ˜ธ ์ €์žฅ โ€˜๋…ธ์„ ๊ณผ ์ •์ฑ… ์ œ1ํ˜ธโ€™(ํŒŒ์ผ๋ช… : NJ-001(20070120).hwp) ์ œํ•˜ ๋ฌธ๊ฑด ์ผ๋ถ€ ์ถœ๋ ฅ๋ฌผ, ์ฆ์ œ131-1ํ˜ธ ์ €์žฅ โ€˜๋ฏธ๊ตญ์„ ํ˜‘์ƒ๊ณผ ๋Œ€๊ฒฐ์˜ ๊ต์ฐจ์ ์œผ๋กœ ๋Œ์–ด๊ฐ„ ๋ฒ ์ด์ง•ํšŒ๋‹ดโ€™(ํŒŒ์ผ๋ช… : 030517.html) ์ œํ•˜ ๋ฌธ๊ฑด ์ผ๋ถ€ ์ถœ๋ ฅ๋ฌผ, ์ฆ์ œ131-1ํ˜ธ ์ €์žฅ โ€˜์„ ๊ตฐ์ •์น˜ ๊ด€๋ จ ๋กœ๋™์‹ ๋ฌธ ์ •๋ก ์ง‘โ€™(ํŒŒ์ผ๋ช… : ์„ ๊ตฐ์ •์น˜๊ด€๋ จ๋กœ๋™์‹ ๋ฌธ์ •๋ก ์ง‘.hwp) ์ œํ•˜ ๋ฌธ๊ฑด ์ผ๋ถ€ ์ถœ๋ ฅ๋ฌผ, ์ฆ์ œ131-1ํ˜ธ ์ €์žฅ โ€˜๊น€์ •์ผ์žฅ๊ตฐ๋‹˜์€ ์–ด๋–ค ๋ถ„์ด์‹ ๊ฐ€โ€™(ํŒŒ์ผ๋ช… : ๊น€์ •์ผ์žฅ๊ตฐ๋‹˜์€์–ด๋–ค๋ถ„์ด์‹ ๊ฐ€.hwp) ์ œํ•˜ ๋ฌธ๊ฑด ์ผ๋ถ€ ์ถœ๋ ฅ๋ฌผ, ์ฆ์ œ131-1ํ˜ธ โ€˜์œ„์ธ์˜ ๋งค๋ ฅโ€™(ํŒŒ์ผ๋ช… : ์œ„์ธ์˜๋งค๋ ฅ.hwp) ์ œํ•˜ ๋ฌธ๊ฑด ์ผ๋ถ€ ์ถœ๋ ฅ๋ฌผ, ์ฆ์ œ131-1ํ˜ธ ์ €์žฅ โ€˜๋ฏผ์กฑ๊ณผ ์„ ๊ตฐ์ •์น˜โ€™(ํŒŒ์ผ๋ช… : ๋ฏผ์กฑ๊ณผ์„ ๊ตฐ์ •์น˜.hwp) ์ œํ•˜ ๋ฌธ๊ฑด ์ผ๋ถ€ ์ถœ๋ ฅ๋ฌผ, ์ฆ์ œ131-1ํ˜ธ ์ €์žฅ โ€˜๊น€์ •์ผ์žฅ๊ตฐ์˜ ํ†ต์ผ์ „๋žตโ€™(ํŒŒ์ผ๋ช… : 1501_๊น€์ •์ผ์žฅ๊ตฐ์˜ ํ†ต์ผ์ „๋žต.hwp) ์ œํ•˜ ๋ฌธ๊ฑด ์ผ๋ถ€ ์ถœ๋ ฅ๋ฌผ, ์ฆ์ œ131-1ํ˜ธ ์ €์žฅ โ€˜๊น€์ •์ผ์žฅ๊ตฐ ์„ ๊ตฐ์ •์น˜๋ฆฌ๋ก โ€™(ํŒŒ์ผ๋ช… : ๊น€์ •์ผ์žฅ๊ตฐ์„ ๊ตฐ์ •์น˜๋ฆฌ๋ก .hwp) ์ œํ•˜ ๋ฌธ๊ฑด ์ผ๋ถ€ ์ถœ๋ ฅ๋ฌผ, ์ฆ์ œ131-1ํ˜ธ ์ €์žฅ โ€˜์„ ๊ตฐ์ •์น˜ ๋…ผ๋ฌธ 10์„ โ€™(ํŒŒ์ผ๋ช… : ์„ ๊ตฐ์ •์น˜๋…ผ๋ฌธ10์„ .hwp) ์ œํ•˜ ๋ฌธ๊ฑด ์ผ๋ถ€ ์ถœ๋ ฅ๋ฌผ, ์ฆ์ œ131-1ํ˜ธ ์ €์žฅ โ€˜์ ˆ์„ธ์˜ ์• ๊ตญ์ž ๊น€์ •์ผ ์žฅ๊ตฐโ€™(ํŒŒ์ผ๋ช… : ์ ˆ์„ธ์˜ ์• ๊ตญ์ž ๊น€์ •์ผ ์žฅ๊ตฐ.hwp) ์ œํ•˜ ๋ฌธ๊ฑด ์ผ๋ถ€ ์ถœ๋ ฅ๋ฌผ, ์ฆ์ œ131-1ํ˜ธ ์ €์žฅ โ€˜ํ•ญ์ผ๋นจ์ฐŒ์‚ฐ ์ฐธ๊ฐ€์ž๋“ค์˜ ํšŒ์ƒ๊ธฐ 4โ€™(ํŒŒ์ผ๋ช… : ํ•ญ์ผ๋นจ์ฐŒ์‚ฐ์ฐธ๊ฐ€์ž๋“ค์˜ํšŒ์ƒ๊ธฐ4.hwp) ์ œํ•˜ ๋ฌธ๊ฑด ์ผ๋ถ€ ์ถœ๋ ฅ๋ฌผ, ์ฆ์ œ131-1ํ˜ธ ์ €์žฅ โ€˜๊น€์ผ์„ฑ๋™์ง€ ํšŒ๊ณ ๋ก ์„ธ๊ธฐ์™€ ๋”๋ถˆ์–ด 1โ€™(ํŒŒ์ผ๋ช… : ์„ธ๊ธฐ์™€๋”๋ถˆ์–ด1.hwp) ์ œํ•˜ ๋ฌธ๊ฑด ์ผ๋ถ€ ์ถœ๋ ฅ๋ฌผ, ์ฆ์ œ131-1ํ˜ธ ์ €์žฅ โ€˜์„ธ๊ธฐ์™€๋”๋ถˆ์–ด ๊ฐ•์ขŒ๋ณธ 1โ€™(ํŒŒ์ผ๋ช… : ์„ธ๊ธฐ์™€๋”๋ถˆ์–ด๊ฐ•์ขŒ๋ณธ1.hwp) ์ œํ•˜ ๋ฌธ๊ฑด ์ผ๋ถ€ ์ถœ๋ ฅ๋ฌผ, ์ฆ์ œ131-1ํ˜ธ ์ €์žฅ โ€˜์œ„๋Œ€ํ•œ ์˜๋„์ž ๊น€์ •์ผ๋™์ง€๊ป˜์„œ ๋ฐํžˆ์‹  ์„ ๊ตฐํ˜๋ช… ๋ น๋„์— ๊ด€ํ•œ ๋…์ฐฝ์  ์‚ฌ์ƒโ€™(ํŒŒ์ผ๋ช… : ์„ ๊ตฐํ˜๋ช…๋ น๋„์—๊ด€ํ•œ๋…์ฐฝ์ ์‚ฌ์ƒ.hwp) ์ œํ•˜ ๋ฌธ๊ฑด ์ผ๋ถ€ ์ถœ๋ ฅ๋ฌผ, ์ฆ์ œ131-1ํ˜ธ ์ €์žฅ โ€˜์„ ๊ตฐ์ •์น˜: ์ฃผ์ฒด์‚ฌํšŒ์ฃผ์˜ ์ƒ๋ช…์„ โ€™(ํŒŒ์ผ๋ช… : ์„ ๊ตฐ์ •์น˜_์ฃผ์ฒด์‚ฌํšŒ์ฃผ์˜์ƒ๋ช…์„ .hwp) ์ œํ•˜ ๋ฌธ๊ฑด ์ผ๋ถ€ ์ถœ๋ ฅ๋ฌผ, ์ฆ์ œ131-1ํ˜ธ ์ €์žฅ โ€˜๋ฌธํ•™์˜ˆ์ˆ ์ž‘ํ’ˆ์˜ ์ข…์žโ€™(ํŒŒ์ผ๋ช… : ์ข…์ž๋ก .hwp) ์ œํ•˜ ๋ฌธ๊ฑด ์ผ๋ถ€ ์ถœ๋ ฅ๋ฌผ, ์ฆ์ œ131-1ํ˜ธ ์ €์žฅ โ€˜์„ธ์ƒ์—์„œ ์ œ์ผ ๊ฐ•ํ•˜์‹  ๋ถ„โ€™(ํŒŒ์ผ๋ช… : 5608_์„ธ์ƒ์—์„œ ์ œ์ผ ๊ฐ•ํ•˜์‹  ๋ถ„.hwp) ์ œํ•˜ ๋ฌธ๊ฑด ์ผ๋ถ€ ์ถœ๋ ฅ๋ฌผ, ์ฆ์ œ131-1ํ˜ธ ์ €์žฅ โ€˜๋ฏผ์กฑ์‚ฌ์— ๊ธธ์ด ๋น›๋‚  ์œ„๋Œ€ํ•œ 5๋…„โ€™(ํŒŒ์ผ๋ช… : w5n.hwp) ์ œํ•˜ ๋ฌธ๊ฑด ์ผ๋ถ€ ์ถœ๋ ฅ๋ฌผ, ์ฆ์ œ131-1ํ˜ธ ์ €์žฅ โ€˜ํ•œ(์กฐ์„ )๋ฐ˜๋„์˜ ์šด๋ช…์„ ๊ฒฐ์ •์ง€์„ ๋‘ ๊ฐœ์˜ ์ „๋žต ์ œ2๋ถ€: ๋ถ(์กฐ์„ )์˜ ๋Œ€๋Ÿ‰๋ณด๋ณต์ „๋žต๊ณผ ๋ฏธ๊ตญ์˜ ํŒจ์ „๊ฒฝํ—˜โ€™(ํŒŒ์ผ๋ช… : 030419.html) ์ œํ•˜ ๋ฌธ๊ฑด ์ผ๋ถ€ ์ถœ๋ ฅ๋ฌผ, ์ฆ์ œ132-1ํ˜ธ ์ €์žฅ โ€˜ํ•œ๊ตญ์‚ฌํšŒ ์„ฑ๊ฒฉ ๋…ผ์˜์˜ ์žฌ์กฐ๋ช…โ€™(ํŒŒ์ผ๋ช… : ํ•œ๊ตญ์‚ฌํšŒ์„ฑ๊ฒฉ๋…ผ์˜์˜์žฌ์กฐ๋ช….hwp)โ€™ ์ œํ•˜ ๋ฌธ๊ฑด ์ผ๋ถ€ ์ถœ๋ ฅ๋ฌผ, ์ฆ์ œ132-1ํ˜ธ ์ €์žฅ โ€˜์• ๊ตญ์‹œ๋Œ€โ€™(ํŒŒ์ผ๋ช… : ์• ๊ตญ์‹œ๋Œ€.hwp) ์ œํ•˜ ๋ฌธ๊ฑด ์ผ๋ถ€ ์ถœ๋ ฅ๋ฌผ, ์ฆ์ œ132-1ํ˜ธ ์ €์žฅ โ€˜๋ฏผ์กฑ๊ณผ ์ฒ ํ•™โ€™(ํŒŒ์ผ๋ช… : ๋ฏผ์กฑ๊ณผ์ฒ ํ•™.hwp) ์ œํ•˜ ๋ฌธ๊ฑด ์ผ๋ถ€ ์ถœ๋ ฅ๋ฌผ, ์ฆ์ œ132-1ํ˜ธ ์ €์žฅ โ€˜์ •์น˜์‚ฌ์ „1 ์œ„๋Œ€ํ•œ ์ˆ˜๋ น ๊น€์ผ์„ฑ๋™์ง€~๋ฒจ์ง€๋„โ€™(ํŒŒ์ผ๋ช… : ์ •์น˜์‚ฌ์ „1.hwp) ์ œํ•˜ ๋ฌธ๊ฑด ์ผ๋ถ€ ์ถœ๋ ฅ๋ฌผ, ์ฆ์ œ132-1ํ˜ธ ์ €์žฅ โ€˜์‚ฌํšŒ์ฃผ์˜์— ๋Œ€ํ•œ ์ฃผ์ฒด์  ๋ฆฌํ•ดโ€™(ํŒŒ์ผ๋ช… : ์‚ฌํšŒ์ฃผ์˜์—๋Œ€ํ•œ์ฃผ์ฒด์ ๋ฆฌํ•ด.hwp) ์ œํ•˜ ๋ฌธ๊ฑด ์ผ๋ถ€ ์ถœ๋ ฅ๋ฌผ, ์ฆ์ œ132-1ํ˜ธ โ€˜๊น€์ผ์„ฑ์ฃผ์˜ ๋„ํ•ดโ€™(ํŒŒ์ผ๋ช… : ๊น€์ผ์„ฑ์ฃผ์˜๋„ํ•ด.hwp) ์ œํ•˜ ๋ฌธ๊ฑด ์ผ๋ถ€ ์ถœ๋ ฅ๋ฌผ, ์ฆ์ œ132-1ํ˜ธ ์ €์žฅ โ€˜๊น€์ผ์„ฑ๋™์ง€ ํšŒ๊ณ ๋ก ์„ธ๊ธฐ์™€ ๋”๋ถˆ์–ด 1~8โ€™(ํŒŒ์ผ๋ช… : ์„ธ๊ธฐ์™€๋”๋ถˆ์–ด1.hwp ~ ์„ธ๊ธฐ์™€๋”๋ถˆ์–ด8.hwp) ์ œํ•˜ ๋ฌธ๊ฑด ์ผ๋ถ€ ์ถœ๋ ฅ๋ฌผ, ์ฆ์ œ132-1ํ˜ธ ์ €์žฅ โ€˜์ •ํ†ต๊ณผ ๊ณ„์Šน ์œ„๋Œ€ํ•œ ์ธ๊ฐ„, ์ƒˆ๋กœ์šด ๋ฌธ๋ช…์„ ์œ„ํ•˜์—ฌโ€™(ํŒŒ์ผ๋ช… : Jungtong.hwp) ์ œํ•˜ ๋ฌธ๊ฑด ์ผ๋ถ€ ์ถœ๋ ฅ๋ฌผ, ์ฆ์ œ132-1ํ˜ธ ์ €์žฅ โ€˜์„ ๊ตฐ์ •์น˜: ์ฃผ์ฒด์‚ฌํšŒ์ฃผ์˜ ์ƒ๋ช…์„ โ€™(ํŒŒ์ผ๋ช… : ์„ ๊ตฐ์ •์น˜_์ฃผ์ฒด์‚ฌํšŒ์ฃผ์˜์ƒ๋ช…์„ .hwp) ์ œํ•˜ ๋ฌธ๊ฑด ์ผ๋ถ€ ์ถœ๋ ฅ๋ฌผ, ์ฆ์ œ132-1ํ˜ธ ์ €์žฅ โ€˜๊น€์ผ์„ฑ ์ €์ž‘์ง‘ ์ œ1๊ถŒโ€™(ํŒŒ์ผ๋ช… : ๊น€์ผ์„ฑ์ €์ž‘์ง‘01.hwp) ์ œํ•˜ ๋ฌธ๊ฑด ์ผ๋ถ€ ์ถœ๋ ฅ๋ฌผ, ์ฆ์ œ132-1ํ˜ธ ์ €์žฅ โ€˜๋ ฅ์‚ฌ์˜ ๋Œ€ํ•˜'(ํŒŒ์ผ๋ช… : ์—ญ์‚ฌ์˜๋Œ€ํ•˜.HWP) ์ œํ•˜ ๋ฌธ๊ฑด ์ผ๋ถ€ ์ถœ๋ ฅ๋ฌผ, ์ฆ์ œ132-1ํ˜ธ ์ €์žฅ โ€˜(์กฐ์„ )๋ฐ˜๋„์˜ ์šด๋ช…์„ ๊ฒฐ์ •์ง€์„ ๋‘ ๊ฐœ์˜ ๊ตฐ์‚ฌ์ „๋žต ์ œ1๋ถ€ : ๋ฏธ๊ตญ์˜ ์„ ์ œ๊ณต๊ฒฉ์ „๋žต๊ณผ โ€˜์ž‘์ „๊ณ„ํš5027โ€™(ํŒŒ์ผ๋ช… : ํ•œ๋ฐ˜๋„์˜์šด๋ช…์„๊ฒฐ์ •์ง€์„๋‘๊ฐœ์˜๊ตฐ์‚ฌ์ „๋žต.hwp) ์ œํ•˜ ๋ฌธ๊ฑด ์ผ๋ถ€ ์ถœ๋ ฅ๋ฌผ, ์ฆ์ œ132-1ํ˜ธ ์ €์žฅ โ€˜21์„ธ๊ธฐ ๊น€์ •์ผ์‹œ๋Œ€โ€™(ํŒŒ์ผ๋ช… : 21์„ธ๊ธฐ๊น€์ •์ผ์‹œ๋Œ€.HWP) ์ œํ•˜ ๋ฌธ๊ฑด ์ผ๋ถ€ ์ถœ๋ ฅ๋ฌผ, ์ฆ์ œ132-1ํ˜ธ ์ €์žฅ โ€˜ํ”ผ๋ฐ”๋‹คโ€™(ํŒŒ์ผ๋ช… : ํ”ผ๋ฐ”๋‹ค.hwp) ์ œํ•˜ ๋ฌธ๊ฑด ์ผ๋ถ€ ์ถœ๋ ฅ๋ฌผ, ์ฆ์ œ132-1ํ˜ธ ์ €์žฅ โ€˜์ •์น˜๊ฒฝ์ œํ•™๊ฐœ๋ก  ์ฃผ์ฒด์˜ ์ •์น˜๊ฒฝ์ œํ•™โ€™(ํŒŒ์ผ๋ช… : ์ •๊ฒฝ.hwp) ์ œํ•˜ ๋ฌธ๊ฑด ์ผ๋ถ€ ์ถœ๋ ฅ๋ฌผ, ์ฆ์ œ132-1ํ˜ธ ์ €์žฅ โ€˜์˜์ƒโ€™(ํŒŒ์ผ๋ช… : ์˜์ƒ.HWP) ์ œํ•˜ ๋ฌธ๊ฑด ์ผ๋ถ€ ์ถœ๋ ฅ๋ฌผ, ์ฆ์ œ132-1ํ˜ธ ์ €์žฅ โ€˜์ฒ ์˜ ์‹ ๋…โ€™(ํŒŒ์ผ๋ช… : ์ฒ ์˜์‹ ๋….hwp) ์ œํ•˜ ๋ฌธ๊ฑด ์ผ๋ถ€ ์ถœ๋ ฅ๋ฌผ, ์ฆ์ œ132-1ํ˜ธ ์ €์žฅ โ€˜๊น€์ผ์„ฑ์ฃผ์„๊ณผ ๋ฏผ์กฑ๋Œ€๋‹จ๊ฒฐโ€™(ํŒŒ์ผ๋ช… : ๊น€์ผ์„ฑ์ฃผ์„๊ณผ๋ฏผ์กฑ๋Œ€๋‹จ๊ฒฐ.hwp) ์ œํ•˜ ๋ฌธ๊ฑด ์ผ๋ถ€ ์ถœ๋ ฅ๋ฌผ, ์ฆ์ œ132-1ํ˜ธ ์ €์žฅ โ€˜๋”๋””๊ฐ€๋„ ์‚ฌ๋žŒ์ƒ๊ฐ ํ•˜์ง€์š”โ€™(ํŒŒ์ผ๋ช… : ๋”๋””๊ฐ€๋„์‚ฌ๋žŒ์ƒ๊ฐํ•˜์ง€์š”.hwp) ์ œํ•˜ ๋ฌธ๊ฑด ์ผ๋ถ€ ์ถœ๋ ฅ๋ฌผ, ์ฆ์ œ132-1ํ˜ธ ์ €์žฅ โ€˜๋ฌธํ•™์˜ˆ์ˆ ์ž‘ํ’ˆ์˜ ์ข…์žโ€™(ํŒŒ์ผ๋ช… : ์ข…์ž๋ก .hwp) ์ œํ•˜ ๋ฌธ๊ฑด ์ผ๋ถ€ ์ถœ๋ ฅ๋ฌผ, ์ฆ์ œ132-1ํ˜ธ ์ €์žฅ โ€˜๋‹น์‹ ์ด ์—†์œผ๋ฉด ์กฐ๊ตญ๋„ ์—†๋‹คโ€™(ํŒŒ์ผ๋ช… : ๋‹น์‹ ์ด์—†์œผ๋ฉด์กฐ๊ตญ๋„์—†๋‹ค.mp3) ์ œํ•˜ ์Œ์„ฑํŒŒ์ผ ๋‚ด์šฉ ์ถœ๋ ฅ๋ฌผ, ์ฆ์ œ132-1ํ˜ธ ์ €์žฅ โ€˜๊น€์ผ์„ฑ ์žฅ๊ตฐ์˜ ๋…ธ๋ž˜โ€™(ํŒŒ์ผ๋ช… : so-051.wav) ์ œํ•˜ ์Œ์„ฑํŒŒ์ผ ๋‚ด์šฉ ์ถœ๋ ฅ๋ฌผ, ์ฆ์ œ132-1ํ˜ธ ์ €์žฅ โ€˜์ฒœ๋งŒ์ด ์ดํญํƒ„ ๋˜๋ฆฌ๋ผโ€™(ํŒŒ์ผ๋ช… : ์ฒœ๋งŒ์ด_์ดํญํƒ„_๋˜๋ฆฌ๋ผ.mp3) ์ œํ•˜ ์Œ์„ฑํŒŒ์ผ ๋‚ด์šฉ ์ถœ๋ ฅ๋ฌผ, ์ฆ์ œ132-1ํ˜ธ ์ €์žฅ โ€˜์„ธ๊ธฐ์™€ ๋”๋ถˆ์–ด 1๊ถŒ 3์žฅ 1์ ˆโ€™(ํŒŒ์ผ๋ช… : huigo-3-1-01.mp3) ์ œํ•˜ ์Œ์„ฑํŒŒ์ผ ๋‚ด์šฉ ์ถœ๋ ฅ๋ฌผ 1. ์ฆ์ œ122-1ํ˜ธ ํด๋” ๊ตฌ์กฐ Encase ์บก์ณํ™”๋ฉด ์ถœ๋ ฅ๋ฌผ, ์ฆ์ œ128-1ํ˜ธ ์ €์žฅ โ€˜๊ฐœ์„ โ€™(ํŒŒ์ผ๋ช… : ์ด์„œใ€Š๋ถˆ๋ฉธ์˜๋ ฅ์‚ฌใ€‹- ๊ฐœ์„ .pdf) ์ œํ•˜ ํŒŒ์ผ ํด๋”๊ตฌ์กฐ ๋ฐ ์ €์žฅํ˜•ํƒœ Encase ์บก์ณํ™”๋ฉด ์ถœ๋ ฅ๋ฌผ, ์ฆ์ œ128-1ํ˜ธ ์ €์žฅ โ€˜์กฐ์„ ๋กœ๋™๋‹น๋žต์‚ฌโ€™(ํŒŒ์ผ๋ช… : ์กฐ์„ ๋กœ๋™๋‹น๋žต์‚ฌ.HWP) ์ œํ•˜ ํŒŒ์ผ ํด๋”๊ตฌ์กฐ ๋ฐ ์ €์žฅํ˜•ํƒœ Encase ์บก์ณํ™”๋ฉด ์ถœ๋ ฅ๋ฌผ, ์ฆ์ œ128-1ํ˜ธ ์ €์žฅ โ€˜๊น€์ •์ผ ์ €์ž‘์ง‘ ์ œ1๊ถŒโ€™(ํŒŒ์ผ๋ช… : ๊น€์ •์ผ์ €์ž‘์ง‘01.hwp) ์ œํ•˜ ํŒŒ์ผ ํด๋”๊ตฌ์กฐ ๋ฐ ์ €์žฅํ˜•ํƒœ Encase ์บก์ณํ™”๋ฉด ์ถœ๋ ฅ๋ฌผ, ์ฆ์ œ129-1ํ˜ธ ํด๋” ๊ตฌ์กฐ Encase ์บก์ณํ™”๋ฉด ์ถœ๋ ฅ๋ฌผ, ์ฆ์ œ131-1ํ˜ธ ์ €์žฅ โ€˜๋…ธ์„ ๊ณผ ์ •์ฑ… ์ œ1ํ˜ธโ€™(ํŒŒ์ผ๋ช… : NJ-001(20070120).hwp) ์ œํ•˜ ํŒŒ์ผ ํด๋”๊ตฌ์กฐ ๋ฐ ์ €์žฅํ˜•ํƒœ Encase ์บก์ณํ™”๋ฉด ์ถœ๋ ฅ๋ฌผ, ์ฆ์ œ131-1ํ˜ธ ์ €์žฅ โ€˜์ ˆ์„ธ ์œ„์ธ์˜ ๋ช…์–ธโ€™(ํŒŒ์ผ๋ช… : Kim-JI-G_myung.hwp) ์ œํ•˜ ํŒŒ์ผ ํด๋”๊ตฌ์กฐ ๋ฐ ์ €์žฅํ˜•ํƒœ Encase ์บก์ณํ™”๋ฉด ์ถœ๋ ฅ๋ฌผ, ์ฆ์ œ131-1ํ˜ธ ์ €์žฅ โ€˜๋ฏธ๊ตญ์„ ํ˜‘์ƒ๊ณผ ๋Œ€๊ฒฐ์˜ ๊ต์ฐจ์ ์œผ๋กœ ๋Œ์–ด๊ฐ„ ๋ฒ ์ด์ง•ํšŒ๋‹ดโ€™(ํŒŒ์ผ๋ช… : 030517.html) ์ œํ•˜ ํŒŒ์ผ ํด๋”๊ตฌ์กฐ ๋ฐ ์ €์žฅํ˜•ํƒœ Encase ์บก์ณํ™”๋ฉด ์ถœ๋ ฅ๋ฌผ, ์ฆ์ œ132-1ํ˜ธ ์ €์žฅ โ€˜๋ฏผ์กฑ๊ณผ ์ฒ ํ•™โ€™(ํŒŒ์ผ๋ช… : ๋ฏผ์กฑ๊ณผ์ฒ ํ•™.hwp) ์ œํ•˜ ํŒŒ์ผ ํด๋”๊ตฌ์กฐ ๋ฐ ์ €์žฅํ˜•ํƒœ Encase ์บก์ณํ™”๋ฉด ์ถœ๋ ฅ๋ฌผ, ์ฆ์ œ132-1ํ˜ธ ์ €์žฅ โ€˜์„ธ๊ธฐ์™€ ๋”๋ถˆ์–ด 1โ€™(ํŒŒ์ผ๋ช… : ์„ธ๊ธฐ์™€๋”๋ถˆ์–ด1.hwp) ์ œํ•˜ ํŒŒ์ผ ํด๋”๊ตฌ์กฐ ๋ฐ ์ €์žฅํ˜•ํƒœ Encase ์บก์ณํ™”๋ฉด ์ถœ๋ ฅ๋ฌผ, ์ฆ์ œ132-1ํ˜ธ ์ €์žฅ โ€˜์›”๋ฏธ๋„โ€™(ํŒŒ์ผ๋ช… : ์›”๋ฏธ๋„.wmv) ์ œํ•˜ ํŒŒ์ผ ํด๋”๊ตฌ์กฐ ๋ฐ ์ €์žฅํ˜•ํƒœ Encase ์บก์ณํ™”๋ฉด ์ถœ๋ ฅ๋ฌผ, ์ฆ์ œ132-1ํ˜ธ ์ €์žฅ โ€˜์ฒœ๋งŒ์ด ์ดํญํƒ„ ๋˜๋ฆฌ๋ผโ€™(ํŒŒ์ผ๋ช… : ์ฒœ๋งŒ์ด_์ดํญํƒ„_๋˜๋ฆฌ๋ผ.mp3) ์ œํ•˜ ํŒŒ์ผ ํด๋”๊ตฌ์กฐ ๋ฐ ์ €์žฅํ˜•ํƒœ Encase ์บก์ณํ™”๋ฉด ์ถœ๋ ฅ๋ฌผ 1. ์ฆ์ œ129-1ํ˜ธ ์ €์žฅ โ€˜์กฐ์„ ๋กœ๋™๋‹น๋งŒ์„ธโ€™(ํŒŒ์ผ๋ช… : ์กฐ์„ ๋กœ๋™๋‹น๋งŒ์„ธ.SWF) ์ œํ•˜ ๋™์˜์ƒํŒŒ์ผ ์บก์ณํ™”๋ฉด ์ถœ๋ ฅ๋ฌผ, ์ฆ์ œ129-1ํ˜ธ ์ €์žฅ โ€˜๊น€์ผ์„ฑ๋Œ€์›์ˆ˜ ๋งŒ๋งŒ์„ธโ€™(ํŒŒ์ผ๋ช… : ํ™”๋ฉด์Œ์•… ๊น€์ผ์„ฑ๋Œ€์›์ˆ˜ ๋งŒ๋งŒ์„ธ_(360p).flv) ์ œํ•˜ ๋™์˜์ƒํŒŒ์ผ ์บก์ณํ™”๋ฉด ์ถœ๋ ฅ๋ฌผ, ์ฆ์ œ132-1ํ˜ธ ์ €์žฅ โ€˜์‹ฌ์žฅ์— ๋‚จ๋Š” ์‚ฌ๋žŒ 1~2๋ถ€โ€™(ํŒŒ์ผ๋ช… : heart1.wmv~heart2-2.wmv) ์ œํ•˜ ๋™์˜์ƒํŒŒ์ผ ์บก์ณํ™”๋ฉด ์ถœ๋ ฅ๋ฌผ, ์ฆ์ œ132-1ํ˜ธ ์ €์žฅ โ€˜์›”๋ฏธ๋„โ€™(ํŒŒ์ผ๋ช… : ์›”๋ฏธ๋„.wmv) ์ œํ•˜ ๋™์˜์ƒํŒŒ์ผ ์บก์ณํ™”๋ฉด ์ถœ๋ ฅ๋ฌผ 1. Micro SD ์นด๋“œ(16G, ์‚ผ์„ฑEVO, MBMPAGVDDDCW-P, ์ฆ ์ œ122ํ˜ธ), USB(8G, Sandisk Blade Cruzer, ์ฆ ์ œ128ํ˜ธ), USB(32G, DUCO, ์ฆ์ œ129ํ˜ธ), ํ•˜๋“œ๋””์Šคํฌ(40G, โ—‹โ—‹โ—‹, ์ฆ ์ œ131ํ˜ธ), ์‚ผ์„ฑ ์™ธ์žฅํ•˜๋“œ S2(500G, ์ฆ ์ œ132ํ˜ธ) ๋ฐ ์œ„ ๊ฐ ์ฆ๊ฑฐ๋“ค์˜ ์„ ๋ณ„์••์ˆ˜์ด๋ฏธ์ง€(์ฆ ์ œ122-1, 128-1, 129-1, 131-1, 132-1ํ˜ธ)์˜ ๊ฐ ํ˜„์กด [2016๊ณ ํ•ฉ558] 1. ์ฆ์ธ ๊ณต์†Œ์™ธ 23, ๊ณต์†Œ์™ธ 47(2016. 7. 27. ๊ณตํŒ๊ธฐ์ผ), ๊ณต์†Œ์™ธ 14, ๊ณต์†Œ์™ธ 47(2016. 8. 9. ๊ณตํŒ๊ธฐ์ผ), ๊ณต์†Œ์™ธ 19, ๊ณต์†Œ์™ธ 30, ๊ณต์†Œ์™ธ 48, ๊ณต์†Œ์™ธ 12, ๊ณต์†Œ์™ธ 18, ๊ณต์†Œ์™ธ 49, ๊ณต์†Œ์™ธ 50, ๊ณต์†Œ์™ธ 15, ๊ณต์†Œ์™ธ 51, ๊ณต์†Œ์™ธ 11, ๊ณต์†Œ์™ธ 52, ๊ณต์†Œ์™ธ 53, ๊ณต์†Œ์™ธ 28, ๊ณต์†Œ์™ธ 54, ๊ณต์†Œ์™ธ 47(2016. 8. 29. ๊ณตํŒ๊ธฐ์ผ), ๊ณต์†Œ์™ธ 46, ๊ณต์†Œ์™ธ 7, ๊ณต์†Œ์™ธ 55์˜ ๊ฐ ๋ฒ•์ •์ง„์ˆ  1. ์ฆ์ธ ๊ณต์†Œ์™ธ 8์˜ ์ผ๋ถ€ ๋ฒ•์ •์ง„์ˆ  1. ์„œ์šธ์ค‘์•™์ง€๋ฐฉ๋ฒ•์› 2015๊ณ ๋‹จ7843 ์‚ฌ๊ฑด์˜ ์ œ5ํšŒ ๊ณตํŒ์กฐ์„œ ์ค‘ ์ฆ์ธ ๊ณต์†Œ์™ธ 47์ด ํ•œ, ์ œ6ํšŒ ๊ณตํŒ์กฐ์„œ ์ค‘ ์ฆ์ธ ๊ณต์†Œ์™ธ 3์ด ํ•œ, ์ œ7ํšŒ ๊ณตํŒ์กฐ์„œ ์ค‘ ๊ณต์†Œ์™ธ 56, ๊ณต์†Œ์™ธ 57์ด ํ•œ, ์ œ8ํšŒ ๊ณตํŒ์กฐ์„œ ์ค‘ ๊ณต์†Œ์™ธ 58, ๊ณต์†Œ์™ธ 59, ๊ณต์†Œ์™ธ 60, ๊ณต์†Œ์™ธ 31์ด ํ•œ, ์ œ9ํšŒ ๊ณตํŒ์กฐ์„œ ์ค‘ ๊ณต์†Œ์™ธ 46, ๊ณต์†Œ์™ธ 49, ๊ณต์†Œ์™ธ 58, ๊ณต์†Œ์™ธ 10์ด ํ•œ, ์ œ10ํšŒ ๊ณตํŒ์กฐ์„œ ์ค‘ ๊ณต์†Œ์™ธ 17, ๊ณต์†Œ์™ธ 16์ด ํ•œ ๊ฐ ์ง„์ˆ ๊ธฐ์žฌ 1. ์ด ๋ฒ•์›์˜ ์ฆ๊ฑฐ๋ชฉ๋ก ์ˆœ๋ฒˆ 751 ๋‚ด์ง€ 796, 798 ๋‚ด์ง€ 809, 810์— ๋Œ€ํ•œ ๊ฐ ๊ฒ€์ฆ๊ฒฐ๊ณผ 1. ๊ณต์†Œ์™ธ 23์— ๋Œ€ํ•œ ๊ฐ ๊ฒฝ์ฐฐ ์ง„์ˆ ์กฐ์„œ 1. ๊ฐ ์••์ˆ˜์กฐ์„œ ์‚ฌ๋ณธ 1๋ถ€, ์••์ˆ˜๋ชฉ๋ก ์‚ฌ๋ณธ 1๋ถ€, ์••์ˆ˜์กฐ์„œ(๊ณต์†Œ์™ธ 61 ํšŒ์‚ฌ) ์‚ฌ๋ณธ 1๋ถ€, ๊ฒ€์ฆ์กฐ์„œ[2013. 12. 14. (๋ช…์นญ 3 ์ƒ๋žต) ์ปคํ”ผ์ˆ] ์‚ฌ๋ณธ 1๋ถ€, ๊ฒ€์ฆ์กฐ์„œ[2014. 1. 4. (๋ช…์นญ 3 ์ƒ๋žต) ์ปคํ”ผ์ˆ] ์‚ฌ๋ณธ 1๋ถ€, ์••์ˆ˜์กฐ์„œ(์‹ ์ฒดยท์ฃผ๊ฑฐ์ง€ยท์ฐจ๋Ÿ‰ยท์˜คํ† ๋ฐ”์ด) ์‚ฌ๋ณธ 1๋ถ€(*์ฒจ๋ถ€์ž๋ฃŒ ํฌํ•จ), ์••์ˆ˜์กฐ์„œ2(์ „์ž์ •๋ณด๋งค์ฒด์— ๋Œ€ํ•œ ์„ ๋ณ„ ์••์ˆ˜, ์ฃผ๊ฑฐ์ง€ ๋“ฑ) ์‚ฌ๋ณธ 1๋ถ€(*์ฒจ๋ถ€์ž๋ฃŒ ํฌํ•จ), ์••์ˆ˜์กฐ์„œ2(์ „์ž์ •๋ณด๋งค์ฒด์— ๋Œ€ํ•œ ์„ ๋ณ„ ์••์ˆ˜, ์‚ฌ๋ฌด์‹ค) ์‚ฌ๋ณธ 1๋ถ€(*์ฒจ๋ถ€์ž๋ฃŒ ํฌํ•จ), ์••์ˆ˜์กฐ์„œ(โ–ณโ–ณ๋Œ€ ์ œ1์ข…ํ•ฉ๊ด€ ๋กœ๋น„ ์ธํ„ฐ๋„ท ์ปดํ“จํ„ฐ ๋ฐ CCTV์˜์ƒ) ์‚ฌ๋ณธ 1๋ถ€(*์ฒจ๋ถ€์ž๋ฃŒ ํฌํ•จ), ์••์ˆ˜์กฐ์„œ2(์ „์ž์ •๋ณด๋งค์ฒด์— ๋Œ€ํ•œ ์„ ๋ณ„ ์••์ˆ˜, โ–ณโ–ณ๋Œ€) ์‚ฌ๋ณธ 1๋ถ€(*์ฒจ๋ถ€์ž๋ฃŒ ํฌํ•จ), ์••์ˆ˜์กฐ์„œ(2015. 10. 8.์ž โ–ณโ–ณ๋Œ€ ์ œ1์ข…ํ•ฉ๊ด€ ๋กœ๋น„ PC ๋กœ๊ทธ๊ธฐ๋ก ๋ฐ ๊ด€๋ จ CCTV์˜์ƒ ์ž„์˜์ œ์ถœ) ์‚ฌ๋ณธ 1๋ถ€(*์ฒจ๋ถ€์ž๋ฃŒ ํฌํ•จ), ์••์ˆ˜์กฐ์„œ(์™ธ๊ตญ๊ณ„์ด๋ฉ”์ผ) ์‚ฌ๋ณธ 1๋ถ€(*์ฒจ๋ถ€์ž๋ฃŒ ํฌํ•จ), ์••์ˆ˜์กฐ์„œ2(์™ธ๊ตญ๊ณ„์ด๋ฉ”์ผ) ์‚ฌ๋ณธ 1๋ถ€(*์ฒจ๋ถ€์ž๋ฃŒ ํฌํ•จ), ์••์ˆ˜์กฐ์„œ(โ–ณโ–ณ๋Œ€ ์ œ1์ข…ํ•ฉ๊ด€ ๋กœ๋น„ ์ธํ„ฐ๋„ท ์ปดํ“จํ„ฐ ํ•˜๋“œ๋””์Šคํฌ ๋ณต์ œ๋ณธ, *์ฒจ๋ถ€์ž๋ฃŒ ํฌํ•จ), ๊ฐ ์••์ˆ˜์ˆ˜์ƒ‰๊ฒ€์ฆ์˜์žฅ, ๊ฐ ํ†ต์‹ ์ œํ•œ์กฐ์น˜ ํ—ˆ๊ฐ€์„œ, ๊ฐ ํ†ต์‹ ์ œํ•œ์กฐ์น˜ ์ง‘ํ–‰์กฐ์„œ, ๊ฐ ํ†ต์‹ ์ œํ•œ์กฐ์น˜ ๋ฐ ๋Œ€ํ™”๋…น์Œยท์ฒญ์ทจ ํ—ˆ๊ฐ€์„œ 1. 2015. 9. 18. ๊ตญ๋ฆฝ๊ณผํ•™์ˆ˜์‚ฌ์—ฐ๊ตฌ์› ๋ฐœ์†ก โ€˜๊ฐ์ •์˜๋ขฐ ํšŒ๋ณด(2015-M-25783)โ€™ ์ œํ•˜ ๊ณต๋ฌธ 1๋ถ€, 2015. 10. 23. ๊ตญ๋ฆฝ๊ณผํ•™์ˆ˜์‚ฌ์—ฐ๊ตฌ์› ๋ฐœ์†ก โ€˜๊ฐ์ •์˜๋ขฐ ํšŒ๋ณด(2015-M-30015)โ€™ ์ œํ•˜ ๊ณต๋ฌธ 1๋ถ€, ๊ฐ ๋””์ง€ํ„ธ ์ฆ๊ฑฐ๋ถ„์„ ๋ณด๊ณ ์„œ, 2015. 11. 24 ๊ณต์†Œ์™ธ 13 ์ฃผ์‹ํšŒ์‚ฌ ์ž‘์„ฑ ๋ฐ์ดํ„ฐ ๋ฌด๊ฒฐ์„ฑ ์œ ์ง€ ํ™•์ธ์„œ 1๋ถ€, ํ•ด์‰ฌ๊ฐ’ ํ™•์ธ์„œ, 2015. 11. 17 ๊ณต์†Œ์™ธ 13 ์ฃผ์‹ํšŒ์‚ฌ ์ž‘์„ฑ ๋ฐ์ดํ„ฐ ๋ฌด๊ฒฐ์„ฑ ์œ ์ง€ ํ™•์ธ์„œ 1๋ถ€, USB ์ด๋ฏธ์ง€ ๋ถ„์„ ๊ฒฐ๊ณผ ๋ณด๊ณ ์„œ, 2016. 1. 6. ๊ตญ๊ณผ์ˆ˜๋กœ๋ถ€ํ„ฐ ํšŒ์‹ ๋ฐ›์€ ๊ณต๋ฌธ(๊ฐ์ •์˜๋ขฐ ํšŒ๋ณด 2015-M-34296) 1๋ถ€, 2015. 12. 9. ๊ตญ๊ณผ์ˆ˜๋กœ๋ถ€ํ„ฐ ํšŒ์‹ ๋ฐ›์€ ๊ณต๋ฌธ(๊ฐ์ •์˜๋ขฐ ํšŒ๋ณด 2015-M-33366) 1๋ถ€, 2015. 12. 9. ๊ตญ๊ณผ์ˆ˜๋กœ๋ถ€ํ„ฐ ํšŒ์‹ ๋ฐ›์€ ๊ณต๋ฌธ(๊ฐ์ •์˜๋ขฐ ํšŒ๋ณด 2015-M-33367) 1๋ถ€, 2016. 2. 2. ๊ตญ๊ณผ์ˆ˜๋กœ๋ถ€ํ„ฐ ํšŒ์‹ ๋ฐ›์€ ๊ณต๋ฌธ(๊ฐ์ •์˜๋ขฐ ํšŒ๋ณด 2015-M-36834) 1๋ถ€, 2015. 12. 18. (๋ช…์นญ 55 ์ƒ๋žต)์—ฐ๊ตฌ์†Œ ์—ฐ๊ตฌ์†Œ์žฅ ๊ณต์†Œ์™ธ 52 ๊ฐ์ •์„œ 1๋ถ€, ์—ฐ๊ตฌ์†Œ์žฅ ๊ณต์†Œ์™ธ 52์˜ ํŠน์ˆ˜๋ถ„์•ผ ์ „๋ฌธ๊ฐ€ ๊ฒฝ๋ ฅ์นด๋“œ 1๋ถ€, 2016. 1. 14. ๊ตญ๊ณผ์ˆ˜ ๊ณต๋ฌธ(๊ฐ์ •์˜๋ขฐ ํšŒ๋ณด 2015-M-33365) 1๋ถ€, 2016. 2. 3. ๊ตญ๊ณผ์ˆ˜ ๊ณต๋ฌธ(๊ฐ์ •์˜๋ขฐ ์ถ”๊ฐ€ํšŒ๋ณด 2015-M-33365) 1๋ถ€, 2016. 2. 26. ๊ตญ๊ณผ์ˆ˜ ๊ณต๋ฌธ(๊ฐ์ •์˜๋ขฐ ํšŒ๋ณด 2016-M-3140) 1๋ถ€, 2016. 3. 22. ๊ตญ๊ณผ์ˆ˜ ๊ณต๋ฌธ(๊ฐ์ •์˜๋ขฐ ํšŒ๋ณด(2์ฐจ) 2016-M-3140) 1๋ถ€, 2016. 4. 5. ๊ตญ๊ณผ์ˆ˜ ๊ณต๋ฌธ(๊ฐ์ •์˜๋ขฐ ํšŒ๋ณด(3์ฐจ) 2016-M-3140) 1๋ถ€, 2016. 4. 11. ๊ตญ๊ณผ์ˆ˜ ๊ณต๋ฌธ(๊ฐ์ •์˜๋ขฐ ํšŒ๋ณด(4์ฐจ) 2016-M-3140) 1๋ถ€, ๋ฐ์ดํ„ฐ ๋ณต๊ตฌ์ž‘์—… ๋‚ด์—ญ์„œ 1. 2013. 8. 18. (๋ช…์นญ 4 ์ƒ๋žต) PC๋ฐฉ ์‚ฌ์žฅ ๊ณต์†Œ์™ธ 19๋กœ๋ถ€ํ„ฐ ์ž„์˜์ œ์ถœ ๋ฐ›์€ 16๋ฒˆPC ํ•˜๋“œ๋””์Šคํฌ ๋“ฑ์— ๋Œ€ํ•œ ์••์ˆ˜์กฐ์„œ ๋ฐ ๋ชฉ๋ก ์‚ฌ๋ณธ 1๋ถ€, ํ™•์ธ์„œ(2013. 8. 18. ๊ณต์†Œ์™ธ 19) ์‚ฌ๋ณธ 1๋ถ€, ๊ฐ ๋””์ง€ํ„ธ๋งค์ฒด ์›๋ณธยท์‚ฌ๋ณธ ์ž„์˜์ œ์ถœ ๋™์˜์„œ ์‚ฌ๋ณธ 1๋ถ€, ๊ฐ ํ•ด์‹œ๊ฐ’ํ™•์ธ์„œ ์‚ฌ๋ณธ 1๋ถ€, 2013. 8. 21. (๋ช…์นญ 4 ์ƒ๋žต) PC๋ฐฉ ์‚ฌ์žฅ ๊ณต์†Œ์™ธ 19๋กœ๋ถ€ํ„ฐ ์ž„์˜์ œ์ถœ ๋ฐ›์€ CCTV ๋…นํ™”๋ฌผ์— ๋Œ€ํ•œ ์••์ˆ˜์กฐ์„œ ๋ฐ ๋ชฉ๋ก ์‚ฌ๋ณธ 1๋ถ€, ํ™•์ธ์„œ(2013. 8. 21. ๊ณต์†Œ์™ธ 19) ์‚ฌ๋ณธ 1๋ถ€, ๊ฐ โ€˜๋””์ง€ํ„ธ์••์ˆ˜๋ฌผ[(๋ช…์นญ 4 ์ƒ๋žต) PC๋ฐฉ 16๋ฒˆ HDD] ๋ถ„์„โ€™ ๋ณด๊ณ ์„œ 1๋ถ€, 2013. 10. 27. (๋ช…์นญ 4 ์ƒ๋žต) PC๋ฐฉ ์‚ฌ์žฅ ๊ณต์†Œ์™ธ 19๋กœ๋ถ€ํ„ฐ ์ž„์˜์ œ์ถœ ๋ฐ›์€ 16๋ฒˆ PCํ•˜๋“œ๋””์Šคํฌ ๋“ฑ์— ๋Œ€ํ•œ ์••์ˆ˜์กฐ์„œ ๋ฐ ๋ชฉ๋ก ์‚ฌ๋ณธ 1๋ถ€, ํ™•์ธ์„œ(2013. 10. 31. ๊ณต์†Œ์™ธ 19) ์‚ฌ๋ณธ 1๋ถ€, ๊ฐ ๋””์ง€ํ„ธ๋งค์ฒด ์›๋ณธยท์‚ฌ๋ณธ ์ž„์˜์ œ์ถœ ๋™์˜์„œ ์‚ฌ๋ณธ 1๋ถ€, 2013. 10. 31. (๋ช…์นญ 4 ์ƒ๋žต) PC๋ฐฉ ์‚ฌ์žฅ ๊ณต์†Œ์™ธ 19๋กœ๋ถ€ํ„ฐ ์ž„์˜์ œ์ถœ ๋ฐ›์€ CCTV ๋…นํ™”๋ฌผ์— ๋Œ€ํ•œ ์••์ˆ˜์กฐ์„œ ๋ฐ ๋ชฉ๋ก ์‚ฌ๋ณธ 1๋ถ€, 2013. 12. 5. (๋ช…์นญ 4 ์ƒ๋žต) PC๋ฐฉ ์ข…์—…์› ๊ณต์†Œ์™ธ 30์œผ๋กœ๋ถ€ํ„ฐ ์ž„์˜์ œ์ถœ ๋ฐ›์€ 3๋ฒˆ PC ํ•˜๋“œ๋””์Šคํฌ ๋“ฑ์— ๋Œ€ํ•œ ์••์ˆ˜์กฐ์„œ ๋ฐ ๋ชฉ๋ก ์‚ฌ๋ณธ 1๋ถ€, ๊ฐ ํ™•์ธ์„œ(๊ณต์†Œ์™ธ 30) ์‚ฌ๋ณธ 1๋ถ€, 2013. 12. 6. (๋ช…์นญ 4 ์ƒ๋žต) PC๋ฐฉ ์‚ฌ์žฅ ๊ณต์†Œ์™ธ 19๋กœ๋ถ€ํ„ฐ ์ž„์˜์ œ์ถœ ๋ฐ›์€ CCTV ๋…นํ™”๋ฌผ์— ๋Œ€ํ•œ์••์ˆ˜์กฐ์„œ ๋ฐ ๋ชฉ๋ก ์‚ฌ๋ณธ 1๋ถ€, ๊ฐ ํ™•์ธ์„œ(๊ณต์†Œ์™ธ 19) ์‚ฌ๋ณธ 1๋ถ€, โ€˜๋””์ง€ํ„ธ์••์ˆ˜๋ฌผ[(๋ช…์นญ 4 ์ƒ๋žต) PC๋ฐฉ 3๋ฒˆ HDD] ๋ถ„์„โ€™ ๋ณด๊ณ ์„œ 1๋ถ€, 2014. 1. 18. (๋ช…์นญ 4 ์ƒ๋žต) PC๋ฐฉ ์ข…์—…์› ๊ณต์†Œ์™ธ 30์œผ๋กœ๋ถ€ํ„ฐ ์ž„์˜์ œ์ถœ ๋ฐ›์€ 46๋ฒˆ PC ํ•˜๋“œ๋””์Šคํฌ ๋“ฑ์— ๋Œ€ํ•œ ์••์ˆ˜์กฐ์„œ ๋ฐ ๋ชฉ๋ก ์‚ฌ๋ณธ 1๋ถ€, 2014. 1. 21. (๋ช…์นญ 4 ์ƒ๋žต) PC๋ฐฉ ์‚ฌ์žฅ ๊ณต์†Œ์™ธ 19๋กœ๋ถ€ํ„ฐ ์ž„์˜์ œ์ถœ ๋ฐ›์€ CCTV ๋…นํ™”๋ฌผ์— ๋Œ€ํ•œ ์••์ˆ˜์กฐ์„œ ๋ฐ ๋ชฉ๋ก ์‚ฌ๋ณธ 1๋ถ€, โ€˜๋””์ง€ํ„ธ์••์ˆ˜๋ฌผ[(๋ช…์นญ 4 ์ƒ๋žต) PC๋ฐฉ 46๋ฒˆ HDD] ๋ถ„์„โ€™ ๋ณด๊ณ ์„œ 1๋ถ€, 2014. 3. 21 ์••์ˆ˜์กฐ์„œยท์••์ˆ˜๋ชฉ๋กยท์••์ˆ˜๋ฌผ์†Œ์œ ๊ถŒํฌ๊ธฐ์—ฌ๋ถ€ํ™•์ธ์„œยทํ™•์ธ์„œ(CCTV๋…นํ™”์˜์ƒ๋ฌผยทPC๋ฐฉ ์‚ฌ์šฉ๋‚ด์—ญ์„œ ์ž„์˜์ œ์ถœ) ๋“ฑ ์‚ฌ๋ณธ 1๋ถ€, 2014. 3. 21 ์••์ˆ˜์กฐ์„œยท์••์ˆ˜๋ชฉ๋กยท์••์ˆ˜๋ฌผยท์••์ˆ˜๋ฌผ์†Œ์œ ๊ถŒํฌ๊ธฐ์—ฌ๋ถ€ํ™•์ธ์„œยทํ•ด์‹œ๊ฐ’ํ™•์ธ์„œ(ํ•˜๋“œ๋””์Šคํฌ+E75 ์ž„์˜์ œ์ถœ) ๋“ฑ ์‚ฌ๋ณธ 1๋ถ€, 2014. 3. 18. (๋ช…์นญ 4 ์ƒ๋žต) PC๋ฐฉ์—์„œ ์‚ฌ์šฉํ•œ PC ํ•˜๋“œ๋””์Šคํฌ(#31) ๋ถ„์„์„œ 1๋ถ€, 2014. 4. 29. ์••์ˆ˜์กฐ์„œยท์••์ˆ˜๋ชฉ๋กยท์••์ˆ˜๋ฌผ์†Œ์œ ๊ถŒํฌ๊ธฐ์—ฌ๋ถ€ํ™•์ธ์„œยทํ™•์ธ์„œ ๋“ฑ(CCTV๋…นํ™”์˜์ƒ๋ฌผยทPC๋ฐฉ ์‚ฌ์šฉ๋‚ด์—ญ์„œ ์ž„์˜์ œ์ถœ) ์‚ฌ๋ณธ 1๋ถ€, 2014. 6. 5. ์••์ˆ˜์กฐ์„œยท์••์ˆ˜๋ชฉ๋กยท์••์ˆ˜๋ฌผ์†Œ์œ ๊ถŒํฌ๊ธฐ์—ฌ๋ถ€ํ™•์ธ์„œยทํ•ด์‹œ๊ฐ’ํ™•์ธ์„œ, ํ™•์ธ์„œ ๋“ฑ(CCTV๋…นํ™”์˜์ƒ๋ฌผยทPC๋ฐฉ ์‚ฌ์šฉ๋‚ด์—ญ์„œ ์ž„์˜์ œ์ถœ) ์‚ฌ๋ณธ 1๋ถ€, 2014. 6. 12. ์••์ˆ˜์กฐ์„œยท์••์ˆ˜๋ชฉ๋กยท์••์ˆ˜๋ฌผ์†Œ์œ ๊ถŒํฌ๊ธฐ์—ฌ๋ถ€ํ™•์ธ์„œยทํ•ด์‹œ๊ฐ’ํ™•์ธ์„œ, ํ™•์ธ์„œ ๋“ฑ(ํ•˜๋“œ๋””์Šคํฌ ์ž„์˜์ œ์ถœ) ๋“ฑ ์‚ฌ๋ณธ 1๋ถ€, 2014. 6. 12. ์••์ˆ˜์กฐ์„œยท์••์ˆ˜๋ชฉ๋กยท์••์ˆ˜๋ฌผ์†Œ์œ ๊ถŒํฌ๊ธฐ์—ฌ๋ถ€ํ™•์ธ์„œยทํ•ด์‹œ๊ฐ’ํ™•์ธ์„œ(CCTV๋…นํ™”์˜์ƒ๋ฌผยทPC๋ฐฉ ์‚ฌ์šฉ๋‚ด์—ญ์„œ ์ž„์˜์ œ์ถœ) ๋“ฑ ์‚ฌ๋ณธ 1๋ถ€, 2014. 6. 11. (๋ช…์นญ 4 ์ƒ๋žต) PC๋ฐฉ์—์„œ ์‚ฌ์šฉํ•œ PC ํ•˜๋“œ๋””์Šคํฌ(#17) ๋ถ„์„์„œ 1๋ถ€, 2014. 6. 20. ์••์ˆ˜์กฐ์„œยท์••์ˆ˜๋ชฉ๋กยท์••์ˆ˜๋ฌผ์†Œ์œ ๊ถŒํฌ๊ธฐ์—ฌ๋ถ€ํ™•์ธ์„œยทํ™•์ธ์„œยทํ•ด์‹œ๊ฐ’ํ™•์ธ์„œ(ํ•˜๋“œ๋””์Šคํฌ ์ž„์˜์ œ์ถœ) ๋“ฑ ์‚ฌ๋ณธ 1๋ถ€, 2014. 6. 20. ์••์ˆ˜์กฐ์„œยท์••์ˆ˜๋ชฉ๋กยท์••์ˆ˜๋ฌผ์†Œ์œ ๊ถŒํฌ๊ธฐ์—ฌ๋ถ€ํ™•์ธ์„œยทํ•ด์‹œ๊ฐ’ํ™•์ธ์„œ(CCTV๋…นํ™”์˜์ƒ๋ฌผยทPC๋ฐฉ ์‚ฌ์šฉ๋‚ด์—ญ์„œ ์ž„์˜์ œ์ถœ) ๋“ฑ ์‚ฌ๋ณธ 1๋ถ€, 2014. 6. 20. (๋ช…์นญ 4 ์ƒ๋žต) PC๋ฐฉ์—์„œ ์‚ฌ์šฉํ•œ PC ํ•˜๋“œ๋””์Šคํฌ(#18) ๋ถ„์„์„œ 1๋ถ€, 2014. 7. 14. ์••์ˆ˜์กฐ์„œยท์••์ˆ˜๋ชฉ๋กยท์••์ˆ˜๋ฌผ์†Œ์œ ๊ถŒํฌ๊ธฐ์—ฌ๋ถ€ํ™•์ธ์„œยทํ™•์ธ์„œยทํ•ด์‹œ๊ฐ’ํ™•์ธ์„œ(ํ•˜๋“œ๋””์Šคํฌ ์ž„์˜์ œ์ถœ) ๋“ฑ ์‚ฌ๋ณธ 1๋ถ€, 2014. 7. 14. ์••์ˆ˜์กฐ์„œยท์••์ˆ˜๋ชฉ๋กยท์••์ˆ˜๋ฌผ์†Œ์œ ๊ถŒํฌ๊ธฐ์—ฌ๋ถ€ํ™•์ธ์„œยทํ•ด์‹œ๊ฐ’ํ™•์ธ์„œ(CCTV๋…นํ™”์˜์ƒ๋ฌผยทPC๋ฐฉ ์‚ฌ์šฉ๋‚ด์—ญ์„œ ์ž„์˜์ œ์ถœ) ๋“ฑ ์‚ฌ๋ณธ 1๋ถ€, 2014. 7. 12. (๋ช…์นญ 4 ์ƒ๋žต) PC๋ฐฉ์—์„œ ์‚ฌ์šฉํ•œ PC ํ•˜๋“œ๋””์Šคํฌ(#31) ๋ถ„์„์„œ 1๋ถ€, 2014. 9. 2. ์••์ˆ˜์กฐ์„œยท์••์ˆ˜๋ชฉ๋กยท์••์ˆ˜๋ฌผ์†Œ์œ ๊ถŒํฌ๊ธฐ์—ฌ๋ถ€ํ™•์ธ์„œยทํ•ด์‹œ๊ฐ’ํ™•์ธ์„œ(CCTV๋…นํ™”์˜์ƒ๋ฌผยทPC๋ฐฉ ์‚ฌ์šฉ๋‚ด์—ญ์„œ ์ž„์˜์ œ์ถœ) ๋“ฑ ์‚ฌ๋ณธ 1๋ถ€, 2015. 3. 17. ์••์ˆ˜์กฐ์„œยท์••์ˆ˜๋ชฉ๋กยท์••์ˆ˜๋ฌผ์†Œ์œ ๊ถŒํฌ๊ธฐ์—ฌ๋ถ€ํ™•์ธ์„œยทํ™•์ธ์„œยทํ•ด์‹œ๊ฐ’ํ™•์ธ์„œ(ํ•˜๋“œ๋””์Šคํฌ ์ž„์˜์ œ์ถœ) ๋“ฑ ์‚ฌ๋ณธ 1๋ถ€, 2015. 3. 16. (๋ช…์นญ 4 ์ƒ๋žต) PC๋ฐฉ์—์„œ ์‚ฌ์šฉํ•œ PC ํ•˜๋“œ๋””์Šคํฌ(#16) ๋ถ„์„์„œ 1๋ถ€, 2015. 3. 25. ์••์ˆ˜์กฐ์„œยท์••์ˆ˜๋ชฉ๋กยท์••์ˆ˜๋ฌผ์†Œ์œ ๊ถŒํฌ๊ธฐ์—ฌ๋ถ€ํ™•์ธ์„œยทํ™•์ธ์„œยทํ•ด์‹œ๊ฐ’ํ™•์ธ์„œ(ํ•˜๋“œ๋””์Šคํฌ ์ž„์˜์ œ์ถœ) ๋“ฑ ์‚ฌ๋ณธ 1๋ถ€, 2015. 3. 24. (๋ช…์นญ 4 ์ƒ๋žต) PC๋ฐฉ์—์„œ ์‚ฌ์šฉํ•œ PC ํ•˜๋“œ๋””์Šคํฌ(#45) ๋ถ„์„์„œ 1๋ถ€, 2015. 4. 10. ์••์ˆ˜์กฐ์„œยท์••์ˆ˜๋ชฉ๋กยท์••์ˆ˜๋ฌผ์†Œ์œ ๊ถŒํฌ๊ธฐ์—ฌ๋ถ€ํ™•์ธ์„œยทํ™•์ธ์„œยทํ•ด์‹œ๊ฐ’ํ™•์ธ์„œ(ํ•˜๋“œ๋””์Šคํฌ ์ž„์˜์ œ์ถœ) ๋“ฑ ์‚ฌ๋ณธ 1๋ถ€, 2015. 4. 10. (๋ช…์นญ 4 ์ƒ๋žต) PC๋ฐฉ์—์„œ ์‚ฌ์šฉํ•œ PC ํ•˜๋“œ๋””์Šคํฌ(#19) ๋ถ„์„์„œ 1๋ถ€ 1. ์ˆ˜์‚ฌ๋ณด๊ณ (์ฆ๊ฑฐ๋ชฉ๋ก ์ˆœ๋ฒˆ 310, ํ”ผ๊ณ ์ธ์ด 2011. 4. 19.~23.๊ฐ„ ์ค‘๊ตญ ๋ฐฉ๋ฌธ, ์ƒ๋ถ€์„ ์ธ โ˜†์ง€๋„์› ๋“ฑ ๋ถ ใ€Œ225๊ตญใ€ ๊ณต์ž‘์กฐ์™€ ํšŒํ•ฉํ•œ ์‚ฌ์‹ค ํ™•์ธ), ์ˆ˜์‚ฌ๋ณด๊ณ (์ฆ๊ฑฐ๋ชฉ๋ก ์ˆœ๋ฒˆ 317, ํ”ผ๊ณ ์ธ์ด 2012. 5. 29.~6. 3.๊ฐ„ ๋ฒ ํŠธ๋‚จ ๋ฐฉ๋ฌธ, ์ƒ๋ถ€์„ ์ธ โ˜†์ง€๋„์› ๋“ฑ ๋ถ ใ€Œ225๊ตญใ€ ๊ณต์ž‘์กฐ์™€ ํšŒํ•ฉํ•œ ์‚ฌ์‹ค ํ™•์ธ), ์ˆ˜์‚ฌ๋ณด๊ณ (์ฆ๊ฑฐ๋ชฉ๋ก ์ˆœ๋ฒˆ 331, ๋Œ€์ƒ์ž ํ”ผ๊ณ ์ธ์ด 2013. 3. 20.~23.๊ฐ„ ๋ฒ ํŠธ๋‚จ ๋ฐฉ๋ฌธ ์‚ฌ์‹ค ๋ฐ ๊ท€๊ตญ์‹œ ์ถœ์ฒ˜๋ฏธ์ƒ ๊ธˆ์•ก์„ ๊ตญ๋‚ด๋ฐ˜์ž…ํ•œ ์‚ฌ์‹ค ํ™•์ธ), ์ˆ˜์‚ฌ๋ณด๊ณ (์ฆ๊ฑฐ๋ชฉ๋ก ์ˆœ๋ฒˆ 347, ๋Œ€์ƒ์ž์˜ ์—ฐ๊ณ„ํ˜์˜์ž ๊ณต์†Œ์™ธ 6์ด 2015. 4. ๋ง๋ ˆ์ด์‹œ์•„ ์ฟ ์•Œ๋ผ๋ฃธํ‘ธ๋ฅด ๋ฉ”๋ฅด๋ฐ์นด ๊ด‘์žฅ ๋“ฑ์—์„œ ๋ถ ๊ณต์ž‘์กฐ์ง ใ€Œ225๊ตญใ€ ์†Œ์† ๋Œ€๋‚จ ๊ณต์ž‘์›๊ณผ ๋น„๋ฐ€ํšŒํ•ฉํ•œ ์‚ฌ์‹ค ํ™•์ธ), ์ˆ˜์‚ฌ๋ณด๊ณ (์ฆ๊ฑฐ๋ชฉ๋ก ์ˆœ๋ฒˆ 397, ํ”ผ๊ณ ์ธ์ด 2014. 4.~2015. 8.๊ฐ„ PC๋ฐฉ์— ์ถœ์ž…ํ•˜๋ฉฐ ์ค‘๊ตญ๊ณ„ ์ด๋ฉ”์ผ โ€˜โ–ฒโ–ฒโ–ฒโ–ฒ.net'์— ์ ‘์†ํ•œ ์‚ฌ์‹ค ํ™•์ธ), ์ˆ˜์‚ฌ๋ณด๊ณ (์ฆ๊ฑฐ๋ชฉ๋ก ์ˆœ๋ฒˆ 514, ํ”ผ๊ณ ์ธ์ด 2014. 10. 5. ์—ฐ๊ณ„ํ˜์˜์ž ๊ณต์†Œ์™ธ 6์—๊ฒŒ ๋ณธ์ธ ์†Œ์ง€ ํƒœ๋ธ”๋ฆฟPC์— ์ €์žฅ๋œ 2014. 10. 3.์ž ๋ถ โ€˜๋…ธ๋™์‹ ๋ฌธโ€™ ๊ธฐ์‚ฌ๋ฅผ ํ™œ์šฉํ•˜์—ฌ ์‚ฌ์ƒ๊ต์–‘ํ•™์Šต์„ ์‹œํ‚จ ์‚ฌ์‹ค ํ™•์ธ) 1. โ€˜๋…น์ทจ๋ก-(๋ช…์นญ 29 ์ƒ๋žต)์—ฐ๊ตฌ์†Œ ๋Œ€ํ™”๋‚ด์šฉ(2013. 10. 22.), ๋…น์ทจ๋ก-(๋ช…์นญ 29 ์ƒ๋žต)์—ฐ๊ตฌ์†Œ ๋Œ€ํ™”๋‚ด์šฉ(2013. 11. 4.), ๋…น์ทจ๋ก-์„ธ๋ฅ˜์—ญ (๋ช…์นญ 3 ์ƒ๋žต) ํ”ผ๊ณ ์ธยท๊ณต์†Œ์™ธ 5 ๋Œ€ํ™”๋‚ด์šฉ(2013. 12. 14.), ๋…น์ทจ๋ก-์„ธ๋ฅ˜์—ญ (๋ช…์นญ 3 ์ƒ๋žต) ํ”ผ๊ณ ์ธยท๊ณต์†Œ์™ธ 5 ๋Œ€ํ™”๋‚ด์šฉ(2014. 1. 4.), ๋…น์ทจ๋ก-ํ•œ๊ตญ(๋ช…์นญ 29 ์ƒ๋žต)์—ฐ๊ตฌ์†Œ ํšŒํ•ฉ(2014. 1. 22.), ๋…น์ทจ๋ก-๊ฐ•๋ณ€์—ญ (๋ช…์นญ 56 ์ƒ๋žต) ํ”ผ๊ณ ์ธยท๊ณต์†Œ์™ธ 6 ๋Œ€ํ™”๋‚ด์šฉ(2014. 2. 2.), ๋…น์ทจ๋ก-(๋ช…์นญ 29 ์ƒ๋žต)์—ฐ๊ตฌ์†Œ ํ”ผ๊ณ ์ธยท๊ณต์†Œ์™ธ 4 ๋Œ€ํ™”๋‚ด์šฉ(2014. 2. 3.), ๋…น์ทจ๋ก-(๋ช…์นญ 29 ์ƒ๋žต)์—ฐ๊ตฌ์†Œ ํ”ผ๊ณ ์ธยท๊ณต์†Œ์™ธ 4 ๋Œ€ํ™”๋‚ด์šฉ(2014. 4. 28.), ๋…น์ทจ๋ก-์ž ์‹ค์—ญ (๋ช…์นญ 49 ์ƒ๋žต) ํ”ผ๊ณ ์ธยท๊ณต์†Œ์™ธ 6 ๋Œ€ํ™”๋‚ด์šฉ(2015. 3. 8.), ๋…น์ทจ๋ก-์ข…๊ฐ์—ญ (๋ช…์นญ 50 ์ƒ๋žต) ํ”ผ๊ณ ์ธยท๊ณต์†Œ์™ธ 6 ๋Œ€ํ™”๋‚ด์šฉ(2015. 3. 30.), ๋…น์ทจ๋ก-์ž ์‹ค (๋ช…์นญ 49 ์ƒ๋žต) ํ”ผ๊ณ ์ธยท๊ณต์†Œ์™ธ 6 ๋Œ€ํ™”๋‚ด์šฉ(2015. 11. 1.) 1. ๊ณต์†Œ์™ธ 5 ํ•ญ์†Œ์‹ฌํŒ๊ฒฐ๋ฌธ(93๋…ธ935) ์‚ฌ๋ณธ 1๋ถ€, 1993. 2. 24. ์„œ์šธํ˜•์‚ฌ์ง€๋ฒ• ์„ ๊ณ  92๊ณ ํ•ฉ1781 ํŒ๊ฒฐ๋ฌธ(ํ”ผ๊ณ ์ธ ๊ณต์†Œ์™ธ 62) ์‚ฌ๋ณธ 1๋ถ€, 1993. 2. 23. ์„œ์šธํ˜•์‚ฌ์ง€๋ฒ• ์„ ๊ณ  92๊ณ ํ•ฉ1788 ํŒ๊ฒฐ๋ฌธ(ํ”ผ๊ณ ์ธ ๊ณต์†Œ์™ธ 5) ์‚ฌ๋ณธ 1๋ถ€, 2015. 11. 6.์ž ํ†ต์ผ๋ถ€๋กœ๋ถ€ํ„ฐ ๋ฐ›์€ ์—…๋ฌดํ˜‘์กฐ ํšŒ์‹  ๊ณต๋ฌธ(์‚ฌํšŒ๋ฌธํ™”๊ต๋ฅ˜๊ณผ-1202) 1๋ถ€, ํ”ผ๊ณ ์ธ ์ถœ์ž…๊ตญ ์ž๋ฃŒ 1๋ถ€, 2015. 11. 2. ๊ฒฝ์ฐฐ์ฒญ ๋ฐœ์†ก โ€˜์ˆ˜์‚ฌํ˜‘์ฃ„์˜๋ขฐโ€™ ์ œํ•˜ ๊ณต๋ฌธ(๊ฒฝ์ฐฐ์ฒญ ๋ณด์•ˆ3๊ณผ-2631) ์Šค์บ”๋ณธ 1๋ถ€, 2015. 11. 2. ์„œ๋Œ€๋ฌธ๊ตฌ ๋ฐœ์†ก โ€˜์ˆ˜์‚ฌ ํ˜‘์กฐ ํšŒ์‹ โ€™ ์ œํ•˜ ๊ณต๋ฌธ(๊ตํ†ตํ–‰์ •๊ณผ-48351) ์Šค์บ”๋ณธ 1๋ถ€, โ—€โ—€์ž๋™์ฐจ(โ–ฃโ–ฃ๋Œ€๋ฆฌ์ )์— ๋ณด๋‚ธ ๊ฒฝ์ฐฐ์ฒญ ๊ณต๋ฌธ(๋ณด์•ˆ3๊ณผ-2887) 1๋ถ€, ์ž๋™์ฐจ ๋“ฑ๋ก์ฆ(์ œ201511-002845ํ˜ธ) ์‚ฌ๋ณธ 1๋ถ€, ์ž๋™์ฐจ ๋งค๋งค๊ณ„์•ฝ์„œ(ํšŒ์‚ฌ์šฉ) ์‚ฌ๋ณธ 1๋ถ€, ์ถœ๊ณ ํ’ˆ์˜๊ด€๋ฆฌ์„œ ์‚ฌ๋ณธ 1๋ถ€, โ€˜๊ณต์†Œ์™ธ 63 ํšŒ์‚ฌโ€™ ์ž๋™์ฐจ ์˜คํ† ๋ก (๋Œ€์ถœ)์‹ ์ฒญ์„œยท์•ฝ์ •์„œ, ๊ธˆ์œต๊ฑฐ๋ž˜ํ™•์ธ์„œ, ๋Œ€์ถœ๊ฑฐ๋ž˜ ๋ฐ ์ž”์•ก์ฆ๋ช…์„œ, ์ฃผ์š”์•ฝ์ •๋‚ด์šฉ ์•ˆ๋‚ด ์‚ฌ๋ณธ ๊ฐ 1๋ถ€, ใ€Œ๋ฏผํ˜๋‹นใ€ ๋Œ€๋ฒ•์› ํŒ๊ฒฐ๋ฌธ 2000๋„2965 ์‚ฌ๋ณธ 1๋ถ€, ใ€Œ์™•์žฌ์‚ฐใ€ 1์‹ฌ ํŒ๊ฒฐ๋ฌธ 2011๊ณ ํ•ฉ1131 ์‚ฌ๋ณธ 1๋ถ€, ใ€ŒROใ€ 1์‹ฌ ํŒ๊ฒฐ๋ฌธ 2013๊ณ ํ•ฉ620 ์‚ฌ๋ณธ 1๋ถ€, ๊ฐ ๋Œ€์ƒ์ž ํ”ผ๊ณ ์ธ ์ถœ์ž…๊ตญ ๊ธฐ๋ก 1๋ถ€, ์—ฐ๊ณ„ํ˜์˜์ž ๊ณต์†Œ์™ธ 6 ์ถœ์ž…๊ตญ ๊ธฐ๋ก 1๋ถ€, ๊ณต์†Œ์™ธ 6์— ๋Œ€ํ•œ ์™ธ๊ตญํ™˜๊ฑฐ๋ž˜๋ฒ•์œ„๋ฐ˜ ๊ณต์†Œ์žฅ, ๊ณต์†Œ์™ธ 6 ์™ธ๊ตญํ™˜๊ฑฐ๋ž˜๋ฒ•์œ„๋ฐ˜ ์‚ฌ๊ฑด ์••์ˆ˜๋ฌผ ์ด๋ชฉ๋ก, ๊ณต์†Œ์™ธ 6 ์™ธ๊ตญํ™˜๊ฑฐ๋ž˜๋ฒ•์œ„๋ฐ˜ ์ธ์ง€๋ณด๊ณ ์„œ, ๊ณต์†Œ์™ธ 6 ๋ฒ”์ฃ„๊ฒฝ๋ ฅ ์กฐํšŒ 1. 2013. 3. 23 ๋Œ€์ƒ์ž์˜ ์™ธํ™˜์€ํ–‰ ํ™˜์ „์ž๋ฃŒ ์‚ฌ๋ณธ 1๋ถ€, 2013. 3. 23 ๋Œ€์ƒ์ž์˜ ํ•˜๋‚˜์€ํ–‰ ํ™˜์ „์ž๋ฃŒ ์‚ฌ๋ณธ 1๋ถ€, 2014. 1. 22.์ž โ€˜์ˆ˜์‚ฌํ˜‘์กฐ ์˜๋ขฐ ๊ฒฐ๊ณผโ€™ ์ œํ•˜ KT๊ณต๋ฌธ 1๋ถ€, โ–ณโ–ณ๋Œ€ํ•™๊ต ์ œ1์ข…ํ•ฉ๊ด€ ์ „์‚ฐ์ •๋ณดํŒ€ ๊ณต์†Œ์™ธ 31์ด ์ž„์˜์ œ์ถœํ•œ โ–ณโ–ณ๋Œ€ํ•™๊ต ์ œ1์ข…ํ•ฉ๊ด€ ๋กœ๋น„ PC ๋กœ๊ทธ๊ธฐ๋ก ์ถœ๋ ฅ๋ฌผ 1๋ถ€, (๋ช…์นญ 20 ์ƒ๋žต) ๊ฐ€์ž…๋‚ด์—ญ ์ถœ๋ ฅ๋ฌผ 1๋ถ€, (๋ช…์นญ 21 ์ƒ๋žต) ๊ฐ€์ž…๋‚ด์—ญ ์ถœ๋ ฅ๋ฌผ 1๋ถ€, โ™ฃโ™ฃ ์ธ์ ์‚ฌํ•ญ ์ถœ๋ ฅ๋ฌผ 1๋ถ€, ํ”ผ์˜์ž ๊ฐ€์ž… (๋ช…์นญ 22 ์ƒ๋žต)_โ™ฃโ™ฃ์นดํŽ˜ํšŒ์›์ธ์ ์‚ฌํ•ญ ์ถœ๋ ฅ๋ฌผ 1๋ถ€, โ™ฃโ™ฃโ™งโ™งโ™ง ์ธ์ ์‚ฌํ•ญ ์ถœ๋ ฅ๋ฌผ 1๋ถ€, 2015. 11. 26.์ž โ€˜๊ธˆ์œต๊ฑฐ๋ž˜์ •๋ณด ์ œ๊ณต์š”๊ตฌ์— ๋Œ€ํ•œ ํšŒ์‹ โ€™ ์ œํ•˜ ๊ตญ๋ฏผ์€ํ–‰ ํšŒ์‹  ๊ณต๋ฌธ(์—…๋ฌด์ง€์›์„ผํ„ฐ-096513) ๊ณต๋ฌธ ํ‘œ์ง€ 1๋ถ€, KB๊ตญ๋ฏผ์€ํ–‰ ์ œ๊ณต, ํ”ผ๊ณ ์ธ ์™ธํ™˜๊ฑฐ๋ž˜๋‚ด์—ญ ์ถœ๋ ฅ๋ฌผ 1๋ถ€, KB๊ตญ๋ฏผ์€ํ–‰ ์ œ๊ณต, ๊ณต์†Œ์™ธ 8 ์™ธํ™˜๊ฑฐ๋ž˜๋‚ด์—ญ ์ถœ๋ ฅ๋ฌผ 1๋ถ€, KB๊ตญ๋ฏผ์€ํ–‰ ํ™ˆํŽ˜์ด์ง€๋‚ด โ€˜์ง€์ ์•ˆ๋‚ด - ์ง€์ ๊ฒ€์ƒ‰โ€™ ์ถœ๋ ฅ๋ฌผ 1๋ถ€, 2015. 4. 14.์ž ๊ณต์†Œ์™ธ 64 ๋ช…์˜ ํ™˜์ „ ๋งค๋งค ๊ฑฐ๋ž˜ํ‘œ(๋ฏธํ™” 2,200๋ถˆ ๋งค๋„) ์Šค์บ” ์ถœ๋ ฅ๋ฌผ 1๋ถ€, 2015. 7. 15.์ž ๊ณต์†Œ์™ธ 64 ๋ช…์˜ ํ™˜์ „ ๋งค๋งค ๊ฑฐ๋ž˜ํ‘œ(๋ฏธํ™” 5,000๋ถˆ ๋งค๋„) ์Šค์บ” ์ถœ๋ ฅ๋ฌผ 1๋ถ€, ํ”ผ๊ณ ์ธ ์†Œ์ง€ โ€˜์•„์ดํŒจ๋“œ ์—์–ด2โ€™(์ฆ์ œ135ํ˜ธ)์˜ ์ธํ„ฐ๋„ท ๊ฒ€์ƒ‰ ๊ธฐ๋ก 1๋ถ€ 1. ์‚ผ์„ฑ ์™ธ์žฅํ•˜๋“œ S2(์ฆ์ œ132ํ˜ธ)์—์„œ ๋ฐœ๊ฒฌํ•œ โ€˜์ฃผ์ฒด์˜ ํ•œ๊ตญ์‚ฌํšŒ ๋ณ€ํ˜์šด๋™๋ก โ€™(์ €์žฅ๊ฒฝ๋กœ : C\\2 ์ž๋ฃŒ๋ชจ์Œ\\๊ฐœ์ •์ž‘์—…\\2๋‹จ๊ณ„-๋ณ€ํ˜์ด๋ก 4๋Œ€๊ฐœ๋ก \\4๋Œ€๊ฐœ๋ก ์„œ\\์ฃผ์ฒด์˜ํ•œ๊ตญ์‚ฌํšŒ๋ณ€ํ˜์šด๋™๋ก .HWP) ์ถœ๋ ฅ๋ฌผ 1๋ถ€, 2015kijo.hwp ํŒŒ์ผ ์ถœ๋ ฅ๋ฌผ 1๋ถ€, ํ”ผ์˜์ž ์˜คํ† ๋ฐ”์ด์—์„œ ์••์ˆ˜ํ•œ โ€˜์‚ผ์„ฑ ์™ธ์žฅํ•˜๋“œ S2โ€™(์ฆ์ œ132ํ˜ธ) ๋‚ด โ€˜2013๋…„ ํ•˜๋ฐ˜๊ธฐ 11์›”~12์›” ๊ต์–‘์ž๋ฃŒโ€™ ๋ฌธ๊ฑด ์ถœ๋ ฅ๋ฌผ 1๋ถ€ (3811), โ€˜์กฐ์„ ๋ฏผ์ฃผ์ฃผ์˜์ธ๋ฏผ๊ณตํ™”๊ตญ์€ ๋ถˆํŒจ์˜ ์œ„๋ ฅ์„ ์ง€๋‹Œ ์ฃผ์ฒด์˜ ์‚ฌํšŒ์ฃผ์˜๊ตญ๊ฐ€์ด๋‹คโ€™ ์ œํ•˜๋ฌธ๊ฑด ์ถœ๋ ฅ๋ฌผ 1๋ถ€, โ€˜21์„ธ๊ธฐ ์‚ฌํšŒ๋ณ€ํ˜์šด๋™๊ณผ ์ง„๋ณด์  ์ •๊ถŒ๊ต์ฒด: ๋ฐ˜์„ฑ๊ณผ ์ „๋งโ€™ ์ œํ•˜ ๋ฌธ๊ฑด 1๋ถ€, ํ”ผ๊ณ ์ธ์ด ํ•˜๋ถ€๋ง ๊ณต์†Œ์™ธ 5์—๊ฒŒ ํƒœ๋ธ”๋ฆฟPC๋ฅผ ํ†ตํ•˜์—ฌ ํ•˜๋‹ฌํ•œ ์‚ฌ์ƒ๊ต์–‘์ž๋ฃŒ ๋‚ด์šฉ(2013. 12. 14.), 2013. 12. 9.์ž โ€˜์กฐ์„ ๋กœ๋™๋‹น ์ค‘์•™์œ„์›ํšŒ ์ •์น˜๊ตญ ํ™•๋Œ€ํšŒ์˜์— ๊ด€ํ•œ ๋ณด๋„โ€™ ์ œํ•˜ ์กฐ์„ ์ค‘์•™ํ†ต์‹  ๋ฌธ๊ฑด ์ถœ๋ ฅ๋ฌผ 1๋ถ€, 2013. 12. 9.์ž โ€˜์กฐ์„ ๋กœ๋™๋‹น ์ค‘์•™์œ„์›ํšŒ ์ •์น˜๊ตญ ํ™•๋Œ€ํšŒ์˜์— ๊ด€ํ•œ ๋ณด๋„โ€™ ์ œํ•˜ ๊ธฐ์‚ฌ๊ฐ€ ํฌํ•จ๋œ ๋…ธ๋™์‹ ๋ฌธ ์›๋ฌธ ์ถœ๋ ฅ๋ฌผ 1๋ถ€, 2013. 12. 10.์ž โ€˜ใ€ˆ์‚ฌ์„คใ€‰๊ฒฝ์• ํ•˜๋Š” ๊น€์ •์€ ๋™์ง€์˜ ๋‘๋ฆฌ์— ์ฒ ํ†ต๊ฐ™์ด ๋ญ‰์ณ ์ฃผ์ฒดํ˜๋ช…์˜ ํ•œ๊ธธ๋กœ ์–ต์„ธ๊ฒŒ ๋‚˜์•„๊ฐ€์žโ€™ ์ œํ•˜ ๊ธฐ์‚ฌ๊ฐ€ ํฌํ•จ๋œ ๋…ธ๋™์‹ ๋ฌธ ์›๋ฌธ ์ถœ๋ ฅ๋ฌผ 1๋ถ€, 2013. 12. 13.์ž โ€˜์กฐ์„ ๋กœ๋™๋‹น์€ ์ผ์‹ฌ๋‹จ๊ฒฐ์˜ ์œ„๋ ฅ์œผ๋กœ ์ „์ง„ํ•˜๋Š” ๋ถˆํŒจ์˜ ํ˜๋ช…์ ๋‹น์ด๋‹คโ€™ ์ œํ•˜ ๊ธฐ์‚ฌ ๋ฐ โ€˜์ฒœ๋งŒ๊ตฐ๋ฏผ์˜ ์น˜์†Ÿ๋Š” ๋ถ„๋…ธ์˜ ํญ๋ฐœ. ๋งŒ๊ณ ์—ญ์  ๋‹จํ˜ธํžˆ ์ฒ˜๋‹จโ€™ ์ œํ•˜ ๊ธฐ์‚ฌ ๋“ฑ์ด ํฌํ•จ๋œ ๋…ธ๋™์‹ ๋ฌธ ์›๋ฌธ ์ถœ๋ ฅ๋ฌผ 1๋ถ€, ํ”ผ์˜์ž ์˜คํ† ๋ฐ”์ด์—์„œ ์••์ˆ˜ํ•œ โ€˜์‚ผ์„ฑ ์™ธ์žฅํ•˜๋“œ S2โ€™(์ฆ์ œ132ํ˜ธ) ๋‚ดโ€˜JSTโ€™์ œํ•˜ ๋ฌธ๊ฑด ์ถœ๋ ฅ๋ฌผ 1๋ถ€, ํ”ผ๊ณ ์ธ์ด ํ•˜๋ถ€๋ง ๊ณต์†Œ์™ธ 5์—๊ฒŒ ํƒœ๋ธ”๋ฆฟPC๋ฅผ ํ†ตํ•˜์—ฌ ํ•˜๋‹ฌํ•œ ์‚ฌ์ƒ๊ต์–‘์ž๋ฃŒ ๋‚ด์šฉ(2014. 1. 4.), 2014. 1. 1.์ž โ€˜์‹ ๋…„์‚ฌโ€™ ์ œํ•˜ ๋ถ ๋…ธ๋™์‹ ๋ฌธ ์›๋ฌธ ์ถœ๋ ฅ๋ฌผ 1๋ถ€, 2014. 1. 4.์ž โ€˜์ผ์‹ฌ๋‹จ๊ฒฐ์˜ ์œ„๋ ฅ ๋–จ์น˜๋ฉฐ ์กฐ๊ตญ์ด์—ฌ ์•ž์œผ๋กœ!โ€™ ์ œํ•˜ ๋ถ ๋…ธ๋™์‹ ๋ฌธ ์›๋ฌธ ์ถœ๋ ฅ๋ฌผ 1๋ถ€, 2014. 1. 4.์ž โ€˜ใ€ˆ์ •๋ก ใ€‰์Šน๋ฆฌ๋ฅผ ํ–ฅํ•˜์—ฌ ์ง„๊ตฐ ๋˜ ์ง„๊ตฐโ€™ ์ œํ•˜ ๋ถ ๋…ธ๋™์‹ ๋ฌธ ์›๋ฌธ ์ถœ๋ ฅ๋ฌผ 1๋ถ€, 2014. 10. 3.์ž โ€˜๋ฐฑ๋‘๋ น ๋”ฐ๋ผ ์ตœํ›„ ์Šน๋ฆฌ์˜ ์ถ•ํฌ์„ฑ ํ„ฐ์ณ ์˜ฌ๋ฆฌ๋ฆฌ๋ผโ€™ ์ œํ•˜ ๋…ธ๋™์‹ ๋ฌธ ๊ธฐ์‚ฌ ์ถœ๋ ฅ๋ฌผ 1๋ถ€, โ€˜์ •์„ธ๋ถ„์„ ๋ณด๊ณ ์„œ.hwpโ€™ ์ œํ•˜ ํŒŒ์ผ ์ถœ๋ ฅ๋ฌผ 1๋ถ€, ํ”ผ๊ณ ์ธ ์†Œ์ง€ ์ฃผ๊ฑฐ์ง€์—์„œ ์••์ˆ˜ํ•œ USB(์ฆ์ œ129ํ˜ธ)๋‚ด ์ €์žฅ๋œ ํŒŒ์ผ โ€˜daesastudy2-1.hwpโ€™ ๋ฌธ๊ฑด ์ถœ๋ ฅ๋ฌผ 1๋ถ€, โ€˜์กฐ์ง๊ด€.hwp' ์ œํ•˜ ํŒŒ์ผ ์ถœ๋ ฅ๋ฌผ 1๋ถ€ 1. ์ฆ์ œ126ํ˜ธ USB๋‚ด โ€˜โ—ˆ๊ตญ์™ธ(๊ตญ๋‚ด)์—ฌํ–‰์‹ ์ฒญ ๋ฐ ๊ณ„์•ฝ์„œโ—ˆโ€™ ์ œํ•˜ ์Šคํ…Œ๊ฐ€๋…ธ๊ทธ๋ผํ”ผ ๋ฌธ์„œํŒŒ์ผ(info.docx) ์ถœ๋ ฅ๋ฌผ 1๋ถ€, ์ฆ์ œ126ํ˜ธ USB๋‚ด ์Šคํ…Œ๊ฐ€๋…ธ๊ทธ๋ผํ”ผ ํŒŒ์ผ(info.docx)์„ ๋ณตํ˜ธํ™”ํ•œ ๋ถํ•œ ์ง€๋ น๋ฌธ(2011. 11.) ์ถœ๋ ฅ๋ฌผ 1๋ถ€, ์ฆ์ œ126ํ˜ธ USB๋‚ด โ€˜๊ณ ๋‚œ์ฃผ๊ฐ„ ์„ค๊ตโ€™ ์ œํ•˜ ์Šคํ…Œ๊ฐ€๋…ธ๊ทธ๋ผํ”ผ ํŒŒ์ผ(to you.docx) ์ถœ๋ ฅ๋ฌผ 1๋ถ€, ์ฆ์ œ126ํ˜ธ USB๋‚ด ์Šคํ…Œ๊ฐ€๋…ธ๊ทธ๋ผํ”ผ ํŒŒ์ผ(to you.docx)์„ ๋ณตํ˜ธํ™”ํ•œ ๋Œ€๋ถ๋ณด๊ณ ๋ฌธ ๋ฌธ๊ฑด (2011. 12. 1.์ž) ์ถœ๋ ฅ๋ฌผ 1๋ถ€, โ€˜to you7-7.docxโ€™ ํŒŒ์ผ ๋‚ด์šฉ ์ถœ๋ ฅ๋ฌผ 1๋ถ€, โ€˜to you7-7.docxโ€™ ํŒŒ์ผ์„ ๋ณตํ˜ธํ™”ํ•œ ๊ฒฐ๊ณผ ์ถœ๋ ฅ๋ฌผ (2013. 7. 7.์ž ๋Œ€๋ถ๋ณด๊ณ ๋ฌธ) 1๋ถ€, ์ฆ์ œ123ํ˜ธ Micro SD์นด๋“œ ๋‚ด โ€œ๋ณธ๋ฌธ์€ ์ด์Šค๋ผ์—˜ ๋ฐฑ์„ฑ๋“ค์—๊ฒŒ...(ํ›„๋žต)...โ€๋กœ ์‹œ์ž‘ํ•˜๋Š” ์Šคํ…Œ๊ฐ€๋…ธ๊ทธ๋ผํ”ผ ๋ฌธ์„œํŒŒ์ผ(toyou11-12.docx) ์ถœ๋ ฅ๋ฌผ 1๋ถ€, ์ฆ์ œ123ํ˜ธ Micro SD์นด๋“œ ๋‚ด ์Šคํ…Œ๊ฐ€๋…ธ๊ทธ๋ผํ”ผ ๋ฌธ์„œํŒŒ์ผ(toyou11-12.docx)์„ ๋ณตํ˜ธํ™”ํ•œ ๋Œ€๋ถ๋ณด๊ณ ๋ฌธ ์ถœ๋ ฅ๋ฌผ 1๋ถ€ 1. ํ”ผ๊ณ ์ธ์ด ์ž…๊ตญ์žฅ ๊ทผ์ฒ˜์—์„œ ํ™˜์ „์ค€๋น„ ๋ฐ ํ™˜์ „ํ•˜๋Š” ์‚ฌ์ง„ 6๋งค, ๊ณต์†Œ์™ธ 6 ๋ง๋ ˆ์ด์‹œ์•„ ๋ฐฉ๋ฌธ ๊ธฐ๊ฐ„ ์ค‘ ์ˆ˜์‚ฌ๊ด€ ์ดฌ์˜ ์ฑ„์ฆ์˜์ƒ๋ฌผ 42๋งค(์ฆ๊ฑฐ๊ธฐ๋ก 4,159, 4,160์ชฝ ์‚ฌ์ง„์— ํ•œํ•œ๋‹ค), 2014. 3.~2015. 8.๊ฐ„ ํ”ผ๊ณ ์ธ โ€˜(๋ช…์นญ 4 ์ƒ๋žต) PC๋ฐฉโ€™ ์ถœ์ž… ์ฑ„์ฆ์‚ฌ์ง„ 1๋ถ€, โ€˜(์˜๋ฌธ ID 1 ์ƒ๋žต)@โ–ฝโ–ฝโ–ฝโ–ฝ.COMโ€™ ์ด๋ฉ”์ผ ๊ณ„์ •์˜ ๋ณด๋‚ธํŽธ์ง€ํ•จ ํ™”๋ฉด ์บก์ณ ์‚ฌ์ง„ 1๋ถ€, โ€˜(์˜๋ฌธ ID 1 ์ƒ๋žต)@โ–ฝโ–ฝโ–ฝโ–ฝ.COMโ€™ ๊ณ„์ • ์••์ˆ˜์‹œ ๋ณด๋‚ธํŽธ์ง€ํ•จ ํ™”๋ฉด ์บก์ณ ์‚ฌ์ง„ 1๋ถ€ (4996), 2013. 12. 14. ํ”ผ๊ณ ์ธยท๊ณต์†Œ์™ธ 5 ์ ‘์„ ์žฅ๋ฉด ์ฑ„์ฆ CCTV ์บก์ณ์‚ฌ์ง„ 83๋งค, 2014. 1. 4. ํ”ผ๊ณ ์ธยท๊ณต์†Œ์™ธ 5 ์ ‘์„ ์žฅ๋ฉด ์ฑ„์ฆ ์˜์ƒ 45๋งค, 2014. 10. 5. ํ”ผ๊ณ ์ธ-๊ณต์†Œ์™ธ 6 ์ ‘์„  ์žฅ๋ฉด ๋ฐ ํ”ผ๊ณ ์ธ ์†Œ์ง€ ํƒœ๋ธ”๋ฆฟPC์— ์ €์žฅ๋œ ๋ฌธ๊ฑด ๋‚ด์šฉ ์ฑ„์ฆ ์‚ฌ์ง„ 1๋ถ€ 1. ์••์ˆ˜๋œ DVD-R[2010. 7. 22. ๊ณต์†Œ์™ธ 21 ์ฃผ์‹ํšŒ์‚ฌ, ์ฆ ์ œ28ํ˜ธ), ๋ฌธ๊ฑด(A4 18๋งค, ์ฆ ์ œ107ํ˜ธ), Micro SD ์นด๋“œ(16G, S/N MBMSAGVDDDCW-P, ์ฆ ์ œ123ํ˜ธ), USB(HP 4G, v165w, S/N : AA0000 0000 003489, ์ฆ ์ œ124ํ˜ธ), SD ์นด๋“œ(์†Œ๋‹ˆ, CHDC036GA, ์ฆ ์ œ125ํ˜ธ), USB(HP 4G, S/N : AA0000 0000 000102, ์ฆ ์ œ126ํ˜ธ), ์•„์ดํŒจ๋“œ ์—์–ด2(A-1566, S/N DLXP4CCEGSVY) ์ด๋ฏธ์ง• ํŒŒ์ผ(์ฆ ์ œ135ํ˜ธ), SanDisk USB(๋ชจ๋ธ๋ช… : Cruzer Blade 4GB, S/N : BH140624725N, ์ฆ ์ œ182ํ˜ธ), 20151008-2.avi(2015. 10. 8.์ž โ–ณโ–ณ๋Œ€ ์ œ1์ข…ํ•ฉ๊ด€ PC CCTV, ์ฆ ์ œ186ํ˜ธ)์˜ ๊ฐ ํ˜„์กด 1. ํ”ผ์˜์ž ์กฐํšŒ๊ฒฐ๊ณผ์„œ 1๋ถ€ ใ€ํ”ผ๊ณ ์ธ ๋ฐ ๋ณ€ํ˜ธ์ธ์˜ ์ฃผ์žฅ์— ๊ด€ํ•œ ํŒ๋‹จใ€‘ [์†Œ์ถ”์š”๊ฑด] 1. ๊ณต์†Œ์žฅ์ผ๋ณธ์ฃผ์˜ ์œ„๋ฐ˜์— ๋Œ€ํ•˜์—ฌ ๊ฐ€. ์ฃผ์žฅ ๊ฒ€์‚ฌ๋Š” ์ด ์‚ฌ๊ฑด ๊ณต์†Œ์žฅ์— ๊ณต์†Œ์‚ฌ์‹ค๊ณผ ๋ฌด๊ด€ํ•œ ์‚ฌ์‹ค์„ ๋ชจ๋‘์‚ฌ์‹ค๋กœ ๊ธฐ์žฌํ•˜๊ณ  ์ฆ๊ฑฐ๋Šฅ๋ ฅ์ด ์—†๋Š” ๋งŽ์€ ์ฆ๊ฑฐ๋ฌผ์˜ ๋‚ด์šฉ์„ ์ธ์šฉํ•˜๋Š” ๋“ฑ ๊ธˆ์ง€๋˜๋Š” ์‚ฌํ•ญ๋“ค์„ ๊ธฐ์žฌํ•˜์˜€๋‹ค. ์ด๋Š” ๋ฒ•๊ด€์—๊ฒŒ ์˜ˆ๋‹จ์„ ์ƒ๊ธฐ๊ฒŒ ํ•˜์—ฌ ๋ฒ”์ฃ„์‚ฌ์‹ค์˜ ์‹ค์ฒด๋ฅผ ํŒŒ์•…ํ•˜๋Š” ๋ฐ ์žฅ์• ๊ฐ€ ๋˜๋Š” ๊ฒƒ์ด๋‹ค. ๋”ฐ๋ผ์„œ ๊ณต์†Œ์ œ๊ธฐ์˜ ๋ฐฉ์‹์ด ๊ณต์†Œ์žฅ์ผ๋ณธ์ฃผ์˜์— ์œ„๋ฐฐ๋˜์–ด ๋ฌดํšจ์ด๋ฏ€๋กœ, ์ด ์‚ฌ๊ฑด ๊ณต์†Œ๋Š” ๊ธฐ๊ฐ๋˜์–ด์•ผ ํ•œ๋‹ค. ๋‚˜. ๊ด€๋ จ๋ฒ•๋ฆฌ ๊ณต์†Œ์žฅ์ผ๋ณธ์ฃผ์˜๋Š” ๊ฒ€์‚ฌ๊ฐ€ ๊ณต์†Œ๋ฅผ ์ œ๊ธฐํ•  ๋•Œ์—๋Š” ์›์น™์ ์œผ๋กœ ๊ณต์†Œ์žฅ ํ•˜๋‚˜๋งŒ์„ ์ œ์ถœํ•˜์—ฌ์•ผ ํ•˜๊ณ  ๊ทธ ๋ฐ–์— ์‚ฌ๊ฑด์— ๊ด€ํ•˜์—ฌ ๋ฒ•์›์— ์˜ˆ๋‹จ์ด ์ƒ๊ธฐ๊ฒŒ ํ•  ์ˆ˜ ์žˆ๋Š” ์„œ๋ฅ˜ ๊ธฐํƒ€ ๋ฌผ๊ฑด์„ ์ฒจ๋ถ€ํ•˜๊ฑฐ๋‚˜ ๊ทธ ๋‚ด์šฉ์„ ์ธ์šฉํ•˜์—ฌ์„œ๋Š” ์•„๋‹ˆ ๋œ๋‹ค๋Š” ์›์น™์ด๋‹ค(ํ˜•์‚ฌ์†Œ์†ก๊ทœ์น™ ์ œ118์กฐ ์ œ2ํ•ญ). ๊ณต์†Œ์žฅ์— ๋ฒ•๋ น์ด ์š”๊ตฌํ•˜๋Š” ์‚ฌํ•ญ ์ด์™ธ์˜ ์‚ฌ์‹ค๋กœ์„œ ๋ฒ•์›์— ์˜ˆ๋‹จ์ด ์ƒ๊ธฐ๊ฒŒ ํ•  ์ˆ˜ ์žˆ๋Š” ์‚ฌ์œ ๋ฅผ ๋‚˜์—ดํ•˜๋Š” ๊ฒƒ์ด ํ—ˆ์šฉ๋˜์ง€ ์•Š๋Š”๋‹ค๋Š” ๊ฒƒ๋„ ์ด๋ฅธ๋ฐ” โ€˜๊ธฐํƒ€ ์‚ฌ์‹ค์˜ ๊ธฐ์žฌ ๊ธˆ์ง€โ€™๋กœ์„œ ๊ณต์†Œ์žฅ์ผ๋ณธ์ฃผ์˜์˜ ๋‚ด์šฉ์— ํฌํ•จ๋œ๋‹ค. ๊ณต์†Œ์žฅ์ผ๋ณธ์ฃผ์˜์˜ ์œ„๋ฐฐ ์—ฌ๋ถ€๋Š” ๊ณต์†Œ์‚ฌ์‹ค๋กœ ๊ธฐ์žฌ๋œ ๋ฒ”์ฃ„์˜ ์œ ํ˜•๊ณผ ๋‚ด์šฉ ๋“ฑ์— ๋น„์ถ”์–ด ๋ณผ ๋•Œ, ๊ณต์†Œ์žฅ์— ์ฒจ๋ถ€ ๋˜๋Š” ์ธ์šฉ๋œ ์„œ๋ฅ˜ ๊ธฐํƒ€ ๋ฌผ๊ฑด์˜ ๋‚ด์šฉ, ๊ทธ๋ฆฌ๊ณ  ๋ฒ•๋ น์ด ์š”๊ตฌํ•˜๋Š” ์‚ฌํ•ญ ์ด์™ธ์— ๊ณต์†Œ์žฅ์— ๊ธฐ์žฌ๋œ ์‚ฌ์‹ค์ด ๋ฒ•๊ด€ ๋˜๋Š” ๋ฐฐ์‹ฌ์›์—๊ฒŒ ์˜ˆ๋‹จ์„ ์ƒ๊ธฐ๊ฒŒ ํ•˜์—ฌ ๋ฒ•๊ด€ ๋˜๋Š” ๋ฐฐ์‹ฌ์›์ด ๋ฒ”์ฃ„์‚ฌ์‹ค์˜ ์‹ค์ฒด๋ฅผ ํŒŒ์•…ํ•˜๋Š” ๋ฐ ์žฅ์• ๊ฐ€ ๋  ์ˆ˜ ์žˆ๋Š”์ง€ ์—ฌ๋ถ€๋ฅผ ๊ธฐ์ค€์œผ๋กœ ๋‹นํ•ด ์‚ฌ๊ฑด์—์„œ ๊ตฌ์ฒด์ ์œผ๋กœ ํŒ๋‹จํ•˜์—ฌ์•ผ ํ•œ๋‹ค. ํ•œํŽธ, ํ˜•์‚ฌ์†Œ์†ก๋ฒ• ์ œ254์กฐ ์ œ4ํ•ญ์€ โ€œ๊ณต์†Œ์‚ฌ์‹ค์˜ ๊ธฐ์žฌ๋Š” ๋ฒ”์ฃ„์˜ ์‹œ์ผ, ์žฅ์†Œ์™€ ๋ฐฉ๋ฒ•์„ ๋ช…์‹œํ•˜์—ฌ ์‚ฌ์‹ค์„ ํŠน์ •ํ•  ์ˆ˜ ์žˆ๋„๋ก ํ•˜์—ฌ์•ผ ํ•œ๋‹คโ€๊ณ  ๊ทœ์ •ํ•˜์—ฌ ๊ณต์†Œ์‚ฌ์‹ค์„ ๊ตฌ์ฒด์ ์œผ๋กœ ํŠน์ •ํ•  ๊ฒƒ์„ ์š”๊ตฌํ•˜๊ณ  ์žˆ๋‹ค. ์ด๋Š” ๋ฒ•์›์— ๋Œ€ํ•˜์—ฌ ์‹ฌํŒ์˜ ๋Œ€์ƒ์„ ํ•œ์ •ํ•จ์œผ๋กœ์จ ์‹ฌํŒ์˜ ๋Šฅ๋ฅ ๊ณผ ์‹ ์†์„ ๊พ€ํ•จ๊ณผ ๋™์‹œ์— ๋ฐฉ์–ด์˜ ๋ฒ”์œ„๋ฅผ ํŠน์ •ํ•˜์—ฌ ํ”ผ๊ณ ์ธ์˜ ๋ฐฉ์–ด๊ถŒ ํ–‰์‚ฌ๋ฅผ ์‰ฝ๊ฒŒ ํ•ด์ฃผ๋ ค๋Š” ๋ฐ ๊ทธ ์ทจ์ง€๊ฐ€ ์žˆ๋‹ค. ๊ทธ๋Ÿฌ๋ฏ€๋กœ ๊ณต์†Œ์‚ฌ์‹ค์€ ๊ฐ€๋Šฅํ•œ ํ•œ ๋ช…ํ™•ํ•˜๊ฒŒ ์ด๋ฅผ ํŠน์ •ํ•  ์ˆ˜ ์žˆ๋„๋ก ๊ธฐ์žฌํ•˜๋Š” ๊ฒƒ์ด ๋ฐ”๋žŒ์งํ•˜๊ณ  ์ด๋Ÿฌํ•œ ํ•„์š”์„ฑ์€ ๊ณต์†Œ์žฅ์ผ๋ณธ์ฃผ์˜ ์›์น™๊ณผ ๋น„๊ตํ•˜๋”๋ผ๋„ ๊ฐ€๋ณ๊ฒŒ ๋‹ค๋ฃฐ ๊ฒƒ์ด ์•„๋‹ˆ๋‹ค. ๋˜ํ•œ, ๊ณต์†Œ์‚ฌ์‹ค์˜ ๊ธฐ์žฌ๋Š” ๋ณธ์งˆ์ ์œผ๋กœ ์—ญ์‚ฌ์ ์œผ๋กœ ์ด๋ฏธ ๋ฐœ์ƒํ•œ ์‚ฌ์‹ค์„ ๊ทธ์— ๊ด€ํ•œ ์ž๋ฃŒ๋ฅผ ๊ธฐ์ดˆ๋กœ ๋ฒ”์ฃ„์‚ฌ์‹ค๋กœ ์žฌ๊ตฌ์„ฑํ•˜์—ฌ ํ‘œํ˜„ํ•˜๋Š” ๊ฒƒ์ด์–ด์„œ ๊ทธ ์ •๋„์˜ ์ฐจ์ด๊ฐ€ ์žˆ์„ ๋ฟ ํ•„์—ฐ์ ์œผ๋กœ ์žฅ์ฐจ ์ฆ๊ฑฐ๋กœ ์ œ์ถœ๋  ์„œ๋ฅ˜ ๊ธฐํƒ€ ๋ฌผ๊ฑด์— ๋‹ด๊ธด ์ •๋ณด๋ฅผ ๊ธฐ์ˆ ํ•˜๋Š” ํ˜•์‹์— ์˜ํ•˜๊ฒŒ ๋˜๊ณ , ์˜ˆ์ปจ๋Œ€ ๋ช…์˜ˆํ›ผ์†ยท๋ชจ์š•ยทํ˜‘๋ฐ• ๋“ฑ๊ณผ ๊ฐ™์ด ํŠน์ •ํ•œ ํ‘œํ˜„์˜ ๊ตฌ์ฒด์ ์ธ ๋‚ด์šฉ์— ๋”ฐ๋ผ ๋ฒ”์ฃ„์˜ ์„ฑ๋ถ€๊ฐ€ ํŒ๊ฐ€๋ฆ„๋˜๋Š” ๊ฒฝ์šฐ๋‚˜ ํŠนํ—ˆ๊ถŒยท์ƒํ‘œ๊ถŒ ์นจํ•ด์‚ฌ๋ฒ”์ฒ˜๋Ÿผ ์‚ฌ์•ˆ์˜ ์„ฑ์งˆ์ƒ ๋„๋ฉด ๋“ฑ์— ์˜ํ•œ ํŠน์ •์ด ํ•„์š”ํ•œ ๊ฒฝ์šฐ ๋“ฑ์—๋Š” ์„œ๋ฅ˜ ๊ธฐํƒ€ ๋ฌผ๊ฑด์˜ ๋‚ด์šฉ์„ ์ง์ ‘ ์ธ์šฉํ•˜๊ฑฐ๋‚˜ ์š”์•ฝ ๋˜๋Š” ์‚ฌ๋ณธํ•˜์—ฌ ์ฒจ๋ถ€ํ•  ์ˆ˜๋ฐ–์— ์—†๋‹ค. ๊ฒฐ๊ตญ, ๊ณต์†Œ์žฅ์ผ๋ณธ์ฃผ์˜๋Š” ๊ณต์†Œ์‚ฌ์‹ค ํŠน์ •์˜ ํ•„์š”์„ฑ์ด๋ผ๋Š” ๋˜ ๋‹ค๋ฅธ ์š”์ฒญ์— ์˜ํ•˜์—ฌ ํ•„์—ฐ์ ์œผ๋กœ ์ œ์•ฝ์„ ๋ฐ›์„ ์ˆ˜๋ฐ–์— ์—†๋Š” ๊ฒƒ์ด๋ฏ€๋กœ, ์–‘์ž์˜ ์ทจ์ง€์™€ ์ •์‹ ์ด ์กฐํ™”๋ฅผ ์ด๋ฃฐ ์ˆ˜ ์žˆ๋Š” ์„ ์—์„œ ๊ณต์†Œ์‚ฌ์‹ค ๊ธฐ์žฌ ๋˜๋Š” ํ‘œํ˜„์˜ ํ—ˆ์šฉ๋ฒ”์œ„์™€ ํ•œ๊ณ„๊ฐ€ ์„ค์ •๋˜์–ด์•ผ ํ•œ๋‹ค(๋Œ€๋ฒ•์› 2009. 10. 22. ์„ ๊ณ  2009๋„7436 ์ „์›ํ•ฉ์˜์ฒด ํŒ๊ฒฐ ๋“ฑ ์ฐธ์กฐ). ๋‹ค. ํŒ๋‹จ ์ด ์‚ฌ๊ฑด ๊ณต์†Œ์‚ฌ์‹ค ์ค‘ ๋ชจ๋‘์‚ฌ์‹ค์„ ๋น„๋กฏํ•˜์—ฌ ๊ฐœ๋ณ„ ๋ฒ”์ฃ„์‚ฌ์‹ค์—์„œ ๊ตฌ์ฒด์ ์œผ๋กœ ๊ธฐ์žฌํ•œ ๋‚ด์šฉ๋“ค์€ ๋ชจ๋‘ ์ด ์‚ฌ๊ฑด ๋ฒ”์ฃ„๋“ค์˜ ๊ตฌ์„ฑ์š”๊ฑด๊ณผ ๊ด€๋ จ์ด ์žˆ๋‹ค. ์ด ์‚ฌ๊ฑด ๋ฒ”์ฃ„๋“ค์€ ์ด์ ํ‘œํ˜„๋ฌผ์„ ์†Œ์ง€ํ•˜๊ณ , ๋ฐ˜๊ตญ๊ฐ€๋‹จ์ฒด์˜ ๊ตฌ์„ฑ์›๊ณผ ํšŒํ•ฉยทํ†ต์‹ ํ•˜๊ณ  ๊ทธ๋“ค์—๊ฒŒ ํŽธ์˜๋ฅผ ์ œ๊ณตํ•˜๋ฉฐ ๊ทธ๋“ค์˜ ํ™œ๋™์— ํ˜ธ์‘ยท๊ฐ€์„ธํ•˜์˜€๋‹ค๋Š” ๊ฒƒ์œผ๋กœ์„œ ์ด๋Ÿฌํ•œ ๋ฒ”์ฃ„๊ฐ€ ์„ฑ๋ฆฝํ•˜๊ธฐ ์œ„ํ•ด์„œ๋Š” โ€˜ํ”ผ๊ณ ์ธ์ด ์ž์‹ ์˜ ํ–‰์œ„๊ฐ€ ๊ตญ๊ฐ€์˜ ์กด๋ฆฝยท์•ˆ์ „์ด๋‚˜ ์ž์œ ๋ฏผ์ฃผ์  ๊ธฐ๋ณธ์งˆ์„œ๋ฅผ ์œ„ํƒœ๋กญ๊ฒŒ ํ•œ๋‹ค๋Š” ์ •์„ ์ธ์‹ํ•  ๊ฒƒโ€™ ๋˜๋Š” โ€˜์‹ค์งˆ์  ํ•ด์•…์„ ๋ผ์น  ๋ช…๋ฐฑํ•œ ์œ„ํ—˜์„ฑโ€˜ ๋“ฑ์ด ์š”๊ตฌ๋˜๋Š”๋ฐ, ์ด๋Ÿฌํ•œ ์‚ฌ์ •์€ ์ง์ ‘์ ์ธ ์ฆ๊ฑฐ๋“ค๋งŒ์œผ๋กœ ํŒ๋‹จํ•˜๊ธฐ๋Š” ์–ด๋ ต๊ณ  ํ”ผ๊ณ ์ธ์˜ ์„ฑํ–‰, ๊ทธ ๋™์•ˆ์˜ ํ™œ๋™๊ฒฝ๋ ฅ ๋“ฑ ์—ฌ๋Ÿฌ ๊ฐ„์ ‘์‚ฌ์‹ค๋“ค๊ณผ ํ•จ๊ป˜ ์ข…ํ•ฉ์ ์œผ๋กœ ํŒ๋‹จํ•  ์ˆ˜๋ฐ–์— ์—†๋‹ค. ๋˜ํ•œ, ํ”ผ๊ณ ์ธ์ด ๋ฒ”ํ•œ ๋ฒ”์ฃ„๋Š” ์€๋ฐ€ํ•˜๊ณ  ๊ณ„ํš์ ์œผ๋กœ ํ–‰ํ•ด์ง€๋Š” ์„ฑ๊ฒฉ์„ ๊ฐ€์ง€๋Š” ๊ฒƒ์œผ๋กœ์„œ ๊ฒ€์‚ฌ๋กœ์„œ๋Š” ํ”ผ๊ณ ์ธ์˜ ๋ฒ”ํ–‰ ์ž์ฒด๋Š” ๋ฌผ๋ก ์ด๊ณ  ๋ฒ”์˜, ๊ณต๋ฒ”๋“ค๊ณผ์˜ ๊ณต๋ชจ๊ด€๊ณ„, ๋ฒ”ํ–‰์˜ ๋™๊ธฐ๋‚˜ ๊ฒฝ์œ„ ๋“ฑ์„ ๋ช…ํ™•ํžˆ ํ•˜๊ธฐ ์œ„ํ•˜์—ฌ ๊ตฌ์ฒด์ ์ธ ์‚ฌ์ •์„ ์ ์‹œํ•  ํ•„์š”๊ฐ€ ์–ด๋Š ์ •๋„ ์žˆ๋‹ค. ๋”ฐ๋ผ์„œ ์ด ์‚ฌ๊ฑด ๊ณต์†Œ์žฅ์—์„œ ์›์šฉํ•˜๊ณ  ์žˆ๋Š” ์ด์ ํ‘œํ˜„๋ฌผ์˜ ๋‚ด์šฉ์ด๋‚˜ ํ†ต์‹ ์—ฐ๋ฝ๋ฌธ์˜ ๊ธฐ์žฌ๋‚ด์šฉ, ๋…น์ทจ๋ก์— ๋”ฐ๋ฅธ ๋Œ€ํ™”๋‚ด์šฉ ๋“ฑ์€ ์ด ์‚ฌ๊ฑด ๊ฐ ๊ตญ๊ฐ€๋ณด์•ˆ๋ฒ•์œ„๋ฐ˜ ๋ฒ”์ฃ„์˜ ๋‚ด์šฉ๊ณผ ์ˆ˜๋‹จยท๋ฐฉ๋ฒ• ์ž์ฒด, ๋˜๋Š” ๊ทธ ๋ฒ”์ฃ„์— ์ด๋ฅด๊ฒŒ ๋œ ๊ฒฝ์œ„๋ฅผ ์ ์‹œํ•˜๊ณ , ๊ณต์†Œ์‚ฌ์‹ค์„ ๋ช…ํ™•ํžˆ ํŠน์ •ํ•˜๊ธฐ ์œ„ํ•œ ๊ฒƒ์œผ๋กœ ๋ณด์ธ๋‹ค. ๊ทธ๋ฆฌ๊ณ  ์ด๋Š” ์ด ์‚ฌ๊ฑด ์‹ฌํŒ์˜ ๋Œ€์ƒ์„ ํ•œ์ •ํ•˜๊ณ  ํ”ผ๊ณ ์ธ์˜ ๋ฐฉ์–ด๊ถŒ ํ–‰์‚ฌ๋ฅผ ์‰ฝ๊ฒŒ ํ•˜๋Š”๋ฐ ์˜คํžˆ๋ ค ๋„์›€์ด ๋  ์ˆ˜ ์žˆ๋‹ค. ์ด๋Ÿฌํ•œ ์‚ฌ์ •์„ ์ข…ํ•ฉํ•˜๋ฉด, ์ด ์‚ฌ๊ฑด ๊ณต์†Œ์žฅ์˜ ๊ธฐ์žฌ๋‚ด์šฉ์ด ๊ณต์†Œ์žฅ์ผ๋ณธ์ฃผ์˜๋ฅผ ์œ„๋ฐ˜ํ•˜์—ฌ ๋ฒ•๊ด€์—๊ฒŒ ์˜ˆ๋‹จ์„ ์ƒ๊ธฐ๊ฒŒ ํ•˜๊ณ  ์‹ค์ฒด๋ฅผ ํŒŒ์•…ํ•˜๋Š” ๋ฐ ์žฅ์• ๊ฐ€ ๋œ๋‹ค๊ณ  ๋ณผ ์ˆ˜๋Š” ์—†๋‹ค. ๋”ฐ๋ผ์„œ ์ด ์‚ฌ๊ฑด ๊ณต์†Œ์ œ๊ธฐ ๋ฐฉ์‹์ด ์œ„๋ฒ•ํ•˜๋‹ค๊ณ  ํ•  ์ˆ˜ ์—†๋‹ค. ํ”ผ๊ณ ์ธ ๋ฐ ๋ณ€ํ˜ธ์ธ์˜ ์ด ๋ถ€๋ถ„ ์ฃผ์žฅ์€ ์ด์œ  ์—†๋‹ค. 2. ๊ณต์†Œ์‚ฌ์‹ค์˜ ํŠน์ •์— ๋Œ€ํ•˜์—ฌ ๊ฐ€. ์ฃผ์žฅ ์ด ์‚ฌ๊ฑด ๊ณต์†Œ์‚ฌ์‹ค ์ค‘ 2011. 11.๊ฒฝ ํ†ต์‹ ์—ฐ๋ฝ(๊ณต์†Œ์‚ฌ์‹ค โ…ก. 2์˜ ๋‚˜.ํ•ญ)๊ณผ 2011. 11.~12.๊ฒฝ ํ†ต์‹ ์—ฐ๋ฝ ๋ฐ ํŽธ์˜์ œ๊ณต(๊ณต์†Œ์‚ฌ์‹ค โ…ก. 2์˜ ๋‹ค.ํ•ญ)๊ณผ ๊ฐ™์€ ๊ฒฝ์šฐ, ํ”ผ๊ณ ์ธ์ด ์–ธ์ œ, ๋ˆ„๊ตฌ๋กœ๋ถ€ํ„ฐ ์Šคํ…Œ๊ฐ€๋…ธ๊ทธ๋ผํ”ผ๋กœ ์•”ํ˜ธํ™”๋œ ํŒŒ์ผ์„ ๋ฐ›์•˜๋Š”์ง€ ๋˜๋Š” ํ”ผ๊ณ ์ธ์ด ์ด๋ฅผ ๋ˆ„๊ตฌ์—๊ฒŒ ์ „๋‹ฌํ•˜์˜€๋Š”์ง€ ๋“ฑ์ด ํŠน์ •๋˜์–ด ์žˆ์ง€ ์•Š์•„ ์œ„๋ฒ•ํ•˜๋‹ค. ๋‚˜. ํŒ๋‹จ ๊ณต์†Œ์‚ฌ์‹ค์˜ ํŠน์ •๋ฐฉ๋ฒ•์„ ๊ทœ์ •ํ•œ ํ˜•์‚ฌ์†Œ์†ก๋ฒ• ์ œ254์กฐ ์ œ4ํ•ญ์—์„œ ๋งํ•˜๋Š” ๋ฒ”์ฃ„์˜ โ€˜์‹œ์ผโ€™์€ ์ด์ค‘๊ธฐ์†Œ๋‚˜ ์‹œํšจ์— ์ €์ด‰๋˜์ง€ ์•Š๋Š” ์ •๋„์˜ ๊ธฐ์žฌ๋ฅผ ์š”ํ•˜๊ณ , โ€˜์žฅ์†Œโ€™๋Š” ํ† ์ง€๊ด€ํ• ์„ ๊ฐ€๋Š ํ•  ์ˆ˜ ์žˆ๋Š” ์ •๋„์˜ ๊ธฐ์žฌ๋ฅผ ์š”ํ•˜๋Š”๋ฐ, ์ด์™€ ๊ฐ™์€ ์š”์†Œ๋“ค์— ์˜ํ•˜์—ฌ ๊ณต์†Œ์‚ฌ์‹ค์˜ ํŠน์ •์„ ์š”๊ตฌํ•˜๋Š” ๋ฒ•์˜ ์ทจ์ง€๋Š” ํ”ผ๊ณ ์ธ์˜ ๋ฐฉ์–ด๊ถŒ ํ–‰์‚ฌ๋ฅผ ์‰ฝ๊ฒŒ ํ•ด์ฃผ๊ธฐ ์œ„ํ•œ ๊ฒƒ์ด๋‹ค. ๊ทธ๋Ÿฌ๋ฏ€๋กœ ๊ณต์†Œ์‚ฌ์‹ค์€ ์ด๋Ÿฌํ•œ ์š”์†Œ๋ฅผ ์ข…ํ•ฉํ•˜์—ฌ ๊ตฌ์„ฑ์š”๊ฑด ํ•ด๋‹น ์‚ฌ์‹ค์„ ๋‹ค๋ฅธ ์‚ฌ์‹ค๊ณผ ์‹๋ณ„ํ•  ์ˆ˜ ์žˆ๋Š” ์ •๋„๋กœ ๊ธฐ์žฌํ•˜๋ฉด ์กฑํ•˜๊ณ , ๊ณต์†Œ์žฅ์— ๋ฒ”์ฃ„์˜ ์‹œ์ผ, ์žฅ์†Œ ๋“ฑ์ด ๊ตฌ์ฒด์ ์œผ๋กœ ์ ์‹œ๋˜์ง€ ์•Š์•˜๋”๋ผ๋„ ์œ„์—์„œ ๋ณธ โ€˜์‹œ์ผโ€™, โ€˜์žฅ์†Œโ€™ ๋“ฑ์˜ ๊ธฐ์žฌ๋ฅผ ํ•„์š”๋กœ ํ•œ ์ •๋„์— ๋ฐ˜ํ•˜์ง€ ์•„๋‹ˆํ•˜๊ณ  ๋”๊ตฌ๋‚˜ ๊ณต์†Œ๋ฒ”์ฃ„์˜ ์„ฑ๊ฒฉ์— ๋น„์ถ”์–ด ๊ทธ ๊ฐœ๊ด„์  ํ‘œ์‹œ๊ฐ€ ๋ถ€๋“์ดํ•˜๋ฉฐ ๋˜ํ•œ ๊ทธ์— ๋Œ€ํ•œ ํ”ผ๊ณ ์ธ์˜ ๋ฐฉ์–ด๊ถŒ ํ–‰์‚ฌ์— ์ง€์žฅ์ด ์—†๋‹ค๊ณ  ๋ณด์ด๋Š” ๊ฒฝ์šฐ์—๋Š” ๊ทธ ๊ณต์†Œ๋‚ด์šฉ์ด ํŠน์ •๋˜์ง€ ์•Š์•„ ๊ณต์†Œ์ œ๊ธฐ๊ฐ€ ์œ„๋ฒ•ํ•˜๋‹ค๊ณ  ํ•  ์ˆ˜๋Š” ์—†๋‹ค(๋Œ€๋ฒ•์› 1991. 10. 25. ์„ ๊ณ  91๋„2085 ํŒ๊ฒฐ ์ฐธ์กฐ). ํ•œํŽธ ์ด ์‚ฌ๊ฑด๊ณผ ๊ฐ™์ด ๋ฐ˜๊ตญ๊ฐ€๋‹จ์ฒด ๊ตฌ์„ฑ์›๊ณผ ํ†ต์‹ ์—ฐ๋ฝ์„ ํ•˜์˜€๋‹ค๊ณ  ๊ณต์†Œ ์ œ๊ธฐ๋œ ์‚ฌ๊ฑด์—์„œ๋Š” ๊ทธ ํ†ต์‹ ์˜ ์ˆ˜๋‹จ์ด ๋œ ์•”ํ˜ธํ™”๋œ ๋ฌธ๊ฑด ๋“ฑ์€ ์€๋ฐ€ํ•˜๊ฒŒ ์ „๋‹ฌ๋˜๋Š” ๊ฒƒ์ผ ์ˆ˜๋ฐ–์— ์—†๊ณ , ๊ทธ ๋ฌธ๊ฑด์„ ์ „๋‹ฌํ•˜๊ฑฐ๋‚˜ ๋ฐ›๋Š” ์ƒ๋Œ€๋ฐฉ๋„ ๋Œ€ํ•œ๋ฏผ๊ตญ์˜ ์‹คํšจ์  ์ง€๋ฐฐ ํ•˜์— ์žˆ์ง€ ์•Š์€ ๊ณณ์— ์žˆ์„ ์ˆ˜๋ฐ–์— ์—†๋‹ค. ๋”ฐ๋ผ์„œ ๊ทธ์— ๊ด€ํ•œ ๊ธฐ์žฌ์— ๋Œ€ํ•˜์—ฌ ์–ด๋Š ์ •๋„๋Š” ๊ฐœ๊ด„์  ํ‘œ์‹œ๊ฐ€ ๋ถ€๋“์ดํ•˜๋‹ค. ์ด์™€ ๊ฐ™์€ ๋ฒ•๋ฆฌ์— ๋”ฐ๋ผ ์ด ๋ถ€๋ถ„ ๊ณต์†Œ์‚ฌ์‹ค์„ ์‚ดํ”ผ๊ฑด๋Œ€, ๊ตฌ์„ฑ์š”๊ฑด ํ•ด๋‹น ์‚ฌ์‹ค์„ ๋‹ค๋ฅธ ์‚ฌ์‹ค๊ณผ ์‹๋ณ„ํ•  ์ˆ˜ ์žˆ์„ ์ •๋„๋กœ ๊ทธ ๋ฒ”ํ–‰์˜ ์ผ์‹œ, ์žฅ์†Œ, ๋ฐฉ๋ฒ• ๋“ฑ์ด ํŠน์ •๋˜์–ด ์žˆ๊ณ , ํ”ผ๊ณ ์ธ์—๊ฒŒ ์•”ํ˜ธํ™”๋œ ๋ฌธ๊ฑด์„ ์ฃผ์—ˆ๊ฑฐ๋‚˜ ํ”ผ๊ณ ์ธ์œผ๋กœ๋ถ€ํ„ฐ ๋Œ€๋ถ๋ณด๊ณ ๋ฌธ์„ ๋ฐ›์€ ์ž์˜ ์„ฑ๋ช…์„ ๋ช…์‹œํ•˜์ง€ ์•Š์•˜๋‹ค๊ณ  ํ•˜๋”๋ผ๋„ ๊ทธ ์ƒ๋Œ€๋ฐฉ์ด ๋ถํ•œ โ€˜225๊ตญโ€™ ์†Œ์† ๊ณต์ž‘์›์ด๋ผ๋Š” ์ ์— ๊ด€ํ•˜์—ฌ๋Š” ๋ช…ํ™•ํ•˜๊ฒŒ ๊ธฐ์žฌํ•˜๊ณ  ์žˆ๋‹ค. ๊ทธ ๋ฐ–์— ์ด ์‚ฌ๊ฑด ๊ณต์†Œ์ œ๊ธฐ๋œ ๋ฒ”์ฃ„์˜ ์„ฑ๊ฒฉ ๋“ฑ์— ๋น„์ถ”์–ด๋ณด๋ฉด ์–ด๋Š ์ •๋„ ๊ฐœ๊ด„์  ๊ธฐ์žฌ๋Š” ๋ถˆ๊ฐ€ํ”ผํ•œ ๊ฒƒ์œผ๋กœ ๋ณด์ด๊ณ , ์ด๋Ÿฌํ•œ ๊ธฐ์žฌ๋ฅผ ๊ฐ€์ง€๊ณ  ํ”ผ๊ณ ์ธ์˜ ๋ฐฉ์–ด๊ถŒ ํ–‰์‚ฌ์— ์ง€์žฅ์ด ์žˆ๋‹ค๊ณ ๋Š” ๋ณด์ด์ง€ ์•Š์œผ๋ฏ€๋กœ, ๊ฒฐ๊ตญ ์ด ์‚ฌ๊ฑด์€ ๊ทธ ๊ณต์†Œ๋‚ด์šฉ์ด ํŠน์ •๋˜์–ด ์žˆ๋Š” ๊ฒƒ์œผ๋กœ ๋ณผ ์ˆ˜ ์žˆ๋‹ค. ๋”ฐ๋ผ์„œ ํ”ผ๊ณ ์ธ ๋ฐ ๋ณ€ํ˜ธ์ธ์˜ ์ด ๋ถ€๋ถ„ ์ฃผ์žฅ์€ ์ด์œ  ์—†๋‹ค. [์ „์ œ๋˜๋Š” ์‚ฌ์‹ค ๋ฐ ์ ์šฉ๋ฒ•๋ฅ , ์žฌํŒ์ ˆ์ฐจ ๋“ฑ] 3. ๋ถํ•œ์˜ ๋ฐ˜๊ตญ๊ฐ€๋‹จ์ฒด์„ฑ๊ณผ ๊ตญ๊ฐ€๋ณด์•ˆ๋ฒ•์— ๋Œ€ํ•˜์—ฌ ๊ฐ€. ์ฃผ์žฅ ์ทจ์ง€ ๋ถํ•œ์€ ๋ฐ˜๊ตญ๊ฐ€๋‹จ์ฒด๊ฐ€ ์•„๋‹ˆ๊ณ  ํ‰ํ™”ํ†ต์ผ์˜ ์ƒ๋Œ€๋ฐฉ์ด๋‹ค. ๋ถํ•œ์„ ์กด์ค‘ํ•˜์—ฌ ์„œ๋กœ ํ˜‘๋ ฅํ•˜๊ณ  ๊ด€๊ณ„๋ฅผ ๊ฐœ์„ ํ•˜๋Š” ๊ฒƒ์ด ์šฐ๋ฆฌ ๋ฏผ์กฑ์ด ๋‚˜๊ฐ€์•ผ ํ•  ๋ฐฉํ–ฅ์ด๋‹ค. ํ”ผ๊ณ ์ธ์€ ๋ถํ•œ๊ณผ์˜ ํ‰ํ™” ํ†ต์ผ์„ ์œ„ํ•˜์—ฌ ๋…ธ๋ ฅํ–ˆ์„ ๋ฟ์ด๋‹ค. ๋ถํ•œ๊ณผ์˜ ๋ชจ๋“  ์ ‘์ด‰, ๋ถํ•œ ์›์ „ ๋ฌธ์„œ์˜ ์†Œ์ง€ ๋“ฑ์„ ๊ธˆ์ง€ํ•˜๊ณ  ์žˆ๋Š” ๊ตญ๊ฐ€๋ณด์•ˆ๋ฒ•์€ ํ์ง€๋˜์–ด์•ผ ํ•  ๋ฒ•๋ฅ ์ผ ๋ฟ, ๊ทธ ํšจ๋ ฅ์„ ์ธ์ •ํ•  ์ˆ˜ ์—†๋‹ค. ๋‚˜. ํŒ๋‹จ ํ”ผ๊ณ ์ธ ์ธก์ด ์ฃผ์žฅํ•˜๋Š” ๋ฐ”์™€ ๊ฐ™์ด ๋ถํ•œ์€ ๊ฐ™์€ ๋ฏผ์กฑ์œผ๋กœ ํ†ต์ผ์„ ์œ„ํ•ด ํ™”ํ•ดํ•˜๊ณ  ํ˜‘๋ ฅํ•˜์—ฌ์•ผ ํ•  ๋™๋ฐ˜์ž์ด๊ณ , ๋‚จํ•œ๊ณผ ๋ถํ•œ ์ง€์—ญ์—์„œ ๊ฐ ์ •๋ถ€๋ฅผ ์ˆ˜๋ฆฝํ•˜์—ฌ ํ†ต์น˜์ฒด์ œ๋ฅผ ๊ฐ–์ถ”๊ณ  ์žˆ์€ ๊ฒƒ์€ ๋ช…๋ฐฑํ•œ ์ฃผ1) ์‚ฌ์‹ค์ด๋‹ค. ๊ทธ๋Ÿฌ๋‚˜ ๋‚จยท๋ถํ•œ์˜ ์—ญ์‚ฌ์  ํŠน์ˆ˜์„ฑ์— ๋น„์ถ”์–ด ๋ณผ ๋•Œ ๋Œ€ํ•œ๋ฏผ๊ตญ์ด ๋ถํ•œ์„ ๊ต๋ฅ˜์™€ ํ˜‘๋ ฅ์˜ ๋Œ€์ƒ์ž๋กœ ์ธ์ •ํ•˜์—ฌ ์™”๋‹ค๋Š” ์‚ฌ์ •๋งŒ์„ ๋“ค์–ด ๋ถํ•œ์„ ํ•˜๋‚˜์˜ ๋…๋ฆฝํ•œ ๊ตญ๊ฐ€๋กœ ์Šน์ธํ•˜์˜€๋‹ค๊ณ  ๋ณผ ์ˆ˜ ์—†๊ณ , ๋‚จยท๋ถํ•œ ์‚ฌ์ด์˜ ๋ฒ•๋ฅ ๊ด€๊ณ„๋Š” ์—ฌ์ „ํžˆ ์šฐ๋ฆฌ์˜ ํ—Œ๋ฒ•๊ณผ ๋ฒ•๋ฅ ์— ๋”ฐ๋ผ ํŒ๋‹จํ•ด์•ผ ํ•˜๋Š” ๊ฒƒ์ด๋‹ค. ์šฐ๋ฆฌ ํ—Œ๋ฒ• ์ œ3์กฐ๋Š” โ€˜๋Œ€ํ•œ๋ฏผ๊ตญ์˜ ์˜ํ† ๋Š” ํ•œ๋ฐ˜๋„์™€ ๊ทธ ๋ถ€์†๋„์„œ๋กœ ํ•œ๋‹คโ€™๊ณ  ์ •ํ•˜๊ณ  ์žˆ๊ณ , ๋‚จ๋ถ๊ด€๊ณ„ ๋ฐœ์ „์— ๊ด€ํ•œ ๋ฒ•๋ฅ  ์ œ3์กฐ๋Š” โ€˜๋‚จํ•œ๊ณผ ๋ถํ•œ์˜ ๊ด€๊ณ„๋Š” ๊ตญ๊ฐ€๊ฐ„์˜ ๊ด€๊ณ„๊ฐ€ ์•„๋‹Œ ํ†ต์ผ์„ ์ง€ํ–ฅํ•˜๋Š” ๊ณผ์ •์—์„œ ์ž ์ •์ ์œผ๋กœ ํ˜•์„ฑ๋˜๋Š” ํŠน์ˆ˜๊ด€๊ณ„โ€™๋ผ๊ณ  ์ •ํ•˜๊ณ  ์žˆ๋‹ค. ์ด์™€ ๊ฐ™์€ ํ—Œ๋ฒ•๊ณผ ๋ฒ•๋ฅ ์˜ ๊ทœ์ •์— ์˜ํ•˜๋ฉด, ๋‚จยท๋ถํ•œ์ด ์ผ์ •ํ•œ ๋ฒ”์œ„ ๋‚ด์—์„œ ๊ต๋ฅ˜์™€ ํ˜‘๋ ฅ์„ ํ•˜์—ฌ ์™”๋‹ค๊ณ  ํ•˜๋”๋ผ๋„ ๊ทธ์™€ ๊ฐ™์€ ์‚ฌ์ •๋งŒ์„ ๋“ค์–ด ๋ถํ•œ์„ ์ •์น˜ยท๊ฒฝ์ œยท๋ฒ•๋ฅ ยท๊ตฐ์‚ฌยท๋ฌธํ™” ๋“ฑ ๋ชจ๋“  ์˜์—ญ์—์„œ ์šฐ๋ฆฌ์™€ ๋Œ€๋“ฑํ•œ ๋ณ„๊ฐœ์˜ ๋…๋ฆฝ๋œ ๊ตญ๊ฐ€๋กœ ๋ณผ ์ˆ˜๋Š” ์—†๋‹ค. ํ•œํŽธ ์šฐ๋ฆฌ ํ—Œ๋ฒ•์ด ์ „๋ฌธ๊ณผ ์ œ4์กฐ, ์ œ5์กฐ์—์„œ ์ฒœ๋ช…ํ•œ ๊ตญ์ œํ‰ํ™”์ฃผ์˜์™€ ํ‰ํ™”ํ†ต์ผ์˜ ์›์น™์€ ์ž์œ ๋ฏผ์ฃผ์ฃผ์˜์  ๊ธฐ๋ณธ์งˆ์„œ๋ผ๋Š” ์šฐ๋ฆฌ ํ—Œ๋ฒ•์˜ ๋Œ€์ „์ œ๋ฅผ ํ•ด์น˜์ง€ ์•Š๋Š” ๊ฒƒ์„ ์ „์ œ๋กœ ํ•˜๋Š” ๊ฒƒ์ด๋‹ค. ๊ทธ๋Ÿฐ๋ฐ ๋ถํ•œ์€ ํ˜„์‹œ์ ์—์„œ๋„ ์šฐ๋ฆฌ ํ—Œ๋ฒ•์˜ ๊ธฐ๋ณธ์›๋ฆฌ์™€ ์„œ๋กœ ์กฐํ™”๋  ์ˆ˜ ์—†์œผ๋ฉฐ ์ ๋Œ€์ ์ด๊ธฐ๋„ ํ•œ ๊ทธ๋“ค์˜ ์‚ฌํšŒ์ฃผ์˜ ํ—Œ๋ฒ•๊ณผ ๊ทธ ํ—Œ๋ฒ•์„ ์˜๋„ํ•˜๋Š” ์กฐ์„ ๋กœ๋™๋‹น๊ทœ์•ฝ์„ ํ†ตํ•˜์—ฌ ๋ถํ•œ์˜ ์ตœ์ข… ๋ชฉ์ ์ด ์ฃผ์ฒด์‚ฌ์ƒํ™”์™€ ๊ณต์‚ฐ์ฃผ์˜ ์‚ฌํšŒ๋ฅผ ๊ฑด์„คํ•˜๋Š” ๋ฐ์— ์žˆ๋‹ค๋Š” ๊ฒƒ๊ณผ ์ด๋Ÿฌํ•œ ์ ํ™”ํ†ต์ผ์˜ ๋ชฉํ‘œ๋ฅผ ์œ„ํ•˜์—ฌ ์ด๋ฅธ๋ฐ” ๋‚จํ•œ์˜ ๋ฐ˜์™ธ์„ธ ํˆฌ์Ÿ ๋“ฑ์„ ์ ๊ทน ์ง€์›ํ•˜๋Š” ์ •์ฑ…์„ ๋ช…๋ฌธ์œผ๋กœ ์„ ์–ธํ•˜๊ณ  ๊ทธ์— ๋”ฐ๋ฅธ ์ •์ฑ…๋“ค์„ ์ˆ˜ํ–‰ํ•˜๋ฉด์„œ ์ด์— ๋Œ€ํ•˜์—ฌ ๋ณ€๊ฒฝ์„ ๊ฐ€ํ•  ์ง•ํ›„๋ฅผ ๋ณด์ด๊ณ  ์žˆ์ง€ ์•Š๋‹ค. ๋”ฐ๋ผ์„œ ๋ถํ•œ์€ ์กฐ๊ตญ์˜ ํ‰ํ™”์  ํ†ต์ผ์„ ์œ„ํ•œ ๋Œ€ํ™”์™€ ํ˜‘๋ ฅ์˜ ๋™๋ฐ˜์ž์ด๊ณ  ๋™์‹œ์— ๋‚จยท๋ถํ•œ ๊ด€๊ณ„์˜ ์ผ๋ถ€ ๋ณ€ํ™”๋ผ๋Š” ์‚ฌ์ •์—๋„ ๋ถˆ๊ตฌํ•˜๊ณ , ์ ํ™”ํ†ต์ผ๋…ธ์„ ์„ ๊ณ ์ˆ˜ํ•˜๋ฉด์„œ ์šฐ๋ฆฌ์˜ ์ž์œ ๋ฏผ์ฃผ์ฃผ์˜ ์ฒด์ œ๋ฅผ ์ „๋ณตํ•˜๊ณ ์ž ํš์ฑ…ํ•˜๋Š” ๋ฐ˜๊ตญ๊ฐ€๋‹จ์ฒด๋ผ๋Š” ์„ฑ๊ฒฉ๋„ ์•„์šธ๋Ÿฌ ๊ฐ€์ง€๊ณ  ์žˆ๋‹ค. ํŠนํžˆ ์ด ์‚ฌ๊ฑด ๊ณต์†Œ์‚ฌ์‹ค ๊ธฐ์žฌ์™€ ๊ฐ™์ด, ๋ถํ•œ์˜ ๋Œ€๋‚จ๊ณต์ž‘๋ถ€์„œ์ธ โ€˜ํ†ต์ผ์ „์„ ๋ถ€โ€™ ์‚ฐํ•˜ โ€˜225๊ตญโ€™์€ ํŠน์ˆ˜๊ต์œก์„ ํ†ตํ•˜์—ฌ ์–‘์„ฑํ•œ ๋Œ€๋‚จ๊ณต์ž‘์›์„ ์งํŒŒ ๋˜๋Š” ์šฐํšŒ ์นจํˆฌ์‹œ์ผœ ๋Œ€ํ•œ๋ฏผ๊ตญ ๋‚ด์˜ ๊ฐ๊ณ„๊ฐ์ธต์˜ ์ธ์‚ฌ๋ฅผ ํฌ์„ญํ•˜์—ฌ ์กฐ์„ ๋…ธ๋™๋‹น์˜ ์ง€๋ น์— ๋”ฐ๋ฅด๋Š” ์ง€ํ•˜๋‹น ์กฐ์ง์„ ๊ฒฐ์„ฑํ•˜์—ฌ, ์ด๋ฅธ๋ฐ” โ€˜๋‚จ์กฐ์„  ํ˜๋ช…์˜ ๊ฒฐ์ •์  ์‹œ๊ธฐโ€™์— ์ง€ํ•˜๋‹น ์กฐ์ง์„ ํ˜๋ช…๋งค๊ฐœ์ฒด๋กœ ํ™œ์šฉํ•˜์—ฌ ๋Œ€ํ•œ๋ฏผ๊ตญ ์ฒด์ œ ์ „๋ณต์„ ๋ชฉํ‘œ๋กœ ํ™œ๋™ํ•˜๋ฉด์„œ ๊ตญ๊ฐ€๊ธฐ๋ฐ€์˜ ํƒ์ง€ยท์ˆ˜์ง‘, ๋ถํ•œ์ฒด์ œ์˜ ์šฐ์›”์„ฑ๊ณผ ๊น€์ผ์„ฑ ์ผ๊ฐ€ ์œ„๋Œ€์„ฑ ์„ ์ „ ๋ฐ ์š”์ธ์•”์‚ดยทํ…Œ๋Ÿฌ ๋“ฑ ๋Œ€๋‚จ๊ณต์ž‘ ์ž„๋ฌด๋ฅผ ์ˆ˜ํ–‰ํ•˜๊ณ  ์žˆ๋‹ค. ์ด๋Ÿฌํ•œ ์—ฌ๋Ÿฌ ์‚ฌ์ •์„ ๊ฐ์•ˆํ•˜์—ฌ ๋ณผ ๋•Œ ๋ฐ˜๊ตญ๊ฐ€๋‹จ์ฒด ๋“ฑ์„ ๊ทœ์œจํ•˜๋Š” ๊ตญ๊ฐ€๋ณด์•ˆ๋ฒ•์˜ ๊ทœ๋ฒ”๋ ฅ์€ ์—ฌ์ „ํžˆ ์œ ํšจํ•˜๋‹ค๊ณ  ํ•  ๊ฒƒ์ด๋‹ค(๋Œ€๋ฒ•์› 2010. 7. 23. ์„ ๊ณ  2010๋„1189 ์ „์›ํ•ฉ์˜์ฒด ํŒ๊ฒฐ, ๋Œ€๋ฒ•์› 2010. 12. 9. ์„ ๊ณ  2007๋„10121 ํŒ๊ฒฐ ๋“ฑ ์ฐธ์กฐ). ๋”ฐ๋ผ์„œ ํ”ผ๊ณ ์ธ ๋ฐ ๋ณ€ํ˜ธ์ธ์˜ ์ด ๋ถ€๋ถ„ ์ฃผ์žฅ์€ ์ด์œ  ์—†๋‹ค. 4. ๊ณต๊ฐœ์žฌํŒ์ฃผ์˜ ์œ„๋ฐฐ ์ฃผ์žฅ ๊ฐ€. ์ฃผ์žฅ 2016. 4. 11. ๊ณตํŒ๊ธฐ์ผ์— ๊ณต์†Œ์™ธ 3์— ๋Œ€ํ•œ ์ฆ์ธ์‹ ๋ฌธ์ด ๋น„๊ณต๊ฐœ๋กœ ์ง„ํ–‰๋˜์—ˆ๊ณ , ๋ณ€ํ˜ธ์ธ์ด ์ด์˜์‹ ์ฒญ์„ ํ•˜์˜€์œผ๋‚˜ ๋‹จ๋…ํŒ์‚ฌ๊ฐ€ ์ด๋ฅผ ๊ธฐ๊ฐํ•˜์˜€๋‹ค. ๋”ฐ๋ผ์„œ ๋น„๊ณต๊ฐœ ์žฌํŒ์—์„œ ๋‚˜์˜จ ์ฆ์–ธ์€ ์œ„๋ฒ•ํ•˜๊ฒŒ ์ˆ˜์ง‘๋œ ์ฆ๊ฑฐ๋กœ์„œ ์ฆ๊ฑฐ๋กœ ์‚ฌ์šฉํ•  ์ˆ˜ ์—†๋‹ค. ๋‚˜. ๊ด€๋ จ๋ฒ•๋ฆฌ ํ—Œ๋ฒ• ์ œ27์กฐ ์ œ3ํ•ญ ํ›„๋ฌธ์€ ํ˜•์‚ฌํ”ผ๊ณ ์ธ์€ ์ƒ๋‹นํ•œ ์ด์œ ๊ฐ€ ์—†๋Š” ํ•œ ์ง€์ฒด ์—†์ด ๊ณต๊ฐœ์žฌํŒ์„ ๋ฐ›์„ ๊ถŒ๋ฆฌ๋ฅผ ๊ฐ€์ง„๋‹ค๊ณ  ๊ทœ์ •ํ•˜์—ฌ ๊ณต๊ฐœ์žฌํŒ์„ ๋ฐ›์„ ๊ถŒ๋ฆฌ๊ฐ€ ํ˜•์‚ฌํ”ผ๊ณ ์ธ์˜ ๊ธฐ๋ณธ์  ์ธ๊ถŒ์ž„์„ ์„ ์–ธํ•˜๊ณ  ์žˆ๊ณ , ํ—Œ๋ฒ• ์ œ109์กฐ๋Š” ์žฌํŒ์˜ ์‹ฌ๋ฆฌ์™€ ํŒ๊ฒฐ์€ ๊ณต๊ฐœํ•œ๋‹ค. ๋‹ค๋งŒ, ์‹ฌ๋ฆฌ๋Š” ๊ตญ๊ฐ€์˜ ์•ˆ์ „๋ณด์žฅ ๋˜๋Š” ์•ˆ๋…•์งˆ์„œ๋ฅผ ๋ฐฉํ•ดํ•˜๊ฑฐ๋‚˜ ์„ ๋Ÿ‰ํ•œ ํ’์†์„ ํ•ดํ•  ์—ผ๋ ค๊ฐ€ ์žˆ์„ ๋•Œ์—๋Š” ๋ฒ•์›์˜ ๊ฒฐ์ •์œผ๋กœ ๊ณต๊ฐœํ•˜์ง€ ์•„๋‹ˆํ•  ์ˆ˜ ์žˆ๋‹ค๊ณ  ๊ทœ์ •ํ•˜๊ณ  ์žˆ์œผ๋ฉฐ, ๋ฒ•์›์กฐ์ง๋ฒ• ์ œ57์กฐ ์ œ1ํ•ญ๋„ ์žฌํŒ์˜ ์‹ฌ๋ฆฌ์™€ ํŒ๊ฒฐ์€ ๊ณต๊ฐœํ•œ๋‹ค. ๋‹ค๋งŒ, ์‹ฌ๋ฆฌ๋Š” ๊ตญ๊ฐ€์˜ ์•ˆ์ „๋ณด์žฅยท์•ˆ๋…•์งˆ์„œ ๋˜๋Š” ์„ ๋Ÿ‰ํ•œ ํ’์†์„ ํ•ดํ•  ์šฐ๋ ค๊ฐ€ ์žˆ๋Š” ๋•Œ์—๋Š” ๊ฒฐ์ •์œผ๋กœ ์ด๋ฅผ ๊ณต๊ฐœํ•˜์ง€ ์•„๋‹ˆํ•  ์ˆ˜ ์žˆ๋‹ค๊ณ  ๊ทœ์ •ํ•˜์—ฌ ์‹ฌ๋ฆฌ์˜ ๊ณต๊ฐœ๊ธˆ์ง€์‚ฌ์œ ๋ฅผ ์—„๊ฒฉํ•˜๊ฒŒ ์ œํ•œํ•˜๊ณ  ์žˆ์œผ๋ฏ€๋กœ, ํ—Œ๋ฒ• ์ œ109์กฐ, ๋ฒ•์›์กฐ์ง๋ฒ• ์ œ57์กฐ ์ œ1ํ•ญ์ด ์ •ํ•œ ๊ณต๊ฐœ๊ธˆ์ง€์‚ฌ์œ ๊ฐ€ ์—†์Œ์—๋„ ๋ถˆ๊ตฌํ•˜๊ณ  ์„ ๊ณ ๋œ ๊ณต๊ฐœ๊ธˆ์ง€๊ฒฐ์ •์€ ํ”ผ๊ณ ์ธ์˜ ๊ณต๊ฐœ์žฌํŒ์„ ๋ฐ›์„ ๊ถŒ๋ฆฌ๋ฅผ ์นจํ•ดํ•œ ๊ฒƒ์œผ๋กœ์„œ ๊ทธ ์ ˆ์ฐจ์— ์˜ํ•˜์—ฌ ์ด๋ฃจ์–ด์ง„ ์ฆ์ธ์˜ ์ฆ์–ธ์€ ์ฆ๊ฑฐ๋Šฅ๋ ฅ์ด ์—†๋‹ค๊ณ  ํ•  ๊ฒƒ์ด๊ณ , ๋ณ€ํ˜ธ์ธ์˜ ๋ฐ˜๋Œ€์‹ ๋ฌธ๊ถŒ์ด ๋ณด์žฅ๋˜์—ˆ๋”๋ผ๋„ ๋‹ฌ๋ฆฌ ๋ณผ ์ˆ˜ ์—†๋‹ค(๋Œ€๋ฒ•์› 2013. 7. 26. ์„ ๊ณ  2013๋„2511 ํŒ๊ฒฐ ๋“ฑ ์ฐธ์กฐ). ๋‹ค. ํŒ๋‹จ 1) 2016. 4. 11. ๊ณตํŒ์กฐ์„œ์— โ€˜๊ฒ€์‚ฌ์˜ ๊ตญ๊ฐ€์•ˆ์ „๋ณด์žฅ ๋“ฑ์— ๊ด€๋ จ๋œ ์ธก๋ฉด์ด ์žˆ๋‹ค๋Š” ์ด์œ ๋กœ ํ•œ ๋น„๊ณต๊ฐœ ์‹ ์ฒญ์€ ์ด์œ  ์žˆ์œผ๋ฏ€๋กœ ์ด๋ฒˆ ์ฆ์ธ์— ๋Œ€ํ•œ ์‹ ๋ฌธ์ ˆ์ฐจ ๋ถ€๋ถ„์˜ ๊ณต๊ฐœ๋ฅผ ์ •์ง€ํ•œ๋‹คโ€™๊ณ  ๊ฒฐ์ •ํ•œ ๊ฒƒ์œผ๋กœ ๊ธฐ์žฌ๋˜์–ด ์žˆ๋Š”๋ฐ”, ๋‹จ๋…ํŒ์‚ฌ๋Š” ๊ฒ€์‚ฌ๊ฐ€ ๋ฏธ๋ฆฌ ์ œ์‹œํ•œ ์ฃผ2) ์‚ฌ์œ ์™€ ์ด์— ๋Œ€ํ•œ ํ”ผ๊ณ ์ธ ๋ณ€ํ˜ธ์ธ์˜ ์ฃผ์žฅ๊ณผ ์ด์˜ ๋“ฑ์„ ๋ชจ๋‘ ๊ฒ€ํ† ํ•˜๋Š” ์ ˆ์ฐจ๋ฅผ ๊ฑฐ์ณ ์žฌํŒ์˜ ๋น„๊ณต๊ฐœ ๊ฒฐ์ •์„ ์„ ๊ณ ํ•œ ๊ฒƒ์œผ๋กœ ์ธ์ •๋œ๋‹ค. 2) ์œ„ ๋น„๊ณต๊ฐœ๋กœ ์ง„ํ–‰๋œ ์žฌํŒ์€ ๊ตญ๊ฐ€์ •๋ณด์›(์ดํ•˜ โ€˜๊ตญ์ •์›โ€™์ด๋ผ๊ณ ๋งŒ ํ•œ๋‹ค)์—์„œ ์•”ํ˜ธ๋ณตํ˜ธ ๋ถ„์„์„ ๋‹ด๋‹นํ•˜๊ณ  ์žˆ๋Š” ์ฆ์ธ ๊ณต์†Œ์™ธ 3์ด ์Šคํ…Œ๊ฐ€๋…ธ๊ทธ๋ผํ”ผ๋กœ ์•”ํ˜ธํ™”๋œ ๋Œ€๋ถ์—ฐ๋ฝ๋ฌธ์˜ ๋ณตํ˜ธํ™” ๊ณผ์ •์„ ์„ค๋ช…ํ•˜๊ณ  ์ด๋ฅผ ์‹ค์—ฐํ•˜๋Š” ๊ฒƒ์ด์—ˆ๋‹ค. ๊ทธ๋Ÿฐ๋ฐ ๋‹น์‹œ ๊ทธ ๊ตญ์ •์› ์ง์›์€ ํ˜„์žฅ ์š”์›์ด ์•„๋‹ˆ์–ด์„œ ์‹ ๋ถ„์„ ๋…ธ์ถœ์‹œํ‚ค์ง€ ์•Š์„ ํ•„์š”๊ฐ€ ์žˆ์—ˆ๋˜ ์ , ๊ทธ๊ฐ€ ํ•˜๋Š” ๋ณตํ˜ธํ™” ๊ณผ์ •์€ ์ƒ๋‹นํ•œ ์ˆ˜์‚ฌ๊ธฐ๋ฐ€์— ํ•ด๋‹นํ•  ์ˆ˜ ์žˆ๋Š” ์ , ๋‹น์‹œ ์žฌํŒ์ด ์ง„ํ–‰๋˜๋Š” ๊ณผ์ •์— ๋น„์ถ”์–ด ๋ณผ ๋•Œ ์‹ ๋ถ„์ƒ์˜ ๋ณด์•ˆ์ด ํ•„์š”ํ•œ ๊ตญ์ •์› ์ง์›์ด ์Šคํ…Œ๊ฐ€๋…ธ๊ทธ๋ผํ”ผ๋ฅผ ๋ณตํ˜ธํ™”ํ•˜๋Š” ๋ชจ์Šต๊ณผ ๊ณผ์ •์ด ์™ธ๋ถ€์— ๊ณต๊ฐœ๋  ๊ฒฝ์šฐ ๊ตญ๊ฐ€์•ˆ๋ณด๋ฅผ ํ•ดํ•  ์šฐ๋ ค๊ฐ€ ์žˆ๋‹ค๊ณ  ํŒ๋‹จํ•จ์— ํ•ฉ๋ฆฌ์  ์ด์œ ๊ฐ€ ์žˆ์–ด ๋ณด์ด๋Š” ์ฃผ3) ์ , ํ•œํŽธ ๋น„๊ณต๊ฐœ๋กœ ํ•˜๋Š” ๋Œ€์‹  ํ”ผ๊ณ ์ธ๊ณผ ์ฆ์ธ ์‚ฌ์ด์— ์ฐจํ๋ง‰์„ ์„ค์น˜ํ•˜์ง€๋Š” ์•Š๋Š” ๋“ฑ ํ”ผ๊ณ ์ธ๊ณผ ๋ณ€ํ˜ธ์ธ์ด ๊ทธ ๊ณผ์ •์„ ์ง์ ‘ ๋ณด๋ฉด์„œ ๋ฐ˜๋ฐ•ํ•  ์ˆ˜ ์žˆ๋„๋ก ํ”ผ๊ณ ์ธ๊ณผ ๋ณ€ํ˜ธ์ธ์˜ ๊ถŒ๋ฆฌ๋ฅผ ๋ณดํ˜ธํ•˜๊ธฐ ์œ„ํ•œ ๋…ธ๋ ฅ์„ ๊ธฐ์šธ์˜€๋˜ ์  ๋“ฑ์„ ์ข…ํ•ฉํ•˜๋ฉด, ์œ„ ์ฆ์ธ์‹ ๋ฌธ์„ ๋น„๊ณต๊ฐœ๋กœ ํ•˜์˜€๋‹ค๊ณ  ํ•˜์—ฌ ์œ„๋ฒ•ํ•˜๋‹ค๊ณ  ๋ณผ ์ˆ˜๋Š” ์ฃผ4) ์—†๋‹ค. ๋”ฐ๋ผ์„œ ํ”ผ๊ณ ์ธ ๋ฐ ๋ณ€ํ˜ธ์ธ์˜ ์ด ๋ถ€๋ถ„ ์ฃผ์žฅ์€ ๋ฐ›์•„๋“ค์ด์ง€ ์•Š๋Š”๋‹ค. 5. ๊ตญ๊ฐ€์ •๋ณด์› ์ง์› ๋“ฑ์— ๋Œ€ํ•œ ์ฆ์ธ์‹ ๋ฌธ์‹œ ์ฐจํ๋ง‰ ์„ค์น˜์˜ ์œ„๋ฒ•์„ฑ ์ฃผ์žฅ ๊ฐ€. ์ฃผ์žฅ ๊ตญ์ •์› ์ง์›๋“ค ๋˜๋Š” ์ผ๋ถ€ ๊ฒฝ์ฐฐ ์ˆ˜์‚ฌ๊ด€์— ๋Œ€ํ•œ ์ฃผ5) ์ฆ์ธ์‹ ๋ฌธ ์‹œ ์ฆ์ธ๊ณผ ํ”ผ๊ณ ์ธ ๋ฐ ๋ฐฉ์ฒญ์„ ์‚ฌ์ด์— ์ฐจํ๋ง‰์„ ์„ค์น˜ํ•˜๊ณ  ์ฆ์ธ์‹ ๋ฌธ์„ ํ•˜์˜€๋‹ค. ์ด๋Š” ๋ฒ•์  ๊ทผ๊ฑฐ๊ฐ€ ์—†๊ณ  ํ”ผ๊ณ ์ธ์˜ ๋ฐฉ์–ด๊ถŒ์„ ์นจํ•ดํ•˜๋Š” ๊ฒƒ์œผ๋กœ์„œ ์œ„๋ฒ•ํ•˜๋‹ค. ๋”ฐ๋ผ์„œ ์ด ๋ถ€๋ถ„ ์ฆ์–ธ์€ ์ฆ๊ฑฐ๋กœ ์‚ฌ์šฉ๋˜์–ด์„œ๋Š” ์•ˆ๋œ๋‹ค. ๋‚˜. ํŒ๋‹จ ํ˜•์‚ฌ์†Œ์†ก๋ฒ• ์ œ165์กฐ์˜2 ์ œ3ํ˜ธ, ํ˜•์‚ฌ์†Œ์†ก๊ทœ์น™ ์ œ84์กฐ์˜9์— ๋”ฐ๋ฅด๋ฉด, ๋ฒ”์ฃ„์˜ ์„ฑ์งˆ, ์ฆ์ธ์˜ ์—ฐ๋ น, ์‹ฌ์‹ ์˜ ์ƒํƒœ, ํ”ผ๊ณ ์ธ๊ณผ์˜ ๊ด€๊ณ„, ๊ทธ ๋ฐ–์˜ ์‚ฌ์ •์œผ๋กœ ์ธํ•˜์—ฌ ํ”ผ๊ณ ์ธ ๋“ฑ๊ณผ ๋Œ€๋ฉดํ•˜์—ฌ ์ง„์ˆ ํ•˜๋Š” ๊ฒฝ์šฐ ์‹ฌ๋ฆฌ์ ์ธ ๋ถ€๋‹ด์œผ๋กœ ์ •์‹ ์˜ ํ‰์˜จ์„ ํ˜„์ €ํ•˜๊ฒŒ ์žƒ์„ ์šฐ๋ ค๊ฐ€ ์žˆ๋‹ค๊ณ  ์ธ์ •๋˜๋Š” ๊ฒฝ์šฐ์—๋Š” ์ฐจํ์‹œ์„ค ๋“ฑ์„ ์„ค์น˜ํ•  ์ˆ˜ ์žˆ๊ณ , ์ด ๋•Œ ํ”ผ๊ณ ์ธ๊ณผ ์ฆ์ธ์ด ์„œ๋กœ ๋ชจ์Šต์„ ๋ณผ ์ˆ˜ ์—†๋„๋ก ํ•„์š”ํ•œ ์กฐ์น˜๋ฅผ ํ•˜๋„๋ก ๋˜์–ด ์ฃผ6) ์žˆ๋‹ค. ๋˜ํ•œ ๊ตญ๊ฐ€์ •๋ณด์›์ง์›๋ฒ• ์ œ17์กฐ ์ œ6ํ•ญ์— ๋”ฐ๋ฅด๋ฉด, ๋ฒ•์›์€ ๊ตญ์ •์› ์ง์›์„ ์ฆ์–ธํ•˜๋„๋ก ํ•จ์— ์žˆ์–ด์„œ ๊ณต๋ฌด์ƒ ๋น„๋ฐ€๋ณดํ˜ธ ๋“ฑ์„ ์œ„ํ•˜์—ฌ ๋น„๊ณต๊ฐœ์ฆ์–ธ ๋“ฑ ์ ์ ˆํ•œ ์กฐ์น˜๋ฅผ ์ทจํ•  ์ˆ˜ ์žˆ๋„๋ก ๊ทœ์ •ํ•˜๊ณ  ์žˆ๋‹ค. ๋”ฐ๋ผ์„œ ์ด ์‚ฌ๊ฑด์—์„œ ๋ฒ”์ฃ„์˜ ์„ฑ์งˆ, ์ฆ์ธ์˜ ์‹ ๋ถ„๊ณผ ์ง๋ฌด์ƒ ๋น„๋ฐ€์˜ ๋ณดํ˜ธ, ํ”ผ๊ณ ์ธ๊ณผ์˜ ๊ด€๊ณ„ ๋“ฑ ์ œ๋ฐ˜์‚ฌ์ •์— ๋น„์ถ”์–ด ์œ„ ๊ทœ์ •์— ๋”ฐ๋ผ ์ฐจํ์‹œ์„ค์„ ํ•  ์ˆ˜ ์žˆ๋Š” ๊ฒƒ์ด๋ฏ€๋กœ, ๋ฒ•๋ น์˜ ๊ทผ๊ฑฐ๊ฐ€ ์—†๋‹ค๊ฑฐ๋‚˜ ์œ„๋ฒ•ํ•˜๋‹ค๊ณ  ๋ณผ ์ˆ˜๋Š” ์—†๋‹ค. ํŠนํžˆ ๊ตญ์ •์› ์ง์›๋“ค ๋˜๋Š” ๊ฒฝ์ฐฐ ์ˆ˜์‚ฌ๊ด€ ๊ณต์†Œ์™ธ 14๋Š” ํ˜„์žฅ์—์„œ ์ง์ ‘ ์ฑ„์ฆ์„ ํ•˜๋Š” ์š”์›๋“ค๋กœ์„œ ๊ทธ๋“ค์˜ ์‹ ์›์ด ๋ฌด์ œํ•œ์ ์œผ๋กœ ๋…ธ์ถœ๋  ๊ฒฝ์šฐ ์ถ”ํ›„ ์ˆ˜์‚ฌ์™€ ๊ธฐ๋ฐ€์„ฑ ์œ ์ง€์— ์–ด๋ ค์›€์„ ๊ฒช์„ ์ˆ˜๋„ ์žˆ๊ณ , ๋˜ ๊ทธ๋Ÿฌํ•œ ์‚ฌ์ • ๋•Œ๋ฌธ์— ํ”ผ๊ณ ์ธ๊ณผ ๋Œ€๋ฉดํ•˜๊ณ  ์—ฌ๋Ÿฌ ๋ฐฉ์ฒญ๊ฐ๋“ค ์•ž์—์„œ ์ง„์ˆ ํ•˜๋Š” ๊ฒƒ์ด ์ƒ๋‹นํžˆ ๊ณค๋ž€ํ•˜๋‹ค๊ณ  ๋ณด์ด๋ฏ€๋กœ ๊ทธ๋“ค์˜ ์‹ ๋ณ€ ๋ณดํ˜ธ๋ฅผ ์œ„ํ•œ ํ•„์š”์„ฑ์ด ์ธ์ •๋œ๋‹ค. ๋˜ํ•œ, ํ”ผ๊ณ ์ธ์ด ์ฆ์ธ์˜ ๋ชจ์Šต์„ ๋ณด์ง€ ๋ชปํ•œ ๊ฒƒ ์™ธ์—๋Š” ๊ทธ ์ง„์ˆ ์„ ๋“ฃ๊ณ  ์งˆ๋ฌธ์„ ํ•˜๋Š” ๋ฐ์— ์•„๋ฌด๋Ÿฐ ์žฅ์• ๊ฐ€ ์—†๊ณ , ํ”ผ๊ณ ์ธ์˜ ๋ณ€ํ˜ธ์ธ ์ชฝ์—๋Š” ์ฐจํ๋ฅผ ํ•˜์ง€ ์•Š์•˜์œผ๋ฏ€๋กœ ๋ณ€ํ˜ธ์ธ๋“ค์€ ์•„๋ฌด๋Ÿฐ ์ œํ•œ ์—†์ด ์ฆ์ธ์„ ๋ณผ ์ˆ˜ ์žˆ์—ˆ๋‹ค. ์‹ค์ œ๋กœ, ์ดฌ์˜์ผ์‹œยท์žฅ์†Œ, ์ดฌ์˜๊ธฐ๊ธฐ์˜ ์ข…๋ฅ˜ ๋ฐ ๊ทธ์— ๋”ฐ๋ฅธ ์ดฌ์˜๊ธฐ๋ฒ•, ์ดฌ์˜๊ฒฝ์œ„, ์ดฌ์˜ ๋‹น์‹œ ํ”ผ๊ณ ์ธ๊ณผ ์ง€์ธ๋“ค์˜ ๋Œ€ํ™”๋‚ด์šฉ ๋“ฑ ๋‹น์‹œ ์ƒํ™ฉ ๋“ฑ์— ๊ด€ํ•˜์—ฌ ๋ฐ˜๋Œ€์‹ ๋ฌธ์ด ๋งค์šฐ ์ƒ์„ธํžˆ ์ด๋ฃจ์–ด์กŒ๊ณ , ๋ณ€ํ˜ธ์ธ๋“ค์€ ์ด๋Ÿฌํ•œ ์ฆ์ธ์˜ ์ฆ์–ธ ํƒœ๋„๋‚˜ ๋ชจ์Šต์„ ์ž์„ธํžˆ ์‚ดํŽด๋ณผ ์ˆ˜ ์žˆ์—ˆ์œผ๋ฉฐ, ์œ„ ์ฆ์ธ๋“ค์€ ์ผ๋ถ€ ๊ณต๋ฌด์ƒ ๋น„๋ฐ€๊ณผ ๊ด€๋ จ์ด ์žˆ๋Š” ๋ถ€๋ถ„์„ ์ œ์™ธํ•œ ๋‚˜๋จธ์ง€ ๋ถ€๋ถ„์— ๋Œ€ํ•˜์—ฌ ๋น„๊ต์  ์ž์„ธํžˆ ๋‹ต๋ณ€์„ ํ•˜์˜€๋‹ค. ๋˜ํ•œ, ํ”ผ๊ณ ์ธ์—๊ฒŒ๋„ ์ถฉ๋ถ„ํ•œ ๋ฐ˜๋Œ€์‹ ๋ฌธ์„ ํ•  ์ˆ˜ ์žˆ๋Š” ๊ธฐํšŒ๋ฅผ ๋ถ€์—ฌํ•˜์˜€๋‹ค(๊ทธ๋Ÿฌ๋‚˜ ๋Œ€์ฒด๋กœ ํ”ผ๊ณ ์ธ์€ ์ด ์‚ฌ๊ฑด ์žฌํŒ ๋‚ด๋‚ด ์ง„์ˆ ์„ ๊ฑฐ๋ถ€ํ•˜์˜€๋‹ค). ์ด์™€ ๊ฐ™์€ ์ฐจํ๋ง‰ ์„ค์น˜์˜ ๊ทผ๊ฑฐ์™€ ์ด์œ , ์ฆ์ธ์‹ ๋ฌธ์˜ ๊ณผ์ • ๋ฐ ์ฆ์–ธ๋‚ด์šฉ ๋“ฑ์— ๋น„์ถ”์–ด ๋ณด๋ฉด, ์ด ์‚ฌ๊ฑด์—์„œ ์ฆ์ธ๊ณผ ํ”ผ๊ณ ์ธ ๋ฐ ๋ฐฉ์ฒญ์„ ์‚ฌ์ด์— ์ฐจํ๋ง‰์ด ์„ค์น˜๋˜์—ˆ๋‹ค๊ณ  ํ•ด์„œ ํ”ผ๊ณ ์ธ๊ณผ ๋ณ€ํ˜ธ์ธ์˜ ๋ฐ˜๋Œ€์‹ ๋ฌธ๊ถŒ์ด๋‚˜ ๋ฐฉ์–ด๊ถŒ์ด ์‹ค์งˆ์ ์œผ๋กœ ์นจํ•ด๋˜์–ด ์œ„ ์ฆ์–ธ์˜ ์ฆ๊ฑฐ๋Šฅ๋ ฅ์„ ๋ถ€์ •ํ•ด์•ผ ํ•  ์ •๋„์˜ ์œ„๋ฒ•์ด ์žˆ๋‹ค๊ณ  ํ•  ์ˆ˜๋Š” ์—†๋‹ค. ๋”ฐ๋ผ์„œ ํ”ผ๊ณ ์ธ ๋ฐ ๋ณ€ํ˜ธ์ธ์˜ ์ด ๋ถ€๋ถ„ ์ฃผ์žฅ์€ ๋ฐ›์•„๋“ค์ด์ง€ ์•Š๋Š”๋‹ค. [์ˆ˜์‚ฌ์ ˆ์ฐจ ๋ฐ ์ฆ๊ฑฐ๋“ค์˜ ์ฆ๊ฑฐ๋Šฅ๋ ฅ ๋“ฑ] 6. ํ”ผ๊ณ ์ธ์˜ ์‹ ์ฒด, ์ฃผ๊ฑฐ์ง€, ์ฐจ๋Ÿ‰, ์˜คํ† ๋ฐ”์ด์—์„œ ์••์ˆ˜ํ•œ ๋ฌผ๊ฑด๋“ค์ด ์œ„๋ฒ•์ˆ˜์ง‘ ์ฆ๊ฑฐ๋ผ๋Š” ์ฃผ์žฅ ๊ฐ€. ์ฃผ์žฅ 1) ์ด ์‚ฌ๊ฑด์—์„œ ๊ตญ์ •์› ์ˆ˜์‚ฌ๊ด€๋“ค์˜ ์••์ˆ˜ยท์ˆ˜์ƒ‰์ด ์ž์ • ์ดํ›„ ์‹œ์ž‘๋˜์–ด ๋ฐค์ƒˆ๋„๋ก ์ง„ํ–‰๋˜์—ˆ๋‹ค. ์œ„ ์••์ˆ˜์ˆ˜์ƒ‰๊ณผ์ •์—๋Š” ๊ทธ ์ง€์—ญ ํ†ต์žฅ์ธ ๊ณต์†Œ์™ธ 7์ด ์ฐธ์—ฌํ•˜์˜€๋Š”๋ฐ, ๊ทธ๋Š” ์žฅ์‹œ๊ฐ„ ํ”ผ๊ณคํ•œ ๊ด€๊ณ„๋กœ ์ƒํ™ฉ์„ ์ œ๋Œ€๋กœ ํŒŒ์•…ํ•˜์ง€ ๋ชปํ•˜๋Š” ๋“ฑ ์ฐธ์—ฌ์ธ์˜ ์—ญํ• ์„ ์ œ๋Œ€๋กœ ํ•˜์ง€ ๋ชปํ–ˆ๊ณ , ์••์ˆ˜์ˆ˜์ƒ‰์ ˆ์ฐจ๊ฐ€ ๋๋‚˜๊ธฐ ์ „์— ๊ฐœ์ธ์ ์ธ ์ด์œ ๋กœ ํ˜„์žฅ์—์„œ ์ดํƒˆํ•˜์˜€๋‹ค. ๋”ฐ๋ผ์„œ ์ด๋Š” ์œ„๋ฒ•ํ•œ ์••์ˆ˜ยท์ˆ˜์ƒ‰์ด๋‹ค. 2) ํ”ผ๊ณ ์ธ์ด ๋ช…์‹œ์ ์œผ๋กœ ์••์ˆ˜์ˆ˜์ƒ‰์ ˆ์ฐจ์— ์ฐธ์—ฌ๋ฅผ ๊ฑฐ๋ถ€ํ•˜์˜€์Œ์—๋„ ํ”ผ๊ณ ์ธ์„ ๊ตฌ์น˜์†Œ๋กœ ์ธ์น˜ํ•˜์ง€ ์•Š๊ณ  ํ˜„์žฅ์— ์ˆ˜๊ฐ‘์„ ์ฑ„์šด ์ฑ„๋กœ ๋ถˆ๋ฒ• ๊ตฌ๊ธˆํ•˜์˜€๊ณ , ์ฐธ์—ฌ๋ฅผ ๊ฐ•์š”ํ•˜์˜€๋‹ค. ํ”ผ๊ณ ์ธ๊ณผ ๊ทธ์˜ ์ฒ˜๋Š” ๋ฐค์ƒˆ๋„๋ก ์••์ˆ˜์ˆ˜์ƒ‰์ ˆ์ฐจ์— ์ฐธ์—ฌ๋ฅผ ๊ฐ•์š”๋‹นํ•˜์˜€๊ณ  ์ž ์‹œ์˜ ์ˆ˜๋ฉด๋„ ํ—ˆ์šฉ๋˜์ง€ ์•Š์•˜๋‹ค. ์ด๋Š” ๊ฐ•์ œ์ˆ˜์‚ฌ ๋น„๋ก€์˜ ์›์น™ ๋ฐ ๊ณผ์ž‰๊ธˆ์ง€์›์น™์— ๋ฐ˜ํ•˜๋Š” ์œ„๋ฒ•ํ•œ ์ง‘ํ–‰์ด๋‹ค. 3) ๋˜ํ•œ ํ”ผ๊ณ ์ธ์˜ ๋ณ€ํ˜ธ์ธ์ด ํ˜„์žฅ์— ๋„์ฐฉํ•˜์—ฌ ์ฒดํฌ์˜์žฅ์˜ ๋“ฑ์‚ฌ๋ฅผ ์š”๊ตฌํ•˜์˜€์œผ๋‚˜ ์ˆ˜์‚ฌ๊ด€๋“ค์ด ์ด๋ฅผ ๊ฑฐ์ ˆํ•˜๋Š” ๋“ฑ ๋ณ€ํ˜ธ์ธ์˜ ์กฐ๋ ฅ์„ ๋ฐ›์„ ๊ถŒ๋ฆฌ๊ฐ€ ์›์ฒœ์ ์œผ๋กœ ์นจํ•ด๋˜์–ด ์œ„๋ฒ•ํ•œ ์ง‘ํ–‰์ด๋‹ค. ๋‚˜. ์ธ์ •๋˜๋Š” ์‚ฌ์‹ค(์••์ˆ˜์ˆ˜์ƒ‰ ์ ˆ์ฐจ์˜ ์ง„ํ–‰ ๊ณผ์ •) ํŒ์‹œ ์ฆ๊ฑฐ์— ์˜ํ•˜๋ฉด ๋‹ค์Œ๊ณผ ๊ฐ™์€ ์‚ฌ์‹ค์ด ์ธ์ •๋œ๋‹ค. 1) ๊ตญ์ •์›๊ณผ ๊ฒ€์ฐฐ์€ 2015. 11. 5. ์˜์žฅ ์ง‘ํ–‰์˜ ๋Œ€์ƒ์„ ํ”ผ๊ณ ์ธ์˜ ์‹ ์ฒด, ์ฃผ๊ฑฐ์ง€, ์ฃผ7) ๊ทผ๋ฌด์ง€ ๋‚ด ํ”ผ๊ณ ์ธ ์‚ฌ์šฉ ์บ๋น„๋„ทยท์ฑ…์ƒยท์„œ๋ž ๋“ฑ ๊ฐœ์ธ๊ณต๊ฐ„ ๋ฐ ๊ณต์šฉ๊ณต๊ฐ„ ์ค‘ ํ”ผ๊ณ ์ธ์ด ์‚ฌ์šฉํ•˜๋Š” ๋ฌผํ’ˆ, ํ”ผ๊ณ ์ธ ๋ช…์˜ ์ฐจ๋Ÿ‰(ํˆฌ๋ฆฌ์Šค๋ชจ), ํ”ผ๊ณ ์ธ ๋ช…์˜ ์˜คํ† ๋ฐ”์ด๋กœ ํ•˜๊ณ , ์••์ˆ˜ยท์ˆ˜์ƒ‰ยท๊ฒ€์ฆํ•  ๋ฌผ๊ฑด์„ ๋ฒ”์ฃ„์‚ฌ์‹ค๊ณผ ๊ด€๋ จ๋œ ๋ฌธ๊ฑด, ๊ธˆ์ „๊ฑฐ๋ž˜ ์ž๋ฃŒ, ๊ฐ์ข… ๋””์ง€ํ„ธ ์ •๋ณด์žฅ์น˜์™€ ๊ทธ ์žฅ์น˜์— ์ˆ˜๋ก๋œ ๋‚ด์šฉ ๋“ฑ์œผ๋กœ ํ•˜๋Š” ์••์ˆ˜์ˆ˜์ƒ‰๊ฒ€์ฆ์˜์žฅ์„ ๋ฐ›์•˜๊ณ , ๊ทธ ์˜์žฅ์—๋Š” ์•ผ๊ฐ„์ง‘ํ–‰์ด ๊ฐ€๋Šฅํ•˜๋‹ค๋Š” ๊ธฐ์žฌ๊ฐ€ ๋˜์–ด ์žˆ๋‹ค(์ฆ๊ฑฐ๊ธฐ๋ก ๋ณ„๊ถŒ 903์ชฝ ์ฐธ์กฐ). ๊ทธ๋ฆฌ๊ณ  ๊ฐ™์€ ๋‚  ํ”ผ๊ณ ์ธ์— ๋Œ€ํ•œ ์ฒดํฌ์˜์žฅ๋„ ๋ฐœ๋ถ€๋ฐ›์•˜๋‹ค. 2) ๊ตญ์ •์› ์ˆ˜์‚ฌ๊ด€๋“ค์€ 2015. 11. 13. 00:36๊ฒฝ ํ”ผ๊ณ ์ธ ์ฃผ๊ฑฐ์ง€ ์•ž์—์„œ ํ”ผ๊ณ ์ธ์„ ์ฒดํฌํ•˜์˜€๋‹ค. ์ˆ˜์‚ฌ๊ด€๋“ค์€ 2015. 11. 13. 00:41๊ฒฝ๋ถ€ํ„ฐ ์••์ˆ˜ยท์ˆ˜์ƒ‰์„ ๊ฐœ์‹œํ•˜์˜€์œผ๋ฉฐ, ๊ด‘๋ช…์‹œ โ—‡โ—‡๋™ ํ†ต์žฅ ๊ณต์†Œ์™ธ 7๊ณผ (๋ช…์นญ 1 ์ƒ๋žต)๋Œ€ํ•™๊ต (๋ช…์นญ 2 ์ƒ๋žต)๋Œ€ํ•™์› ๊ต์ˆ˜์ธ ๊ณต์†Œ์™ธ 10์ด ์••์ˆ˜ยท์ˆ˜์ƒ‰ ๊ณผ์ •์— ์ฐธ์—ฌํ•˜์˜€๋‹ค. ์••์ˆ˜์ˆ˜์ƒ‰์กฐ์„œ์—๋Š” โ€œํ”ผ๊ณ ์ธ์ด ์••์ˆ˜์ˆ˜์ƒ‰์˜์žฅ์˜ ์—ด๋žŒ์„ ๊ฑฐ๋ถ€ํ•˜๋ฉฐ ๊ณ ์„ฑ์„ ์ง€๋ฅด๊ธฐ ๋•Œ๋ฌธ์— ํ”ผ๊ณ ์ธ์˜ ์ฒ˜ ๊ณต์†Œ์™ธ 8์—๊ฒŒ ์˜์žฅ์„ ์—ด๋žŒ์‹œ์ผœ ์ฃผ์—ˆ๋‹ค. ๊ณต์†Œ์™ธ 8์€ ์˜์žฅ์„ ์—ด๋žŒํ•˜์˜€์œผ๋‚˜ โ€™์˜์žฅ ์—ด๋žŒ ํ™•์ธโ€˜ ์„œ๋ช…์„ ๊ฑฐ๋ถ€ํ•˜์˜€๋‹คโ€๊ณ  ๊ธฐ์žฌ๋˜์–ด ์žˆ๋‹ค. 3) ๊ตญ์ •์› ์ˆ˜์‚ฌ๊ด€๋“ค์€ 00:48๊ฒฝ ํ”ผ๊ณ ์ธ์˜ ์‹ ์ฒด์—์„œ ํœด๋Œ€์ „ํ™”ยทUSBยทSD ๋ฉ”๋ชจ๋ฆฌ์นด๋“œ ๋“ฑ์„ ์••์ˆ˜ํ•˜์˜€๋‹ค. 4) ํ”ผ๊ณ ์ธ์ด 00:50๊ฒฝ ๋ณ€ํ˜ธ์‚ฌ์—๊ฒŒ ์—ฐ๋ฝ์„ ํ•˜๋ผ๊ณ  ํ•˜์—ฌ ์ฒ˜ ๊ณต์†Œ์™ธ 8์ด ๋ณ€ํ˜ธ์‚ฌ์™€์˜ ์—ฐ๋ฝ์„ ๊ณ„์† ์‹œ๋„ํ•˜์˜€๋‹ค. 5) ์ˆ˜์‚ฌ๊ด€๋“ค์€ 00:58๊ฒฝ ํ”ผ๊ณ ์ธ์—๊ฒŒ ์••์ˆ˜์ˆ˜์ƒ‰ ์ฐธ์—ฌ์˜์‚ฌ๋ฅผ ๋ฌผ์–ด๋ณด์•˜์œผ๋‚˜ ํ”ผ๊ณ ์ธ์ด ์ผ์ฒด์˜ ์˜์‚ฌํ‘œ์‹œ๋ฅผ ํ•˜์ง€ ์•Š์•„ ๊ฑฐ์‹ค ์‹ํƒ ์˜์ž์— ํ”ผ๊ณ ์ธ์„ ์•‰๊ฒŒ ํ•œ ํ›„ ๊ณต์†Œ์™ธ 8์„ ์ฐธ์—ฌํ•˜๊ฒŒ ํ•˜๊ณ  ์••์ˆ˜์ˆ˜์ƒ‰์„ ์ง„ํ–‰ํ•˜์˜€๋‹ค. 01:01๊ฒฝ ํ”ผ๊ณ ์ธ์—๊ฒŒ ๊ทผ๋ฌด์ง€ ์••์ˆ˜์ˆ˜์ƒ‰์— ์ฐธ์—ฌํ•  ๊ฒƒ์ธ์ง€๋„ ๋ฌผ์–ด๋ณด์•˜์œผ๋‚˜, ํ”ผ๊ณ ์ธ์€ ์•„๋ฌด๋Ÿฐ ๋Œ€๋‹ต์„ ํ•˜์ง€ ์•Š์•˜๋‹ค. 6) ์••์ˆ˜์ˆ˜์ƒ‰์€ ๊ฑฐ์‹ค, ์•ˆ๋ฐฉ, ์ž‘์€๋ฐฉ, ํ™”์žฅ์‹ค, ๋ฒ ๋ž€๋‹ค, ์‹ ๋ฐœ์žฅ ์ˆœ์œผ๋กœ ์ง„ํ–‰๋˜์—ˆ๋‹ค. ์••์ˆ˜ยท์ˆ˜์ƒ‰์ด ์ง„ํ–‰๋˜๋Š” ๋™์•ˆ ํ”ผ๊ณ ์ธ์€ ์‹ํƒ ์˜์ž์— ์•‰์•„ ์žˆ์—ˆ๊ณ , ๊ณต์†Œ์™ธ 8์€ ์••์ˆ˜ยท์ˆ˜์ƒ‰ ๊ณผ์ •์„ ์ง€์ผœ๋ณด๋ฉฐ ๋”ฐ๋ผ๋‹ค๋‹ˆ๊ธฐ๋„ ํ•˜์˜€๋‹ค. 7) ์ˆ˜์‚ฌ๊ด€๋“ค์€ 01:11๊ฒฝ ~ 03:59๊ฒฝ ๊ฑฐ์‹ค์„ ์ˆ˜์ƒ‰ํ•˜์—ฌ ์„œ์ , ํ†ต์žฅ, USB, ์™ธ์žฅํ•˜๋“œ๋””์Šคํฌ ๋“ฑ์„ ์••์ˆ˜ํ•˜์˜€๋‹ค. 02:24๊ฒฝ ํ”ผ๊ณ ์ธ๊ณผ ๊ณต์†Œ์™ธ 8์—๊ฒŒ ๋””์ง€ํ„ธ ์ €์žฅ๋งค์ฒด๋Š” ๋ด‰์ธ ๋“ฑ์„ ํ•˜๊ณ  ์›๋ณธ์„ ์™ธ๋ถ€๋กœ ๋ฐ˜์ถœํ•  ์ˆ˜ ์žˆ๋‹ค๊ณ  ์„ค๋ช…ํ•˜์˜€๋‹ค. 8) ๊ฐ™์€ ๋‚  03:44๊ฒฝ ๋ณ€ํ˜ธ์‚ฌ ๊ณต์†Œ์™ธ 9๊ฐ€ ๋„์ฐฉํ•˜์˜€๋‹ค. ๋ณ€ํ˜ธ์‚ฌ๋Š” ์˜์žฅ์— ๋Œ€ํ•˜์—ฌ ๋“ฑ์‚ฌ๋ฅผ ์š”์ฒญํ•˜๋‹ค๊ฐ€ ์ด๋Ÿฌํ•œ ์š”๊ตฌ๊ฐ€ ๊ฑฐ์ ˆ๋˜์ž, ์ค‘์š”๋ถ€๋ถ„์„ ๋ฉ”๋ชจํ•˜๊ฒŒ ํ•ด๋‹ฌ๋ผ๊ณ  ํ•˜์—ฌ ๊ณต์†Œ์™ธ 8์ด ์˜์žฅ์˜ ์ค‘์š” ๋ถ€๋ถ„์„ ๋ฉ”๋ชจํ•˜์˜€๋‹ค. 03:58๊ฒฝ ๋ณ€ํ˜ธ์‚ฌ๋Š” ํ˜„์žฅ์„ ๋– ๋‚ฌ๋‹ค. 9) ์ˆ˜์‚ฌ๊ด€๋“ค์€ 05:30๊ฒฝ ํ”ผ๊ณ ์ธ๊ณผ ๊ณต์†Œ์™ธ 8์—๊ฒŒ ํ”ผ๊ณ ์ธ์˜ ์ฐจ๋Ÿ‰ ์••์ˆ˜ยท์ˆ˜์ƒ‰์— ์ฐธ์—ฌํ•  ๊ฒƒ์ธ์ง€ ๋ฌผ์–ด๋ณด์•˜์œผ๋‚˜ ํ”ผ๊ณ ์ธ์€ ๋Œ€๋‹ตํ•˜์ง€ ์•Š๊ณ , ๊ณต์†Œ์™ธ 8์€ ์ฐธ์„ํ•˜์ง€ ์•Š๊ฒ ๋‹ค๊ณ  ํ•˜์˜€๋‹ค. 05:42๊ฒฝ ๊ตญ์ •์› ์ˆ˜์‚ฌ๊ด€๊ณผ ๊ณต์†Œ์™ธ 7์€ ํ”ผ๊ณ ์ธ์˜ ์ฐจ๋Ÿ‰์ด ์ฃผ์ฐจ๋˜์–ด ์žˆ๋Š” โ–ณโ–ณ๋Œ€ํ•™๊ต๋กœ ์ถœ๋ฐœํ•˜์˜€๋‹ค. 06:15๊ฒฝ ~ 06:30๊ฒฝ ๊ณต์†Œ์™ธ 7์˜ ์ฐธ์—ฌ ํ•˜์— ํ”ผ๊ณ ์ธ์˜ ์ฐจ๋Ÿ‰์„ ์••์ˆ˜ยท์ˆ˜์ƒ‰ํ•˜์—ฌ LG ๋…ธํŠธ๋ถ, ์•„์ดํŒจ๋“œ, ์‚ผ์„ฑ ๊ฐค๋Ÿญ์‹œํƒญ, USB, SD์นด๋“œ ๋“ฑ์„ ์••์ˆ˜ํ•˜์˜€๋‹ค. ์„ ๋ณ„์••์ˆ˜๊ฐ€ ์–ด๋ ค์šด ์ƒํ™ฉ์ด๋ผ, ์œ„ ์••์ˆ˜๋ฌผ๋“ค์„ ์ƒ์ž์— ๋„ฃ๊ณ  ๊ณต์†Œ์™ธ 7์ด ๋ด‰์ธํ•˜์˜€๋‹ค. 10) ์ˆ˜์‚ฌ๊ด€๋“ค์€ 06:58๊ฒฝ ๊ณต์†Œ์™ธ 7๊ณผ ํ•จ๊ป˜ ํ”ผ๊ณ ์ธ์˜ ์ง‘์œผ๋กœ ์ด๋™ํ•˜์—ฌ ํ”ผ๊ณ ์ธ๊ณผ ๊ณต์†Œ์™ธ 8์—๊ฒŒ ๋ด‰์ธ์„ ํ™•์ธ์‹œํ‚จ ํ›„ ์„ ๋ณ„์••์ˆ˜๋ฅผ ์‹ค์‹œํ•˜์˜€๋‹ค. ๊ทธ๋Ÿฐ๋ฐ ๊ทธ ํ›„ ๊ณต์†Œ์™ธ 7์€ ์–ด๋จธ๋‹ˆ๋ฅผ ๋ณ‘์›์— ๋ชจ์…”๋‹ค ๋“œ๋ ค์•ผ ํ–ˆ๊ธฐ ๋•Œ๋ฌธ์— ์•„์นจ 7์‹œ๊ฐ€ ์กฐ๊ธˆ ๋„˜์–ด ํ˜„์žฅ์„ ๋– ๋‚ฌ๋‹ค. 11) ์ˆ˜์‚ฌ๊ด€๋“ค์€ 07:48 ํ”ผ๊ณ ์ธ๊ณผ ๊ณต์†Œ์™ธ 8์—๊ฒŒ ์˜คํ† ๋ฐ”์ด ์••์ˆ˜ยท์ˆ˜์ƒ‰์— ์ฐธ์—ฌํ•  ๊ฒƒ์ธ์ง€ ๋ฌผ์–ด๋ณด์•˜์œผ๋‚˜ ํ”ผ๊ณ ์ธ๊ณผ ๊ณต์†Œ์™ธ 8์€ ์••์ˆ˜์ˆ˜์ƒ‰์˜์žฅ์„ ๋“ฑ์‚ฌํ•ด์ฃผ์ง€ ์•Š์œผ๋ฉด ์ฐธ์—ฌํ•˜์ง€ ์•Š๊ฒ ๋‹ค๋ฉฐ ๋ช…์‹œ์ ์œผ๋กœ ์ฐธ์—ฌ๊ฑฐ์ ˆ์˜์‚ฌ๋ฅผ ๋ฐํ˜”๋‹ค. 07:53 ~ 07:59๊ฒฝ ํ”ผ๊ณ ์ธ์˜ ์ฃผ๊ฑฐ์ง€ ์ฃผ์ฐจ์žฅ์— ์ฃผ์ฐจ๋˜์–ด ์žˆ๋˜ ํ”ผ๊ณ ์ธ์˜ ์˜คํ† ๋ฐ”์ด๋ฅผ ์••์ˆ˜ยท์ˆ˜์ƒ‰ํ•˜์˜€๊ณ , ์•ˆ์žฅ ๋ฐ‘ ๋ณด๊ด€ํ•จ์—์„œ ์™ธ์žฅํ˜• ํ•˜๋“œ๋””์Šคํฌ, USB, ํ•˜๋“œ๋””์Šคํฌ ๋“ฑ์„ ์••์ˆ˜ํ•˜์˜€๊ณ  ๊ทธ ์••์ˆ˜๋ฌผ์„ ๋ด‰ํˆฌ์— ๋„ฃ์–ด ๋ด‰์ธํ•˜์˜€๋‹ค. 12) ๊ตญ์ •์› ์ˆ˜์‚ฌ๊ด€๋“ค์€ ์œ„ ์••์ˆ˜ยท์ˆ˜์ƒ‰ ๊ณผ์ • ์ „์ฒด๋ฅผ ๋…นํ™”ํ•˜์˜€๋‹ค(๊ทธ ์บก์ณ ์‚ฌ์ง„๋“ค ์ค‘ ์ผ๋ถ€๊ฐ€ ์ด ์‚ฌ๊ฑด์— ์ œ์ถœ๋˜์—ˆ๋‹ค). ๋‹ค. ํŒ๋‹จ 1) ํ”ผ๊ณ ์ธ์˜ ์ฃผ๊ฑฐ์ง€ ์••์ˆ˜ยท์ˆ˜์ƒ‰ ์œ„ ์ธ์ •์‚ฌ์‹ค์— ์˜ํ•˜๋ฉด, ํ”ผ๊ณ ์ธ๊ณผ ๊ณต์†Œ์™ธ 8์€ ๊ตญ์ •์› ์ˆ˜์‚ฌ๊ด€๋“ค๋กœ๋ถ€ํ„ฐ ์••์ˆ˜ยท์ˆ˜์ƒ‰ ์˜์žฅ์„ ์ œ์‹œ๋ฐ›๊ณ  ์ฐธ์—ฌ์˜ ๊ธฐํšŒ๋ฅผ ๋ณด์žฅ๋ฐ›์•„ ๊ทธ์— ์ฐธ์—ฌํ•œ ๊ฒƒ์œผ๋กœ ๋ณด์ด๋ฏ€๋กœ ์ด ๋ถ€๋ถ„ ์••์ˆ˜ยท์ˆ˜์ƒ‰์€ ์ ๋ฒ•ํ•˜๋‹ค. 2) ํ”ผ๊ณ ์ธ ์ฐจ๋Ÿ‰, ์˜คํ† ๋ฐ”์ด ์••์ˆ˜ยท์ˆ˜์ƒ‰ ์œ„ ์ธ์ •์‚ฌ์‹ค๊ณผ ์ฆ๊ฑฐ์— ์˜ํ•ด ์ธ์ •๋˜๋Š” ๋‹ค์Œ์˜ ์‚ฌ์ •์„ ์ข…ํ•ฉํ•˜๋ฉด, ์ด ์‚ฌ๊ฑด ์ฐจ๋Ÿ‰๊ณผ ์˜คํ† ๋ฐ”์ด์˜ ์••์ˆ˜ยท์ˆ˜์ƒ‰ ์ ˆ์ฐจ๊ฐ€ ์œ„๋ฒ•ํ•˜๋‹ค๊ณ  ํ•  ์ˆ˜๋Š” ์—†๋‹ค. ๊ฐ€) ํ”ผ๊ณ ์ธ๊ณผ ๊ณต์†Œ์™ธ 8์€ ์ฃผ๊ฑฐ์ง€ ์••์ˆ˜ยท์ˆ˜์ƒ‰์— ์ฐธ์—ฌํ•˜์˜€๊ณ , ๊ตญ์ •์› ์ˆ˜์‚ฌ๊ด€๋“ค์ด ํ”ผ๊ณ ์ธ๊ณผ ๊ณต์†Œ์™ธ 8์—๊ฒŒ ์ˆ˜ํšŒ์— ๊ฑธ์ณ ์ฐจ๋Ÿ‰๊ณผ ์˜คํ† ๋ฐ”์ด์˜ ์••์ˆ˜ยท์ˆ˜์ƒ‰์— ์ฐธ์—ฌํ•  ๊ฒƒ์ธ์ง€ ๋ฌผ์–ด๋ณด๋Š” ๋“ฑ ํ”ผ๊ณ ์ธ๊ณผ ๊ณต์†Œ์™ธ 8์˜ ์ฐธ์—ฌ๊ถŒ์„ ๋ณด์žฅํ•˜๊ธฐ ์œ„ํ•œ ์กฐ์น˜๋ฅผ ์ทจํ–ˆ๋‹ค. ๋‚˜) ์ฐจ๋Ÿ‰์˜ ์••์ˆ˜ยท์ˆ˜์ƒ‰์—๋Š” ํ†ต์žฅ์ธ ๊ณต์†Œ์™ธ 7์ด ์ฐธ์—ฌํ•˜์˜€๋‹ค. ๊ณต์†Œ์™ธ 7์€ โ€˜๊ตญ์ •์› ์ˆ˜์‚ฌ๊ด€์ด ๋ฐฉ๋งˆ๋‹ค ์ž์‹ ์„ ๋ฐ๋ฆฌ๊ณ  ๋‹ค๋‹ˆ๋ฉด์„œ ์••์ˆ˜ยท์ˆ˜์ƒ‰๊ณผ์ •์„ ๋ณด์—ฌ์คฌ๊ณ , ํ˜„์žฅ์„ ์ดํƒˆํ•˜๊ธฐ ์ „๊นŒ์ง€๋Š” ์••์ˆ˜ยท์ˆ˜์ƒ‰๊ณผ์ •์„ ์ฒ˜์Œ๋ถ€ํ„ฐ ๋๊นŒ์ง€ ์ง€์ผœ๋ดค๋‹ค. ๋ฐ–์— ๋‚˜๊ฐ„ ์ ์ด ์—†๊ณ  ์ฐจ๋Ÿ‰์˜ ์••์ˆ˜ยท์ˆ˜์ƒ‰์‹œ ๋™ํ–‰ํ•˜์—ฌ ์••์ˆ˜ยท์ˆ˜์ƒ‰ ์ ˆ์ฐจ๋ฅผ ์ง€์ผœ๋ณด์•˜๋‹คโ€™๋ผ๊ณ  ์ง„์ˆ ํ•˜๊ณ  ์žˆ์œผ๋ฏ€๋กœ ๊ณต์†Œ์™ธ 7์ด ์••์ˆ˜ยท์ˆ˜์ƒ‰ ์ ˆ์ฐจ์— ์ œ๋Œ€๋กœ ์ฐธ์—ฌํ•˜์ง€ ๋ชปํ•œ ๊ฒƒ์œผ๋กœ ๋ณด์ด์ง€๋Š” ์•Š๋Š”๋‹ค. ๋‹ค) ํ”ผ๊ณ ์ธ์˜ ์˜คํ† ๋ฐ”์ด ์••์ˆ˜ยท์ˆ˜์ƒ‰์€ ๊ณต์†Œ์™ธ 7์ด ์ฐธ์—ฌํ•˜์ง€ ์•„๋‹ˆํ•˜์—ฌ ์ˆ˜์‚ฌ๊ด€๋“ค๋งŒ์— ์˜ํ•ด ์ด๋ฃจ์–ด์ง€๊ธฐ๋Š” ํ–ˆ๋‹ค. ๊ทธ๋Ÿฌ๋‚˜ ์ด ์‚ฌ๊ฑด ์˜คํ† ๋ฐ”์ด ์••์ˆ˜ยท์ˆ˜์ƒ‰์€ ์ž์ •๊ฒฝ๋ถ€ํ„ฐ ์‹œ์ž‘๋œ ํ”ผ๊ณ ์ธ์˜ ์ฒดํฌ์™€ ์ˆ˜ ์‹œ๊ฐ„์— ๊ฑธ์นœ ์ฃผ๊ฑฐ์ง€ ์••์ˆ˜ยท์ˆ˜์ƒ‰, 06์‹œ ๋ฌด๋ ต์˜ ์ฐจ๋Ÿ‰ ์••์ˆ˜ยท์ˆ˜์ƒ‰์— ์ด์–ด ์—ฐ์†์ ์ธ ์ผ๋ จ์˜ ๊ณผ์ •์œผ๋กœ ์ด๋ฃจ์–ด์ง„ ๊ฒƒ์ด๋‹ค. ํ”ผ๊ณ ์ธ์˜ ์˜คํ† ๋ฐ”์ด ์••์ˆ˜ยท์ˆ˜์ƒ‰์„ ํ•˜๊ธฐ ์ „์— ์ˆ˜์‚ฌ๊ด€๋“ค์€ ํ”ผ๊ณ ์ธ๊ณผ ์ฒ˜ ๊ณต์†Œ์™ธ 8์—๊ฒŒ ์ฐธ์—ฌํ•˜๋ผ๊ณ  ์•ฝ 20๋ถ„๊ฐ„ ์ˆ˜์ฐจ๋ก€ ์„ค๋“ํ•˜๋Š” ๋“ฑ ์ถฉ๋ถ„ํžˆ ๊ธฐํšŒ๋ฅผ ์ฃผ์—ˆ์Œ์—๋„ ํ”ผ๊ณ ์ธ๊ณผ ๊ณต์†Œ์™ธ 8์€ ์ด๋ฅผ ๋ช…์‹œ์ ์œผ๋กœ ๊ฑฐ๋ถ€ํ•˜์˜€๋‹ค. ๊ทธ๋ฆฌ๊ณ  ์ด ์‚ฌ๊ฑด ์˜คํ† ๋ฐ”์ด๋Š” ์••์ˆ˜ยท์ˆ˜์ƒ‰์ฒ˜๋ถ„์„ ๋ฐ›๋Š” ๋‹น์‚ฌ์ž์ธ ํ”ผ๊ณ ์ธ ๋ณธ์ธ์ด ์†Œ์œ ยท๊ด€๋ฆฌํ•˜๋Š” ๋ฌผ๊ฑด์œผ๋กœ ๊ฐ„์ˆ˜์ž๊ฐ€ ๋”ฐ๋กœ ์žˆ๋Š” ๊ฒƒ์ด ์•„๋‹ˆ๊ณ , ํ”ผ๊ณ ์ธ์˜ ์ฃผ๊ฑฐ์ง€ ์•„ํŒŒํŠธ ์ฃผ์ฐจ์žฅ์— ์„ธ์›Œ์ ธ ์žˆ๋˜ ๊ฒƒ์ด๋ฉฐ, ๊ทธ ์ˆ˜์ƒ‰์˜ ๋Œ€์ƒ๋„ ์˜คํ† ๋ฐ”์ด์˜ ์™ธ๋ถ€์™€ ์•ˆ์žฅ ๋ฐ‘์˜ ์ˆ˜๋‚ฉํ•จ ์ •๋„์— ๋ถˆ๊ณผํ•˜๋‹ค(์ด ์ ์—์„œ โ–ณโ–ณ๋Œ€ํ•™๊ต์— ์ฃผ์ฐจ๋˜์–ด ์žˆ๋˜ ํ”ผ๊ณ ์ธ์˜ ์ฐจ๋Ÿ‰์˜ ๊ฒฝ์šฐ์™€๋„ ๋‹ค๋ฅด๋‹ค). ๋”ฐ๋ผ์„œ ์ด ๊ฒฝ์šฐ ํ˜•์‚ฌ์†Œ์†ก๋ฒ• ์ œ123์กฐ์ฃผ8) ์—์„œ ๊ทœ์ •ํ•˜๋Š” โ€˜๊ฐ„์ˆ˜์ž๊ฐ€ ์žˆ๋Š” ์„ ์ฐจโ€™๋กœ์„œ ํ”ผ๊ณ ์ธ ๋ณธ์ธ ์™ธ์— ๋‹ค๋ฅธ ๊ฐ„์ˆ˜์ž๋‚˜ ์ธ๊ฑฐ์ธ(์ธ๊ฑฐ์ธ) ๋“ฑ์„ ๋ณ„๋„๋กœ ์ฐธ์—ฌ์‹œ์ผœ์•ผ๋งŒ ํ•œ๋‹ค๊ณ ๋Š” ํ•  ์ˆ˜ ์—†๋‹ค. ๋˜ํ•œ ๋‹น์ดˆ ์ˆ˜์‚ฌ๊ด€์€ ํ”ผ์ฒ˜๋ถ„์ž์ธ ํ”ผ๊ณ ์ธ์˜ ๊ถŒ์ต๊ณผ ์ง‘ํ–‰์˜ ์ ์ •์„ฑ์„ ๋‹ด๋ณดํ•˜๊ธฐ ์œ„ํ•˜์—ฌ ์ธ๊ฑฐ์ธ์ธ ํ†ต์žฅ ๊ณต์†Œ์™ธ 7์„ ์ด ์‚ฌ๊ฑด ์˜์žฅ์ง‘ํ–‰์— ์ฐธ์—ฌ์‹œ์ผฐ๋˜ ๊ฒƒ์ธ๋ฐ, ๊ณต์†Œ์™ธ 7์ด ์˜คํ† ๋ฐ”์ด ์ด์ „์˜ ์••์ˆ˜ยท์ˆ˜์ƒ‰ ๊ณผ์ •์—๋Š” ๋ชจ๋‘ ์ ๋ฒ•ํ•˜๊ฒŒ ์ฐธ์—ฌํ•˜์˜€๊ธฐ ๋•Œ๋ฌธ์— ๊ตญ์ •์› ์ˆ˜์‚ฌ๊ด€๋“ค๋กœ์„œ๋Š” ๊ณต์†Œ์™ธ 7์ด ์ดํ›„์˜ ์••์ˆ˜ยท์ˆ˜์ƒ‰ ๊ณผ์ •์—๋„ ์ฐธ์—ฌํ•  ๊ฒƒ์ด๋ผ๊ณ  ๊ธฐ๋Œ€ํ•˜๋Š” ๊ฒƒ์ด ๋‹น์—ฐํ•˜์˜€๋‹ค๊ณ  ๋ณด์ด๊ณ , ๊ฐœ์ธ์ ์ธ ์‚ฌ์ •์œผ๋กœ ๊ฐ‘์ž๊ธฐ ํ˜„์žฅ์„ ์ดํƒˆํ•  ๊ฒƒ์„ ์˜ˆ์ƒํ•˜๊ธฐ๋Š” ์–ด๋ ค์› ์„ ๊ฒƒ์œผ๋กœ ๋ณด์ธ๋‹ค(๊ณต์†Œ์™ธ 7 ๋˜ํ•œ ์••์ˆ˜ยท์ˆ˜์ƒ‰์ด ์•„์นจ๊นŒ์ง€ ์ด์–ด์งˆ ์ •๋„๋กœ ๊ธธ์–ด์งˆ ๊ฒƒ์ด๋ผ๊ณ  ์˜ˆ์ƒํ•˜๊ธฐ๋Š” ์–ด๋ ค์› ์„ ๊ฒƒ์ด๊ณ  ๋ถ€๋ชจ๋‹˜ ๋ณ‘์›์ง„๋ฃŒ ๋•Œ๋ฌธ์— ์•„์นจ 07์‹œ๊ฒฝ ๊ท€๊ฐ€ํ•˜๊ฒŒ ๋˜์—ˆ๋‹ค). ํฌ๋ Œ์‹ ์ž…ํšŒ์ธ ๊ณต์†Œ์™ธ 10๋„ ๋‹น์‹œ ํฌ๋ Œ์‹ ๋ถ„์„ ์ค‘์ด์–ด์„œ ์ฐธ์—ฌํ•˜๊ธฐ ๊ณค๋ž€ํ•œ ์ƒํ™ฉ์ด์—ˆ๊ณ , ์‹œ๊ฐ„๋„ 07์‹œ๊ฐ€ ์กฐ๊ธˆ ๋„˜์—ˆ์„ ๋•Œ๋ผ ๋‹ค๋ฅธ ์ธ๊ฑฐ์ธ์ด๋‚˜ ์ง€๋ฐฉ๊ณต๊ณต๋‹จ์ฒด ์ง์›์„ ๊ธ‰ํžˆ ์„ญ์™ธํ•˜๊ธฐ๋„ ๊ณค๋ž€ํ•œ ์ƒํ™ฉ์ด์—ˆ๋‹ค. ๋”ฐ๋ผ์„œ ๊ตญ์ •์› ์ˆ˜์‚ฌ๊ด€๋“ค์ด ์˜๋„์ ์œผ๋กœ ๊ณต์†Œ์™ธ 7์˜ ์ฐธ์—ฌ๋ฅผ ๋ฐฐ์ œํ•œ ๊ฒƒ๋„ ์•„๋‹ˆ๋‹ค. ํ”ผ๊ณ ์ธ์˜ ์˜คํ† ๋ฐ”์ด๋Š” ํ”ผ๊ณ ์ธ ์•„ํŒŒํŠธ ์ฃผ์ฐจ์žฅ์— ์ฃผ์ฐจ๋˜์–ด ์žˆ๋˜ ๊ฒƒ์œผ๋กœ, ํ”ผ๊ณ ์ธ๊ณผ ๊ณต์†Œ์™ธ 8์€ ๊ทธ์— ๋Œ€ํ•œ ์••์ˆ˜ยท์ˆ˜์ƒ‰ ๋„์ค‘ ์–ธ์ œ๋“ ์ง€ ์ฐธ์—ฌํ•  ์ˆ˜ ์žˆ์—ˆ์„ ๊ฒƒ์ด๊ณ , ํ”ผ๊ณ ์ธ๊ณผ ๊ณต์†Œ์™ธ 8์—๊ฒŒ ์ฐธ์—ฌ์˜์‚ฌ๋ฅผ ํ™•์ธํ•  ๋•Œ๋ถ€ํ„ฐ ์˜คํ† ๋ฐ”์ด์— ๋Œ€ํ•œ ์••์ˆ˜ยท์ˆ˜์ƒ‰์„ ์ข…๋ฃŒํ•œ ์‹œ๊ฐ„๊นŒ์ง€ ์•ฝ 11๋ถ„ ๋ฐ–์— ๊ฒฝ๊ณผํ•˜์ง€ ์•Š์•˜์œผ๋ฉฐ, ์••์ˆ˜ยท์ˆ˜์ƒ‰ ์ž์ฒด๋Š” 6๋ถ„ ๋™์•ˆ ์ด๋ฃจ์–ด์กŒ๋‹ค(์ˆ˜์‚ฌ๊ด€์€ ๊ทธ ๊ณผ์ •์„ ์ดฌ์˜ํ•ด ๋†“๊ธฐ๋„ ํ•˜์˜€๋‹ค). ์ด๋Ÿฌํ•œ ์งง์€ ์‹œ๊ฐ„ ๋™์•ˆ ์ˆ˜์‚ฌ๊ด€๋“ค์ด ์–ด๋– ํ•œ ์กฐ์ž‘์„ ๊ฐ€ํ•  ์ˆ˜ ์žˆ์—ˆ์„ ๊ฒƒ์œผ๋กœ๋Š” ๋ณด์ด์ง€ ์•Š๋Š”๋‹ค. ๋˜ํ•œ ์ˆ˜์‚ฌ๊ด€๋“ค์€ ์••์ˆ˜ยท์ˆ˜์ƒ‰ํ•œ ๋ฌผ๊ฑด๋“ค์„ ๊ณง๋ฐ”๋กœ ๋ด‰์ธํ•˜์—ฌ ํ”ผ๊ณ ์ธ์˜ ์ง‘์œผ๋กœ ๋“ค๊ณ  ์™€์„œ ํ”ผ๊ณ ์ธ๊ณผ ๊ณต์†Œ์™ธ 8์—๊ฒŒ ํ™•์ธ์‹œ์ผœ ์ฃผ์—ˆ๊ณ  ํฌ๋ Œ์‹ ์ž‘์—…์„ ๊ณ„์†ํ•˜์˜€๋‹ค. ์ด๋Ÿฌํ•œ ์ ์„ ๋ชจ๋‘ ์ข…ํ•ฉํ•˜๋ฉด, ์ด ์‚ฌ๊ฑด ์••์ˆ˜ยท์ˆ˜์ƒ‰ ๊ณผ์ •์— ์ง‘ํ–‰ ์ฐธ์—ฌ์™€ ๊ด€๋ จํ•œ ํŠน๋ณ„ํ•œ ์œ„๋ฒ•์€ ์—†๋Š” ๊ฒƒ์œผ๋กœ ๋ณด์ธ๋‹ค. 3) ํ˜•์‚ฌ์†Œ์†ก๋ฒ•์ƒ ์••์ˆ˜ยท์ˆ˜์ƒ‰์˜์žฅ ์ง‘ํ–‰ํ˜„์žฅ์— ๊ทธ ์ฒ˜๋ถ„ ๋‹น์‚ฌ์ž์ด์ž ์ฃผ๊ฑฐ์ฃผ์ธ ํ”ผ๊ณ ์ธ์ด ์ฐธ์—ฌํ•˜๋„๋ก ํ•˜๋Š” ๊ฒƒ์€ ๋‹น์‚ฌ์ž๋ฅผ ๋ณดํ˜ธํ•˜๊ณ  ์˜์žฅ์ง‘ํ–‰ ์ ˆ์ฐจ์˜ ์ ์ •์„ฑ์„ ๋ณด์žฅํ•˜๋ ค๋Š” ๊ฒƒ์ธ ์ , ํ˜•์‚ฌ์†Œ์†ก๋ฒ• ์ œ125์กฐ๋Š” ์•ผ๊ฐ„์ง‘ํ–‰์ด ๊ฐ€๋Šฅํ•œ ๊ฒƒ์„ ์ „์ œ๋กœ ๊ทœ์ •ํ•˜๊ณ  ์žˆ๊ณ  ์ด ์‚ฌ๊ฑด ์••์ˆ˜์ˆ˜์ƒ‰ ์˜์žฅ์— ์•ผ๊ฐ„์ง‘ํ–‰์ด ๊ฐ€๋Šฅํ•˜๋‹ค๋Š” ๊ธฐ์žฌ๊ฐ€ ๋˜์–ด ์žˆ๋Š” ์ , ํ”ผ๊ณ ์ธ์ด ์••์ˆ˜๋Œ€์ƒ๋ฌผ์„ ์€๋‹‰ยทํ›ผ์†ํ•˜๋Š” ๊ฒƒ์„ ๋ฐฉ์ง€ํ•˜๊ธฐ ์œ„ํ•˜์—ฌ ์••์ˆ˜ยท์ˆ˜์ƒ‰์ด ์ „๊ฒฉ์ ์œผ๋กœ ์ด๋ฃจ์–ด์ ธ์•ผ ํ•  ํ•„์š”์„ฑ์ด ์žˆ์—ˆ๋˜ ๊ฒƒ์œผ๋กœ ๋ณด์ด๋Š” ์ , ํ”ผ๊ณ ์ธ๊ณผ ๊ทธ ๊ฐ€์กฑ์ด ๋‹น์žฅ ๊ฐ์ˆ˜ํ•ด์•ผ ํ•˜๋Š” ๋ถˆํŽธํ•จ๋„ ์žˆ์œผ๋‚˜ ์••์ˆ˜ยท์ˆ˜์ƒ‰์ด ์ง„ํ–‰๋œ ๊ธฐ๊ฐ„์ด ํ•˜๋ฃจ ์ด๋‚ด(ํฌ๋ Œ์‹ ๊ณผ์ •์„ ํฌํ•จํ•˜์—ฌ ์ด 17์‹œ๊ฐ„ ์ •๋„)๋กœ์„œ ํ”ผ๊ณ ์ธ๊ณผ ๊ทธ ๊ฐ€์กฑ์ด ์ž…์–ด์•ผ ํ•  ํ”ผํ•ด์— ๋น„ํ•ด์„œ ๊ทธ๋กœ ์ธํ•ด ๋‹ฌ์„ฑํ•  ์ˆ˜ ์žˆ๋Š” ๋ฒ•์ต์ด ๋” ํฐ ๊ฒƒ์œผ๋กœ ๋ณด์ด๋Š” ์ , ์ˆ˜์ผ์— ๊ฑธ์ณ ์••์ˆ˜ยท์ˆ˜์ƒ‰์„ ํ•˜๋Š” ๊ฒƒ๋ณด๋‹ค๋Š” ์—ฐ์†์ ์œผ๋กœ ์••์ˆ˜ยท์ˆ˜์ƒ‰์„ ํ•˜๊ณ  ํ•˜๋ฃจ ๋งŒ์— ๋งˆ์น˜๋Š” ๊ฒƒ์ด ํ”ผ๊ณ ์ธ๊ณผ ๊ทธ ๊ฐ€์กฑ๋“ค์˜ ์ฃผ๊ฑฐ์˜ ํ‰์˜จ์„ ์ƒ๋Œ€์ ์œผ๋กœ ๋œ ์นจํ•ดํ•˜๋Š” ๊ฒƒ์œผ๋กœ ๋ณด์ด๋Š” ์  ๋“ฑ์„ ์ข…ํ•ฉํ•˜๋ฉด ์ด ์‚ฌ๊ฑด ์••์ˆ˜ยท์ˆ˜์ƒ‰์ด ๋ฐค์ƒˆ ์žฅ์‹œ๊ฐ„ ๊ณ„์†๋˜์—ˆ๋‹ค๊ณ  ํ•ด์„œ ๋น„๋ก€์˜ ์›์น™ ๋ฐ ๊ณผ์ž‰๊ธˆ์ง€์›์น™์— ๋ฐ˜ํ•˜๋Š” ๊ฒƒ์œผ๋กœ ๋ณด์ด์ง€๋Š” ์•Š๋Š”๋‹ค. 4) ํ”ผ๊ณ ์ธ์ด ์ฒดํฌ๋œ ํ›„ ๊ณง๋ฐ”๋กœ ์„œ์šธ๊ตฌ์น˜์†Œ๋กœ ์ด๋™๋˜์ง€ ์•Š๊ณ  ํ˜„์žฅ(์ฃผ๊ฑฐ์ง€)์—์„œ ์†์„ ๋’ค๋กœ ํ•˜์—ฌ ์ˆ˜๊ฐ‘์„ ์ฑ„์šด ์ฑ„๋กœ ์žˆ์—ˆ๋˜ ์‚ฌ์‹ค, ์ด๋Ÿฌํ•œ ์ƒํƒœ๊ฐ€ ์ƒˆ๋ฒฝ 04์‹œ๊ฒฝ๊นŒ์ง€ ์ด์–ด์ง„ ๊ฒƒ์€ ์ธ์ •๋œ๋‹ค. ๊ทธ๋Ÿฌ๋‚˜ ์‚ดํ”ผ๊ฑด๋Œ€, ํ”ผ๊ณ ์ธ์ด ๋’ค์— ์ฐจ๋Ÿ‰๊ณผ ์˜คํ† ๋ฐ”์ด ์••์ˆ˜ยท์ˆ˜์ƒ‰์—๋Š” ์ฐธ์—ฌ๊ฑฐ์ ˆ ์˜์‚ฌ๋ฅผ ํ‘œ์‹œํ•˜์˜€์œผ๋‚˜ ์ดˆ๋ฐ˜์— ์ฃผ๊ฑฐ์ง€์— ๋Œ€ํ•œ ์••์ˆ˜ยท์ˆ˜์ƒ‰ ๋‹น์‹œ์—๋Š” ์ฐธ์—ฌ๊ฑฐ์ ˆ ์˜์‚ฌ๋ฅผ ๋ช…์‹œ์ ์œผ๋กœ ํ‘œ์‹œํ•˜์ง€๋Š” ์•Š์•˜์œผ๋ฏ€๋กœ ํ”ผ๊ณ ์ธ์„ ์ฒดํฌํ•˜์˜€๋‹ค๊ณ  ํ•ด์„œ ํ”ผ๊ณ ์ธ์„ ๋ฐ”๋กœ ์„œ์šธ๊ตฌ์น˜์†Œ๋กœ ๋ณด๋‚ด๋ฒ„๋ฆฌ๋Š” ๊ฒƒ์€ ์ ์ ˆํ•˜์ง€ ์•Š๊ณ , ์ˆ˜ ์‹œ๊ฐ„์— ๊ฑธ์ณ ๊ณ„์†๋˜๋Š” ์ฃผ๊ฑฐ์ง€ ์••์ˆ˜ยท์ˆ˜์ƒ‰ ๊ทธ๋ฆฌ๊ณ  ์ดํ›„์˜ ์ฐจ๋Ÿ‰ ๋˜๋Š” ์˜คํ† ๋ฐ”์ด ์••์ˆ˜ยท์ˆ˜์ƒ‰(ํฌ๋ Œ์‹ ํฌํ•จ)์—๋„ ํ”ผ๊ณ ์ธ์˜ ์ฐธ์—ฌ๊ธฐํšŒ๋ฅผ ๊ณ„์† ๋ณด์žฅํ•˜๊ธฐ ์œ„ํ•ด์„œ๋Š” ํ”ผ๊ณ ์ธ์„ ํ˜„์žฅ์ธ ์ฃผ๊ฑฐ์ง€์— ๊ณ„์† ๋จธ๋ฌด๋ฅด๊ฒŒ ํ•  ํ•„์š”์„ฑ๋„ ์žˆ์—ˆ๋˜ ์ , ๋‹ค๋งŒ ์ฒดํฌ๋œ ํ”ผ๊ณ ์ธ์ด ๊ณ ์„ฑ์„ ์ง€๋ฅด๊ณ  ์ €ํ•ญํ•˜์˜€๊ธฐ ๋•Œ๋ฌธ์— ์••์ˆ˜ยท์ˆ˜์ƒ‰ ์ ˆ์ฐจ์˜ ์›ํ™œํ•œ ์ง„ํ–‰์„ ์œ„ํ•ด์„œ ์–ด๋Š ์ •๋„ ํ”ผ๊ณ ์ธ์„ ์ œ์••ํ•  ํ•„์š”์„ฑ์ด ์ธ์ •๋˜๋Š” ์ , ์ดํ›„ ํ”ผ๊ณ ์ธ์ด ์„ฑ๊ฒฝ์ฑ…์„ ๋‹ฌ๋ผ๋ฉฐ ์ง„์ •์ด ๋œ ์ดํ›„๋กœ๋Š” ์†์„ ์•ž์œผ๋กœ ํ•ด์„œ ์ˆ˜๊ฐ‘์„ ์ฑ„์šด ์ , ๊ทธ๋ฆฌ๊ณ  ํ”ผ๊ณ ์ธ ์ธก์˜ ์—ฐ๋ฝ์œผ๋กœ ์ƒˆ๋ฒฝ์— ๋ณ€ํ˜ธ์‚ฌ๊ฐ€ ์ฃผ๊ฑฐ์ง€ ํ˜„์žฅ์œผ๋กœ ์™€์„œ ํ”ผ๊ณ ์ธ์„ ๋งŒ๋‚˜๊ธฐ๋„ ํ–ˆ๋˜ ์  ๋“ฑ์„ ๋ชจ๋‘ ์ข…ํ•ฉํ•˜๋ฉด, ์œ„ ์‚ฌ์ •๋งŒ์œผ๋กœ๋Š” ๊ตญ์ •์› ์ˆ˜์‚ฌ๊ด€๋“ค์ด ํ”ผ๊ณ ์ธ์„ ๋ถˆ๋ฒ• ๊ตฌ๊ธˆํ–ˆ๋‹ค๊ฑฐ๋‚˜ ์••์ˆ˜ยท์ˆ˜์ƒ‰์ด ๊ฐ•์ œ์ˆ˜์‚ฌ ๋น„๋ก€์˜ ์›์น™ ๋ฐ ๊ณผ์ž‰๊ธˆ์ง€์›์น™์— ๋ฐ˜ํ•˜์—ฌ ์œ„๋ฒ•ํ•œ ์ง‘ํ–‰์ด๋ผ๊ณ  ๋ณผ ์ˆ˜๋Š” ์—†๋‹ค. 5) ํ˜•์‚ฌ์†Œ์†ก๊ทœ์น™ ์ œ101์กฐ์ฃผ9) ์— ์˜ํ•˜๋ฉด, ํ”ผ๊ณ ์ธ๊ณผ ๊ทธ ๋ณ€ํ˜ธ์ธ ๋“ฑ์€ ์ฒดํฌยท๊ตฌ์†์˜์žฅ ๋“ฑ๋ณธ์˜ ๊ต๋ถ€๋ฅผ ์ฒญ๊ตฌํ•  ์ˆ˜ ์žˆ์œผ๋‚˜, ๊ทธ ์‹œ๊ธฐ์— ๋Œ€ํ•ด์„œ๋Š” ๋ช…๋ฐฑํžˆ ๊ทœ์ •ํ•˜๊ณ  ์žˆ์ง€ ์•Š๋‹ค. ๊ทธ๋Ÿฌ๋‚˜ ์œ„ ์กฐ๋ฌธ์˜ ์ œ๋ชฉ(์ฒดํฌยท๊ตฌ์†์ ๋ถ€์‹ฌ์ฒญ๊ตฌ๊ถŒ์ž์˜ ์ฒดํฌยท๊ตฌ์†์˜์žฅ๋“ฑ๋ณธ ๊ต๋ถ€์ฒญ๊ตฌ๋“ฑ)๊ณผ ๋‚ด์šฉ(โ€˜์ฒดํฌ๋œโ€™ ํ”ผ์˜์ž, โ€˜์ฒดํฌ์˜์žฅ ๋“ฑ์„ ๋ณด๊ด€ํ•˜๊ณ  ์žˆ๋Š”โ€™ ๊ฒ€์‚ฌ, ์‚ฌ๋ฒ•๊ฒฝ์ฐฐ๊ด€ ๋“ฑ)์— ๋น„์ถ”์–ด ๋ณด๋ฉด ์ด๋Š” ์ฒดํฌ ์ดํ›„ ์›ํ™œํ•œ ์ฒดํฌยท๊ตฌ์†์ ๋ถ€์‹ฌ์‚ฌ์ฒญ๊ตฌ๋ฅผ ๋ณด์žฅํ•˜๊ธฐ ์œ„ํ•œ ์ทจ์ง€๋ผ ํ•  ๊ฒƒ์ด๋‹ค. ๋”ฐ๋ผ์„œ ์ˆ˜์‚ฌ๊ธฐ๊ด€์€ ํ”ผ๊ณ ์ธ์„ ์ฒดํฌํ•œ ๋’ค ์ฒดํฌ์ ๋ถ€์‹ฌ์‚ฌ๋ฅผ ์ฒญ๊ตฌํ•˜๋Š”๋ฐ ์ง€์žฅ์ด ์—†๋„๋ก ๊ตฌ์ฒด์ ์ธ ์ƒํ™ฉ์— ๋”ฐ๋ผ ๊ฐ€๋Šฅํ•œ ํ•œ ์กฐ์†ํžˆ ๊ทธ ์˜์žฅ์˜ ๋“ฑ๋ณธ์„ ๊ต๋ถ€ํ•ด์ฃผ๋ฉด ๋œ๋‹ค๊ณ  ํ•  ๊ฒƒ์ด๊ณ , ๋ฐ˜๋“œ์‹œ ์ฒดํฌ๋œ ํ˜„์žฅ์—์„œ ๋ฐ”๋กœ ๊ต๋ถ€ํ•ด์ฃผ์–ด์•ผ๋งŒ ํ•œ๋‹ค๊ณ  ํ•  ์ˆ˜๋Š” ์—†๋‹ค. ๋‹น์‹œ๋Š” ์ฒดํฌ ์ดํ›„ ์‹ฌ์•ผ์— ์••์ˆ˜ยท์ˆ˜์ƒ‰์ด ๊ณ„์† ์ค‘์ด์—ˆ์œผ๋ฏ€๋กœ ์ฒดํฌ์˜์žฅ์˜ ๋“ฑ๋ณธ์„ ๊ต๋ถ€ํ•˜๊ธฐ์— ํ˜„์‹ค์ ์ธ ์–ด๋ ค์›€๋„ ์žˆ์—ˆ๋‹ค. ๊ทธ๋ฆฌํ•˜์—ฌ ์ˆ˜์‚ฌ๊ด€๋“ค์€ ์ฒดํฌ์˜์žฅ์„ ์ œ์‹œํ•˜์—ฌ ์—ด๋žŒ์‹œ์ผœ ์ฃผ์—ˆ๊ณ  ๋˜ ํ”ผ๊ณ ์ธ ์ธก์ด ์˜์žฅ์˜ ์ผ๋ถ€ ๋‚ด์šฉ์„ ๋ฉ”๋ชจํ•˜๊ธฐ๋„ ํ•˜์˜€๋‹ค. ๊ทธ๋ฆฌ๊ณ  ๊ตญ์ •์› ์ˆ˜์‚ฌ๊ด€์€ ๋‹ค์Œ ๋‚ ์ธ 2015. 11. 14. ์˜ค์ „ ํ”ผ๊ณ ์ธ์ด ๊ตญ์ •์›์œผ๋กœ ์ด๊ฐ๋œ ํ›„ ํ”ผ๊ณ ์ธ์—๊ฒŒ ์ฒดํฌ์˜์žฅ ๋“ฑ๋ณธ์„ ๊ต๋ถ€ํ•˜์˜€๋‹ค. ์ด๋Ÿฌํ•œ ์ ์„ ๋ชจ๋‘ ์ข…ํ•ฉํ•˜๋ฉด, ์ฒดํฌ์˜์žฅ์˜ ๋“ฑ๋ณธ ๊ต๋ถ€์— ๊ด€ํ•œ ์ˆ˜์‚ฌ๊ธฐ๊ด€์˜ ์กฐ์น˜๊ฐ€ ์œ„๋ฒ•ํ•˜๋‹ค๊ณ  ๋ณผ ์ˆ˜๋Š” ์—†๋‹ค. 6) ๋”ฐ๋ผ์„œ ํ”ผ๊ณ ์ธ ๋ฐ ๋ณ€ํ˜ธ์ธ์˜ ์ด ๋ถ€๋ถ„ ์ฃผ์žฅ์€ ๋ชจ๋‘ ์ด์œ  ์—†๋‹ค. 7. ํ•ด์™ธ์ดฌ์˜์˜ ์œ„๋ฒ•์„ฑ, ๊ทธ๋กœ๋ถ€ํ„ฐ ํŒŒ์ƒ๋œ ์ฆ๊ฑฐ๋“ค์˜ ์ฆ๊ฑฐ๋Šฅ๋ ฅ ๊ด€๋ จ ์ฃผ์žฅ ๊ฐ€. ์ฃผ์žฅ 1) ๊ฒ€์ฆ๋ชฉ๋ก ์ˆœ๋ฒˆ 14๋ฒˆ ~ 28๋ฒˆ์€ ์ค‘๊ตญ, ๋ฒ ํŠธ๋‚จ, ๋ง๋ ˆ์ด์‹œ์•„์—์„œ์˜ ์ ‘์„ ์žฅ๋ฉด์„ ์ดฌ์˜ํ•œ ๋™์˜์ƒ ๋˜๋Š” ์‚ฌ์ง„์ธ๋ฐ, ์ด๋Š” ์ดฌ์˜ ๋Œ€์ƒ์ž์˜ ์˜์‚ฌ์— ๋ฐ˜ํ•˜์—ฌ ์ฐ์€ ์‚ฌ์ง„์œผ๋กœ์„œ ์˜์žฅ์ฃผ์˜์— ์œ„๋ฐฐ๋œ ๊ฒƒ์ด๋‹ค. 2) ๋˜ํ•œ ์™ธ๊ตญ์—์„œ ํ˜•์‚ฌ์‚ฌ๋ฒ•๊ณต์กฐ ์—†์ด ๋ฌด๋‹จ์œผ๋กœ ์ˆ˜์‚ฌ๋ฅผ ํ•œ ๊ฒƒ์€ ๊ทธ ๋‚˜๋ผ์˜ ์˜ํ† ์ฃผ๊ถŒ์„ ์นจํ•ดํ•œ ๊ฒƒ์œผ๋กœ์„œ ์œ„๋ฒ•ํ•˜๋‹ค. ๋”ฐ๋ผ์„œ ์œ„ ์‚ฌ์ง„๋“ค์€ ์œ„๋ฒ•์ˆ˜์ง‘์ฆ๊ฑฐ๋กœ์„œ ์ฆ๊ฑฐ๋Šฅ๋ ฅ์ด ์—†๊ณ , ์œ„๋ฒ•ํ•˜๊ฒŒ ์ˆ˜์ง‘๋œ ์ฆ๊ฑฐ์—์„œ ํŒŒ์ƒ๋œ ์ฆ๊ฑฐ๋“ค(์‚ฌ์ง„ ์† ์ธ๋ฌผ์— ๋Œ€ํ•œ ๋™์ผ์„ฑ ๊ฐ์ •๊ฒฐ๊ณผ ๋“ฑ) ์—ญ์‹œ ์ฆ๊ฑฐ๋Šฅ๋ ฅ์ด ์—†๋‹ค. ๋‚˜. ํŒ๋‹จ 1) ์˜์žฅ์ฃผ์˜ ์œ„๋ฐ˜ ์—ฌ๋ถ€ ๋ˆ„๊ตฌ๋“ ์ง€ ์ž๊ธฐ์˜ ์–ผ๊ตด ๊ธฐํƒ€ ๋ชจ์Šต์„ ํ•จ๋ถ€๋กœ ์ดฌ์˜๋‹นํ•˜์ง€ ์•Š์„ ์ž์œ ๋ฅผ ๊ฐ€์ง€๋‚˜ ์ด๋Ÿฌํ•œ ์ž์œ ๋„ ๊ตญ๊ฐ€๊ถŒ๋ ฅ์˜ ํ–‰์‚ฌ๋กœ๋ถ€ํ„ฐ ๋ฌด์ œํ•œ์œผ๋กœ ๋ณดํ˜ธ๋˜๋Š” ๊ฒƒ์€ ์•„๋‹ˆ๊ณ  ๊ตญ๊ฐ€์˜ ์•ˆ์ „๋ณด์žฅยท์งˆ์„œ์œ ์ง€ยท๊ณต๊ณต๋ณต๋ฆฌ๋ฅผ ์œ„ํ•˜์—ฌ ํ•„์š”ํ•œ ๊ฒฝ์šฐ์—๋Š” ์ƒ๋‹นํ•œ ์ œํ•œ์ด ๋”ฐ๋ฅด๋Š” ๊ฒƒ์ด๊ณ , ์ˆ˜์‚ฌ๊ธฐ๊ด€์ด ๋ฒ”์ฃ„๋ฅผ ์ˆ˜์‚ฌํ•จ์— ์žˆ์–ด ํ˜„์žฌ ๋ฒ”ํ–‰์ด ํ–‰ํ•˜์—ฌ์ง€๊ณ  ์žˆ๊ฑฐ๋‚˜ ํ–‰ํ•˜์—ฌ์ง„ ์งํ›„์ด๊ณ , ์ฆ๊ฑฐ๋ณด์ „์˜ ํ•„์š”์„ฑ ๋ฐ ๊ธด๊ธ‰์„ฑ์ด ์žˆ์œผ๋ฉฐ, ์ผ๋ฐ˜์ ์œผ๋กœ ํ—ˆ์šฉ๋˜๋Š” ์ƒ๋‹นํ•œ ๋ฐฉ๋ฒ•์— ์˜ํ•˜์—ฌ ์ดฌ์˜์„ ํ•œ ๊ฒฝ์šฐ๋ผ๋ฉด ์œ„ ์ดฌ์˜์ด ์˜์žฅ ์—†์ด ์ด๋ฃจ์–ด์กŒ๋‹ค ํ•˜์—ฌ ์ด๋ฅผ ์œ„๋ฒ•ํ•˜๋‹ค๊ณ  ๋‹จ์ •ํ•  ์ˆ˜ ์—†๋‹ค(๋Œ€๋ฒ•์› 1999. 9. 3. ์„ ๊ณ  99๋„2317 ํŒ๊ฒฐ ๋“ฑ ์ฐธ์กฐ). ์‚ดํ”ผ๊ฑด๋Œ€, ์‚ฌ์ง„์ด๋‚˜ ๋น„๋””์˜ค์ดฌ์˜๊ณผ ๊ด€๋ จ๋œ ๊ถŒ๋ฆฌ์ธ ์ดˆ์ƒ๊ถŒ์€ โ‘  ์ดฌ์˜๊ฑฐ์ ˆ๊ถŒ(์–ผ๊ตด ๊ธฐํƒ€ ์‚ฌํšŒํ†ต๋…์ƒ ํŠน์ •์ธ์„ ์•Œ ์ˆ˜ ์žˆ๋Š” ์‹ ์ฒด์  ํŠน์ง•์„ ํ•จ๋ถ€๋กœ ์ดฌ์˜๋˜๋Š” ๊ฒƒ์„ ๊ฑฐ์ ˆํ•  ๊ถŒ๋ฆฌ), โ‘ก ๊ณตํ‘œ๊ฑฐ์ ˆ๊ถŒ(์ดฌ์˜๋œ ์ดˆ์ƒ์ด ํ•จ๋ถ€๋กœ ๊ณตํ‘œ๋˜์ง€ ์•„๋‹ˆํ•  ๊ถŒ๋ฆฌ), โ‘ข ์ดˆ์ƒ์˜๋ฆฌ๊ถŒ(์ดˆ์ƒ์ด ํ•จ๋ถ€๋กœ ์˜๋ฆฌ๋ชฉ์ ์— ์ด์šฉ๋˜์ง€ ์•„๋‹ˆํ•  ๊ถŒ๋ฆฌ) ๋“ฑ์„ ํฌ๊ด„ํ•˜๋Š” ๊ถŒ๋ฆฌ์ธ๋ฐ, ๊ทธ์ค‘ ์ดฌ์˜๊ฑฐ์ ˆ๊ถŒ์€ ๋‹ค๋ฅธ ๊ถŒ๋ฆฌ์— ๋น„ํ•˜์—ฌ ์ƒ๋Œ€์ ์œผ๋กœ ๋ณดํ˜ธ์˜ ์ด์ต์ด ํฐ ๊ฒƒ์ด ์•„๋‹ˆ๊ณ , ์ ์–ด๋„ ๊ณต๊ฐœ๋œ ์žฅ์†Œ์—์„œ ์ดฌ์˜๋œ ๊ฒฝ์šฐ์—๋Š” ๋ฒ”์ฃ„์˜ ์ˆ˜์‚ฌ๋ฅผ ์œ„ํ•˜์—ฌ ๋ฐ˜๋“œ์‹œ ํ”ผ์ดฌ์˜์ž์˜ ๋™์˜๋ฅผ ์–ป๊ฑฐ๋‚˜ ์˜์žฅ์„ ๋ฐœ๋ถ€๋ฐ›์•„์•ผ๋งŒ ํ•˜๋Š” ๊ฐ•์ œ์ˆ˜์‚ฌ๋ผ ๋‹จ์ •ํ•˜๊ธฐ ์–ด๋ ค์šด ์ , ์ด ์‚ฌ๊ฑด ์ดฌ์˜์€ ํ”ผ๊ณ ์ธ ๋“ฑ์ด ๋ฐ˜๊ตญ๊ฐ€๋‹จ์ฒด์˜ ๊ตฌ์„ฑ์›๊ณผ ํšŒํ•ฉ ์ค‘์ธ ๋ชจ์Šต์ด๋‚˜ ํšŒํ•ฉํ•˜๊ธฐ ์ง์ „ยท์งํ›„์˜ ๋ชจ์Šต์„ ์ดฌ์˜ํ•œ ๊ฒƒ์œผ๋กœ์„œ, ๊ตญ๊ฐ€์˜ ์กด๋ฆฝยท์•ˆ์ „์ด๋‚˜ ์ž์œ ๋ฏผ์ฃผ์  ๊ธฐ๋ณธ์งˆ์„œ๋ฅผ ํ•ด์น  ์šฐ๋ ค๊ฐ€ ์žˆ๋Š” ๊ตญ๊ฐ€๋ณด์•ˆ๋ฒ•์œ„๋ฐ˜(ํšŒํ•ฉ)์˜ ์ฆ๊ฑฐ๋ฅผ ๋ณด์ „ํ•˜๊ธฐ ์œ„ํ•œ ํ•„์š”์—์„œ ๊ธด๊ธ‰ํ•˜๊ฒŒ ์ด๋ฃจ์–ด์ง„ ๊ฒƒ์ธ ์ , ๊ทธ ์ดฌ์˜ ์žฅ์†Œ๋„ ๊ธธ๊ฑฐ๋ฆฌ๋‚˜ ์‹๋‹น ๋“ฑ ๊ฐ๊ด€์ ์ธ ์€๋น„(์€๋น„) ๊ธฐ๋Œ€๊ฐ€ ๊ทธ๋‹ค์ง€ ๋†’์ง€ ์•Š์€ ๊ณต๊ฐœ์ ์ธ ์žฅ์†Œ์ด๋ฏ€๋กœ ์ด๋Ÿฌํ•œ ์ดฌ์˜์œผ๋กœ์„œ ํ”ผ๊ณ ์ธ์˜ ์‚ฌ์ƒํ™œ์˜ ๋น„๋ฐ€์ด ํ˜„์ €ํžˆ ์นจํ•ด๋˜์—ˆ๋‹ค๊ณ  ๋ณด๊ธฐ ์–ด๋ ค์šด ์ , ์ผ๋ฐ˜์ ์ธ ๋™์˜์ƒ ์นด๋ฉ”๋ผ๋กœ ์ดฌ์˜ํ•œ ๊ฒƒ์ด์–ด์„œ ์ดฌ์˜ ๋ฐฉ๋ฒ•์˜ ์ƒ๋‹น์„ฑ์ด ์ธ์ •๋˜๋Š” ์  ๋“ฑ์— ๋น„์ถ”์–ด ๋ณด๋ฉด, ์ด ์‚ฌ๊ฑด ์‚ฌ์ง„ ์ดฌ์˜์ด ์˜์žฅ์ฃผ์˜์— ์œ„๋ฐฐ๋˜๋Š” ๊ฒƒ์œผ๋กœ์„œ ์ฆ๊ฑฐ๋Šฅ๋ ฅ์„ ๋ถ€์ •ํ•  ์ •๋„์˜ ์œ„๋ฒ•์ด ์žˆ๋‹ค๊ณ  ํ•  ์ˆ˜๋Š” ์—†๋‹ค. 2) ์˜ํ† ์ฃผ๊ถŒ์˜ ์นจํ•ด ์—ฌ๋ถ€ ์ด ์‚ฌ๊ฑด ์ดฌ์˜์€ ์ƒ๋Œ€๋ฐฉ์ด ์šฐ๋ฆฌ๋‚˜๋ผ ๊ตญ๋ฏผ์ด๊ณ  ์œ„์—์„œ ๋ณธ ๋ฐ”์™€ ๊ฐ™์€ ํ•„์š”์„ฑ ํ•˜์—์„œ ์ผ๋ฐ˜์ ์œผ๋กœ ํ—ˆ์šฉ๋˜๋Š” ์ƒ๋‹นํ•œ ๋ฐฉ๋ฒ•์œผ๋กœ ์ด๋ฃจ์–ด์กŒ๋‹ค. ๊ทธ๋ฆฌ๊ณ  ์„ค์‚ฌ ์ค‘๊ตญยท๋ฒ ํŠธ๋‚จยท๋ง๋ ˆ์ด์‹œ์•„์™€ ์‚ฌ์ด์— ํ˜•์‚ฌ์‚ฌ๋ฒ•๊ณต์กฐ์ ˆ์ฐจ๋ฅผ ๊ฑฐ์น˜์ง€ ์•„๋‹ˆํ•˜๊ณ  ์œ„์™€ ๊ฐ™์ด ์ˆ˜์‚ฌํ–‰์œ„์— ํ•ด๋‹นํ•˜๋Š” ์‚ฌ์ง„ ์ดฌ์˜์„ ํ•จ์œผ๋กœ์จ ์ด๋กœ ์ธํ•œ ์˜ํ† ์ฃผ๊ถŒ ์นจํ•ด์˜ ๋ฌธ์ œ๊ฐ€ ์ƒ๊ฒจ๋‚  ๊ฐ€๋Šฅ์„ฑ์ด ์žˆ๋‹ค๊ณ  ํ•˜๋”๋ผ๋„, ์ด๋Š” ์šฐ๋ฆฌ๋‚˜๋ผ์™€ ์ค‘๊ตญยท๋ฒ ํŠธ๋‚จยท๋ง๋ ˆ์ด์‹œ์•„ ์‚ฌ์ด์˜ ๊ตญ์ œ๋ฒ•์  ๋ฌธ์ œ๋กœ์„œ ํ”ผ๊ณ ์ธ์€ ์ค‘๊ตญยท๋ฒ ํŠธ๋‚จยท๋ง๋ ˆ์ด์‹œ์•„์™€ ์‚ฌ์ด์— ๊ตญ์ œ๋ฒ•์ƒ ๊ด€ํ• ์˜ ์›์ธ์ด ๋  ๋งŒํ•œ ํŠน๋ณ„ํ•œ ์—ฐ๊ด€์„ฑ๋„ ๊ฐ–๊ณ  ์žˆ์ง€ ์•„๋‹ˆํ•˜๋‹ค. ๋”ฐ๋ผ์„œ ์œ„์™€ ๊ฐ™์ด ์ทจ๋“ํ•œ ์ž๋ฃŒ๋ฅผ ์šฐ๋ฆฌ๋‚˜๋ผ ๊ตญ๋ฏผ์ธ ํ”ผ๊ณ ์ธ์˜ ๊ตญ๊ฐ€๋ณด์•ˆ๋ฒ•์œ„๋ฐ˜ ๋ฒ”์ฃ„์— ๋Œ€ํ•œ ๊ตญ๋‚ด์˜ ํ˜•์‚ฌ์†Œ์†ก์ ˆ์ฐจ์—์„œ ์‚ฌ์šฉํ•˜๋Š” ์ด์ƒ ์œ„์™€ ๊ฐ™์€ ์‚ฌ์œ ๋กœ ์ธํ•˜์—ฌ ์œ„๋ฒ•์ˆ˜์ง‘์ฆ๊ฑฐ๋ฐฐ์ œ๋ฒ•์น™์ด ์ ์šฉ๋˜์–ด ์ฆ๊ฑฐ๋Šฅ๋ ฅ์ด ๋ถ€์ •๋œ๋‹ค๊ณ  ๋ณผ ์ˆ˜๋Š” ์—†๋‹ค(๋Œ€๋ฒ•์› 2011. 7. 14. ์„ ๊ณ  2011๋„3809 ํŒ๊ฒฐ, ๋Œ€๋ฒ•์› 2013. 7. 26. ์„ ๊ณ  2013๋„2511 ํŒ๊ฒฐ ๋“ฑ ์ฐธ์กฐ). 3) ๋™์ผ์„ฑ ๊ฐ์ •๊ฒฐ๊ณผ ์œ„์™€ ๊ฐ™์ด ํ•ด์™ธ์ดฌ์˜์ด ์œ„๋ฒ•ํ•œ ์ฆ๊ฑฐ์ˆ˜์ง‘์ด๋ผ๊ณ  ํ•  ์ˆ˜ ์—†๊ณ , ์ ๋ฒ•ํ•œ ๊ฐ์ •์˜๋ขฐ์— ๊ทผ๊ฑฐํ•˜์—ฌ ๊ฐ์ •์ด ์ด๋ฃจ์–ด์ง„ ๊ฒƒ์ด๋ฏ€๋กœ ๊ทธ ํŒŒ์ƒ์ฆ๊ฑฐ์—๋„ ์–ด๋– ํ•œ ์œ„๋ฒ•์ด ์žˆ๋‹ค๊ณ  ํ•  ์ˆ˜ ์—†๋‹ค. 4) ๋”ฐ๋ผ์„œ ํ”ผ๊ณ ์ธ ๋ฐ ๋ณ€ํ˜ธ์ธ์˜ ์ด ๋ถ€๋ถ„ ์ฃผ์žฅ์€ ์ด์œ  ์—†๋‹ค. 8. ์••์ˆ˜๋œ ๋””์ง€ํ„ธ ์ €์žฅ๋งค์ฒด์™€ ์ „์ž์ •๋ณด ํŒŒ์ผ์˜ ๋™์ผ์„ฑ, ๋ฌด๊ฒฐ์„ฑ ๊ด€๋ จ ์ฃผ์žฅ ๊ฐ€. ์ฃผ์žฅ 1) ์ฆ๊ฑฐ๋กœ ์ œ์ถœ๋œ ์••์ˆ˜๋ฌผ์ธ ๋””์ง€ํ„ธ ์ €์žฅ๋งค์ฒด์™€ ์ „์ž์ •๋ณด๋“ค์ด ํ”ผ๊ณ ์ธ์œผ๋กœ๋ถ€ํ„ฐ ์••์ˆ˜ํ•œ ๊ฒƒ๊ณผ ๋™์ผํ•˜๊ณ  ๋ฌด๊ฒฐํ•˜๋‹ค๋Š” ๊ฒƒ์„ ์ธ์ •ํ•  ์ˆ˜ ์—†๋‹ค. ์ด๋Š” ์••์ˆ˜ ๋‹น์‹œ์˜ ์ƒํ™ฉ์„ ์ดฌ์˜ํ•œ ๋™์˜์ƒ์˜ ๊ฒ€์ฆ ๋“ฑ์„ ํ†ตํ•˜์—ฌ ์—„๊ฒฉํ•˜๊ฒŒ ์ž…์ฆํ•ด์•ผ ํ•จ์—๋„ ๋ฒ•์›์€ ์ด๋Ÿฌํ•œ ์ ˆ์ฐจ๋ฅผ ๊ฑฐ์น˜์ง€ ์•Š์•˜๊ณ , ๊ฒ€์ฐฐ์€ ์‹ฌ์ง€์–ด ๊ทธ ๋™์˜์ƒ์˜ ์—ด๋žŒยท๋“ฑ์‚ฌ ์‹ ์ฒญ์กฐ์ฐจ ๊ฑฐ๋ถ€ํ•˜์˜€๋‹ค. 2) ๋ด‰์ธ์„ ํ•ด์ œํ•  ๋•Œ ๋ด‰์ธ์„ ํ•œ ์‚ฌ๋žŒ์ด ๋ฐ˜๋“œ์‹œ ์ฐธ์„ํ•ด์•ผ ํ•˜๋Š”๋ฐ๋„, ๋‹ค๋ฅธ ์‚ฌ๋žŒ์ด ๋ด‰์ธ์„ ํ•ด์ œํ•˜๋Š” ๋“ฑ ์ด ์‚ฌ๊ฑด ์ฆ๊ฑฐ๋“ค์˜ ๋ด‰์ธ์˜ ์—ฐ์†์„ฑ์„ ์ธ์ •ํ•  ์ˆ˜ ์—†๋‹ค. ํŠนํžˆ โ€˜toyou 11-12.docxโ€™ ํŒŒ์ผ์ด ๋“ค์–ด์žˆ๋Š” ๋””์ง€ํ„ธ์ €์žฅ๋งค์ฒด์ธ Micro SD์นด๋“œ๋ฅผ ๋ด‰์ธ์ž ๊ณต์†Œ์™ธ 12๊ฐ€ ์•„๋‹Œ ๊ณต์†Œ์™ธ 11์ด ๋ณต๊ตฌ๋ฅผ ์œ„ํ•˜์—ฌ ํ•ด์ œํ•˜์˜€๋‹ค๊ฐ€ ๋‹ค์‹œ ๋ด‰์ธํ•˜์˜€์œผ๋ฏ€๋กœ, ๋ด‰์ธ์˜ ์—ฐ์†์„ฑ์„ ์ธ์ •ํ•  ์ˆ˜ ์—†๋‹ค. ๋‚˜. ๊ด€๋ จ๋ฒ•๋ฆฌ ์••์ˆ˜๋ฌผ์ธ ์ปดํ“จํ„ฐ์šฉ ๋””์Šคํฌ ๊ทธ ๋ฐ–์— ์ด์™€ ๋น„์Šทํ•œ ์ •๋ณด์ €์žฅ๋งค์ฒด(์ดํ•˜ โ€˜์ •๋ณด์ €์žฅ๋งค์ฒดโ€™๋ผ๊ณ ๋งŒ ํ•œ๋‹ค)์— ์ž…๋ ฅํ•˜์—ฌ ๊ธฐ์–ต๋œ ๋ฌธ์ž์ •๋ณด ๋˜๋Š” ๊ทธ ์ถœ๋ ฅ๋ฌผ(์ดํ•˜ โ€˜์ถœ๋ ฅ ๋ฌธ๊ฑดโ€™์ด๋ผ ํ•œ๋‹ค)์„ ์ฆ๊ฑฐ๋กœ ์‚ฌ์šฉํ•˜๊ธฐ ์œ„ํ•ด์„œ๋Š” ์ •๋ณด์ €์žฅ๋งค์ฒด ์›๋ณธ์— ์ €์žฅ๋œ ๋‚ด์šฉ๊ณผ ์ถœ๋ ฅ ๋ฌธ๊ฑด์˜ ๋™์ผ์„ฑ์ด ์ธ์ •๋˜์–ด์•ผ ํ•˜๊ณ , ์ด๋ฅผ ์œ„ํ•ด์„œ๋Š” ์ •๋ณด์ €์žฅ๋งค์ฒด ์›๋ณธ์ด ์••์ˆ˜์‹œ๋ถ€ํ„ฐ ๋ฌธ๊ฑด ์ถœ๋ ฅ์‹œ๊นŒ์ง€ ๋ณ€๊ฒฝ๋˜์ง€ ์•Š์•˜๋‹ค๋Š” ์‚ฌ์ •, ์ฆ‰ ๋ฌด๊ฒฐ์„ฑ์ด ๋‹ด๋ณด๋˜์–ด์•ผ ํ•œ๋‹ค. ํŠนํžˆ ์ •๋ณด์ €์žฅ๋งค์ฒด ์›๋ณธ์„ ๋Œ€์‹ ํ•˜์—ฌ ์ €์žฅ๋งค์ฒด์— ์ €์žฅ๋œ ์ž๋ฃŒ๋ฅผ โ€˜ํ•˜๋“œ์นดํ”ผโ€™ ๋˜๋Š” โ€˜์ด๋ฏธ์ง•โ€™ํ•œ ๋งค์ฒด๋กœ๋ถ€ํ„ฐ ์ถœ๋ ฅํ•œ ๋ฌธ๊ฑด์˜ ๊ฒฝ์šฐ์—๋Š” ์ •๋ณด์ €์žฅ๋งค์ฒด ์›๋ณธ๊ณผ โ€˜ํ•˜๋“œ์นดํ”ผโ€™ ๋˜๋Š” โ€˜์ด๋ฏธ์ง•โ€™ํ•œ ๋งค์ฒด ์‚ฌ์ด์— ์ž๋ฃŒ์˜ ๋™์ผ์„ฑ๋„ ์ธ์ •๋˜์–ด์•ผ ํ•  ๋ฟ๋งŒ ์•„๋‹ˆ๋ผ, ์ด๋ฅผ ํ™•์ธํ•˜๋Š” ๊ณผ์ •์—์„œ ์ด์šฉํ•œ ์ปดํ“จํ„ฐ์˜ ๊ธฐ๊ณ„์  ์ •ํ™•์„ฑ, ํ”„๋กœ๊ทธ๋žจ์˜ ์‹ ๋ขฐ์„ฑ, ์ž…๋ ฅยท์ฒ˜๋ฆฌยท์ถœ๋ ฅ์˜ ๊ฐ ๋‹จ๊ณ„์—์„œ ์กฐ์ž‘์ž์˜ ์ „๋ฌธ์ ์ธ ๊ธฐ์ˆ ๋Šฅ๋ ฅ๊ณผ ์ •ํ™•์„ฑ์ด ๋‹ด๋ณด๋˜์–ด์•ผ ํ•œ๋‹ค(๋Œ€๋ฒ•์› 2007. 12. 13. ์„ ๊ณ  2007๋„7257 ํŒ๊ฒฐ ๋“ฑ ์ฐธ์กฐ). ์ด ๊ฒฝ์šฐ ์ถœ๋ ฅ ๋ฌธ๊ฑด๊ณผ ์ •๋ณด์ €์žฅ๋งค์ฒด์— ์ €์žฅ๋œ ์ž๋ฃŒ๊ฐ€ ๋™์ผํ•˜๊ณ  ์ •๋ณด์ €์žฅ๋งค์ฒด ์›๋ณธ์ด ๋ฌธ๊ฑด ์ถœ๋ ฅ์‹œ๊นŒ์ง€ ๋ณ€๊ฒฝ๋˜์ง€ ์•Š์•˜๋‹ค๋Š” ์ ์€, ํ”ผ์••์ˆ˜ยท์ˆ˜์ƒ‰ ๋‹น์‚ฌ์ž๊ฐ€ ์ •๋ณด์ €์žฅ๋งค์ฒด ์›๋ณธ๊ณผ โ€˜ํ•˜๋“œ์นดํ”ผโ€™ ๋˜๋Š” โ€˜์ด๋ฏธ์ง•โ€™ํ•œ ๋งค์ฒด์˜ ํ•ด์‹œ(Hash) ๊ฐ’์ด ๋™์ผํ•˜๋‹ค๋Š” ์ทจ์ง€๋กœ ์„œ๋ช…ํ•œ ํ™•์ธ์„œ๋ฉด์„ ๊ต๋ถ€๋ฐ›์•„ ๋ฒ•์›์— ์ œ์ถœํ•˜๋Š” ๋ฐฉ๋ฒ•์— ์˜ํ•˜์—ฌ ์ฆ๋ช…ํ•˜๋Š” ๊ฒƒ์ด ์›์น™์ด๋‚˜, ๊ทธ์™€ ๊ฐ™์€ ๋ฐฉ๋ฒ•์— ์˜ํ•œ ์ฆ๋ช…์ด ๋ถˆ๊ฐ€๋Šฅํ•˜๊ฑฐ๋‚˜ ํ˜„์ €ํžˆ ๊ณค๋ž€ํ•œ ๊ฒฝ์šฐ์—๋Š”, ์ •๋ณด์ €์žฅ๋งค์ฒด ์›๋ณธ์— ๋Œ€ํ•œ ์••์ˆ˜, ๋ด‰์ธ, ๋ด‰์ธํ•ด์ œ, โ€˜ํ•˜๋“œ์นดํ”ผโ€™ ๋˜๋Š” โ€˜์ด๋ฏธ์ง•โ€™ ๋“ฑ ์ผ๋ จ์˜ ์ ˆ์ฐจ์— ์ฐธ์—ฌํ•œ ์ˆ˜์‚ฌ๊ด€์ด๋‚˜ ์ „๋ฌธ๊ฐ€ ๋“ฑ์˜ ์ฆ์–ธ์— ์˜ํ•ด ์ •๋ณด์ €์žฅ๋งค์ฒด ์›๋ณธ๊ณผ โ€˜ํ•˜๋“œ์นดํ”ผโ€™ ๋˜๋Š” โ€˜์ด๋ฏธ์ง•โ€™ํ•œ ๋งค์ฒด ์‚ฌ์ด์˜ ํ•ด์‹œ๊ฐ’์ด ๋™์ผํ•˜๋‹ค๊ฑฐ๋‚˜ ์ •๋ณด์ €์žฅ๋งค์ฒด ์›๋ณธ์ด ์ตœ์ดˆ ์••์ˆ˜์‹œ๋ถ€ํ„ฐ ๋ฐ€๋ด‰๋˜์–ด ์ฆ๊ฑฐ ์ œ์ถœ์‹œ๊นŒ์ง€ ์ „ํ˜€ ๋ณ€๊ฒฝ๋˜์ง€ ์•Š์•˜๋‹ค๋Š” ๋“ฑ์˜ ์‚ฌ์ •์„ ์ฆ๋ช…ํ•˜๋Š” ๋ฐฉ๋ฒ• ๋˜๋Š” ๋ฒ•์›์ด ๊ทธ ์›๋ณธ์— ์ €์žฅ๋œ ์ž๋ฃŒ์™€ ์ฆ๊ฑฐ๋กœ ์ œ์ถœ๋œ ์ถœ๋ ฅ ๋ฌธ๊ฑด์„ ๋Œ€์กฐํ•˜๋Š” ๋ฐฉ๋ฒ• ๋“ฑ์œผ๋กœ๋„ ๊ทธ์™€ ๊ฐ™์€ ๋ฌด๊ฒฐ์„ฑยท๋™์ผ์„ฑ์„ ์ธ์ •ํ•  ์ˆ˜ ์žˆ๋‹ค๊ณ  ํ•  ๊ฒƒ์ด๋ฉฐ, ๋ฐ˜๋“œ์‹œ ์••์ˆ˜ยท์ˆ˜์ƒ‰ ๊ณผ์ •์„ ์ดฌ์˜ํ•œ ์˜์ƒ๋…นํ™”๋ฌผ ์žฌ์ƒ ๋“ฑ์˜ ๋ฐฉ๋ฒ•์œผ๋กœ๋งŒ ์ฆ๋ช…ํ•˜์—ฌ์•ผ ํ•œ๋‹ค๊ณ  ๋ณผ ๊ฒƒ์€ ์•„๋‹ˆ๋‹ค(๋Œ€๋ฒ•์› 2013. 7. 26. ์„ ๊ณ  2013๋„2511 ์ฐธ์กฐ). ๋‹ค. ์ธ์ •์‚ฌ์‹ค ใ€ˆ์••์ˆ˜๊ณผ์ •ใ€‰ 1) Micro SD ์นด๋“œ(16G, ์‚ผ์„ฑ EVO, MBMPAGVDDDCE-P, ์ฆ ์ œ122ํ˜ธ) : ์ด ์‚ฌ๊ฑด ์••์ˆ˜ยท์ˆ˜์ƒ‰์ผ์ธ 2015. 11. 13. 00:48๊ฒฝ ํ”ผ๊ณ ์ธ์ด ์†Œ์ง€ํ•œ ์ง€๊ฐ‘์—์„œ ๋ฐœ๊ฒฌํ•˜์˜€๋‹ค. 11:25๊ฒฝ ์›๋ณธ์„ ์••์ˆ˜ํ•˜๊ณ  ์ด๋ฏธ์ง•ํ•˜์—ฌ ํ•ด์‹œ๊ฐ’์„ ์‚ฐ์ถœํ•˜์˜€๋‹ค. 2) Micro SD ์นด๋“œ(16G, S/N MBMSAGVDDDCW-P, ์ฆ ์ œ123ํ˜ธ) : 00:48๊ฒฝ ํ”ผ๊ณ ์ธ ์‹ ์ฒด(์…”์ธ  ์ฃผ๋จธ๋‹ˆ)๋ฅผ ์ˆ˜์ƒ‰ํ•˜์—ฌ ๋ฐœ๊ฒฌํ•˜์˜€๋‹ค. 12:37๊ฒฝ ์›๋ณธ์„ ์••์ˆ˜, ์ด๋ฏธ์ง•ํ•˜๊ณ  ํ•ด์‹œ๊ฐ’์„ ์‚ฐ์ถœํ•˜์˜€๋‹ค. 3) USB(HP 4G, v165w, S/N : AA0000 0000 003498, ์ฆ ์ œ124ํ˜ธ) : 07:53 ~ 07:59๊ฒฝ ํ”ผ๊ณ ์ธ ์˜คํ† ๋ฐ”์ด ์•ˆ์žฅ ๋ฐ‘ ์ˆ˜๋‚ฉ๊ณต๊ฐ„์—์„œ ์••์ˆ˜ํ•˜์˜€๋‹ค. ์ด๋ฅผ ๋ด‰์ธํ•˜์—ฌ ํ”ผ๊ณ ์ธ ์ฃผ๊ฑฐ์ง€๋กœ ์˜ฎ๊ฒผ๊ณ , ํ”ผ๊ณ ์ธ๊ณผ ์ฒ˜ ๊ณต์†Œ์™ธ 8์—๊ฒŒ ํ™•์ธ์„ ์‹œ์ผœ์ฃผ์—ˆ๋‹ค. 09:20๊ฒฝ ์›๋ณธ์„ ์••์ˆ˜ํ•˜๊ณ  ์ด๋ฏธ์ง•ํ•˜์—ฌ ํ•ด์‹œ๊ฐ’์„ ์‚ฐ์ถœํ•˜์˜€๋‹ค. 4) SD ์นด๋“œ(์†Œ๋‹ˆ, CHDC036GA, ์ฆ ์ œ125ํ˜ธ) ๋ฐ USB(HP, 4G, S/N : AA0000 0000 000102, ์ฆ ์ œ126ํ˜ธ) : 06:15 ~ 06:30๊ฒฝ ํ”ผ๊ณ ์ธ์˜ ์ฐจ๋Ÿ‰ ๋‚ด ์žฌ๋–จ์ด ์•ˆ ๋™์ „๋”๋ฏธ์—์„œ ๋ฐœ๊ฒฌํ•˜์˜€๋‹ค. 07:59๊ฒฝ ๊ฐ ์›๋ณธ์„ ์••์ˆ˜, ์ด๋ฏธ์ง•ํ•˜๊ณ  ํ•ด์‹œ๊ฐ’์„ ์‚ฐ์ถœํ•˜์˜€๋‹ค. 5) LG ๋…ธํŠธ๋ถ SSD(128G, LG13Z94 ์ด๋ฏธ์ง• ํŒŒ์ผ, ์ฆ ์ œ127ํ˜ธ) : 06:15 ~ 06:30๊ฒฝ ํ”ผ๊ณ ์ธ ์ฐจ๋Ÿ‰์—์„œ ๋ฐœ๊ฒฌํ•˜์˜€๋‹ค. 12:37๊ฒฝ ์›๋ณธ ์••์ˆ˜ํ•˜๊ณ  ์ด๋ฏธ์ง•์„ ํ•˜์—ฌ ํ•ด์‹œ๊ฐ’์„ ์‚ฐ์ถœํ•˜์˜€๋‹ค. 6) USB(8G, Sandisk Blade Cruzer, ์ฆ ์ œ128ํ˜ธ) : 06:15 ~ 06:30๊ฒฝ ํ”ผ๊ณ ์ธ ์ฐจ๋Ÿ‰ ๋‚ด ๋ณ€์†๊ธฐ ์•ž ์ˆ˜๋‚ฉ๊ณต๊ฐ„์—์„œ ๋ฐœ๊ฒฌํ•˜์˜€๋‹ค. 08:45๊ฒฝ ์›๋ณธ์„ ์••์ˆ˜ํ•˜๊ณ  ์ด๋ฏธ์ง•ํ•˜์—ฌ ํ•ด์‹œ๊ฐ’์„ ์‚ฐ์ถœํ•˜์˜€๋‹ค. 7) USB(32G, DUCO, ์ฆ ์ œ129ํ˜ธ) : 01:11 ~ 03:59๊ฒฝ ํ”ผ๊ณ ์ธ ์ฃผ๊ฑฐ์ง€ ๊ฑฐ์‹ค์—์„œ ๋ฐœ๊ฒฌํ•˜์˜€๋‹ค. 13:16๊ฒฝ ์›๋ณธ์„ ์••์ˆ˜ํ•˜๊ณ  ์ด๋ฏธ์ง•ํ•˜์—ฌ ํ•ด์‹œ๊ฐ’์„ ์‚ฐ์ถœํ•˜์˜€๋‹ค. 8) ๋„์‹œ๋ฐ” ๋…ธํŠธ๋ถ(120G, S/N 180XT018T ์ด๋ฏธ์ง•ํŒŒ์ผ, ์ฆ ์ œ130ํ˜ธ) : 01:11 ~ 03:59๊ฒฝ ํ”ผ๊ณ ์ธ ์ฃผ๊ฑฐ์ง€ ๊ฑฐ์‹ค์—์„œ ๋ฐœ๊ฒฌํ•˜์˜€๋‹ค. 06:30๊ฒฝ ์ด๋ฏธ์ง•ํ•˜์—ฌ 2๊ฐœ์˜ ์‚ฌ๋ณธ์„ ๋งŒ๋“ค์–ด ์••์ˆ˜ํ•˜๊ณ  ์›๋ณธ์€ ๋ฐ˜ํ™˜ํ•˜์˜€๋‹ค. 9) ํ•˜๋“œ๋””์Šคํฌ(40G, โ—‹โ—‹โ—‹, ์ฆ ์ œ131ํ˜ธ) : 01:11 ~ 03:59๊ฒฝ ํ”ผ๊ณ ์ธ ์ฃผ๊ฑฐ์ง€ ๊ฑฐ์‹ค์—์„œ ๋ฐœ๊ฒฌํ•˜์˜€๋‹ค. 13:35๊ฒฝ ์ด๋ฏธ์ง• ๊ณผ์ •์—์„œ ์˜ค๋ฅ˜๊ฐ€ ๋ฐœ์ƒํ•˜์—ฌ ์™ธ๋ถ€์—…์ฒด๋ฅผ ํ†ตํ•ด ๋ณต๊ตฌ๋ฅผ ํ•˜๊ธฐ ์œ„ํ•ด ์›๋ณธ์„ ๋ฐ˜์ถœํ•˜์˜€๋‹ค. 10) ์‚ผ์„ฑ ์™ธ์žฅํ•˜๋“œ S2(500G, ์ฆ ์ œ132ํ˜ธ) : 07:53 ~ 07:59๊ฒฝ ํ”ผ๊ณ ์ธ ์˜คํ† ๋ฐ”์ด ์•ˆ์žฅ ๋ฐ‘ ๋ณด๊ด€ํ•จ์—์„œ ๋ฐœ๊ฒฌํ•˜์˜€๋‹ค. 12:30๊ฒฝ ์›๋ณธ์„ ์••์ˆ˜ํ•˜๊ณ  ์ด๋ฏธ์ง•ํ•˜์—ฌ ํ•ด์‹œ๊ฐ’์„ ์‚ฐ์ถœํ•˜์˜€๋‹ค. 11) CD(1, 2, ์ฆ ์ œ133ํ˜ธ) : ํ”ผ๊ณ ์ธ ์ฃผ๊ฑฐ์ง€ ์ฑ…์žฅ ์˜† ์†Œ์ฑ…์žฅ์—์„œ ๋ฐœ๊ฒฌํ•˜์˜€๋‹ค. 08:30๊ฒฝ ์›๋ณธ ์••์ˆ˜ํ•˜๊ณ  ์ด๋ฏธ์ง•ํ•˜์—ฌ ํ•ด์‹œ๊ฐ’์„ ์‚ฐ์ถœํ•˜์˜€๋‹ค. 12) ์Šค๋งˆํŠธํฐ(LG-F300S, S/N 310KPDT0024778, ์ฆ ์ œ134ํ˜ธ) : ํ”ผ๊ณ ์ธ์˜ ์‹ ์ฒด์—์„œ ๋ฐœ๊ฒฌํ•˜์—ฌ ์••์ˆ˜ํ•˜์˜€๋‹ค. 13) ์•„์ดํŒจ๋“œ ์—์–ด2(A-1566, S/N DLXP4CCEGSVY ์ด๋ฏธ์ง• ํŒŒ์ผ, ์ฆ ์ œ135ํ˜ธ) : ํ”ผ๊ณ ์ธ์˜ ์ฐจ๋Ÿ‰ ์กฐ์ˆ˜์„ ๊ฐ€๋ฐฉ ์•ˆ์—์„œ ๋ฐœ๊ฒฌํ•˜์˜€๋‹ค. 11:35๊ฒฝ 2๊ฐœ๋ฅผ ์ด๋ฏธ์ง•ํ•˜์—ฌ ์••์ˆ˜ํ•˜๊ณ  ์›๋ณธ์€ ํ”ผ๊ณ ์ธ์—๊ฒŒ ๋ฐ˜ํ™˜ํ•˜์˜€๋‹ค. 14) ๊ฐค๋Ÿญ์‹œ ํƒญ(SHW-M500W, S/N R34D600A76V, ์ฆ ์ œ136ํ˜ธ) : ํ”ผ๊ณ ์ธ์˜ ์ฐจ๋Ÿ‰ ์กฐ์ˆ˜์„ ๊ฐ€๋ฐฉ ์•ˆ์—์„œ ๋ฐœ๊ฒฌํ•˜์—ฌ ์••์ˆ˜ํ•˜์˜€๋‹ค. 15) ์Šค๋งˆํŠธํฐ(SHV-E330S, S/N R33D60V7PBA ์ด๋ฏธ์ง• ํŒŒ์ผ, ์ฆ ์ œ137ํ˜ธ) : ํ”ผ๊ณ ์ธ์˜ ์ฃผ๊ฑฐ์ง€ ๊ฑฐ์‹ค ๋ฒฝ๋ฉด ์ฑ…์žฅ ์œ„ ์ฒœ์ •์—์„œ ๋ฐœ๊ฒฌํ•˜์—ฌ ์••์ˆ˜ํ•˜์˜€๋‹ค. 16) ์•„์ดํฐ 6 ํ”Œ๋Ÿฌ์Šค(A-1524, IMEI : 35437063388512 ์ด๋ฏธ์ง• ํŒŒ์ผ, ์ฆ ์ œ138ํ˜ธ) : ํ”ผ๊ณ ์ธ์˜ ์‹ ์ฒด๋ฅผ ์ˆ˜์ƒ‰ํ•˜๋˜ ์ค‘ ์ƒ์˜ ์•ˆ์ฃผ๋จธ๋‹ˆ์—์„œ ๋ฐœ๊ฒฌํ•˜์˜€๋‹ค. 11:20๊ฒฝ 2๊ฐœ๋ฅผ ์ด๋ฏธ์ง•ํ•˜์—ฌ ์••์ˆ˜ํ•˜๊ณ  ์›๋ณธ์€ ํ”ผ๊ณ ์ธ์—๊ฒŒ ๋ฐ˜ํ™˜ํ•˜์˜€๋‹ค. 17) ์œ„ ๋””์ง€ํ„ธ ๋งค์ฒด๋“ค์— ๋Œ€ํ•˜์—ฌ ์••์ˆ˜์ ˆ์ฐจ๊ฐ€ ๋๋‚œ ํ›„ ๊ตญ์ •์› ์ˆ˜์‚ฌ๊ด€์€ 14:20๊ฒฝ๋ถ€ํ„ฐ ํ”ผ๊ณ ์ธ์—๊ฒŒ ๋ด‰์ธ์„ ์š”์ฒญํ•˜์˜€์œผ๋‚˜, ํ”ผ๊ณ ์ธ์ด ์ด๋ฅผ ๊ฑฐ์ ˆํ•˜์—ฌ ๊ณต์†Œ์™ธ 10(๊ต์ˆ˜, ํฌ๋ Œ์‹ํ•™ํšŒ)์„ ๋ด‰์ธ์ ˆ์ฐจ์— ์ฐธ์—ฌ์‹œ์ผœ ๋ด‰์ธํ•œ ํ›„ ๊ณต์†Œ์™ธ 10์ด ์„œ๋ช…ํ•˜์˜€๋‹ค. ๋ด‰์ธ ์ ˆ์ฐจ๋Š” 16:55๊ฒฝ ์™„๋ฃŒ๋˜์—ˆ๋‹ค. 18) ๊ตญ์ •์› ์ˆ˜์‚ฌ๊ด€์€ ์••์ˆ˜๋ชฉ๋ก๊ณผ ์••์ˆ˜์ฆ๋ช…์„œ๋ฅผ ํ”ผ๊ณ ์ธ์—๊ฒŒ ๊ต๋ถ€ํ•˜์˜€์œผ๋‚˜ ํ”ผ๊ณ ์ธ์€ ์••์ˆ˜๋ชฉ๋ก ์ˆ˜๋ นํ™•์ธ ์„œ๋ช… ๋ฐ ๋‚ ์ธ์„ ๊ฑฐ๋ถ€ํ•˜์˜€๋‹ค. ใ€ˆ์ดํ›„์˜ ์••์ˆ˜ ๋˜๋Š” ์„ ๋ณ„์••์ˆ˜ ๊ณผ์ •ใ€‰ ์ดํ›„ ๊ตญ์ •์› ์ˆ˜์‚ฌ๊ด€์€ 2015. 11. 14. ~ 2015. 11. 29.๊ฒฝ ๊ตญ์ •์› ์กฐ์‚ฌ์‹ค์—์„œ ์œ„์™€ ๊ฐ™์ด ์••์ˆ˜ํ•œ ๋””์ง€ํ„ธ ์ €์žฅ๋งค์ฒด๋“ค(๋˜๋Š” ์ด๋ฅผ ์ด๋ฏธ์ง•ํ•œ ์‚ฌ๋ณธ)์— ๊ด€ํ•˜์—ฌ ์„ ๋ณ„์••์ˆ˜๋ฅผ ์ง„ํ–‰ํ–ˆ๋‹ค. ๋‹น์‹œ ์„ ๋ณ„์••์ˆ˜์— ๊ด€ํ•˜์—ฌ ํ”ผ๊ณ ์ธ์—๊ฒŒ ์ฐธ์—ฌ์˜์‚ฌ๋ฅผ ๋ฌผ์—ˆ์œผ๋‚˜ ํ”ผ๊ณ ์ธ์ด ์ฐธ์—ฌ๋ฅผ ๊ฑฐ๋ถ€ํ•˜์—ฌ, ๊ณต์†Œ์™ธ 10์„ ์ฐธ์—ฌ์‹œํ‚ค๊ณ  ์„ ๋ณ„์••์ˆ˜๋ฅผ ์ง„ํ–‰ํ•˜์˜€๋‹ค. ์„ ๋ณ„์••์ˆ˜๊ฐ€ ์ข…๋ฃŒ๋œ ๋’ค, ์„ ๋ณ„๋œ ํŒŒ์ผ์˜ ํ•ด์‹œ๊ฐ’์„ ์‚ฐ์ถœํ•˜์—ฌ ๊ณต์†Œ์™ธ 10์—๊ฒŒ ์ด๋ฅผ ํ™•์ธ์‹œ์ผฐ๋‹ค. ์ž์„ธํ•œ ๋‚ด์—ญ์€ ์•„๋ž˜์™€ ๊ฐ™๋‹ค. 1) ์ฆ ์ œ122ํ˜ธ ์ด๋ฏธ์ง• ์‚ฌ๋ณธ : 2015. 11. 14. 15:51๊ฒฝ ~ 2015. 11. 18. 16:26๊ฒฝ ํŒŒ์ผ ์„ ๋ณ„์ž‘์—…์„ ํ•œ ๋’ค, 2015. 11. 18. 17:12 ~ 17:33๊ฒฝ ์„ ๋ณ„์••์ˆ˜ํ•˜์˜€๋‹ค. 2) ์ฆ ์ œ123ํ˜ธ ์ด๋ฏธ์ง• ์‚ฌ๋ณธ : 2015. 11. 17. 15:22๊ฒฝ ์ „์ฒด๋ฅผ ์••์ˆ˜ํ•˜์˜€๋‹ค. 3) ์ฆ ์ œ124ํ˜ธ ์ด๋ฏธ์ง• ์‚ฌ๋ณธ : 2015. 11. 17. 15:30๊ฒฝ ์ „์ฒด๋ฅผ ์••์ˆ˜ํ•˜์˜€๋‹ค. 4) ์ฆ ์ œ125ํ˜ธ ์ด๋ฏธ์ง• ์‚ฌ๋ณธ : 2015. 11. 17. 15:40๊ฒฝ ์ „์ฒด๋ฅผ ์••์ˆ˜ํ•˜์˜€๋‹ค. 5) ์ฆ ์ œ126ํ˜ธ ์ด๋ฏธ์ง• ์‚ฌ๋ณธ : 2015. 11. 17. 15:43๊ฒฝ ์ „์ฒด๋ฅผ ์••์ˆ˜ํ•˜์˜€๋‹ค, 6) ์ฆ ์ œ127ํ˜ธ ์ด๋ฏธ์ง• ์‚ฌ๋ณธ : 2015. 11. 14. 15:51๊ฒฝ ~ 2015. 11. 29. 13:30๊ฒฝ ํŒŒ์ผ ์„ ๋ณ„์ž‘์—…์„ ํ•œ ๋’ค, 2015. 11. 29. 14:00๊ฒฝ ~ 14:30๊ฒฝ ์„ ๋ณ„์••์ˆ˜ํ•˜์˜€๋‹ค. 7) ์ฆ ์ œ128ํ˜ธ ์ด๋ฏธ์ง• ์‚ฌ๋ณธ : 2015. 11. 14. 15:51๊ฒฝ ~ 2015. 11. 18. 16:43๊ฒฝ ํŒŒ์ผ ์„ ๋ณ„์ž‘์—…์„ ํ•œ ๋’ค, 2015. 11. 18. 17:37 ~ 17:57๊ฒฝ ์„ ๋ณ„์••์ˆ˜ํ•˜์˜€๋‹ค. 8) ์ฆ ์ œ129ํ˜ธ ์ด๋ฏธ์ง• ์‚ฌ๋ณธ : 2015. 11. 14. 15:51๊ฒฝ ~ 2015. 11. 18. 17:03๊ฒฝ ํŒŒ์ผ ์„ ๋ณ„์ž‘์—…์„ ํ•œ ๋’ค, 2015. 11. 18. 17:58๊ฒฝ ~ 18:59๊ฒฝ ์„ ๋ณ„์••์ˆ˜ํ•˜์˜€๋‹ค. 9) ์ฆ ์ œ130ํ˜ธ ์ด๋ฏธ์ง• ์‚ฌ๋ณธ : 2015. 11. 14. 15:51๊ฒฝ ~ 2015. 11. 29. 13:30๊ฒฝ ํŒŒ์ผ ์„ ๋ณ„์ž‘์—…์„ ํ•œ ๋’ค, 2015. 11. 29. 15:05๊ฒฝ ~ 15:40๊ฒฝ ์„ ๋ณ„์••์ˆ˜ํ•˜์˜€๋‹ค. 10) ์ฆ ์ œ131ํ˜ธ : 2015. 11. 13. ์˜ค๋ฅ˜๊ฐ€ ๋ฐœ์ƒํ•˜์—ฌ ๋‚ด์šฉ์„ ํ™•์ธํ•˜์ง€ ๋ชปํ•˜์˜€๋‹ค. 2015. 11. 24. ๊ณต์†Œ์™ธ 13 ์ฃผ์‹ํšŒ์‚ฌ๋ฅผ ํ†ตํ•ด ๋ณต๊ตฌ๋ฅผ ์™„๋ฃŒํ•˜์˜€๋‹ค. 2015. 11. 25. 14:18๊ฒฝ ~ 14:29๊ฒฝ ํฌ๋ Œ์‹ ์ „๋ฌธ๊ฐ€ ๊ณต์†Œ์™ธ 12๋ฅผ ์ž…ํšŒ์‹œํ‚ค๊ณ  ๋ด‰์ธ์„ ํ•ด์ œํ•˜๊ณ  ํ•ด์‹œ๊ฐ’์„ ํ™•์ธํ•˜์˜€๋‹ค. 2015. 11. 25. 14:30๊ฒฝ ~ 2015. 11. 29. 13:30๊ฒฝ ํŒŒ์ผ ์„ ๋ณ„์ž‘์—…์„ ํ•œ ๋’ค 2015. 11. 29. 14:40๊ฒฝ ~ 15:03๊ฒฝ ์„ ๋ณ„์••์ˆ˜ํ•˜์˜€๋‹ค. 11) ์ฆ ์ œ132ํ˜ธ ์ด๋ฏธ์ง• ์‚ฌ๋ณธ : 2015. 11. 14. 15:51๊ฒฝ ~ 2015. 11. 18. 17:05๊ฒฝ ํŒŒ์ผ ์„ ๋ณ„์ž‘์—…์„ ํ•œ ๋’ค 2015. 11. 18. 19:00๊ฒฝ ~ 2015. 11. 19. 08:34๊ฒฝ ์„ ๋ณ„์••์ˆ˜ํ•˜์˜€๋‹ค. 12) ์ฆ ์ œ133ํ˜ธ ์ด๋ฏธ์ง• ์‚ฌ๋ณธ : 2015. 11. 20. 14:33๊ฒฝ ์ „์ฒด๋ฅผ ์••์ˆ˜ํ•˜์˜€๋‹ค. 13) ์ฆ ์ œ134ํ˜ธ ์ด๋ฏธ์ง• ์‚ฌ๋ณธ : 2015. 11. 17. 19:00๊ฒฝ ์„ ๋ณ„์••์ˆ˜ํ•˜์˜€๋‹ค. 14) ์ฆ ์ œ135ํ˜ธ ์ด๋ฏธ์ง• ์‚ฌ๋ณธ : 2015. 11. 14. 15:51๊ฒฝ ์„ ๋ณ„์••์ˆ˜ํ•˜์˜€๋‹ค. 15) ์ฆ ์ œ136ํ˜ธ : 2015. 11. 13. ๋ณต์ œ๊ฐ€ ๋ถˆ๊ฐ€๋Šฅํ•˜์—ฌ 2015. 11. 16. ๊ณต์†Œ์™ธ 65 ์ฃผ์‹ํšŒ์‚ฌ๋ฅผ ํ†ตํ•ด ๋ณต๊ตฌ๋ฅผ ์™„๋ฃŒํ•˜์˜€๊ณ , 2015. 11. 18. 13:30๊ฒฝ ~ 2015. 11. 20. 15:28๊ฒฝ ํŒŒ์ผ ์„ ๋ณ„์ž‘์—…์„ ํ•œ ๋’ค 2015. 11. 20. 15:29๊ฒฝ ~ 15:45๊ฒฝ ์„ ๋ณ„์••์ˆ˜ํ•˜์˜€๋‹ค. 16) ์ฆ ์ œ137ํ˜ธ ์ด๋ฏธ์ง• ์‚ฌ๋ณธ : 2015. 11. 14. 15:51๊ฒฝ ์„ ๋ณ„์••์ˆ˜ํ•˜์˜€๋‹ค. 17) ์ฆ ์ œ138ํ˜ธ ์ด๋ฏธ์ง• ์‚ฌ๋ณธ : 2015. 11. 17. 19:52๊ฒฝ ์„ ๋ณ„์••์ˆ˜ํ•˜์˜€๋‹ค. ใ€ˆ์ด ๋ฒ•์›์˜ ์ฃผ10) ๊ฒ€์ฆใ€‰ 1) ์ฆ ์ œ123, 124, 125, 126ํ˜ธ : ๊ฐ 2016. 9. 27. ๊ฒ€์ฆํ•˜์—ฌ 2015. 11. 13. ๋ด‰์ธ๋˜์–ด ์žˆ๋Š” ์ƒํƒœ๋ฅผ ํ™•์ธํ•˜์˜€๊ณ , ๊ฒ€์ฆ ๋Œ€์ƒ ํŒŒ์ผ๋“ค์— ๋Œ€ํ•œ ํ•ด์‹œ๊ฐ’์ด ํ˜„์žฅ์—์„œ ์‚ฐ์ถœํ•œ ํ•ด์‹œ๊ฐ’๊ณผ ๋™์ผํ•จ์„ ํ™•์ธํ•˜์˜€๋‹ค(๊ทธ ๋ฐ–์— ํŒŒ์ผ์˜ ์ด๋ฆ„, ํฌ๊ธฐ, ์ˆ˜์ •์‹œ๊ฐ„ ๋“ฑ์˜ ํŒŒ์ผ์ •๋ณด๋“ค๋„ ๊ฒ€์ฆ๊ณผ์ •์—์„œ ํ•จ๊ป˜ ํ™•์ธํ•˜์˜€๋‹ค, ์ดํ•˜ ๊ฐ™๋‹ค). 2) ์ฆ ์ œ122ํ˜ธ : 2016. 10. 4. ๊ฒ€์ฆํ•˜์—ฌ 2015. 11. 13. 11:40 ๋ด‰์ธ๋˜์–ด ์žˆ๋Š” ์ƒํƒœ๋ฅผ ํ™•์ธํ•˜์˜€๋‹ค(2016. 10. 11.์—๋„ ๊ฒ€์ฆ ๊ณ„์†). ๊ฒ€์ฆ ๋Œ€์ƒ ํŒŒ์ผ๋“ค์— ๋Œ€ํ•œ ํ•ด์‹œ๊ฐ’์ด ํ˜„์žฅ์—์„œ ์‚ฐ์ถœํ•œ ํ•ด์‹œ๊ฐ’๊ณผ ๋™์ผํ•จ์„ ํ™•์ธํ•˜์˜€๋‹ค. 3) ์ฆ ์ œ128ํ˜ธ : 2016. 10. 4. ๊ฒ€์ฆํ•˜์—ฌ 2015. 11. 13. 09:11 ๋ด‰์ธ๋˜์–ด ์žˆ๋Š” ์ƒํƒœ๋ฅผ ํ™•์ธํ•˜์˜€๋‹ค. ๊ฒ€์ฆ ๋Œ€์ƒ ํŒŒ์ผ๋“ค์— ๋Œ€ํ•œ ํ•ด์‹œ๊ฐ’์ด ํ˜„์žฅ์—์„œ ์‚ฐ์ถœํ•œ ํ•ด์‹œ๊ฐ’๊ณผ ๋™์ผํ•จ์„ ํ™•์ธํ•˜์˜€๋‹ค. 4) ์ฆ ์ œ129ํ˜ธ : 2016. 10. 10. ๊ฒ€์ฆํ•˜์—ฌ 2015. 11. 13. ๋ด‰์ธ๋˜์–ด ์žˆ๋Š” ์ƒํƒœ๋ฅผ ํ™•์ธํ•˜์˜€๋‹ค. ๊ฒ€์ฆ ๋Œ€์ƒ ํŒŒ์ผ๋“ค์— ๋Œ€ํ•œ ํ•ด์‹œ๊ฐ’์ด ํ˜„์žฅ์—์„œ ์‚ฐ์ถœํ•œ ํ•ด์‹œ๊ฐ’๊ณผ ๋™์ผํ•จ์„ ํ™•์ธํ•˜์˜€๋‹ค. 5) ์ฆ ์ œ131ํ˜ธ : 2016. 10. 17. ๊ฒ€์ฆํ•˜์—ฌ 2015. 11. 24. 14:32 ๋ด‰์ธ(๋ด‰์ธยท์ฐธ๊ด€์ž : ๊ณต์†Œ์™ธ 11)๋˜์–ด ์žˆ๋Š” ์ƒํƒœ๋ฅผ ํ™•์ธํ•˜์˜€๋‹ค. ๋ด‰ํˆฌ ์•ˆ์—๋Š” ์ฆ ์ œ131ํ˜ธ๊ฐ€ ๋‹ด๊ธด ๋ด‰ํˆฌ์™€ ์ด์— ๋Œ€ํ•˜์—ฌ ๋ณต๊ตฌ ์ž‘์—…์„ ํ•œ ํ•˜๋“œ๋””์Šคํฌ๊ฐ€ ๋‹ด๊ธด ๋ด‰ํˆฌ(๊ฐ 2015. 11. 24. 14:26 ๋ด‰์ธ, ๋ด‰์ธยท์ฐธ๊ด€์ž : ๊ณต์†Œ์™ธ 11, ์ด๋ฅผ โ€˜๋ณต๊ตฌ์‚ฌ๋ณธโ€˜์ด๋ผ ํ•œ๋‹ค)๊ฐ€ ๋“ค์–ด์žˆ์—ˆ๋‹ค. ์›๋ณธ์€ ์ผ๋ถ€ ์†์ƒ๋˜์–ด ์ฝ์„ ์ˆ˜ ์—†๊ณ  ๋ณต๊ตฌ์‚ฌ๋ณธ์˜ ํŒŒ์ผํ™•์žฅ์ž๋ฅผ ์ฝ์„ ์ˆ˜ ์žˆ๋Š” ํ”„๋กœ๊ทธ๋žจ์ด ์—†์–ด์„œ, ๋ณต๊ตฌ์‚ฌ๋ณธ์— ๋Œ€ํ•œ ๊ฒ€์ฆ์€ ์ฐจํšŒ ๊ธฐ์ผ์— ํ•˜๊ธฐ๋กœ ํ•˜๊ณ  ๋ณต๊ตฌ์‚ฌ๋ณธ์€ ์žฌ๋ด‰์ธํ•˜์˜€๋‹ค. ์ดํ›„ 2016. 10. 24. ๊ฒ€์ฆ๊ธฐ์ผ์— ๋ณต๊ตฌ์‚ฌ๋ณธ์˜ ๋ด‰์ธ์„ ํ•ด์ œํ•˜๊ณ  ์ €์žฅ๋˜์–ด ์žˆ๋Š” ํŒŒ์ผ๋“ค์˜ ํ•ด์‹œ๊ฐ’, ํŒŒ์ผ์ƒ์„ฑ์‹œ๊ฐ„, ๋งˆ์ง€๋ง‰์ˆ˜์ •์‹œ๊ฐ„, ํŒŒ์ผ์šฉ๋Ÿ‰ ๋“ฑ์ด ๊ฒ€์ฆ์šฉ ์‚ฌ๋ณธ๊ณผ ๋™์ผํ•จ์„ ํ™•์ธํ•˜์˜€๋‹ค. ใ€ˆ์ฆ์ธ๋“ค์˜ ์ฆ์–ธใ€‰ 1) ๊ตญ์ •์› ์ˆ˜์‚ฌ๊ด€ ๊ณต์†Œ์™ธ 47 : ํ”ผ๊ณ ์ธ ์ฃผ๊ฑฐ์ง€ ์••์ˆ˜ยท์ˆ˜์ƒ‰ ํ˜„์žฅ์—๋Š” ํ”ผ๊ณ ์ธ ๋ณธ์ธ๊ณผ ์ฒ˜ ๊ณต์†Œ์™ธ 8 ์™ธ์— ํฌ๋ Œ์‹์ „๋ฌธ๊ฐ€์ธ ๊ณต์†Œ์™ธ 10์ด ์ฐธ์—ฌํ•˜์˜€๋‹ค. ํ”ผ๊ณ ์ธ์œผ๋กœ๋ถ€ํ„ฐ ๋””์ง€ํ„ธ ์ €์žฅ๋งค์ฒด๋ฅผ ์••์ˆ˜ํ•˜๊ณ  ์ด๋ฅผ ๋ด‰์ธํ•  ๋•Œ, ๊ณต์†Œ์™ธ 10์œผ๋กœ ํ•˜์—ฌ๊ธˆ ๊ฐ ์ €์žฅ๋งค์ฒด์˜ ๋ด‰ํˆฌ ๊ฒ‰๋ฉด์— ์ผ์ผ์ด ํ™•์ธยท์„œ๋ช…ํ•˜๋„๋ก ํ•˜์˜€๋‹ค. ํ”ผ๊ณ ์ธ์˜ ๋ฉด์ „์—์„œ ์••์ˆ˜๋ชฉ๋ก๊ณผ ์••์ˆ˜๋ฌผ์„ ์„ค๋ช…ํ•ด ์ฃผ์—ˆ์œผ๋‚˜ ๋ณด๊ธฐ๋งŒํ•˜๊ณ  ๋ณ„๋‹ค๋ฅธ ๋ง์ด ์—†์—ˆ๋‹ค. ์ดํ›„ ์„ ๋ณ„์••์ˆ˜ ๋‹น์‹œ ํ”ผ๊ณ ์ธ๊ณผ ๋ณ€ํ˜ธ์ธ์—๊ฒŒ ์ „์ž์ •๋ณด ์„ ๋ณ„์••์ˆ˜ ๊ณผ์ •์— ์ฐธ์—ฌํ•  ๊ถŒ๋ฆฌ๊ฐ€ ์žˆ๋‹ค๊ณ  ๊ณ ์ง€ํ•˜์˜€์œผ๋‚˜ ์ฐธ์—ฌ๋ฅผ ๊ฑฐ์ ˆํ•˜์˜€๋‹ค. ์„ ๋ณ„์ด ์™„๋ฃŒ๋œ ์ „์ž์ •๋ณด๋ฅผ ์ €์žฅํ•œ ๋งค์ฒด๋Š” 2015. 11. 29. 20:07๋ถ€ํ„ฐ 20:41๊นŒ์ง€ ๊ณต์†Œ์™ธ 10์˜ ์ž…ํšŒํ•˜์— ๋ด‰์ธํ•˜์˜€๋‹ค. ์„ ๋ณ„์ด ์ข…๋ฃŒ๋œ ์ „์ž์ •๋ณด ์ƒ์„ธ๋ชฉ๋ก๊ณผ ์••์ˆ˜๋ชฉ๋ก์„ ํŒŒ์ผ ํ˜•ํƒœ๋กœ CD์— ์ €์žฅํ•œ ํ›„ ํ”ผ๊ณ ์ธ์—๊ฒŒ ๋“ฑ๊ธฐ์šฐํŽธ์œผ๋กœ ๊ต๋ถ€ํ•˜์˜€๋‹ค. 2) ๊ณต์†Œ์™ธ 10 : ํฌ๋ Œ์‹์ „๋ฌธ๊ฐ€๋กœ์„œ ํ”ผ๊ณ ์ธ ์ฃผ๊ฑฐ์ง€ ์••์ˆ˜ยท์ˆ˜์ƒ‰์— ํฌ๋ Œ์‹ ์ž…ํšŒ์ธ ์ž๊ฒฉ์œผ๋กœ ์ฐธ์—ฌํ•˜์˜€๋‹ค. ๋‹น์‹œ ์ „์ž์ •๋ณด ์ €์žฅ๋งค์ฒด๋“ค์— ๋Œ€ํ•œ ์ˆ˜์ƒ‰์— ์•ž์„œ ์“ฐ๊ธฐ๋ฐฉ์ง€ ์žฅ์น˜์— ์—ฐ๊ฒฐํ•˜์˜€๋‹ค. ์ €์žฅ๋งค์ฒด๋ฅผ ์‚ฌ๋ณธํ˜•ํƒœ๋กœ ์••์ˆ˜ํ•˜๋Š” ๊ฒฝ์šฐ, ์›๋ณธ ๋™์ผ์„ฑ์„ ๋ณด์žฅํ•˜๊ธฐ ์œ„ํ•ด ์›๋ณธ๊ณผ ์‚ฌ๋ณธ์˜ ํ•ด์‹œ๊ฐ’์„ ๋น„๊ตํ•˜์—ฌ ํ™•์ธํ•˜์˜€๋‹ค. ๊ตญ์ •์› ์ˆ˜์‚ฌ๊ด€์ด ํ”ผ๊ณ ์ธ์—๊ฒŒ ๋ด‰์ธ์„ ์š”์ฒญํ•˜์˜€์œผ๋‚˜ ํ”ผ๊ณ ์ธ์ด ์ด๋ฅผ ๊ฑฐ์ ˆํ•˜์˜€๋‹ค. ์ด์— ์ฆ์ธ์ด ๋ด‰์ธ์ ˆ์ฐจ์— ์ฐธ์—ฌํ•˜์˜€๊ณ , ๊ฐ๊ฐ์˜ ๋ด‰ํˆฌ ๊ฒ‰๋ฉด์— ์ง์ ‘ ํ™•์ธยท์„œ๋ช…์„ ํ•˜์˜€๋‹ค. ์„ ๋ณ„์••์ˆ˜๋ฅผ ์ง„ํ–‰ํ•  ๋•Œ๋„ ์ž…ํšŒ์ธ์œผ๋กœ ์ฐธ์—ฌํ•˜์˜€๋‹ค. ์„ ๋ณ„์••์ˆ˜์— ์••์„œ ํ”ผ๊ณ ์ธ ์ฃผ๊ฑฐ์ง€์—์„œ ๋ฐ˜์ถœํ•œ ์ „์ž์ •๋ณด ์••์ˆ˜๋ฌผ ๊ฐ๊ฐ์˜ ๋ด‰์ธ ์ƒํƒœ์™€ ๋ด‰์ธํ…Œ์ดํ”„์— ๊ธฐ์žฌ๋œ ์„œ๋ช…์˜ ํ›ผ์† ์—ฌ๋ถ€๋ฅผ ํ™•์ธํ•˜์—ฌ ์ด์ƒ ์—†์Œ์„ ํ™•์ธํ•˜์˜€๋‹ค. ์ „์ž์ •๋ณด ์„ ๋ณ„์••์ˆ˜ ์ „๊ณผ์ •์— ์ฐธ์—ฌํ•˜์˜€๋‹ค. ์„ ๋ณ„์••์ˆ˜๋ฅผ ๋งˆ์นœ ํ›„, ๊ทธ ์ด์™ธ์˜ ์ฆ๊ฑฐ๋ฌผ์„ ํ๊ธฐํ•˜์˜€๊ณ , ๊ฐ ์ €์žฅ๋งค์ฒด๋ฅผ ๋ด‰์ธํ•˜๊ณ  ๋ด‰ํˆฌ ๊ฒ‰๋ฉด์— ์ง์ ‘ ์„œ๋ช…ํ•˜์˜€๋‹ค. 3) ๊ณต์†Œ์™ธ 49 : ๊ตญ์ •์›์—์„œ ํฌ๋ Œ์‹์„ ๋‹ด๋‹นํ•˜๋Š” ์ง์›์ด๋‹ค. ํ”ผ๊ณ ์ธ ์ง‘์„ ์••์ˆ˜ยท์ˆ˜์ƒ‰ํ•  ๋•Œ ๋””์Šคํฌ์— ์žˆ๋Š” ์ €์žฅ๋งค์ฒด๋ฅผ ํฌ๋ Œ์‹ํ•˜๋Š” ์—…๋ฌด๋ฅผ ๋‹ด๋‹นํ•˜์˜€๋‹ค. ๋‹น์‹œ ํ•˜๋“œ๋””์Šคํฌ๋ฅผ ๋ณต์ œํ•  ๋•Œ ์บ ์ฝ”๋”๋กœ ํฌ๋ Œ์‹์˜ ์ „๊ณผ์ •์„ ์ดฌ์˜ํ•˜์˜€๊ณ , ํฌ๋ Œ์‹ ๋ณต์ œ์žฅ๋น„๋ฅผ ์ด์šฉํ•˜์—ฌ ํ•ด์‹œ๊ฐ’ ์‚ฐ์ถœ์„ ํ•˜์˜€์œผ๋ฉฐ, ํฌ๋ Œ์‹์ „๋ฌธ๊ฐ€๋ฅผ ์ž…ํšŒ์‹œ์ผœ ๋ชจ๋“  ๊ณผ์ •์„ ์„ค๋ช…ํ•˜์˜€๋‹ค. ์„ ๋ณ„์••์ˆ˜๋ฅผ ํ•  ๋•Œ๋„ ์„ ๋ณ„์••์ˆ˜์— ์•ž์„œ ์‚ฌ๋ณธ ํ•˜๋“œ๋””์Šคํฌ์˜ ๋ด‰์ธ์„ ํ•ด์ œํ•˜๊ณ  ์„ ๋ณ„์••์ˆ˜ ์ข…๋ฃŒ ํ›„ ์žฌ๋ด‰์ธํ•œ ์ ˆ์ฐจ์— ๊ณต์†Œ์™ธ 10์„ ์ฐธ์—ฌ์‹œ์ผฐ๋‹ค. 4) ๊ณต์†Œ์™ธ 11 : ๊ณต์†Œ์™ธ 13 ์ฃผ์‹ํšŒ์‚ฌ์˜ ๋ถ€์žฅ์ด๋‹ค. 2015. 11. 17. ์ฆ ์ œ131ํ˜ธ(โ—‹โ—‹โ—‹ ํ•˜๋“œ๋””์Šคํฌ)๋ฅผ ๋ฐ›์•„์„œ 2015. 11. 24. ๋ณต๊ตฌ๋ฅผ ์™„๋ฃŒํ•˜์˜€๋‹ค. ์œ„ ํ•˜๋“œ๋””์Šคํฌ์˜ ํ•ด์‹œ๊ฐ’ ํ™•์ธ์„œ๋ฅผ ์ง์ ‘ ์ž‘์„ฑํ•˜์—ฌ ๊ตญ์ •์› ์ˆ˜์‚ฌ๊ด€์—๊ฒŒ ๊ต๋ถ€ํ•˜์˜€๋‹ค. ์œ„ ํ•˜๋“œ๋””์Šคํฌ๋Š” ๋ด‰์ธ๋˜์–ด ์žˆ์—ˆ๋‹ค. ์ฆ์ธ์ด ์ง์ ‘ ๋ด‰์ธ์„ ํ•ด์ œํ•˜๊ณ  ๋ณต๊ตฌ์ž‘์—…์„ ์‹ค์‹œํ•˜์˜€๋‹ค. ํ•˜๋“œ๋””์Šคํฌ๋ฅผ ์ •์ƒ์ ์œผ๋กœ ๋™์ž‘ํ•  ์ˆ˜ ์žˆ๋„๋ก ๋ณต๊ตฌํ•˜์—ฌ ์ด๋ฏธ์ง•ํ•˜์˜€๋‹ค. ๋ฐ์ดํ„ฐ๋ณต๊ตฌ ๊ณผ์ •์€ ๋ฐ์ดํ„ฐ์˜ ๋ณ€๊ฒฝ์ด ์—†๋‹ค. ์ฆ์ธ์˜ ์ž‘์—…์‹ค์˜ ๋ฐ์ดํ„ฐ๋ณต๊ตฌ ์žฅ๋น„๋Š” ์“ฐ๊ธฐ๋ฐฉ์ง€์žฅ์น˜๊ฐ€ ๋˜์–ด ์žˆ์œผ๋ฏ€๋กœ ๋‹ค๋ฅธ ๋ฐ์ดํ„ฐ๊ฐ€ ๋“ค์–ด๊ฐˆ ์—ผ๋ ค๋Š” ์ „ํ˜€ ์—†๋‹ค. ๋ณต๊ตฌ ์ž‘์—…๊ณผ ์ด๋ฏธ์ง•์ด ๋๋‚œ ํ›„ ํ•ด์‹œ๊ฐ’์„ ์ง์ ‘ ์ƒ์„ฑํ•˜์—ฌ ์›๋ณธ(๋ณต๊ตฌ์‚ฌ๋ณธ์„ ๋งํ•˜๋Š” ๊ฒƒ์œผ๋กœ ๋ณด์ธ๋‹ค)๊ณผ ์‚ฌ๋ณธ์˜ ๋™์ผ์„ฑ์„ ํ™•์ธํ•˜๊ณ , ๋ชจ๋‘ ์žฌ๋ด‰์ธํ•˜์—ฌ ์ˆ˜์‚ฌ๊ด€์—๊ฒŒ ์ฃผ์—ˆ๋‹ค. ๋ผ. ํŒ๋‹จ 1) ์•ž์„œ ๋ณธ ๋ฒ•๋ฆฌ์— ์˜ํ•˜๋ฉด ์ฆ๊ฑฐ๋ฌผ์˜ ๋™์ผ์„ฑ์„ ๋ฐ˜๋“œ์‹œ ์••์ˆ˜ํ˜„์žฅ์—์„œ ์ดฌ์˜ํ•œ ์˜์ƒ๋งŒ์œผ๋กœ ์ž…์ฆํ•ด์•ผ ํ•˜๋Š” ๊ฒƒ์€ ์•„๋‹ˆ๋ฏ€๋กœ ์ด๋Ÿฌํ•œ ์ ˆ์ฐจ๋ฅผ ๊ฑฐ์น˜์ง€ ์•Š์•˜๋‹ค๊ณ  ํ•˜์—ฌ ์ฆ๊ฑฐ๋ฌผ๊ณผ ์••์ˆ˜๋ฌผ์˜ ๋™์ผ์„ฑ์„ ์ธ์ •ํ•  ์ˆ˜ ์—†๋‹ค๊ฑฐ๋‚˜ ๊ฒ€์ฆ์ ˆ์ฐจ์— ์œ„๋ฒ•์ด ์žˆ๋‹ค๊ณ  ํ•  ์ˆ˜๋Š” ์—†๋‹ค(์ฆ‰, ์ˆ˜์‚ฌ๊ธฐ๊ด€์€ ํ˜•์‚ฌ์†Œ์†ก๋ฒ• ์ œ219์กฐ, ์ œ120์กฐ ์ œ1ํ•ญ์— ์˜ํ•˜์—ฌ ์••์ˆ˜ยท์ˆ˜์ƒ‰ ์ง‘ํ–‰์— ์žˆ์–ด์„œ ์••์ˆ˜ยท์ˆ˜์ƒ‰ ์˜์žฅ์˜ ๋ชฉ์ ์„ ๋‹ฌ์„ฑํ•˜๊ธฐ ์œ„ํ•˜์—ฌ ํ•„์š”ํ•œ ์ตœ์†Œํ•œ๋„์—์„œ ์‚ฌํšŒํ†ต๋…์ƒ ์ƒ๋‹นํ•˜๋‹ค๊ณ  ์ธ์ •๋˜๋Š” ๋ฐฉ๋ฒ•์œผ๋กœ ํ•„์š”ํ•œ ์ฒ˜๋ถ„์„ ํ•  ์ˆ˜ ์žˆ๋Š”๋ฐ”, ์ˆ˜์‚ฌ๊ธฐ๊ด€์ด ์••์ˆ˜ยท์ˆ˜์ƒ‰์„ ํ•˜๋ฉด์„œ ๊ทธ ์ƒํ™ฉ์„ ์ดฌ์˜ํ•˜๋Š” ๊ฒƒ์€ ์œ„ ํ•„์š”ํ•œ ์ฒ˜๋ถ„์— ํ•ด๋‹นํ•˜๋Š” ๊ฒƒ์ผ ๋ฟ, ๊ทธ ์ž์ฒด๊ฐ€ ๊ณต์†Œ์‚ฌ์‹ค์— ๋Œ€ํ•œ ์ฆ๊ฑฐ๊ฐ€ ๋˜๋Š” ๊ฒƒ์ด ์•„๋‹ˆ๊ณ  ์ฆ๊ฑฐ๋กœ ์ œ์ถœ๋˜์ง€ ์•Š๋Š” ์ด์ƒ ๋ฐ˜๋“œ์‹œ ์••์ˆ˜ยท์ˆ˜์ƒ‰ ๊ณผ์ •์„ ์ดฌ์˜ํ•œ ์˜์ƒ๋…นํ™”๋ฌผ์˜ ์žฌ์ƒ์ด ํ•„์š”ํ•œ ๊ฒƒ์€ ์•„๋‹ˆ๋ผ๊ณ  ํ•  ๊ฒƒ์ด๋ฉฐ, ๋”ฐ๋ผ์„œ ํŠน๋ณ„ํ•œ ์‚ฌ์ •์ด ์—†๋Š” ํ•œ ๋ฒ•์ •์—์„œ ์˜์ƒ๋…นํ™”๋ฌผ์˜ ์žฌ์ƒ์ด ์—†๋‹ค๋Š” ์‚ฌ์ •๋งŒ์œผ๋กœ ๊ณง๋ฐ”๋กœ ๋ฌด๊ฒฐ์„ฑ์˜ ์ž…์ฆ์ด ๋ถ€์กฑํ•˜๋‹ค๊ณ  ๋‹จ์ •ํ•  ์ˆ˜๋Š” ์—†๋‹ค). ๋˜ํ•œ ํ˜•์‚ฌ์†Œ์†ก๋ฒ• ์ œ266์กฐ์˜3์— ์˜ํ•˜์—ฌ ํ”ผ๊ณ ์ธ ๋˜๋Š” ๋ณ€ํ˜ธ์ธ์ด ์—ด๋žŒยท๋“ฑ์‚ฌ๋ฅผ ์‹ ์ฒญํ•  ์ˆ˜ ์žˆ๋Š” ์„œ๋ฅ˜๋Š” ๊ฒ€์‚ฌ๊ฐ€ ์ฆ๊ฑฐ๋กœ ์‹ ์ฒญํ•  ์„œ๋ฅ˜ ๋“ฑ, ๊ฒ€์‚ฌ๊ฐ€ ์ฆ์ธ์œผ๋กœ ์‹ ์ฒญํ•  ์‚ฌ๋žŒ์˜ ์„ฑ๋ช…ยท์‚ฌ๊ฑด๊ณผ์˜ ๊ด€๊ณ„ ๋“ฑ์„ ๊ธฐ์žฌํ•œ ์„œ๋ฉด ๋˜๋Š” ๊ทธ ์‚ฌ๋žŒ์ด ๊ณตํŒ๊ธฐ์ผ ์ „์— ํ–‰ํ•œ ์ง„์ˆ ์„ ๊ธฐ์žฌํ•œ ์„œ๋ฅ˜ ๋“ฑ, ๊ทธ ์„œ๋ฉด ๋˜๋Š” ์„œ๋ฅ˜ ๋“ฑ์˜ ์ฆ๋ช…๋ ฅ๊ณผ ๊ด€๋ จ๋œ ์„œ๋ฅ˜ ๋“ฑ, ํ”ผ๊ณ ์ธ ๋˜๋Š” ๋ณ€ํ˜ธ์ธ์ด ํ–‰ํ•œ ๋ฒ•๋ฅ ์ƒยท์‚ฌ์‹ค์ƒ ์ฃผ์žฅ๊ณผ ๊ด€๋ จ๋œ ์„œ๋ฅ˜ ๋“ฑ์— ํ•œ์ •๋˜๊ณ , ์ œ2ํ•ญ ์†Œ์ •์˜ ์‚ฌ์œ ๊ฐ€ ์žˆ๋Š” ๊ฒฝ์šฐ์—๋Š” ๊ฒ€์‚ฌ๋Š” ์—ด๋žŒยท๋“ฑ์‚ฌ๋ฅผ ๊ฑฐ๋ถ€ํ•  ์ˆ˜๋„ ์žˆ์œผ๋ฉฐ, ์„ค๋ น ํ˜•์‚ฌ์†Œ์†ก๋ฒ• ์ œ266์กฐ์˜4์— ๋”ฐ๋ผ ๋ฒ•์›์ด ๊ฒ€์‚ฌ์— ๋Œ€ํ•˜์—ฌ ๊ทธ ์„œ๋ฅ˜ ๋“ฑ์˜ ์—ด๋žŒยท๋“ฑ์‚ฌ ํ—ˆ์šฉ๋ช…๋ น์„ ํ•˜๋Š” ๊ฒฝ์šฐ์—๋„ ๊ฒ€์‚ฌ๊ฐ€ ์ด๋ฅผ ์ดํ–‰ํ•˜์ง€ ์•„๋‹ˆํ•  ๋•Œ์—๋Š” ํ•ด๋‹น ์„œ๋ฅ˜ ๋“ฑ์„ ์ฆ๊ฑฐ๋กœ ์‹ ์ฒญํ•  ์ˆ˜ ์—†๋Š” ํšจ๊ณผ๊ฐ€ ์žˆ์Œ์— ๊ทธ์นœ๋‹ค. ๊ฒฐ๊ตญ, ์œ„์™€ ๊ฐ™์€ ์••์ˆ˜ยท์ˆ˜์ƒ‰ ์ง‘ํ–‰์˜ ์ฐธ๊ณ ์ž๋ฃŒ์ผ ๋ฟ, ์ด ์‚ฌ๊ฑด์˜ ์ฆ๊ฑฐ์— ํ•ด๋‹นํ•˜์ง€์•„๋‹ˆํ•˜๊ณ  ์ด ์‚ฌ๊ฑด ์ฆ๊ฑฐ๋“ค์˜ ์ฆ๊ฑฐ๋Šฅ๋ ฅ ์ธ์ •์— ํ•„์ˆ˜์ ์ธ ๊ฒƒ๋„ ์•„๋‹Œ ์••์ˆ˜ยท์ˆ˜์ƒ‰ ํ˜„์žฅ ์˜์ƒ์— ๋Œ€ํ•˜์—ฌ ๊ฒ€์‚ฌ๊ฐ€ ์—ด๋žŒยท๋“ฑ์‚ฌ๋ฅผ ํ•ด์ฃผ์ง€ ์•Š์•˜๋‹ค๊ณ  ํ•˜์—ฌ ์œ„๋ฒ•ํ•˜๋‹ค๊ณ  ๋‹จ์ •ํ•˜๊ธฐ ์–ด๋ ต๊ณ , ๋˜ ๊ทธ๋กœ ์ธํ•ด ์‹ค์ œ ์ด ์‚ฌ๊ฑด ์žฌํŒ์— ์–ด๋– ํ•œ ์˜ํ–ฅ์„ ๋ฏธ์น  ์ˆ˜๋„ ์—†์œผ๋ฉฐ, ๋”์šฑ์ด ๊ทธ๋กœ ์ธํ•ด ์œ„ ์••์ˆ˜ยท์ˆ˜์ƒ‰์ ˆ์ฐจ๊ฐ€ ์œ„๋ฒ•ํ•˜๊ฒŒ ๋œ๋‹ค๊ณ  ๋ณผ ์ˆ˜๋„ ์—†๋‹ค. 2) ๋˜ํ•œ ์œ„ ์ธ์ •์‚ฌ์‹ค์— ์˜ํ•˜๋ฉด ์œ„ ๊ฐ ์••์ˆ˜๋ฌผ๋“ค์€ ํ˜„์žฅ์—์„œ ๋ด‰์ธ์ด ์ œ๋Œ€๋กœ ๋˜์—ˆ๊ณ , ๊ทธ ๋ด‰์ธ ์ƒํƒœ๊ฐ€ ์ด ๋ฒ•์›์—์„œ ๊ฒ€์ฆํ•  ๋•Œ๊นŒ์ง€ ์œ ์ง€๋˜์—ˆ์œผ๋ฉฐ, ์••์ˆ˜ ๋‹น์‹œ ์‚ฐ์ถœํ•˜์˜€๋˜ ํ•ด์‹œ๊ฐ’๊ณผ ๊ธฐํƒ€ ํŒŒ์ผ์ •๋ณด ๋“ฑ์ด ๋ด‰์ธ์„ ํ•ด์ œํ•œ ๋’ค ์‚ฐ์ถœํ•œ ํ•ด์‹œ๊ฐ’ ๋“ฑ๊ณผ ๋™์ผํ•˜๋ฏ€๋กœ ์œ„ ์ฆ๊ฑฐ๋ฌผ๋“ค์— ๋Œ€ํ•œ ๋ฌด๊ฒฐ์„ฑยท๋™์ผ์„ฑ์„ ์ธ์ •ํ•  ์ˆ˜ ์žˆ๋‹ค. 3) โ—‹โ—‹โ—‹ ํ•˜๋“œ๋””์Šคํฌ๊ฐ€ ๋ณต๊ตฌ๋œ ์ฆ ์ œ131ํ˜ธ์˜ ๊ฒฝ์šฐ, ๊ณต์†Œ์™ธ 11์˜ ์ฆ์–ธ์— ์˜ํ•˜๋ฉด ๋ณต๊ตฌํ•˜๋Š” ํ•˜๋“œ๋””์Šคํฌ์˜ ํ•˜๋“œ์›จ์–ด์ ์ธ ๋ฌธ์ œ๋ฅผ ํ•ด๊ฒฐํ•œ ๊ฒƒ์ด์ง€ ๊ทธ ์•ˆ์— ์ €์žฅ๋˜์–ด ์žˆ๋Š” ๋ฐ์ดํ„ฐ์— ์˜ํ–ฅ์„ ์ฃผ๋Š” ๊ฒƒ์€ ์•„๋‹ˆ๊ณ , ์ด ๋ฒ•์›์˜ ๊ฒ€์ฆ๊ฒฐ๊ณผ ๊ทธ ๋ด‰์ธ์˜ ์—ฐ์†์„ฑ์ด ์ธ์ •๋˜๋ฏ€๋กœ ์ด ์ฆ๊ฑฐ๋ฌผ์— ๋Œ€ํ•˜์—ฌ๋„ ๋ฌด๊ฒฐ์„ฑยท๋™์ผ์„ฑ์ด ์ธ์ •๋œ๋‹ค. ๋”ฐ๋ผ์„œ ํ”ผ๊ณ ์ธ๊ณผ ๋ณ€ํ˜ธ์ธ์˜ ์ด ๋ถ€๋ถ„ ์ฃผ์žฅ์€ ์ด์œ  ์—†๋‹ค. 4) ํ”ผ๊ณ ์ธ๊ณผ ๋ณ€ํ˜ธ์ธ์€, ๋ด‰์ธ์„ ํ•ด์ œํ•  ๋•Œ ๋ด‰์ธ์„ ํ•œ ์‚ฌ๋žŒ์ด ๋ฐ˜๋“œ์‹œ ์ฐธ์„ํ•ด์•ผ ํ•˜๋Š”๋ฐ๋„ ๋‹ค๋ฅธ ์‚ฌ๋žŒ์ด ๋ด‰์ธ์„ ๋œฏ์Œ์œผ๋กœ์จ ๋ด‰์ธ์˜ ์—ฐ์†์„ฑ์„ ๋‹ด๋ณดํ•  ์ˆ˜ ์—†๊ฒŒ ๋˜์—ˆ๋‹ค๊ณ  ์ฃผ์žฅํ•œ๋‹ค. ๊ทธ๋Ÿฌ๋‚˜ ๋ด‰์ธ์„ ํ•ด์ œํ•œ ์‚ฌ๋žŒ์€ ์ž‘์—…์„ ์™„๋ฃŒํ•œ ๋’ค ํ•ด์ œํ•œ ๋ด‰์ธ์ง€์™€ ๋ด‰ํˆฌ๋ฅผ ์ƒˆ๋กœ์šด ๋ด‰ํˆฌ์— ํ•จ๊ป˜ ๋„ฃ๊ณ  ์žฌ๋ด‰์ธ์„ ํ•˜๊ฒŒ ๋˜์–ด ์žˆ์œผ๋ฏ€๋กœ, ๋ด‰์ธ์„ ํ•ด์ œํ•œ ์‚ฌ๋žŒ์˜ ์ง„์ˆ , ํ•ด์ œ๋œ ๋ด‰์ธ์ง€์™€ ๋ด‰ํˆฌ์˜ ์ƒํƒœ, ์žฌ๋ด‰์ธ์— ์‚ฌ์šฉ๋œ ๋ด‰์ธ์ง€, ์žฌ๋ด‰์ธ๋œ ๋ด‰ํˆฌ์˜ ์ƒํƒœ ๋“ฑ์„ ํ†ตํ•˜์—ฌ ์‚ฌํ›„์ ์œผ๋กœ ๋ด‰์ธ์˜ ์—ฐ์†์„ฑ์„ ํŒ๋‹จํ•  ์ˆ˜ ์žˆ๋Š” ๊ฒƒ์ด๋‹ค. ๊ฒŒ๋‹ค๊ฐ€ ํŒŒ์ผ์˜ ํ•ด์‹œ๊ฐ’์„ ์‚ฐ์ถœํ•˜์—ฌ ํŒŒ์ผ์˜ ๋™์ผ์„ฑ์„ ํ™•์ธํ•˜๋Š” ๋ฐฉ๋ฒ•๋„ ์ฆ๊ฑฐ๋ฌผ์˜ ๋ฌด๊ฒฐ์„ฑยท๋™์ผ์„ฑ์„ ํ™•์ธํ•˜๊ธฐ ์œ„ํ•ด ํ•จ๊ป˜ ์‚ฌ์šฉ๋˜๋ฏ€๋กœ, ์œ„์™€ ๊ฐ™์€ ๋ด‰์ธํ•ด์ œ์™€ ์žฌ๋ด‰์ธ์˜ ๋ฐฉ๋ฒ•๊ณผ ๊ณผ์ •์— ์–ด๋– ํ•œ ์ž˜๋ชป์ด ์žˆ๋‹ค๊ณ  ๋ณผ ์ˆ˜๋Š” ์—†๋‹ค. ํ•œํŽธ, ํ”ผ๊ณ ์ธ๊ณผ ๋ณ€ํ˜ธ์ธ์€ 'toyou11-12.docx' ํŒŒ์ผ์˜ ์ฆ๊ฑฐ๋Šฅ๋ ฅ์„ ๋‹คํˆฌ๋ฉด์„œ 2015. 11. 13. ์ฆ ์ œ125ํ˜ธ์˜ ์••์ˆ˜์ˆ˜์ƒ‰๊ณผ์ •์— ์ฐธ์—ฌํ•œ ๊ณต์†Œ์™ธ 12๊ฐ€ ๋ด‰์ธํ•ด์ œ์— ๊ด€์—ฌํ•˜์ง€ ์•Š์•˜๋‹ค๊ณ  ์ฃผ์žฅํ•œ๋‹ค. ๊ทธ๋Ÿฌ๋‚˜ ์œ„ ํŒŒ์ผ์€ ์ฆ ์ œ123ํ˜ธ์— ์ €์žฅ๋˜์–ด ์žˆ๋˜ ํŒŒ์ผ์ด๊ณ , ๊ณต์†Œ์™ธ 12๋Š” ํ”ผ๊ณ ์ธ์˜ ์‚ฌ๋ฌด์‹ค์˜ ์••์ˆ˜ยท์ˆ˜์ƒ‰๊ณผ ๊ทธ ์ดํ›„์˜ ์„ ๋ณ„์••์ˆ˜๊ณผ์ •์—์„œ๋งŒ ์ฐธ์—ฌํ–ˆ์„ ๋ฟ, ์œ„ ์ฆ ์ œ123ํ˜ธ๋‚˜ ์ œ125ํ˜ธ๊ฐ€ ์••์ˆ˜๋œ ์ง‘, ์ฐจ๋Ÿ‰์˜ ์••์ˆ˜ยท์ˆ˜์ƒ‰์ด๋‚˜ ๊ทธ ์ดํ›„ ์„ ๋ณ„๊ณผ์ •์— ์ฐธ์—ฌํ•˜์ง€ ์•Š์€ ๊ฒƒ์œผ๋กœ ๋ณด์ธ๋‹ค. ์ด๋ฅผ ๋ด‰์ธ์ž์™€ ํ•ด์ œ์ž๊ฐ€ ๋‹ค๋ฅธ ๊ฒฝ์šฐ(์ฆ ์ œ131ํ˜ธ)์— ๋Œ€ํ•œ ์ฃผ์žฅ์œผ๋กœ ์„ ํ•ดํ•˜๋”๋ผ๋„, ๊ณต์†Œ์™ธ 11์˜ ์ฆ์–ธ ๋“ฑ์— ์˜ํ•˜๋ฉด ๊ทธ ๋ด‰์ธ์˜ ์—ฐ์†์„ฑ์„ ์ธ์ •ํ•จ์— ์•„๋ฌด๋Ÿฐ ๋ฌธ์ œ๊ฐ€ ์—†์Œ์€ ์•ž์„œ ๋ณธ ๋ฐ”์™€ ๊ฐ™๋‹ค. 5) ๋”ฐ๋ผ์„œ ํ”ผ๊ณ ์ธ ๋ฐ ๋ณ€ํ˜ธ์ธ์˜ ์ด ๋ถ€๋ถ„ ์ฃผ์žฅ์€ ์ด์œ  ์—†๋‹ค. 9. ํ•ด์™ธ ์ดฌ์˜ ๋™์˜์ƒยท์‚ฌ์ง„์˜ ๋™์ผ์„ฑ, ๋ฌด๊ฒฐ์„ฑ์„ ์ธ์ •ํ•  ์ˆ˜ ์—†๋‹ค๋Š” ์ฃผ์žฅ ๊ฐ€. ์ฃผ์žฅ ๋‹ค์Œ๊ณผ ๊ฐ™์€ ์ด์œ ๋กœ ์•„๋ž˜ ์ฆ๊ฑฐ๋“ค์€ ๋™์ผ์„ฑ๊ณผ ๋ฌด๊ฒฐ์„ฑ์„ ์ธ์ •ํ•  ์ˆ˜ ์—†๋‹ค. 1) ์ค‘๊ตญ ๋Œ€๋ จ์—์„œ ์ดฌ์˜ํ•œ ๋™์˜์ƒ(๊ฒ€์ฆ๋ชฉ๋ก ์ˆœ๋ฒˆ 14)์€ ์›๋ณธ์ด ์‚ญ์ œ๋˜์–ด ์กด์žฌํ•˜์ง€ ์•Š๊ฑฐ๋‚˜ ๋™์ผ์„ฑ, ๋ฌด๊ฒฐ์„ฑ์„ ์ธ์ •ํ•  ์ˆ˜ ์—†๋‹ค. 2) ๋ฒ ํŠธ๋‚จ ํ˜ธ์น˜๋ฏผ์—์„œ ์ดฌ์˜ํ•œ ๋™์˜์ƒ(๊ฒ€์ฆ๋ชฉ๋ก ์ˆœ๋ฒˆ 15 ๋‚ด์ง€ 20)๊ณผ ๋ง๋ ˆ์ด์‹œ์•„ ์ฟ ์•Œ๋ผ๋ฃธํ‘ธ๋ฅด์—์„œ ์ดฌ์˜ํ•œ ๋™์˜์ƒ๊ณผ ์‚ฌ์ง„(๊ฒ€์ฆ๋ชฉ๋ก 21 ๋‚ด์ง€ 28)์€ ๊ฐ๊ฐ ์ฑ„์ฆ ์งํ›„ ๋ด‰์ธ์ ˆ์ฐจ ๋ฐ ํ•ด์‹œ๊ฐ’ ์‚ฐ์ถœ ๋“ฑ ์›๋ณธ๋™์ผ์„ฑ๊ณผ ๋ณด๊ด€์˜ ์—ฐ์†์„ฑ์„ ๋‹ด๋ณดํ•˜๊ธฐ ์œ„ํ•œ ์ ˆ์ฐจ๋ฅผ ๊ฑฐ์น˜์ง€ ์•Š์•˜๋‹ค. ๋‚˜. ๊ด€๋ จ๋ฒ•๋ฆฌ ๋Œ€ํ™” ๋‚ด์šฉ์„ ๋…น์Œํ•œ ํŒŒ์ผ ๋“ฑ์˜ ์ „์ž๋งค์ฒด๋Š” ๊ทธ ์„ฑ์งˆ์ƒ ์ž‘์„ฑ์ž๋‚˜ ์ง„์ˆ ์ž์˜ ์„œ๋ช… ํ˜น์€ ๋‚ ์ธ์ด ์—†์„ ๋ฟ๋งŒ ์•„๋‹ˆ๋ผ, ๋…น์Œ์ž์˜ ์˜๋„๋‚˜ ํŠน์ •ํ•œ ๊ธฐ์ˆ ์— ์˜ํ•˜์—ฌ ๊ทธ ๋‚ด์šฉ์ด ํŽธ์ง‘ยท์กฐ์ž‘๋  ์œ„ํ—˜์„ฑ์ด ์žˆ์Œ์„ ๊ณ ๋ คํ•˜์—ฌ ๊ทธ ๋Œ€ํ™” ๋‚ด์šฉ์„ ๋…น์Œํ•œ ์›๋ณธ์ด๊ฑฐ๋‚˜ ํ˜น์€ ์›๋ณธ์œผ๋กœ๋ถ€ํ„ฐ ๋ณต์‚ฌํ•œ ์‚ฌ๋ณธ์ผ ๊ฒฝ์šฐ์—๋Š” ๋ณต์‚ฌ ๊ณผ์ •์—์„œ ํŽธ์ง‘๋˜๋Š” ๋“ฑ ์ธ์œ„์  ๊ฐœ์ž‘ ์—†์ด ์›๋ณธ์˜ ๋‚ด์šฉ ๊ทธ๋Œ€๋กœ ๋ณต์‚ฌ๋œ ์‚ฌ๋ณธ์ž„์ด ์ž…์ฆ๋˜์–ด์•ผ๋งŒ ํ•˜๊ณ , ๊ทธ๋Ÿฌํ•œ ์ž…์ฆ์ด ์—†๋Š” ๊ฒฝ์šฐ์—๋Š” ์‰ฝ๊ฒŒ ๊ทธ ์ฆ๊ฑฐ๋Šฅ๋ ฅ์„ ์ธ์ •ํ•  ์ˆ˜ ์—†๋‹ค. ๊ทธ๋ฆฌ๊ณ  ์ฆ๊ฑฐ๋กœ ์ œ์ถœ๋œ ๋…น์ŒํŒŒ์ผ์ด ๋Œ€ํ™” ๋‚ด์šฉ์„ ๋…น์Œํ•œ ์›๋ณธ์ด๊ฑฐ๋‚˜ ํ˜น์€ ๋ณต์‚ฌ ๊ณผ์ •์—์„œ ํŽธ์ง‘๋˜๋Š” ๋“ฑ ์ธ์œ„์  ๊ฐœ์ž‘ ์—†์ด ์›๋ณธ ๋‚ด์šฉ์„ ๊ทธ๋Œ€๋กœ ๋ณต์‚ฌํ•œ ์‚ฌ๋ณธ์ด๋ผ๋Š” ์ ์€ ๋…น์ŒํŒŒ์ผ์˜ ์ƒ์„ฑ๊ณผ ์ „๋‹ฌ ๋ฐ ๋ณด๊ด€ ๋“ฑ์˜ ์ ˆ์ฐจ์— ๊ด€์—ฌํ•œ ์‚ฌ๋žŒ์˜ ์ฆ์–ธ์ด๋‚˜ ์ง„์ˆ , ์›๋ณธ์ด๋‚˜ ์‚ฌ๋ณธ ํŒŒ์ผ ์ƒ์„ฑ ์งํ›„์˜ ํ•ด์‹œ(Hash)๊ฐ’๊ณผ์˜ ๋น„๊ต, ๋…น์ŒํŒŒ์ผ์— ๋Œ€ํ•œ ๊ฒ€์ฆยท๊ฐ์ • ๊ฒฐ๊ณผ ๋“ฑ ์ œ๋ฐ˜ ์‚ฌ์ •์„ ์ข…ํ•ฉํ•˜์—ฌ ํŒ๋‹จํ•  ์ˆ˜ ์žˆ๋‹ค(๋Œ€๋ฒ•์› 2015. 1. 22. ์„ ๊ณ  2014๋„10978 ํŒ๊ฒฐ ๋“ฑ ์ฐธ์กฐ). ์ด๋Ÿฌํ•œ ๋ฒ•๋ฆฌ๋Š” ํ”ผ๊ณ ์ธ์˜ ๋ชจ์Šต์„ ๋…นํ™”ํ•œ ๋™์˜์ƒ ํŒŒ์ผ๊ณผ ์ด๋ฅผ ์บก์ณํ•œ ์‚ฌ์ง„์˜ ๊ฒฝ์šฐ์—๋„ ๋งˆ์ฐฌ๊ฐ€์ง€๋กœ ์ ์šฉ๋œ๋‹ค๊ณ  ํ•  ๊ฒƒ์ด๋‹ค. ๋”ฐ๋ผ์„œ ๋™์˜์ƒ ๋˜๋Š” ์บก์ณ์‚ฌ์ง„์˜ ์ฆ๊ฑฐ๋Šฅ๋ ฅ์„ ์ธ์ •ํ•˜๊ธฐ ์œ„ํ•ด์„œ๋Š” โ€˜์›๋ณธ ์ž์ฒดโ€™์ด๊ฑฐ๋‚˜ โ€˜์‚ฌ๋ณธ์ผ ๊ฒฝ์šฐ, ํŽธ์ง‘ ๋“ฑ์˜ ์ธ์œ„์  ๊ฐœ์ž‘ ์—†์ด ์›๋ณธ์˜ ๋‚ด์šฉ ๊ทธ๋Œ€๋กœ ๋ณต์‚ฌ๋œ ๊ฒƒ์ผ ๊ฒƒโ€™์ž„์ด ์ œ๋ฐ˜ ์ฆ๊ฑฐ์— ์˜ํ•ด ์ž…์ฆ๋˜์–ด์•ผ ํ•œ๋‹ค. ๋‹ค. ์ฆ๊ฑฐ์— ์˜ํ•ด ์ธ์ •๋˜๋Š” ์‚ฌ์‹ค ใ€ˆ2011. 4. 21. ์ค‘๊ตญ ๋Œ€๋ จ ์ฑ„์ฆใ€‰ 1) ๊ฒฝ์ฐฐ์ฒญ ๋ณด์•ˆ์ˆ˜์‚ฌ๋Œ€ ์†Œ์†์ด์—ˆ๋˜ ๊ฒฝ์ฐฐ๊ด€ ๊ณต์†Œ์™ธ 14 ๋“ฑ 5~6๋ช…์€ ํ”ผ๊ณ ์ธ์ด ์ค‘๊ตญ ๋Œ€๋ จ์„ ๋ฐฉ๋ฌธํ•  ๋•Œ ํ”ผ๊ณ ์ธ์˜ ๋™ํ–ฅ์„ ํŒŒ์•…ํ•˜๊ณ  ๋ถํ•œ ๊ณต์ž‘์›๊ณผ ํšŒํ•ฉํ•˜๋Š” ์žฅ๋ฉด์„ ์ดฌ์˜ํ•˜์˜€๋‹ค. ์ˆ˜์‚ฌ๊ด€๋“ค์€ ๊ฐ์ž ๋™์˜์ƒ ๋˜๋Š” ์‚ฌ์ง„์„ ์ดฌ์˜ํ•˜์˜€๊ณ , ๋‚˜์ค‘์— ์‚ฌ์ง„์„ ์ทจํ•ฉํ•œ ๊ฒƒ์€ ์œ„ ๊ณต์†Œ์™ธ 14๊ฐ€๋‹ค. ๊ณต์†Œ์™ธ 14๊ฐ€ ์ดฌ์˜ํ•  ๋•Œ ์‚ฌ์šฉํ•œ ๊ฒƒ์€ ์‚ฐ์š” ์บ ์ฝ”๋”์ด๋‹ค. 2) ๊ณต์†Œ์™ธ 14๊ฐ€ ํ”ผ๊ณ ์ธ์˜ ๋ชจ์Šต์„ ์ดฌ์˜ํ•˜๋Š”๋ฐ ์‚ฌ์šฉํ•œ ๋ฉ”๋ชจ๋ฆฌ์นฉ์€ ํ•œ ๊ฐœ๋‹ค. ํ˜„์žฅ์—์„œ ์ดฌ์˜ํ•˜๋ฉด์„œ ์บ ์ฝ”๋”๋ฅผ ๊ณ„์† ์ผœ๋‘” ๊ฒƒ์€ ์•„๋‹ˆ๊ณ , ์ˆ˜์‚ฌ๊ด€๋“ค์ด ํ•„์š”ํ•˜๋‹ค๊ณ  ํŒ๋‹จ๋˜๋Š” ์ƒํ™ฉ์— ์บ ์ฝ”๋”๋ฅผ ์ผœ์„œ ์ดฌ์˜ํ•˜์˜€๋‹ค. ํ•„์š” ์—†๋Š” ์˜์ƒ์€ ํ˜„์žฅ์—์„œ ์‚ญ์ œํ•˜๊ธฐ๋„ ํ•˜์˜€๋‹ค. 3) ๊ณต์†Œ์™ธ 14๋Š” ๊ตญ๋‚ด์— ๋“ค์–ด์˜จ ๋’ค ์œ„ ์ดฌ์˜ ์˜์ƒ์„ ํ•˜๋“œ๋””์Šคํฌ ๊ฐ™์€ ๊ณณ์— ๊ทธ๋Œ€๋กœ ์ €์žฅ์„ ํ•˜์—ฌ ๋ณด๊ด€ํ•˜์˜€๋‹ค. ๋‹ค๋งŒ, ๊ณต์†Œ์™ธ 14๋Š” 2011๋…„ ๋‹น์‹œ์—๋Š” ์›๋ณธ ๋””์ง€ํ„ธ ๋งค์ฒด๋ฅผ ๋ณด๊ด€ํ•˜๋Š” ์ ˆ์ฐจ๋‚˜ ๋ฐฉ๋ฒ•์ด ํ™•๋ฆฝ๋˜์–ด ์žˆ์ง€ ์•Š์•„ ์™„๋ฒฝ(๋ด‰์ธ, ํ•ด์‹œ๊ฐ’ ์‚ฐ์ถœ ๋“ฑ)์„ ๊ธฐํ•˜์ง€๋Š” ๋ชปํ–ˆ๋‹ค๊ณ  ์ง„์ˆ ํ•œ๋‹ค. 4) ๊ตญ์ •์›์€ 2015. 10. 14. ์ด ์˜์ƒ์— ๊ด€ํ•˜์—ฌ ๊ตญ๋ฆฝ๊ณผํ•™์ˆ˜์‚ฌ์—ฐ๊ตฌ์›์— ๊ทธ ์กฐ์ž‘ ์—ฌ๋ถ€์— ๊ด€ํ•œ ๊ฐ์ •์„ ์˜๋ขฐํ•˜์˜€๊ณ  ๊ตญ๋ฆฝ๊ณผํ•™์ˆ˜์‚ฌ์—ฐ๊ตฌ์› ์†Œ์† ๊ณต์†Œ์™ธ 15๋Š” 2015. 10. 23. โ€˜๋””์ง€ํ„ธ ๋ฐ์ดํ„ฐ์˜ ํŠน์„ฑ์ƒ ์ •๊ตํ•œ ์œ„๋ณ€์กฐ์˜ ๊ฒฝ์šฐ ๊ทธ ํ”์ ์„ ๋ฐœ๊ฒฌํ•˜์ง€ ๋ชปํ•  ๊ฐ€๋Šฅ์„ฑ์„ ์™„์ „ํžˆ ๋ฐฐ์ œํ•  ์ˆ˜๋Š” ์ฃผ11) ์—†์œผ๋‚˜, ๊ฐ์ • ๋™์˜์ƒ ํŒŒ์ผ์€ ํ˜•์‹ ๋ฐ ๋ฐ์ดํ„ฐ ๊ตฌ์กฐ์—์„œ ๊ฐ์ •๋ฌผ ์บ ์ฝ”๋”๋กœ ์‹œํ—˜ ์ดฌ์˜ํ•œ ์˜์ƒ๊ณผ ๋™์ผํ•œ ํ˜•์‹์œผ๋กœ ๊ตฌ์„ฑ๋˜์–ด ์žˆ์œผ๋ฉฐ, ์œ„๋ณ€์กฐ๋˜์—ˆ๋‹ค๊ณ  ํŒ๋‹จํ•  ๋งŒํ•œ ํŠน์ด์ ์ด ๋ฐœ๊ฒฌ๋˜์ง€ ์•Š์Œโ€™์ด๋ผ๊ณ  ๊ฐ์ •ํšŒ์‹ ํ•˜์˜€๋‹ค. ใ€ˆ2012. 5. 31. ๋ฒ ํŠธ๋‚จ ํ˜ธ์น˜๋ฏผ ์ฑ„์ฆใ€‰ 1) ๊ณต์†Œ์™ธ 14์™€ ๊ตญ์ •์› ์ˆ˜์‚ฌ๊ด€ 5~6๋ช…์€ ํ”ผ๊ณ ์ธ์ด ๋ฒ ํŠธ๋‚จ ํ˜ธ์น˜๋ฏผ์„ ๋ฐฉ๋ฌธํ•  ๋•Œ ํ”ผ๊ณ ์ธ์˜ ๋™ํ–ฅ์„ ํŒŒ์•…ํ•˜๊ณ  ๋ถํ•œ ๊ณต์ž‘์›๊ณผ ํšŒํ•ฉํ•˜๋Š” ์žฅ๋ฉด์„ ์ดฌ์˜ํ•˜์˜€๋‹ค. ์ด๋•Œ ์‚ฌ์šฉํ•œ ์บ ์ฝ”๋”๋Š” ์‚ฐ์š” ์บ ์ฝ”๋”์ด๋‹ค. 2) ๊ณต์†Œ์™ธ 14๊ฐ€ ์ดฌ์˜ํ•œ ์˜์ƒ์€ ๊ตญ์ •์› ๋‚ด ํฌ๋ Œ์‹ ํŒ€์— ์ „๋‹ฌ๋˜์—ˆ๊ณ , ๊ณต์†Œ์™ธ 14๊ฐ€ ์ž…ํšŒํ•œ ์ƒํƒœ์—์„œ ์“ฐ๊ธฐ๋ฐฉ์ง€ ํ”„๋กœ๊ทธ๋žจ์„ ์‚ฌ์šฉํ•˜์—ฌ ์‚ฌ๋ณธ์„ ๋œจ๊ณ  ์›๋ณธ์€ ๊ทธ๋Œ€๋กœ ๋ณด์กดํ•˜์˜€๋‹ค. ์›๋ณธ์— ๋Œ€ํ•œ ํ•ด์‹œ๊ฐ’์€ ์‚ฐ์ถœํ•˜์ง€ ์•Š์•˜๋‹ค. 3) ๊ตญ์ •์›์€ 2015. 11. 26. ์ด ์˜์ƒ๋“ค์— ๊ด€ํ•˜์—ฌ ๊ตญ๋ฆฝ๊ณผํ•™์ˆ˜์‚ฌ์—ฐ๊ตฌ์›์— ๊ทธ ์กฐ์ž‘ ์—ฌ๋ถ€์— ๊ด€ํ•œ ๊ฐ์ •์„ ์˜๋ขฐํ•˜์˜€๊ณ  ๊ตญ๋ฆฝ๊ณผํ•™์ˆ˜์‚ฌ์—ฐ๊ตฌ์› ์†Œ์† ๊ณต์†Œ์™ธ 15๋Š” 2016. 1. 6. โ€˜๋””์ง€ํ„ธ ๋ฐ์ดํ„ฐ์˜ ํŠน์„ฑ์ƒ ์ •๊ตํ•œ ์œ„๋ณ€์กฐ์˜ ๊ฒฝ์šฐ ๊ทธ ํ”์ ์„ ๋ฐœ๊ฒฌํ•˜์ง€ ๋ชปํ•  ๊ฐ€๋Šฅ์„ฑ์„ ์™„์ „ํžˆ ๋ฐฐ์ œํ•  ์ˆ˜๋Š” ์ฃผ12) ์—†์œผ๋‚˜, ๊ฐ์ •๋ฌผ ๋™์˜์ƒ SANYO004.MP4, SANYO007.MP4์—์„œ๋Š” ์œ„๋ณ€์กฐ ๋˜์—ˆ๋‹ค๊ณ  ํŒ๋‹จํ•  ๋งŒํ•œ ํŠน์ด์ ์€ ๋ฐœ๊ฒฌ๋˜์ง€ ์•Š์Œโ€™์ด๋ผ๊ณ  ๊ฐ์ •ํšŒ์‹ ํ•˜์˜€๋‹ค. ใ€ˆ2015. 4. 5. ๋ง๋ ˆ์ด์‹œ์•„ ์ฟ ์•Œ๋ผ๋ฃธํ‘ธ๋ฅด ์ฑ„์ฆใ€‰ 1) ๊ณต์†Œ์™ธ 14์™€ ๊ตญ์ •์› ์ˆ˜์‚ฌ๊ด€ ์•ฝ 10๋ช…์€ ๊ณต์†Œ์™ธ 6์ด ๋ง๋ ˆ์ด์‹œ์•„ ์ฟ ์•Œ๋ผ๋ฃธํ‘ธ๋ฅด๋ฅผ ๋ฐฉ๋ฌธํ•  ๋•Œ ๊ณต์†Œ์™ธ 6์˜ ๋™ํ–ฅ์„ ํŒŒ์•…ํ•˜๊ณ  ๋ถํ•œ ๊ณต์ž‘์›๊ณผ ํšŒํ•ฉํ•˜๋Š” ์žฅ๋ฉด์„ ์ดฌ์˜ํ•˜์˜€๋‹ค. ์ด๋•Œ ์‚ฌ์šฉํ•œ ์บ ์ฝ”๋”๋Š” ์†Œ๋‹ˆ ์บ ์ฝ”๋”์ด๋‹ค. 2) ๊ณต์†Œ์™ธ 14๊ฐ€ ์ดฌ์˜ํ•œ ์˜์ƒ์€ ๊ตญ์ •์› ๋‚ด ํฌ๋ Œ์‹ ํŒ€์— ์ „๋‹ฌ๋˜์—ˆ๊ณ , ๊ณต์†Œ์™ธ 14๊ฐ€ ์ž…ํšŒํ•œ ์ƒํƒœ์—์„œ ์“ฐ๊ธฐ๋ฐฉ์ง€ ํ”„๋กœ๊ทธ๋žจ์„ ์‚ฌ์šฉํ•˜์—ฌ ์‚ฌ๋ณธ์„ ๋œจ๊ณ  ์›๋ณธ์€ ๊ทธ๋Œ€๋กœ ๋ณด์กดํ•˜์˜€๋‹ค. ์›๋ณธ์— ๋Œ€ํ•œ ํ•ด์‹œ๊ฐ’์€ ์‚ฐ์ถœํ•˜์ง€ ์•Š์•˜๋‹ค. 3) ๊ตญ์ •์›์€ 2015. 11. 17. ์ด ์˜์ƒ๋“ค์— ๊ด€ํ•˜์—ฌ ๊ตญ๋ฆฝ๊ณผํ•™์ˆ˜์‚ฌ์—ฐ๊ตฌ์›์— ๊ทธ ์กฐ์ž‘ ์—ฌ๋ถ€์— ๊ด€ํ•œ ๊ฐ์ •์„ ์˜๋ขฐํ•˜์˜€๊ณ  ๊ตญ๋ฆฝ๊ณผํ•™์ˆ˜์‚ฌ์—ฐ๊ตฌ์› ์†Œ์† ๊ณต์†Œ์™ธ 55๋Š” 2015. 12. 9. โ€˜๋””์ง€ํ„ธ ๋ฐ์ดํ„ฐ์˜ ํŠน์„ฑ์ƒ ์ •๊ตํ•œ ์œ„๋ณ€์กฐ์˜ ๊ฒฝ์šฐ ๊ทธ ํ”์ ์„ ๋ฐœ๊ฒฌํ•˜์ง€ ๋ชปํ•  ๊ฐ€๋Šฅ์„ฑ์„ ์™„์ „ํžˆ ๋ฐฐ์ œํ•  ์ˆ˜๋Š” ์ฃผ13) ์—†์œผ๋‚˜, ๊ฐ์ •๋ฌผ ๋™์˜์ƒ 00031.MTS, 00032.MTS, 00022.MTS, VID20150406073158.MP4, MVI_1216.MOV์—์„œ๋Š” ์œ„๋ณ€์กฐ ๋˜๋Š” ํŽธ์ง‘๋˜์—ˆ๋‹ค๊ณ  ํŒ๋‹จํ•  ๋งŒํ•œ ํŠน์ด์ ์€ ๋ฐœ๊ฒฌ๋˜์ง€ ์•Š์Œโ€™์ด๋ผ๊ณ  ๊ฐ์ •ํšŒ์‹ ํ•˜์˜€๋‹ค. ใ€ˆ์œ„ ์ค‘๊ตญ ๋Œ€๋ จ ์ฑ„์ฆ, ๋ฒ ํŠธ๋‚จ ํ˜ธ์น˜๋ฏผ ์ฑ„์ฆ์— ๊ด€ํ•œ ๊ณต์†Œ์™ธ 15์˜ ์ฆ์–ธใ€‰ ํˆด์ธ ์ธ์ผ€์ด์Šค, ์œˆํ—ฅ์Šค์™€ ๊ฐ™์€ ๋™์˜์ƒํŒŒ์ผ ๋ถ„์„๊ธฐ์™€ ๋‹ค์Œ ํŒŸํ”Œ๋ ˆ์ด์–ด์™€ ๊ฐ™์€ ๋™์˜์ƒ ํ”Œ๋ ˆ์ด์–ด๋ฅผ ๊ฐ™์ด ์‚ฌ์šฉํ•˜์—ฌ ์œ„ ์˜์ƒ์— ๊ด€ํ•˜์—ฌ ๋ฐ์ดํ„ฐ ๊ตฌ์กฐ๋ถ„์„๊ณผ ์˜์ƒ ํ”„๋ ˆ์ž„ ๋‹จ์œ„๋ถ„์„์„ ์‹ค์‹œํ–ˆ๋‹ค. ๋ณต์‚ฌ๋ณธ์„ ๊ฒ€์ฆํ–ˆ๊ณ  ์ด์— ๋Œ€ํ•œ ํ•ด์‹œ๊ฐ’์„ ์‚ฐ์ถœํ•˜์˜€๋‹ค. ๋ฐ์ดํ„ฐ ๊ตฌ์กฐ๋ถ„์„์€ ์ดฌ์˜๊ธฐ๊ธฐ์—์„œ ์ œ๊ณตํ•˜๋Š” ํฌ๋ฑ์ •๋ณด๋‚˜ ๋ฉ”ํƒ€๋ฐ์ดํ„ฐ๋“ค์ด ์‹ค์ œ๋กœ ๊ฐ์ •๋ฌผ์—๋„ ๊ทธ๋Ÿฐ ํŠน์„ฑ๋“ค์ด ๋‚˜ํƒ€๋‚˜ ์žˆ๋Š”์ง€ ํ™•์ธํ•˜๋Š” ํŒŒ์ผ์„ ๋ถ„์„ํ•˜๋Š” ๊ฒƒ์ด๊ณ , ํ”„๋ ˆ์ž„๋‹จ์œ„๋ถ„์„์€ ํ”„๋ ˆ์ž„๋ณ„๋กœ ์—ฐ์†์„ฑ์„ ๊ฐ€์ง€๊ณ  ์žˆ๋Š”์ง€, ๋™์˜์ƒ์ด ์‚ฌ์ง„์˜ ํ•œ ์ปท ํ•œ ์ปท์ด ์—ฐ์†์„ฑ์„ ๊ฐ€์ง€๊ณ  ์žˆ๋Š”์ง€, ๋งŒ์•ฝ ๋™์˜์ƒ์— ์Œ์„ฑ์ด ๊ฐ™์ด ์ €์žฅ๋˜์–ด ์žˆ๋‹ค๋ฉด ์Œ์„ฑ์ƒ์œผ๋กœ๋„ ์—ฐ์†์„ฑ์ด ์žˆ๋Š”์ง€ ๊ฐ™์ด ํ™•์ธํ•˜๋Š” ๊ธฐ๋ฒ•์ด๋‹ค. ๊ฐ์ •๋Œ€์ƒ ๊ฐ ๋™์˜์ƒ์€ ์ดฌ์˜ํ•œ ๊ฒƒ๊ณผ ๋™์ผํ•œ ๊ธฐ์ข…์œผ๋กœ ์ƒ์„ฑํ•œ ์‹œํ—˜ ๋™์˜์ƒ๊ณผ ๋ฉ”ํƒ€๋ฐ์ดํ„ฐ๊ฐ€ ์ฃผ14) ๋™์ผํ–ˆ๊ณ , ์˜์ƒ์ด ํ”„๋ ˆ์ž„๋ณ„๋กœ ์—ฐ์†์„ฑ์„ ๊ฐ€์ง€๊ณ  ์žˆ์—ˆ๋‹ค. ๋”ฐ๋ผ์„œ ์œ„๋ณ€์กฐ๋˜์—ˆ์„ ๊ฐ€๋Šฅ์„ฑ์ด ์ ๋‹ค. ใ€ˆ์œ„ ๋ง๋ ˆ์ด์‹œ์•„ ์ฟ ์•Œ๋ผ๋ฃธํ‘ธ๋ฅด ์ฑ„์ฆ์— ๊ด€ํ•œ ๊ณต์†Œ์™ธ 55์˜ ์ฆ์–ธใ€‰ ํˆด์ธ ์ธ์ผ€์ด์Šค, ์œˆํ—ฅ์Šค ๋˜๋Š” ์ž์ฒด์ ์œผ๋กœ ๊ฐœ๋ฐœํ•œ ์—”์—ํ”„์—์Šค๋””์ง€ํ„ธ์ด๋ผ๋Š” ํ”„๋กœ๊ทธ๋žจ์„ ์ด์šฉํ•˜์—ฌ ์˜์ƒํŒŒ์ผ๋ถ„์„๊ณผ ํ”„๋ ˆ์ž„๋ณ„ ์˜์ƒ์—ฐ์†์„ฑ ๋ถ„์„์„ ํ•˜์˜€๋‹ค. ์˜๋ขฐ๋ฐ›์€ ๋™์˜์ƒ ํŒŒ์ผ์€ ์‹œํ—˜์˜์ƒ๊ณผ ํŒŒ์ผ๋ช…์ด๋‚˜ ์ฝ”๋ฑ์ข…๋ฅ˜, ์••์ถ• ์ •๋ณด, ๋ฉ”ํƒ€๋ฐ์ดํ„ฐ ๋“ฑ ๋ฐ์ดํ„ฐ ๊ตฌ์กฐ์— ์žˆ์–ด์„œ ๋ชจ๋‘ ๋ถ€ํ•ฉํ–ˆ๋‹ค. ๋˜ํ•œ ๊ฐ์ •์˜๋ขฐ ๋™์˜์ƒ์ด ํ”„๋ ˆ์ž„๊ฐ„ ์‹œ๊ณต๊ฐ„์ ์ธ ์—ฐ์†์„ฑ์„ ๊ฐ€์ง€๋ฉฐ ์‹œํ—˜ ๋™์˜์ƒ๊ณผ ๋น„๊ตํ•  ๋•Œ ํ™”์งˆ ๋ฐ ์Œ์งˆ์˜ ์ผ๊ด€์„ฑ์„ ๊ฐ€์ง„๋‹ค. ๋”ฐ๋ผ์„œ ์œ„๋ณ€์กฐ๋˜์—ˆ์„ ๊ฐ€๋Šฅ์„ฑ์ด ์ ๋‹ค. ใ€ˆ์ด ๋ฒ•์›์˜ ๊ฒ€์ฆใ€‰ 2016. 9. 19. ๊ฐ์ • ์‹œ, ์ฆ๊ฑฐ๋ชฉ๋ก ์ˆœ๋ฒˆ 764~778(๊ฒ€์ฆ๋ชฉ๋ก ์ˆœ๋ฒˆ 14~28)์˜ ๋™์˜์ƒ๋“ค์˜ ์›๋ณธ ๋ด‰์ธ์ƒํƒœ๋ฅผ ์•„๋ž˜์™€ ๊ฐ™์ด ํ™•์ธํ•˜๊ณ  ๋ด‰์ธํ•ด์ œํ•œ ๋’ค ๊ฐ ํ•ด์‹œ๊ฐ’์„ ์‚ฐ์ถœํ•˜์—ฌ ์›๋ณธํŒŒ์ผ์˜ ํ•ด์‹œ๊ฐ’๊ณผ ๊ฒ€์ฆ์šฉ ํŒŒ์ผ์˜ ํ•ด์‹œ๊ฐ’์ด ๋™์ผํ•จ์„ ํ™•์ธํ•˜์˜€๋‹ค. ์ดํ›„ ๊ฐ ๊ฒ€์ฆ์šฉ ์‚ฌ๋ณธ ํŒŒ์ผ์„ ์žฌ์ƒํ•˜์—ฌ ์˜์ƒ์„ ์‹œ์ฒญํ•˜๋Š” ๋ฐฉ๋ฒ•์œผ๋กœ ๊ฒ€์‚ฌ๊ฐ€ ์ œ์ถœํ•œ โ€˜๊ฒ€์ฆ๋Œ€์ƒ ์„ค๋ช…์„œโ€™ ๊ธฐ์žฌ์™€ ๊ฐ™์€ ์‚ฌ์‹ค์ด ์žˆ์—ˆ์Œ์„ ํ™•์ธํ•˜์˜€๋‹ค. 1) 2011. 4. 21. ์ค‘๊ตญ ๋Œ€๋ จ ์ฑ„์ฆ๊ด€๋ จ ๋™์˜์ƒ์ธ ์ฆ๊ฑฐ๋ชฉ๋ก ์ˆœ๋ฒˆ 764 : 2015. 12. 14. ๋ด‰์ธ๋˜์–ด ์žˆ์—ˆ๋‹ค. 2) 2012. 5. 31. ๋ฒ ํŠธ๋‚จ ํ˜ธ์น˜๋ฏผ ์ฑ„์ฆ๊ด€๋ จ ๋™์˜์ƒ์ธ ์ฆ๊ฑฐ๋ชฉ๋ก ์ˆœ๋ฒˆ 765~770 : 2015. 12. 14. ๋ด‰์ธ๋˜์–ด ์žˆ์—ˆ๋‹ค. 3) 2015. 4. 5. ๋ง๋ ˆ์ด์‹œ์•„ ์ฟ ์•Œ๋ผ๋ฃธํ‘ธ๋ฅด ์ฑ„์ฆ๊ด€๋ จ ๋™์˜์ƒ์ธ ์ฆ๊ฑฐ๋ชฉ๋ก ์ˆœ๋ฒˆ 771~778 : 2015. 11. 12. ๋ด‰์ธ๋˜์–ด ์žˆ์—ˆ๋‹ค. ๋ผ. ํŒ๋‹จ 1) ์ฆ๊ฑฐ๋ชฉ๋ก ์ˆœ๋ฒˆ 764 ๊ฐ€) ์ด ๋ฒ•์›์˜ ๊ฒ€์ฆ๊ฒฐ๊ณผ ๋“ฑ์— ์˜ํ•˜๋ฉด, ๋ด‰์ธ๋˜์–ด ์žˆ๋˜ ์˜์ƒ๊ณผ ๊ฒ€์ฆ์šฉ ์‚ฌ๋ณธ์˜ ํ•ด์‹œ๊ฐ’์ด ๋™์ผํ•˜๋ฏ€๋กœ ์ด ๋ฒ•์›์ด ๊ฒ€์ฆํ•œ ํŒŒ์ผ์€ ๋‹น์ดˆ ๋ด‰์ธ๋˜์–ด ์žˆ๋˜ ์˜์ƒ๊ณผ ๋™์ผํ•œ ์˜์ƒ์ž„์ด ์ธ์ •๋œ๋‹ค. ๋‚˜) ๋‹ค๋งŒ, ์ด ์˜์ƒ์€ ์ดฌ์˜๋œ ํ›„ ๊ณง๋ฐ”๋กœ ๋ด‰์ธ๋˜๊ณ  ํ•ด์‹œ๊ฐ’์ด ์‚ฐ์ถœ๋œ ๊ฒƒ์€ ์•„๋‹ˆ๋‹ค. ๊ทธ๋Ÿฌ๋‚˜ ์ฆ์ธ ๊ณต์†Œ์™ธ 14, ๊ณต์†Œ์™ธ 15์˜ ๊ฐ ์ฆ์–ธ, ๊ฐ์ •์„œ ๊ธฐ์žฌ ์™ธ์— ์ด ๋ฒ•์›์˜ ๊ฒ€์ฆ๊ฒฐ๊ณผ์ƒ ์˜์ƒ์— ํŠน๋ณ„ํžˆ ๋Š์–ด์ง์ด๋‚˜ ์กฐ์ž‘์˜ ํ”์ ์„ ๋ฐœ๊ฒฌํ•  ์ˆ˜ ์—†๋Š” ์ , ํŒŒ์ผ์˜ ๋งˆ์ง€๋ง‰ ์ˆ˜์ •์‹œ๊ฐ„์ด 2011. 4. 22. ๊ธˆ์š”์ผ ์˜ค์ „ 12:50๊ฒฝ์ธ๋ฐ, ์ด๋Š” ํ”ผ๊ณ ์ธ์ด ์ดฌ์˜๋œ ์‹œ๊ฐ„๋Œ€์™€ ์ผ์น˜ํ•˜๋Š” ์ฃผ15) ์  ๋“ฑ๊นŒ์ง€ ์ข…ํ•ฉํ•˜๋ฉด, ๋ด‰์ธ๋œ ์˜์ƒ์€ ์œ„์กฐ ๋˜๋Š” ๋ณ€์กฐ๋˜์ง€ ์•Š์€ ์›๋ณธ์ž„์„ ์ธ์ •ํ•  ์ˆ˜ ์žˆ๊ณ , ๊ฒ€์ฆ์šฉ ํŒŒ์ผ๋„ ์œ„ ์›๋ณธ์„ ์ธ์œ„์  ๊ฐœ์ž‘ ์—†์ด ๊ทธ๋Œ€๋กœ ๋ณต์‚ฌํ•œ ์‚ฌ๋ณธ์ž„์ด ์ธ์ •๋œ๋‹ค. 2) ์ฆ๊ฑฐ๋ชฉ๋ก ์ˆœ๋ฒˆ 765, 766 ๊ฐ€) ์ด ์˜์ƒ๊ณผ ๊ฒ€์ฆ์šฉ ์‚ฌ๋ณธ์˜ ํ•ด์‹œ๊ฐ’์ด ๋™์ผํ•˜๋ฏ€๋กœ ๊ฒ€์ฆ๋Œ€์ƒ ํŒŒ์ผ์€ ๋ด‰์ธ๋œ ์˜์ƒ๊ณผ ๋™์ผํ•œ ์˜์ƒ์ž„์ด ์ธ์ •๋œ๋‹ค. ๋‚˜) ๊ทธ๋Ÿฐ๋ฐ ์ด ์˜์ƒ๋„ ์ดฌ์˜๋œ ํ›„ ๊ณง๋ฐ”๋กœ ๋ด‰์ธ๋˜๊ณ  ํ•ด์‹œ๊ฐ’์ด ์‚ฐ์ถœ๋œ ๊ฒƒ์€ ์•„๋‹ˆ๋‹ค. ๊ทธ๋Ÿฌ๋‚˜ ์œ„ ๊ธฐ์žฌ ์ฆ๊ฑฐ๋“ค ์™ธ์— ์ด ๋ฒ•์›์˜ ๊ฒ€์ฆ๊ฒฐ๊ณผ์ƒ ๋Š์–ด์ง์ด๋‚˜ ์กฐ์ž‘์˜ ํ”์ ์„ ๋ฐœ๊ฒฌํ•  ์ˆ˜ ์—†๋Š” ์ , ๋งˆ์ง€๋ง‰ ์ˆ˜์ •์‹œ๊ฐ„์ด 2012. 5. 31. ๋ชฉ์š”์ผ 21:31, 22:06๊ฒฝ์ธ๋ฐ, ์ด๋Š” ํ”ผ๊ณ ์ธ์ด ์ดฌ์˜๋œ ์‹œ๊ฐ„๋Œ€์™€ ์ผ์น˜ํ•˜๋Š” ์ฃผ16) ์  ๋“ฑ์„ ๋ชจ๋‘ ์ข…ํ•ฉํ•˜๋ฉด, ๋ด‰์ธ๋œ ์˜์ƒ์€ ์œ„์กฐ ๋˜๋Š” ๋ณ€์กฐ๋˜์ง€ ์•Š์€ ์›๋ณธ์ž„์„ ์ธ์ •ํ•  ์ˆ˜ ์žˆ๊ณ , ๊ฒ€์ฆ์šฉ ํŒŒ์ผ๋„ ์œ„ ์›๋ณธ์„ ์ธ์œ„์  ๊ฐœ์ž‘ ์—†์ด ๊ทธ๋Œ€๋กœ ๋ณต์‚ฌํ•œ ์‚ฌ๋ณธ์ž„์ด ์ธ์ •๋œ๋‹ค. 3) ์ฆ๊ฑฐ๋ชฉ๋ก ์ˆœ๋ฒˆ 771~774, 777 ๊ฐ€) ์ด ์˜์ƒ๊ณผ ๊ฒ€์ฆ์šฉ ์‚ฌ๋ณธ์˜ ํ•ด์‹œ๊ฐ’์ด ๋™์ผํ•˜๋ฏ€๋กœ ๊ฒ€์ฆ๋Œ€์ƒ ํŒŒ์ผ์€ ๋ด‰์ธ๋œ ์˜์ƒ๊ณผ ๋™์ผํ•œ ์˜์ƒ์ž„์ด ์ธ์ •๋œ๋‹ค. ๋‚˜) ์ด ์˜์ƒ๋„ ์ดฌ์˜๋œ ํ›„ ๊ณง๋ฐ”๋กœ ๋ด‰์ธ๋˜๊ณ  ํ•ด์‹œ๊ฐ’์ด ์‚ฐ์ถœ๋œ ๊ฒƒ์€ ์•„๋‹ˆ๋‹ค. ๊ทธ๋Ÿฌ๋‚˜ ์œ„ ๊ธฐ์žฌ ์ฆ๊ฑฐ๋“ค ์™ธ์— ์ด ๋ฒ•์›์˜ ๊ฒ€์ฆ๊ฒฐ๊ณผ์ƒ ๋Š์–ด์ง์ด๋‚˜ ์กฐ์ž‘์˜ ํ”์ ์„ ๋ฐœ๊ฒฌํ•  ์ˆ˜ ์—†๋Š” ์ , ๋ฉ”ํƒ€๋ฐ์ดํ„ฐ์ƒ ๋…นํ™”์‹œ๊ฐ„์ด ์ˆœ๋ฒˆ 771(00031.MTS)์€ 2015. 4. 5. 18:08:37 ~ 18:13:24, ์ˆœ๋ฒˆ 772(00032.MTS)์€ 2015. 4. 5. 18:13:38 ~ 18:14:20, ์ˆœ๋ฒˆ 773(00022.MTS)์€ 2015. 4. 5. 18:18:10 ~ 18:20:58์ธ๋ฐ, ์ด๋Š” ๊ณต์†Œ์™ธ 6์ด ์ดฌ์˜๋œ ์‹œ๊ฐ„๋Œ€์™€ ์ผ์น˜ํ•˜๋Š” ์  ๋“ฑ์„ ๋ชจ๋‘ ์ข…ํ•ฉํ•˜๋ฉด, ๋ด‰์ธ๋œ ์˜์ƒ์€ ์œ„์กฐ ๋˜๋Š” ๋ณ€์กฐ๋˜์ง€ ์•Š์€ ์›๋ณธ์ž„์„ ์ธ์ •ํ•  ์ˆ˜ ์žˆ๊ณ , ๊ฒ€์ฆ์šฉ ํŒŒ์ผ๋„ ์œ„ ์›๋ณธ์„ ์ธ์œ„์  ๊ฐœ์ž‘ ์—†์ด ๊ทธ๋Œ€๋กœ ๋ณต์‚ฌํ•œ ์‚ฌ๋ณธ์ž„์ด ์ธ์ •๋œ๋‹ค. 4) ์ฆ๊ฑฐ๋ชฉ๋ก ์ˆœ๋ฒˆ 767 ๋‚ด์ง€ 770, 775, 776, 778 ์ด ์˜์ƒ๋“ค์€ ๊ตญ๋ฆฝ๊ณผํ•™์ˆ˜์‚ฌ์—ฐ๊ตฌ์›์˜ ๊ฐ์ • ๋Œ€์ƒ๋ฌผ์— ํฌํ•จ๋˜์ง€ ์•Š์•˜๋‹ค. ๊ทธ๋Ÿฌ๋‚˜ ๊ณต์†Œ์™ธ 14์˜ ์ฆ์–ธ๊ณผ ์ด ๋ฒ•์›์˜ ๊ฒ€์ฆ๊ฒฐ๊ณผ๋ฅผ ๋น„๋กฏํ•˜์—ฌ, ์ด ์˜์ƒ๋“ค์€ ์œ„์—์„œ ์ฆ๊ฑฐ๋Šฅ๋ ฅ์ด ์ธ์ •๋œ ํŒŒ์ผ๊ณผ ๋™์ผํ•œ ์‹œ๊ธฐ์— ๋™์ผํ•œ ์บ ์ฝ”๋”๋ฅผ ๊ฐ€์ง€๊ณ  ์ดฌ์˜ํ•œ ๊ฒƒ์œผ๋กœ์„œ ์ดฌ์˜๋œ ๋‚ด์šฉ์ด ์œ ์‚ฌํ•˜์—ฌ ์ „ํ›„ ์—ฐ์†์„ฑ์„ ๊ฐ€์ง€๊ณ  ์žˆ๋Š” ์ , ์ฆ๊ฑฐ๋Šฅ๋ ฅ์ด ์ธ์ •๋˜๋Š” ํŒŒ์ผ๋“ค๊ณผ ๊ทธ ํŒŒ์ผ์˜ ํ™•์žฅ์ž๊ฐ€ ๋™์ผํ•˜๊ณ , ํŒŒ์ผ๋ช…๋„ ๋™์ผํ•œ ํ˜•์‹์ธ ์  ๋“ฑ์„ ์ข…ํ•ฉํ•˜๋ฉด, ์œ„ ์ฆ๊ฑฐ๋“ค๋„ ์›๋ณธ์œผ๋กœ ์ธ์ •ํ•  ์ˆ˜ ์žˆ๋‹ค. 5) ๋”ฐ๋ผ์„œ ์œ„ ์˜์ƒ๋“ค์€ ๋ชจ๋‘ ์ฆ๊ฑฐ๋Šฅ๋ ฅ์„ ์ธ์ •ํ•  ์ˆ˜ ์žˆ๊ณ , ํ”ผ๊ณ ์ธ๊ณผ ๋ณ€ํ˜ธ์ธ์˜ ์ด ๋ถ€๋ถ„ ์ฃผ์žฅ์€ ์ด์œ  ์—†๋‹ค. 10. ํ”ผ๊ณ ์ธ ๋˜๋Š” ๊ณต์†Œ์™ธ 6์˜ ํ•ด์™ธ ํ–‰์ ์— ๊ด€ํ•œ ์บก์ณ ์‚ฌ์ง„ ๊ด€๋ จ ์ฃผ์žฅ ๊ฐ€. ์ฃผ์žฅ ์ค‘๊ตญ ๋Œ€๋ จ(์ฆ๊ฑฐ๋ชฉ๋ก ์ˆœ๋ฒˆ 312), ๋ฒ ํŠธ๋‚จ ํ˜ธ์น˜๋ฏผ(์ฆ๊ฑฐ๋ชฉ๋ก ์ˆœ๋ฒˆ 319), ๋ง๋ ˆ์ด์‹œ์•„ ์ฟ ์•Œ๋ผ๋ฃธํ‘ธ๋ฅด(์ฆ๊ฑฐ๋ชฉ๋ก ์ˆœ๋ฒˆ 349)์—์„œ ํ”ผ๊ณ ์ธ ๋˜๋Š” ๊ณต์†Œ์™ธ 6์„ ์ดฌ์˜ํ•œ ์˜์ƒ์˜ ์บก์ณ ์‚ฌ์ง„๋“ค ์ค‘ ์›๋ณธ ์˜์ƒ์ด ํ™•์ธ๋˜์ง€ ์•Š์€ ์‚ฌ์ง„์€ ์ฆ๊ฑฐ๋Šฅ๋ ฅ์ด ์—†๋‹ค. ๋‚˜. ํŒ๋‹จ ๊ฒ€์‚ฌ๋Š” 2016. 10. 31. ์ฆ๊ฑฐ๋ชฉ๋ก ์ˆœ๋ฒˆ 312, 319์— ๋Œ€ํ•œ ์ฆ๊ฑฐ์‹ ์ฒญ์„ ์ฒ ํšŒํ•˜์˜€๊ณ , ์ฆ๊ฑฐ๋ชฉ๋ก ์ˆœ๋ฒˆ 349๋„ ๊ทธ์ค‘ 4,159, 4160์ชฝ์˜ ์บก์ณ ์‚ฌ์ง„์„ ์ œ์™ธํ•˜๊ณ ๋Š” ๋ชจ๋‘ ์ฒ ํšŒํ•˜์˜€๋‹ค. ๋”ฐ๋ผ์„œ ์ด ์‚ฌ์ง„๋“ค์€ ์ฆ๊ฑฐ๋กœ ์‚ฌ์šฉํ•˜์ง€ ์•Š๋Š”๋‹ค. ํ•œํŽธ 4,159, 4,160์ชฝ ์‚ฌ์ง„๋“ค์€ ์•ž์„œ ์ฆ๊ฑฐ๋Šฅ๋ ฅ์ด ์ธ์ •๋˜์–ด ์ ๋ฒ•ํ•˜๊ฒŒ ์ฆ๊ฑฐ์กฐ์‚ฌํ•œ ์˜์ƒ ์ค‘์—์„œ ๊ทธ๋Œ€๋กœ ์บก์ณํ•œ ๊ฒƒ์ž„์ด ์ธ์ •๋˜๋ฏ€๋กœ ๊ทธ ์ฆ๊ฑฐ๋Šฅ๋ ฅ์ด ์ธ์ •๋œ๋‹ค. 11. ๋ถํ•œ ์ƒ๋ถ€์„  ๊ณต์†Œ์™ธ 39ยท๊ณต์†Œ์™ธ 40์˜ ๋ถํ•œ์—ฌ๊ถŒ ์‚ฌ๋ณธ(์ฆ๊ฑฐ๋ชฉ๋ก ์ˆœ๋ฒˆ 339๋ฒˆ)์˜ ์ฆ๊ฑฐ๋Šฅ๋ ฅ ๊ด€๋ จ ๊ฐ€. ์ฃผ์žฅ 1) ์ž…์ˆ˜๊ฒฝ์œ„์— ๋Œ€ํ•œ ๊ตฌ์ฒด์  ์„ค๋ช…๊ณผ ์ž…์ฆ ์—†์ด ์œ„๋ฒ•ํ•˜๊ฒŒ ์ˆ˜์ง‘๋œ ์ฆ๊ฑฐ์ด๋‹ค. 2) ์›๋ณธ๊ณผ์˜ ๋™์ผ์„ฑ์„ ์ธ์ •ํ•  ์ˆ˜ ์—†๋‹ค. 3) ๊ทธ ๊ธฐ์žฌ๋‚ด์šฉ์˜ ์ง„์ •์„ฑ์„ ์ธ์ •ํ•  ์ˆ˜ ์—†๋‹ค. ๋‚˜. ํŒ๋‹จ 1) ๊ตญ์ •์› ์ˆ˜์‚ฌ๊ด€ ๊ณต์†Œ์™ธ 47์€ ์œ„ ์—ฌ๊ถŒ ์ฃผ17) ์‚ฌ๋ณธ์˜ ์ž…์ˆ˜๊ฒฝ์œ„์— ๊ด€ํ•˜์—ฌ โ€˜์ฆ์ธ์ด ํ”ผ๊ณ ์ธ์˜ ๋ถํ•œ ์ƒ๋ถ€์„ ์„ ์ถ”์ ํ•˜๋Š” ๊ณผ์ •์—์„œ โ˜†์ง€๋„์›, ๊ณต์†Œ์™ธ 40, ๊ณต์†Œ์™ธ 39์˜ ์กด์žฌ์— ๋Œ€ํ•ด์„œ ํŒŒ์•…ํ•˜๊ณ  ์žˆ์—ˆ๋‹ค. ๊ทธ๋Ÿฌ๋˜ ์ฐจ์— 2014. 7.๊ฒฝ ์ž์นด๋ฅดํƒ€ ๊ณตํ•ญ์— ๊ณต์†Œ์™ธ 40๊ณผ ๊ณต์†Œ์™ธ 39๊ฐ€ ์ถœํ˜„ํ•œ ์‚ฌ์‹ค์„ ์ฆ์ธ์ด ํ™•์ธํ–ˆ๋‹ค. ๊ทธ๋ž˜์„œ ์ •ํ™•ํ•œ ์ƒ๋ถ€์„  ์‹ ์›๊ทœ๋ช… ์ฐจ์›์—์„œ ๊ณต์†Œ์™ธ 40๊ณผ ๊ณต์†Œ์™ธ 39์˜ ์—ฌ๊ถŒ์‚ฌ์ง„์„ ํ•ด์™ธํ˜‘์กฐ๋ง์„ ํ†ตํ•ด ์ž…์ˆ˜ํ•˜๊ฒŒ ๋˜์—ˆ๋‹ค. ์ถœ์ฒ˜์™€ ๊ฒฝ๋กœ์— ๋Œ€ํ•ด์„œ๋Š” ์—ฌ๋Ÿฌ ๊ฐ€์ง€ ์ •์ฑ…์ ์ธ ๋ถ€๋ถ„๋„ ์žˆ๊ธฐ ๋•Œ๋ฌธ์— ๋งํ•  ์ˆ˜ ์—†๋‹คโ€™๋Š” ์ทจ์ง€๋กœ๋งŒ ์ง„์ˆ ํ•˜์˜€๋‹ค. 2) ์œ„ ์ง„์ˆ ์— ์˜ํ•˜๋”๋ผ๋„ ์œ„ ์—ฌ๊ถŒ ์‚ฌ๋ณธ์„ ์–ด๋– ํ•œ ์ ˆ์ฐจ๋ฅผ ๊ฑฐ์ณ์„œ ์ทจ๋“ํ–ˆ๋Š”์ง€ ๋ถˆ๋ถ„๋ช…ํ•˜๊ณ , ๊ทธ ์›๋ณธ์˜ ์กด์žฌ ๋ฐ ์›๋ณธ๊ณผ์˜ ๋™์ผ์„ฑ๋„ ์ธ์ •ํ•˜๊ธฐ ์–ด๋ ค์šฐ๋ฉฐ, ๋‹ฌ๋ฆฌ ์ด๋ฅผ ์ธ์ •ํ•  ๋งŒํ•œ ์ž๋ฃŒ๊ฐ€ ์—†๋‹ค. 3) ๋”ฐ๋ผ์„œ ์œ„ ์ฆ๊ฑฐ๋Š” ์ฆ๊ฑฐ๋Šฅ๋ ฅ์ด ์—†์œผ๋ฏ€๋กœ ์ฆ๊ฑฐ๋กœ ์‚ฌ์šฉํ•  ์ˆ˜ ์—†๋‹ค. ํ”ผ๊ณ ์ธ๊ณผ ๋ณ€ํ˜ธ์ธ์˜ ์ด ๋ถ€๋ถ„ ์ฃผ์žฅ์€ ์ด์œ  ์ฃผ18) ์žˆ๋‹ค. 12. ์™ธ๊ตญ๊ณ„ ์ด๋ฉ”์ผ ์••์ˆ˜์ˆ˜์ƒ‰์ ˆ์ฐจ๊ฐ€ ์œ„๋ฒ•ํ•˜์—ฌ ์ฆ๊ฑฐ๋Šฅ๋ ฅ์ด ์—†๋‹ค๋Š” ์ฃผ์žฅ ๊ฐ€. ์ฃผ์žฅ 1) ์ˆ˜์‚ฌ๊ธฐ๊ด€์ด ํ”ผ๊ณ ์ธ์˜ ์ˆ˜์ฒฉ์— ๊ธฐ์žฌ๋œ ์ด๋ฉ”์ผ ์•„์ด๋””์™€ ๋น„๋ฐ€๋ฒˆํ˜ธ๋ฅผ ์ด์šฉํ•˜์—ฌ ๋งˆ์น˜ ์ด๋ฉ”์ผ์‚ฌ์šฉ์ž ๋ณธ์ธ์ธ ๊ฒƒ์ฒ˜๋Ÿผ ์„œ๋ฒ„๊ด€๋ฆฌ์ž๋ฅผ ๊ธฐ๋งํ•˜์—ฌ ํ•ด๋‹น ์ด๋ฉ”์ผ์— ์ ‘์†ํ•˜์˜€๋Š”๋ฐ”, ์ด๋Š” ์ •๋ณดํ†ต์‹ ๋ง ์ด์šฉ์ด‰์ง„ ๋ฐ ์ •๋ณด๋ณดํ˜ธ ๋“ฑ์— ๊ด€ํ•œ ๋ฒ•๋ฅ  ์ œ48์กฐ ์ œ1ํ•ญ์˜ โ€˜์ •๋ณดํ†ต์‹ ๋ง ์นจํ•ดํ–‰์œ„โ€™์— ํ•ด๋‹นํ•˜๊ณ , ํƒ€์ธ์˜ ๊ฐœ์ธ์ •๋ณด๋ฅผ ์œ„๋ฒ•ํ•˜๊ฒŒ ์ˆ˜์ง‘ยท์‚ฌ์šฉํ•œ ๊ฒƒ์ด๋‹ค. 2) ์™ธ๊ตญ์— ์„œ๋ฒ„๊ฐ€ ์žˆ๋Š” ์ด๋ฉ”์ผ๊ณ„์ •์— ์•„์ด๋””์™€ ๋น„๋ฐ€๋ฒˆํ˜ธ๋ฅผ ์ž…๋ ฅํ•œ ๊ฒƒ์€ ๊ด€ํ• ๊ถŒ์ด ์—†๋Š” ๋Œ€์ƒ์— ๋Œ€ํ•˜์—ฌ ์ˆ˜์‚ฌ๋ฅผ ํ•œ ๊ฒƒ์ด๊ณ , ์ ๋ฒ•ํ•œ ์‚ฌ๋ฒ•๊ณต์กฐ์ ˆ์ฐจ๋ฅผ ๊ฑฐ์น˜์ง€ ์•Š์•˜์œผ๋ฏ€๋กœ ์œ„๋ฒ•ํ•œ ์ฆ๊ฑฐ ์ˆ˜์ง‘์ด๋‹ค. 3) ์ธํ„ฐ๋„ท ์„œ๋ฒ„์— ์ €์žฅ๋œ ์ด๋ฉ”์ผ ์ •๋ณด์™€ ์ฆ๊ฑฐ๋กœ ์ œ์ถœ๋œ ์ด๋ฉ”์ผ ๋‚ด์šฉ์˜ ๋™์ผ์„ฑ๊ณผ ๋ฌด๊ฒฐ์„ฑ์„ ์ธ์ •ํ•  ์ˆ˜ ์—†๋‹ค. ๋‚˜. ์ฆ๊ฑฐ์— ์˜ํ•ด ์ธ์ •๋˜๋Š” ์‚ฌ์‹ค ใ€ˆ์˜์žฅ์ง‘ํ–‰๊ณผ์ •ใ€‰ 1) ๊ตญ์ •์› ์ˆ˜์‚ฌ๊ด€์€ ํ”ผ๊ณ ์ธ์˜ ์ฐจ๋Ÿ‰ ์•ˆ์—์„œ ๋ฐœ๊ฒฌํ•œ USB(์ฆ ์ œ126ํ˜ธ) ์•ˆ์— ์žˆ๋˜ info.docx ํŒŒ์ผ์„ ๋ณตํ˜ธํ™”ํ•œ ๋ฌธ์„œ์—์„œ โ€˜๋ถ„๊ธฐ๋งˆ๋‹ค ์‚ฌ์šฉํ•  ์ด๋ฉ”์ผ ์ฃผ์†Œ์™€ ์•”ํ˜ธโ€™๋ฅผ ์•Œ๊ฒŒ ๋˜์—ˆ๋‹ค. ๊ตญ์ •์›๊ณผ ๊ฒ€์ฐฐ์€ 2015. 11. 23. ๋ฒ•์›์œผ๋กœ๋ถ€ํ„ฐ ์œ„์™€ ๊ฐ™์ด ์•Œ๊ฒŒ๋œ ์™ธ๊ตญ๊ณ„ ์„œ๋ฒ„์˜ ์ด๋ฉ”์ผ ์ฃผ์†Œ 10๊ฐœ๋ฅผ ๋Œ€์ƒ์œผ๋กœ ํ•˜๋Š” ์••์ˆ˜์ˆ˜์ƒ‰๊ฒ€์ฆ์˜์žฅ์„ ๋ฐ›์•˜๋‹ค. ๊ทธ ๋Œ€์ƒ ์ด๋ฉ”์ผ์—๋Š” (์˜๋ฌธ ID 1 ์ƒ๋žต)@โ–ฝโ–ฝโ–ฝโ–ฝ.COM ๊ณ„์ •๋„ ํฌํ•จ๋˜์–ด ์žˆ๋‹ค. ์œ„ ์˜์žฅ์—๋Š” ์••์ˆ˜ยท์ˆ˜์ƒ‰ยท๊ฒ€์ฆ์˜ ๋ฐฉ๋ฒ•์„ โ€˜๊ตญ๊ฐ€ ์ •๋ณดํ†ต์‹  ์ธ์ฆ ๊ณต๊ณต๊ธฐ๊ด€์ธ ํ•œ๊ตญ์ธํ„ฐ๋„ท์ง„ํฅ์›(KISA) ์‚ฌ๋ฌด์‹ค์— ์„ค์น˜๋œ ์ธํ„ฐ๋„ท์šฉ PC์—์„œ ์˜์ƒ๋…นํ™” ๋ฐ ๋™ ๊ธฐ๊ด€์˜ ์ „๋ฌธ๊ฐ€, ์ผ๋ฐ˜์ธ ํฌ๋ Œ์‹ ์ „๋ฌธ๊ฐ€๊ฐ€ ์ž…ํšŒํ•œ ๊ฐ€์šด๋ฐ ์ค‘๊ตญ ๊ณต์†Œ์™ธ 1 ํšŒ์‚ฌ ๋ฐ ์ค‘๊ตญ ๊ณต์†Œ์™ธ 2 ํšŒ์‚ฌ ์ด๋ฉ”์ผ ํ™ˆํŽ˜์ด์ง€ ๋กœ๊ทธ์ธ ์ž…๋ ฅ์ฐฝ์— ๊ตญ๊ฐ€์ •๋ณด์›์ด ์••์ˆ˜ยท์ˆ˜์ƒ‰๊ณผ์ •์—์„œ ์ž…์ˆ˜ํ•œ ์œ„ ์ด๋ฉ”์ผ ๊ณ„์ •ยท๋น„๋ฐ€๋ฒˆํ˜ธ๋ฅผ ์ž…๋ ฅ, ๋กœ๊ทธ์ธํ•œ ํ›„ ๊ตญ๊ฐ€๋ณด์•ˆ๋ฒ• ์œ„๋ฐ˜ ๋ฒ”์ฆ ์ž๋ฃŒ ์ถœ๋ ฅ๋ฌผ ๋ฐ ๋™ ์ž๋ฃŒ๋ฅผ ์„ ๋ณ„ํ•˜์—ฌ ์ €์žฅํ•œ ์ €์žฅ๋งค์ฒด ๋ด‰์ธยท์••์ˆ˜ํ•  ๊ฒƒโ€™, โ€˜ํ”ผ๊ณ ์ธ์—๊ฒŒ ์••์ˆ˜ยท์ˆ˜์ƒ‰์— ์ฐธ์—ฌํ•  ๊ธฐํšŒ๋ฅผ ๋ถ€์—ฌํ•œ ๋’ค ๋ณธ๋ฌธ ๊ธฐ์žฌ์™€ ๊ฐ™์€ ๋ฐฉ์‹์œผ๋กœ ์••์ˆ˜ยท์ˆ˜์ƒ‰ํ•  ์ˆ˜ ์žˆ์Œ. ํ”ผ๊ณ ์ธ์—๊ฒŒ ์••์ˆ˜ยท์ˆ˜์ƒ‰์— ์ฐธ์—ฌํ•  ๊ธฐํšŒ๋ฅผ ๋ถ€์—ฌํ•˜๋ฉด ์ถฉ๋ถ„ํ•˜๊ณ , ํ”ผ๊ณ ์ธ์ด ์••์ˆ˜ยท์ˆ˜์ƒ‰์— ์ฐธ์—ฌํ•˜๊ธฐ๋ฅผ ํฌ๊ธฐํ•˜๋Š” ๋“ฑ์˜ ๊ฒฝ์šฐ์—๋Š” ํ”ผ๊ณ ์ธ ์ฐธ์—ฌ ์—†์ด ์••์ˆ˜์ˆ˜์ƒ‰ํ•  ์ˆ˜ ์žˆ์Œโ€™์œผ๋กœ ์ •ํ•˜๊ณ  ์žˆ๋‹ค. 2) ๊ตญ์ •์› ์ˆ˜์‚ฌ๊ด€๋“ค์€ 2015. 11. 24. 11:40 ํ”ผ๊ณ ์ธ ๋ฐ ๋ณ€ํ˜ธ์ธ์—๊ฒŒ ์••์ˆ˜์ˆ˜์ƒ‰๊ฒ€์ฆ์˜์žฅ์ด ๋ฐœ๋ถ€๋œ ์‚ฌ์‹ค์„ ์„ค๋ช…ํ•˜๊ณ  ์œ„ ์˜์žฅ์„ ์ œ์‹œํ•˜๋ฉฐ ์ฐธ์—ฌ์˜์‚ฌ๋ฅผ ๋ฌผ์—ˆ์œผ๋‚˜, ํ”ผ๊ณ ์ธ์€ ๋Œ€๋‹ตํ•˜์ง€ ์•Š์•˜๊ณ  ํ”ผ๊ณ ์ธ์˜ ๋ณ€ํ˜ธ์ธ๋„ ์˜์žฅ์„ ์—ด๋žŒํ–ˆ์„ ๋ฟ, ์ฐธ์—ฌ์˜์‚ฌ๋ฅผ ๋ฐํžˆ์ง€ ์•Š์•˜๋‹ค. ๊ตญ์ •์› ์ˆ˜์‚ฌ๊ด€๋“ค์€ 2015. 11. 26. 10:41๊ฒฝ ํ•œ๊ตญ์ธํ„ฐ๋„ท์ง„ํฅ์›์—์„œ ์ฃผ์ž„์—ฐ๊ตฌ์› ๊ณต์†Œ์™ธ 16๊ณผ ๋””์ง€ํ„ธํฌ๋ Œ์‹ ์ „๋ฌธ๊ฐ€ ๊ณต์†Œ์™ธ 17์„ ์ž…ํšŒํ•˜๊ฒŒ ํ•œ ํ›„ ์˜์žฅ์„ ์ง‘ํ–‰ํ•˜์˜€๋‹ค. 3) ์••์ˆ˜ยท์ˆ˜์ƒ‰์— ๋…ธํŠธ๋ถ์„ ์‚ฌ์šฉํ•˜์˜€๊ณ , Internet Explore ๋ฐ Chrome์„ ํ†ตํ•ด ์˜์žฅ์— ๊ธฐ์žฌ๋œ ์ด๋ฉ”์ผ ์ฃผ์†Œ ๋ฐ ๋น„๋ฐ€๋ฒˆํ˜ธ๋ฅผ ์ž…๋ ฅํ•˜์—ฌ ๋กœ๊ทธ์ธ์ด ์‹คํ–‰๋œ ๊ณ„์ •์— ๋Œ€ํ•ด ๋ฒ”์ฃ„์‚ฌ์‹ค๊ณผ ๊ด€๋ จ๋œ ์ž๋ฃŒ๋ฅผ ์„ ๋ณ„ํ•˜์˜€๋‹ค. ์ฆ‰, ๊ฐ™์€ ๋‚  11:17๊ฒฝ (์˜๋ฌธ ID 1 ์ƒ๋žต)@โ–ฝโ–ฝโ–ฝโ–ฝ.COM ์ด๋ฉ”์ผ์— ๋น„๋ฐ€๋ฒˆํ˜ธ๋ฅผ ์ž…๋ ฅํ•˜์—ฌ ์ „์ฒด๋ณด๊ด€ํ•จ์— ์ €์žฅ๋˜์–ด ์žˆ๋Š” ์ด๋ฉ”์ผ(์ด 17๊ฑด) ์ค‘ ์ด 15๊ฑด์˜ ์ด๋ฉ”์ผ์„ ์„ ๋ณ„ํ•˜์—ฌ ์ด๋ฉ”์ผ ๋ฐ ๊ทธ ์ฒจ๋ถ€ํŒŒ์ผ์„ ์ถ”์ถœํ•˜์—ฌ ์ถœ๋ ฅยท์ €์žฅํ•˜๋Š” ๋ฐฉ๋ฒ•์œผ๋กœ ์••์ˆ˜ํ•˜์˜€๋‹ค. ์ด ๋•Œ, ๋…ธํŠธ๋ถ ๋ฐ”ํƒ•ํ™”๋ฉด์— โ€˜20151126_ํ”ผ๊ณ ์ธ ์ด๋ฉ”์ผโ€™ ํด๋”๋ฅผ ์ƒ์„ฑํ•˜๊ณ , ๊ทธ ํด๋” ์•ˆ์— ๊ฐ๊ฐ์˜ ์ด๋ฉ”์ผ ์ฃผ์†Œ๋ช…์„ ์ด๋ฆ„์œผ๋กœ ํ•˜๋Š” ํ•˜์œ„ ํด๋”๋ฅผ ์ƒ์„ฑํ•œ ํ›„ ๊ทธ ์•ˆ์— ๊ฐ๊ฐ์˜ ์ด๋ฉ”์ผ ๊ณ„์ •์—์„œ ์„ ๋ณ„ํ•œ ์ž๋ฃŒ๋ฅผ ์ €์žฅํ•˜์˜€๋‹ค. 4) ์œ„ ํด๋” ๋‚ด์˜ ์ž๋ฃŒ๋“ค์— ๋Œ€ํ•ด ๋””์ง€ํ„ธ ํฌ๋ Œ์‹ ํ”„๋กœ๊ทธ๋žจ์ธ Encase7์„ ์ด์šฉํ•˜์—ฌ ๊ฐ๊ฐ์˜ ํŒŒ์ผ์— ๋Œ€ํ•œ ํ•ด์‹œ๊ฐ’์„ ์ƒ์„ฑํ•œ ํ›„ ์ „์ฒด ํŒŒ์ผ์„ ๋ณต์‚ฌํ•˜์—ฌ USB 2๊ฐœ์— ๊ฐ๊ฐ ์ €์žฅํ•˜๊ณ  ๊ฐ๊ฐ์˜ ํŒŒ์ผ์— ๋Œ€ํ•œ ํ•ด์‹œ๊ฐ’์ด ๊ธฐ๋ก๋œ โ€˜์ „์ž์ƒ์„ธ์ •๋ณด๋ชฉ๋กโ€™์„ ์ถœ๋ ฅํ•˜์—ฌ ์›๋ณธํŒŒ์ผ๊ณผ ์‚ฌ๋ณธํŒŒ์ผ ๊ฐ๊ฐ์— ๋Œ€ํ•œ ํ•ด์‹œ๊ฐ’์„ ์ผ์ผ์ด ๋น„๊ตํ•˜์—ฌ ํ•ด์‹œ๊ฐ’์ด ๋™์ผํ•จ์„ ํ™•์ธํ•˜์˜€๋‹ค. 5) ๊ฐ™์€ ๋‚  16:00๊ฒฝ ์œ„ ํด๋” ๋‚ด ํŒŒ์ผ๋“ค์„ ์ €์žฅํ•œ USB 2๊ฐœ ์ค‘ 1๊ฐœ๋Š” ๋ด‰์ธํ•˜์—ฌ ๊ณต์†Œ์™ธ 16 ๋ฐ ๊ณต์†Œ์™ธ 17์—๊ฒŒ ๊ฐ๊ฐ ์„œ๋ช…ํ•˜๊ฒŒ ํ•˜์˜€๋‹ค. ใ€ˆ์„ ๋ณ„์••์ˆ˜ ๊ณผ์ •ใ€‰ ๊ตญ์ •์› ์ˆ˜์‚ฌ๊ด€์€ 2015. 11. 27. 10:00๊ฒฝ ~ 14:10๊ฒฝ ๊ตญ์ •์› CT์„ผํ„ฐ B11ํ˜ธ์—์„œ ๊ณต์†Œ์™ธ 17์„ ์ž…ํšŒ์‹œํ‚ค๊ณ  2015. 11. 26. ์••์ˆ˜ํ•œ ํ”ผ๊ณ ์ธ ์ด๋ฉ”์ผ์— ์ €์žฅ๋˜์–ด ์žˆ๋˜ ํŒŒ์ผ์ด ๋“ค์–ด์žˆ๋Š” USB์— ๊ด€ํ•˜์—ฌ ์•„๋ž˜์™€ ๊ฐ™์ด ์„ ๋ณ„์••์ˆ˜๋ฅผ ์ง„ํ–‰ํ•˜์˜€๋‹ค. 1) ๊ฐ™์€ ๋‚  10:02๊ฒฝ ๊ณต์†Œ์™ธ 17๋กœ ํ•˜์—ฌ๊ธˆ USB์˜ ๋ด‰์ธ์ƒํƒœ ๋ฐ ๋ด‰ํˆฌ์— ๊ธฐ์žฌ๋œ ์„œ๋ช…์„ ํ™•์ธํ•˜๋„๋ก ํ•˜์˜€๋‹ค. 10:05๊ฒฝ ๋ฏธ๋ด‰์ธ๋œ ๋ณต์ œ๋ณธ USB 1๊ฐœ๋Š” ๊ณต์†Œ์™ธ 17์˜ ์ž…ํšŒํ•˜์— ํ๊ธฐํ•˜์˜€๋‹ค. 2) 10:10๊ฒฝ ๊ณต์†Œ์™ธ 17 ์ž…ํšŒํ•˜์— ๋ด‰์ธ๋œ USB(์ƒŒ๋””์Šคํฌ 16G, S/N 4C531001530713112074)๋ฅผ ๊ฐœ๋ด‰ํ•˜๊ณ  ์“ฐ๊ธฐ๋ฐฉ์ง€์žฅ์น˜์— ์—ฐ๊ฒฐํ•˜์—ฌ ๊ทธ ์•ˆ์— ์žˆ๋Š” ํŒŒ์ผ๋“ค์— ๋Œ€ํ•˜์—ฌ ์„ ๋ณ„์••์ˆ˜๋ฅผ ์ง„ํ–‰ํ•˜์˜€๋‹ค. 3) 11:23๊ฒฝ ์„ ๋ณ„์™„๋ฃŒํ•œ ์ „์ž์ •๋ณด(์ด 7๊ฑด)์— ๋Œ€ํ•ด ์ƒ์„ธ์••์ˆ˜๋ชฉ๋ก์„ ์ถœ๋ ฅํ•˜์—ฌ ์›๋ณธ๊ณผ ํŒŒ์ผ๋ณ„ ํ•ด์‹œ๊ฐ’๊ณผ ๋น„๊ตํ•˜๊ณ , 11:44๊ฒฝ ์ƒˆ USB๋ฅผ ๋…ธํŠธ๋ถ์— ์—ฐ๊ฒฐํ•˜์—ฌ ์„ ๋ณ„ํ•œ ํŒŒ์ผ์„ ์ธ์ผ€์ด์Šค ์ด๋ฏธ์ง€ํ˜•ํƒœ๋กœ USB(์ƒŒ๋””์Šคํฌ, 16G S/N 4C530001140713110304)์— ์ €์žฅํ•˜์˜€๋‹ค. 4) 12:00๊ฒฝ ์œ„ USB(์ƒŒ๋””์Šคํฌ, 16G S/N 4C530001140713110304)๋ฅผ ๋ด‰์ธํ•œ ํ›„ ๊ณต์†Œ์™ธ 17์—๊ฒŒ ์„œ๋ช…ํ•˜๋„๋ก ํ•˜์˜€๋‹ค. 5) 12:07~13:50๊ฒฝ USB(์ƒŒ๋””์Šคํฌ 16G, S/N 4C531001530713112074)์™€ ์ด๋ฉ”์ผ ์ถœ๋ ฅ๋ฌผ(A4์šฉ์ง€ 63๋งค)์„ ๊ณต์†Œ์™ธ 17 ์ฐธ์—ฌํ•˜์— ๋ชจ๋‘ ํ๊ธฐํ•˜์˜€๋‹ค. ใ€ˆ์ด ๋ฒ•์›์˜ ๊ฒ€์ฆใ€‰ 2016. 9. 27. ๊ฒ€์ฆ ์‹œ 2015. 11. 27. ๋ด‰์ธ๋˜์–ด ์žˆ๋Š” ๊ฒƒ์„ ํ™•์ธํ•˜์˜€๋‹ค. ๋ด‰์ธ์„ ํ•ด์ œํ•œ ๋’ค ํ•ด์‹œ๊ฐ’์„ ์‚ฐ์ถœํ•˜์—ฌ ํ™•์ธํ•˜๊ณ  ์ €์žฅ๋งค์ฒด ์•ˆ์— ์žˆ๋Š” ํŒŒ์ผ์„ ๋ฒ•์ • ๊ฒ€์ฆ์šฉ ์ปดํ“จํ„ฐ์— ๋‹ค์šด๋กœ๋“œํ•˜์—ฌ ํŒŒ์ผ์„ ๋ณตํ˜ธํ™”ํ•˜์—ฌ ์ฝ์—ˆ๋‹ค. ใ€ˆ์ฆ์ธ๋“ค์˜ ์ฆ์–ธใ€‰ 1) ๊ณต์†Œ์™ธ 57 : ๊ตญ์ •์› ์†Œ์†์œผ๋กœ ์••์ˆ˜ยท์ˆ˜์ƒ‰ ์ง‘ํ–‰ ์ˆ˜์‚ฌ๊ด€์ด๋‹ค. ํ”ผ๊ณ ์ธ๊ณผ ๋ณ€ํ˜ธ์ธ์—๊ฒŒ ์••์ˆ˜์ˆ˜์ƒ‰์— ์ฐธ์—ฌํ•  ๊ฒƒ์„ ๋‹ค๋ฅธ ์ˆ˜์‚ฌ๊ด€์ด ๊ณ ์ง€ํ•˜์˜€์œผ๋‚˜ ๊ฑฐ๋ถ€ํ•˜์˜€๋‹ค. ์••์ˆ˜์ˆ˜์ƒ‰์˜์žฅ์˜ ์ทจ์ง€์— ๋”ฐ๋ผ ์••์ˆ˜์ˆ˜์ƒ‰์กฐ์„œ์— ๊ธฐ์žฌ๋œ ๋Œ€๋กœ ์••์ˆ˜ยท์ˆ˜์ƒ‰ยท๊ฒ€์ฆํ•˜์˜€๋‹ค. ์‹ค์ œ ์ˆ˜์ƒ‰ํ•  ์ˆ˜ ์žˆ์—ˆ๋˜ ๊ณ„์ •์€ (์˜๋ฌธ ID 1 ์ƒ๋žต)@โ–ฝโ–ฝโ–ฝโ–ฝ.COM ๋ฐ–์— ์—†๋‹ค. ๋ฒ”์ฃ„ํ–‰์œ„ ๊ด€๋ จ์„ฑ์€ ์ฒ˜์Œ ๋ดค์„ ๋•Œ ์Šคํ…Œ๊ฐ€๋…ธ์ธ์ง€ ์˜์‹ฌ์ด ๋˜์ง€๋งŒ ์ง์ ‘์ ์ธ ๊ด€๋ จ์„ฑ์€ ํŒŒ์•…ํ•  ์ˆ˜ ์—†๊ธฐ ๋•Œ๋ฌธ์— 2์ฐจ์ ์œผ๋กœ ์„ ๋ณ„์••์ˆ˜์ž‘์—…์„ ํ•˜๊ธฐ ์œ„ํ•ด์„œ ํ”ผ๊ณ ์ธ์˜ ๋ฉ”์ผ์„ ์••์ˆ˜ํ•˜์˜€๋‹ค. ์••์ˆ˜์ˆ˜์ƒ‰ ์ข…๋ฃŒ ํ›„ ์••์ˆ˜๋ชฉ๋ก๊ณผ ์ „์ž์ •๋ณด์ƒ์„ธ๋ชฉ๋ก์„ ํ•œ๊ตญ์ธํ„ฐ๋„ท์ง„ํฅ์› ์ง์› ๊ณต์†Œ์™ธ 16์—๊ฒŒ ์ œ๊ณตํ•˜์˜€๋‹ค. 2) ๊ณต์†Œ์™ธ 16 : ํ•œ๊ตญ์ธํ„ฐ๋„ท์ง„ํฅ์› ์ง์›์œผ๋กœ ์ด ์‚ฌ๊ฑด ์••์ˆ˜ยท์ˆ˜์ƒ‰ยท๊ฒ€์ฆ ์ ˆ์ฐจ์— ์ฐธ์—ฌํ•˜์˜€๋‹ค. ์••์ˆ˜์กฐ์„œ์™€ ํ•ด์‹œ๊ฐ’ํ™•์ธ์„œ, ์••์ˆ˜์ˆ˜์ˆ˜์ƒ‰์ฐธ์—ฌํ™•์ธ์„œ์— ์„œ๋ช…ยท๋‚ ์ธํ•œ ๊ฒƒ์ด ๋งž๋‹ค. ์••์ˆ˜์ˆ˜์ƒ‰ ๊ณผ์ •์„ ์ดฌ์˜ํ•˜๋ฉด์„œ ์˜์žฅ์— ๊ธฐ์žฌ๋˜์–ด ์žˆ๋Š” ์ด๋ฉ”์ผ์ฃผ์†Œ๋กœ ๋กœ๊ทธ์ธํ•˜์—ฌ ๊ด€๋ จ๋œ ๋‚ด์šฉ์„ ํ™•์ธํ•œ ํ›„ ์บก์ณ๋ฅผ ํ•˜์˜€๋‹ค. ์ด๋ฉ”์ผ ๋ณธ๋ฌธ์€ โ–กโ–กโ–ก ์บก์ณ ํ”„๋กœ๊ทธ๋žจ์„ ์‚ฌ์šฉํ•˜์—ฌ ์ด๋ฏธ์ง€ ํŒŒ์ผ๋กœ ์ €์žฅํ•˜๊ณ , ์ฒจ๋ถ€๋ฌธ์„œ๊ฐ€ ์žˆ๋Š” ๊ฒฝ์šฐ๋Š” ์›๋ณธ ํŒŒ์ผ๋ช…์— โ€˜๋ฐœ์‹ ์ž๋ช…โ€™์„ ์ถ”๊ฐ€ํ•˜๋Š” ๋ฐฉ๋ฒ•์œผ๋กœ ์ €์žฅํ•˜๋ฉฐ, ์ฒจ๋ถ€๋ฌธ์„œ์— ๋งํฌํŒŒ์ผ์ด ํฌํ•จ๋˜์–ด ์žˆ๋Š” ๊ฒฝ์šฐ์—๋Š” ๋งํฌํŒŒ์ผ์— ์ ‘์†, ํ•ด๋‹น ์‚ฌ์ดํŠธ๋กœ ์ด๋™ํ•˜์—ฌ ํ˜„์ถœ๋œ ํ™”๋ฉด์„ ์ด๋ฉ”์ผ ๋ณธ๋ฌธ๊ณผ ๋™์ผํ•œ ๋ฐฉ๋ฒ•์œผ๋กœ ์บก์ณ, ์ €์žฅํ•˜์˜€๋‹ค. ์••์ˆ˜๊ฐ€ ์™„๋ฃŒ๋œ ํŒŒ์ผ์€ USB์— ์ €์žฅํ•˜์˜€๊ณ , ์ €์žฅ๋œ ๊ฐ๊ฐ์˜ ํŒŒ์ผ์— ๋Œ€ํ•œ ํ•ด์‹œ๊ฐ’ ๋ฐ USB ๋งค์ฒด์— ๋Œ€ํ•œ ํ•ด์‹œ๊ฐ’์„ ํ™•์ธํ–ˆ๋‹ค. USB ๋ด‰์ธ ์ ˆ์ฐจ์— ์ฐธ์—ฌํ•˜์—ฌ ๋ด‰ํˆฌ ๊ฒ‰ํ‘œ๋ฉด์— ํ™•์ธ ์„œ๋ช…์„ ํ•˜์˜€๋‹ค. 3) ๊ณต์†Œ์™ธ 17 : ๋””์ง€ํ„ธํฌ๋ Œ์‹ ๊ด€๋ จ ํšŒ์‚ฌ์—์„œ ๊ทผ๋ฌดํ•˜๊ณ  ์žˆ๋‹ค. ์ด ์‚ฌ๊ฑด ์••์ˆ˜ยท์ˆ˜์ƒ‰ยท๊ฒ€์ฆ ์ ˆ์ฐจ์— ์ฐธ์—ฌํ•˜์˜€๋‹ค. ๊ตญ์ •์› ์ˆ˜์‚ฌ๊ด€๋“ค๊ณผ ํ•œ๊ตญ์ธํ„ฐ๋„ท์ง„ํฅ์› ์ง์›๋“ค์ด ํ•จ๊ป˜ ์˜์žฅ์„ ํ™•์ธํ•œ ํ›„ ํ•ด๋‹น ํ•œ๊ตญ์ธํ„ฐ๋„ท์ง„ํฅ์› ์ง์›์ด ์•„์ด๋””์™€ ํŒจ์Šค์›Œ๋“œ๋ฅผ ์ž…๋ ฅํ•œ ํ›„ ๋ฉ”์ผ์— ์ ‘์†ํ•ด ํ•„์š”ํ•œ ์ฆ๊ฑฐ๋ฅผ ์ถ”์ถœํ•˜์˜€๋‹ค. ๋…ธํŠธ๋ถ์— ๋‹ด๊ธด ์›๋ณธ ์••์ˆ˜ ํŒŒ์ผ๋“ค์„ USB์— ์˜ฎ๊ฒจ ์‚ฌ๋ณธํ˜•ํƒœ๋กœ ์ €์žฅํ•˜๋Š” ๊ณผ์ •์—์„œ ๊ฐœ๋ณ„ ํŒŒ์ผ์˜ ํ•ด์‹œ๊ฐ’์ด ๋ชจ๋‘ ์ผ์น˜ํ•จ์„ ํ™•์ธํ–ˆ๋‹ค. USB๋ฅผ ๋ด‰์ธํ•œ ํ›„ ๋ด‰ํˆฌ ๊ฒ‰๋ฉด์— ํ™•์ธ์„œ๋ช…์„ ํ•˜์˜€๋‹ค. 2015. 11. 27. ๊ตญ์ •์›์—์„œ ์ง„ํ–‰ํ•œ ์„ ๋ณ„์••์ˆ˜์ ˆ์ฐจ์—๋„ ์ฐธ์—ฌํ•˜์˜€๋‹ค. ์••์ˆ˜์กฐ์„œ, ํ•ด์‹œ๊ฐ’ํ™•์ธ์„œ์— ์žํ•„๋กœ ์„œ๋ช…, ๋‚ ์ธํ•œ ๊ฒƒ์ด ๋งž๋‹ค. ์„ ๋ณ„์••์ˆ˜์‹œ USB์˜ ๋ด‰์ธ์ƒํƒœ ๋ฐ ๋ด‰ํˆฌ์— ๊ธฐ์žฌ๋œ ๋ณธ์ธ์˜ ์„œ๋ช…์„ ํ™•์ธํ•˜์˜€๋‹ค. USB๋ฅผ ๊ฐœ๋ด‰ํ•˜๊ณ  ์“ฐ๊ธฐ๋ฐฉ์ง€์žฅ์น˜์— ์—ฐ๊ฒฐํ•œ ํ›„์— ์„ ๋ณ„์••์ˆ˜๋ฅผ ์‹œ์ž‘ํ•˜์˜€๋‹ค. ์„ ๋ณ„์™„๋ฃŒ ํ›„ ์„ ๋ณ„ํ•œ ํŒŒ์ผ๋“ค์„ ์ƒˆ๋กœ์šด USB์— ๋‹ด์•„ ์ €์žฅํ•œ ํ›„ ๋ด‰์ธํ•˜๊ณ  ์ง์ ‘ ์„œ๋ช…ํ•˜์˜€๋‹ค. ๋‹ค. ํŒ๋‹จ 1) ์ •๋ณดํ†ต์‹ ๋ง ์ด์šฉ์ด‰์ง„ ๋ฐ ์ •๋ณด๋ณดํ˜ธ ๋“ฑ์— ๊ด€ํ•œ ๋ฒ•๋ฅ  ์ œ48์กฐ์—์„œ๋Š” โ€˜์ •๋‹นํ•œ ์ ‘๊ทผ๊ถŒํ•œ ์—†์ด ๋˜๋Š” ํ—ˆ์šฉ๋œ ์ ‘๊ทผ๊ถŒํ•œ์„ ๋„˜์–ด ์ •๋ณดํ†ต์‹ ๋ง์— ์นจ์ž…ํ•˜๋Š” ํ–‰์œ„โ€™๋ฅผ ๊ธˆ์ง€ํ•˜๊ณ  ์žˆ๋‹ค. ๊ทธ๋Ÿฌ๋‚˜ ๊ตญ์ •์› ์ˆ˜์‚ฌ๊ด€์ด ํ”ผ๊ณ ์ธ ์ด๋ฉ”์ผ์— ์ ‘์†ํ•œ ๊ฒƒ์€ ์ˆ˜์‚ฌ์˜ ํ•„์š”์ƒ ๋ฒ•์›์˜ ์˜์žฅ์— ์˜ํ•˜์—ฌ ์˜์žฅ์— ๊ธฐ์žฌ๋œ ์ƒ๋‹นํ•œ ๋ฐฉ๋ฒ•์— ๋”ฐ๋ผ์„œ ์ฑ„์ฆํ™œ๋™์„ ํ•œ ๊ฒƒ์ด๋ฏ€๋กœ, ์ˆ˜์‚ฌ๊ด€์€ ์ •๋‹นํ•œ ์ ‘๊ทผ๊ถŒํ•œ์„ ๊ฐ€์ง€๊ณ  ํ”ผ๊ณ ์ธ์˜ ์ด๋ฉ”์ผ์— ์ ‘์†ํ•œ ๊ฒƒ์— ํ•ด๋‹นํ•œ๋‹ค. ๋˜ํ•œ, ํ˜•์‚ฌ์†Œ์†ก๋ฒ• ์ œ120์กฐ ์ œ1ํ•ญ์—์„œ โ€˜์••์ˆ˜ยท์ˆ˜์ƒ‰์˜์žฅ์˜ ์ง‘ํ–‰์— ์žˆ์–ด์„œ๋Š” ๊ฑด์ •(๊ฑด์ •, ์ž๋ฌผ์‡ )์„ ์—ด๊ฑฐ๋‚˜ ๊ฐœ๋ด‰ ๊ธฐํƒ€ ํ•„์š”ํ•œ ์ฒ˜๋ถ„์„ ํ•  ์ˆ˜ ์žˆ๋‹คโ€™๊ณ  ๊ทœ์ •ํ•˜๊ณ  ์žˆ๊ณ , ์ด๋Š” ๊ฒ€์ฆ์˜์žฅ์„ ์ง‘ํ–‰ํ•˜๋Š” ๊ฒฝ์šฐ์—๋„ ์ค€์šฉ๋œ๋‹ค. ์ˆ˜์‚ฌ๊ด€์ด ์ ๋ฒ•ํ•˜๊ฒŒ ์•Œ์•„๋‚ธ ํ”ผ๊ณ ์ธ์˜ ์ด๋ฉ”์ผ ์•„์ด๋””์™€ ๋น„๋ฐ€๋ฒˆํ˜ธ๋ฅผ ์ž…๋ ฅํ•˜๋Š” ๊ฒƒ๋„ ์ด๋Ÿฌํ•œ '๊ธฐํƒ€ ํ•„์š”ํ•œ ์ฒ˜๋ถ„โ€˜์— ํ•ด๋‹นํ•œ๋‹ค๊ณ  ๋ด„์ด ์ƒ๋‹นํ•˜๋‹ค. 2) ํ•œํŽธ ๊ฐœ์ธ์ •๋ณด ๋ณดํ˜ธ๋ฒ•์€ ๊ฐœ์ธ์ •๋ณด์ฒ˜๋ฆฌ์ž๊ฐ€ ์ •๋ณด์ฃผ์ฒด์˜ ๋™์˜๋ฅผ ์–ป์ง€ ์•Š๊ณ  ๊ฐœ์ธ์ •๋ณด๋ฅผ ์ œ3์ž์—๊ฒŒ ์ œ๊ณตํ•˜๋Š” ๋“ฑ์˜ ํ–‰์œ„๋ฅผ ๊ธˆ์ง€ํ•˜๊ณ  ์žˆ์„ ๋ฟ, ๋ฒ•์›์˜ ์˜์žฅ์— ์˜ํ•œ ๊ฐœ์ธ์ •๋ณด์˜ ์ทจ๋“๊นŒ์ง€ ๋ชจ๋‘ ๊ธˆ์ง€ํ•˜๋Š” ๋ฒ•๋ฅ ์ด๋ผ๊ณ  ๋ณผ ์ˆ˜๋Š” ์—†๋‹ค. ๋”ฐ๋ผ์„œ ์ด๋Ÿฌํ•œ ์ˆ˜์‚ฌ๋ฐฉ์‹์ด ๊ฐœ์ธ์ •๋ณด ๋ณดํ˜ธ๋ฒ•์„ ์œ„๋ฐ˜ํ•œ ๊ฒƒ์œผ๋กœ ๋ณผ ์ˆ˜๋„ ์—†๋‹ค. 3) โ–ฝโ–ฝโ–ฝโ–ฝ.COM์˜ ์ด๋ฉ”์ผ์€ ์ „์„ธ๊ณ„ ์–ด๋””์„œ๋‚˜ ์ ‘์†ํ•  ์ˆ˜ ์žˆ์œผ๋ฏ€๋กœ ์ด๋ฉ”์ผ ์„œ๋ฒ„ ๊ด€๋ฆฌ์ž์˜ ์˜์‚ฌ๋Š” ์ •๋‹นํ•œ ๊ถŒํ•œ์„ ๊ฐ€์ง€๊ณ  ์•„์ด๋””์™€ ํŒจ์Šค์›Œ๋“œ๋ฅผ ์•„๋Š” ์ž๋ผ๋ฉด ์–ด๋””์„œ๋“ ์ง€ ์ ‘์†ํ•  ์ˆ˜ ์žˆ๋„๋ก ํ•˜๋ ค๋Š” ๊ฒƒ์œผ๋กœ ์ถ”์ •ํ•  ์ˆ˜ ์žˆ๋‹ค. ๊ตญ์ •์› ์ˆ˜์‚ฌ๊ด€๋“ค์ด ๋ฒ•์›์˜ ์˜์žฅ์— ๊ธฐํ•˜์—ฌ ์œ„ ์ด๋ฉ”์ผ์— ์ ‘๊ทผํ•˜๋Š” ์ •๋‹นํ•œ ๊ถŒํ•œ์„ ๊ฐ€์กŒ๋‹ค๋Š” ์ ์€ ์•ž์„œ ๋ณธ ๋ฐ”์™€ ๊ฐ™์œผ๋ฏ€๋กœ ๊ตญ์ •์› ์ˆ˜์‚ฌ๊ด€์ด ๋Œ€ํ•œ๋ฏผ๊ตญ์—์„œ ์ด๋ฉ”์ผ์— ์ ‘๊ทผํ–ˆ๋‹ค๊ณ  ํ•˜๋”๋ผ๋„ ์–ด๋– ํ•œ ์œ„๋ฒ•์ด ์žˆ๋‹ค๊ณ  ํ•  ์ˆ˜ ์—†๊ณ  ๊ตญ์ œ์ ์ธ ๊ด€ํ• ๊ถŒ์˜ ๋ฌธ์ œ๊ฐ€ ์ƒ๊ธด๋‹ค๊ณ  ๋ณผ ์ˆ˜๋„ ์—†๋‹ค. 4) ๋˜ํ•œ, ๊ตญ์ •์› ์ˆ˜์‚ฌ๊ด€์€ ์™ธ๊ตญ๊ณ„ ์„œ๋ฒ„์— ์ ‘์†ํ•˜์—ฌ ๋ฒ”์ฃ„ํ˜์˜์™€ ๊ด€๋ จ๋œ ํŒŒ์ผ์„ ์ถ”์ถœํ•˜์—ฌ ์ €์žฅํ•˜๋Š” ๋ฐฉ๋ฒ•์œผ๋กœ ์••์ˆ˜ํ•œ ๊ฒƒ์ผ ๋ฟ, ์™ธ๊ตญ์— ์œ„์น˜ํ•œ ์„œ๋ฒ„ ๊ทธ ์ž์ฒด์— ๋Œ€ํ•ด์„œ ์••์ˆ˜ยท์ˆ˜์ƒ‰์„ ํ•œ ๊ฒƒ์ด ์•„๋‹ˆ๊ณ , ๊ตญ์ •์› ์ˆ˜์‚ฌ๊ด€์˜ ์ด๋Ÿฌํ•œ ํ–‰์œ„๊ฐ€ ๊ตญ์ œ๋ฒ•์ƒ ๊ด€ํ• ์˜ ์›์ธ์ด ๋˜๋Š” ํŠน๋ณ„ํ•œ ๋ฌธ์ œ๋ฅผ ์•ผ๊ธฐํ•˜๋Š” ๊ฒƒ๋„ ์•„๋‹ˆ๋ฏ€๋กœ ์ด๋Ÿฌํ•œ ์ˆ˜์‚ฌ๋ฅผ ํ•จ์— ์žˆ์–ด์„œ ๋ฐ˜๋“œ์‹œ ์‚ฌ๋ฒ•๊ณต์กฐ๋ฅผ ๊ฑฐ์ณ์•ผ ํ•œ๋‹ค๊ณ  ๋ณผ ์ˆ˜๋„ ์—†๋‹ค. 5) ์œ„ ์ด๋ฉ”์ผ์€ ์•ž์„œ ์ธ์ •ํ•œ ๋ฐ”์™€ ๊ฐ™์ด ์ด๋ฉ”์ผ ์„œ๋ฒ„์— ์ง์ ‘ ์ ‘์†ํ•˜์—ฌ ๊ฑฐ๊ธฐ์— ์ €์žฅ๋˜์–ด ์žˆ๋Š” ํŒŒ์ผ์„ ์••์ˆ˜ํ•œ ๊ฒƒ์ด๋‹ค. ์ปดํ“จํ„ฐ๋ฅผ ์กฐ์ž‘ํ•˜์—ฌ ํŒŒ์ผ์„ USB์— ์ €์žฅํ•œ ์‚ฌ๋žŒ์€ ๊ตญ์ •์›๊ณผ๋Š” ์ „ํ˜€ ๋ฌด๊ด€ํ•œ ํ•œ๊ตญ์ธํ„ฐ๋„ท์ง„ํฅ์› ์ง์›์ธ ๊ณต์†Œ์™ธ 16์ด๊ณ , ํ˜„์žฅ์—๋Š” ๋˜ ๋‹ค๋ฅธ ๊ฐ๊ด€์  ์ œ3์ž์ธ ๊ณต์†Œ์™ธ 17๋„ ์žˆ์—ˆ์œผ๋ฏ€๋กœ ์ด๋ฉ”์ผ์˜ ํŒŒ์ผ์„ ๋…ธํŠธ๋ถ์— ์ €์žฅํ•˜๋Š” ๊ณผ์ •์—์„œ ์–ด๋– ํ•œ ์กฐ์ž‘์ด ์žˆ์„ ์ˆ˜๋Š” ์—†์—ˆ๋˜ ๊ฒƒ์œผ๋กœ ๋ณด์ธ๋‹ค. ์ผ๋ฐ˜์ ์œผ๋กœ ์ด๋ฉ”์ผ ์„œ๋ฒ„์— ์ ‘์†ํ•˜์—ฌ ํŠน์ • ํŒŒ์ผ์„ ์—ด์–ด๋ณด์•˜์„ ๋•Œ, ๊ทธ ํŒŒ์ผ์ด ์ด๋ฉ”์ผ ์„œ๋ฒ„์— ์ €์žฅ๋˜์–ด ์žˆ๋Š” ํŒŒ์ผ๊ณผ ๋™์ผํ•œ ๊ฒƒ์ž„์€ ๊ฒฝํ—˜์น™์ƒ ๋‹น์—ฐํ•œ ๊ฒƒ์ด๋ฏ€๋กœ, ์œ„ USB์— ์ €์žฅ๋œ ํŒŒ์ผ์€ โ–ฝโ–ฝโ–ฝโ–ฝ.COM ์„œ๋ฒ„์— ์ €์žฅ๋˜์–ด ์žˆ๋Š” ํŒŒ์ผ๊ณผ ๋™์ผํ•จ์ด ์ธ์ •๋œ๋‹ค. ๋˜ํ•œ ์œ„ USB์— ์ €์žฅ๋œ ํŒŒ์ผ๊ณผ ์„ ๋ณ„์••์ˆ˜ํ•˜๊ณ  ๋‚œ ํ›„์˜ ํŒŒ์ผ, ์ด ๋ฒ•์›์—์„œ ๋ด‰์ธ์„ ํ•ด์ œํ•˜๊ณ  ๊ฒ€์ฆํ•œ ํŒŒ์ผ์˜ ํ•ด์‹œ๊ฐ’์ด ๋™์ผํ•˜๊ณ , ๊ด€๋ จ์ž๋“ค์˜ ์ฆ์–ธ๋„ ์ด ๋ฒ•์›์— ์ด๋ฅด๊ธฐ๊นŒ์ง€ ์–ด๋– ํ•œ ์œ„ยท๋ณ€์กฐ๋„ ์—†์—ˆ๋‹ค๋Š” ๊ฒƒ์ด๋ฏ€๋กœ ์ด ๋ฒ•์›์—์„œ ๊ฒ€์ฆํ•œ ์ฆ๊ฑฐ ํŒŒ์ผ๊ณผ ์ด๋ฉ”์ผ ์„œ๋ฒ„์— ์ €์žฅ๋œ ์›๋ณธ ํŒŒ์ผ์˜ ๋™์ผ์„ฑ๊ณผ ๋ฌด๊ฒฐ์„ฑ์„ ์ธ์ •ํ•  ์ˆ˜ ์žˆ๋‹ค. 6) ๋”ฐ๋ผ์„œ ํ”ผ๊ณ ์ธ๊ณผ ๋ณ€ํ˜ธ์ธ์˜ ์ด ๋ถ€๋ถ„ ์ฃผ์žฅ์€ ์ด์œ  ์—†๋‹ค. 13. ํ”ผ๊ณ ์ธ์˜ ๋Œ€ํ™”๋ฅผ ๋…น์Œํ•œ ํŒŒ์ผ๊ณผ ๊ทธ ๋…น์ทจ๋ก์ด ์œ„๋ฒ•์ˆ˜์ง‘์ฆ๊ฑฐ๋ผ๋Š” ์ฃผ์žฅ ๊ฐ€. ์ฃผ์žฅ 1) ํ”ผ๊ณ ์ธ์˜ ๋Œ€ํ™”๋ฅผ ๋…น์Œํ•œ ํŒŒ์ผ๋“ค์€ 2012๋…„๊ฒฝ๋ถ€ํ„ฐ 2015๋…„๊ฒฝ๊นŒ์ง€ ํ”ผ๊ณ ์ธ์„ ๋ฏธํ–‰ํ•˜๋ฉด์„œ ๋ชฐ๋ž˜ ๋…น์Œํ•œ ๊ฒƒ์ด๋ฏ€๋กœ ํ”ผ๊ณ ์ธ์˜ ์ธ๊ฒฉ๊ถŒ๊ณผ ์‚ฌ์ƒํ™œ์„ ๊ณผ๋„ํ•˜๊ฒŒ ์นจํ•ดํ•˜๋Š” ์œ„๋ฒ•ํ•œ ์ˆ˜์‚ฌ์ด๋‹ค. 2) ์œ„ ์ฆ๊ฑฐ๋“ค์€ ์ตœ์ดˆ ๋…น์Œ ์›๋ณธ๊ณผ ๋ฌด๊ฒฐ์„ฑยท๋™์ผ์„ฑ์„ ์ธ์ •ํ•  ์ˆ˜ ์—†๋‹ค. ๋‚˜. ๊ด€๋ จ๋ฒ•๋ฆฌ ์•ž์„œ ๋Œ€๋ฒ•์› 2015. 1. 22. ์„ ๊ณ  2014๋„10978 ํŒ๊ฒฐ ๋“ฑ์—์„œ ๋ณธ ๋ฐ”์™€ ๊ฐ™์ด, ๋…น์ŒํŒŒ์ผ์˜ ์ฆ๊ฑฐ๋Šฅ๋ ฅ์„ ์ธ์ •ํ•˜๊ธฐ ์œ„ํ•ด์„œ๋Š” โ€˜์›๋ณธ์ผ ๊ฒƒโ€™ ๋˜๋Š” โ€˜์‚ฌ๋ณธ์ผ ๊ฒฝ์šฐ, ํŽธ์ง‘ ๋“ฑ์˜ ์ธ์œ„์  ๊ฐœ์ž‘ ์—†์ด ์›๋ณธ์˜ ๋‚ด์šฉ ๊ทธ๋Œ€๋กœ ๋ณต์‚ฌ๋œ ๊ฒƒ์ผ ๊ฒƒโ€™์ด ์ž…์ฆ๋˜์–ด์•ผ ํ•œ๋‹ค. ๋‹ค. ์ฆ๊ฑฐ์— ์˜ํ•ด ์ธ์ •๋˜๋Š” ์‚ฌ์‹ค ใ€ˆ๋…น์Œํ•˜๊ฒŒ ๋œ ๊ฒฝ์œ„ใ€‰ 1) 2013. 10. 22. ๋…น์Œ(์ฆ๊ฑฐ๋ชฉ๋ก ์ˆœ๋ฒˆ 757), 2013. 11. 4. ๋…น์Œ(์ˆœ๋ฒˆ 758) : 2013. 10. 8. ๋ฒ•์›์œผ๋กœ๋ถ€ํ„ฐ ๋Œ€ํ™”์˜ ๋…น์Œ๋„ ๊ฐ€๋Šฅํ•œ ํ†ต์‹ ์ œํ•œ์กฐ์น˜ ๋ฐ ๋Œ€ํ™”๋…น์Œยท์ฒญ์ทจํ—ˆ๊ฐ€์„œ๋ฅผ ๋ฐœ๋ถ€๋ฐ›์•˜๊ณ , ์˜์žฅ์˜ ์ง‘ํ–‰๊ธฐ๊ฐ„์€ 2013. 10. 9. ~ 2013. 12. 8.์ด๋‹ค. ๊ตญ์ •์› ์ˆ˜์‚ฌ๊ด€์€ 2013. 10. 22.๊ณผ 2013. 11. 4.์˜ 2ํšŒ์— ๊ฑธ์ณ (๋ช…์นญ 53 ์ƒ๋žต)ํšŒ๊ด€ (ํ˜ธ์ˆ˜ ์ƒ๋žต)ํ˜ธ (๋ช…์นญ 29 ์ƒ๋žต)์—ฐ๊ตฌ์†Œ์—์„œ ๋…น์Œํ•˜์˜€๋‹ค. 2) 2013. 12. 14. ๋…น์Œ(์ˆœ๋ฒˆ 754), 2014. 1. 4. ๋…น์Œ(์ˆœ๋ฒˆ 755, 756) : 2013. 12. 11. ๋ฒ•์›์œผ๋กœ๋ถ€ํ„ฐ ๋Œ€ํ™”์˜ ๋…น์Œ๋„ ๊ฐ€๋Šฅํ•œ ํ†ต์‹ ์ œํ•œ์กฐ์น˜ ๋ฐ ๋Œ€ํ™”๋…น์Œยท์ฒญ์ทจํ—ˆ๊ฐ€์„œ๋ฅผ ๋ฐœ๋ถ€๋ฐ›์•˜๊ณ , ์˜์žฅ์˜ ์ง‘ํ–‰๊ธฐ๊ฐ„์€ 2013. 12. 12. ~ 2014. 2. 11.์ด๋‹ค. ๊ตญ์ •์› ์ˆ˜์‚ฌ๊ด€์€ 2013. 12. 14.๊ณผ 2014. 1. 4. ์ˆ˜์›์‹œ์— ์žˆ๋Š” โ€˜(๋ช…์นญ 3 ์ƒ๋žต) ์„ธ๋ฅ˜์—ญ์ โ€™ ์ปคํ”ผ์ˆ์—์„œ ๋…น์Œํ•˜์˜€๋‹ค. 3) 2014. 1. 22. ๋…น์Œ(์ˆœ๋ฒˆ 759), 2014. 2. 3. ๋…น์Œ(์ˆœ๋ฒˆ 760) : 2013. 12. 11. ๋ฒ•์›์œผ๋กœ๋ถ€ํ„ฐ ๋Œ€ํ™”์˜ ๋…น์Œ๋„ ๊ฐ€๋Šฅํ•œ ํ†ต์‹ ์ œํ•œ์กฐ์น˜ ๋ฐ ๋Œ€ํ™”๋…น์Œยท์ฒญ์ทจํ—ˆ๊ฐ€์„œ๋ฅผ ๋ฐœ๋ถ€๋ฐ›์•˜๊ณ , ์˜์žฅ์˜ ์ง‘ํ–‰๊ธฐ๊ฐ„์€ 2013. 12. 12. ~ 2014. 2. 11.์ด๋‹ค. ๊ฒฝ์ฐฐ๊ด€ ๊ณต์†Œ์™ธ 14๊ฐ€ (๋ช…์นญ 29 ์ƒ๋žต)์—ฐ๊ตฌ์†Œ โ—‘์ธต์— ์ง์ ‘ ์˜ฌ๋ผ๊ฐ€ ๋ณต๋„ ์ชฝ์—์„œ ์ฐฝ๊ฐ€์— ๋…น์Œ๊ธฐ๋ฅผ ์˜ฌ๋ ค๋†“๊ณ  ๋…น์Œํ•˜์˜€๋‹ค. 4) 2014. 2. 2. ๋…น์Œ(์ˆœ๋ฒˆ 762, 763) : 2013. 12. 11. ๋ฒ•์›์œผ๋กœ๋ถ€ํ„ฐ ๋Œ€ํ™”์˜ ๋…น์Œ๋„ ๊ฐ€๋Šฅํ•œ ํ†ต์‹ ์ œํ•œ์กฐ์น˜ ๋ฐ ๋Œ€ํ™”๋…น์Œยท์ฒญ์ทจํ—ˆ๊ฐ€์„œ๋ฅผ ๋ฐœ๋ถ€๋ฐ›์•˜๊ณ , ์˜์žฅ์˜ ์ง‘ํ–‰๊ธฐ๊ฐ„์€ 2013. 12. 12. ~ 2014. 2. 11.์ด๋‹ค. ๊ตญ์ •์› ์ˆ˜์‚ฌ๊ด€ ๊ณต์†Œ์™ธ 47์ด ๊ฐ•๋ณ€์—ญ ์ธ๊ทผ โ€˜(๋ช…์นญ 56 ์ƒ๋žต)โ€™ ์ปคํ”ผ์ˆ์—์„œ ๋…น์Œํ•˜์˜€๋‹ค. 5) 2014. 4. 28. ๋…น์Œ(์ˆœ๋ฒˆ 761) : 2014. 4. 16. ๋ฒ•์›์œผ๋กœ๋ถ€ํ„ฐ ๋Œ€ํ™”์˜ ๋…น์Œ๋„ ๊ฐ€๋Šฅํ•œ ํ†ต์‹ ์ œํ•œ์กฐ์น˜ ๋ฐ ๋Œ€ํ™”๋…น์Œยท์ฒญ์ทจํ—ˆ๊ฐ€์„œ๋ฅผ ๋ฐœ๋ถ€๋ฐ›์•˜๊ณ , ์˜์žฅ์˜ ์ง‘ํ–‰๊ธฐ๊ฐ„์€ 2014. 4. 17. ~ 2014. 6. 16.์ด๋‹ค. ๊ฒฝ์ฐฐ๊ด€ ๊ณต์†Œ์™ธ 14, ๊ตญ์ •์› ์ˆ˜์‚ฌ๊ด€ ๊ณต์†Œ์™ธ 47์ด (๋ช…์นญ 29 ์ƒ๋žต)์—ฐ๊ตฌ์†Œ โ—‘์ธต์— ์ง์ ‘ ์˜ฌ๋ผ๊ฐ€ ๋ณต๋„ ์ชฝ์—์„œ ์ฐฝ๊ฐ€์— ๋…น์Œ๊ธฐ๋ฅผ ์˜ฌ๋ ค๋†“๊ณ  ๋…น์Œํ•˜์˜€๋‹ค. 6) 2015. 3. 8. ๋…น์Œ(์ˆœ๋ฒˆ 751), 2015. 3. 30. ๋…น์Œ(์ˆœ๋ฒˆ 752) : 2015. 2. 28. ๋ฒ•์›์œผ๋กœ๋ถ€ํ„ฐ ๋Œ€ํ™”๋…น์Œ์ด ๊ฐ€๋Šฅํ•œ ํ†ต์‹ ์ œํ•œ์กฐ์น˜ํ—ˆ๊ฐ€์„œ๋ฅผ ๋ฐœ๋ถ€๋ฐ›์•˜๊ณ , ์˜์žฅ์˜ ์ง‘ํ–‰๊ธฐ๊ฐ„์€ 2015. 3. 1. ~ 2015. 4. 28.์ด๋‹ค. ๊ตญ์ •์› ์ˆ˜์‚ฌ๊ด€ ๊ณต์†Œ์™ธ 47์ด ์ž ์‹ค (๋ช…์นญ 49 ์ƒ๋žต) ํ˜ธํ”„, ์ข…๊ฐ (๋ช…์นญ 50 ์ƒ๋žต) ์นดํŽ˜์—์„œ ๋…น์Œํ•˜์˜€๋‹ค. 7) 2015. 11. 1. ๋…น์Œ(์ˆœ๋ฒˆ 753) : 2015. 10. 20. ๋ฒ•์›์œผ๋กœ๋ถ€ํ„ฐ ๋Œ€ํ™”๋…น์Œ์ด ๊ฐ€๋Šฅํ•œ ํ†ต์‹ ์ œํ•œ์กฐ์น˜ํ—ˆ๊ฐ€์„œ๋ฅผ ๋ฐœ๋ถ€๋ฐ›์•˜๊ณ , ์˜์žฅ์˜ ์ง‘ํ–‰๊ธฐ๊ฐ„์€ 2015. 10. 21. ~ 2015. 12. 20.์ด๋‹ค. ๊ตญ์ •์› ์ˆ˜์‚ฌ๊ด€ ๊ณต์†Œ์™ธ 47์ด ์ž ์‹ค (๋ช…์นญ 49 ์ƒ๋žต) ํ˜ธํ”„์—์„œ ๋…น์Œํ•˜์˜€๋‹ค. ใ€ˆ์ด ๋ฒ•์›์˜ ๊ฒ€์ฆใ€‰ 1) 2016. 9. 5. ๊ฒ€์ฆ๊ธฐ์ผ์— ์•„๋ž˜ ์›๋ณธ ์ €์žฅ๋งค์ฒด์˜ ๋ด‰์ธ์ƒํƒœ๋ฅผ ํ™•์ธํ•˜๊ณ , ๋ด‰์ธ์„ ํ•ด์ œํ•œ ํ›„ ์›๋ณธ ํŒŒ์ผ๊ณผ ๊ฒ€์ฆ์šฉ ์‚ฌ๋ณธ ํŒŒ์ผ์— ๋Œ€ํ•œ ๊ฐ ํ•ด์‹œ๊ฐ’ ํ™•์ธ ๋“ฑ์„ ํ†ตํ•˜์—ฌ ๋™์ผ์„ฑ์—ฌ๋ถ€๋ฅผ ํ™•์ธํ•˜์˜€๋‹ค. ์ดํ›„ ์žฌ์ƒยท์‹œ์ฒญํ•˜๊ณ  ์žฌ๋ด‰์ธํ•˜์˜€๋‹ค. โ—‹ ์ฆ๊ฑฐ๋ชฉ๋ก ์ˆœ๋ฒˆ 751(0308_193959.MP3) : 2015. 3. 10. ๋ด‰์ธ ์ดํ›„ 2015. 11. 19. ์žฌ๋ด‰์ธํ•œ ๊ฒƒ์„ ํ™•์ธํ•˜์˜€๋‹ค. โ—‹ ์ˆœ๋ฒˆ 752(0330_215048.MP3) : 2015. 3. 31. ๋ด‰์ธ ์ดํ›„ 2015. 11. 19. ์žฌ๋ด‰์ธํ•œ ๊ฒƒ์„ ํ™•์ธํ•˜์˜€๋‹ค. โ—‹ ์ˆœ๋ฒˆ 753(R09_0001.WAV) : 2015. 11. 22. ๋ด‰์ธ ์ดํ›„ 2015. 11. 22. ์žฌ๋ด‰์ธํ•œ ๊ฒƒ์„ ํ™•์ธํ•˜์˜€๋‹ค. โ—‹ ์ˆœ๋ฒˆ 754(1214_162806.WAV) : 2014. 1. 5. ๋ด‰์ธ ์ดํ›„ 2015. 11. 19. ์žฌ๋ด‰์ธํ•œ ๊ฒƒ์„ ํ™•์ธํ•˜์˜€๋‹ค. โ—‹ ์ˆœ๋ฒˆ 755, 756(0104_161112.WAV, 0104_170156.WAV) : 2014. 1. 4. ๋ด‰์ธ ์ดํ›„ 2015. 11. 19. ์žฌ๋ด‰์ธํ•œ ๊ฒƒ์„ ํ™•์ธํ•˜์˜€๋‹ค. 2) 2016. 9. 6. ๊ฒ€์ฆ๊ธฐ์ผ์— ์•„๋ž˜ ์›๋ณธ ์ €์žฅ๋งค์ฒด์˜ ๋ด‰์ธ์ƒํƒœ๋ฅผ ํ™•์ธํ•˜๊ณ , ๋ด‰์ธ์„ ํ•ด์ œํ•œ ํ›„ ์›๋ณธ ํŒŒ์ผ๊ณผ ๊ฒ€์ฆ์šฉ ์‚ฌ๋ณธ ํŒŒ์ผ์— ๋Œ€ํ•œ ๊ฐ ํ•ด์‹œ๊ฐ’ ํ™•์ธ ๋“ฑ์„ ํ†ตํ•˜์—ฌ ๋™์ผ์„ฑ์—ฌ๋ถ€๋ฅผ ํ™•์ธํ•˜์˜€๋‹ค. ์ดํ›„ ์žฌ์ƒยท์‹œ์ฒญํ•˜๊ณ  ์žฌ๋ด‰์ธํ•˜์˜€๋‹ค. ๋‹จ, ์ˆœ๋ฒˆ 758์€ ์˜ค๋ฅ˜๋กœ ์ธํ•ด ํ•ด์‹œ๊ฐ’์„ ํ™•์ธํ•˜์ง€ ๋ชปํ•˜์˜€๋‹ค. โ—‹ ์ฆ๊ฑฐ๋ชฉ๋ก ์ˆœ๋ฒˆ 757(V001.WAV) : 2015. 11. 18. ๋ด‰์ธ ์ดํ›„ 2016. 9. 6. ์žฌ๋ด‰์ธํ•œ ๊ฒƒ์„ ํ™•์ธํ•˜์˜€๋‹ค. โ—‹ ์ˆœ๋ฒˆ 758(V001.WAV) : 2015. 11. 19. ๋ด‰์ธ ์ดํ›„ 2016. 9. 6. ์žฌ๋ด‰์ธํ•œ ๊ฒƒ์„ ํ™•์ธํ•˜์˜€๋‹ค. โ—‹ ์ˆœ๋ฒˆ 759(V001.WAV) : 2015. 11. 18. ๋ด‰์ธ ์ดํ›„ 2016. 9. 6. ์žฌ๋ด‰์ธํ•œ ๊ฒƒ์„ ํ™•์ธํ•˜์˜€๋‹ค. โ—‹ ์ˆœ๋ฒˆ 760(V001.WAV) : 2015. 11. 12. ๋ด‰์ธ ์ดํ›„ 2016. 9. 6. ์žฌ๋ด‰์ธํ•œ ๊ฒƒ์„ ํ™•์ธํ•˜์˜€๋‹ค. โ—‹ ์ˆœ๋ฒˆ 761(V001.WAV) : 2015. 11. 19. ๋ด‰์ธ ์ดํ›„ 2016. 9. 6. ์žฌ๋ด‰์ธํ•œ ๊ฒƒ์„ ํ™•์ธํ•˜์˜€๋‹ค. โ—‹ ์ˆœ๋ฒˆ 762, 763(RO9_0001.WAV, RO9_0002.WAV) : ๊ฐ 2014. 2. 2. ๋ด‰์ธ ์ดํ›„ 2016. 9. 6. ์žฌ๋ด‰์ธํ•œ ๊ฒƒ์„ ํ™•์ธํ•˜์˜€๋‹ค. 3) 2016. 9. 27. ๊ฒ€์ฆ๊ธฐ์ผ์— ์ฆ๊ฑฐ๋ชฉ๋ก ์ˆœ๋ฒˆ 798๋ฒˆ(์ฆ๊ฑฐ๋ชฉ๋ก ์ˆœ๋ฒˆ 758์„ ๋ณต๊ตฌํ•œ ํŒŒ์ผ)์— ๊ด€ํ•˜์—ฌ ๋™์ผ์„ฑ์„ ์ฃผ19) ํ™•์ธํ•˜๊ณ  ํŒŒ์ผ์„ ์žฌ์ƒ, ์ฒญ์ทจํ•˜์˜€๋‹ค. 2015. 11. 19. ๋ด‰์ธ ๋ฐ 2016. 9. 6. ๋ด‰์ธํ•ด์ œ ๋ฐ ์žฌ๋ด‰์ธ, 2016. 9. 8. ๋ด‰์ธํ•ด์ œ ๋ฐ ์žฌ๋ด‰์ธ ์ƒํƒœ์ธ ๊ฒƒ์„ ํ™•์ธํ•˜์˜€๋‹ค. ใ€ˆ์ฆ์ธ๋“ค์˜ ์ฆ์–ธใ€‰ 1) 2016. 7. 27. ์ฆ์ธ ๊ณต์†Œ์™ธ 47 [2013. 10. 22. ๋…น์Œ(์ฆ๊ฑฐ๋ชฉ๋ก ์ˆœ๋ฒˆ 757), 2013. 11. 4. ๋…น์Œ(์ˆœ๋ฒˆ 758), 2013. 12. 14. ๋…น์Œ(์ˆœ๋ฒˆ 754), 2014. 1. 4. ๋…น์Œ(์ˆœ๋ฒˆ 755, 756), 2014. 2. 2. ๋…น์Œ(์ˆœ๋ฒˆ 762, 763), 2014. 4. 28. ๋…น์Œ(์ˆœ๋ฒˆ 761) ๊ด€๋ จ] : (๋ช…์นญ 29 ์ƒ๋žต)์—ฐ๊ตฌ์†Œ์—์„œ ๋…น์Œํ•  ๋•Œ๋Š” ๋…น์Œ๊ธฐ๋ฅผ 1ํšŒ์”ฉ๋งŒ ์‚ฌ์šฉํ•˜์˜€๋‹ค. ์ด๋•Œ ์‚ฌ์šฉํ•œ ๊ฒƒ์€ โ€˜(๋ช…์นญ 57 ์ƒ๋žต)โ€˜์ด๋ผ๋Š” ๋…น์Œ๊ธฐ์ธ๋ฐ ๋…น์Œ์ด ๋๋‚˜๋ฉด ๋ผ๋ฒจ์„ ๋ถ™์—ฌ์„œ ๊ด€๋ฆฌ๋ฅผ ํ•ด์„œ ์‚ฌ๋ณธ์„ ๋งŒ๋“ ๋‹ค. ๊ทธ ์‚ฌ๋ณธ์„ ๊ฐ€์ง€๊ณ  ๋…น์ทจ๋ก์ด๋‚˜ ๊ธฐํƒ€ ํ™œ์šฉํ•˜๋Š” ๋ฐ ์‚ฌ์šฉํ•˜์˜€๋‹ค. ๋…น์Œ ์ฆ‰์‹œ ์“ฐ๊ธฐ๋ฐฉ์ง€์žฅ์น˜๋ฅผ ์—ฐ๊ฒฐํ•˜์—ฌ ์‚ฌ๋ณธ์„ ๋งŒ๋“ค์–ด ๋…น์ทจ๋ก์„ ๋งŒ๋“ค์—ˆ๊ณ , ์ผ๋ถ€ ๋…น์ŒํŒŒ์ผ์˜ ๊ฒฝ์šฐ์—๋Š” ๋…น์Œ ์ฆ‰์‹œ ์›๋ณธ์„ ๋ด‰์ธํ•œ ํ›„, ์ด๋ฏธ์ง• ์ž‘์—…์„ ํ†ตํ•ด์„œ ๋…น์ทจ๋ก์„ ๋งŒ๋“ค์—ˆ๋‹ค. 2) ๊ณต์†Œ์™ธ 14 [2013. 10. 22. ๋…น์Œ(์ฆ๊ฑฐ๋ชฉ๋ก ์ˆœ๋ฒˆ 757), 2013. 11. 4. ๋…น์Œ(์ˆœ๋ฒˆ 758), 2014. 1. 22. ๋…น์Œ(์ˆœ๋ฒˆ 759), 2014. 2. 3. ๋…น์Œ(์ˆœ๋ฒˆ 760), 2014. 4. 28. ๋…น์Œ(์ˆœ๋ฒˆ 761) ๊ด€๋ จ] : ๋…น์Œ์„ ํ•  ๋•Œ (๋ช…์นญ 57 ์ƒ๋žต)์ด๋ผ๋Š” ๋…น์Œ๊ธฐ๋ฅผ ์‚ฌ์šฉํ•˜์˜€๋‹ค. ํ”ผ๊ณ ์ธ์ด (๋ช…์นญ 29 ์ƒ๋žต)์—ฐ๊ตฌ์†Œ๋ผ๋“ ์ง€ ์ด๋Ÿฐ ๊ณณ์— ๋“ค์–ด๊ฐ”์„ ๋•Œ ์ฆ์ธ์ด ์ง์ ‘ โ—‘์ธต์— ์˜ฌ๋ผ๊ฐ€ ๋…น์Œ๊ธฐ๋ฅผ ์˜ฌ๋ ค๋†“๊ณ  ์™”๋‹ค. ํ˜„์žฅ์—์„œ ๋…น์Œํ•  ๋•Œ ์ฆ์ธ์ด ๋…น์Œ๊ธฐ๋ฅผ ํ‚ค๋Š” ์‹œ๊ฐ„, ๋„๋Š” ์‹œ๊ฐ„์„ ์ •ํ™•ํžˆ ์ ์–ด ๋†“์•˜๋‹ค. ๋…น์Œํ•˜๊ณ  ๋‚˜์„œ ๋…น์Œ๊ธฐ๋ฅผ ํšŒ์ˆ˜ํ•˜๋ฉด ์ฆ‰์‹œ ๋ผ๋ฒจ์„ ๋งŒ๋“ค์–ด์„œ ๋ถ™์˜€๋‹ค. ๋…น์Œ๊ธฐ ์•ˆ์˜ ํŒŒ์ผ๋ช…์€ ์ž๋™์ ์œผ๋กœ ์ƒ์„ฑ๋˜๋Š” ๊ฒƒ์ด๋‹ค. ๋งค๋ฒˆ ์ƒˆ ๋…น์Œ๊ธฐ๋ฅผ ํ•œ ๋ฒˆ๋งŒ ์‚ฌ์šฉํ–ˆ๊ธฐ ๋•Œ๋ฌธ์— ํŒŒ์ผ๋ช…์ด ๋˜‘๊ฐ™์„ ์ˆ˜๋ฐ–์— ์—†๋‹ค. 3) 2016. 8. 29. ์ฆ์ธ ๊ณต์†Œ์™ธ 66 [2015. 3. 8. ๋…น์Œ(์ˆœ๋ฒˆ 751), 2015. 3. 30. ๋…น์Œ(์ˆœ๋ฒˆ 752), 2015. 11. 1. ๋…น์Œ(์ˆœ๋ฒˆ 753)] : ๋…น์Œ ๋ฐฉ๋ฒ•์€ ๋ฌด์„ ํ˜• ์†ก์ˆ˜์‹ ๊ธฐ์— ๋…น์Œ๊ธฐ๋ฅผ ๋ถ€์ฐฉํ•ด์„œ ๋…น์Œํ–ˆ๋‹ค. 2015. 3. 8.์˜ ๊ฒƒ์€ ํฌ๋ Œ์‹ ์ˆ˜์‚ฌ๊ด€์˜ ๋„์›€์„ ๋ฐ›์•„์„œ ์ง์ ‘ ์‚ฌ๋ณธ์„ ์ƒ์„ฑํ•˜๊ณ  ์›๋ณธ์€ ๋ด‰์ธํ–ˆ๋‹ค. 2015. 3. 30.์˜ ๊ฒƒ์€ ์ฆ์ธ์ด ๋ง๋ ˆ์ด์‹œ์•„ ์ถœ์žฅ ์ค‘์ด์–ด์„œ ํฌ๋ Œ์‹ ์ˆ˜์‚ฌ๊ด€์ด ํ•œ ๊ฒƒ์œผ๋กœ ์•Œ๊ณ  ์žˆ๋‹ค. 2015. 11. 1.์˜ ๊ฒƒ์€ ํฌ๋ Œ์‹ ์ˆ˜์‚ฌ๊ด€์˜ ๋„์›€์„ ๋ฐ›์•„์„œ ์ง์ ‘ ์‚ฌ๋ณธ์„ ์ƒ์„ฑํ•˜๊ณ  ์›๋ณธ์€ ๋ด‰์ธํ–ˆ๋‹ค. 4) ๊ณต์†Œ์™ธ 51 : ๊ตญ๋ฆฝ๊ณผํ•™์ˆ˜์‚ฌ์—ฐ๊ตฌ์› ๋””์ง€ํ„ธ๋ถ„์„๊ณผ์—์„œ ์Œ์„ฑ๋ถ„์„, ์Œํ–ฅ๋ถ„์„ ๊ด€๋ จ๋œ ์ผ์„ ํ•˜๊ณ  ์žˆ๊ณ  ์ฆ๊ฑฐ๋ชฉ๋ก ์ˆœ๋ฒˆ 751๋ถ€ํ„ฐ 762๊นŒ์ง€์˜ ํŒŒ์ผ์˜ ์œ„๋ณ€์กฐ ์—ฌ๋ถ€์— ๊ด€ํ•˜์—ฌ ๊ฐ์ •ํ•˜์˜€๋‹ค. ์Œ์„ฑํŒŒ์ผ์„ ๊ฐ์ •ํ•  ๋•Œ ๊ธฐ๋ณธ์ ์œผ๋กœ ์ฃผํŒŒ์ˆ˜๋ถ„์„, ํŒŒํ˜•๋ถ„์„, ์ฒญ๊ฐ์— ์˜ํ•œ ๊ด€๋Šฅ๊ฒ€์‚ฌ๋ฅผ ๋™๋ฐ˜ํ•ด์„œ ํ•œ๋‹ค. ์‚ฌ์šฉ๋˜๋Š” ์žฅ๋น„๋“ค์€ ์ฃผํŒŒ์ˆ˜๋ฅผ ๋ถ„์„ํ•  ์ˆ˜ ์žˆ๋Š” ๋‹ค์–‘ํ•œ ์žฅ๋น„๋“ค, ํŒŒํ˜•์„ ๋ถ„์„ํ•  ์ˆ˜ ์žˆ๋Š” ๋‹ค์–‘ํ•œ ์žฅ๋น„๋“ค์„ ์‚ฌ์šฉํ•˜๊ณ  ์žˆ๋‹ค. ์ „์ฒด ๋…น์Œ๋‚ด์šฉ ์ค‘์—์„œ ํ•œ ๋ฌธ์žฅ์€ ๋ฌผ๋ก ์ด๊ณ  ํ•œ ๋‹จ์–ด ๋˜๋Š” ํ•œ ์Œ์ ˆ์„ ์ธ์œ„์ ์œผ๋กœ ์ถ”๊ฐ€ํ•˜๋”๋ผ๋„ ํ™•์ธํ•  ์ˆ˜ ์žˆ๋‹ค. ํ˜„์žฌ๊นŒ์ง€ ํ™•์ธํ•˜์ง€ ๋ชปํ•œ ์ ์€ ์—†๋‹ค. ์ด ์‚ฌ๊ฑด๊ณผ ๊ด€๋ จํ•˜์—ฌ ์˜๋ขฐ๋ฅผ ๋ฐ›์€ ํŒŒ์ผ์€ ์‚ฌ๋ณธ์ด์—ˆ๋‹ค. ์‚ฌ๋ณธ์ด๋ผ๊ณ  ํ•ด๋„ ์œ„๋ณ€์กฐ๋ฅผ ํŒ๋‹จํ•จ์— ์žˆ์–ด์„œ๋Š” ๋ฌธ์ œ๊ฐ€ ์—†๋‹ค. ๊ฐ์ •์˜๋ขฐ ๋ฐ›์€ ํŒŒ์ผ๋“ค์€ ์–ด๋–ค ํŒŒํ˜•์˜ ๋Š๊น€์ด ์—†์—ˆ๊ณ , ์ฃผํŒŒ์ˆ˜ ๋Œ€์—ญ์ด ๊ณ ๋ฅด๊ฒŒ ๋ถ„ํฌ๋˜์–ด ์žˆ์–ด์„œ ํŽธ์ง‘๋œ ๊ฒƒ์ด๋ผ๊ณ  ๋ณผ ์ˆ˜ ์—†์—ˆ๋‹ค. 5) ๊ณต์†Œ์™ธ 67 : 2016. 9. 8. ๊ตญ์ •์› ์ˆ˜์‚ฌ๊ด€์œผ๋กœ๋ถ€ํ„ฐ (๋ช…์นญ 57 ์ƒ๋žต)ํ…Œํฌ ๋…น์Œ๊ธฐ์˜ ๋ฐ์ดํ„ฐ ๋ณต๊ตฌ๋ฅผ ์˜๋ขฐ๋ฐ›์•„ ๊ฐ™์€ ๋‚  ๋ณต๊ตฌ๋ฅผ ๋งˆ์น˜๊ณ  ์ง์ ‘ ๋ด‰์ธ์„ ํ•˜์—ฌ ์ˆ˜์‚ฌ๊ธฐ๊ด€์— ๋ฐ˜ํ™˜ํ•˜์˜€๋‹ค. ๋ณต๊ตฌ์ž‘์—…์„ ํ•˜๊ธฐ ์ „์— ์“ฐ๊ธฐ๋ณดํ˜ธ์žฅ์น˜๋ฅผ ์ด์šฉํ•ด์„œ ๊ธฐ๋ก์ด ๋  ์ˆ˜ ์—†๋Š” ์ƒํƒœ๋กœ ๋จผ์ € ๋งŒ๋“ค์–ด์„œ ๋ถ„์„์ž‘์—…์„ ํ–ˆ๋‹ค. ๋ด‰์ธ์„ ํ•ด์ œํ•  ๋•Œ๋ถ€ํ„ฐ ๋ณต๊ตฌ๋ฅผ ์™„๋ฃŒํ•˜์—ฌ ์žฌ๋ด‰์ธํ•˜๋Š” ๊ณผ์ •์„ ์˜์ƒ์ดฌ์˜ ํ•˜์˜€๋‹ค. ๋ณด์ด์Šค๋ ˆ์ฝ”๋”๋ฅผ ๋ฐ›์•˜์„ ๋•Œ ๊ทธ ์•ˆ์— ๋ฌธ์„œ ๊ฐ™์€ ๊ฒƒ๋“ค์€ ์ „ํ˜€ ์—†์—ˆ๊ณ , WAV ํŒŒ์ผ์ด๋ผ๊ณ  ํ•˜๋Š” ์Œ์„ฑํŒŒ์ผ ๋”ฑ ํ•˜๋‚˜๋งŒ ํ™•์ธ๋˜์—ˆ๋‹ค. ์œˆํ—ฅ์Šค๋ผ๋Š” ํˆด์„ ์ด์šฉํ•ด ์ผ๋‹จ ์กฐ์‚ฌ๋ฅผ ํ•˜์˜€๊ณ  ์†์ƒ๋œ ํŒŒ์ผ์„ ๋ณต๊ตฌํ•˜์˜€๋‹ค. ๋ณต๊ตฌํ•œ ํŒŒ์ผ๊ณผ ์†์ƒ๋œ ํŒŒ์ผ์€ ๋ฐ์ดํ„ฐ๋ฅผ ์ €์žฅํ•˜๋Š” ๋ฐ ์‚ฌ์šฉํ•œ ํด๋Ÿฌ์Šคํ„ฐ ๋ถ€๋ถ„์ด ๋™์ผํ•˜๋‹ค๋ฉด ํ•ด์‹œ๊ฐ’์ด ๋™์ผํ•  ์ฃผ20) ๊ฒƒ์ด๋‹ค. ๋ผ. ๋…น์ŒํŒŒ์ผ์— ๊ด€ํ•œ ํŒ๋‹จ 1) ์ผ๋‹จ ๋ด‰์ธ์„ ๋œฏ์€ ํ”์ ์ด ์—†๊ณ  ๋ด‰์ธ๋œ ํŒŒ์ผ๋“ค์˜ ํ•ด์‹œ๊ฐ’์ด ๋ด‰์ธํ•˜๋ฉด์„œ ์‚ฐ์ถœํ•ด ๋†“์€ ํ•ด์‹œ๊ฐ’๊ณผ ๋™์ผํ•˜๋ฏ€๋กœ ์œ„ ํŒŒ์ผ๋“ค์ด ๋ด‰์ธํ•œ ์‹œ์ ๋ถ€ํ„ฐ ์ด ๋ฒ•์›์˜ ๊ฒ€์ฆ์‹œ๊นŒ์ง€๋Š” ์œ„์กฐยท๋ณ€์กฐ๊ฐ€ ์—†์—ˆ๋‹ค๊ณ  ์ธ์ •๋œ๋‹ค. 2) ๊ทธ๋Ÿฐ๋ฐ ๋…น์Œํ•œ ๋‚  ๋˜๋Š” ๊ทธ๋กœ๋ถ€ํ„ฐ ํ•˜๋ฃจ, ์ดํ‹€์ด ์ง€๋‚œ ์‹œ์ ์— ๋ด‰์ธ์ด ๋œ ๊ฒƒ(์ฆ๊ฑฐ๋ชฉ๋ก ์ˆœ๋ฒˆ 751, 752, 755, 756, 762, 763) ์™ธ์—๋„, ๋…น์Œํ•œ ๋‚ ๋ถ€ํ„ฐ ๋ด‰์ธํ•œ ๋‚ ๊นŒ์ง€์˜ ๊ธฐ๊ฐ„์ด ์งง์ง€ ์•Š์€ ํŒŒ์ผ๋“ค์ด ์žˆ์œผ๋ฏ€๋กœ ๊ฐ ๋…น์Œ์ผ์‹œ๋ถ€ํ„ฐ ๋ด‰์ธํ•œ ์‹œ์ ๊นŒ์ง€ ๊ทธ ํŒŒ์ผ์˜ ์›๋ณธ์„ฑ ๋ฐ ์œ„ยท๋ณ€์กฐ ์—ฌ๋ถ€๊ฐ€ ๋ฌธ์ œ๋œ๋‹ค. ๊ทธ๋Ÿฌ๋‚˜ โ‘  ์ง์ ‘ ๋…น์Œ์„ ํ•œ ์ฆ์ธ๋“ค์ด ๋…น์Œํ•œ ํ›„์— ์–ด๋– ํ•œ ํŽธ์ง‘๋„ ์—†์—ˆ๋‹ค๊ณ  ์ง„์ˆ ํ•˜๊ณ  ์žˆ๋Š” ์ , โ‘ก ์œ„ ๊ฒ€์ฆ๋Œ€์ƒ๋ฌผ๋“ค์€ ๋…น์Œ๊ธฐ ์ƒํƒœ๋กœ ์ œ์ถœ๋˜์—ˆ๋Š”๋ฐ, ๋…น์Œ๊ธฐ์—์„œ ํŽธ์ง‘์„ ํ•  ๊ธฐ๋Šฅ์€ ์—†๋Š” ์ , โ‘ข ๊ณต์†Œ์™ธ 51์˜ ๊ฐ์ • ๊ฒฐ๊ณผ๋„ ํŽธ์ง‘์˜ ํ”์ ์„ ์ฐพ์„ ์ˆ˜ ์—†๋‹ค๋Š” ๊ฒƒ์ด๊ณ , ๋ฌธ์„œํŒŒ์ผ๊ณผ๋Š” ๋‹ฌ๋ฆฌ ๋…น์ŒํŒŒ์ผ์˜ ๊ฒฝ์šฐ์—๋Š” ๊ทธ ํŠน์„ฑ์ƒ ๋‚ด์šฉ์„ ํŽธ์ง‘ํ•˜๋Š” ๊ฒฝ์šฐ ๊ฐ์ •์— ์˜ํ•ด ์ด๋ฅผ ์‹๋ณ„ํ•  ์ˆ˜ ์žˆ์„ ๊ฒƒ์œผ๋กœ ๋ณด์ด๋Š” ์ (์ด์— ์–ด๊ธ‹๋‚˜๋Š” ์ฆ์ธ ๊ณต์†Œ์™ธ 68์˜ ๋ฒ•์ •์ง„์ˆ ์€ ๋””์ง€ํ„ธ ํฌ๋ Œ์‹๊ณผ ์Œ์„ฑํŒŒ์ผ์˜ ํŠน์„ฑ์— ๊ด€ํ•œ ๊ฐœ์ธ์˜ ํ•™๋ ฅ๊ณผ ๊ฒฝํ—˜, ์ง€์‹์ˆ˜์ค€์— ๊ธฐํ•œ ์ถ”์ธก์„ฑ ์ง„์ˆ ์ด์–ด์„œ ๊ทธ๋Œ€๋กœ ๋ฏฟ๊ธฐ ์–ด๋ ต๋‹ค. ๊ณต์†Œ์™ธ 68์€ ๊ฐ™์€ ์Œ์„ฑ์ด ๋…น์Œ๋˜์–ด ์žˆ๋‹ค๊ณ  ํ•˜๋”๋ผ๋„ ๋””์ง€ํ„ธ ์ฆ๊ฑฐ์™€ ์•„๋‚ ๋กœ๊ทธ ์ฆ๊ฑฐ๋Š” ๋‹ค๋ฅธ ํŠน์„ฑ์„ ๊ฐ€์ง€๊ณ  ๋””์ง€ํ„ธ ์Œ์„ฑ ํŒŒ์ผ์€ ๋ถ€๋ถ„ ๋ถ€๋ถ„์„ ์ž˜๋ผ๋‚ด์„œ ๋‹ค๋ฅธ ์Œ์„ฑ์— ์„ž๋Š” ๋ฐฉ์‹์œผ๋กœ ์œ„ยท๋ณ€์กฐ๋ฅผ ํ•  ์ˆ˜ ์žˆ๋‹ค๋Š” ์ทจ์ง€๋กœ ์ฆ์–ธํ•˜์˜€์œผ๋‚˜, ์—ฐ์†์ ์œผ๋กœ ๋…น์Œ๋˜์–ด ์žˆ๋Š” ์Œ์„ฑํŒŒ์ผ์„ ์œ„ ๋ฐฉ๋ฒ•์œผ๋กœ ์œ„ยท๋ณ€์กฐํ•˜๋Š” ๊ฒƒ์€ ์ƒ๋‹นํžˆ ์–ด๋ ค์šด ๊ฒƒ์œผ๋กœ ๋ณด์ด๊ณ , ๊ณต์†Œ์™ธ 68์ด ๋งํ•œ ๋ฐฉ๋ฒ•์— ์˜ํ•˜๋”๋ผ๋„ ํ”ผ๊ณ ์ธ์ด ๋งํ•˜์ง€ ์•Š์€ ๋‚ด์šฉ์„ ๋‹ค๋ฅธ ํŒŒ์ผ์— ์‚ฝ์ž…ํ•˜์—ฌ ๋ณ€์กฐํ•˜๋Š” ๊ฒƒ์€ ๊ทนํžˆ ์–ด๋ ค์šธ ๊ฒƒ์ด๋‹ค), โ‘ฃ ์ด ๋ฒ•์›์˜ ๊ฒ€์ฆ๊ณผ์ •์—์„œ๋„ ํŒŒ์ผ์ด ์ด์ƒํ•˜๋‹ค๊ฑฐ๋‚˜ ํŽธ์ง‘๋œ ๊ฒƒ์œผ๋กœ ๋ณผ ํ”์ ์ด ์—†์—ˆ๋˜ ์ , โ‘ค ์œ„ ์Œ์„ฑํŒŒ์ผ๋“ค์€ ๊ธธ๊ฒŒ๋Š” ๋‹ค์„ฏ ์‹œ๊ฐ„์„ ๋„˜๋Š” ์‹œ๊ฐ„ ๋™์•ˆ ์—ฐ์†์ ์œผ๋กœ ๋…น์Œ๋œ ๊ฒƒ์ธ๋ฐ ๊ทธ ํŒŒ์ผ ์ค‘๊ฐ„์— ์–ด๋–ค ๋‚ด์šฉ์„ ์‚ฝ์ž…ํ•  ๊ฐ€๋Šฅ์„ฑ์€ ์ ์€ ๊ฒƒ์œผ๋กœ ๋ณด์ด๋Š” ์  ๋“ฑ์„ ์ข…ํ•ฉํ•˜๋ฉด, ์œ„ ํŒŒ์ผ๋“ค์€ ๋…น์Œ ์ดํ›„๋ถ€ํ„ฐ ๋ด‰์ธํ•  ๋•Œ๊นŒ์ง€๋„ ์›๋ณธ๊ณผ ๋™์ผ์„ฑ์„ ์œ ์ง€ํ•˜๊ณ  ์žˆ์—ˆ๋‹ค๊ณ  ์ธ์ •๋œ๋‹ค(์ฆ๊ฑฐ๋ชฉ๋ก ์ˆœ๋ฒˆ 763 ํŒŒ์ผ์€ ๊ณต์†Œ์™ธ 51์˜ ๊ฐ์ • ๋Œ€์ƒ์— ํฌํ•จ๋˜์ง€ ์•Š์•˜์œผ๋‚˜, ์ด๋Š” ์ฆ๊ฑฐ๋Šฅ๋ ฅ์ด ์ธ์ •๋˜๋Š” ์ฆ๊ฑฐ๋ชฉ๋ก ์ˆœ๋ฒˆ 762๋ฒˆ ํŒŒ์ผ๊ณผ ๋™์ผํ•œ ๋‚ ์— ์—ฐ์†์ ์œผ๋กœ ๋…น์Œ๋œ ๊ฒƒ์ด๊ณ  ๊ทธ ํŒŒ์ผ์˜ ํ˜•์‹๋„ ๋™์ผํ•˜๋ฏ€๋กœ ์›๋ณธ๊ณผ ๋™์ผํ•จ์„ ์ธ์ •ํ•  ์ˆ˜ ์žˆ๋‹ค). 3) ์ˆœ๋ฒˆ 758์„ ๋ณต๊ตฌํ•œ ํŒŒ์ผ(์ฆ๊ฑฐ๋ชฉ๋ก ์ˆœ๋ฒˆ 798๋ฒˆ) ์—ญ์‹œ ์ฆ์ธ ๊ณต์†Œ์™ธ 67์˜ ์ง„์ˆ ๊ณผ ์ด ๋ฒ•์›์˜ ๊ฒ€์ฆ๊ฒฐ๊ณผ, ๋ณต๊ตฌํ•œ ํŒŒ์ผ๊ณผ ๊ฒ€์‚ฌ๊ฐ€ ์ œ์ถœํ•œ ์›๋ณธํŒŒ์ผ์˜ ํ•ด์‹œ๊ฐ’์ด ์ผ์น˜ํ•˜๋Š” ์  ๋“ฑ์— ๋น„์ถ”์–ด ๋ณด๋ฉด, ์›๋ณธ๊ณผ ๋™์ผํ•จ์„ ์ธ์ •ํ•  ์ˆ˜ ์žˆ๋‹ค. 4) ์ด ์‚ฌ๊ฑด์—์„œ ๋ฒ”์ฃ„์˜ ์„ฑ๊ฒฉ์ƒ ๊ฐ•์ œ์ˆ˜์‚ฌ์˜ ํ•„์š”์„ฑ์ด ์ธ์ •๋˜๊ณ , ๋ฒ•์›๋„ ๊ทธ ํ•„์š”์„ฑ์„ ์ธ์ •ํ•˜์—ฌ ์˜์žฅ(๋…น์Œยท์ฒญ์ทจํ—ˆ๊ฐ€์„œ)์„ ๋ฐœ๋ถ€ํ•˜์˜€์œผ๋ฉฐ, ์œ„ ๊ฐ•์ œ์ˆ˜์‚ฌ๋Š” ์˜์žฅ์— ๊ธฐ์žฌ๋œ ๋ฒ”์œ„ ๋‚ด์—์„œ ํ–‰ํ•ด์ง„ ๊ฒƒ์ด๋ฏ€๋กœ ๊ทธ ๊ธฐ๊ฐ„์ด ์žฅ๊ธฐ๊ฐ„์ด๋ผ๋Š” ์‚ฌ์‹ค ๋“ฑ์˜ ์‚ฌ์œ ๋งŒ์œผ๋กœ๋Š” ํ”ผ๊ณ ์ธ์˜ ์ธ๊ฒฉ๊ถŒ๊ณผ ์‚ฌ์ƒํ™œ์„ ๊ณผ๋„ํ•˜๊ฒŒ ์นจํ•ดํ•˜์—ฌ ์œ„๋ฒ•ํ•œ ์ˆ˜์‚ฌ๋ผ๊ณ  ๋ณผ ์ˆ˜๋Š” ์—†๋‹ค. 5) ํ”ผ๊ณ ์ธ๊ณผ ๋ณ€ํ˜ธ์ธ๋“ค์˜ ์ด ๋ถ€๋ถ„ ์ฃผ์žฅ์€ ์ด์œ  ์—†๋‹ค. ๋งˆ. ๋…น์ทจ๋ก์— ๊ด€ํ•œ ํŒ๋‹จ 1) ๋…น์ทจ๋ก์€ ๋…น์Œํ…Œ์ดํ”„๋‚˜ ๋…น์ŒํŒŒ์ผ ๋“ฑ์— ์ˆ˜๋ก๋œ ์Œ์„ฑ์ •๋ณด๋ฅผ ๊ทธ ๋‚ด์šฉ ๊ทธ๋Œ€๋กœ ๋ฌธ์ž๋ผ๋Š” ๋‹ค๋ฅธ ํ˜•ํƒœ์˜ ์ธ์‹์ˆ˜๋‹จ์œผ๋กœ ๋ฐ”๊พผ ๊ฒƒ์— ๋ถˆ๊ณผํ•˜๊ณ , ๊ทธ ์ž‘์„ฑ ์ทจ์ง€๋„ ๋…น์Œํ…Œ์ดํ”„๋‚˜ ๋…น์ŒํŒŒ์ผ ๋“ฑ์— ์ˆ˜๋ก๋œ ๋‚ด์šฉ์„ ๊ทธ๋Œ€๋กœ ์ „๋‹ฌํ•˜๋ ค๋Š” ๊ฒƒ์— ๋ถˆ๊ณผํ•˜๋ฏ€๋กœ, ๋…น์ทจ๋ก์˜ ์ฆ๊ฑฐ๋Šฅ๋ ฅ๊ณผ ์ฆ๊ฑฐ๊ฐ€์น˜๋Š” ๊ทธ ์ž‘์„ฑ์˜ ๊ธฐ์ดˆ๊ฐ€ ๋œ ๋…น์Œํ…Œ์ดํ”„๋‚˜ ๋…น์ŒํŒŒ์ผ ๋“ฑ์˜ ์ฆ๊ฑฐ๋Šฅ๋ ฅ ๋ฐ ์ฆ๊ฑฐ๊ฐ€์น˜๋ฅผ ์ดˆ๊ณผํ•  ์ˆ˜๋Š” ์—†๋‹ค. ๋”ฐ๋ผ์„œ ๋…น์ทจ๋ก์˜ ์ฆ๊ฑฐ๋Šฅ๋ ฅ์„ ์ธ์ •ํ•˜๊ธฐ ์œ„ํ•ด์„œ๋Š” ๊ทธ ์ž‘์„ฑ์˜ ๊ธฐ์ดˆ๊ฐ€ ๋œ ๋…น์Œํ…Œ์ดํ”„๋‚˜ ๋…น์ŒํŒŒ์ผ ๋“ฑ์˜ ์ฆ๊ฑฐ๋Šฅ๋ ฅ์ด ์ธ์ •๋˜์–ด์•ผ ํ•˜๊ณ , ๋‚˜์•„๊ฐ€ ๊ทธ ๋…น์ทจ๋ก์˜ ๊ธฐ์žฌ๊ฐ€ ํ•ด๋‹น ๋…น์Œํ…Œ์ดํ”„๋‚˜ ๋…น์ŒํŒŒ์ผ ๋“ฑ์˜ ๋‚ด์šฉ๊ณผ ๋™์ผํ•˜๋‹ค๋Š” ์ ์ด ๋‹ด๋ณด๋˜์–ด์•ผ ํ•œ๋‹ค. 2) ์œ„์—์„œ ๋ณธ ๋ฐ”์™€ ๊ฐ™์ด ๋…น์ŒํŒŒ์ผ๋“ค์— ๋Œ€ํ•œ ์ฆ๊ฑฐ๋Šฅ๋ ฅ์ด ์ธ์ •๋˜๋Š”๋ฐ”(๊ทธ๊ฒƒ์„ ์žฌ์ƒ, ์ฒญ์ทจ์˜ ๋ฐฉ๋ฒ•์œผ๋กœ ์ฆ๊ฑฐ์กฐ์‚ฌํ•œ ๊ฒƒ๋งŒ์œผ๋กœ๋„ ์ถฉ๋ถ„ํ•˜๋‹ค), ์ด ์‚ฌ๊ฑด ๋…น์ทจ๋ก๋“ค์€ ๊ทธ ๋…น์ŒํŒŒ์ผ์˜ ๋‚ด์šฉ์„ ๊ทธ๋Œ€๋กœ ๊ธฐ์žฌํ•˜๊ณ  ์žˆ์„ ๋ฟ์ธ ์ž๋ฃŒ๋“ค๋กœ์„œ(์ •ํ™•ํ•˜๊ฒŒ ๋“ฃ์ง€ ๋ชปํ•œ ๋ถ€๋ถ„์€ โ€˜ยทโ€˜ ๋“ฑ์œผ๋กœ ํ‘œ์‹œํ•˜๊ณ  ์žˆ๋‹ค), ๊ทธ ์ž‘์„ฑ์ž๋“ค์˜ ์ง„์ˆ  ๋“ฑ์— ์˜ํ•ด ์ง„์ •์„ฑ๋ฆฝ์ด ์ธ์ •๋˜๋ฏ€๋กœ ์ด ๋…น์ทจ๋ก๋“ค์˜ ์ฆ๊ฑฐ๋Šฅ๋ ฅ์—๋„ ๋ณ„๋‹ค๋ฅธ ๋ฌธ์ œ๊ฐ€ ์—†๋‹ค. 3) ๋”ฐ๋ผ์„œ ํ”ผ๊ณ ์ธ๊ณผ ๋ณ€ํ˜ธ์ธ๋“ค์˜ ์ด ๋ถ€๋ถ„ ์ฃผ์žฅ์€ ์ด์œ  ์—†๋‹ค. 14. (๋ช…์นญ 3 ์ƒ๋žต) ์ปคํ”ผ์ˆ์—์„œ ์ดฌ์˜ํ•œ ์˜์ƒ๋“ค์ด ์œ„๋ฒ•์ˆ˜์ง‘ ์ฆ๊ฑฐ์ด๋ฏ€๋กœ ์ฆ๊ฑฐ๋Šฅ๋ ฅ์ด ์—†๋‹ค๋Š” ์ฃผ์žฅ ๊ฐ€. ์ฃผ์žฅ 1) 2013. 11. 2. (๋ช…์นญ 3 ์ƒ๋žต) ์ปคํ”ผ์ˆ์—์„œ ํ”ผ๊ณ ์ธ ๋“ฑ์„ ๋ชฐ๋ž˜ ์ดฌ์˜ํ•œ ๊ฒƒ์€ ์˜์žฅ์ด ์—†๊ณ , (๋ช…์นญ 3 ์ƒ๋žต) ์ปคํ”ผ์ˆ ์‚ฌ์žฅ์˜ ๋™์˜๋ฅผ ๋ฐ›์•˜๋‹ค ํ•˜๋”๋ผ๋„ ๊ทธ๋Š” ํ”ผ๊ณ ์ธ์˜ ๊ฐœ์ธ์ •๋ณด๋ฅผ ์ฒ˜๋ฆฌํ•  ๊ถŒํ•œ์ด ์—†์œผ๋ฏ€๋กœ, ๊ฒฐ๊ตญ ์œ„ ์ดฌ์˜์€ ์œ„๋ฒ•ํ•˜๋‹ค. 2) 2013. 12. 14.๊ณผ 2014. 1. 4. ์œ„ ์ปคํ”ผ์ˆ์—์„œ ํ”ผ๊ณ ์ธ์„ ์ดฌ์˜ํ•œ ๊ฒƒ์€ ์˜์žฅ์— ์˜ํ•œ ๊ฒƒ์ด๋ผ๊ณ  ํ•˜๋”๋ผ๋„ ๊ทธ ์˜์žฅ์„ ์ฒญ๊ตฌํ•˜๋ฉด์„œ ์œ„๋ฒ•ํ•˜๊ฒŒ ์ˆ˜์ง‘ํ•œ ์œ„ 2013. 11. 2.์ž ์˜์ƒ์„ ์ฒจ๋ถ€ํ•˜์˜€์œผ๋ฏ€๋กœ ์ด๋Š” ์œ„๋ฒ•ํ•œ ์ฆ๊ฑฐ๋ฅผ ๊ธฐ์ดˆ๋กœ ์ทจ๋“ํ•œ ์ฆ๊ฑฐ์ด๋‹ค. ๋‚˜. ์ฆ๊ฑฐ์— ์˜ํ•˜์—ฌ ์ธ์ •๋˜๋Š” ์‚ฌ์‹ค 1) 2013. 11. 2. ์ดฌ์˜ ๊ตญ์ •์› ์ˆ˜์‚ฌ๊ด€์€ ํ”ผ๊ณ ์ธ๊ณผ ๊ณต์†Œ์™ธ 5์— ๋Œ€ํ•ด์„œ ๋™ํ–ฅ ๋‚ด์‚ฌ๋ฅผ ํ•˜๋Š” ์ค‘์— ํ”ผ๊ณ ์ธ์ด ๊ณต์†Œ์™ธ 5์™€ ํšŒํ•ฉํ•œ๋‹ค๋Š” ์ •ํ™ฉ์„ ํ™•์ธํ•˜์˜€๋‹ค. ์ด์— ์˜์žฅ์„ ๋ฐ›์ง€ ์•Š์€ ์ฑ„ (๋ช…์นญ 3 ์ƒ๋žต) ์ปคํ”ผ์ˆ ์‚ฌ์žฅ ๊ณต์†Œ์™ธ 28์˜ ๋™์˜๋ฅผ ์–ป์–ด ํ”ผ๊ณ ์ธ๊ณผ ๊ณต์†Œ์™ธ 5๊ฐ€ ์•‰์„ ๊ฒƒ ๊ฐ™์€ ๊ณ ๊ฐ์šฉ ํ…Œ์ด๋ธ” ๊ทผ์ฒ˜ ์ฒœ์žฅ ์ชฝ์— ํ”ผ๊ณ ์ธ๊ณผ ๊ณต์†Œ์™ธ 5๊ฐ€ ์•Œ์•„์ฐจ๋ฆฌ์ง€ ๋ชปํ•˜๊ฒŒ๋” ๋ฏธ๋ฆฌ ๋„คํŠธ์›Œํฌ ์นด๋ฉ”๋ผ(CCTV์™€ ์œ ์‚ฌํ•œ ๋ชจ์–‘์œผ๋กœ, ์คŒ ๊ธฐ๋Šฅ์ด ์žˆ๊ณ  ์‹ค์‹œ๊ฐ„์œผ๋กœ ์›๊ฒฉ ์กฐ์ข…์ด ๊ฐ€๋Šฅํ•˜๋‹ค)๋ฅผ ์„ค์น˜ํ•˜์˜€๋‹ค. ๊ทธ ํ›„ ์ด ์นด๋ฉ”๋ผ๋กœ ํ”ผ๊ณ ์ธ๊ณผ ๊ณต์†Œ์™ธ 5๊ฐ€ ํ…Œ์ด๋ธ”์— ๋งˆ์ฃผ ์•‰์•„์„œ ๋Œ€ํ™”ํ•˜๋ฉฐ ํƒœ๋ธ”๋ฆฟ PC๋ฅผ ๋ณด์—ฌ์ฃผ๋Š” ์žฅ๋ฉด, ํƒœ๋ธ”๋ฆฟ PC์˜ ํ™”๋ฉด๋‚ด์šฉ ๋“ฑ์„ ์ดฌ์˜ํ•˜์˜€๋‹ค. 2) 2013. 12. 14. ์ดฌ์˜ 2013. 12. 11. ๋ฒ•์›์œผ๋กœ๋ถ€ํ„ฐ ์••์ˆ˜์ˆ˜์ƒ‰๊ฒ€์ฆ์˜์žฅ์„ ๋ฐœ๋ถ€๋ฐ›์•˜๊ณ , ์˜์žฅ์˜ ์œ ํšจ๊ธฐ๊ฐ„์€ 2014. 1. 4.๊นŒ์ง€, ๊ฒ€์ฆํ•  ์žฅ์†Œ๋Š” ์ˆ˜์› (๋ช…์นญ 58 ์ƒ๋žต)๋นŒ๋”ฉ์— ์žˆ๋Š” โ€˜(๋ช…์นญ 3 ์ƒ๋žต) ์„ธ๋ฅ˜์—ญ์ โ€™, ๊ฒ€์ฆ์˜ ๋Œ€์ƒ ๋ฐ ๋ฐฉ๋ฒ•์€ โ€˜ํ”ผ๊ณ ์ธ, ๊ณต์†Œ์™ธ 5์˜ ํšŒํ•ฉ ์˜ˆ์ • ์žฅ์†Œ์ธ (๋ช…์นญ 3 ์ƒ๋žต) ์„ธ๋ฅ˜์—ญ์  ๋‚ดยท์™ธ์— ์ •์‚ฌ์ง„ ๋ฐ ๋™์˜์ƒ ์ดฌ์˜์žฅ์น˜ ์„ค์น˜, ๋Œ€์ƒ์ž๊ฐ„ ํšŒํ•ฉ ๋ฐ ์ง€๋ น์ˆ˜์ˆ˜ ์žฅ๋ฉด ๋“ฑ ๊ตญ๊ฐ€๋ณด์•ˆ๋ฒ• ์œ„๋ฐ˜ ๋ฒ”์ฃ„์˜ ๋ชจ์˜ ๋‚ด์ง€ ์‹คํ–‰์žฅ๋ฉด ์ดฌ์˜โ€™์ด๋‹ค. ๊ตญ์ •์› ์ˆ˜์‚ฌ๊ด€์€ 2013. 12. 14. ์œ„ (๋ช…์นญ 3 ์ƒ๋žต) ์„ธ๋ฅ˜์—ญ์ ์—์„œ ๊ทธ๊ณณ ์‚ฌ์žฅ ๊ณต์†Œ์™ธ 28์˜ ํ˜‘์กฐ ํ•˜์— ๋ฏธ๋ฆฌ ์„ค์น˜ํ•ด ๋‘” ๋„คํŠธ์›Œํฌ ์นด๋ฉ”๋ผ(AXIS-213-PTZ)๋ฅผ ์ด์šฉํ•˜์—ฌ ์˜์ƒ์„ ์ดฌ์˜ํ•˜์˜€๋‹ค. 3) 2014. 1. 4. ์ดฌ์˜ 2014. 1. 2. ๋ฒ•์›์œผ๋กœ๋ถ€ํ„ฐ ์••์ˆ˜์ˆ˜์ƒ‰๊ฒ€์ฆ์˜์žฅ์„ ๋ฐœ๋ถ€๋ฐ›์•˜๊ณ , ์˜์žฅ์˜ ์œ ํšจ๊ธฐ๊ฐ„์€ 2014. 1. 11.๊นŒ์ง€, ๊ฒ€์ฆํ•  ์žฅ์†Œ๋Š” ์œ„ โ€˜(๋ช…์นญ 3 ์ƒ๋žต) ์„ธ๋ฅ˜์—ญ์ โ€™์ด๋‹ค. ๊ตญ์ •์› ์ˆ˜์‚ฌ๊ด€์€ 2014. 1. 4. ์œ„ (๋ช…์นญ 3 ์ƒ๋žต) ์„ธ๋ฅ˜์—ญ์ ์—์„œ ์œ„ ๊ณต์†Œ์™ธ 28์˜ ํ˜‘์กฐ ํ•˜์— ๋ฏธ๋ฆฌ ์„ค์น˜ํ•ด ๋‘” ๋„คํŠธ์›Œํฌ ์นด๋ฉ”๋ผ(AXIS-213-PTZ)๋ฅผ ์ด์šฉํ•˜์—ฌ ์˜์ƒ์„ ์ดฌ์˜ํ•˜์˜€๋‹ค. ๋‹ค. ํŒ๋‹จ 1) 2013. 11. 2. ์ดฌ์˜ ๊ด€๋ จ ๊ณต์†Œ์™ธ 28์˜ ์ฒ˜๋ถ„์ด ๊ฐœ์ธ์ •๋ณด ์นจํ•ด์ธ์ง€ ์—ฌ๋ถ€ ์šฐ์„ , ํ”ผ๊ณ ์ธ์˜ ๋ชจ์Šต์ด ์ดฌ์˜๋œ ๋„คํŠธ์›Œํฌ ์นด๋ฉ”๋ผ ์˜์ƒ๋ฌผ์€ ํ”ผ๊ณ ์ธ ๊ฐœ์ธ์„ ์•Œ์•„๋ณผ ์ˆ˜ ์žˆ๋Š” ๊ฐœ์ธ์ •๋ณด์— ํ•ด๋‹นํ•˜๊ณ , ๋˜ ํ”ผ๊ณ ์ธ์„ ๋ฌด๋‹จ์œผ๋กœ ์ดฌ์˜ํ•œ ๊ฒƒ์€ ์ผ์ • ๋ถ€๋ถ„ ์ดˆ์ƒ๊ถŒ, ์ธ๊ฒฉ๊ถŒ, ์‚ฌ์ƒํ™œ์˜ ๋น„๋ฐ€, ๊ฐœ์ธ์ •๋ณด ์ž๊ธฐ๊ฒฐ์ •๊ถŒ์— ๋Œ€ํ•œ ์นจํ•ด์— ํ•ด๋‹นํ•  ์†Œ์ง€๊ฐ€ ์žˆ๊ธฐ๋Š” ํ•˜๋‹ค. ๊ทธ๋Ÿฌ๋‚˜ ๊ณต์†Œ์™ธ 28์ด ์ˆ˜์‚ฌ๊ธฐ๊ด€์— ๋™์˜ํ•œ ๋‚ด์šฉ์€ ๊ฐ€๊ฒŒ์— ๋„คํŠธ์›Œํฌ ์นด๋ฉ”๋ผ๋ฅผ ์„ค์น˜ํ•˜๋„๋ก ๋™์˜ํ•œ๋‹ค๋Š” ๊ฒƒ๋ฟ์ด์–ด์„œ ๊ทธ ์นด๋ฉ”๋ผ๊ฐ€ ๊ณต์†Œ์™ธ 28์˜ ์†Œ์œ ๋กœ ๋œ๋‹ค๊ฑฐ๋‚˜ ๊ทธ ์˜์ƒ ๋…นํ™”๋ฌผ์— ๊ด€ํ•˜์—ฌ ๊ณต์†Œ์™ธ 28์ด ์–ด๋– ํ•œ ์ฒ˜๋ถ„๊ถŒ์„ ๊ฐ€์ง€๊ฒŒ ๋˜๋Š” ๊ฒƒ์€ ์•„๋‹ˆ๋‹ค. ๋”ฐ๋ผ์„œ ๊ณต์†Œ์™ธ 28์ด ์ง์ ‘ ํ”ผ๊ณ ์ธ์˜ ๊ฐœ์ธ์ •๋ณด๋ฅผ ์ฒ˜๋ฆฌํ•œ ๊ฒƒ์ด๋ผ๊ณ ๋Š” ๋ณผ ์ˆ˜ ์—†๊ณ , 2013. 11. 2.์ž ์˜์ƒ๋ฌผ์ด ๊ฐœ์ธ์ •๋ณด ๋ณดํ˜ธ๋ฒ•์„ ์œ„๋ฐ˜ํ•˜์˜€๋‹ค๊ณ  ๋ณผ ์ˆ˜๋„ ์—†๋‹ค. 2) 2013. 11. 2. ์ดฌ์˜์ด ์˜์žฅ์ฃผ์˜ ๋“ฑ์— ์œ„๋ฐ˜ํ•˜์—ฌ ์œ„๋ฒ•ํ•œ์ง€ ์—ฌ๋ถ€ ์ˆ˜์‚ฌ, ์ฆ‰ ๋ฒ”์ฃ„ํ˜์˜์˜ ์œ ๋ฌด๋ฅผ ๋ช…๋ฐฑํžˆ ํ•˜์—ฌ ๊ณต์†Œ๋ฅผ ์ œ๊ธฐยท์œ ์ง€ํ•  ๊ฒƒ์ธ๊ฐ€์˜ ์—ฌ๋ถ€๋ฅผ ๊ฒฐ์ •ํ•˜๊ธฐ ์œ„ํ•˜์—ฌ ๋ฒ”์ธ์„ ๋ฐœ๊ฒฌยทํ™•๋ณดํ•˜๊ณ  ์ฆ๊ฑฐ๋ฅผ ์ˆ˜์ง‘ยท๋ณด์ „ํ•˜๋Š” ์ˆ˜์‚ฌ๊ธฐ๊ด€์˜ ํ™œ๋™์€ ์ˆ˜์‚ฌ ๋ชฉ์ ์„ ๋‹ฌ์„ฑํ•จ์— ํ•„์š”ํ•œ ๊ฒฝ์šฐ์— ํ•œํ•˜์—ฌ ์‚ฌํšŒํ†ต๋…์ƒ ์ƒ๋‹นํ•˜๋‹ค๊ณ  ์ธ์ •๋˜๋Š” ๋ฐฉ๋ฒ• ๋“ฑ์— ์˜ํ•˜์—ฌ ์ˆ˜ํ–‰๋˜์–ด์•ผ ํ•œ๋‹ค(๋Œ€๋ฒ•์› 1999. 12. 7. ์„ ๊ณ  98๋„3329 ํŒ๊ฒฐ ๋“ฑ ์ฐธ์กฐ). ์ˆ˜์‚ฌ๊ธฐ๊ด€์ด ์˜์žฅ ์—†์ด ํ”ผ๊ณ ์ธ์„ ๋ชฐ๋ž˜ ์ดฌ์˜ํ•œ ์ฃผ21) ํ–‰์œ„๋ฅผ ์ ๋ฒ•ํ•˜๋‹ค๊ณ  ํ•˜์—ฌ ์ฆ๊ฑฐ๋Šฅ๋ ฅ์„ ์ธ์ •ํ•˜๊ธฐ ์œ„ํ•ด์„œ๋Š” ์•ž์„œ ๋ณธ ๋ฒ•๋ฆฌ(๋Œ€๋ฒ•์› 1999. 9. 3. ์„ ๊ณ  99๋„2317 ํŒ๊ฒฐ ๋“ฑ ์ฐธ์กฐ)์™€ ๊ฐ™์ด ์ฆ๊ฑฐ๋ณด์ „์˜ ํ•„์š”์„ฑ, ๊ธด๊ธ‰์„ฑ ๋ฐ ์ดฌ์˜์˜ ์ƒ๋‹น์„ฑ ๋“ฑ์ด ์žˆ์–ด์•ผ ํ•œ๋‹ค. ๊ทธ๋Ÿฐ๋ฐ ์ด ์‚ฌ๊ฑด ์ฆ๊ฑฐ๋“ค์— ์˜ํ•˜์—ฌ ์ธ์ •ํ•  ์ˆ˜ ์žˆ๋Š” ๋‹ค์Œ๊ณผ ๊ฐ™์€ ์‚ฌ์ •๋“ค์„ ์ข…ํ•ฉํ•˜๋ฉด, ๊ตญ์ •์› ์ˆ˜์‚ฌ๊ด€์ด 2013. 11. 2. ํ”ผ๊ณ ์ธ๊ณผ ๊ทธ์˜ ํƒœ๋ธ”๋ฆฟ PC์˜ ์˜์ƒ์„ ๋ชฐ๋ž˜ ์ดฌ์˜ํ•œ ๊ฒƒ์€ ์œ„ ์š”๊ฑด์„ ๊ฐ–์ถ”์ง€ ๋ชปํ•˜์˜€๊ณ  ๋”ฐ๋ผ์„œ ์ด๋Š” ์ ๋ฒ•์ ˆ์ฐจ ๋ฐ ์˜์žฅ์ฃผ์˜ ์›์น™์„ ์œ„๋ฐ˜ํ•˜์—ฌ ์œ„๋ฒ•ํ•˜๊ฒŒ ์ˆ˜์ง‘ํ•œ ์ฆ๊ฑฐ๋ผ๊ณ  ํŒ๋‹จ๋œ๋‹ค. ๊ฐ€) ํ”ผ๊ณ ์ธ์ด ๋ฐ›๊ณ  ์žˆ์—ˆ๋˜ ๋ฒ”์ฃ„ํ˜์˜๋Š” ๋ฐ˜๊ตญ๊ฐ€๋‹จ์ฒด ๊ตฌ์„ฑ์›๊ณผ ๋น„๋ฐ€๋ฆฌ์— ํšŒํ•ฉํ•˜๊ฑฐ๋‚˜, ์‚ฌ์ƒํ•™์Šต์„ ํ•˜๋ฉด์„œ ๊ทธ ํ™œ๋™์„ ์ฐฌ์–‘ยท๊ณ ๋ฌดํ•œ๋‹ค๋Š” ๊ฒƒ ๋“ฑ์œผ๋กœ ๊ทธ ๋‚ด์šฉ์ด ๊ตญ๊ฐ€์˜ ์กด๋ฆฝยท์•ˆ์ „์ด๋‚˜ ์ž์œ ๋ฏผ์ฃผ์  ๊ธฐ๋ณธ์งˆ์„œ๋ฅผ ์œ„ํƒœ๋กญ๊ฒŒ ํ•  ์ •๋„๋กœ ์ค‘๋Œ€ํ•˜๋‹ค. ๋˜ ๊ทธ์— ๊ด€ํ•œ ์ฆ๊ฑฐ๋ฌผ์„ ์ทจ๋“ํ•˜๊ธฐ ์œ„ํ•ด์„œ๋Š” ํ”ผ๊ณ ์ธ์ด ๊ณต์†Œ์™ธ 5์™€ ๋งŒ๋‚˜์„œ ๋Œ€ํ™”ํ•˜๋ฉฐ ์ œ์‹œํ•˜๋Š” ํƒœ๋ธ”๋ฆฟ PC์˜ ํ™”๋ฉด ๋“ฑ์„ ํ˜„์žฅ์—์„œ ๋ชฐ๋ž˜ ์ดฌ์˜ํ•  ํ•„์š”์„ฑ์ด ์žˆ์—ˆ๋˜ ๊ฒƒ์œผ๋กœ๋Š” ์ธ์ •๋œ๋‹ค. ๋‚˜) ๊ทธ๋Ÿฐ๋ฐ ์ด ์‚ฌ๊ฑด ์ดฌ์˜์€ ํ”ผ๊ณ ์ธ๊ณผ ๊ณต์†Œ์™ธ 5๊ฐ€ ์•Œ์•„์ฑ„์ง€ ๋ชปํ•˜๋„๋ก ๋ฏธ๋ฆฌ ์ปคํ”ผ์ˆ ๋‚ด๋ถ€ ํ…Œ์ด๋ธ” ๊ทผ์ฒ˜ ์ฒœ์žฅ์— ๋ณ„๋„์˜ ํŠน์ˆ˜ํ•œ ๋„คํŠธ์›Œํฌ ์นด๋ฉ”๋ผ ์žฅ๋น„๋ฅผ ์„ค์น˜ํ•˜์—ฌ ์ดฌ์˜ํ•œ ๊ฒƒ์ด๋‹ค. ์ด๋Ÿฌํ•œ ์ดฌ์˜ ๋ฐฉ์‹์€ ์—…์†Œ์˜ ๋‚ดยท์™ธ๋ถ€์—์„œ ์ผ๋ฐ˜์ ์ธ ์นด๋ฉ”๋ผ ์ดฌ์˜๋ฐฉ๋ฒ•์œผ๋กœ ํƒ€์ธ์˜ ๋ชจ์Šต์„ ์ดฌ์˜ํ•˜๋Š” ๊ฒƒ๊ณผ ๋‹ค๋ฅด๋ฏ€๋กœ ๊ทธ ์ž์ฒด๋กœ ์นจํ•ด์˜ ๋ฐฉ๋ฒ•๊ณผ ์ •๋„๊ฐ€ ์ค‘ํ•˜๋‹ค๊ณ  ๋ณด์ธ๋‹ค. ์ผ๋ฐ˜์ ์œผ๋กœ ์ปคํ”ผ์ˆ์— ์žˆ๋Š” ์‚ฌ๋žŒ์œผ๋กœ์„œ๋Š” ๋ˆ„๊ตฐ๊ฐ€๊ฐ€ ํ†ต์ƒ์˜ ์นด๋ฉ”๋ผ๋‚˜ CCTV ๋“ฑ์„ ํ†ตํ•˜์—ฌ์„œ๋ผ๋„ ์ž์‹ ์˜ ๋ชจ์Šต ์ž์ฒด๋ฅผ ๋ณผ ์ˆ˜ ์žˆ๋‹ค๋Š” ์ ์— ๋Œ€ํ•˜์—ฌ๋Š” ์–ด๋Š ์ •๋„ ์˜ˆ์ƒํ•  ์ˆ˜ ์žˆ๋‹ค๊ณ  ํ•  ๊ฒƒ์ด์ง€๋งŒ, ํ…Œ์ด๋ธ” ๊ทผ์ฒ˜์— ํŠน์ˆ˜ํ•œ ๋ชฐ๋ž˜ ์นด๋ฉ”๋ผ๊ฐ€ ๋‹ฌ๋ ค์žˆ๊ณ  ์ด๋ฅผ ํ†ตํ•˜์—ฌ ์ž์‹ ์˜ ์€๋ฐ€ํ•œ ํ–‰๋™๊ณผ ํƒœ๋ธ”๋ฆฟ PC์˜ ํ™”๋ฉด๋‚ด์šฉ๊นŒ์ง€ ์„ธ์„ธํžˆ ๊ด€์ฐฐํ•˜์—ฌ ์ดฌ์˜ํ•  ๊ฒƒ์ด๋ผ๊ณ ๋Š” ์˜ˆ์ธกํ•˜๊ธฐ ์–ด๋ ต๊ธฐ ๋•Œ๋ฌธ์ด๋‹ค. ๋”ฐ๋ผ์„œ ์ด ์‚ฌ๊ฑด์—์„œ ํŠน๋ณ„ํ•œ ๋„คํŠธ์›Œํฌ ์นด๋ฉ”๋ผ ์žฅ๋น„๋ฅผ ๋ฏธ๋ฆฌ ์„ค์น˜ํ•˜์—ฌ ํ”ผ๊ณ ์ธ์„ ๋ชฐ๋ž˜ ์ดฌ์˜ํ•œ ๊ฒƒ์€ ์ผ๋ฐ˜์ ์œผ๋กœ ํ—ˆ์šฉ๋˜๋Š” ์ƒ๋‹นํ•œ ๋ฐฉ๋ฒ•์ด๋ผ๊ณ  ๋ณด๊ธฐ ์–ด๋ ต๋‹ค. ๋‹ค) ์œ„ ์นด๋ฉ”๋ผ๋ฅผ ์ด์šฉํ•˜์—ฌ ์ดฌ์˜ํ•œ ๊ฒƒ์€ ๋‹จ์ˆœํžˆ ํ”ผ๊ณ ์ธ์˜ ์™ธ์–‘๊ณผ ํ–‰ํƒœ๊ฐ€ ์•„๋‹ˆ๊ณ  ํ”ผ๊ณ ์ธ์ด ์†Œ์ง€ํ•˜์—ฌ ๊ณต์†Œ์™ธ 5์—๊ฒŒ ์—ด๋žŒ์‹œํ‚จ ํƒœ๋ธ”๋ฆฟ PC์˜ ํ™”๋ฉด ๋‚ด์šฉ์ธ๋ฐ”, ์ด๋Š” ๊ฐœ์ธ์˜ ๋‚ด๋ฐ€ํ•œ ์˜์—ญ์— ํ•ด๋‹นํ•˜๋Š” ๊ฒƒ์ด๋‹ค. ๋”ฐ๋ผ์„œ ์˜์žฅ์„ ๋ฐ›์ง€ ์•Š์€ ์ฑ„ ํ”ผ๊ณ ์ธ์˜ ํƒœ๋ธ”๋ฆฟ PC์˜ ๋‚ด์šฉ๊นŒ์ง€ ์ดฌ์˜ํ•œ ๊ฒƒ์€ ํ”ผ๊ณ ์ธ์˜ ์‚ฌ์ƒํ™œ์˜ ๋น„๋ฐ€ ๋“ฑ์„ ์ง€๋‚˜์น˜๊ฒŒ ์นจ๋ฒ”ํ•œ ๊ฒƒ์œผ๋กœ์จ ๊ทธ ์ดฌ์˜์˜ ๋Œ€์ƒ๊ณผ ๋‚ด์šฉ์ƒ์œผ๋กœ๋„ ์ƒ๋‹น์„ฑ์ด ์ธ์ •๋œ๋‹ค๊ณ  ๋ณด๊ธฐ ์–ด๋ ต๋‹ค. ๋ผ) 2013. 11. 2. ๋…นํ™” ์˜์ƒ์€ ๊ตญ์ •์› ์ˆ˜์‚ฌ๊ด€ ๊ณต์†Œ์™ธ 18์ด ์ดฌ์˜ํ•œ ๊ฒƒ์ด๋‹ค. ๊ณต์†Œ์™ธ 18์€ ์ด ๋ฒ•์›์—์„œ โ€˜ํ”ผ๊ณ ์ธ๊ณผ ๊ณต์†Œ์™ธ 5์— ๋Œ€ํ•˜์—ฌ ๋™ํ–ฅ ๋‚ด์‚ฌ๋ฅผ ํ•˜๋Š” ๋„์ค‘์— ํšŒํ•ฉํ•œ๋‹ค๋Š” ์ •ํ™ฉ์„ ๊ธด๊ธ‰ํ•˜๊ฒŒ ํ™•์ธํ•˜์—ฌ ์˜์žฅ์„ ๋ฐ›์ง€ ์•Š์€ ์ฑ„ ๊ณต์†Œ์™ธ 28์˜ ๋™์˜๋ฅผ ์–ป์–ด ๋„คํŠธ์›Œํฌ ์นด๋ฉ”๋ผ๋ฅผ ์„ค์น˜ํ•˜์˜€๋‹คโ€™๋ผ๊ณ  ์ง„์ˆ ํ•˜์˜€์œผ๋‚˜, ์ดํ›„์˜ ์ดฌ์˜ ๋•Œ๋Š” ๋ฒ•์›์œผ๋กœ๋ถ€ํ„ฐ ์˜์žฅ์„ ๋ฐ›์•„์„œ ์ดฌ์˜ํ•˜์˜€๋˜ ์ ์— ๋น„์ถ”์–ด ๋ณด๋ฉด, ์ด ๋‹น์‹œ์—๋„ ๊ทธ ํ•„์š”์„ฑ์„ ์†Œ๋ช…ํ•˜๊ณ  ์˜์žฅ์„ ๋ฐ›์•„์„œ ์ดฌ์˜ํ•  ์ˆ˜ ์žˆ์—ˆ์„ ๊ฒƒ์œผ๋กœ ๋ณด์ธ๋‹ค. ์„ค์‚ฌ ๋‹น์‹œ ๊ธด๊ธ‰์„ ์š”ํ•˜์—ฌ ์˜์žฅ์„ ๋ฐ›์„ ์ˆ˜ ์—†์—ˆ๋”๋ผ๋„ ์ผ๋‹จ ์œ„์™€ ๊ฐ™์ด ์ดฌ์˜์„ ํ•œ ํ›„ ์ด๋ฅผ ์œ ์ฃ„์˜ ์ฆ๊ฑฐ๋กœ ์‚ฌ์šฉํ•˜๋ ค๋ฉด ์‚ฌํ›„์— ์ง€์ฒด ์—†์ด ์˜์žฅ์„ ๋ฐ›์•˜์–ด์•ผ ํ•จ์—๋„(ํ˜•์‚ฌ์†Œ์†ก๋ฒ• ์ œ216์กฐ ์ œ3ํ•ญ ์ ์šฉ ๋˜๋Š” ์œ ์ถ”์ ์šฉ), ๊ทธ๋Ÿฌํ•œ ์กฐ์น˜๋ฅผ ์ทจํ•˜์ง€ ์ฃผ22) ์•Š์•˜๋‹ค. 3) 2013. 12. 14.๊ณผ 2014. 1. 4. ์ดฌ์˜์˜ ๊ฒฝ์šฐ 2013. 12. 14. ๋ฐ 2014. 1. 4. ์ˆ˜์‚ฌ๊ด€๋“ค์€ ๋ณ„๋„๋กœ ๋ฒ•์›์œผ๋กœ๋ถ€ํ„ฐ ์ ๋ฒ•ํ•œ ์˜์žฅ์„ ๋ฐœ๋ถ€๋ฐ›์•„ ๊ทธ ์˜์žฅ์— ๊ธฐ์žฌ๋œ ๋ฐฉ๋ฒ•์œผ๋กœ ํ”ผ๊ณ ์ธ์˜ ํƒœ๋ธ”๋ฆฟ PC๋ฅผ ์ดฌ์˜ํ•˜์˜€๋Š”๋ฐ, ์ด๋•Œ ์•ž์˜ 2013. 11. 2. ์ดฌ์˜์œผ๋กœ ์ธํ•ด ์ทจ๋“ํ•œ ์ •๋ณด๋งŒ์ด ์˜์žฅ ๋ฐœ๋ถ€์˜ ๊ทผ๊ฑฐ๊ฐ€ ๋œ ๊ฒƒ๋„ ์•„๋‹ˆ๋‹ค. ์ˆ˜์‚ฌ๊ด€๋“ค์€ ๋ณ„๋„์˜ ์ˆ˜์‚ฌ์ƒ ํ•„์š”์„ฑ์— ๋”ฐ๋ผ ๋ฒ•๊ด€์˜ ์˜์žฅ์— ๊ทผ๊ฑฐํ•˜์—ฌ ํ”ผ๊ณ ์ธ์˜ ๋ชจ์Šต์„ ์ดฌ์˜ํ•œ ๊ฒƒ์ด๋ฏ€๋กœ ์ด๋ฅผ ์œ„๋ฒ•ํ•˜๊ฒŒ ์ˆ˜์ง‘ํ•œ ์ฆ๊ฑฐ๋ผ๊ณ  ํ•  ์ˆ˜๋Š” ์—†๋‹ค. ๋”ฐ๋ผ์„œ ๋น„๋ก ์ˆ˜์‚ฌ๊ธฐ๊ด€์ด 2013. 11. 2. ํ”ผ๊ณ ์ธ์„ ์ดฌ์˜ํ•œ ๊ฒƒ์ด ๊ทธ์˜ ๊ฐœ์ธ์ •๋ณด๋‚˜ ์‚ฌ์ƒํ™œ์˜ ๋น„๋ฐ€ ๋“ฑ์„ ์นจํ•ดํ•œ ์œ„๋ฒ•ํ•œ ๊ฒƒ์ด๊ณ , ๊ทธ๊ฒƒ์ด ์ดํ›„์˜ ์ˆ˜์‚ฌ์— ์ผ๋ถ€ ์˜ํ–ฅ์„ ๋ฏธ์ณค๋‹ค ํ•˜๋”๋ผ๋„, 2013. 12. 14. ๋ฐ 2014. 1. 4. ์ดฌ์˜์€ ์•ž์„  ์ดฌ์˜ํ–‰์œ„์˜ ์œ„๋ฒ•ํ–‰์œ„์™€ ์ธ๊ณผ๊ด€๊ณ„๊ฐ€ ๋‹จ์ ˆ ๋˜๋Š” ํฌ์„๋˜์—ˆ๋‹ค๊ณ  ๋ณด์ง€ ์•Š์„ ์ˆ˜ ์—†๋‹ค. 4) ํ”ผ๊ณ ์ธ ๋ฐ ๋ณ€ํ˜ธ์ธ๋“ค์˜ ์ด ๋ถ€๋ถ„ ์ฃผ์žฅ ์ค‘ 2013. 11. 2. ์ดฌ์˜์— ๊ด€ํ•œ ์ฃผ์žฅ์€ ์ด์œ  ์žˆ๊ณ , 2013. 12. 14. ๋ฐ 2014. 1. 4. ์ดฌ์˜์— ๊ด€ํ•œ ์ฃผ์žฅ์€ ์ด์œ  ์—†๋‹ค. 15. PC๋ฐฉ์—์„œ ์ž„์˜์ œ์ถœ ๋ฐ›์€ CCTV ์˜์ƒ๊ณผ PC ์‚ฌ์šฉ์ •๋ณด๊ฐ€ ์œ„๋ฒ•์ˆ˜์ง‘์ฆ๊ฑฐ๋ผ๋Š” ์ฃผ์žฅ ๊ฐ€. ์ฃผ์žฅ 1) (๋ช…์นญ 4 ์ƒ๋žต) PC๋ฐฉ ์‚ฌ์žฅ ๊ณต์†Œ์™ธ 19๋กœ๋ถ€ํ„ฐ ์ž„์˜์ œ์ถœ ๋ฐ›์€ CCTV ์˜์ƒ๋…นํ™”๋ฌผ ๋“ฑ์€ ๊ณต์†Œ์™ธ 19์˜ ์ž์˜์— ์˜ํ•˜์ง€ ์•Š์€ ๊ฒƒ์ด๊ณ , ์žฅ๊ธฐ๊ฐ„ ์ˆ˜์‚ฌ๊ธฐ๊ด€์˜ ์••๋ฐ•์— ์˜ํ•œ ๊ฒƒ์ด๋ฏ€๋กœ ์ง„์ •ํ•œ ์ž„์˜์ œ์ถœ์ด๋ผ๊ณ  ํ•  ์ˆ˜ ์—†๋‹ค. 2) CCTV ์˜์ƒ๋…นํ™”๋ฌผ๊ณผ PC ์‚ฌ์šฉ์ •๋ณด๋Š” PC๋ฐฉ ์‚ฌ์žฅ์ด ์ž„์˜๋กœ ์ œ์ถœํ•  ์ˆ˜ ์—†๋Š” ํ”ผ๊ณ ์ธ์˜ ๊ฐœ์ธ์ •๋ณด์ด๋‹ค. ์ด๋Ÿฌํ•œ ๊ฐœ์ธ์ •๋ณด๋ฅผ ์ทจ๋“ํ•˜๊ธฐ ์œ„ํ•ด์„œ๋Š” ํ”ผ๊ณ ์ธ์œผ๋กœ๋ถ€ํ„ฐ ์ž„์˜์ œ์ถœ์„ ๋ฐ›๊ฑฐ๋‚˜ ํ”ผ๊ณ ์ธ์˜ ๋™์˜๋ฅผ ์–ป์–ด์•ผ ํ•˜๋ฉฐ, ๋‹น์‚ฌ์ž์˜ ๋™์˜๊ฐ€ ์—†๋Š” ๊ฒฝ์šฐ์—๋Š” ์••์ˆ˜์ˆ˜์ƒ‰๊ฒ€์ฆ์˜์žฅ์„ ๋ฐ›์•„์„œ ์ทจ๋“ํ•ด์•ผ ํ•œ๋‹ค. ๋‚˜. ํŒ๋‹จ 1) ๊ณต์†Œ์™ธ 19์˜ ์ž์˜์— ์˜ํ•œ ์ž„์˜์ œ์ถœ ์—ฌ๋ถ€ ํ˜•์‚ฌ์†Œ์†ก๋ฒ• ์ œ218์กฐ(์˜์žฅ์— ์˜ํ•˜์ง€ ์•„๋‹ˆํ•œ ์••์ˆ˜)๋Š” โ€˜๊ฒ€์‚ฌ, ์‚ฌ๋ฒ•๊ฒฝ์ฐฐ๊ด€์€ ํ”ผ์˜์ž ๊ธฐํƒ€์ธ์˜ ์œ ๋ฅ˜ํ•œ ๋ฌผ๊ฑด์ด๋‚˜ ์†Œ์œ ์ž, ์†Œ์ง€์ž ๋˜๋Š” ๋ณด๊ด€์ž๊ฐ€ ์ž„์˜๋กœ ์ œ์ถœํ•œ ๋ฌผ๊ฑด์„ ์˜์žฅ์—†์ด ์••์ˆ˜ํ•  ์ˆ˜ ์žˆ๋‹คโ€™๊ณ  ๊ทœ์ •ํ•˜๊ณ  ์žˆ์œผ๋ฏ€๋กœ ๊ตญ์ •์› ์ˆ˜์‚ฌ๊ด€์€ ๊ณต์†Œ์™ธ 19๊ฐ€ ์ž„์˜๋กœ ์ œ์ถœํ•˜๋Š” ๋ฌผ๊ฑด์€ ์••์ˆ˜ํ•  ์ˆ˜ ์žˆ๋‹ค. ๊ณต์†Œ์™ธ 19๋Š” ์ด ๋ฒ•์ •์—์„œ โ€˜์••์ˆ˜์กฐ์„œ, ํ•ด์‹œ๊ฐ’ํ™•์ธ์„œ, ์ž„์˜์ œ์ถœ ๋™์˜์„œ, ํ™•์ธ์„œ ๋“ฑโ€™์— ๊ด€ํ•˜์—ฌ ์ง์ ‘ ์žํ•„๋กœ ์ž‘์„ฑํ•˜๊ณ  ์„œ๋ช…, ๋‚ ์ธํ•œ ๊ฒƒ์ด๋‹ค. ์ž„์˜์ œ์ถœ ๊ณผ์ •์—์„œ ๊ตญ์ •์› ์ˆ˜์‚ฌ๊ด€์œผ๋กœ๋ถ€ํ„ฐ ํ•ด๋‹น ์ž๋ฃŒ์˜ ์••์ˆ˜ํ•„์š”์„ฑ ๋ฐ ๊ด€๋ จ์ ˆ์ฐจ์— ๋Œ€ํ•ด ์„ค๋ช…์„ ๋“ฃ๊ณ  ๋‚ด์šฉ์„ ์ดํ•ดํ•˜๊ณ  ์ž„์˜๋กœ ์ œ์ถœํ•˜์˜€๋‹ค. 2๋…„์— ๊ฑธ์ณ ๊ฝค ์—ฌ๋Ÿฌ ์ฐจ๋ก€ ํ•˜๋“œ๋””์Šคํฌ์™€ CCTV ์˜์ƒ์„ ์ œ์ถœํ•œ ๊ฒƒ์€ ๋ชจ๋‘ ์ž์˜๋กœ, ์ œํ•œ ์—†์ด ์ œ์ถœํ•œ ๊ฒƒ์ด๋‹คโ€˜๋ผ๋Š” ์ทจ์ง€๋กœ ์ง„์ˆ ํ•˜๊ณ  ์žˆ์œผ๋ฏ€๋กœ ๊ณต์†Œ์™ธ 19๊ฐ€ ์ž„์˜์ œ์ถœํ•œ ๊ฒƒ์€ ํ™•์‹คํ•˜๋‹ค. 2) CCTV ์˜์ƒ๋…นํ™”๋ฌผ๊ณผ PC ์‚ฌ์šฉ์ •๋ณด์˜ ์ž„์˜์ œ์ถœ์— ์˜ํ•œ ์••์ˆ˜๊ฐ€ ์œ„๋ฒ•ํ•œ์ง€ ๊ฐ€) ๊ตญ๋ฏผ์˜ ์ธ๊ฐ„์œผ๋กœ์„œ์˜ ์กด์—„๊ณผ ๊ฐ€์น˜๋ฅผ ๋ณด์žฅํ•˜๋Š” ๊ฒƒ์€ ๊ตญ๊ฐ€๊ธฐ๊ด€์˜ ๊ธฐ๋ณธ์ ์ธ ์˜๋ฌด์— ์†ํ•˜๋Š” ๊ฒƒ์ด๊ณ  ์ด๋Š” ํ˜•์‚ฌ์ ˆ์ฐจ์—์„œ๋„ ๋‹น์—ฐํžˆ ๊ตฌํ˜„๋˜์–ด์•ผ ํ•˜๋Š” ๊ฒƒ์ด์ง€๋งŒ, ๊ตญ๋ฏผ์˜ ์‚ฌ์ƒํ™œ ์˜์—ญ์— ๊ด€๊ณ„๋œ ๋ชจ๋“  ์ฆ๊ฑฐ์˜ ์ œ์ถœ์ด ๊ณง๋ฐ”๋กœ ๊ธˆ์ง€๋˜๋Š” ๊ฒƒ์œผ๋กœ ๋ณผ ์ˆ˜๋Š” ์—†์œผ๋ฏ€๋กœ ๋ฒ•์›์œผ๋กœ์„œ๋Š” ํšจ๊ณผ์ ์ธ ํ˜•์‚ฌ์†Œ์ถ” ๋ฐ ํ˜•์‚ฌ์†Œ์†ก์—์„œ์˜ ์ง„์‹ค๋ฐœ๊ฒฌ์ด๋ผ๋Š” ๊ณต์ต๊ณผ ๊ฐœ์ธ์˜ ์ธ๊ฒฉ์  ์ด์ต ๋“ฑ์˜ ๋ณดํ˜ธ์ด์ต์„ ๋น„๊ตํ˜•๋Ÿ‰ํ•˜์—ฌ ๊ทธ ํ—ˆ์šฉ์—ฌ๋ถ€๋ฅผ ๊ฒฐ์ •ํ•˜์—ฌ์•ผ ํ•œ๋‹ค(๋Œ€๋ฒ•์› 1997. 9. 30. ์„ ๊ณ  97๋„1230 ํŒ๊ฒฐ, ๋Œ€๋ฒ•์› 2010. 9. 9. ์„ ๊ณ  2008๋„3990 ํŒ๊ฒฐ ๋“ฑ ์ฐธ์กฐ). ์ด๋•Œ ๋ฒ•์›์ด ๊ทธ ๋น„๊ตํ˜•๋Ÿ‰์„ ํ•จ์— ์žˆ์–ด์„œ๋Š” ์ฆ๊ฑฐ์ˆ˜์ง‘ ์ ˆ์ฐจ์™€ ๊ด€๋ จ๋œ ๋ชจ๋“  ์‚ฌ์ •, ์ฆ‰ ์‚ฌ์ƒํ™œ ๋‚ด์ง€ ์ธ๊ฒฉ์  ์ด์ต์„ ๋ณดํ˜ธํ•˜์—ฌ์•ผ ํ•  ํ•„์š”์„ฑ ์—ฌ๋ถ€ ๋ฐ ๊ทธ ์ •๋„, ์ฆ๊ฑฐ์ˆ˜์ง‘ ๊ณผ์ •์—์„œ ์‚ฌ์ƒํ™œ ๊ธฐํƒ€ ์ธ๊ฒฉ์  ์ด์ต์„ ์นจํ•ดํ•˜๊ฒŒ ๋œ ๊ฒฝ์œ„์™€ ๊ทธ ์นจํ•ด์˜ ๋‚ด์šฉ ๋ฐ ์ •๋„, ํ˜•์‚ฌ์†Œ์ถ”์˜ ๋Œ€์ƒ์ด ๋˜๋Š” ๋ฒ”์ฃ„์˜ ๊ฒฝ์ค‘ ๋ฐ ์„ฑ๊ฒฉ, ํ”ผ๊ณ ์ธ์˜ ์ฆ๊ฑฐ๋™์˜ ์—ฌ๋ถ€ ๋“ฑ์„ ์ „์ฒด์ ยท์ข…ํ•ฉ์ ์œผ๋กœ ๊ณ ๋ คํ•˜์—ฌ์•ผ ํ•˜๊ณ , ๋‹จ์ง€ ํ˜•์‚ฌ์†Œ์ถ”์— ํ•„์š”ํ•œ ์ฆ๊ฑฐ๋ผ๋Š” ์‚ฌ์ •๋งŒ์„ ๋“ค์–ด ๊ณง๋ฐ”๋กœ ํ˜•์‚ฌ์†Œ์†ก์—์„œ์˜ ์ง„์‹ค๋ฐœ๊ฒฌ์ด๋ผ๋Š” ๊ณต์ต์ด ๊ฐœ์ธ์˜ ์ธ๊ฒฉ์  ์ด์ต ๋“ฑ์˜ ๋ณดํ˜ธ์ด์ต๋ณด๋‹ค ์šฐ์›”ํ•œ ๊ฒƒ์œผ๋กœ ์„ฃ๋ถˆ๋ฆฌ ๋‹จ์ •ํ•˜์—ฌ์„œ๋Š” ์•„๋‹ˆ ๋œ๋‹ค(๋Œ€๋ฒ•์› 2013. 11. 28. ์„ ๊ณ  2010๋„12244 ํŒ๊ฒฐ ๋“ฑ ์ฐธ์กฐ). ๊ทธ๋Ÿฐ๋ฐ ์ด ์‚ฌ๊ฑด์—์„œ ๊ณต์†Œ์™ธ 19๊ฐ€ ์ž์‹ ์ด ์†Œ์œ ยท๊ด€๋ฆฌํ•˜๋Š” CCTV ์˜์ƒ๋…นํ™”๋ฌผ๊ณผ PC ์‚ฌ์šฉ์ •๋ณด๋ฅผ ์ž„์˜์ œ์ถœํ•˜์—ฌ ์ˆ˜์‚ฌ๊ธฐ๊ด€์ด ์˜์žฅ ์—†์ด ์••์ˆ˜ํ•˜์˜€๋‹ค๋Š” ํ˜•์‚ฌ์†Œ์†ก๋ฒ• ์ œ218์กฐ์— ์˜ํ•œ ์ ˆ์ฐจ ๊ทธ ์ž์ฒด๋กœ๋Š” ๋ฌธ์ œ๊ฐ€ ์—†์œผ๋‚˜, ์œ„ ์ œ์ถœ๋œ ์ž๋ฃŒ์˜ ๋‚ด์šฉ๊ณผ ๋ฐฉ๋ฒ•์— ๋น„์ถ”์–ด ํ”ผ๊ณ ์ธ์˜ ์ธ๊ฒฉ๊ถŒ ๋‚ด์ง€ ์‚ฌ์ƒํ™œ์˜ ๋น„๋ฐ€์— ๋Œ€ํ•œ ์นจํ•ด, ํŠนํžˆ ๊ทธ ํ–‰์œ„๊ฐ€ ๊ฐœ์ธ์ •๋ณด์˜ ์œ ์ถœํ–‰์œ„๋กœ์„œ ๊ฐœ์ธ์ •๋ณด ๋ณดํ˜ธ๋ฒ• ์œ„๋ฐ˜์ด ๋˜๋Š” ๊ฒƒ์€ ์•„๋‹Œ์ง€๊ฐ€ ๋ฌธ์ œ๋˜๋ฏ€๋กœ ์ด์— ๋Œ€ํ•ด ์‚ดํ•€๋‹ค. ๋‚˜) ํ”ผ๊ณ ์ธ์˜ ๊ฐœ์ธ์ •๋ณด์ธ์ง€ ๊ฐœ์ธ์ •๋ณด ๋ณดํ˜ธ๋ฒ• ์ œ2์กฐ ์ œ1ํ˜ธ๋Š” โ€œ๊ฐœ์ธ์ •๋ณดโ€๋ž€ ์‚ด์•„ ์žˆ๋Š” ๊ฐœ์ธ์— ๊ด€ํ•œ ์ •๋ณด๋กœ์„œ ์„ฑ๋ช…, ์ฃผ๋ฏผ๋“ฑ๋ก๋ฒˆํ˜ธ ๋ฐ ์˜์ƒ ๋“ฑ์„ ํ†ตํ•˜์—ฌ ๊ฐœ์ธ์„ ์•Œ์•„๋ณผ ์ˆ˜ ์žˆ๋Š” ์ •๋ณด ๋˜๋Š” ํ•ด๋‹น ์ •๋ณด๋งŒ์œผ๋กœ๋Š” ํŠน์ • ๊ฐœ์ธ์„ ์•Œ์•„๋ณผ ์ˆ˜ ์—†๋”๋ผ๋„ ๋‹ค๋ฅธ ์ •๋ณด์™€ ์‰ฝ๊ฒŒ ๊ฒฐํ•ฉํ•˜์—ฌ ์•Œ์•„๋ณผ ์ˆ˜ ์žˆ๋Š” ์ •๋ณด๋ฅผ ๋งํ•œ๋‹ค๊ณ  ์ •ํ•˜๊ณ  ์žˆ์–ด ๊ทธ ์š”๊ฑด์œผ๋กœ ๋น„๊ณต๊ฐœ์„ฑ์„ ์š”ํ•˜์ง€ ์•„๋‹ˆํ•˜๊ณ  ์žˆ์œผ๋ฏ€๋กœ ๊ฒฐ๊ตญ ๊ฐœ์ธ์ •๋ณด ๋ณดํ˜ธ๋ฒ•์ƒ์˜ ๊ฐœ์ธ์ •๋ณด์— ํ•ด๋‹นํ•˜๋Š”์ง€ ์—ฌ๋ถ€๋Š” ํ•ด๋‹น ์ •๋ณด๊ฐ€ ๊ณต๊ฐœ๋œ ๊ฒƒ์ธ์ง€ ์—ฌ๋ถ€์™€๋Š” ๋ฌด๊ด€ํ•˜๊ฒŒ ํ•ด๋‹น ์ •๋ณด๊ฐ€ ํŠน์ • ๊ฐœ์ธ์— ๊ด€ํ•œ ์ •๋ณด๋กœ์„œ ๊ทธ ๊ฐœ์ธ์„ ์‹๋ณ„ํ•  ์ˆ˜ ์žˆ๋Š” ์ •๋ณด์— ํ•ด๋‹นํ•˜๋Š”์ง€ ์—ฌ๋ถ€์— ์˜ํ•˜์—ฌ ํŒ๋‹จํ•˜์—ฌ์•ผ ํ•˜๋Š” ๊ฒƒ์ด๋ฉฐ, ๊ทธ ํŒ๋‹จ์—๋Š” ์‚ฌ์ƒํ™œ์˜ ๋น„๋ฐ€์˜ ์ž์œ ์— ๋Œ€ํ•œ ๋ถ€๋‹นํ•œ ์นจํ•ด๋ฅผ ๋ฐฉ์ง€ํ•˜๊ณ  ๊ตญ๋ฏผ์˜ ๊ถŒ๋ฆฌ๋ฅผ ๋ณด์žฅํ•˜๊ณ ์ž ํ•˜๋Š” ๊ฐœ์ธ์ •๋ณด ๋ณดํ˜ธ๋ฒ•์˜ ์ž…๋ฒ•์ทจ์ง€ ๋˜ํ•œ ๊ณ ๋ ค๋˜์–ด์•ผ ํ•  ๊ฒƒ์ด๋‹ค. ์ด๋•Œ โ€˜๋‹ค๋ฅธ ์ •๋ณด์™€ ์‰ฝ๊ฒŒ ๊ฒฐํ•ฉํ•˜์—ฌ ์•Œ์•„๋ณผ ์ˆ˜ ์žˆ๋‹คโ€™๋Š” ๊ฒƒ์€ ๊ฐœ์ธ์ •๋ณด์ฒ˜๋ฆฌ์ž ๋˜๋Š” ์ž„์˜์˜ ๋‹ค๋ฅธ ์‚ฌ๋žŒ ๋“ฑ์ด ์ด๋ฏธ ๋ณด์œ ํ•˜๊ณ  ์žˆ๊ฑฐ๋‚˜ ์‰ฝ๊ฒŒ ์–ป์„ ์ˆ˜ ์žˆ๋Š” ๋‹ค๋ฅธ ์ •๋ณด๋ฅผ ๋ฐ”ํƒ•์œผ๋กœ ํ•ด๋‹น ์ •๋ณด์™€ ๋‹ค๋ฅธ ์ •๋ณด๋ฅผ ํŠน๋ณ„ํ•œ ์–ด๋ ค์›€ ์—†์ด ๊ฒฐํ•ฉํ•˜์—ฌ ํŠน์ • ๊ฐœ์ธ์„ ์•Œ์•„๋ณผ ์ˆ˜ ์žˆ๊ฒŒ ๋˜๋Š” ๊ฒƒ์„ ๋งํ•˜๋Š” ๊ฒƒ์œผ๋กœ์„œ, ๋‹ค๋ฅธ ์ •๋ณด์˜ ์ทจ๋“ ๋ฐ ํ•ด๋‹น ์ •๋ณด์™€์˜ ๊ฒฐํ•ฉ์„ ํ†ตํ•œ ํŠน์ • ๊ฐœ์ธ์˜ ์‹๋ณ„์ด ๋ชจ๋‘ ์‰ฝ๊ฒŒ ์ด๋ฃจ์–ด์ ธ์•ผ ํ•˜๋Š” ๊ฒƒ์„ ์˜๋ฏธํ•œ๋‹ค๊ณ  ๋ณผ ๊ฒƒ์ด๋‹ค. ๋‹ค๋ฅธ ์ •๋ณด์™€ ์‰ฝ๊ฒŒ ๊ฒฐํ•ฉํ•˜์—ฌ ํŠน์ • ๊ฐœ์ธ์„ ์•Œ์•„๋ณผ ์ˆ˜ ์žˆ๋Š” ์ •๋ณด์ธ์ง€๋ฅผ ํŒ๋‹จํ•˜๊ธฐ ์œ„ํ•ด์„œ๋Š” ๊ฐœ์ธ์ •๋ณด์ฒ˜๋ฆฌ์ž ๋˜๋Š” ์ž„์˜์˜ ๋‹ค๋ฅธ ์‚ฌ๋žŒ ๋“ฑ์ด ํ•ฉ๋ฆฌ์ ์œผ๋กœ ์‚ฌ์šฉํ•  ๊ฐ€๋Šฅ์„ฑ์ด ์žˆ๋Š” ๋ชจ๋“  ์ˆ˜๋‹จ์„ ๊ณ ๋ คํ•˜์—ฌ์•ผ ํ•˜๊ณ , ๋งŒ์•ฝ ํŠน์ • ๊ฐœ์ธ์„ ์•Œ์•„๋ณด๊ธฐ ์œ„ํ•˜์—ฌ ๋ถˆํ•ฉ๋ฆฌํ•  ์ •๋„์˜ ์‹œ๊ฐ„, ๋…ธ๋ ฅ, ๋น„์šฉ ๋“ฑ์ด ํˆฌ์ž…๋˜์–ด์•ผ ํ•œ๋‹ค๋ฉด ํ•ด๋‹น ์ •๋ณด๋Š” ๋‹ค๋ฅธ ์ •๋ณด์™€ ์‰ฝ๊ฒŒ ๊ฒฐํ•ฉํ•˜์—ฌ ํŠน์ • ๊ฐœ์ธ์„ ์•Œ์•„๋ณผ ์ˆ˜ ์žˆ๋Š” ๊ฐœ์ธ์ •๋ณด์— ํ•ด๋‹นํ•œ๋‹ค๊ณ  ๋ณผ ์ˆ˜ ์—†๋‹ค. ์œ„์™€ ๊ฐ™์€ ๊ธฐ์ค€์— ๋น„์ถ”์–ด ์‚ดํ”ผ๊ฑด๋Œ€, โ‘  ์ด ์‚ฌ๊ฑด CCTV ์˜์ƒ์€ ์ดฌ์˜๋œ ํ”ผ๊ณ ์ธ์˜ ๋ชจ์Šต ๊ทธ ์ž์ฒด๋กœ์„œ ํ”ผ๊ณ ์ธ์„ ์•Œ์•„๋ณผ ์ˆ˜ ์žˆ๋Š” ์ž๋ฃŒ์ด๋ฏ€๋กœ ๊ฐœ์ธ์ •๋ณด์— ํ•ด๋‹นํ•œ๋‹ค๊ณ  ํ•  ์ˆ˜ ์žˆ๋‹ค. ๊ทธ๋Ÿฌ๋‚˜ โ‘ก ์ด ์‚ฌ๊ฑด์—์„œ ๊ตญ์ •์› ์ˆ˜์‚ฌ๊ด€๋“ค์ด ์ทจ๋“ํ•œ PC ์‚ฌ์šฉ์ •๋ณด์˜ ๋‚ด์šฉ์€ '์ปดํ“จํ„ฐ ์‚ฌ์šฉ์‹œ๊ฐ„, USB ์—ฐ๊ฒฐ์‹œ๊ฐ„, โ–ฒโ–ฒโ–ฒโ–ฒ.net ์‚ฌ์ดํŠธ ์ ‘์† ๋ฐ ๋ฉ”์ผ๋ฐœ์‹ ๋‚ด์—ญ ํ™•์ธ, ํ”ผ๊ณ ์ธ์ด ์‚ฌ์šฉํ•œ ์ด๋ฉ”์ผ ์•„์ด๋””, ๋ฉ”์ผ์˜ ์ œ๋ชฉโ€˜ ๋“ฑ์ด๋‹ค. ๊ทธ๋Ÿฐ๋ฐ ๋‹น์‹œ ํ”ผ๊ณ ์ธ์ด ์‚ฌ์šฉํ•œ ์•„์ด๋””๋Š” โ€™(์˜๋ฌธ ID 2 ์ƒ๋žต)โ€˜์ผ ๋ฟ ๊ทธ ์ž์ฒด์—์„œ ํ”ผ๊ณ ์ธ์˜ ์‹ ์›์„ ์ถ”์ •ํ•  ๋งŒํ•œ ์–ด๋– ํ•œ ๊ธฐ์žฌ๋„ ์—†๊ณ , ์ด๋Š” ์ค‘๊ตญ ์‚ฌ์ดํŠธ www.โ–ฒโ–ฒโ–ฒโ–ฒ.net์— ๊ด€ํ•œ ๊ฒƒ์œผ๋กœ ํ”ผ๊ณ ์ธ์˜ ๋ช…์˜๋กœ ๊ฐ€์ž…๋œ ๊ฒƒ๋„ ์•„๋‹ˆ๋‹ค. ๋”ฐ๋ผ์„œ ์ด๋Ÿฌํ•œ ์ •๋ณด๋กœ๋Š” ์ทจ๋“ํ•œ ๋‹ค๋ฅธ ์ •๋ณด์™€์˜ ๊ฒฐํ•ฉ์„ ํ†ตํ•˜์—ฌ๋„ ํ”ผ๊ณ ์ธ์„ ์‰ฝ๊ฒŒ ์•Œ์•„๋ณผ ์ˆ˜๋Š” ์—†๋‹ค๊ณ  ํ•  ๊ฒƒ์ด๋ฏ€๋กœ ์ด ์‚ฌ๊ฑด PC ์‚ฌ์šฉ์ •๋ณด๋Š” ํ”ผ๊ณ ์ธ์˜ ๊ฐœ์ธ์ •๋ณด๋ผ๊ณ  ํ•˜๊ธฐ ์–ด๋ ต๋‹ค. ๋‹ค) ๊ณต์†Œ์™ธ 19๊ฐ€ ๊ฐœ์ธ์ •๋ณด์ฒ˜๋ฆฌ์ž์ธ์ง€ ๊ฐœ์ธ์ •๋ณด ๋ณดํ˜ธ๋ฒ•์ƒ์˜ โ€œ๊ฐœ์ธ์ •๋ณด์ฒ˜๋ฆฌ์žโ€๋Š” ์—…๋ฌด๋ฅผ ๋ชฉ์ ์œผ๋กœ ๊ฐœ์ธ์ •๋ณดํŒŒ์ผ์„ ์šด์šฉํ•˜๊ธฐ ์œ„ํ•˜์—ฌ ์Šค์Šค๋กœ ๋˜๋Š” ๋‹ค๋ฅธ ์‚ฌ๋žŒ์„ ํ†ตํ•˜์—ฌ ๊ฐœ์ธ์ •๋ณด๋ฅผ ์ฒ˜๋ฆฌํ•˜๋Š” ๊ณต๊ณต๊ธฐ๊ด€, ๋ฒ•์ธ, ๋‹จ์ฒด ๋ฐ ๊ฐœ์ธ ๋“ฑ์„ ๋งํ•˜๋Š”๋ฐ(์ œ2์กฐ ์ œ5ํ˜ธ), ์—ฌ๊ธฐ์—์„œ ๊ฐœ์ธ์ •๋ณดํŒŒ์ผ์ด๋ž€ ๊ฐœ์ธ์ •๋ณด๋ฅผ ์‰ฝ๊ฒŒ ๊ฒ€์ƒ‰ํ•  ์ˆ˜ ์žˆ๋„๋ก ์ผ์ •ํ•œ ๊ทœ์น™์— ๋”ฐ๋ผ ์ฒด๊ณ„์ ์œผ๋กœ ๋ฐฐ์—ดํ•˜๊ฑฐ๋‚˜ ๊ตฌ์„ฑํ•œ ๊ฐœ์ธ์ •๋ณด์˜ ์ง‘ํ•ฉ๋ฌผ์„ ๋งํ•œ๋‹ค(์ œ2์กฐ ์ œ4ํ˜ธ). ๊ทธ๋Ÿฐ๋ฐ ๊ณต์†Œ์™ธ 19๋Š” PC๋ฐฉ์„ ์šด์˜ํ•˜๋ฉด์„œ ๋‹จ์ง€ ๋ฒ”์ฃ„์˜ ์˜ˆ๋ฐฉ, ์‹œ์„ค์•ˆ์ „ ๋ฐ ํ™”์žฌ ์˜ˆ๋ฐฉ ๋“ฑ์„ ์œ„ํ•˜์—ฌ CCTV(์˜์ƒ์ •๋ณด์ฒ˜๋ฆฌ์žฅ์น˜)๋ฅผ ์„ค์น˜ํ•œ ๊ฒƒ์œผ๋กœ ๋ณด์ด๊ณ (๊ฐœ์ธ์ •๋ณด ๋ณดํ˜ธ๋ฒ• ์ œ25์กฐ ์ œ1ํ•ญ ์ฐธ์กฐ), ๋˜ ๊ณ ๊ฐ๋“ค์—๊ฒŒ PC์™€ ์ธํ„ฐ๋„ท ์„ค๋น„๋ฅผ ์ œ๊ณตํ•˜์—ฌ ์ด๋ฅผ ์ด์šฉํ•˜๊ฒŒ ํ•˜์˜€์„ ๋ฟ, ์—…๋ฌด๋ฅผ ๋ชฉ์ ์œผ๋กœ ๊ฐœ์ธ์ •๋ณดํŒŒ์ผ์„ ์šด์šฉํ•˜๊ธฐ ์œ„ํ•˜์—ฌ ๊ฐœ์ธ์ •๋ณด๋ฅผ ์ฒ˜๋ฆฌํ•˜๋Š” ์ž๋ผ๊ณ  ํ•˜๊ธฐ๋Š” ์–ด๋ ต๋‹ค. ๊ณต์†Œ์™ธ 19๋Š” ์ด ๋ฒ•์ •์—์„œ โ€˜PC ์‚ฌ์šฉ์ •๋ณด๋Š” PC ์‚ฌ์šฉ์ข…๋ฃŒ์™€ ๋™์‹œ์— ์ž๋™์œผ๋กœ ์‚ญ์ œ๋œ๋‹คโ€™๊ณ  ์ง„์ˆ ํ•˜๊ณ  ์žˆ๋Š”๋ฐ”, ๊ณต์†Œ์™ธ 19๋Š” ๊ฐœ์ธ์ •๋ณด๋ฅผ ์‰ฝ๊ฒŒ ๊ฒ€์ƒ‰ํ•  ์ˆ˜ ์žˆ๋„๋ก ์ผ์ •ํ•œ ๊ทœ์น™์— ๋”ฐ๋ผ ์ฒด๊ณ„์ ์œผ๋กœ ๋ฐฐ์—ดํ•˜๊ฑฐ๋‚˜ ๊ตฌ์„ฑํ•˜์—ฌ ์ •๋ณด์˜ ์ง‘ํ•ฉ๋ฌผ์„ ์ƒ์„ฑํ•  ์˜๋„๊ฐ€ ์ „ํ˜€ ์—†๋Š” ๊ฒƒ์œผ๋กœ ๋ณด์ด๋ฏ€๋กœ ๊ฐœ์ธ์ •๋ณดํŒŒ์ผ์„ ์šด์šฉํ•  ๋ชฉ์ ์ด ์—†๊ณ , PC๋ฐฉ ์šด์˜ ์—…๋ฌด๋ฅผ ๋ชฉ์ ์œผ๋กœ ๊ฐœ์ธ์ •๋ณด๋ฅผ ์ฒ˜๋ฆฌํ•˜๋Š” ํ–‰์œ„๋„ ํ•˜์ง€ ์•Š๋Š”๋‹ค. ๊ฒฐ๊ตญ ๊ณต์†Œ์™ธ 19๊ฐ€ ์ด ์‚ฌ๊ฑด CCTV ์˜์ƒ์ด๋‚˜ PC ์‚ฌ์šฉ์ •๋ณด์— ๊ด€ํ•˜์—ฌ ๊ฐœ์ธ์ •๋ณด ๋ณดํ˜ธ๋ฒ•์ƒ์˜ ๊ฐœ์ธ์ •๋ณด์ฒ˜๋ฆฌ์ž๋ผ๊ณ  ํ•˜๊ธฐ ์–ด๋ ต๋‹ค. ๋ผ) ๊ฐœ์ธ์ •๋ณด์˜ ์œ ์ถœ๊ณผ ์ˆ˜์‚ฌ์˜ ์œ„๋ฒ•์„ฑ๊ณผ์˜ ๊ด€๊ณ„ ๊ทธ๋Ÿผ์—๋„ ๋ถˆ๊ตฌํ•˜๊ณ  ํ”ผ๊ณ ์ธ์˜ ๊ฐœ์ธ์ ์ธ ์ •๋ณด๊ฐ€ ํ”ผ๊ณ ์ธ์˜ ๋™์˜ ์—†์ด PC๋ฐฉ ์—…์ฃผ์— ์˜ํ•˜์—ฌ ์œ ์ถœ๋œ ๊ฒƒ์€ ์ฃผ23) ์‚ฌ์‹ค์ด๊ณ , ๊ทธ๋กœ ์ธํ•˜์—ฌ ์ผ์ • ๋ถ€๋ถ„ ํ”ผ๊ณ ์ธ์˜ ์‚ฌ์ƒํ™œ์˜ ๋น„๋ฐ€ ๋‚ด์ง€ ๊ฐœ์ธ์ •๋ณด์— ๋Œ€ํ•œ ์ž๊ธฐ๊ฒฐ์ •๊ถŒ์ด ์นจํ•ด๋œ ๊ฒƒ์€ ๋งž๋‹ค๊ณ  ํŒ๋‹จ๋œ๋‹ค. ๊ทธ๋Ÿฌ๋‚˜ ์ด ์‚ฌ๊ฑด์—์„œ ํ”ผ๊ณ ์ธ์ด ํ–‰ํ•œ ๋ฒ”์ฃ„๋Š” ๋ฐ˜๊ตญ๊ฐ€๋‹จ์ฒด ๊ตฌ์„ฑ์›๊ณผ ํšŒํ•ฉยทํ†ต์‹ ํ•˜๊ฑฐ๋‚˜ ๊ทธ ํ™œ๋™์— ๋™์กฐํ•œ๋‹ค๋Š” ๊ฒƒ ๋“ฑ์œผ๋กœ ๊ทธ ๋‚ด์šฉ์ด ๊ตญ๊ฐ€์˜ ์กด๋ฆฝยท์•ˆ์ „์ด๋‚˜ ์ž์œ ๋ฏผ์ฃผ์  ๊ธฐ๋ณธ์งˆ์„œ๋ฅผ ์œ„ํƒœ๋กญ๊ฒŒ ํ•  ์ •๋„๋กœ ์ค‘๋Œ€ํ•˜๋ฉด์„œ๋„ ๋งค์šฐ ์€๋ฐ€ํ•˜๊ฒŒ ์ด๋ฃจ์–ด์ง€๋Š” ์„ฑ๊ฒฉ์˜ ๋ฒ”์ฃ„์ธ ์ , ์œ„ ์ž„์˜์ œ์ถœ๋กœ์จ ์ˆ˜์‚ฌ๊ธฐ๊ด€์ด ์ทจ๋“ํ•œ ๊ฒƒ์€ ํ”ผ๊ณ ์ธ์ด PC๋ฐฉ ์—…์†Œ(๊ณต์ค‘์ ‘๊ฐ์—…์†Œ์ด๋‹ค)์— ์ถœ์ž…ํ•˜์˜€๋‹ค๋Š” ์‚ฌ์‹ค๊ณผ ๊ทธ๊ณณ์—์„œ ์–ด๋–ค USB๋ฅผ PC๋ฐฉ ์ปดํ“จํ„ฐ์— ์—ฐ๊ฒฐํ•˜๊ณ  ์ธํ„ฐ๋„ท์œผ๋กœ www.โ–ฒโ–ฒโ–ฒโ–ฒ.net์— ์ ‘์†ํ•˜์—ฌ ์ด๋ฉ”์ผ์„ ๋ณด๋‚ด๋Š” ๋“ฑ PC๋ฅผ ์‚ฌ์šฉํ•˜์˜€๋‹ค๋Š” ์‚ฌ์‹ค(๊ทธ ๋ฉ”์ผ์˜ ๋‚ด์šฉ์ด๋‚˜ USB์— ์ €์žฅ๋˜์–ด ์žˆ๋˜ ํŒŒ์ผ์˜ ๋‚ด์šฉ์€ ํŒŒ์•…๋˜์ง€ ์•Š๋Š”๋‹ค)์„ ๊ฐ„์ ‘์ ์œผ๋กœ ์ฑ„์ฆํ•œ๋‹ค๋Š” ๊ฒƒ์— ๋ถˆ๊ณผํ•˜์—ฌ ์ด ์ž์ฒด๋กœ๋Š” ํ”ผ๊ณ ์ธ์— ๋Œ€ํ•œ ๊ถŒ์ต์˜ ์นจํ•ด๊ฐ€ ์•„์ฃผ ์ค‘๋Œ€ํ•˜์ง€ ์•Š์€ ์ , ์ผ๋ฐ˜์ ์œผ๋กœ CCTV ์˜์ƒ์€ ์ผ์ • ๋ณด์กด๊ธฐ๊ฐ„์ด ์ง€๋‚˜๋ฉด ์‚ญ์ œ, ํ๊ธฐ๋  ๊ฒƒ์ด๊ณ  ์ปดํ“จํ„ฐ ์‚ฌ์šฉ๊ธฐ๋ก๋„ ๊ณ ๊ฐ์ด ๋– ๋‚œ ํ›„ ๊ทธ ์ปดํ“จํ„ฐ๋ฅผ ๋„๋ฉด ๋ชจ๋‘ ์‚ญ์ œ๋˜๋Š” ์ , ๋”ฐ๋ผ์„œ ์ˆ˜์‚ฌ๊ด€์€ ๊ธ‰๋ฐ•ํ•˜๊ฒŒ PC๋ฐฉ ์‚ฌ์žฅ์ธ ๊ณต์†Œ์™ธ 19์—๊ฒŒ ์ฑ„์ฆ์˜ ํ•„์š”์„ฑ๊ณผ ๊ทธ ๋ฐฉ๋ฒ• ๋“ฑ์„ ์„ค๋ช…ํ•˜๊ณ  ๊ทธ์˜ ์ž๋ฐœ์  ํ˜‘์กฐ ํ•˜์— ์ž„์˜์ œ์ถœ๋กœ์จ ๊ทธ ์ž๋ฃŒ๋ฅผ ํ™•๋ณดํ•œ ๊ฒƒ์ด๊ณ  ๊ทธ ๋ฐฉ๋ฒ•์ด ์ผ๋ฐ˜์ ์œผ๋กœ ํ—ˆ์šฉ๋˜๋Š” ์ •๋„๋ฅผ ๋„˜์ง€ ์•Š๋Š” ์ƒ๋‹นํ•œ ๋ฐฉ๋ฒ•์ด๋ผ๊ณ  ๋ณด์ด๋Š” ์ , ๊ทธ ํ›„ ์ˆ˜์‚ฌ๊ธฐ๊ด€์€ ๋‹ค์ˆ˜์˜ ์••์ˆ˜์ˆ˜์ƒ‰์˜์žฅ ๋ฐ ์ฒดํฌ์˜์žฅ ๋“ฑ์„ ๋ฐœ๋ถ€๋ฐ›๋Š” ๋“ฑ ์ ˆ์ฐจ์— ๋”ฐ๋ผ ํ”ผ๊ณ ์ธ์˜ ๊ตญ๊ฐ€๋ณด์•ˆ๋ฒ•์œ„๋ฐ˜ ํ˜์˜์‚ฌ์‹ค์— ๊ด€ํ•œ ์ˆ˜์‚ฌ๋ฅผ ๊ณ„์†ํ•œ ์ ์„ ๋น„๋กฏํ•˜์—ฌ, ์ด ์‚ฌ๊ฑด ์ˆ˜์‚ฌ์˜ ํ•„์š”์„ฑ๊ณผ ๊ธ‰๋ฐ•์„ฑ, ์‹ค์ฒด์  ์ง„์‹ค์˜ ๊ทœ๋ช…๊ณผ ํ˜•์‚ฌ์‚ฌ๋ฒ•์ •์˜์˜ ์‹คํ˜„์ด๋ผ๋Š” ๊ณต์ต์  ์ด๋… ๋“ฑ์„ ๋ชจ๋‘ ์ข…ํ•ฉํ•˜์—ฌ ๋น„๊ต๊ต๋Ÿ‰ํ•˜์—ฌ ๋ณธ๋‹ค๋ฉด, ๊ตญ์ •์›์ด ์ˆ˜์‚ฌ๋ฅผ ์œ„ํ•˜์—ฌ ๊ณต์†Œ์™ธ 19๋กœ๋ถ€ํ„ฐ ํ”ผ๊ณ ์ธ์— ๊ด€ํ•œ CCTV ์˜์ƒ๊ณผ PC ์‚ฌ์šฉ์ •๋ณด๋ฅผ ์ž„์˜์ œ์ถœ ๋ฐ›์€ ๊ฒƒ์ด ์œ„๋ฒ•ํ•˜๊ฒŒ ์ˆ˜์ง‘ํ•œ ์ฆ๊ฑฐ๋กœ์„œ ์ฆ๊ฑฐ๋Šฅ๋ ฅ์ด ์—†๋‹ค๊ณ  ํ•  ์ˆ˜๋Š” ์—†๋‹ค. 3) ํ”ผ๊ณ ์ธ๊ณผ ๋ณ€ํ˜ธ์ธ๋“ค์˜ ์ด ๋ถ€๋ถ„ ์ฃผ์žฅ์€ ์ด์œ  ์—†๋‹ค. 16. โ–ณโ–ณ๋Œ€ํ•™๊ต์—์„œ ์ž„์˜์ œ์ถœ๋ฐ›์€ CCTV ์˜์ƒ ๋“ฑ์ด ์œ„๋ฒ•ํ•˜๊ฒŒ ์ˆ˜์ง‘ํ•œ ์ฆ๊ฑฐ๋ผ๋Š” ์ฃผ์žฅ ๊ฐ€. ์ฃผ์žฅ CCTV ์˜์ƒ๋…นํ™”๋ฌผ๊ณผ PC ๋กœ๊ทธ(์ ‘์†)์ •๋ณด๋Š” โ–ณโ–ณ๋Œ€ํ•™๊ต๊ฐ€ ์ž„์˜๋กœ ์ œ์ถœํ•  ์ˆ˜ ์—†๋Š” ํ”ผ๊ณ ์ธ์˜ ๊ฐœ์ธ์ •๋ณด์ด๋‹ค. ๋”ฐ๋ผ์„œ ์œ„์™€ ๊ฐ™์€ ๊ฐœ์ธ์ •๋ณด๋ฅผ ์ทจ๋“ํ•˜๊ธฐ ์œ„ํ•ด์„œ๋Š” ํ”ผ๊ณ ์ธ์˜ ๋™์˜ํ•˜์— ์ž„์˜์ œ์ถœ์„ ๋ฐ›๊ฑฐ๋‚˜, ์••์ˆ˜์ˆ˜์ƒ‰๊ฒ€์ฆ์˜์žฅ์„ ๋ฐ›์•„์„œ ์ทจ๋“ํ•ด์•ผ ํ•œ๋‹ค. ๋‚˜. ํŒ๋‹จ 1) ๊ฐœ์ธ์ •๋ณด์ธ์ง€ CCTV ์˜์ƒ๋…นํ™”๋ฌผ์€ ํ”ผ๊ณ ์ธ์˜ ๋ชจ์Šต์ด ์ดฌ์˜๋œ ๊ฒƒ์œผ๋กœ ๊ฐœ์ธ์ •๋ณด๋ผ๊ณ  ํ•  ์ˆ˜ ์žˆ๋‹ค. ๋ฐ˜๋ฉด, ์ด ๋ฒ•์›์— ์ œ์ถœ๋œ PC ๋กœ๊ทธ๊ธฐ๋ก์€ ๋‹นํ•ด ์ปดํ“จํ„ฐ(โ–ณโ–ณ๋Œ€ํ•™๊ต ์ œ1์ข…ํ•ฉ๊ด€ ๋กœ๋น„์— ์„ค์น˜๋œ ๊ณต์šฉ ์ธํ„ฐ๋„ท PC์ด๋‹ค)์—์„œ ์ ‘์†ํ•œ ์‚ฌ์ดํŠธ๊ฐ€ ์–ด๋””์ธ์ง€, ์ถœ๋ฐœ์ง€ ๋ฐ ๋ชฉ์ ์ง€ ํฌํŠธ, ๋ฐœ์‹  ๋ฐ ์ˆ˜์‹  ํŒจํ‚ท ๋“ฑ์„ ๋‚˜ํƒ€๋‚ด๋Š” ๊ฒƒ์œผ๋กœ ๊ธฐ๊ณ„์ ์œผ๋กœ ์ˆ˜์น˜ํ™”๋œ ์ •๋ณด์ผ ๋ฟ, ํ”ผ๊ณ ์ธ ๊ฐœ์ธ์„ ์‹๋ณ„ํ•  ๋งŒํ•  ์–ด๋– ํ•œ ์ •๋ณด๋„ ์—†์œผ๋ฏ€๋กœ ๊ฐœ์ธ์ •๋ณด๋ผ๊ณ  ํ•  ์ˆ˜ ์—†๋‹ค. ๋”ฐ๋ผ์„œ PC ๋กœ๊ทธ๊ธฐ๋ก์„ ์ž„์˜์ œ์ถœ ๋ฐ›์€ ๊ฒƒ์€ ๊ฐœ์ธ์ •๋ณด ๋ณดํ˜ธ๋ฒ•์„ ์œ„๋ฐ˜ํ•œ ์œ„๋ฒ•ํ•œ ์ˆ˜์‚ฌ๋ผ๊ณ  ํ•  ์ˆ˜ ์—†๋‹ค. 2) ๊ฐœ์ธ์ •๋ณด๋ฅผ ์ž„์˜์ œ์ถœ ๋ฐ›์€ ๊ฒƒ์ด ์œ„๋ฒ•ํ•œ ์ˆ˜์‚ฌ์ธ์ง€ ์šฐ์„  โ–ณโ–ณ๋Œ€ํ•™๊ต๋Š” ๊ทธ ์‹œ์„ค ๋‚ด์— CCTV ๋ฐ ๊ณต์šฉ PC๋ฅผ ์„ค์น˜ํ•˜๊ธฐ๋Š” ํ•˜์˜€์œผ๋‚˜ ์—…๋ฌด๋ฅผ ๋ชฉ์ ์œผ๋กœ ์ฃผ24) ๊ฐœ์ธ์ •๋ณดํŒŒ์ผ์„ ์šด์šฉํ•˜๊ธฐ ์œ„ํ•œ โ€˜๊ฐœ์ธ์ •๋ณด์ฒ˜๋ฆฌ์žโ€™๋ผ๊ณ  ๋ณด๊ธฐ ์–ด๋ ต๋‹ค. ๋˜ํ•œ, ๊ณต๊ณต๊ธฐ๊ด€์€ ์ •๋ณด์ฃผ์ฒด ๋˜๋Š” ์ œ3์ž์˜ ์ด์ต์„ ๋ถ€๋‹นํ•˜๊ฒŒ ์นจํ•ดํ•  ์šฐ๋ ค๊ฐ€ ์žˆ์„ ๋•Œ์— ํ•ด๋‹นํ•˜์ง€ ์•„๋‹ˆํ•˜๋Š” ์ด์ƒ โ€˜๋ฒ”์ฃ„์˜ ์ˆ˜์‚ฌ์™€ ๊ณต์†Œ์˜ ์ œ๊ธฐ ๋ฐ ์œ ์ง€๋ฅผ ์œ„ํ•˜์—ฌ ํ•„์š”ํ•œ ๊ฒฝ์šฐโ€™์—๋Š” ๊ฐœ์ธ์ •๋ณด๋ฅผ ์ œ3์ž์—๊ฒŒ ์ œ๊ณตํ•  ์ˆ˜ ์žˆ๋Š”๋ฐ”(๊ฐœ์ธ์ •๋ณด ๋ณดํ˜ธ๋ฒ• ์ œ18์กฐ ์ œ2ํ•ญ ์ œ7ํ˜ธ), โ–ณโ–ณ๋Œ€ํ•™๊ต๋Š” ๊ณ ๋“ฑ๊ต์œก๋ฒ•์—์„œ ์ •ํ•œ ๋Œ€ํ•™๊ต๋กœ์„œ ๊ณต๊ณต๊ธฐ๊ด€์— ํ•ด๋‹นํ•˜๋ฏ€๋กœ(๊ฐœ์ธ์ •๋ณด ๋ณดํ˜ธ๋ฒ• ์‹œํ–‰๋ น ์ œ2์กฐ ์ฐธ์กฐ) ๊ฐœ์ธ์ •๋ณด๋ฅผ ์ˆ˜์‚ฌ๊ธฐ๊ด€์— ์ œ๊ณตํ•  ์ˆ˜ ์žˆ๋‹ค. ๊ฒŒ๋‹ค๊ฐ€, ๋‹น์‹œ ํ”ผ๊ณ ์ธ์ด ๋ฐ›๊ณ  ์žˆ์—ˆ๋˜ ํ˜์˜๋Š” ๋ฐ˜๊ตญ๊ฐ€๋‹จ์ฒด์™€ ๋น„๋ฐ€์ง€๋ น๋ฌธ์„ ์ฃผ๊ณ ๋ฐ›๋Š”๋‹ค๋Š” ๊ฒƒ์œผ๋กœ ๊ตญ๊ฐ€์˜ ์กด๋ฆฝยท์•ˆ์ „์ด๋‚˜ ์ž์œ ๋ฏผ์ฃผ์  ๊ธฐ๋ณธ์งˆ์„œ๋ฅผ ์œ„ํƒœ๋กญ๊ฒŒ ํ•  ์ •๋„๋กœ ์ค‘๋Œ€ํ•˜๊ณ , ๊ทธ ์ˆ˜์‚ฌ๋ฅผ ์œ„ํ•ด์„œ ํ”ผ๊ณ ์ธ์ด โ–ณโ–ณ๋Œ€ํ•™๊ต์˜ PC๋ฅผ ์‚ฌ์šฉํ–ˆ๋Š”์ง€๋ฅผ ํ™•์ธํ•  ํ•„์š”์„ฑ์ด ์žˆ๋‹ค๊ณ  ๋ณด์ด๋Š” ์ , ํ”ผ๊ณ ์ธ์˜ ๋ชจ์Šต์ด ์ดฌ์˜๋œ ๊ณณ์€ โ–ณโ–ณ๋Œ€ํ•™๊ต ์ข…ํ•ฉ๊ด€ ๋กœ๋น„๋กœ์„œ ๋‹ค์ˆ˜์˜ ์‚ฌ๋žŒ๋“ค์ด ์ด์šฉํ•˜๋Š” ๊ณต๊ฐœ๋œ ์žฅ์†Œ์ด๊ณ , ์‹ค์ œ ์ดฌ์˜๋œ ์˜์ƒ๋„ ํ”ผ๊ณ ์ธ์ด PC๋ฅผ ์‚ฌ์šฉํ•˜๋Š” ๋ชจ์Šต์ผ ๋ฟ์ด์–ด์„œ ํ”ผ๊ณ ์ธ์˜ ์‚ฌ์ƒํ™œ ์นจํ•ด ๋“ฑ์ด ๊ฑฐ์˜ ๋ฌธ์ œ๊ฐ€ ๋˜์ง€ ์•Š๋Š” ์  ๋“ฑ์„ ์ข…ํ•ฉํ•˜๋ฉด, ์ด๋ฅผ ์ œ๊ณตํ•จ์— ๋”ฐ๋ผ ํ”ผ๊ณ ์ธ์˜ ์ด์ต์„ ๋ถ€๋‹นํ•˜๊ฒŒ ์นจํ•ดํ•  ์šฐ๋ ค๊ฐ€ ์žˆ๋Š” ๊ฒฝ์šฐ๋ผ๊ณ ๋Š” ๋ณผ ์ˆ˜ ์—†๋‹ค. ๊ฒฐ๊ตญ, ๊ตญ์ •์› ์ˆ˜์‚ฌ๊ด€์ด โ–ณโ–ณ๋Œ€ํ•™๊ต๋กœ๋ถ€ํ„ฐ ํ”ผ๊ณ ์ธ์˜ PC ์ ‘์†๊ธฐ๋ก์€ ๋ฌผ๋ก  CCTV ์˜์ƒ์„ ์ž„์˜์ œ์ถœ ๋ฐ›์€ ๊ฒƒ์„ ๊ฐ€์ง€๊ณ  ์œ„๋ฒ•ํ•˜๋‹ค๊ณ  ํ•  ์ˆ˜๋Š” ์—†๋‹ค. 3) ํ”ผ๊ณ ์ธ ๋ฐ ๋ณ€ํ˜ธ์ธ๋“ค์˜ ์ด ๋ถ€๋ถ„ ์ฃผ์žฅ์€ ์ด์œ  ์—†๋‹ค. 17. โ–กโ–กโ–ก๋ฅผ ์••์ˆ˜ยท์ˆ˜์ƒ‰ํ•˜๋Š” ๊ณผ์ •์ด ์œ„๋ฒ•ํ•˜์—ฌ ๊ทธ ๊ฒฐ๊ณผ๋ฌผ์ด ์œ„๋ฒ•ํ•˜๋‹ค๋Š” ์ฃผ์žฅ ๊ฐ€. ์ฃผ์žฅ 1) ์••์ˆ˜ยท์ˆ˜์ƒ‰ ์ง‘ํ–‰๊ณผ์ •์—์„œ ๊ณต์†Œ์™ธ 21 ์ฃผ์‹ํšŒ์‚ฌ(์ดํ•˜ โ€˜๊ณต์†Œ์™ธ 21 ํšŒ์‚ฌโ€™๋ผ ํ•œ๋‹ค)๊ฐ€ ์••์ˆ˜์ˆ˜์ƒ‰๊ฒ€์ฆ์˜์žฅ์˜ ๋ฒ”์ฃ„ํ˜์˜์™€ ๊ด€๋ จ์„ฑ์ด ์—†๋Š” ์ž๋ฃŒ๊นŒ์ง€๋„ ์ „๋ถ€ ์ถ”์ถœํ•˜์—ฌ ์ด๋ฏธ์ง•ํ•˜์˜€๋‹ค. ๋”ฐ๋ผ์„œ ์••์ˆ˜์ˆ˜์ƒ‰๊ฒ€์ฆ์˜์žฅ์˜ ๋ฒ”์œ„๋ฅผ ๋„˜์–ด์„  ์œ„๋ฒ•ํ•œ ์ง‘ํ–‰์ด๋‹ค. 2) ์›๋ณธ ๋™์ผ์„ฑ๊ณผ ๋ฌด๊ฒฐ์„ฑ์„ ๋ณด์žฅํ•˜๊ธฐ ์œ„ํ•œ ํ•ด์‰ฌ๊ฐ’ ์‚ฐ์ถœ์ ˆ์ฐจ๋ฅผ ๊ฑฐ์น˜์ง€ ์•Š์•˜๋‹ค. ๋˜ํ•œ ์••์ˆ˜์กฐ์„œ์— ์ฐธ์—ฌ์ธ์œผ๋กœ ๊ธฐ์žฌ๋œ ๊ณต์†Œ์™ธ 21 ํšŒ์‚ฌ ์ง์›๊ณผ ๋ด‰์ธยท๋‚ ์ธํ•œ ๊ณต์†Œ์™ธ 21 ํšŒ์‚ฌ ์ง์›์ด ์ƒ์ดํ•˜๋‹ค. ๋‚˜. ์ฆ๊ฑฐ์— ์˜ํ•ด ์ธ์ •๋˜๋Š” ์‚ฌ์‹ค ใ€ˆ์••์ˆ˜ยท์ˆ˜์ƒ‰ ๊ณผ์ •ใ€‰ ์ฆ ์ œ28ํ˜ธ[DVD-R(2010. 7. 22. ๊ณต์†Œ์™ธ 21 ์ฃผ์‹ํšŒ์‚ฌ)], ์ œ30ํ˜ธ[CD(2011. 12. 2. ๊ณต์†Œ์™ธ 21 ์ฃผ์‹ํšŒ์‚ฌ, ID : (์˜๋ฌธ ID 7 ์ƒ๋žต) โ—โ—โ—โ—โ— ์ €์žฅ๋ฌผ)] : ๋ฒ•์›์œผ๋กœ๋ถ€ํ„ฐ 2010. 7. 21. ์ฃผ25) ์••์ˆ˜์ˆ˜์ƒ‰๊ฒ€์ฆ์˜์žฅ์„ ๋ฐœ๋ถ€๋ฐ›์•„ 2010. 7. 22. ๊ณต์†Œ์™ธ 21 ํšŒ์‚ฌ์— ์œ„ํƒ์ง‘ํ–‰์„ ํ•˜์˜€๊ณ , 2011. 11. 30. ๊ฐ™์€ ์ทจ์ง€์˜ ์••์ˆ˜์ˆ˜์ƒ‰๊ฒ€์ฆ์˜์žฅ์„ ๋ฐœ๋ถ€๋ฐ›์•„ 2011. 12. 1. ๊ณต์†Œ์™ธ 21 ํšŒ์‚ฌ์— ์ง‘ํ–‰์œ„ํƒ์„ ํ•˜์˜€๋‹ค. ๊ณต์†Œ์™ธ 21 ํšŒ์‚ฌ๋Š” ํ”ผ๊ณ ์ธ์˜ ์ด๋ฉ”์ผ ๊ณ„์ •์— ๋ณด๊ด€๋˜์–ด ์žˆ๋Š” ์ž๋ฃŒ๋“ค์„ DVD์— ์ €์žฅํ•œ ๋’ค ๋ด‰์ธํ•˜์—ฌ ์ˆ˜์‚ฌ๊ด€์—๊ฒŒ ์ฃผ์—ˆ๊ณ , ๋™์ผํ•œ ํŒŒ์ผ์„ ์ˆ˜์‚ฌ๊ด€์˜ ์ด๋ฉ”์ผ๋กœ๋„ ์ „์†กํ•ด ์ฃผ์—ˆ๋‹ค. ์œ„ ๋‘ ์˜์žฅ์— ๋Œ€ํ•˜์—ฌ ์ง‘ํ–‰ํ•œ ์‚ฌ์‹ค์„ ํ™•์ธํ•˜๋ฉด์„œ ์„œ๋ช…ํ•œ ์‚ฌ๋žŒ์€ ๊ณต์†Œ์™ธ 21 ํšŒ์‚ฌ ๊ฐœ์ธ์ •๋ณด๋ณดํ˜ธํŒ€ ๊ณต์†Œ์™ธ 69์ด๋‹ค. ์ˆ˜์‚ฌ๊ด€์€ ์œ„์™€ ๊ฐ™์ด ์ด๋ฉ”์ผ๋กœ ๋ฐ›์€ ํŒŒ์ผ์—์„œ ํ˜์˜์‚ฌ์‹ค๊ณผ ๊ด€๋ จ๋œ ์ •๋ณด๋ฅผ ์„ ๋ณ„์••์ˆ˜ํ•˜๊ณ  ๊ด€๋ จ ์—†๋Š” ์ •๋ณด๋Š” ์‚ญ์ œยทํ๊ธฐํ•˜์˜€๋‹ค. ใ€ˆ์ด ๋ฒ•์›์˜ ๊ฒ€์ฆใ€‰ ์ฆ ์ œ28ํ˜ธ[DVD-R(2010. 7. 22. ๊ณต์†Œ์™ธ 21 ์ฃผ์‹ํšŒ์‚ฌ)] : 2016. 9. 26. ๊ฒ€์ฆํ•˜์—ฌ ์••์ˆ˜๋‹น์‹œ ์›๋ณธ์ด ๋ด‰์ธ๋˜์–ด ์žˆ๋Š” ๊ฒƒ์„ ํ™•์ธํ•˜๊ณ , ๋ด‰์ธ์„ ํ•ด์ œํ•˜์—ฌ ๊ทธ DVD(์ฆ‰, ๊ณต์†Œ์™ธ 21 ํšŒ์‚ฌ์—์„œ ์ตœ์ดˆ ์ง‘ํ–‰ํ•˜์—ฌ ๋ด‰์ธํ•ด๋‘” ๊ทธ๋Œ€๋กœ์˜ ์›๋ณธ) ๋‚ด์— ์ €์žฅ๋˜์–ด ์žˆ๋Š” ํŒŒ์ผ์„ ํ™•์ธํ•˜์˜€๋‹ค. 2011. 12. 2. ์••์ˆ˜ยท์ˆ˜์ƒ‰ํ•˜์—ฌ ์ฑ„๋“ํ•œ ์ฆ๊ฑฐ๋ฌผ์€ ์ด ์‚ฌ๊ฑด ๊ณตํŒ๊ณผ์ •์— ์ œ์ถœ๋˜์ง€ ์•Š์•˜๋‹ค. ใ€ˆ์ฆ์ธ์˜ ์ฆ์–ธใ€‰ ๊ณต์†Œ์™ธ 14 : ๊ณต์†Œ์™ธ 21 ํšŒ์‚ฌ ์ง์›์ด ๊ณต์†Œ์™ธ 21 ํšŒ์‚ฌ ์„œ๋ฒ„์—์„œ ์˜์žฅ์— ๊ธฐ์žฌ๋œ ํ”ผ๊ณ ์ธ์˜ ์ž๋ฃŒ๋ฅผ ์ถ”์ถœํ•˜์—ฌ DVD์— ๋‹ด์•„ ์ฃผ์—ˆ๋‹ค. ๋ด‰์ธ๋„ ๊ณต์†Œ์™ธ 21 ํšŒ์‚ฌ ์ชฝ์—์„œ ํ•ด์ค€ ๊ทธ๋Œ€๋กœ์ด๋‹ค. ๋ณ„๋„๋กœ ์ด๋ฉ”์ผ๋กœ ๋ฐ›์€ ํŒŒ์ผ์—์„œ ํ˜์˜์‚ฌ์‹ค๊ณผ ๊ด€๋ จ๋œ ์ •๋ณด๋ฅผ ์„ ๋ณ„์••์ˆ˜ํ•˜๊ณ  ๊ด€๋ จ ์—†๋Š” ์ •๋ณด๋Š” ์‚ญ์ œยทํ๊ธฐํ•˜์˜€๋‹ค. ๋‹ค. ํŒ๋‹จ 1) ๊ณต์†Œ์™ธ 21 ํšŒ์‚ฌ์—์„œ ๋ณ„๋„๋กœ ์„ ๋ณ„ ์••์ˆ˜ยท์ˆ˜์ƒ‰ํ•˜์ง€ ์•Š์€ ๊ฒƒ์ด ์œ„๋ฒ•ํ•œ ์••์ˆ˜ยท์ˆ˜์ƒ‰์ธ์ง€(์ „๋ถ€ ์œ„ํƒํ•œ ๊ฒƒ์ด ์œ„๋ฒ•ํ•œ ์••์ˆ˜ยท์ˆ˜์ƒ‰์ธ์ง€) ํ˜•์‚ฌ์†Œ์†ก๋ฒ•์—์„œ ์ˆ˜์‚ฌ๊ถŒ์˜ ์ฃผ์ฒด๋ฅผ ์ˆ˜์‚ฌ๊ธฐ๊ด€์ธ ๊ฒ€์‚ฌ ๋˜๋Š” ์‚ฌ๋ฒ•๊ฒฝ์ฐฐ๊ด€์œผ๋กœ ํ•œ์ •ํ•˜๊ณ  ๊ฐ•์ œ์ˆ˜์‚ฌ์— ํ•ด๋‹นํ•˜๋Š” ์••์ˆ˜ยท์ˆ˜์ƒ‰์˜ ์ง‘ํ–‰์ฃผ์ฒด ์—ญ์‹œ ์ˆ˜์‚ฌ๊ธฐ๊ด€์œผ๋กœ ์ •ํ•œ ์ทจ์ง€๋Š”, ์ˆ˜์‚ฌ๋ฅผ ํ†ตํ•œ ์‹ค์ฒด์  ์ง„์‹ค์˜ ๋ฐœ๊ฒฌ ๋ฐ ๊ตญ๊ฐ€์˜ ์ •๋‹นํ•œ ํ˜•๋ฒŒ๊ถŒ์˜ ํ–‰์‚ฌ์—๋Š” ํ•„์—ฐ์ ์œผ๋กœ ํ•ด๋‹น ๊ฐœ์ธ์˜ ๊ธฐ๋ณธ๊ถŒ์„ ์นจํ•ด ๋˜๋Š” ์ œ์•ฝ์ด ์ˆ˜๋ฐ˜๋  ์ˆ˜๋ฐ–์— ์—†์œผ๋ฏ€๋กœ ๊ตญ๊ฐ€๊ธฐ๊ด€์ด ์•„๋‹Œ ์‚ฌ์ธ์— ์˜ํ•œ ์ž์˜์ ์ธ ์ˆ˜์‚ฌํ–‰์œ„๋ฅผ ๋ฐฉ์ง€ํ•˜๋Š” ๋ฐ์— ๊ทธ ์ทจ์ง€๊ฐ€ ์žˆ๋‹ค๊ณ  ํŒ๋‹จ๋˜๋‚˜, ๊ทธ๋ ‡๋‹ค๊ณ  ํ•˜์—ฌ ๋ฐ˜๋“œ์‹œ ๋ชจ๋“  ์ˆ˜์‚ฌ์ ˆ์ฐจ๊ฐ€ ์˜ค๋กœ์ง€ ์ˆ˜์‚ฌ๊ธฐ๊ด€์— ์˜ํ•˜์—ฌ๋งŒ ์ด๋ฃจ์–ด์ ธ์•ผ ํ•œ๋‹ค๋Š” ์ทจ์ง€๋กœ๋Š” ๋ณผ ์ˆ˜ ์—†๊ณ , ๋”ฐ๋ผ์„œ ์ˆ˜์‚ฌ๊ธฐ๊ด€์ด ๋ฒ”์ฃ„ํ˜์˜์— ๋Œ€ํ•œ ์ˆ˜์‚ฌ๋ฅผ ํ•จ์— ์žˆ์–ด์„œ ์ „๋ฌธ๊ฐ€์ธ ์‚ฌ์ธ์˜ ํ˜‘์กฐ๋ฅผ ๋ฐ›๊ฑฐ๋‚˜ ๊ทธ ์ž๋ฌธ์„ ๊ตฌํ•˜๋Š” ๊ฒƒ ์—ญ์‹œ ์ •๋‹นํ•œ ์ˆ˜์‚ฌ๊ถŒ์˜ ๋ฒ”์œ„์— ์†ํ•˜๋Š” ์ ๋ฒ•ํ•œ ์ ˆ์ฐจ๋ผ๊ณ  ํ•  ์ฃผ26) ๊ฒƒ์ด๋‹ค. ์ด ์‚ฌ๊ฑด๊ณผ ๊ฐ™์ด ์••์ˆ˜ยท์ˆ˜์ƒ‰๋Œ€์ƒ ์ด๋ฉ”์ผ ๊ณ„์ •์„ ์ธํ„ฐ๋„ท์„œ๋น„์Šค์ œ๊ณต์ž๊ฐ€ ๋ณด๊ด€ํ•˜๊ณ  ์žˆ๋Š” ๊ฒฝ์šฐ์—๋Š” ์ˆ˜์‚ฌ๊ธฐ๊ด€์ด ์••์ˆ˜ยท์ˆ˜์ƒ‰์˜์žฅ์˜ ์ง‘ํ–‰ ํ˜„์žฅ์—์„œ ์ธํ„ฐ๋„ท์„œ๋น„์Šค์ œ๊ณต์ž๊ฐ€ ์ด๋ฉ”์ผ ๊ณ„์ •์„ ๋ณด๊ด€ํ•˜๊ณ  ์žˆ๋Š” ์„œ๋ฒ„ ๋“ฑ์— ์ง์ ‘ ์ ‘์†ํ•˜์—ฌ ์ด๋ฅผ ์ˆ˜์ƒ‰ํ•œ ํ›„ ์••์ˆ˜๋Œ€์ƒ ๊ณ„์ •์—์„œ ์••์ˆ˜๋Œ€์ƒ๊ธฐ๊ฐ„ ๋™์•ˆ ์†ก์ˆ˜์‹ ๋œ ์ด๋ฉ”์ผ, ๊ฐ์ข… ๋ณด๊ด€์ž๋ฃŒ ๋“ฑ์„ ์ผ๊ด„ํ•˜์—ฌ ์ถ”์ถœํ•˜๋Š” ๋ฐฉ์‹์œผ๋กœ ์••์ˆ˜ํ•˜๋Š” ๊ฒƒ์€ ๊ธฐ์ˆ ์ ์œผ๋กœ๋‚˜ ์‹œ๊ฐ„์ ์œผ๋กœ ์–ด๋ ค์šธ ์ˆ˜ ์žˆ๋‹ค(์ธํ„ฐ๋„ท์„œ๋น„์Šค์ œ๊ณต์ž์˜ ํ˜‘๋ ฅ ์—†์ด ์ˆ˜์‚ฌ๊ธฐ๊ด€์ด ์ธํ„ฐ๋„ท์„œ๋น„์Šค์ œ๊ณต์ž์˜ ์„œ๋ฒ„์— ์ง์ ‘ ์ ‘์†ํ•˜์—ฌ ๋Œ€์ƒ ์ด๋ฉ”์ผ, ๊ฐ์ข… ๋ณด๊ด€์ž๋ฃŒ ๋“ฑ๋งŒ ์ถ”์ถœํ•˜๋Š” ๊ฒƒ์ด ์‚ฌ์‹ค์ƒ ๋ถˆ๊ฐ€๋Šฅํ•˜๊ณ , ์ถ”์ถœ์ด ๊ฐ€๋Šฅํ•˜๋”๋ผ๋„ ์••์ˆ˜๋Œ€์ƒ๋ฌผ์ด ๋งŽ๊ณ  ์••์ˆ˜๋Œ€์ƒ๊ธฐ๊ฐ„์ด ์žฅ๊ธฐ๊ฐ„์ธ ๊ฒฝ์šฐ์—๋Š” ์˜์žฅ ์ œ์‹œ์ผ ๋‚ด์— ๋ชจ๋“  ๋Œ€์ƒ๋ฌผ์„ ์ถ”์ถœํ•˜๋Š” ๊ฒƒ์ด ์–ด๋ ค์šธ ์ˆ˜ ์žˆ์œผ๋ฉฐ, ํ•œํŽธ ์„œ๋ฒ„ ์ž์ฒด๋ฅผ ์••์ˆ˜ํ•  ์ˆ˜๋Š” ์—†๊ณ , ์ด๋ฉ”์ผ ๋“ฑ์ด ๋ณด๊ด€๋œ ์„œ๋ฒ„๋งŒ์„ ๋ถ„๋ฆฌํ•ด์„œ ์••์ˆ˜ํ•˜๋Š” ๊ฒƒ๋„ ์‚ฌ์‹ค์ƒ ๋ถˆ๊ฐ€๋Šฅํ•˜๋‹ค). ์ด์™€ ๊ฐ™์€ ์ƒํ™ฉ์—์„œ ํ†ต์ƒ ์ˆ˜์‚ฌ๊ธฐ๊ด€์€ ์••์ˆ˜ยท์ˆ˜์ƒ‰์˜์žฅ์„ ๋Œ€์ƒ์ž๊ฐ€ ๊ฐ€์ž…ํ•œ ์ด๋ฉ”์ผ ๊ณ„์ • ๋“ฑ์„ ์‚ฌ์šฉํ•˜๋Š” ์ธํ„ฐ๋„ท์„œ๋น„์Šค์ œ๊ณต์ž์˜ ๋‹ด๋‹น ์ง์›(ํ˜•์‚ฌ์†Œ์†ก๋ฒ• ์ œ118์กฐ์—์„œ ๋งํ•˜๋Š” โ€˜์ฒ˜๋ถ„์„ ๋ฐ›๋Š” ์žโ€™๋ผ ํ•จ์€ ์••์ˆ˜ยท์ˆ˜์ƒ‰์„ ๋‹นํ•˜๋Š” ์ž๋กœ ์••์ˆ˜ํ•  ๋ฌผ๊ฑด ๋˜๋Š” ์ˆ˜์ƒ‰ํ•  ์žฅ์†Œ๋ฅผ ํ˜„์‹ค์ ์œผ๋กœ ์ง€๋ฐฐํ•˜๋Š” ์ž๋ฅผ ์˜๋ฏธํ•œ๋‹ค๊ณ  ํ•  ๊ฒƒ์ด๋‹ค)์—๊ฒŒ ์ œ์‹œํ•˜๊ณ , ๋‹ด๋‹น ์ง์›์œผ๋กœ๋ถ€ํ„ฐ ์˜์žฅ์„ ์ œ์‹œ๋ฐ›์•˜๋‹ค๋Š” ์ทจ์ง€์˜ ์„œ๋ช…ยท๋‚ ์ธ ๋“ฑ์„ ๋ฐ›์€ ๋‹ค์Œ, ๋‹ด๋‹น ์ง์›์ด ์••์ˆ˜๋Œ€์ƒ ์ด๋ฉ”์ผ, ๊ฐ์ข… ๋ณด๊ด€์ž๋ฃŒ ๋“ฑ์„ ์ถ”์ถœํ•˜๊ณ , ๊ทธ ๊ฒฐ๊ณผ๋ฌผ์„ ์ˆ˜์‚ฌ๊ธฐ๊ด€์ด ์‚ฌ์šฉํ•˜๋Š” ์ด๋ฉ”์ผ ๋“ฑ์œผ๋กœ ์†ก์‹ ํ•˜๋Š” ํ˜•์‹์œผ๋กœ ์ด๋ฃจ์–ด์ ธ ์™”๋˜ ๊ฒƒ์œผ๋กœ ๋ณด์ธ๋‹ค. ์ด ์‚ฌ๊ฑด์˜ ๊ฒฝ์šฐ ์••์ˆ˜ยท์ˆ˜์ƒ‰ํ•  ๋Œ€์ƒ๊ณผ ๊ธฐ๊ฐ„์ด ๋งค์šฐ ๊ด‘๋ฒ”์œ„ํ•˜์˜€๊ณ , ์ด์— ์ˆ˜์‚ฌ๊ด€์€ ๊ณต์†Œ์™ธ 21 ํšŒ์‚ฌ ์†Œ์† ๋‹ด๋‹น ์ง์›์œผ๋กœ๋ถ€ํ„ฐ ์••์ˆ˜๋Œ€์ƒ ์ด๋ฉ”์ผ ๋“ฑ์„ ์ €์žฅํ•œ DVD์™€ ๊ทธ์™€ ๋™์ผํ•œ ํŒŒ์ผ์„ ๋ฉ”์ผ๋กœ ๋ฐ›์•˜๋‹ค. ์ˆ˜์‚ฌ๊ธฐ๊ด€์€ ์›๋ณธ์€ ๊ฐœ๋ด‰ํ•˜์ง€ ์•Š์€ ์ฑ„, ์ด๋ฉ”์ผ๋กœ ๋ฐ›์€ ํŒŒ์ผ์—์„œ ๋ฒ”์ฃ„์‚ฌ์‹ค๊ณผ ๊ด€๋ จ์„ฑ ์žˆ๋Š” ํŒŒ์ผ์„ ์„ ๋ณ„ํ•˜์—ฌ ํ•ด๋‹น๋˜๋Š” ์ด๋ฉ”์ผ ๋“ฑ๋งŒ์„ ์••์ˆ˜ํ•œ๋‹ค๋Š” ์ทจ์ง€์˜ ๋ณด๊ณ ์„œ๋ฅผ ์ž‘์„ฑํ•˜์˜€๋‹ค. ์œ„์™€ ๊ฐ™์ด ์ธํ„ฐ๋„ท์„œ๋น„์Šค์ œ๊ณต์ž ๋ณด๊ด€ ์ด๋ฉ”์ผ ๊ณ„์ • ๋“ฑ์— ๋Œ€ํ•œ ์••์ˆ˜ยท์ˆ˜์ƒ‰์ ˆ์ฐจ์—๋Š” ์••์ˆ˜ยท์ˆ˜์ƒ‰ ๊ณผ์ •์—์„œ ์ด๋ฉ”์ผ ๋“ฑ์˜ ๋‚ด์šฉ์„ ์ผ์ผ์ด ํ™•์ธํ•˜์—ฌ ๋ฒ”์ฃ„์‚ฌ์‹ค๊ณผ ๊ด€๋ จ๋œ ์ด๋ฉ”์ผ์„ ์„ ๋ณ„ํ•œ ํ›„ ๋ฒ”์œ„๋ฅผ ์ •ํ•˜์—ฌ ์ถœ๋ ฅ ๋˜๋Š” ๋ณต์ œํ•˜๋Š” ๊ฒƒ์ด ํ˜„์žฅ์˜ ์‚ฌ์ •์ด๋‚˜ ์ „์ž์ •๋ณด์˜ ๋Œ€๋Ÿ‰์„ฑ ๋“ฑ์œผ๋กœ ์ธํ•˜์—ฌ ํ˜„์ €ํžˆ ๊ณค๋ž€ํ•œ ์˜ˆ์™ธ์  ์‚ฌ์ •์ด ์žˆ๋Š” ์ , ์ธํ„ฐ๋„ท์„œ๋น„์Šค์ œ๊ณต์ž ์†Œ์†์˜ ๋‹ด๋‹น ์ง์›์ด ์ˆ˜์‚ฌ๊ธฐ๊ด€์— ์••์ˆ˜๋Œ€์ƒ ์ด๋ฉ”์ผ ๋“ฑ์„ ์ œ๊ณตํ•˜๋Š” ๊ฒƒ์€ ์ •ํ˜•์ ์ธ ์—…๋ฌด๋กœ ๋ณด์—ฌ ๋‹ด๋‹น ์ง์›์˜ ์ฃผ๊ด€์— ๋”ฐ๋ฅธ ์„ ๋ณ„ ์ž‘์—…์ด ๊ฐœ์žฌ๋˜์ง€ ์•„๋‹ˆํ•˜๊ณ , ์ด๋Ÿฌํ•œ ์ง‘ํ–‰ ๋ฐฉ์‹์— ๋Œ€ํ•˜์—ฌ ์ง‘ํ–‰ ๋Œ€์ƒ์ž์ธ ์ธํ„ฐ๋„ท์„œ๋น„์Šค์ œ๊ณต์ž ์ธก์—์„œ ๊ทธ๋™์•ˆ ๋ณ„๋‹ค๋ฅธ ์ด์˜๋ฅผ ์ œ๊ธฐํ•˜์ง€ ์•Š์•˜๋˜ ์  ๋“ฑ์„ ๊ณ ๋ คํ•  ๋•Œ, ์ธํ„ฐ๋„ท์„œ๋น„์Šค์ œ๊ณต์ž ์†Œ์† ๋‹ด๋‹น ์ง์›์ด ์••์ˆ˜๋Œ€์ƒ ์ด๋ฉ”์ผ ๋“ฑ์„ ์ถ”์ถœํ•˜์—ฌ ์ด๋ฅผ DVD์— ์ €์žฅ, ๋ด‰์ธํ•˜์—ฌ ์ „๋‹ฌํ•˜๋˜, ๋ณ„๋„๋กœ ์ˆ˜์‚ฌ๊ธฐ๊ด€์— ์ด๋ฉ”์ผ๋กœ๋„ ์ „์†กํ•˜์—ฌ ๊ทธ์ค‘ ๋‹น์ดˆ์˜ ์˜์žฅ์—์„œ ์ œ์‹œํ•œ ๋ฒ”์ฃ„ ๊ด€๋ จ ํ•„์š”ํ•œ ์ •๋ณด๋งŒ์„ ์„ ๋ณ„ํ•˜์—ฌ ์ˆ˜์‚ฌ์— ์‚ฌ์šฉํ•˜๋„๋ก ํ•˜๋Š” ๋ฐฉ๋ฒ•์œผ๋กœ ์ด๋ฃจ์–ด์ง„ ์••์ˆ˜ยท์ˆ˜์ƒ‰์˜์žฅ์˜ ์ง‘ํ–‰์ด ํ—Œ๋ฒ•๊ณผ ํ˜•์‚ฌ์†Œ์†ก๋ฒ•์—์„œ ๊ทœ์ •ํ•œ ์˜์žฅ์ฃผ์˜ ์›์น™์ด๋‚˜ ๋น„๋ก€์˜ ์›์น™ ๋“ฑ์— ์œ„๋ฐ˜๋˜๊ณ  ์ ๋ฒ•์ ˆ์ฐจ๋ฅผ ์ค€์ˆ˜ํ•˜์ง€ ์•„๋‹ˆํ•˜์—ฌ ์œ„๋ฒ•ํ•˜๋‹ค๊ณ  ๋ณด๊ธฐ๋Š” ์–ด๋ ต๋‹ค. 2) ์›๋ณธ์˜ ๋™์ผ์„ฑยท๋ฌด๊ฒฐ์„ฑ์„ ์ธ์ •ํ•  ์ˆ˜ ์—†๋Š”์ง€ ์ด ๋ฒ•์›์˜ ๊ฒ€์ฆ ๊ณผ์ •์—์„œ DVD ๋ด‰์ธ์— ๋‚ ์ธํ•œ ์‚ฌ๋žŒ๊ณผ ์••์ˆ˜์ˆ˜์ƒ‰์˜์žฅ์— ์ง‘ํ–‰์ฐธ์—ฌ์ž๋กœ ์„œ๋ช…์„ ํ•œ ์‚ฌ๋žŒ์ด ๋‹ค๋ฅด๋‹ค๋Š” ์ ์€ ํ™•์ธํ•˜์ง€ ๋ชปํ•˜์˜€์œผ๋‚˜, ์›๋ณธ์˜ ๋ด‰์ธ์ด ํ™•์‹คํ•˜๊ฒŒ ๋˜์–ด ์žˆ๊ณ  ์ดํ›„ ๋œฏ์€ ํ”์ ์ด ์—†๋‹ค๋Š” ์‚ฌ์‹ค์€ ํ™•์ธํ•˜์˜€๋‹ค. ์„ค์‚ฌ ์œ„ ๋‘ ์‚ฌ๋žŒ์ด ๋‹ค๋ฅด๋‹ค๊ณ  ํ•˜๋”๋ผ๋„, ๊ณต์†Œ์™ธ 21 ํšŒ์‚ฌ ์ธก์—์„œ ๋ด‰์ธํ•˜์—ฌ ์ œ์ถœํ•œ DVD์˜ ๋ด‰์ธ์ด ์ด ๋ฒ•์ •๊นŒ์ง€ ๊ฐœ๋ด‰๋œ ํ”์  ์—†์ด ์ œ์ถœ์ด ๋˜์—ˆ๋‹ค๋ฉด ๊ทธ ์•ˆ์— ๋“ค์–ด์žˆ๋Š” ํŒŒ์ผ์€ ์••์ˆ˜ยท์ˆ˜์ƒ‰ ๋Œ€์ƒ ์›๋ณธํŒŒ์ผ์ž„์„ ์ธ์ •ํ•  ์ˆ˜ ์žˆ๊ณ , ์ด ๋ฒ•์ •์—์„œ ์›๋ณธ์„ ๊ฐœ๋ด‰ํ•˜์—ฌ ์›๋ณธ DVD์— ์žˆ๋Š” ํŒŒ์ผ๊ณผ ์ˆ˜์‚ฌ๊ด€์ด ๋ถ„์„์— ์‚ฌ์šฉํ•œ ํŒŒ์ผ์˜ ํ•ด์‹œ๊ฐ’์ด ๋™์ผํ•จ์„ ํ™•์ธํ•˜๊ณ  ๊ทธ ํŒŒ์ผ์˜ ๋‚ด์šฉ๋„ ๋™์ผํ•จ์„ ํ™•์ธํ•˜์˜€์œผ๋ฉฐ, ๊ฒ€์‚ฌ๊ฐ€ ์ถœ๋ ฅํ•˜์—ฌ ์ œ์ถœํ•œ ๋ฌธ์„œ๋„ ๊ทธ ์›๋ณธ ํŒŒ์ผ์˜ ๋‚ด์šฉ๊ณผ ๋™์ผํ•จ์„ ์ธ์ •ํ•  ์ˆ˜ ์žˆ๋‹ค. 3) ํ”ผ๊ณ ์ธ๊ณผ ๋ณ€ํ˜ธ์ธ์˜ ์ด ๋ถ€๋ถ„ ์ฃผ์žฅ๋„ ๋ฐ›์•„๋“ค์ด์ง€ ์•Š๋Š”๋‹ค. 18. ์ „๋ฌธ๋ฒ•์น™ ์œ„๋ฐฐ ์ฃผ์žฅ ๊ฐ€. ์ฃผ์žฅ 2016๊ณ ํ•ฉ558 ์‚ฌ๊ฑด์˜ ์ฆ๊ฑฐ๋ชฉ๋ก ์ˆœ๋ฒˆ ์ฃผ27) 425, ์ฃผ28) 432, ์ฃผ29) 436, ์ฃผ30) 448 ๋“ฑ ์ฆ๊ฑฐ๋“ค์€ ํšŒํ•ฉ, ํ†ต์‹ ์—ฐ๋ฝ, ๊ธˆํ’ˆ์ˆ˜์ˆ˜, ํŽธ์˜์ œ๊ณต์˜ ๋ฒ”์ฃ„์‚ฌ์‹ค์— ๊ด€ํ•˜์—ฌ๋Š” ์ „๋ฌธ๋ฒ•์น™์ด ์ ์šฉ๋˜๋Š” ์ฆ๊ฑฐ๋ผ๊ณ  ํ•  ๊ฒƒ์ด๋ฏ€๋กœ ์ž‘์„ฑ์ž๊ฐ€ ๋ˆ„๊ตฌ์ธ์ง€ ์•Œ ์ˆ˜ ์—†๊ณ , ์ž‘์„ฑ์ž๊ฐ€ ์„ฑ๋ฆฝ์˜ ์ง„์ •์„ ํ™•์ธํ•˜์ง€ ์•Š์€ ์ด ์‚ฌ๊ฑด์—์„œ ๊ทธ ์ฆ๊ฑฐ๋Šฅ๋ ฅ์ด ์—†๋‹ค. ๋‚˜. ๊ด€๋ จ๋ฒ•๋ฆฌ ํ”ผ๊ณ ์ธ ๋˜๋Š” ํ”ผ๊ณ ์ธ ์•„๋‹Œ ์‚ฌ๋žŒ์ด ์ •๋ณด์ €์žฅ๋งค์ฒด์— ์ž…๋ ฅํ•˜์—ฌ ๊ธฐ์–ต๋œ ๋ฌธ์ž์ •๋ณด ๋˜๋Š” ๊ทธ ์ถœ๋ ฅ๋ฌผ์„ ์ฆ๊ฑฐ๋กœ ์‚ฌ์šฉํ•˜๋Š” ๊ฒฝ์šฐ, ์ด๋Š” ์‹ค์งˆ์— ์žˆ์–ด์„œ ํ”ผ๊ณ ์ธ ๋˜๋Š” ํ”ผ๊ณ ์ธ ์•„๋‹Œ ์‚ฌ๋žŒ์ด ์ž‘์„ฑํ•œ ์ง„์ˆ ์„œ๋‚˜ ๊ทธ ์ง„์ˆ ์„ ๊ธฐ์žฌํ•œ ์„œ๋ฅ˜์™€ ํฌ๊ฒŒ ๋‹ค๋ฅผ ๋ฐ” ์—†๊ณ , ์••์ˆ˜ ํ›„์˜ ๋ณด๊ด€ ๋ฐ ์ถœ๋ ฅ๊ณผ์ •์— ์กฐ์ž‘์˜ ๊ฐ€๋Šฅ์„ฑ์ด ์žˆ์œผ๋ฉฐ, ๊ธฐ๋ณธ์ ์œผ๋กœ ๋ฐ˜๋Œ€์‹ ๋ฌธ์˜ ๊ธฐํšŒ๊ฐ€ ๋ณด์žฅ๋˜์ง€ ์•Š๋Š” ์  ๋“ฑ์— ๋น„์ถ”์–ด ๊ทธ ๋‚ด์šฉ์˜ ์ง„์‹ค์„ฑ์— ๊ด€ํ•˜์—ฌ๋Š” ์ „๋ฌธ๋ฒ•์น™์ด ์ ์šฉ๋˜๊ณ , ๋”ฐ๋ผ์„œ ์›์น™์ ์œผ๋กœ ํ˜•์‚ฌ์†Œ์†ก๋ฒ• ์ œ313์กฐ ์ œ1ํ•ญ์— ์˜ํ•˜์—ฌ ๊ทธ ์ž‘์„ฑ์ž ๋˜๋Š” ์ง„์ˆ ์ž์˜ ์ง„์ˆ ์— ์˜ํ•˜์—ฌ ์„ฑ๋ฆฝ์˜ ์ง„์ •ํ•จ์ด ์ฆ๋ช…๋œ ๋•Œ์— ํ•œํ•˜์—ฌ ์ด๋ฅผ ์ฆ๊ฑฐ๋กœ ์‚ฌ์šฉํ•  ์ˆ˜ ์žˆ๋‹ค. ๋‹ค๋งŒ, ์ •๋ณด์ €์žฅ๋งค์ฒด์— ๊ธฐ์–ต๋œ ๋ฌธ์ž์ •๋ณด์˜ ๋‚ด์šฉ์˜ ์ง„์‹ค์„ฑ์ด ์•„๋‹Œ ๊ทธ์™€ ๊ฐ™์€ ๋‚ด์šฉ์˜ ๋ฌธ์ž์ •๋ณด๊ฐ€ ์กด์žฌํ•˜๋Š” ๊ฒƒ ์ž์ฒด๊ฐ€ ์ฆ๊ฑฐ๋กœ ๋˜๋Š” ๊ฒฝ์šฐ์—๋Š” ์ „๋ฌธ๋ฒ•์น™์ด ์ ์šฉ๋˜์ง€ ์•„๋‹ˆํ•œ๋‹ค. ๋‚˜์•„๊ฐ€ ์–ด๋–ค ์ง„์ˆ ์„ ๋ฒ”์ฃ„์‚ฌ์‹ค์— ๋Œ€ํ•œ ์ง์ ‘์ฆ๊ฑฐ๋กœ ์‚ฌ์šฉํ•  ๋•Œ์—๋Š” ๊ทธ ์ง„์ˆ ์ด ์ „๋ฌธ์ฆ๊ฑฐ๊ฐ€ ๋œ๋‹ค๊ณ  ํ•˜๋”๋ผ๋„ ๊ทธ์™€ ๊ฐ™์€ ์ง„์ˆ ์„ ํ•˜์˜€๋‹ค๋Š” ๊ฒƒ ์ž์ฒด ๋˜๋Š” ๊ทธ ์ง„์ˆ ์˜ ์ง„์‹ค์„ฑ๊ณผ ๊ด€๊ณ„์—†๋Š” ๊ฐ„์ ‘์‚ฌ์‹ค์— ๋Œ€ํ•œ ์ •ํ™ฉ์ฆ๊ฑฐ๋กœ ์‚ฌ์šฉํ•  ๋•Œ์—๋Š” ๋ฐ˜๋“œ์‹œ ์ „๋ฌธ์ฆ๊ฑฐ๊ฐ€ ๋˜๋Š” ๊ฒƒ์€ ์•„๋‹ˆ๋‹ค(๋Œ€๋ฒ•์› 2013. 2. 15. ์„ ๊ณ  2010๋„3504 ํŒ๊ฒฐ, ๋Œ€๋ฒ•์› 2013. 7. 26. ์„ ๊ณ  2013๋„2511 ํŒ๊ฒฐ ๋“ฑ ์ฐธ์กฐ). ๋‹ค. ํŒ๋‹จ 1) ์œ„ ๋ฌธ์„œ๊ฐ€ ์ฆ๊ฑฐ๋กœ ์‚ฌ์šฉ๋˜๋Š” ํ†ต์‹ ์—ฐ๋ฝ ๋ฐ ํŽธ์˜์ œ๊ณต์˜ ๋ฒ”์ฃ„์‚ฌ์‹ค์€ ํ”ผ๊ณ ์ธ์ด ๋ฐ˜๊ตญ๊ฐ€๋‹จ์ฒด์˜ ๊ตฌ์„ฑ์›๊ณผ ๊ทธ ๋ฌธ๊ฑด์„ ์ฃผ๊ณ ๋ฐ›๋Š” ๋ฐฉ๋ฒ•์œผ๋กœ ํ†ต์‹ ํ•˜์˜€๋‹ค๊ฑฐ๋‚˜, ๋ฐ˜๊ตญ๊ฐ€๋‹จ์ฒด์˜ ํŽธ์˜๋ฅผ ์œ„ํ•˜์—ฌ ์ œ๊ณตํ•œ ๋ชฉ์ ๋ฌผ์ด ๊ทธ ๋ฌธ๊ฑด์ธ ๊ฒฝ์šฐ์ด๋‹ค. ์ด๋Ÿฌํ•œ ๊ฒฝ์šฐ ์š”์ฆ์‚ฌ์‹ค๊ณผ์˜ ๊ด€๊ณ„์—์„œ, ๋ฌธ๊ฑด ๋‚ด์šฉ์˜ ์ง„์‹ค์„ฑ์ด ๋ฌธ์ œ๋˜๋Š” ๊ฒƒ์ด ์•„๋‹ˆ๋ผ ๊ทธ๋Ÿฌํ•œ ๋‚ด์šฉ์˜ ๋ฌธ๊ฑด์ด ์กด์žฌํ•˜๋Š” ๊ฒƒ ์ž์ฒด๊ฐ€ ์ฆ๊ฑฐ๊ฐ€ ๋˜๋Š” ๊ฒฝ์šฐ ๋‚ด์ง€ ๊ทธ ์ฆ๊ฑฐ์˜ ์กด์žฌ๋‚˜ ๊ธฐ์žฌ๋‚ด์šฉ ์ž์ฒด๊ฐ€ ์š”์ฆ์‚ฌ์‹ค์ธ ๋ฒ”ํ–‰์˜ ๊ตฌ์„ฑ์š”์†Œ๋ฅผ ์ด๋ฃจ๋Š” ๊ฒฝ์šฐ(์ด๋ฅธ๋ฐ” โ€˜๋ณธ๋ž˜์ฆ๊ฑฐโ€™)๋กœ์„œ, ์ด๋Ÿฌํ•œ ์ฆ๊ฑฐ์— ๋Œ€ํ•˜์—ฌ๋Š” ์ „๋ฌธ๋ฒ•์น™์ด ์ ์šฉ๋˜์ง€ ์•Š๋Š”๋‹ค๊ณ  ๋ณด์•„์•ผ ํ•œ๋‹ค. ํ”ผ๊ณ ์ธ ๋ฐ ๋ณ€ํ˜ธ์ธ์˜ ์ด ๋ถ€๋ถ„ ์ฃผ์žฅ์€ ๋ฐ›์•„๋“ค์ด์ง€ ์•Š๋Š”๋‹ค. 2) ํšŒํ•ฉ๊ณผ ๊ธˆํ’ˆ์ˆ˜์ˆ˜์˜ ๊ฒฝ์šฐ์— ์œ„ ๋ฌธ๊ฑด์˜ ๋‚ด์šฉ์˜ ์ง„์‹ค์„ฑ์„ ์ฆ๊ฑฐ๋กœ ์‚ฌ์šฉํ•˜๋Š” ๊ฒƒ์€ ์ „๋ฌธ๋ฒ•์น™์˜ ์ ์šฉ์„ ๋ฐ›๋Š”๋‹ค๊ณ  ๋ณผ ์ˆ˜ ์žˆ์„ ๊ฒƒ์ด๋‚˜, ๊ทธ๋Ÿฌํ•œ ๋ฌธ์„œ ํŒŒ์ผ์ด ์ž‘์„ฑ๋˜์–ด ์กด์žฌํ•œ๋‹ค๋Š” ์‚ฌ์‹ค ๋ฐ ๊ทธ ๊ธฐ์žฌ๋‚ด์šฉ์˜ ์‹ค์ œ ์ง„์‹ค์„ฑ๊ณผ ๊ด€๊ณ„์—†์ด ๊ฐ„์ ‘์‚ฌ์‹ค์— ๋Œ€ํ•œ ์ •ํ™ฉ์ฆ๊ฑฐ๋กœ์„œ์˜ ์ฆ๊ฑฐ๋Šฅ๋ ฅ์€ ์ธ์ •ํ•  ์ˆ˜ ์žˆ๋Š” ๊ฒƒ์ด๋ฏ€๋กœ ํšŒํ•ฉยท๊ธˆํ’ˆ์ˆ˜์ˆ˜ ๋ฒ”์ฃ„์‚ฌ์‹ค๊ณผ ๊ด€๋ จํ•˜์—ฌ์„œ๋Š” ์œ„์™€ ๊ฐ™์€ ์˜๋ฏธ์˜ ๋น„์ง„์ˆ  ๋‚ด์ง€ ๋น„์ „๋ฌธ์ฆ๊ฑฐ๋กœ์„œ์˜ ์ฆ๊ฑฐ๋Šฅ๋ ฅ๋งŒ์„ ์ธ์ •ํ•˜๊ธฐ๋กœ ํ•œ๋‹ค. [๊ฐœ๋ณ„ ๋ฒ”์ฃ„์‚ฌ์‹ค ๊ด€๋ จ] 19. ์ด์ ํ‘œํ˜„๋ฌผ ์†Œ์ง€๋กœ ์ธํ•œ ๊ตญ๊ฐ€๋ณด์•ˆ๋ฒ•์œ„๋ฐ˜(์ฐฌ์–‘ยท๊ณ ๋ฌด๋“ฑ)์˜ ์ ์— ๊ด€ํ•˜์—ฌ - ํŒ์‹œ โ… . ์ œ3ํ•ญ, โ…ก. ์ œ3์˜ ๋‚˜. ๋‹ค. ๋ฐ”. ์‚ฌ.ํ•ญ ๊ฐ€. ์ฃผ์žฅ ํ”ผ๊ณ ์ธ์€ ๊ณต์†Œ์ œ๊ธฐ๋œ ํ‘œํ˜„๋ฌผ๋“ค์„ ์†Œ์ง€ํ•œ ์ ์ด ์—†๊ณ , ๊ทธ ๋ฌธ๊ฑด์˜ ์ด์ ์„ฑ ๋“ฑ๋„ ์ธ์ •ํ•  ์ˆ˜ ์—†๋‹ค. ๋‚˜. ๊ด€๋ จ๋ฒ•๋ฆฌ 1) ๊ตญ๊ฐ€๋ณด์•ˆ๋ฒ• ์ œ7์กฐ ์ œ5ํ•ญ์˜ ์ด์ ํ‘œํ˜„๋ฌผ๋กœ ์ธ์ •๋˜๊ธฐ ์œ„ํ•ด์„œ๋Š” ๊ทธ ํ‘œํ˜„๋ฌผ์˜ ๋‚ด์šฉ์ด ๊ตญ๊ฐ€๋ณด์•ˆ๋ฒ•์˜ ๋ณดํ˜ธ๋ฒ•์ต์ธ ๊ตญ๊ฐ€์˜ ์กด๋ฆฝยท์•ˆ์ „๊ณผ ์ž์œ ๋ฏผ์ฃผ์  ๊ธฐ๋ณธ์งˆ์„œ๋ฅผ ์œ„ํ˜‘ํ•˜๋Š” ์ ๊ทน์ ์ด๊ณ  ๊ณต๊ฒฉ์ ์ธ ๊ฒƒ์ด์–ด์•ผ ํ•œ๋‹ค. ๊ทธ๋ฆฌ๊ณ  ํ‘œํ˜„๋ฌผ์— ์ด์™€ ๊ฐ™์€ ์ด์ ์„ฑ์ด ์žˆ๋Š”์ง€ ์—ฌ๋ถ€๋Š” ํ‘œํ˜„๋ฌผ์˜ ์ „์ฒด์ ์ธ ๋‚ด์šฉ๋ฟ๋งŒ ์•„๋‹ˆ๋ผ ๊ทธ ์ž‘์„ฑ์˜ ๋™๊ธฐ๋Š” ๋ฌผ๋ก  ํ‘œํ˜„ํ–‰์œ„ ์ž์ฒด์˜ ํƒœ์–‘ ๋ฐ ์™ธ๋ถ€์™€์˜ ๊ด€๋ จ ์‚ฌํ•ญ, ํ‘œํ˜„ํ–‰์œ„ ๋‹น์‹œ์˜ ์ •ํ™ฉ ๋“ฑ ์ œ๋ฐ˜ ์‚ฌ์ •์„ ์ข…ํ•ฉํ•˜์—ฌ ๊ฒฐ์ •ํ•˜์—ฌ์•ผ ํ•˜๋ฉฐ, ํ•ด๋‹น ํ‘œํ˜„๋ฌผ์˜ ์–ด๋Š ํ‘œํ˜„ ํ•˜๋‚˜๋งŒ์„ ๋”ฐ๋กœ ๋–ผ์–ด ๋†“๊ณ  ๋ณผ ๊ฒƒ์ด ์•„๋‹ˆ๋ผ ๋ฌธ๋งฅ์„ ํ†ตํ•ด ๊ทธ ์ „์ฒด์  ๋‚ด์šฉ์„ ๊ฐ๊ด€์ ์œผ๋กœ ๋ถ„์„ํ•˜์—ฌ ์ด์ ์„ฑ ์œ ๋ฌด๋ฅผ ํŒ๋‹จํ•˜์—ฌ์•ผ ํ•œ๋‹ค(๋Œ€๋ฒ•์› 2013. 3. 28. ์„ ๊ณ  2010๋„12836 ํŒ๊ฒฐ, ๋Œ€๋ฒ•์› 2015. 1. 22. ์„ ๊ณ  2014๋„10978 ์ „์›ํ•ฉ์˜์ฒด ํŒ๊ฒฐ ๋“ฑ ์ฐธ์กฐ). 2) ๋˜ํ•œ ๊ทธ๋Ÿฌํ•œ ํ‘œํ˜„๋ฌผ์ด ํ•ฉ๋ฒ•์ ์œผ๋กœ ์ถœํŒยทํŒ๋งค๋˜์–ด ์žˆ๋Š” ๊ฒƒ์ด๋ผ ํ•˜๋”๋ผ๋„ ๊ตญ๊ฐ€๋ณด์•ˆ๋ฒ• ์ œ7์กฐ ์ œ5ํ•ญ, ์ œ1ํ•ญ์˜ ๊ตฌ์„ฑ์š”๊ฑด์„ ์ถฉ์กฑํ•จ์— ์ง€์žฅ์ด ์—†๊ณ (๋Œ€๋ฒ•์› 2009. 10. 29. ์„ ๊ณ  2008๋„2912 ํŒ๊ฒฐ ์ฐธ์กฐ), ํ‘œํ˜„๋ฌผ์˜ ๋‚ด์šฉ์ด ์ผ๋ฐ˜์ธ์—๊ฒŒ ๊ณต๊ฐœ๋œ ์„œ์ ์ด๋‚˜ ์ธํ„ฐ๋„ท์‚ฌ์ดํŠธ ๋“ฑ์—์„œ ์ˆ˜์ง‘ยท์ธ์šฉ๋˜์—ˆ๋‹ค๋Š” ์ด์œ ๋งŒ์œผ๋กœ ๊ทธ์— ๋Œ€ํ•œ ํ‰๊ฐ€๊ฐ€ ๋‹ฌ๋ผ์ ธ์•ผ ํ•˜๋Š” ๊ฒƒ๋„ ์•„๋‹ˆ๋‹ค(๋Œ€๋ฒ•์› 2006. 6. 16. ์„ ๊ณ  2004๋„851 ํŒ๊ฒฐ ์ฐธ์กฐ). 3) ๋ถํ•œ์›์ „์†Œ์„ค ๋“ฑ์˜ ์ค„๊ฑฐ๋ฆฌ๋‚˜ ์ฃผ์š” ๋‚ด์šฉ์ด ๋ฐ˜๋“œ์‹œ ์ง์ ‘์ ์œผ๋กœ ๋ฐ˜๊ตญ๊ฐ€๋‹จ์ฒด์ธ ๋ถํ•œ์˜ ๋Œ€๋‚จ ์„ ์ „ยท์„ ๋™ ๋“ฑ์˜ ํ–‰์œ„์— ๋™์กฐํ•˜๋Š” ๊ฒƒ์ด ์•„๋‹ˆ๋ผ ํ•˜๋”๋ผ๋„, ๊ทธ ๋‚ด์šฉ ์ค‘์˜ ์ผ๋ถ€๊ฐ€ ์‚ฌํšŒ์ฃผ์˜์ž์˜ ์ž…์žฅ์—์„œ ๋ฐ”๋ผ๋ณธ ์„ธ๊ณ„๊ด€๊ณผ ์—ญ์‚ฌ๊ด€์„ ์„œ์ˆ ํ•œ ๊ฒƒ์ด๊ณ  ๋ถํ•œ์˜ ์ฃผ์ฒด์‚ฌ์ƒ๊ณผ ์‚ฌํšŒ์  ์• ๊ตญ์ฃผ์˜๋กœ ๋ฌด์žฅํ•œ ๋ฐ”๋žŒ์งํ•œ ์ธ๊ฐ„์ƒ์˜ ์ „ํ˜•์„ ์ œ์‹œํ•˜๊ณ  ์žˆ์œผ๋ฉฐ ๊น€์ผ์„ฑ์„ '์œ„๋Œ€ํ•œ ์ˆ˜๋ น'์œผ๋กœ ํ‘œํ˜„ํ•˜๋Š” ๋“ฑ ๊น€์ผ์„ฑ ๊ฐœ์ธ๊ณผ ๊ทธ ๊ฐ€๊ณ„๋ฅผ ์ฐฌ์–‘, ๋ฏธํ™”ํ•˜๊ณ  ์žˆ๋‹ค๋ฉด, ๊ทธ์™€ ๊ฐ™์€ ์†Œ์„ค ๋“ฑ์€ ๋Œ€ํ•œ๋ฏผ๊ตญ์˜ ์•ˆ์ „๊ณผ ์ž์œ ๋ฏผ์ฃผ์ฃผ์˜์ฒด์ œ๋ฅผ ์œ„ํƒœ๋กญ๊ฒŒ ํ•˜๋Š” ์ ๊ทน์ ์ธ ๋‚ด์šฉ์„ ๋‹ด๊ณ  ์žˆ๋‹ค๊ณ  ๋ณด์•„์•ผ ํ•  ๊ฒƒ์ด๋ฏ€๋กœ ๊ตญ๊ฐ€๋ณด์•ˆ๋ฒ• ์ œ7์กฐ ์ œ5ํ•ญ ์†Œ์ •์˜ ํ‘œํ˜„๋ฌผ์— ํ•ด๋‹นํ•œ๋‹ค(๋Œ€๋ฒ•์› 1998. 5. 22. ์„ ๊ณ  95๋„1152 ํŒ๊ฒฐ ์ฐธ์กฐ). 4) ๊ตญ๊ฐ€๋ณด์•ˆ๋ฒ• ์ œ7์กฐ ์ œ5ํ•ญ์—์„œ ๊ทœ์ •ํ•˜๊ณ  ์žˆ๋Š” โ€˜์†Œ์ง€โ€™๋Š” ํ‘œํ˜„๋ฌผ์„ ์ž๊ธฐ์˜ ์‚ฌ์‹ค์ƒ์˜ ์ง€๋ฐฐํ•˜์— ๋‘๋Š” ๊ฒƒ์„ ๋งํ•˜๋Š”๋ฐ”, ๊ทธ ํ‘œํ˜„๋ฌผ์ด ํ”ผ๊ณ ์ธ์˜ ์‚ฌ์‹ค์ƒ์˜ ์ง€๋ฐฐํ•˜์— ์žˆ์—ˆ๋Š”์ง€๋Š” ๋ณด๊ด€๋˜์–ด ์žˆ๋˜ ์žฅ์†Œ์˜ ๊ทœ๋ชจ ๋ฐ ํ˜•์ƒ, ์šด์˜ยท๊ด€๋ฆฌ ํ˜•ํƒœ, ๊ฐ ํ‘œํ˜„๋ฌผ์˜ ๋ณด๊ด€์ƒํƒœ ๋“ฑ๊ณผ ๊ฐ™์€ ์‚ฌ์ •๋“ค์„ ์ข…ํ•ฉํ•˜์—ฌ ํŒ๋‹จํ•˜์—ฌ์•ผ ํ•  ๊ฒƒ์ด๋‹ค. 5) ๊ตญ๊ฐ€๋ณด์•ˆ๋ฒ• ์ œ7์กฐ ์ œ5ํ•ญ์˜ ์ฃ„๋Š” ๊ฐ™์€ ์กฐ ์ œ1, 3, 4ํ•ญ์— ๊ทœ์ •๋œ ์ด์ ํ–‰์œ„๋ฅผ ํ•  ๋ชฉ์ ์œผ๋กœ ๋ฌธ์„œยท๋„ํ™” ๊ธฐํƒ€์˜ ํ‘œํ˜„๋ฌผ์„ ์ œ์ž‘ยท์ˆ˜์ž…ยท๋ณต์‚ฌยท์†Œ์ง€ยท์šด๋ฐ˜ยท๋ฐ˜ํฌยทํŒ๋งค ๋˜๋Š” ์ทจ๋“ํ•จ์œผ๋กœ์จ ์„ฑ๋ฆฝํ•˜๋Š” ๋ฒ”์ฃ„๋กœ์„œ ์ด๋ฅธ๋ฐ” ๋ชฉ์ ๋ฒ”์— ํ•ด๋‹นํ•˜๊ณ , ๋ชฉ์ ๋ฒ”์—์„œ์˜ ๋ชฉ์ ์€ ๋ฒ”์ฃ„ ์„ฑ๋ฆฝ์„ ์œ„ํ•œ ์ดˆ๊ณผ์ฃผ๊ด€์  ์œ„๋ฒ•์š”์†Œ๋กœ์„œ ๊ณ ์˜ ์™ธ์— ๋ณ„๋„๋กœ ์š”๊ตฌ๋˜๋Š” ๊ฒƒ์ด๋ฉฐ, ํ˜•์‚ฌ์žฌํŒ์—์„œ ๊ณต์†Œ๊ฐ€ ์ œ๊ธฐ๋œ ๋ฒ”์ฃ„์˜ ๊ตฌ์„ฑ์š”๊ฑด์„ ์ด๋ฃจ๋Š” ์‚ฌ์‹ค์— ๋Œ€ํ•œ ์ฆ๋ช…์ฑ…์ž„์€ ๊ฒ€์‚ฌ์—๊ฒŒ ์žˆ์œผ๋ฏ€๋กœ, ๊ตญ๊ฐ€๋ณด์•ˆ๋ฒ• ์ œ7์กฐ ์ œ5ํ•ญ ๋ฒ”์ฃ„์˜ ์„ฑ๋ฆฝ์„ ์ธ์ •ํ•  ์ˆ˜ ์žˆ์œผ๋ ค๋ฉด ๊ฒ€์‚ฌ๊ฐ€ ํ–‰์œ„์ž์—๊ฒŒ ์ด์ ํ–‰์œ„๋ฅผ ํ•  ๋ชฉ์ ์ด ์žˆ์—ˆ๋‹ค๋Š” ์ ์„ ์ฆ๋ช…ํ•˜์—ฌ์•ผ ํ•œ๋‹ค. ์ด ๊ฒฝ์šฐ ํ–‰์œ„์ž๊ฐ€ ์ด์ ํ‘œํ˜„๋ฌผ์ž„์„ ์ธ์‹ํ•˜๊ณ  ์ด์™€ ๊ด€๋ จํ•˜์—ฌ ๊ตญ๊ฐ€๋ณด์•ˆ๋ฒ• ์ œ7์กฐ ์ œ5ํ•ญ์—์„œ ์ •ํ•œ ์ œ์ž‘ยท์ˆ˜์ž…ยท๋ณต์‚ฌยท์†Œ์ง€ยท์šด๋ฐ˜ยท๋ฐ˜ํฌยทํŒ๋งค ๋˜๋Š” ์ทจ๋“ ๋“ฑ์˜ ํ–‰์œ„๋ฅผ ํ•˜์˜€๋‹ค๋Š” ์‚ฌ์‹ค๋งŒ์œผ๋กœ ๊ทธ์—๊ฒŒ ์ด์ ํ–‰์œ„๋ฅผ ํ•  ๋ชฉ์ ์ด ์žˆ์—ˆ๋‹ค๊ณ  ์ถ”์ •ํ•  ์ˆ˜๋Š” ์—†์ง€๋งŒ, ํ–‰์œ„์ž์—๊ฒŒ ์ด์ ํ–‰์œ„ ๋ชฉ์ ์ด ์žˆ์Œ์„ ์ฆ๋ช…ํ•  ์ง์ ‘ ์ฆ๊ฑฐ๊ฐ€ ์—†๋Š” ๋•Œ์—๋Š” ์•ž์—์„œ ๋ณธ ํ‘œํ˜„๋ฌผ์˜ ์ด์ ์„ฑ์˜ ์ง•ํ‘œ๊ฐ€ ๋˜๋Š” ์—ฌ๋Ÿฌ ์‚ฌ์ •์— ๋”ํ•˜์—ฌ ํ–‰์œ„์ž์˜ ๊ฒฝ๋ ฅ๊ณผ ์ง€์œ„, ํ–‰์œ„์ž๊ฐ€ ์ด์ ํ‘œํ˜„๋ฌผ๊ณผ ๊ด€๋ จํ•˜์—ฌ ์œ„ ๊ทœ์ •์˜ ํ–‰์œ„๋ฅผ ํ•˜๊ฒŒ ๋œ ๊ฒฝ์œ„, ํ–‰์œ„์ž์˜ ์ด์ ๋‹จ์ฒด ๊ฐ€์ž… ์—ฌ๋ถ€ ๋ฐ ์ด์ ํ‘œํ˜„๋ฌผ๊ณผ ํ–‰์œ„์ž๊ฐ€ ์†Œ์†ํ•œ ์ด์ ๋‹จ์ฒด์˜ ์‹ค์งˆ์ ์ธ ๋ชฉํ‘œ ๋ฐ ํ™œ๋™๊ณผ์˜ ์—ฐ๊ด€์„ฑ ๋“ฑ ๊ฐ„์ ‘์‚ฌ์‹ค์„ ์ข…ํ•ฉ์ ์œผ๋กœ ๊ณ ๋ คํ•˜์—ฌ ํ–‰์œ„์ž์˜ ์ด์ ํ–‰์œ„ ๋ชฉ์  ์—ฌ๋ถ€๋ฅผ ํŒ๋‹จํ•  ์ˆ˜ ์žˆ๋‹ค(๋Œ€๋ฒ•์› 2013. 3. 28. ์„ ๊ณ  2010๋„12836 ํŒ๊ฒฐ ๋“ฑ ์ฐธ์กฐ). ๋‹ค. ํ”ผ๊ณ ์ธ์˜ ์ด์ ํ‘œํ˜„๋ฌผ ์†Œ์ง€์— ๊ด€ํ•œ ํŒ๋‹จ ์ด ์‚ฌ๊ฑด ์ด์ ํ‘œํ˜„๋ฌผ๋“ค์€ ์ฆ ์ œ122ํ˜ธ[Micro SD ์นด๋“œ(16G, ์‚ผ์„ฑ EVO, MBMPAGVDDDCE-P)], ์ œ128ํ˜ธ[USB(8G, Sandisk Blade Cruzer], ์ œ129ํ˜ธ[USB(32G, DUCO)], ์ œ131ํ˜ธ[ํ•˜๋“œ๋””์Šคํฌ(40G, โ—‹โ—‹โ—‹)], ์ œ132ํ˜ธ[์‚ผ์„ฑ ์™ธ์žฅํ•˜๋“œ S2(500G)], ํ”ผ๊ณ ์ธ์˜ ํƒœ๋ธ”๋ฆฟ PC, ์ฆ ์ œ28ํ˜ธ[DVD-R(2010. 7. 22. ๊ณต์†Œ์™ธ 21 ์ฃผ์‹ํšŒ์‚ฌ)]์— ์ €์žฅ๋˜์–ด ์žˆ๋˜ ํŒŒ์ผ๋“ค ๋ฐ ํ”ผ๊ณ ์ธ์˜ ์ฃผ๊ฑฐ์ง€์—์„œ ์••์ˆ˜ํ•œ ์ถœ๋ ฅ๋ฌผ[์ฆ ์ œ107ํ˜ธ ๋ฌธ๊ฑด(A4 18๋งค)]์ด๊ณ , ์œ„ ์••์ˆ˜๋ฌผ๋“ค์€ ํ”ผ๊ณ ์ธ์˜ ๋ฐฐํƒ€์  ์ง€๋ฐฐ์˜์—ญ[ํ”ผ๊ณ ์ธ์˜ ์ง€๊ฐ‘(์ œ122ํ˜ธ), ํ”ผ๊ณ ์ธ์˜ ์ฐจ๋Ÿ‰ ๋‚ด ์ˆ˜๋‚ฉ๊ณต๊ฐ„(์ œ128ํ˜ธ), ํ”ผ๊ณ ์ธ ์ฃผ๊ฑฐ์ง€ ๊ฑฐ์‹ค(์ œ129, 131ํ˜ธ), ํ”ผ๊ณ ์ธ์˜ ์˜คํ† ๋ฐ”์ด ์•ˆ์žฅ ๋ฐ‘ ๋ณด๊ด€ํ•จ(์ œ132ํ˜ธ)]์—์„œ ๋ฐœ๊ฒฌ๋˜์—ˆ๊ฑฐ๋‚˜, ํ”ผ๊ณ ์ธ์˜ ํƒœ๋ธ”๋ฆฟ PC, ํ”ผ๊ณ ์ธ ๋ช…์˜๋กœ ๊ฐ€์ž…๋œ โ—โ—โ—โ—โ—์— ์ €์žฅ๋œ ํŒŒ์ผ(์ œ28ํ˜ธ)์ด๊ฑฐ๋‚˜, ํ”ผ๊ณ ์ธ์˜ ์ฃผ๊ฑฐ์ง€ ๊ฑฐ์‹ค ๋ฒฝ๋ฉด ์ฑ…์žฅ์—์„œ ์••์ˆ˜๋œ ๊ฒƒ(์ œ107ํ˜ธ)์œผ๋กœ์„œ ํ”ผ๊ณ ์ธ์ด ์ด๋ฅผ ์†Œ์ง€ํ•˜๊ณ  ์žˆ์—ˆ๋˜ ์‚ฌ์‹ค์„ ๋„‰๋„‰ํžˆ ์ธ์ •ํ•  ์ˆ˜ ์žˆ๋‹ค. ๋ผ. ์ด ์‚ฌ๊ฑด ๋ฌธ๊ฑด๋“ค์˜ ์ด์ ์„ฑ์— ๊ด€ํ•œ ํŒ๋‹จ ์ด ๋ฒ•์›์ด ์ฑ„ํƒํ•˜์—ฌ ์กฐ์‚ฌํ•œ ์ฆ๊ฑฐ๋“ค์— ์˜ํ•˜์—ฌ ์ธ์ •๋˜๋Š” ๋‹ค์Œ๊ณผ ๊ฐ™์€ ์‚ฌ์ •, ์ฆ‰ ์œ„ ๊ฐ ๋ฌธ๊ฑด, ์Œ์› ๋ฐ ์˜์ƒ ํŒŒ์ผ์€ ํŒ์‹œ โ… . ์ œ3ํ•ญ๊ณผ โ…ก. ์ œ3์˜ ๋‚˜. ๋‹ค. ๋ฐ”. ์‚ฌ.ํ•ญ ๊ธฐ์žฌ ๊ฐ ๋ฒ”์ฃ„์‚ฌ์‹ค ๋ฐ [๋ณ„์ง€ 2] ์ด์ ํ‘œํ˜„๋ฌผ ์ฃผ์š”๋‚ด์šฉ์—์„œ ๊ธฐ์žฌํ•œ ๋ฐ”์™€ ๊ฐ™์ด ๊ทธ ์ฃผ๋œ ๋‚ด์šฉ์ด ๋ถํ•œ์˜ ์ฃผ์ฒด์‚ฌ์ƒ, ์„ ๊ตฐ์ •์น˜ ๋“ฑ์„ ์˜นํ˜ธยท์ฐฌ์–‘ํ•˜๊ณ , ๋ถํ•œ์˜ ๋Œ€๋‚จํ˜๋ช…๋ก ์ธ โ€˜๋ฏผ์กฑํ•ด๋ฐฉ๋ฏผ์ค‘๋ฏผ์ฃผ์ฃผ์˜ํ˜๋ช…๋ก (NLPDR)โ€™์„ ์„ ์ „ยท์„ ๋™ํ•˜๋ฉฐ, ๋Œ€ํ•œ๋ฏผ๊ตญ์„ ๋ฏธ์ œ๊ตญ์ฃผ์˜์˜ ์‹๋ฏผ์ง€๋กœ ๊ทœ์ •ํ•˜๊ณ , ๋ฐ˜๋ฏธ์ž์ฃผํ™”ํˆฌ์Ÿ์„ ์œ„ํ•œ ์ฃผํ•œ๋ฏธ๊ตฐ์ฒ ์ˆ˜, ๊ตญ๊ฐ€๋ณด์•ˆ๋ฒ•์ฒ ํ, ์—ฐ๋ฐฉ์ œํ†ต์ผ ๋“ฑ ๋ถํ•œ์˜ ์ฃผ์žฅ์„ ๋ฏธํ™”ยท์ฐฌ์–‘ํ•˜๊ฑฐ๋‚˜ ์„ ์ „ยท์„ ๋™ํ•˜๋Š” ๊ฒƒ์ธ ์ , ๊ทธ ๋‚ด์šฉ๋ฟ ์•„๋‹ˆ๋ผ ํ‘œํ˜„์˜ ๋ฐฉ์‹์ด ๋ถํ•œ์˜ ๊ทธ๊ฒƒ์„ ์ถ”์ข…ํ•˜๋Š” ๋“ฑ ๊ณผ๊ฒฉํ•˜๊ณ  ์„ ๋™์ ์ธ ๊ฒƒ์ธ ์  ๋“ฑ์— ๋น„์ถ”์–ด, ํŒ์‹œ ๊ฐ ํ‘œํ˜„๋ฌผ์€ ๋Œ€ํ•œ๋ฏผ๊ตญ์˜ ์กด๋ฆฝยท์•ˆ์ „๊ณผ ์ž์œ ๋ฏผ์ฃผ์ฃผ์˜ ์ฒด์ œ๋ฅผ ์œ„ํ˜‘ํ•˜๋Š” ์ ๊ทน์ ์ด๊ณ  ๊ณต๊ฒฉ์ ์ธ ๊ฒƒ์œผ๋กœ ํ‘œํ˜„์˜ ์ž์œ ์˜ ํ•œ๊ณ„๋ฅผ ๋ฒ—์–ด๋‚œ ์ด์ ํ‘œํ˜„๋ฌผ์ด๋ผ๊ณ  ์ถฉ๋ถ„ํžˆ ์ธ์ •๋œ๋‹ค. ๋งˆ. ์ด์ ํ–‰์œ„๋ฅผ ํ•  ๋ชฉ์ ์ด ์žˆ์—ˆ๋Š”์ง€ ์—ฌ๋ถ€ ์•ž์„œ ๋ณธ ๋ฒ•๋ฆฌ์—๋‹ค๊ฐ€ ์ด ๋ฒ•์›์ด ์ฑ„ํƒํ•˜์—ฌ ์กฐ์‚ฌํ•œ ์ฆ๊ฑฐ๋“ค์— ์˜ํ•˜์—ฌ ์ธ์ •ํ•  ์ˆ˜ ์žˆ๋Š” ๋‹ค์Œ๊ณผ ๊ฐ™์€ ์‚ฌ์ •์„ ์ข…ํ•ฉํ•˜๋ฉด, ํ”ผ๊ณ ์ธ์€ ์œ„ ์ด์ ํ‘œํ˜„๋ฌผ์„ ์†Œ์ง€ํ•œ๋‹ค๋Š” ๊ณ ์˜ ์™ธ์—๋„ ์ด์ ํ–‰์œ„๋ฅผ ํ•  ๋ชฉ์ ์ด ์žˆ์—ˆ๋‹ค๊ณ  ์ธ์ •ํ•  ์ˆ˜ ์žˆ๋‹ค. 1) ์•ž์„œ ๋ณธ ๊ฒƒ์ฒ˜๋Ÿผ ์œ„ ํ‘œํ˜„๋ฌผ๋“ค์„ ์ผ๊ฒฌํ•˜์—ฌ ๋ณด์•„๋„ ๊ทธ ์ด์ ์„ฑ์ด ์ถฉ๋ถ„ํžˆ ์ธ์ •๋œ๋‹ค. 2) ํ”ผ๊ณ ์ธ์€ ํŒ์‹œ ๋ฒ”์ฃ„์‚ฌ์‹ค ์ค‘ ๋ชจ๋‘์‚ฌ์‹ค(ํ”ผ๊ณ ์ธ์˜ ๊ฒฝ๋ ฅ) ๊ธฐ์žฌ์™€ ๊ฐ™์ด ํ™œ๋™ํ•˜์—ฌ ์™”๋‹ค. 3) ํ”ผ๊ณ ์ธ์€ 2013. 11.๊ฒฝ๋ถ€ํ„ฐ 2014. 10.๊ฒฝ๊นŒ์ง€ ์‚ฌ์ด์— ๊ณต์†Œ์™ธ 6, ๊ณต์†Œ์™ธ 4, ๊ณต์†Œ์™ธ 5์™€ ๋ถํ•œ์˜ ์ฃผ์ฒด์‚ฌ์ƒ ๋“ฑ๊ณผ ๊ด€๋ จ๋œ ์‚ฌ์ƒํ•™์Šต ๋ชจ์ž„์„ ์ •๊ธฐ์ ์œผ๋กœ ๊ฐ€์กŒ๊ณ , ๊ทธ ๋ชจ์ž„์—์„œ ์œ„ ์ด์ ํ‘œํ˜„๋ฌผ๋“ค ์ค‘ ์ผ๋ถ€๋ฅผ ์ฝ๊ณ  ๋ถํ•œ์˜ ์ฃผ์žฅ์— ๋™์กฐํ•˜๊ฑฐ๋‚˜ ์ด์ ํ‘œํ˜„๋ฌผ์„ ์ „ํŒŒํ•˜๋Š” ๋“ฑ์˜ ํ–‰์œ„๋ฅผ ํ•˜์˜€๋‹ค. 4) ๋˜ํ•œ ํ”ผ๊ณ ์ธ์€ ๋Œ€๋‚จ๊ณต์ž‘์›๊ณผ ํ•ด์™ธ์—์„œ ํšŒํ•ฉํ•˜์˜€๊ณ  ๋Œ€ํ•œ๋ฏผ๊ตญ ๋‚ด ๋ถํ•œ์„ ์ถ”์ข…ํ•˜๋Š” ์กฐ์ง์˜ ์ผ์›(์ด์ฑ…)์œผ๋กœ ํ™œ๋™ํ•˜์˜€์œผ๋ฉฐ ๊ทธ ํ˜„ํ™ฉ์„ ๋ถํ•œ์— ๋ณด๊ณ ํ•˜๋Š” ๋“ฑ์˜ ํ–‰์œ„๋ฅผ ํ•˜๊ธฐ๋„ ํ•˜์˜€๋‹ค. ๋ฐ”. ์†Œ๊ฒฐ ํ”ผ๊ณ ์ธ๊ณผ ๋ณ€ํ˜ธ์ธ๋“ค์˜ ์ด ๋ถ€๋ถ„ ์ฃผ์žฅ์€ ์ด์œ  ์—†๋‹ค. 20. 2011. 4. 21., 2012. 5. 31. ๊ฐ ๊ตญ๊ฐ€๋ณด์•ˆ๋ฒ•์œ„๋ฐ˜(ํšŒํ•ฉยทํ†ต์‹ )์˜ ์ ์— ๊ด€ํ•˜์—ฌ ๊ฐ€. ์ฃผ์žฅ 1) ํ”ผ๊ณ ์ธ์€ ์œ„ ๊ฐ ์ผ์‹œ์— ๋ฐ˜๊ตญ๊ฐ€๋‹จ์ฒด ๊ตฌ์„ฑ์›๊ณผ ํšŒํ•ฉํ•œ ์‚ฌ์‹ค์ด ์—†๋‹ค. 2) ํ”ผ๊ณ ์ธ๊ณผ ๊ณต์†Œ์™ธ 6์ด ๋งŒ๋‚œ ์‚ฌ๋žŒ์„ ๋ถํ•œ๊ณต์ž‘์› โ˜†์ง€๋„์›, โ–ทโ–ทโ–ท(๋˜๋Š” โ–ทโ™คโ™ค์ฃผ31) )์ด๋ผ๊ณ  ์ง€๋ชฉํ•œ ๊ณต์†Œ์™ธ 23์˜ ์ง„์ˆ ์€ ๋ฏฟ์„ ์ˆ˜ ์—†๋‹ค. ์ฆ‰, ๊ณต์†Œ์™ธ 23์€ 1995๋…„ ๋ถ€์—ฌ์—์„œ ๊ฒ€๊ฑฐ๋œ ๋‚จํŒŒ๊ณต์ž‘์›์œผ๋กœ ์ดํ›„ 4๋…„๊ฐ„ ๊ตญ์ •์› ์•ˆ๊ฐ€์— ๊ฐ๊ธˆ๋˜์–ด ์žˆ๋‹ค๊ฐ€ 2008๋…„๋ถ€ํ„ฐ ์ง€๊ธˆ๊นŒ์ง€ ๊ตญ์ •์› ์‚ฐํ•˜๊ธฐ๊ด€์—์„œ ๊ทผ๋ฌดํ•˜๊ณ  ์žˆ๋‹ค. ๊ทธ์˜ ๊ฒฝ๋ ฅ์— ๋น„์ถ”์–ด ๋ณผ ๋•Œ ๊ตญ์ •์›์— ์œ ๋ฆฌํ•œ ์ฆ์–ธ์„ ํ•  ์ˆ˜ ๋ฐ–์— ์—†๋‹ค. ํ•œํŽธ ๊ณต์†Œ์™ธ 23์€ ๋‹ค๋ฅธ ๊ตญ๊ฐ€๋ณด์•ˆ๋ฒ•์œ„๋ฐ˜ ์‚ฌ๊ฑด์—๋„ ์ฆ์ธ์œผ๋กœ ์ถœ์„ํ•˜์—ฌ ํ•ด๋‹น ์‚ฌ๊ฑด์—์„œ ์ดฌ์˜๋œ ์‚ฌ๋žŒ์ด ์ž์‹ ์„ ์ง€๋„ํ–ˆ๋˜ ์ง€๋„์›์ด๋ผ๊ณ  ์ง€๋ชฉํ•˜๋ฉด์„œ ๊ทธ๋ฅผ ์•Œ๊ฒŒ ๋œ ๊ฒฝ์œ„ ๋“ฑ์„ ๋™์ผํ•˜๊ฒŒ ์„ค๋ช…ํ•˜๋Š” ๋“ฑ ์‹ ๋น™์„ฑ์ด ์—†๋‹ค. ๋‚˜. ์ฃผ32) ๊ด€๋ จ๋ฒ•๋ฆฌ ๊ตญ๊ฐ€๋ณด์•ˆ๋ฒ• ์ œ8์กฐ ์ œ1ํ•ญ์—์„œ ์ •ํ•œ ํšŒํ•ฉยทํ†ต์‹  ๋“ฑ์˜ ์ฃ„๋Š” ๊ตญ๊ฐ€์˜ ์กด๋ฆฝยท์•ˆ์ „์ด๋‚˜ ์ž์œ ๋ฏผ์ฃผ์  ๊ธฐ๋ณธ์งˆ์„œ๋ฅผ ์œ„ํƒœ๋กญ๊ฒŒ ํ•œ๋‹ค๋Š” ์ •์„ ์•Œ๋ฉด์„œ ๋ฐ˜๊ตญ๊ฐ€๋‹จ์ฒด์˜ ๊ตฌ์„ฑ์› ๋˜๋Š” ๊ทธ ์ง€๋ น์„ ๋ฐ›์€ ์ž์™€ ํšŒํ•ฉยทํ†ต์‹  ๊ธฐํƒ€์˜ ๋ฐฉ๋ฒ•์œผ๋กœ ์—ฐ๋ฝ์„ ํ•˜๊ณ , ๊ทธ ํšŒํ•ฉ ๋“ฑ์˜ ํ–‰์œ„๊ฐ€ ๊ตญ๊ฐ€์˜ ์กด๋ฆฝยท์•ˆ์ „์ด๋‚˜ ์ž์œ ๋ฏผ์ฃผ์  ๊ธฐ๋ณธ์งˆ์„œ์— ์‹ค์งˆ์  ํ•ด์•…์„ ๋ผ์น  ๋ช…๋ฐฑํ•œ ์œ„ํ—˜์„ฑ์ด ์žˆ์„ ๋•Œ ์„ฑ๋ฆฝํ•œ๋‹ค(๋Œ€๋ฒ•์› 2008. 4. 17. ์„ ๊ณ  2003๋„758 ์ „์›ํ•ฉ์˜์ฒด ํŒ๊ฒฐ ๋“ฑ ์ฐธ์กฐ). ๊ตญ๊ฐ€๋ณด์•ˆ๋ฒ• ์ œ8์กฐ ์ œ1ํ•ญ์˜ ํšŒํ•ฉ์ฃ„๋Š” ๋ฐ˜๊ตญ๊ฐ€๋‹จ์ฒด์˜ ์ด์ต์ด ๋œ๋‹ค๋Š” ์ •์„ ์•Œ๋ฉด์„œ ๋˜๋Š” ๊ตญ๊ฐ€์˜ ์กด๋ฆฝ, ์•ˆ์ „์ด๋‚˜ ์ž์œ ๋ฏผ์ฃผ์  ๊ธฐ๋ณธ์งˆ์„œ๋ฅผ ์œ„ํƒœ๋กญ๊ฒŒ ํ•œ๋‹ค๋Š” ์ •์„ ์•Œ๊ณ ์„œ ๊ทธ ๊ตฌ์„ฑ์› ๋˜๋Š” ๊ทธ ์ง€๋ น์„ ๋ฐ›์€ ์ž์™€ ํšŒํ•ฉ์„ ํ•˜๋ฉด ์„ฑ๋ฆฝ๋˜๋Š” ๊ฒƒ์œผ๋กœ์„œ, ๊ทธ๊ฒƒ์ด ์˜๋ก€์ , ์‚ฌ๊ต์ ์ธ ์ฐจ์›์—์„œ์˜ ์ „ํ˜€ ๋‹ค๋ฅธ ์˜๋„ํ•˜์—์„œ์˜ ๋ชจ์ž„์ด ์•„๋‹Œ ํ•œ ํšŒํ•ฉ์ž ์ƒํ˜ธ๊ฐ„์— ์‚ฌ์ „ ๊ณต๋™์˜์‚ฌ๊ฐ€ ์žˆ์–ด์•ผ ํ•˜๋Š” ๊ฒƒ๋„ ์•„๋‹ˆ๊ณ , ํšŒํ•ฉ์˜ ๊ฒฝ์œ„๋‚˜ ๋ฐฉ๋ฒ•๋„ ๋ถˆ๋ฌธํ•˜์—ฌ, ๋ฐ˜๋“œ์‹œ ์ผ์ •ํ•œ ์‚ฌํ•ญ์„ ๋…ผ์˜ํ•˜๊ฑฐ๋‚˜ ๊ฒฐ์ •ํ•˜์—ฌ์•ผ ํ•˜๋Š” ๊ฒƒ๋„ ์•„๋‹ˆ๋ฉฐ, ๋ชฉ์ ์ˆ˜ํ–‰์„ ์œ„ํ•œ ์ผ๋ จ์˜ ํ™œ๋™๊ณผ์ •์—์„œ์˜ ๋ชจ์ž„์œผ๋กœ ์ธ์ •๋˜๋ฉด ์กฑํ•˜๋‹ค(๋Œ€๋ฒ•์› 1997. 9. 9. ์„ ๊ณ  97๋„1656 ํŒ๊ฒฐ ๋“ฑ ์ฐธ์กฐ). ํ•œํŽธ ํ–‰์œ„์ž์—๊ฒŒ ์ด๋Ÿฌํ•œ ์ •์— ๋Œ€ํ•œ ์ธ์‹์ด ์žˆ์—ˆ๋Š”์ง€ ์—ฌ๋ถ€๋Š” ํšŒํ•ฉยทํ†ต์‹  ๋“ฑ ์—ฐ๋ฝ์˜ ๊ฒฝ์œ„, ๋Œ€ํ™”ยท์—ฐ๋ฝ์˜ ๋‚ด์šฉ ๋ฐ ๊ทธ ์ „ํ›„ ์‚ฌ์ • ๋“ฑ์„ ์ข…ํ•ฉํ•˜์—ฌ ๊ฐ๊ด€์ ์œผ๋กœ ํŒ๋‹จํ•˜์—ฌ์•ผ ํ•˜๋Š”๋ฐ, ๊ทธ ์ธ์‹์€ ์ƒ๋‹นํ•œ ์ƒ์‹์„ ๊ฐ€์ง„ ์‚ฌ๋žŒ๋“ค์ด ๊ทธ ํ–‰์œ„๊ฐ€ ๊ตญ๊ฐ€์˜ ์กด๋ฆฝยท์•ˆ์ „์ด๋‚˜ ์ž์œ ๋ฏผ์ฃผ์  ๊ธฐ๋ณธ์งˆ์„œ๋ฅผ ์œ„ํƒœ๋กญ๊ฒŒ ํ•œ๋‹ค๋Š” ์ •์„ ์ธ์‹ํ•˜๊ฑฐ๋‚˜ ๋˜๋Š” ์œ„ํƒœ๋กญ๊ฒŒ ํ•  ์ˆ˜ ์žˆ๋‹ค๋Š” ๋ฏธํ•„์  ์ธ์‹์„ ๊ฐ€์ง€๊ณ  ์žˆ์œผ๋ฉด ์ถฉ๋ถ„ํ•˜๊ณ  ๋ฐ˜๊ตญ๊ฐ€๋‹จ์ฒด์˜ ์ด์ต์„ ๋ชฉ์ ์œผ๋กœ ํ•˜๊ฑฐ๋‚˜ ์ด๋ฅผ ์˜์š•ํ•  ๊ฒƒ๊นŒ์ง€ ํ•„์š”๋กœ ํ•˜๋Š” ๊ฒƒ์€ ์•„๋‹ˆ๋ฉฐ, ๋‹ค๋ฅธ ๋ชฉ์ ์ด ์žˆ์—ˆ๋‹ค ํ•˜๋”๋ผ๋„ ํ˜„์‹ค์ ์œผ๋กœ ๋ฐ˜๊ตญ๊ฐ€๋‹จ์ฒด๊ฐ€ ์ด๋ฅผ ์šฐ๋ฆฌ ๋‚ด๋ถ€์˜ ๊ต๋ž€์ฑ… ๋“ฑ์œผ๋กœ ์•…์šฉํ•˜๋ฉด ์—ญ์‹œ ๋Œ€ํ•œ๋ฏผ๊ตญ์˜ ์กด๋ฆฝยท์•ˆ์ „์ด๋‚˜ ์ž์œ ๋ฏผ์ฃผ์ฒด์ œ๋ฅผ ์œ„ํƒœ๋กญ๊ฒŒ ํ•  ์ˆ˜ ์žˆ๋Š” ๊ฒƒ์— ํ•ด๋‹นํ•˜๊ณ  ๊ทธ๋Ÿฌํ•œ ์•…์šฉ์ด ๊ฐ€๋Šฅํ•˜๋‹ค๋Š” ์ธ์‹์ด ์žˆ์—ˆ๋‹ค๋ฉด ๊ทธ ๋ฒ”์˜๋ฅผ ์ธ์ •ํ•  ์ˆ˜ ์žˆ๋‹ค(๋Œ€๋ฒ•์› 2003. 9. 26. ์„ ๊ณ  2001๋„2209 ํŒ๊ฒฐ, ๋Œ€๋ฒ•์› 2011. 10. 13. ์„ ๊ณ  2011๋„9094 ํŒ๊ฒฐ, ๋Œ€๋ฒ•์› 2012. 10. 25. ์„ ๊ณ  2010๋„6310 ํŒ๊ฒฐ ๋“ฑ ์ฐธ์กฐ). ๋‹ค. ํŒ๋‹จ ์ด ๋ฒ•์›์ด ์ฑ„ํƒํ•˜์—ฌ ์กฐ์‚ฌํ•œ ์ฆ๊ฑฐ๋“ค์— ์˜ํ•˜์—ฌ ์ธ์ •๋˜๋Š” ๋‹ค์Œ๊ณผ ๊ฐ™์€ ์‚ฌ์ •๋“ค์„ ์ข…ํ•ฉํ•˜๋ฉด ํ”ผ๊ณ ์ธ์€ ๋ฐ˜๊ตญ๊ฐ€๋‹จ์ฒด์˜ ์ด์ต์ด ๋จ๊ณผ ์•„์šธ๋Ÿฌ ๊ตญ๊ฐ€์˜ ์กด๋ฆฝยท์•ˆ์ „์ด๋‚˜ ์ž์œ ๋ฏผ์ฃผ์  ๊ธฐ๋ณธ์งˆ์„œ๋ฅผ ์œ„ํƒœ๋กญ๊ฒŒ ํ•œ๋‹ค๋Š” ์ •์„ ์•Œ๋ฉด์„œ ๊ทธ ๊ตฌ์„ฑ์› ๋˜๋Š” ๊ทธ ์ง€๋ น์„ ๋ฐ›์€ ์ž์ธ โ˜†์ง€๋„์› ๋“ฑ๊ณผ ํšŒํ•ฉํ•œ ์‚ฌ์‹ค์„ ์ธ์ •ํ•  ์ˆ˜ ์žˆ๋‹ค. 1) ์ด ๋ฒ•์ •์— ์ฆ์ธ์œผ๋กœ ์ถœ์„ํ•œ ๊ตญ์ •์› ์ˆ˜์‚ฌ๊ด€๋“ค๊ณผ ๊ฒฝ์ฐฐ๊ด€์€ ํ”ผ๊ณ ์ธ์ด 2011. 4. 21.๊ฒฝ ์ค‘๊ตญ ๋Œ€๋ จ์—์„œ, 2012. 5. 31.๊ฒฝ ๋ฒ ํŠธ๋‚จ ํ˜ธ์น˜๋ฏผ์—์„œ ๋ถํ•œใ€Œ225๊ตญใ€์˜ ๊ณต์ž‘์กฐ์ธ โ˜†์ง€๋„์›๊ณผ ๊ณต์†Œ์™ธ 39๋ฅผ ๋งŒ๋‚˜๋Š” ๊ฒƒ์„ ์ง์ ‘ ๋ชฉ๊ฒฉํ•˜๊ณ  ์ด๋ฅผ ๋ฏธํ–‰ํ•˜์—ฌ ์‚ฌ์ง„์„ ์ดฌ์˜ํ•˜์˜€๋‹ค๊ณ  ์ง„์ˆ ํ•˜์˜€๋‹ค. 2) ์ฆ์ธ ๊ณต์†Œ์™ธ 23์€ ๋‚จํŒŒ๊ณต์ž‘์›์œผ๋กœ ํ™œ๋™ํ•˜๋‹ค๊ฐ€ ์ฒดํฌ๋œ ์ดํ›„ ํ˜„์žฌ ๊ตญ๊ฐ€์•ˆ๋ณด์ „๋žต์—ฐ๊ตฌ์†Œ ์—ฐ๊ตฌ์œ„์›์œผ๋กœ ๊ทผ๋ฌดํ•˜๊ณ  ์žˆ๋Š” ์‚ฌ๋žŒ์ธ๋ฐ, โ€˜ํ”ผ๊ณ ์ธ์ด ๋งŒ๋‚œ ์‚ฌ๋žŒ์ด โ˜†์ง€๋„์›๊ณผ โ–ทโ–ทโ–ท์ด๋‹ค. โ˜†์ง€๋„์›์€ ํ•œ๊ตญ์— ๋‘ ๋ฒˆ์— ๊ฑธ์ณ ์นจํˆฌํ•ด ์ง€ํ•˜๋‹น์„ ๊ตฌ์ถ•ํ•ด์„œ ๊ฐ„์ฒฉ์„ ํฌ์„ญํ•ด์„œ ๊ณตํ™”๊ตญ์˜์›… ์นญํ˜ธ๋ฅผ ๋‘ ๋ฒˆ ๋ฐ›์€ ์‚ฌ๋žŒ์ด๋‹ค. โ˜†์ง€๋„์›์€ ์ฒด์ œ์˜ ๋ณด์•ˆ์„ ์š”ํ•˜๋Š” ๊ณต์ž‘๋ถ€์„œ์˜ ๊ฐ„๋ถ€์ด๊ธฐ ๋•Œ๋ฌธ์— ์ผ๋ฐ˜์ธ์ด๋‚˜ ๋ฌด์Šจ ์‚ฌ์—… ๋ชฉ์ ์œผ๋กœ ๋งŒ๋‚  ์ผ์€ ์ „ํ˜€ ์—†๋‹ค. โ–ทโ–ทโ–ท์€ ์ฆ์ธ๋ณด๋‹ค ๋Œ€ํ•™ 2๋…„ ์„ ๋ฐฐ์ด๋‹ค. ๋Œ€ํ•™์—์„œ 1๋…„ ๋™์•ˆ ์ฆ์ธ๊ณผ ๊ฐ™์€ ์ค‘๋Œ€์—์„œ ํ™œ๋™ํ–ˆ๊ธฐ ๋•Œ๋ฌธ์— ๊ฐœ์ธ์ ์œผ๋กœ ์ž˜ ์•ˆ๋‹ค. ๋Œ€ํ•™์„ ์กธ์—…ํ•œ ํ›„์— ๊ณต์ž‘์›์„ ํ•˜๋ฉด์„œ โ–ทโ–ทโ–ท์ด ๋ชจ์ž๊ณต์ž‘์กฐ๋กœ ์œ„์žฅํ•ด์„œ 1988.๊ฒฝ์— ํ•œ๊ตญ์— ์‹ค์ œ๋กœ ์นจํˆฌํ•ด์„œ ๊ณต์†Œ์™ธ 70์„ ์ ‘์„ ํ•ด์„œ ๊ณต์†Œ์™ธ 70๊ณผ ๊ฐ™์ด ์„œ์šธ์—์„œ ํ™œ๋™ํ–ˆ๋‹ค. ๊ธˆ์„ฑ์ •์น˜๊ตฐ์‚ฌ๋Œ€ํ•™ ์ „ํˆฌ์›๋ฐ˜์„ ๊ฐ™์ด 1๋…„ ๋™์•ˆ ๋‹ค๋‹ˆ๋ฉด์„œ ํ›ˆ๋ จ๋„ ๊ฐ™์ด ํ•˜๊ณ  ๊ณ„์† ๋ฐฅ ๋จน์œผ๋Ÿฌ ๋‹ค๋…”๊ธฐ ๋•Œ๋ฌธ์— โ–ทโ–ทโ–ท์„ ๊ธฐ์–ตํ•˜์ง€ ๋ชปํ•  ์ผ์€ ์—†๋‹ค. ๋‹ค๋งŒ ์ด์ œ ๋ณด๋‹ˆ ์‚ด๋งŒ ์ฐŒ๊ณ  ๋‚˜์ด๋งŒ ๋“ค์—ˆ์ง€ ๋ชจ์Šต์€ ๊ทธ๋Œ€๋กœ์ด๋‹ค. ์–ด์ฐจํ”ผ ๊ณต์ž‘์›๋“ค์€ ํ™œ๋™ํ•  ๋•Œ ๊ฐ€๋ช…์„ ๋‹ค ์“ฐ๊ธฐ ๋•Œ๋ฌธ์— ์–ด๋Š ๊ฒƒ์ด ๋ณธ๋ช…์ด๋ผ๊ณ  ํ•  ์ˆ˜๋Š” ์—†๋‹ค. ๋Œ€ํ•™ ๋•Œ ์“ฐ๋˜ ์ด๋ฆ„์ด โ–ทโ–ทโ–ท์ด์—ˆ๊ณ , ๊ณต์ž‘์›์œผ๋กœ ์™€์„œ โ™คโ™ค์ด๋ผ๋Š” ์ด๋ฆ„์„ ์ผ๋Š”๋ฐ, ์„ฑ ์”จ๋Š” ์ž˜ ๋ชจ๋ฅด๊ฒ ๋‹คโ€™๋Š” ์ทจ์ง€๋กœ ๊ตฌ์ฒด์ ์ด๊ณ  ์ผ๊ด€๋˜๊ฒŒ ์ง„์ˆ ํ•˜์˜€๋‹ค. ํ•œํŽธ, ๋ณ€ํ˜ธ์ธ์˜ ์ฃผ์žฅ๋Œ€๋กœ, ๊ณต์†Œ์™ธ 23์ด ๋‹ค๋ฅธ ์‚ฌ๊ฑด์˜ ์ˆ˜์‚ฌ ์ค‘ ์ดฌ์˜๋œ ์‚ฌ๋žŒ์„ ๋ถํ•œ์— ์žˆ์„ ๋•Œ ์ž์‹ ์„ ์ง€๋„ํ•œ ๊ณต์†Œ์™ธ 71์ด๋ผ๊ณ  ์ง€๋ชฉํ•˜๋ฉด์„œ, ๊ทธ๋ฅผ ์•Œ๊ฒŒ ๋œ ๊ฒฝ์œ„์— ๊ด€ํ•˜์—ฌ โ€˜1995๋…„ ์ œ๊ฐ€ ๋‚จํ•œ์— ์นจํˆฌํ•  ๋‹น์‹œ ์ €๋ฅผ ๋‹ด๋‹นํ–ˆ๋˜ ๊ณต์ž‘ ์ง€๋„์›์ด๋‹ค. ๋‹น์‹œ ์ €๋Š” ๊ณต์†Œ์™ธ 71๊ณผ ํ‰์–‘ ์ˆœ์•ˆ ํŠน๋ณ„์ดˆ๋Œ€์†Œ์™€ ๋งŒ๊ฒฝ๋Œ€ ๊ตฌ์—ญ ์ดˆ๋Œ€์†Œ์—์„œ ์„ธ ๋‹ฌ ์ •๋„ ํ•จ๊ป˜ ์ƒํ™œํ•˜์˜€๊ธฐ ๋•Œ๋ฌธ์— ์ง€๊ธˆ๋„ ์ƒ์ƒํžˆ ๊ธฐ์–ตํ•œ๋‹คโ€™๊ณ  ์ง„์ˆ ํ•˜์˜€๊ณ , ์ด๋Š” ์ด ์‚ฌ๊ฑด์—์„œ ๊ณต์†Œ์™ธ 23์ด โ˜†์ง€๋„์›์„ ์•Œ๊ฒŒ ๋œ ๊ฒฝ์œ„์— ๊ด€ํ•œ ์ง„์ˆ ๊ณผ ์œ ์‚ฌํ•˜๊ธฐ๋Š” ํ•˜๋‹ค. ๊ทธ๋Ÿฌ๋‚˜ ๊ณต์†Œ์™ธ 23์€ ๊ทธ ์™ธ์˜ ์ ์— ๊ด€ํ•˜์—ฌ๋„ ๊ตฌ์ฒด์ ์œผ๋กœ ์ง„์ˆ ํ•˜๊ณ  ์žˆ๊ณ  ์ž์‹ ์„ ์ง€๋„ํ•œ ๋‹ด๋‹น์ง€๋„์›์ด ๋ณดํ†ต 3๋ช… ๋‚ด์ง€ 4๋ช…์ด๋ผ๊ณ  ํ•˜๊ณ  ์žˆ์œผ๋ฉฐ, ๋‹ค๋ฅธ ์‚ฌ๊ฑด์˜ ์ง„์ˆ ์กฐ์„œ์™€ ์ด ์‚ฌ๊ฑด ์ง„์ˆ ์กฐ์„œ๋ฅผ ๋น„๊ตํ•ด๋ณด์•„๋„ โ˜†์ง€๋„์›๊ณผ ๊ณต์†Œ์™ธ 71์„ ๊ตฌ๋ถ„ํ•˜๋ฉด์„œ ๋‹ฌ๋ฆฌ ๋ฌ˜์‚ฌํ•˜๊ณ  ์žˆ๋‹ค. ์ด๋Ÿฌํ•œ ์‚ฌ์ •์—๋‹ค๊ฐ€ ๊ณต์†Œ์™ธ 23์ด ๋‚จํŒŒ๊ณต์ž‘์›์œผ๋กœ ๋ถํ•œ์—์„œ ์ด๋ฅผ ์œ„ํ•œ ๊ต์œก์„ ๋ฐ›์€ ์‚ฌ์‹ค์€ ๋ถ„๋ช…ํ•œ ๊ฒƒ์œผ๋กœ ๋ณด์ด๋Š” ์ ์„ ์ข…ํ•ฉํ•ด๋ณด๋ฉด, ๊ทธ๋ฅผ ์ง€๋„ํ•œ ์ง€๋„์›์ด 1์ธ ์ด์ƒ์ด์—ˆ์„ ๊ฐ€๋Šฅ์„ฑ๊ณผ ๊ทธ๊ฐ€ โ˜†์ง€๋„์›๊ณผ ๊ณต์†Œ์™ธ 71์˜ ์–ผ๊ตด์„ ๋ชจ๋‘ ์•Œ๊ณ  ์žˆ์„ ๊ฐ€๋Šฅ์„ฑ์ด ์–ผ๋งˆ๋“ ์ง€ ์žˆ๋‹ค. ๊ทธ ๋ฐ–์— ์ด ์‚ฌ๊ฑด ์žฌํŒ๊ณผ์ •์—์„œ ๋‚˜ํƒ€๋‚œ ์—ฌ๋Ÿฌ ์‚ฌ์ •์— ๋น„์ถ”์–ด ๋ณด๋ฉด, ํ”ผ๊ณ ์ธ๊ณผ ๊ณต์†Œ์™ธ 6์ด ๋งŒ๋‚œ ์‚ฌ๋žŒ์ด ๋ถํ•œ์˜ ๋Œ€๋‚จ๊ณต์ž‘์›๋“ค์ด๋ผ๋Š” ์ ์— ๊ด€ํ•œ ๊ณต์†Œ์™ธ 23์˜ ์ง„์ˆ ์„ ๋ฏฟ์„ ์ˆ˜ ์žˆ๋‹ค. 3) ๊ณต์†Œ์™ธ 52, ๊ณต์†Œ์™ธ 50์˜ ๊ฐ ์ฆ์–ธ๊ณผ ๊ฐ์ •์„œ ๊ธฐ์žฌ, ์ด ๋ฒ•์›์˜ ๊ฒ€์ฆ๊ฒฐ๊ณผ๋ฅผ ์ข…ํ•ฉํ•˜์—ฌ ๋ณด์•„๋„ ํ”ผ๊ณ ์ธ์ด ๋Œ€๋ จ๊ณผ ํ˜ธ์น˜๋ฏผ์—์„œ ๋งŒ๋‚œ ์‚ฌ๋žŒ๊ณผ 2015. 10. 9. ๋ถํ•œ ์กฐ์„ ์ค‘์•™ TV ๋ฐฉ์†ก์— ์ดฌ์˜๋œ ์‚ฌ๋žŒ์€ ๋ชจ๋‘ ๋™์ผํ•œ ์‚ฌ๋žŒ์ธ ๊ฒƒ์œผ๋กœ ๋ณด์ธ๋‹ค. ๋”ฐ๋ผ์„œ ํ”ผ๊ณ ์ธ์ด ๋Œ€๋ จ๊ณผ ํ˜ธ์น˜๋ฏผ์—์„œ ๋งŒ๋‚œ ์‚ฌ๋žŒ์€ โ˜†์ง€๋„์›์ด ๋งž๋Š” ๊ฒƒ์œผ๋กœ ๋ณด์ด๊ณ , ์œ„ ์˜์ƒ์—์„œ ์•‰์€ ์œ„์น˜์™€ ๋Œ€๋ จ๊ณผ ํ˜ธ์น˜๋ฏผ์— ์ˆ˜ํ–‰์›์„ ๋ฐ๋ฆฌ๊ณ  ๋‚˜ํƒ€๋‚ฌ๋˜ ์ ์— ๋น„์ถ”์–ด ๋ณผ ๋•Œ โ˜†์ง€๋„์›์€ ๋ถํ•œ์—์„œ ์ƒ๋‹นํžˆ ๋†’์€ ์ง€์œ„์— ์žˆ๋Š” ๊ฒƒ์œผ๋กœ ๋ณด์ธ๋‹ค. ํ”ผ๊ณ ์ธ์€ ๋‘ ๋ฒˆ ๋‹ค ํ˜ผ์ž ์ถœ๊ตญํ•˜์˜€๊ณ , ๋ฏธ๋ฆฌ ํ˜„์žฅ์„ ๋‹ต์‚ฌํ•˜๊ณ  ๋ฏธํ–‰์ด ์žˆ๋Š”์ง€ ํ™•์ธํ•˜๋Š” ๋“ฑ ํƒ€์ธ์˜ ๋ˆˆ์„ ํ”ผํ•˜๋ ค ๋…ธ๋ ฅํ•˜์˜€๋Š”๋ฐ ์ด๋ ‡๋“ฏ ๋น„์Šทํ•œ ์‹œ๊ธฐ(๋งค๋…„ 4~5์›” ๊ฒฝ)์— ํ•ด์™ธ์—์„œ 2ํšŒ์— ๊ฑธ์ณ, ๋ถํ•œ์˜ ๊ณต์ž‘์›์„ ๋ชฐ๋ž˜ ๋งŒ๋‚˜๋Š” ๊ฒƒ์€ ๊ฐœ์ธ์ ์ด๊ฑฐ๋‚˜ ์‚ฌ๊ต์ ์ธ ๋งŒ๋‚จ์€ ์•„๋‹Œ ๊ฒƒ์œผ๋กœ ๋ณด์ธ๋‹ค. 4) ํ”ผ๊ณ ์ธ์€ ๊ณต์†Œ์™ธ 5 ๋“ฑ๊ณผ ์‚ฌ์ƒํ•™์Šต์„ ํ•˜๊ธฐ๋„ ํ•˜๊ณ , ๋ฐ˜๊ตญ๊ฐ€๋‹จ์ฒด ๊ตฌ์„ฑ์›๋“ค๊ณผ ํ†ต์‹ ์„ ํ•˜๊ธฐ๋„ ํ•˜์˜€๋‹ค. ํ”ผ๊ณ ์ธ์ด ๊ณต์†Œ์™ธ 5 ๋“ฑ๊ณผ ํ•œ ์‚ฌ์ƒํ•™์Šต์˜ ๋‚ด์šฉ์€ ๋ถํ•œ์„ ์ถ”์ข…ํ•˜๊ฑฐ๋‚˜ ๋Œ€๋‚จํ˜๋ช…๋ก  ๋“ฑ์„ ๋ฏธํ™”, ์„ ์ „, ๋™์กฐํ•˜๋Š” ๋‚ด์šฉ์ด๊ณ , ํ†ต์‹ ํ•œ ๋ฌธ๊ฑด์—๋Š” ๋Œ€ํ•œ๋ฏผ๊ตญ ๋‚ด์—์„œ์˜ ๋น„๋ฐ€์กฐ์ง์˜ ํ™œ๋™ ํ˜„ํ™ฉ, ์•ž์œผ๋กœ์˜ ํ™œ๋™ ๊ณ„ํš ๋“ฑ๋„ ๊ธฐ์žฌ๋˜์–ด ์žˆ๋‹ค. ์ด์™€ ๊ฐ™์€ ์ผ๋ จ์˜ ํ–‰์œ„๋Š” ํ”ผ๊ณ ์ธ์ด ํ•ด์™ธ์—์„œ ๊ณต์ž‘์›์„ ๋งŒ๋‚œ ๊ฒƒ๊ณผ ๋ชจ๋‘ ๊ด€๋ จ์ด ์žˆ๋Š” ํ–‰์œ„๋กœ ๋ณด์ธ๋‹ค. 5) ํ”ผ๊ณ ์ธ์€ ๋Œ€๋ จ์—์„œ โ˜†์ง€๋„์›๊ณผ ๋งŒ๋‚  ๋•Œ, ๋‘ ์†์„ ์•ž์œผ๋กœ ๋ชจ์€ ์ฑ„ ์ด์•ผ๊ธฐ๋ฅผ ํ•˜๊ณ  ์•…์ˆ˜๋ฅผ ํ•œ ๋’ค ๊ณ ๊ฐœ๋ฅผ ์ˆ™์—ฌ ์ธ์‚ฌ๋ฅผ ํ•˜๊ธฐ๋„ ํ•˜๋Š” ๋“ฑ ์ƒ๊ธ‰์ž๋ฅผ ๋Œ€ํ•˜๋Š” ๋“ฏํ•œ ํƒœ๋„๋ฅผ ์ทจํ•˜์˜€๋‹ค. ๋ฐ˜๋ฉด์—, ํ”ผ๊ณ ์ธ๊ณผ ๋ณ€ํ˜ธ์ธ์€ ํ”ผ๊ณ ์ธ์ด ์ด ์‚ฌ๊ฑด ์ผ์‹œยท์žฅ์†Œ์—์„œ ๋ˆ„๊ตฌ๋ฅผ ์–ด๋– ํ•œ ๊ฒฝ์œ„๋กœ ๋งŒ๋‚ฌ๋Š”์ง€ ์—ฌํ•˜์— ๋Œ€ํ•ด ์ผ๋ง์˜ ์„ค๋ช…๋„ ํ•˜์ง€ ์•Š๊ณ  ์žˆ๋‹ค. ๋ผ. ์†Œ๊ฒฐ ํ”ผ๊ณ ์ธ๊ณผ ๋ณ€ํ˜ธ์ธ๋“ค์˜ ์ด ๋ถ€๋ถ„ ์ฃผ์žฅ์€ ๋ฐ›์•„๋“ค์ด์ง€ ์•Š๋Š”๋‹ค. 21. 2015. 4. 5.๊ฒฝ ๊ตญ๊ฐ€๋ณด์•ˆ๋ฒ•์œ„๋ฐ˜(ํšŒํ•ฉยทํ†ต์‹ ), ๊ตญ๊ฐ€๋ณด์•ˆ๋ฒ•์œ„๋ฐ˜(์ž์ง„์ง€์›ยท๊ธˆํ’ˆ์ˆ˜์ˆ˜)์˜ ์ ์— ๊ด€ํ•˜์—ฌ ๊ฐ€. ์ฃผ์žฅ ๊ณต์†Œ์™ธ 6์€ ๋ฐ˜๊ตญ๊ฐ€๋‹จ์ฒด ๊ตฌ์„ฑ์›๊ณผ ํšŒํ•ฉํ•˜๊ฑฐ๋‚˜ ๊ธˆํ’ˆ์„ ์ˆ˜์ˆ˜ํ•˜์ง€ ์•Š์•˜๋‹ค. ์„ค์‚ฌ ๊ทธ๋ ‡๋‹ค๊ณ  ํ•˜๋”๋ผ๋„ ํ”ผ๊ณ ์ธ์€ ๊ณต์†Œ์™ธ 6์˜ ๋ฒ”ํ–‰์„ ๋ชฐ๋ž๊ณ , ๊ณต๋ชจ๋‚˜ ์ง€์‹œ ๋“ฑ ์–ด๋– ํ•œ ๊ธฐ๋Šฅ์  ํ–‰์œ„์ง€๋ฐฐ๋„ ์—†๋‹ค. ๋‚˜. ๊ด€๋ จ๋ฒ•๋ฆฌ 1) ๊ตญ๊ฐ€๋ณด์•ˆ๋ฒ• ์ œ5์กฐ ์ œ2ํ•ญ์˜ ๊ธˆํ’ˆ์ˆ˜์ˆ˜์ฃ„๋Š” ๊ตญ๊ฐ€์˜ ์กด๋ฆฝยท์•ˆ์ „์ด๋‚˜ ์ž์œ ๋ฏผ์ฃผ์  ๊ธฐ๋ณธ์งˆ์„œ๋ฅผ ์œ„ํƒœ๋กญ๊ฒŒ ํ•œ๋‹ค๋Š” ์ •์„ ์•Œ๋ฉด์„œ ๋ฐ˜๊ตญ๊ฐ€๋‹จ์ฒด์˜ ๊ตฌ์„ฑ์› ๋˜๋Š” ๊ทธ ์ง€๋ น์„ ๋ฐ›์€ ์ž๋กœ๋ถ€ํ„ฐ ๊ธˆํ’ˆ์„ ์ˆ˜์ˆ˜ํ•˜๋Š” ๊ฒฝ์šฐ ์„ฑ๋ฆฝํ•˜๊ณ , ๊ธˆํ’ˆ์˜ ๊ฐ€์•ก์ด๋‚˜ ๊ฐ€์น˜ ๋˜๋Š” ๊ธˆํ’ˆ์ˆ˜์ˆ˜์˜ ๋ชฉ์ ์„ ๊ฐ€๋ฆฌ์ง€ ์•„๋‹ˆํ•œ๋‹ค(๋Œ€๋ฒ•์› 2012. 10. 11. ์„ ๊ณ  2012๋„7455 ํŒ๊ฒฐ ์ฐธ์กฐ). 2) ๊ตญ๊ฐ€์˜ ์กด๋ฆฝยท์•ˆ์ „์ด๋‚˜ ์ž์œ ๋ฏผ์ฃผ์  ๊ธฐ๋ณธ์งˆ์„œ๋ฅผ ์œ„ํƒœ๋กญ๊ฒŒ ํ•œ๋‹ค๋Š” ์ •์„ ์•Œ๋ฉด์„œ ๋ฐ˜๊ตญ๊ฐ€๋‹จ์ฒด์˜ ๊ตฌ์„ฑ์› ๋˜๋Š” ๊ทธ ์ง€๋ น์„ ๋ฐ›์€ ์ž๋กœ๋ถ€ํ„ฐ ๊ธˆํ’ˆ์„ ์ˆ˜์ˆ˜ํ•œ ๊ฒฝ์šฐ ํ–‰์œ„์ž์—๊ฒŒ ์ด๋Ÿฌํ•œ ์ •์— ๋Œ€ํ•œ ์ธ์‹์ด ์žˆ์–ด์•ผ ํ•˜๊ณ , ์ด๋Ÿฌํ•œ ์ธ์‹์ด ์žˆ์—ˆ๋Š”์ง€ ์—ฌ๋ถ€๋Š” ๊ธˆํ’ˆ ์ˆ˜์ˆ˜ ๊ฒฝ์œ„ ๋ฐ ๊ทธ ์ „ํ›„ ์‚ฌ์ • ๋“ฑ์„ ์ข…ํ•ฉํ•˜์—ฌ ๊ฐ๊ด€์ ์œผ๋กœ ํŒ๋‹จํ•˜์—ฌ์•ผ ํ•˜๋ฉฐ, ๊ทธ ์ธ์‹์€ ์ƒ๋‹นํ•œ ์ƒ์‹์„ ๊ฐ€์ง„ ์‚ฌ๋žŒ๋“ค์ด ๊ทธ ํ–‰์œ„๊ฐ€ ๊ตญ๊ฐ€์˜ ์กด๋ฆฝยท์•ˆ์ „์ด๋‚˜ ์ž์œ ๋ฏผ์ฃผ์  ๊ธฐ๋ณธ์งˆ์„œ๋ฅผ ์œ„ํƒœ๋กญ๊ฒŒ ํ•œ๋‹ค๋Š” ์ •์„ ์ธ์‹ํ•˜๊ฑฐ๋‚˜ ๋˜๋Š” ์œ„ํƒœ๋กญ๊ฒŒ ํ•  ์ˆ˜ ์žˆ๋‹ค๋Š” ๋ฏธํ•„์  ์ธ์‹์„ ๊ฐ€์ง€๊ณ  ์žˆ์œผ๋ฉด ์ถฉ๋ถ„ํ•˜๋ฉฐ ๋ฐ˜๊ตญ๊ฐ€๋‹จ์ฒด์˜ ์ด์ต์„ ๋ชฉ์ ์œผ๋กœ ํ•˜๊ฑฐ๋‚˜ ์ด๋ฅผ ์˜์š•ํ•  ๊ฒƒ๊นŒ์ง€ ํ•„์š”๋กœ ํ•˜๋Š” ๊ฒƒ์€ ์•„๋‹ˆ๊ณ , ๋‹ค๋ฅธ ๋ชฉ์ ์ด ์žˆ์—ˆ๋‹ค ํ•˜๋”๋ผ๋„ ํ˜„์‹ค์ ์œผ๋กœ ๋ฐ˜๊ตญ๊ฐ€๋‹จ์ฒด๊ฐ€ ์ด๋ฅผ ์šฐ๋ฆฌ ๋‚ด๋ถ€์˜ ๊ต๋ž€์ฑ… ๋“ฑ์œผ๋กœ ์•…์šฉํ•˜๋ฉด ์—ญ์‹œ ์šฐ๋ฆฌ๋‚˜๋ผ์˜ ์กด๋ฆฝยท์•ˆ์ „์ด๋‚˜ ์ž์œ ๋ฏผ์ฃผ์ฒด์ œ๋ฅผ ์œ„ํƒœ๋กญ๊ฒŒ ํ•  ์ˆ˜ ์žˆ๋Š” ๊ฒƒ์— ํ•ด๋‹นํ•˜๊ณ  ๊ทธ๋Ÿฌํ•œ ์•…์šฉ์ด ๊ฐ€๋Šฅํ•˜๋‹ค๋Š” ์ธ์‹์ด ์žˆ์—ˆ๋‹ค๋ฉด ๊ทธ ๋ฒ”์˜๋ฅผ ์ธ์ •ํ•  ์ˆ˜ ์žˆ๋‹ค(๋Œ€๋ฒ•์› 2003. 9. 26. ์„ ๊ณ  2001๋„2209 ํŒ๊ฒฐ ๋“ฑ ์ทจ์ง€ ์ฐธ์กฐ). 3) ํ˜•๋ฒ• ์ œ30์กฐ์˜ ๊ณต๋™์ •๋ฒ”์€ ๊ณต๋™๊ฐ€๊ณต์˜ ์˜์‚ฌ์™€ ๊ทธ ๊ณต๋™์˜์‚ฌ์— ์˜ํ•œ ๊ธฐ๋Šฅ์  ํ–‰์œ„์ง€๋ฐฐ๋ฅผ ํ†ตํ•œ ๋ฒ”์ฃ„์‹คํ–‰์ด๋ผ๋Š” ์ฃผ๊ด€์ ยท๊ฐ๊ด€์  ์š”๊ฑด์„ ์ถฉ์กฑํ•จ์œผ๋กœ์จ ์„ฑ๋ฆฝํ•˜๋ฏ€๋กœ, ๊ณต๋ชจ์ž ์ค‘ ๊ตฌ์„ฑ์š”๊ฑดํ–‰์œ„๋ฅผ ์ง์ ‘ ๋ถ„๋‹ดํ•˜์—ฌ ์‹คํ–‰ํ•˜์ง€ ์•„๋‹ˆํ•œ ์‚ฌ๋žŒ๋„ ์œ„ ์š”๊ฑด์˜ ์ถฉ์กฑ ์—ฌ๋ถ€์— ๋”ฐ๋ผ ์ด๋ฅธ๋ฐ” ๊ณต๋ชจ๊ณต๋™์ •๋ฒ”์œผ๋กœ์„œ์˜ ์ฃ„์ฑ…์„ ์งˆ ์ˆ˜๋„ ์ฃผ33) ์žˆ๋‹ค. ํ•œํŽธ ๊ตฌ์„ฑ์š”๊ฑดํ–‰์œ„๋ฅผ ์ง์ ‘ ๋ถ„๋‹ดํ•˜์—ฌ ์‹คํ–‰ํ•˜์ง€ ์•„๋‹ˆํ•œ ๊ณต๋ชจ์ž๊ฐ€ ๊ณต๋ชจ๊ณต๋™์ •๋ฒ”์œผ๋กœ ์ธ์ •๋˜๊ธฐ ์œ„ํ•˜์—ฌ๋Š” ์ „์ฒด ๋ฒ”์ฃ„์— ์žˆ์–ด์„œ ๊ทธ๊ฐ€ ์ฐจ์ง€ํ•˜๋Š” ์ง€์œ„ยท์—ญํ• ์ด๋‚˜ ๋ฒ”์ฃ„๊ฒฝ๊ณผ์— ๋Œ€ํ•œ ์ง€๋ฐฐ ๋‚ด์ง€ ์žฅ์•…๋ ฅ ๋“ฑ์„ ์ข…ํ•ฉํ•˜์—ฌ ๊ทธ๊ฐ€ ๋‹จ์ˆœํ•œ ๊ณต๋ชจ์ž์— ๊ทธ์น˜๋Š” ๊ฒƒ์ด ์•„๋‹ˆ๋ผ ๋ฒ”์ฃ„์— ๋Œ€ํ•œ ๋ณธ์งˆ์  ๊ธฐ์—ฌ๋ฅผ ํ†ตํ•œ ๊ธฐ๋Šฅ์  ํ–‰์œ„์ง€๋ฐฐ๊ฐ€ ์กด์žฌํ•˜๋Š” ๊ฒƒ์œผ๋กœ ์ธ์ •๋˜์–ด์•ผ ํ•œ๋‹ค(๋Œ€๋ฒ•์› 2010. 7. 15. ์„ ๊ณ  2010๋„3544 ํŒ๊ฒฐ ๋“ฑ ์ฐธ์กฐ). ๋‹ค. ํŒ๋‹จ ์ด ๋ฒ•์›์ด ์ฑ„ํƒํ•˜์—ฌ ์กฐ์‚ฌํ•œ ์ฆ๊ฑฐ๋“ค์„ ์ข…ํ•ฉํ•˜๋ฉด, ๊ณต์†Œ์™ธ 6์ด ๋ฐ˜๊ตญ๊ฐ€๋‹จ์ฒด์˜ ์ด์ต์ด ๋œ๋‹ค๋Š” ์ • ๋˜๋Š” ๊ตญ๊ฐ€์˜ ์กด๋ฆฝ, ์•ˆ์ „์ด๋‚˜ ์ž์œ ๋ฏผ์ฃผ์  ๊ธฐ๋ณธ์งˆ์„œ๋ฅผ ์œ„ํƒœ๋กญ๊ฒŒ ํ•œ๋‹ค๋Š” ์ •์„ ์•Œ๋ฉด์„œ ๊ทธ ๊ตฌ์„ฑ์› ๋˜๋Š” ๊ทธ ์ง€๋ น์„ ๋ฐ›์€ ์ž์™€ ํšŒํ•ฉํ•˜๊ณ  ๊ทธ๋กœ๋ถ€ํ„ฐ ๊ธˆํ’ˆ์„ ์ˆ˜์ˆ˜ํ•œ ์‚ฌ์‹ค, ํ”ผ๊ณ ์ธ์€ ๊ณต์†Œ์™ธ 6๊ณผ ๋Œ€๋‚จ๊ณต์ž‘์›์„ ๋งŒ๋‚˜๋Š” ์‹œ๊ฐ„ ๋ฐ ์ˆ˜์ˆ˜ํ•œ ๊ธˆํ’ˆ์„ ๊ฐ€์ง€๊ณ  ๋“ค์–ด์˜ค๋Š” ๋ฐฉ๋ฒ• ๋“ฑ์— ๊ด€ํ•˜์—ฌ ๋ฏธ๋ฆฌ ์ƒ์˜ํ•˜๊ณ  ๋Œ€์ฑ…์„ ๋งˆ๋ จํ•œ ์‚ฌ์‹ค, ์ด ๊ณผ์ •์—์„œ ํ”ผ๊ณ ์ธ์€ ๊ณต์†Œ์™ธ 6์˜ ํšŒํ•ฉ, ๊ธˆํ’ˆ์ˆ˜์ˆ˜์— ๋Œ€ํ•˜์—ฌ ๊ณต๋ชจํ•˜๊ณ  ๋ณธ์งˆ์  ๊ธฐ์—ฌ๋ฅผ ํ†ตํ•˜์—ฌ ๊ธฐ๋Šฅ์  ํ–‰์œ„์ง€๋ฐฐ๊ฐ€ ์กด์žฌํ•˜๋Š” ๊ฒƒ์œผ๋กœ ์ธ์ •๋œ๋‹ค. ์ด ํŒ๋‹จ์€ ํŠนํžˆ ๋‹ค์Œ๊ณผ ๊ฐ™์€ ์‚ฌ์ •๋“ค์— ๊ทผ๊ฑฐํ•œ๋‹ค. 1) ํ”ผ๊ณ ์ธ์€ 2015. 3. 30. ์ข…๊ฐ์—ญ (๋ช…์นญ 50 ์ƒ๋žต)์นดํŽ˜์—์„œ ๊ณต์†Œ์™ธ 6์—๊ฒŒ โ€˜3์ผ๋‚  ์ €๋…, 4์ผ๋‚  ์ €๋…, 5์ผ๋‚  ๋งŒ๋‚˜๊ณ  ๋‚˜์„œ..โ€™, โ€˜๋“ค์–ด์˜ฌ ๋•Œ๊ฐ€ ๋ฌธ์ œ์ง€. (์ง์—์„œ) ๋ˆ์ด ์กฐ๊ธˆ ๋‚˜์˜จ ๊ฒƒ ๊ฐ™๋‹ค. ์–ด๋–ค ์ด์ œ ๋งŒ ๋ถˆ ์ด๋‚ด์˜€๊ธฐ ๋•Œ๋ฌธ์— ์ง€๋‚˜์™”๋‹ค. ๋ณ„๋กœ ํฐ ๋ฌธ์ œ๊ฐ€ ์•„๋‹ˆ์—ˆ์ง€. ๊ทผ๋ฐ ์™œ ์ด๋ ‡๊ฒŒ ๋งŽ์ด ๊ฐ€์ ธ์™”๋ƒ? ์ฐ์–ด์„œ ๋•ก๊ฒผ๋Š”๋ฐ ๋”ฐ์„œ..โ€™, โ€˜์ด์ œ๋Š” ์ข€ ์ง€์›๊ธˆ ๊ทœ๋ชจ๊ฐ€ ์ข€ ์ปค์ ธ์„œ ๋ช‡ ๋…„๊ฐ„... ๊ฐ€์ง€๊ณ , ์ž๊ธˆ์„ ์ข€ ๋ถˆ๋ ค์•ผ ๋  ํ•„์š”๊ฐ€ ์žˆ์–ดโ€™, โ€˜๊ฐ€์„œ ๋ง์„ ๋„ˆ๋ฌด ๋น ๋ฅด๊ฒŒ ํ•˜์ง€ ๋ง๊ณ  ์ฒ˜์Œ ๋ณด๋Š” ๊ฑฐ๋‹ˆ๊นŒ, ์ฒœ์ฒœํžˆ ๋ง์„ ํ•ดโ€™, โ€˜์ด๋ฒˆ์ฒ˜๋Ÿผ ์ฐธ ํž˜๋“ค๊ฒŒ ์กฐ๊ฑด์ด ๋งŒ๋“ค์–ด์ง€๋Š”๊ฒŒ ๊ทธ๋ ‡๊ฒŒ ๋งŽ์ง€ ์•Š์•˜์–ดโ€™, โ€˜๊ทผ๋ฐ ํ•˜๋ฐ˜๊ธฐ์— ์ง‘ํ–‰, ์ƒ๋ฐ˜๊ธฐ์— ์ด๋ ‡๊ฒŒ ํ•ด๊ฐ€์ง€๊ณ  4์›” ์ด์ œ ํšŒ์˜ ๋•Œ...โ€™๋ผ๊ณ  ๋งํ•˜์˜€๋‹ค. ์ด๋ ‡๋“ฏ ํ”ผ๊ณ ์ธ์€ ๊ณต์†Œ์™ธ 6์ด ๋ถํ•œ ๊ณต์ž‘์›์„ ๋งŒ๋‚˜๋Š” ์ผ์ •๊ณผ ๋ฐฉ๋ฒ•, ๊ทธ๋กœ๋ถ€ํ„ฐ ๋ˆ์„ ๋ฐ›์•„์„œ ๋ฐ˜์ž…ํ•˜๋Š” ๋ฐฉ๋ฒ• ๋“ฑ์— ๊ด€ํ•˜์—ฌ ๊ตฌ์ฒด์ ์œผ๋กœ ํ•จ๊ป˜ ๋…ผ์˜ํ•˜์˜€๋‹ค. 2) ์ด ๋ฒ•์ •์— ์ฆ์ธ์œผ๋กœ ์ถœ์„ํ•œ ๊ตญ๊ฐ€์ •๋ณด์› ์ˆ˜์‚ฌ๊ด€๋“ค๊ณผ ๊ฒฝ์ฐฐ๊ด€์€ ๊ณต์†Œ์™ธ 6์ด 2015. 4. 5. ๋ง๋ ˆ์ด์‹œ์•„ ์ฟ ์•Œ๋ผ๋ฃธํ‘ธ๋ฅด์—์„œ ๋ถํ•œ 225๊ตญ์˜ ๊ณต์ž‘์กฐ์ธ ๊ณต์†Œ์™ธ 40์ฃผ34) , ๊ณต์†Œ์™ธ 39์ฃผ35) ๋ฅผ ๋งŒ๋‚˜๋Š” ๊ฒƒ์„ ์ง์ ‘ ๋ชฉ๊ฒฉํ•˜๊ณ  ์ด๋ฅผ ๋ฏธํ–‰ํ•˜์—ฌ ์‚ฌ์ง„์„ ์ดฌ์˜ํ•˜์˜€๋‹ค๊ณ  ์ง„์ˆ ํ•˜์˜€๋‹ค. 3) ์ˆ˜์‚ฌ๊ธฐ๊ด€์€ ๊ณต์†Œ์™ธ 6๊ณผ ์ ‘์ด‰ํ•œ ์ž๋ฅผ ๋”ฐ๋ผ๊ฐ€์„œ ๊ทธ๊ฐ€ (๋ช…์นญ 52 ์ƒ๋žต) ํ˜ธํ…”์— ๋ฌต๊ณ  ์žˆ๋‹ค๋Š” ์‚ฌ์‹ค์„ ์•Œ๊ฒŒ ๋˜์—ˆ๊ณ , ๊ทธ ํ˜ธํ…”์˜ ์กฐ์‹์‹์‚ฌ ๋ช…๋‹จ์— ๊ณต์†Œ์™ธ 40์˜ ์ด๋ฆ„์ด ์˜ฌ๋ผ ์žˆ์—ˆ๋‹ค. ๊ณต์†Œ์™ธ 40์˜ ์˜๋ฌธ ์ด๋ฆ„์€ ๋ถํ•œ์‹ ๊ธฐ์žฌ๋ฐฉ๋ฒ•์ธ โ€˜(์˜๋ฌธ ์„ฑ๋ช… ์ƒ๋žต)โ€™์ด๊ณ , ์ˆ˜์‚ฌ๊ด€์€ ์œ„ ๋ช…๋‹จ์„ ์ดฌ์˜ํ•˜์—ฌ ์ฆ๊ฑฐ๋กœ ์ œ์ถœํ•˜์˜€๋‹ค. 4) ๊ณต์†Œ์™ธ 6์€ 2015. 4. 7. ๊ตญ๋‚ด์— ์ž…๊ตญํ•˜๋ฉด์„œ 18,900๋‹ฌ๋Ÿฌ๋ฅผ ๊ฐ€์ง€๊ณ  ๋“ค์–ด์˜ค๋‹ค๊ฐ€ ์„ธ๊ด€์—์„œ ๋‹จ์†์„ ๋‹นํ•˜์˜€๋‹ค. ๊ณต์†Œ์™ธ 6์€ ์ˆ˜์‚ฌ๋ฅผ ๋ฐ›์œผ๋ฉด์„œ ํ”ผ๊ณ ์ธ๊ณผ ์ƒ์˜ํ•œ ๋‚ด์šฉ๊ณผ ๊ฐ™์ด ์นด์ง€๋…ธ์—์„œ ๋ˆ์„ ๋”ด ๊ฒƒ์ด๋ผ๊ณ  ์ง„์ˆ ํ•˜์˜€๋‹ค. 5) ํ”ผ๊ณ ์ธ์€ 2015. 4. 5. ์ž์‹ ์˜ ์•„์ดํŒจ๋“œ ์—์–ด2(์ฆ ์ œ135ํ˜ธ)๋ฅผ ์ด์šฉํ•˜์—ฌ โ–กโ–กโ–ก์—์„œ โ€˜์™ธ๊ตญ์—์„œ (๋ช…์นญ 59 ์ƒ๋žต)์€ํ–‰ ์†ก๊ธˆโ€™, โ€˜(๋ช…์นญ 59 ์ƒ๋žต)์€ํ–‰ ์™ธ๊ตญ์—์„œ ์†ก๊ธˆโ€™, โ€˜์™ธํ™˜๊ด€๋ฆฌ๋ฒ•์œ„๋ฐ˜โ€™, โ€˜ํ•ด์™ธ ๋„๋ฐ• ํ•œ๋„โ€™, โ€˜ํ•ด์™ธ๋„๋ฐ• ๋ถˆ๋ฒ•โ€™์„ ๊ฒ€์ƒ‰ํ•˜์˜€๋‹ค. ํ”ผ๊ณ ์ธ์€ ๊ณต์†Œ์™ธ 6์ด ๋ฐ›์€ ๋ˆ์„ ๊ตญ๋‚ด์— ๋ฐ˜์ž…ํ•˜๋Š” ๋ฐฉ๋ฒ•์„ ๋ชจ์ƒ‰ํ–ˆ๋˜ ๊ฒƒ์œผ๋กœ ๋ณด์ธ๋‹ค. 6) ํ”ผ๊ณ ์ธ์€ 2015. 11. 12.๊ฒฝ ๋ถํ•œ์— ์Šคํ…Œ๊ฐ€๋…ธ๊ทธ๋ผํ”ผ๋กœ ์•”ํ˜ธํ™”ํ•œ ๋ณด๊ณ ๋ฌธ์„ ๋ณด๋ƒˆ๋Š”๋ฐ, ๊ทธ ๋ฌธ์„œ์—๋„ ์œ„ ๊ณต์†Œ์™ธ 6์˜ ํšŒํ•ฉ ๋ฐ ๊ธˆํ’ˆ์ˆ˜์ˆ˜์™€ ๊ด€๋ จ๋œ ๊ธฐ์žฌ๊ฐ€ ์žˆ๋‹ค. ์ฆ‰, ํ”ผ๊ณ ์ธ์ด ์ž‘์„ฑํ•œ ์Šคํ…Œ๊ฐ€๋…ธ๊ทธ๋ผํ”ผ(toyou11-12.docx)๋ฅผ ํ•ด๋…ํ•œ ๋ฌธ์„œ์— โ€˜์ดˆ๊ธฐ ์ž๊ธˆ ์กฐ๋‹ฌ๋กœ ์ธํ•œ ์–ด๋ ค์›€์€ ์™„์ „ํžˆ ๊ทน๋ณต๋˜์—ˆ์œผ๋ฉฐ, ๋ชจ๋“  ํ™œ๋™์„ ์ •์ƒํ™”ํ–ˆ์Œ. ์ž๊ธˆ๋Œ€์ฑ…์œผ๋กœ ๋‚ด๋…„๋ถ€ํ„ฐ๋Š” ์ผ์ •์ด ์ •ํ•ด์ง€๋ฉด 2์ธ ์ด์ƒ์ด ๊ฐ๊ธฐ ๋‹ค๋ฅธ ๋‚  ์ถœ๊ตญ, ์ž…๊ตญํ•ด ๋ณธ์‚ฌ ๋ฉด๋‹ด ํ›„ ๋งŒ๋‚˜ ์ƒํ˜ธ 1๋งŒ ์”ฉ ๋‚˜๋ˆ„๋Š” ๋ฐฉ๋ฒ•์œผ๋กœ ์•ˆ์ „์„ ๋ณด์žฅํ•˜๋ คํ•จ. ์ตœ๊ทผ ์ž์œ ์—ฌํ–‰ ํ™œ์„ฑํ™”๋กœ ์ ์€ ๋น„์šฉ์œผ๋กœ ํ•ด์™ธ์—ฌํ–‰์ด ๊ฐ€๋Šฅํ•ด์ง. ๋น„์šฉ๋ณด๋‹ค ์•ˆ์ „ ์šฐ์„  ์›์น™์œผ๋กœ ํ•˜๊ฒ ์Œโ€™์ด๋ผ๊ณ  ๊ธฐ์žฌ๋˜์–ด ์ฃผ36) ์žˆ๋‹ค. 7) ์œ„ 2015. 11. 12.๊ฒฝ ๋ณด๊ณ ๋ฌธ์—๋Š” โ€˜์‹ ๋ฌธ์‚ฌ์—…๊ณผ ์—ฐ๊ตฌ์†Œ์‚ฌ์—…, ์šฐ๋ฆฌ์กฐ์ง์‚ฌ์—… ๋“ฑ ์ข…ํ•ฉ์  ๋ชฉ์ ์„ ์œ„ํ•ด 9์ธ์Šน ์ฐจ๋Ÿ‰์„ ๊ตฌ์ž…ํ–ˆ์Œ. 5๋…„(60๊ฐœ์›”) ํ• ๋ถ€๋กœ ์ฐจ๋Ÿ‰๊ฐ€๊ฒฉ์€ 3๋งŒ ๋ถˆ ์ •๋„ํ•จ. ์‹ ๋ฌธ์‚ฌ์˜ ์ˆ˜์ž…์ผ๋ถ€๊ฐ€ ๋“ค์–ด๊ฐ€๋‚˜ ๋ณธ์‚ฌ์˜ ์ง€์›์„ ์—ผ๋‘์— ๋‘” ๊ฒƒ์ž„. ์˜ฌํ•ด์ˆ˜์ค€์—์„œ ์ž๊ธˆ ์ง€์› ์š”์ฒญํ•จ. ํ–ฅํ›„ ์‹ ๋ฌธ ํ†ตํ•ด ์‹ ๋ถ„์•ˆ์ „๊ณผ ์ •๋ณด์ˆ˜์ง‘, ํ•ด์™ธ์—ฌํ–‰, ๋Œ€์ค‘๊ต์–‘ ๋“ฑ ๋‹ค์–‘ํ•œ ์ด์ต์„ ๋ณผ ์ˆ˜ ์žˆ์Œโ€™์ด๋ผ๋Š” ๊ธฐ์žฌ๋„ ์žˆ๋‹ค. ํ”ผ๊ณ ์ธ์€ ์‹ค์ œ๋กœ 2015. 10. 29. ์‹ ๋ฌธ์‚ฌ์ธ (๋ช…์นญ 13 ์ƒ๋žต)์˜ ๋ช…์˜๋กœ 9์ธ์Šน ์ฐจ๋Ÿ‰์ธ ์ฝ”๋ž€๋„ ํˆฌ๋ฆฌ์Šค๋ชจ๋ฅผ ๊ตฌ์ž…ํ•˜์˜€๋‹ค. ๋”ฐ๋ผ์„œ ๊ณต์†Œ์™ธ 6์ด ๋ฐ›์•„์˜จ ๋ˆ์ด ์œ„ ์ฐจ๋Ÿ‰์˜ ๊ตฌ์ž… ๋“ฑ์— ์‚ฌ์šฉ๋˜์—ˆ์„ ๊ฒƒ์œผ๋กœ ๋ณด์ธ๋‹ค. ๋ผ. ์†Œ๊ฒฐ ๋”ฐ๋ผ์„œ ํ”ผ๊ณ ์ธ๊ณผ ๋ณ€ํ˜ธ์ธ๋“ค์˜ ์ด ๋ถ€๋ถ„ ์ฃผ์žฅ์€ ๋ฐ›์•„๋“ค์ด์ง€ ์•Š๋Š”๋‹ค. 21. 2011. 11.๊ฒฝ ํ†ต์‹ ์—ฐ๋ฝ์œผ๋กœ ์ธํ•œ ๊ตญ๊ฐ€๋ณด์•ˆ๋ฒ•์œ„๋ฐ˜(ํšŒํ•ฉยทํ†ต์‹ )์˜ ์ ์— ๊ด€ํ•˜์—ฌ ๊ฐ€. ์ฃผ์žฅ ํ”ผ๊ณ ์ธ์€ ๋ฐ˜๊ตญ๊ฐ€๋‹จ์ฒด์™€ ํ†ต์‹ ์—ฐ๋ฝํ•œ ์‚ฌ์‹ค์ด ์—†๊ณ  ํŒŒ์ผ์„ ๋ณด๋‚ธ ์‚ฌ๋žŒ์ด ๋ถํ•œ์˜ 225๊ตญ ๊ณต์ž‘์›์ด๋ผ๋Š” ์ฆ๊ฑฐ๋„ ์—†๋‹ค. ์ˆ˜์‚ฌ๊ด€๋“ค์€ 2008๋…„๊ฒฝ๋ถ€ํ„ฐ ์ด ๋ฌธ๊ฑด๊ณผ ์œ ์‚ฌํ•œ ๋‚ด์šฉ์˜ ๋ณด๊ณ ๋ฌธ์„ ํ™•๋ณดํ•˜๊ณ  ์žˆ์—ˆ์œผ๋ฏ€๋กœ ์ด ํŒŒ์ผ๋“ค๋„ ์ˆ˜์‚ฌ๊ธฐ๊ด€์ด ๋ชฐ๋ž˜ ์ž‘์„ฑํ•˜์—ฌ ์ €์žฅํ•ด ๋†“์•˜์„ ๊ฐ€๋Šฅ์„ฑ์„ ๋ฐฐ์ œํ•  ์ˆ˜ ์—†๋‹ค. ๋˜ํ•œ ํ”ผ๊ณ ์ธ์€ ๊ตญ๊ฐ€์˜ ์กด๋ฆฝยท์•ˆ์ „์ด๋‚˜ ์ž์œ ๋ฏผ์ฃผ์  ๊ธฐ๋ณธ์งˆ์„œ๋ฅผ ์œ„ํƒœ๋กญ๊ฒŒ ํ•œ๋‹ค๋Š” ์ •์„ ์•Œ๋ฉด์„œ ์œ„์™€ ๊ฐ™์€ ํ–‰์œ„๋ฅผ ํ•œ ๊ฒƒ์ด ์•„๋‹ˆ๋‹ค. ๊ฒฐ๊ตญ ํ”ผ๊ณ ์ธ์€ ๊ณต์†Œ์‚ฌ์‹ค ๊ธฐ์žฌ ๋ฒ”์ฃ„๋ฅผ ์ €์ง€๋ฅด์ง€ ์•Š์•˜๋‹ค. ๋‚˜. ํŒ๋‹จ ์ด ๋ฒ•์›์ด ์ฑ„ํƒํ•˜์—ฌ ์กฐ์‚ฌํ•œ ์ฆ๊ฑฐ๋“ค์— ์˜ํ•˜์—ฌ ์ธ์ •๋˜๋Š” ๋‹ค์Œ๊ณผ ๊ฐ™์€ ์‚ฌ์ •๋“ค์„ ์ข…ํ•ฉํ•˜๋ฉด, ํ”ผ๊ณ ์ธ์ด ๋ถํ•œ์˜ 225๊ตญ ์†Œ์† ๊ณต์ž‘์›์œผ๋กœ๋ถ€ํ„ฐ ๊ณต์†Œ์‚ฌ์‹ค ๊ธฐ์žฌ ํŒŒ์ผ์„ ๋ฐ›์•˜๋‹ค๊ณ  ์ธ์ •ํ•  ์ˆ˜ ์žˆ๊ณ , ํ”ผ๊ณ ์ธ์€ ๊ทธ ํŒŒ์ผ์— ๊ธฐ์žฌ๋œ ๋‚ด์šฉ๋Œ€๋กœ ํ–‰ํ•จ์œผ๋กœ์จ ๋ถํ•œ๊ณผ ์ง€์†์ ์œผ๋กœ ํ†ต์‹ ํ•  ์ˆ˜ ์žˆ๊ฒŒ ๋˜์—ˆ์œผ๋ฉฐ, ๋ถํ•œ์ด ์ด๋ฅผ ์šฐ๋ฆฌ ๋‚ด๋ถ€์˜ ๊ต๋ž€์ฑ… ๋“ฑ์œผ๋กœ ์•…์šฉํ•  ์†Œ์ง€๊ฐ€ ์žˆ๋‹ค. ๋”ฐ๋ผ์„œ ์ด๋Ÿฌํ•œ ํ–‰์œ„๋Š” ๊ตญ๊ฐ€์˜ ์กด๋ฆฝยท์•ˆ์ „์ด๋‚˜ ์ž์œ ๋ฏผ์ฃผ์  ๊ธฐ๋ณธ์งˆ์„œ๋ฅผ ์œ„ํƒœ๋กญ๊ฒŒ ํ•˜๋Š” ํ–‰์œ„๋กœ ํ‰๊ฐ€๋˜๊ณ , ํ”ผ๊ณ ์ธ ์—ญ์‹œ ์ด๋Ÿฌํ•œ ์‚ฌ์ •์„ ์•Œ์•˜๋‹ค๊ณ  ํŒ๋‹จ๋œ๋‹ค. 1) 'info.docx' ํŒŒ์ผ์€ ํ”ผ๊ณ ์ธ ์ฐจ๋Ÿ‰ ๋‚ด ์žฌ๋–จ์ด ์•ˆ์—์„œ ์••์ˆ˜๋œ USB(์ฆ ์ œ126ํ˜ธ)์—์„œ ๋ฐœ๊ฒฌ๋œ ๊ฒƒ์œผ๋กœ ํ”ผ๊ณ ์ธ์˜ ๋ฐฐํƒ€์  ์ง€๋ฐฐ์˜์—ญ์—์„œ ๋ฐœ๊ฒฌ๋œ ๊ฒƒ์ด๋‹ค. ์œ„ ์ฐจ๋Ÿ‰์„ ์••์ˆ˜์ˆ˜์ƒ‰ํ•  ๋•Œ ์ˆ˜์‚ฌ๊ด€์€ ์œ„ USB์— ์—ฐ๊ฒฐํ•  ๋งŒํ•œ ๋””์ง€ํ„ธ ๊ธฐ๊ธฐ๋ฅผ ์†Œ์ง€ํ•˜๊ณ  ์žˆ์ง€๋„ ์•Š์•˜๊ณ , ๊ณต์†Œ์™ธ 7์ด ์ˆ˜์‚ฌ๊ด€๊ณผ ๋™ํ–‰ํ•˜์—ฌ ์ฐธ์—ฌํ•˜์˜€๋‹ค. ๋”ฐ๋ผ์„œ ์ˆ˜์‚ฌ๊ด€์ด ํŒŒ์ผ์„ ๋ชฐ๋ž˜ ๋„ฃ๋Š”๋‹ค๋Š” ๊ฒƒ์€ ๋‹น์‹œ ์‹œ๊ฐ„, ์žฅ์†Œ ๋“ฑ ์ •ํ™ฉ์ƒ ๋ถˆ๊ฐ€๋Šฅํ•˜๋‹ค. 2) ํ”ผ๊ณ ์ธ์œผ๋กœ๋ถ€ํ„ฐ ์••์ˆ˜ํ•œ ์ฐฌ์†ก๊ฐ€ ์ฑ…์ž(์ฆ ์ œ110ํ˜ธ)์— ํ”ผ๊ณ ์ธ์ด ์žํ•„๋กœ ๊ธฐ์žฌํ•ด๋‘” ํŒŒ์ผ์˜ ์ด๋ฆ„๊ณผ ์ฆ ์ œ126ํ˜ธ์— ์ €์žฅ๋˜์–ด ์žˆ๋Š” ํŒŒ์ผ์ด ๋™์ผํ•˜๊ณ  ์œ„ ์ฐฌ์†ก๊ฐ€ ์ฑ…์ž์— ๊ธฐ์žฌ๋œ ๋ฉ”๋ชจ๊ฐ€ ์ฆ ์ œ126ํ˜ธ USB์— ์ €์žฅ๋˜์–ด ์žˆ๋Š” ํŒŒ์ผ์˜ ๋‚ด์šฉ๊ณผ ์—ญ์‹œ ์ผ์น˜ํ•˜๋ฉฐ ์ˆ˜์‚ฌ๊ธฐ๊ด€์€ ์œ„ ๋ฉ”๋ชจ์— ๊ธฐ์žฌ๋œ ๋ฐฉ๋ฒ•๋Œ€๋กœ ์ด ํŒŒ์ผ์„ ๋ณตํ˜ธํ™”ํ•˜์˜€๋‹ค. ๋”ฐ๋ผ์„œ ์œ„ USB๋Š” ํ”ผ๊ณ ์ธ์ด ์‚ฌ์šฉํ•œ ๊ฒƒ์œผ๋กœ ๋ณด์ธ๋‹ค. 3) ์Šคํ…Œ๊ฐ€๋…ธ๊ทธ๋ผํ”ผ ํ”„๋กœ๊ทธ๋žจ์€ ํŒŒ์ผ์„ ์•”ํ˜ธํ™”ํ•  ๋•Œ ํŒŒ์ผ ๋‚ด ๋ฌธ์ž๋ฅผ ์ˆซ์ž๋กœ ์น˜ํ™˜ํ•˜๋Š” ๋ฐฉ์‹์œผ๋กœ ์ž‘๋™ํ•˜๊ณ  ์ด๋ฅผ ์œ„ํ•ด ์ฆ ์ œ126ํ˜ธ USB ๋‚ด โ€˜Setup.exe' ํŒŒ์ผ์—๋Š” 500์—ฌ๊ฐœ ์ด์ƒ์˜ ๋‹จ์–ด๋ฅผ ํฌํ•จํ•˜๊ณ  ์žˆ๋Š”๋ฐ, ๊ทธ ์•ˆ์— โ€™๋‚จ์กฐ์„ ยท์ „๊ตญ๋ จํ•ฉยท๋ฏผ์ฃผ๋กœ์ดโ€˜ ๋“ฑ ๋ถํ•œ์‹ ์šฉ์–ด์™€ โ€™์œ„๋Œ€ํ•œ ๋ น๋„์ž ๊น€์ •์ผ ๋™์ง€ยท์œ„๋Œ€ํ•œ ์ˆ˜๋ น ๊น€์ผ์„ฑ ๋™์ง€โ€˜ ๋“ฑ ๋ถํ•œ์„ ์ฐฌ์–‘ํ•˜๋Š” ๋ฌธ๊ตฌ๊ฐ€ ์žˆ๋‹ค. 4) ์ด ๋ฌธ๊ฑด์˜ ๋‚ด์šฉ์€ ๋ฌธ์„œ๋ฅผ ์•”ํ˜ธํ™”ํ•˜๋Š” ๋‚ด์šฉ, ํ†ต์‹ ์„ ์œ„ํ•œ ์ด๋ฉ”์ผ์ฃผ์†Œ์™€ ๊ทธ ์ฃผ37) ์•”ํ˜ธ์ด๋‹ค. ์ด๋ฅผ ๋‹จ์ˆœํžˆ ์˜๋ก€์ ยท์‚ฌ๊ต์  ์ฐจ์›์˜ ๋ฌธ๊ฑด์ด๋ผ๊ณ  ๋ณด๊ธฐ๋Š” ์–ด๋ ต๊ณ , ์ดํ›„ ์—ฌ๊ธฐ์„œ ์ฐพ์•„๋‚ธ ์ด๋ฉ”์ผ์ฃผ์†Œ๋กœ ์‹ค์ œ๋กœ ๋ถํ•œ๊ณผ์˜ ํ†ต์‹ ยทํŽธ์˜์ œ๊ณต์ด ์ด๋ฃจ์–ด์กŒ๋‹ค. ๋‹ค. ์†Œ๊ฒฐ ํ”ผ๊ณ ์ธ๊ณผ ๋ณ€ํ˜ธ์ธ๋“ค์˜ ์ด ๋ถ€๋ถ„ ์ฃผ์žฅ์€ ๋ฐ›์•„๋“ค์ด์ง€ ์•Š๋Š”๋‹ค. 22. 2011. 11. ~ 12.๊ฒฝ ํ†ต์‹ ์—ฐ๋ฝ์œผ๋กœ ์ธํ•œ ๊ตญ๊ฐ€๋ณด์•ˆ๋ฒ•์œ„๋ฐ˜(ํšŒํ•ฉยทํ†ต์‹ ), ๊ตญ๊ฐ€๋ณด์•ˆ๋ฒ•์œ„๋ฐ˜(ํŽธ์˜์ œ๊ณต)์˜ ์ ์— ๊ด€ํ•˜์—ฌ ๊ฐ€. ์ฃผ์žฅ 1) ํ”ผ๊ณ ์ธ์€ ์œ„ ํŒŒ์ผ์„ ๋งŒ๋“ค๊ฑฐ๋‚˜ ๋ฐ˜๊ตญ๊ฐ€๋‹จ์ฒด ์ธก์— ์ „๋‹ฌํ•˜์ง€ ์•Š์•˜๋‹ค. 2) to you.docx ํŒŒ์ผ์˜ ์ƒ์„ฑ์‹œ๊ฐ„์€ 2011. 12. 12. 11:25:48์ž„์— ๋ฐ˜ํ•ด ์ตœ์ข…์ˆ˜์ •์‹œ๊ฐ„์ด 2011. 11. 29. 15:50:32๋กœ ์ƒ์„ฑ์‹œ๊ฐ„๊ณผ ์ˆ˜์ •์‹œ๊ฐ„์ด ๊ฑฐ๊พธ๋กœ ๋˜์–ด ์žˆ์–ด ์ œ3์ž๊ฐ€ ํŒŒ์ผ์„ ์ž‘์„ฑํ•˜์—ฌ ํ”ผ๊ณ ์ธ์˜ USB์— ์˜ฎ๊ฒจ ๋†“์€ ๊ฒƒ์ด๋‹ค. ๋˜ํ•œ, ์ˆ˜์‚ฌ๊ด€๋“ค์€ 2008๋…„๊ฒฝ๋ถ€ํ„ฐ ํ”ผ๊ณ ์ธ์ด ์ž‘์„ฑํ–ˆ๋‹ค๋Š” ์ด ์‚ฌ๊ฑด ๋ฌธ๊ฑด๊ณผ ์œ ์‚ฌํ•œ ๋‚ด์šฉ์˜ ๋ณด๊ณ ๋ฌธ๋„ ํ™•๋ณดํ•˜๊ณ  ์žˆ์—ˆ์œผ๋ฏ€๋กœ ์ด ํŒŒ์ผ๋“ค๋„ ์ˆ˜์‚ฌ๊ธฐ๊ด€์ด ๋ชฐ๋ž˜ ์ž‘์„ฑํ•˜์—ฌ ์ €์žฅํ•ด ๋†“์•˜์„ ๊ฐ€๋Šฅ์„ฑ์ด ์žˆ๋‹ค. 3) ์ด ํŒŒ์ผ์„ ์ „๋‹ฌ๋ฐ›์€ ์‚ฌ๋žŒ์ด ๋ฐ˜๊ตญ๊ฐ€๋‹จ์ฒด ๊ตฌ์„ฑ์›์ด๋ผ๋Š” ์ ๋„ ์ธ์ •ํ•  ์ˆ˜ ์—†๋‹ค. ๋‚˜. ๊ด€๋ จ๋ฒ•๋ฆฌ ๊ตญ๊ฐ€๋ณด์•ˆ๋ฒ• ์ œ9์กฐ ์ œ2ํ•ญ์˜ ๊ธฐํƒ€์˜ ๋ฐฉ๋ฒ•์œผ๋กœ ํŽธ์˜๋ฅผ ์ œ๊ณตํ•œ ๊ฒฝ์šฐ๋ž€ ๊ตญ๊ฐ€๋ณด์•ˆ๋ฒ• ์œ„๋ฐ˜ ๋ฒ”์ธ์— ๋Œ€ํ•˜์—ฌ ์ดํฌยทํƒ„์•ฝ, ๊ธฐํƒ€ ๋ฌด๊ธฐ, ๊ธˆํ’ˆ ๊ธฐํƒ€ ์žฌ์‚ฐ์ƒ ์ด์ต์˜ ์ œ๊ณตํ–‰์œ„, ์ž ๋ณตยทํšŒํ•ฉยทํ†ต์‹ ยท์—ฐ๋ฝ์„ ์œ„ํ•œ ์žฅ์†Œ์ œ๊ณตํ–‰์œ„๋ฅผ ์ œ์™ธํ•œ ๋ชจ๋“  ๋ฐฉ๋ฒ•์œผ๋กœ ํ•˜๋Š” ์ผ์ฒด์˜ ํŽธ์˜์ œ๊ณต ํ–‰์œ„๋ฅผ ํฌํ•จํ•œ๋‹ค๊ณ  ํ•  ๊ฒƒ์ด๊ณ (๋Œ€๋ฒ•์› 1984. 10. 10. ์„ ๊ณ  84๋„1796 ํŒ๊ฒฐ ์ฐธ์กฐ), ํŽธ์˜์ œ๊ณต์ฃ„์˜ ์ฒ˜๋ฒŒ ๋Œ€์ƒ์ธ ํ–‰์œ„๋Š” ๊ตญ๊ฐ€์˜ ์กด๋ฆฝยท์•ˆ์ „์ด๋‚˜ ์ž์œ ๋ฏผ์ฃผ์  ๊ธฐ๋ณธ์งˆ์„œ์— ์‹ค์งˆ์  ํ•ด์•…์„ ๋ฏธ์น  ๊ตฌ์ฒด์ ์ด๊ณ  ๋ช…๋ฐฑํ•œ ์œ„ํ—˜์„ฑ์ด ์žˆ๋Š” ๊ฒฝ์šฐ๋กœ ์ œํ•œ๋˜์–ด์•ผ ํ•œ๋‹ค(๋Œ€๋ฒ•์› 2013. 6. 28. ์„ ๊ณ  2010๋„3810 ํŒ๊ฒฐ ์ฐธ์กฐ). ๊ทธ๋ฆฌ๊ณ  ๊ตญ๊ฐ€๋ณด์•ˆ๋ฒ• ์ œ9์กฐ ์ œ2ํ•ญ์—์„œ ์ •ํ•œ ํŽธ์˜์ œ๊ณต์˜ ์ฃ„๋Š” ๊ฐ™์€ ๋ฒ• ์ œ3์กฐ ๋‚ด์ง€ ์ œ8์กฐ์˜ ์ฃ„๋ฅผ ๋ฒ”ํ•˜๊ฑฐ๋‚˜ ๋ฒ”ํ•˜๋ ค๋Š” ์ž๋ผ๋Š” ์ •์„ ์•Œ๋ฉด์„œ ์œ„์™€ ๊ฐ™์€ ํŽธ์˜๋ฅผ ์ œ๊ณตํ•œ ๋•Œ์— ์„ฑ๋ฆฝํ•˜๋Š”๋ฐ, ์—ฌ๊ธฐ์„œ ํŽธ์˜ ์ œ๊ณต์˜ ์ƒ๋Œ€๋ฐฉ์ด ๊ตญ๊ฐ€๋ณด์•ˆ๋ฒ• ์ œ3์กฐ ๋‚ด์ง€ ์ œ8์กฐ์˜ ์ฃ„๋ฅผ ๋ฒ”ํ•˜๊ฑฐ๋‚˜ ๋ฒ”ํ•˜๋ ค๋Š” ์ž๋ผ๋Š” ์ •์— ๋Œ€ํ•˜์—ฌ๋Š” ํ™•์ •์  ์ธ์‹๊นŒ์ง€๋Š” ํ•„์š” ์—†๊ณ  ๋ฏธํ•„์  ์ธ์‹์„ ๊ฐ€์ง€๊ณ  ์žˆ์œผ๋ฉด ์ถฉ๋ถ„ํ•˜๋‹ค(๋Œ€๋ฒ•์› 2012. 10. 25. ์„ ๊ณ  2010๋„6310 ํŒ๊ฒฐ ์ฐธ์กฐ). ๋‹ค. ํŒ๋‹จ ์ด ๋ฒ•์›์ด ์ฑ„ํƒํ•˜์—ฌ ์กฐ์‚ฌํ•œ ์ฆ๊ฑฐ๋“ค์— ์˜ํ•˜์—ฌ ์ธ์ •๋˜๋Š” ๋‹ค์Œ๊ณผ ๊ฐ™์€ ์‚ฌ์ •๋“ค์„ ์ข…ํ•ฉํ•˜๋ฉด, ํ”ผ๊ณ ์ธ์ด ๋ถํ•œ์˜ 225๊ตญ ์†Œ์† ๊ณต์ž‘์›์—๊ฒŒ ํŒ์‹œ ๋ฒ”์ฃ„์‚ฌ์‹ค ๊ธฐ์žฌ ํŒŒ์ผ์„ ๋ถˆ์ƒ์˜ ๋ฐฉ๋ฒ•์œผ๋กœ ์ „์†กํ•˜์—ฌ ํ†ต์‹ ์—ฐ๋ฝํ•˜์˜€๊ณ , ๋ถํ•œ ๋‚ด์ง€ ๊ทธ ์ง€๋ น์„ ๋ฐ›์€ ์ž์—๊ฒŒ ์ •๋ณดยท์ž๋ฃŒ๋ฅผ ์ œ๊ณตํ•˜์—ฌ ๊ทธ ํ™œ๋™์— ๋„์›€์ด ๋˜๋„๋ก ํŽธ์˜๋ฅผ ์ œ๊ณตํ•˜์˜€๋‹ค๊ณ  ์ธ์ •๋œ๋‹ค. ์ด๋Ÿฌํ•œ ํ–‰์œ„๋Š” ๊ตญ๊ฐ€์˜ ์กด๋ฆฝยท์•ˆ์ „์ด๋‚˜ ์ž์œ ๋ฏผ์ฃผ์  ๊ธฐ๋ณธ์งˆ์„œ์— ์‹ค์งˆ์  ํ•ด์•…์„ ๋ฏธ์น  ์œ„ํ—˜ํ•œ ํ–‰์œ„๋กœ ํ‰๊ฐ€๋˜๊ณ , ํ”ผ๊ณ ์ธ ์—ญ์‹œ ์ด๋Ÿฌํ•œ ์‚ฌ์ •์„ ์•Œ์•˜๋‹ค๊ณ  ๋ด„์ด ์ƒ๋‹นํ•˜๋‹ค. 1) 'toyou.docx' ํŒŒ์ผ์€ ์ฆ ์ œ126ํ˜ธ USB์—์„œ ๋ฐœ๊ฒฌ๋œ ๊ฒƒ์ด๊ณ , ์•ž์„œ ๋ณธ ๋ฐ”์™€ ๊ฐ™์ด ์ˆ˜์‚ฌ๊ธฐ๊ด€์ด ๋ชฐ๋ž˜ ํŒŒ์ผ์„ ๋„ฃ๋Š”๋‹ค๋Š” ๊ฒƒ์€ ๋ถˆ๊ฐ€๋Šฅํ•ด ๋ณด์ธ๋‹ค. 2) ์œ„ ํŒŒ์ผ์˜ ์ƒ์„ฑ์‹œ๊ฐ„์ด ์ตœ์ข…์ˆ˜์ •์‹œ๊ฐ„ ๋ณด๋‹ค ๋Šฆ์€ ์‹œ์ ์ธ ๊ฒƒ์€ ์ธ์ •๋œ๋‹ค. ๊ทธ๋Ÿฌ๋‚˜ ๊ตญ์ •์› ์ˆ˜์‚ฌ๊ด€ ๊ณต์†Œ์™ธ 3์˜ ์ฆ์–ธ์— ์˜ํ•˜๋ฉด, ์ €์žฅ๋งค์ฒด์˜ ์ƒ์„ฑ์‹œ๊ฐ„๊ณผ ์ˆ˜์ •์‹œ๊ฐ„์€ ์ €์žฅ๊ธฐ๊ธฐ์˜ ํŠน์„ฑ์„ ํƒ€๋Š”๋ฐ, toyou.docx ํŒŒ์ผ์ด ์ƒ์„ฑ๋œ ํ›„ ๋งˆ์ง€๋ง‰์œผ๋กœ ์ˆ˜์ •์ด ๋˜๋ฉด ๊ทธ ์ˆ˜์ •์ž‘์—…์„ ํ•œ ์‹œ๊ฐ„์ด ์ ํžˆ๊ฒŒ ๋˜๋Š” ํ•œํŽธ, ๊ทธ ํŒŒ์ผ์ด ๋‹ค๋ฅธ ๋งค์ฒด(์œ„ ์ฆ ์ œ126ํ˜ธ)๋กœ ๊ทธ๋Œ€๋กœ ๋ณต์‚ฌ๊ฐ€ ๋˜๋ฉด ๊ทธ ์‹œ๊ฐ„์ด ์ƒ์„ฑ์‹œ๊ฐ„์ด ๋˜๊ณ  ์ˆ˜์ •์‹œ๊ฐ„์€ ๊ณผ๊ฑฐ์— ์žˆ์—ˆ๋˜ ๊ทธ ์ˆ˜์ •์‹œ๊ฐ„์„ ์œ ์ง€ํ•˜๊ฒŒ ๋˜์–ด ์ˆ˜์ •์‹œ๊ฐ„๊ณผ ์ƒ์„ฑ์‹œ๊ฐ„์˜ ์—ญ์ „์ด ์ƒ๊ธธ ์ˆ˜ ์žˆ๋‹ค ํ•˜๋ฏ€๋กœ ํฌ๊ฒŒ ์ด์ƒํ•˜๊ฒŒ ๋ณด์ด์ง€๋Š” ์•Š๋Š”๋‹ค. ๋˜ํ•œ, ์œ„ ํŒŒ์ผ์˜ ์ƒ์„ฑ์‹œ๊ฐ„๊ณผ ์ˆ˜์ •์‹œ๊ฐ„์€ ๋ชจ๋‘ ์ด ์‚ฌ๊ฑด ์••์ˆ˜์ˆ˜์ƒ‰๋ณด๋‹ค๋Š” ํ›จ์”ฌ ์ด์ „์ด๋ฏ€๋กœ ์ˆ˜์‚ฌ๊ธฐ๊ด€์ด ์••์ˆ˜์ˆ˜์ƒ‰ ๋ฌด๋ ต์— ๊ทธ ํŒŒ์ผ์„ ๋ณต์‚ฌํ•ด์„œ ๋„ฃ์—ˆ๋‹ค๊ณ  ํ•  ์ˆ˜๋Š” ์—†๋‹ค. 3) ํŒŒ์ผ์˜ ๋‚ด์šฉ ์ค‘ โ€˜์•”ํ˜ธํ”„๋กœ๊ทธ๋žจ ์‚ฌ์šฉ ๋ฐฉ๋ฒ•์„ ์ˆ™์ง€ํ–ˆ๋‹ค. ์ด ํŒŒ์ผ ๋ณด๊ณ ๊ฐ€ ๊ทธ ์ฆ๊ฑฐ์ด๋‹คโ€™๋ผ๋Š” ๊ธฐ์žฌ๊ฐ€ ์žˆ์œผ๋ฏ€๋กœ ์œ„ ํŒŒ์ผ์„ ์•”ํ˜ธํ™”ํ•œ ๊ณผ์ •์€ ์œ„ 'info.docx' ํŒŒ์ผ์— ๊ธฐ์žฌ๋œ ๋ฐฉ๋ฒ•๋Œ€๋กœ์ธ ๊ฒƒ์œผ๋กœ ๋ณด์ด๊ณ , ์ด ํŒŒ์ผ์„ ๋ณตํ˜ธํ™”ํ•˜๋Š” ๊ฒƒ์€ ์œ„ 'info.docx' ํŒŒ์ผ์„ ๋ณตํ˜ธํ™”ํ•œ ๋ฐฉ๋ฒ•๊ณผ ๋™์ผํ•˜๋‹ค. ์ด ํŒŒ์ผ์—๋Š” 'info.docx' ํŒŒ์ผ์—์„œ ์š”๊ตฌํ•˜์˜€๋˜ ๋ฐ”(์‹ ๋…„๋ฉ”์‹œ์ง€๋ฅผ 12์›” 12์ผ์— ๋ณด๋‚ด๋‹ฌ๋ผ)๋Œ€๋กœ โ€˜12์ผ ์‹ ๋…„๋ณด๊ณ ๋ฅผ ๋“œ๋ฆฌ๊ฒ ๋‹คโ€™๋Š” ๊ธฐ์žฌ๊ฐ€ ์žˆ๋Š”๋ฐ”, ์ด ํŒŒ์ผ์„ ํ•ด๋…ํ•œ ๋ฌธ์„œ๋Š” ์•ž์„œ ๋ณธ 'info.docx' ํŒŒ์ผ์„ ํ•ด๋…ํ•œ ๋ฌธ์„œ์™€ ๊ทธ ๊ธฐ์žฌ ๋‚ด์šฉ์ด ์—ฐ๊ด€๋˜์–ด ์žˆ๋‹ค. 4) ๊ทธ ์™ธ์—๋„ ๋Œ€ํ•œ๋ฏผ๊ตญ ๋‚ด ๋น„๋ฐ€์กฐ์ง์˜ ์šด์˜ ๋ฐ ํ™œ๋™ํ˜„ํ™ฉ, ์กฐ์ง์— ๋Œ€ํ•œ ๊ตญ์ •์›์˜ ์••์ˆ˜ยท์ˆ˜์ƒ‰ ํ˜„ํ™ฉ ๋“ฑ ๋ถํ•œ์ด ๊ถ๊ธˆํ•ดํ•˜๊ณ  ํ•„์š”๋กœ ํ•˜๋Š” ๋‚ด์šฉ์ด ๊ธฐ์žฌ๋˜์–ด ์žˆ๋‹ค. ๋‹ค. ์†Œ๊ฒฐ ํ”ผ๊ณ ์ธ๊ณผ ๋ณ€ํ˜ธ์ธ๋“ค์˜ ์ด ๋ถ€๋ถ„ ์ฃผ์žฅ์€ ๋ฐ›์•„๋“ค์ด์ง€ ์•Š๋Š”๋‹ค. 23. 2013. 7. 7.๊ฒฝ ํ†ต์‹ ์—ฐ๋ฝ์œผ๋กœ ์ธํ•œ ๊ตญ๊ฐ€๋ณด์•ˆ๋ฒ•์œ„๋ฐ˜(ํšŒํ•ฉยทํ†ต์‹ ), ๊ตญ๊ฐ€๋ณด์•ˆ๋ฒ•์œ„๋ฐ˜(ํŽธ์˜์ œ๊ณต)์˜ ์ ์— ๊ด€ํ•˜์—ฌ ๊ฐ€. ์ฃผ์žฅ 1) ํ”ผ๊ณ ์ธ์€ โ€˜toyou7-7.docx' ํŒŒ์ผ์„ ์ƒ์„ฑํ•˜์—ฌ ๋ฐœ์†กํ•˜์ง€ ์•Š์•˜๋‹ค. 2) ์œ„ ํŒŒ์ผ์€ (์˜๋ฌธ ID 1 ์ƒ๋žต)@โ–ฝโ–ฝโ–ฝโ–ฝ.COM์—์„œ (์˜๋ฌธ ID 1 ์ƒ๋žต)@โ–ฝโ–ฝโ–ฝโ–ฝ.COM๊ณผ (์˜๋ฌธ ID 1 ์ƒ๋žต)@โ—Žโ—Žโ—Žโ—Žโ—Ž.com ์•ž์œผ๋กœ ์ „์†ก๋œ ๊ฒƒ์ผ ๋ฟ์ด๊ณ  ์œ„ ์ด๋ฉ”์ผ ๊ณ„์ •์„ ๋ถํ•œ๊ณต์ž‘์›๊ณผ ํ•จ๊ป˜ ์‚ฌ์šฉํ•œ๋‹ค๋Š” ์ ์— ๋Œ€ํ•œ ์ž…์ฆ์ด ์—†๋‹ค. ๋‚˜. ํŒ๋‹จ ์ด ๋ฒ•์›์ด ์ฑ„ํƒํ•˜์—ฌ ์กฐ์‚ฌํ•œ ์ฆ๊ฑฐ๋“ค์— ์˜ํ•˜์—ฌ ์ธ์ •๋˜๋Š” ๋‹ค์Œ๊ณผ ๊ฐ™์€ ์‚ฌ์ •๋“ค์„ ์ข…ํ•ฉํ•˜๋ฉด, ํ”ผ๊ณ ์ธ์ด ๋ถํ•œ์˜ 225๊ตญ ์†Œ์† ๊ณต์ž‘์›์—๊ฒŒ ํŒ์‹œ ๋ฒ”์ฃ„์‚ฌ์‹ค ๊ธฐ์žฌ ํŒŒ์ผ์„ ๋ฉ”์ผ๋กœ ๋ณด๋‚ด ํ†ต์‹ ์—ฐ๋ฝํ•˜์˜€๊ณ , ๋ถํ•œ ๋‚ด์ง€ ๊ทธ ์ง€๋ น์„ ๋ฐ›์€ ์ž์—๊ฒŒ ์ •๋ณดยท์ž๋ฃŒ๋ฅผ ์ œ๊ณตํ•˜์—ฌ ๊ทธ ํ™œ๋™์— ๋„์›€์ด ๋˜๋„๋ก ํŽธ์˜๋ฅผ ์ œ๊ณตํ•˜์˜€๋‹ค๊ณ  ์ธ์ •๋œ๋‹ค. ์ด๋Ÿฌํ•œ ํ–‰์œ„๋Š” ๊ตญ๊ฐ€์˜ ์กด๋ฆฝยท์•ˆ์ „์ด๋‚˜ ์ž์œ ๋ฏผ์ฃผ์  ๊ธฐ๋ณธ์งˆ์„œ์— ์‹ค์งˆ์  ํ•ด์•…์„ ๋ฏธ์น  ์œ„ํ—˜ํ•œ ํ–‰์œ„๋กœ ํ‰๊ฐ€๋˜๊ณ , ํ”ผ๊ณ ์ธ ์—ญ์‹œ ์ด๋Ÿฌํ•œ ์‚ฌ์ •์„ ์•Œ์•˜๋‹ค๊ณ  ๋ด„์ด ์ƒ๋‹นํ•˜๋‹ค. 1) ์œ„ ํŒŒ์ผ์€ ์œ„ โ€˜info.docx ํŒŒ์ผโ€™์˜ ๋ณตํ˜ธํ™”๋œ ๋ฌธ์„œ์—์„œ ์ด๋ฉ”์ผ์ฃผ์†Œ ์ด์šฉ์— ๊ด€ํ•œ โ€˜์•ฝ์†์‚ฌํ•ญโ€™์œผ๋กœ ๊ธฐ์žฌ๋˜์–ด ์žˆ๋˜ ์ด๋ฉ”์ผ ์ฃผ์†Œ(์˜๋ฌธ ID 1 ์ƒ๋žต)์™€ ๋น„๋ฐ€๋ฒˆํ˜ธ๋ฅผ ์ˆ˜์‚ฌ๊ธฐ๊ด€์ด ํ•œ๊ตญ์ธํ„ฐ๋„ท์ง„ํฅ์›์—์„œ ์ž…๋ ฅํ•˜๋Š” ๋ฐฉ๋ฒ•์œผ๋กœ ์••์ˆ˜ยท์ˆ˜์ƒ‰ยท๊ฒ€์ฆํ•œ ๊ฒฐ๊ณผ ๊ทธ๋Œ€๋กœ ์ ‘์†์ด ๋˜์–ด, ๊ทธ๊ณณ์— ์ฒจ๋ถ€๋˜์–ด ์žˆ๋Š” ํŒŒ์ผ์„ ์••์ˆ˜ํ•œ ๊ฒƒ์ด๋‹ค. 2) ์•ž์„œ ๋ณธ ๊ฒƒ๊ณผ ๊ฐ™์ด โ€˜info.docx ํŒŒ์ผโ€™์€ ํ”ผ๊ณ ์ธ์ด ๋ถํ•œ ๊ณต์ž‘์›์œผ๋กœ๋ถ€ํ„ฐ ๋ฐ›์€ ํŒŒ์ผ๋กœ ๋ณด์ด๊ณ , ๊ทธ ๋ฌธ์„œ์˜ ๊ธฐ์žฌ์— ์˜ํ•˜๋ฉด ์œ„ ์ด๋ฉ”์ผ ๊ณ„์ •์€ โ€˜๋ณธ์‚ฌ์™€ ์ง€์‚ฌ ๊ฐ„์— ์‚ฌ์šฉโ€™, ์ฆ‰ ๋ถํ•œ ๊ณต์ž‘์›๊ณผ ํ”ผ๊ณ ์ธ ์‚ฌ์ด์— ์•”ํ˜ธํ™”๋œ ๋ฌธ์„œ๋ฅผ ๊ตํ™˜ํ•˜๊ธฐ ์œ„ํ•œ ์ˆ˜๋‹จ์œผ๋กœ ๋ณด์ธ๋‹ค. 3) โ€˜toyou7-7.docx' ํŒŒ์ผ์„ ๋ณตํ˜ธํ™”ํ•œ ๋ฌธ์„œ์—๋Š” โ€™์ด๋ฉ”์ผ์„ ํ™•์ธํ–ˆ๋‹ค. ์ •์ƒ์„ ๊ณผ ์˜ˆ๋น„์„ ์ด ์ •ํ™•ํ•˜๊ฒŒ ์ ‘์† ๊ฐ€๋™๋˜์—ˆ๋‹ค. ์˜ค๋Š˜ 7์›” 7์ผ ์ดํ›„๋กœ ์ƒˆ๋กœ์šด ์ด๋ฉ”์ผ์„ ์ •์ƒ์„ ๊ณผ ์˜ˆ๋น„์„ ์œผ๋กœ ์“ธ ๊ฒƒ์„ ์ œ์•ˆํ•œ๋‹คโ€˜๊ณ  ๊ธฐ์žฌ๋˜์–ด ์žˆ๊ณ  ์ด๋Š” ์œ„ โ€˜info.docx ํŒŒ์ผโ€™์˜ ๊ธฐ์žฌ์™€ ๋ถ€ํ•ฉํ•˜๋Š” ๊ฒƒ์ด๋‹ค. ๋˜ํ•œ, ์ด๋ฉ”์ผ์„ ๋ฐœ์‹ ํ•œ ๊ณ„์ •๊ณผ ์†ก์‹ ํ•œ ๊ณ„์ •์ด ๋™์ผํ•˜๋‹ค๋Š” ๊ฒƒ์€ ๊ทธ ๋ฉ”์ผ์ž๋ฃŒ๋ฅผ ์ด๋ฉ”์ผ ์„œ๋ฒ„์— ๋‚จ๊ธด๋‹ค, ๊ทธ๋ฆฌํ•˜์—ฌ ๊ทธ๊ณณ์— ์ ‘์†ํ•˜๋Š” ์ž๊ฐ€ ์ด๋ฅผ ์—ด์–ด๋ณผ ์ˆ˜ ์žˆ๊ฒŒ ํ•œ๋‹ค๋Š” ์˜๋ฏธ๋กœ ๋ณด์ด๊ณ , ์•ž์„œ ํ”ผ๊ณ ์ธ์ด ๋ฐ›์€ ๋ฌธ์„œ์™€ ๋ณด๋‚ธ ๋ฌธ์„œ, ์ด ์‚ฌ๊ฑด ๋ฌธ์„œ์˜ ๊ธฐ์žฌ ๋‚ด์šฉ์œผ๋กœ ๋ณด์•„ ๊ทธ ํŒŒ์ผ์„ ์—ด์–ด๋ณผ ์‚ฌ๋žŒ์€ ๋ถํ•œ ๊ณต์ž‘์›์ธ ๊ฒƒ์œผ๋กœ ๋ณด์ธ๋‹ค. 4) ์œ„ ๋ฌธ์„œ์—๋Š” โ€˜ํ˜๋ช…์  ์ธ์‚ฌโ€™์™€ โ€˜์ •์ƒ์„ , ์˜ˆ๋น„์„ ์˜ ์‚ฌ์šฉโ€™ ๊ด€๊ณ„ ์™ธ์—๋„, โ€˜๋น„๋ฐ€์กฐ์ง์˜ ์šด์˜ํ˜„ํ™ฉ, ์›”๋ก€ ๋ชจ์ž„์˜ ์ง„ํ–‰์ƒํ™ฉ, ๋ฏผ์‹ฌ๋™ํ–ฅ ๋ณด๊ณ  ๊ณ„ํšโ€™ ๋“ฑ์— ๊ด€ํ•œ ๊ธฐ์žฌ๋„ ์žˆ๊ณ , ์ด๋Š” ๋ถํ•œ์ด ๊ถ๊ธˆํ•ดํ•˜๊ณ  ํ•„์š”๋กœ ํ•  ๋งŒํ•œ ๋‚ด์šฉ์ด๋‹ค. ๋‹ค. ์†Œ๊ฒฐ ํ”ผ๊ณ ์ธ๊ณผ ๋ณ€ํ˜ธ์ธ๋“ค์˜ ์ด ๋ถ€๋ถ„ ์ฃผ์žฅ์€ ๋ฐ›์•„๋“ค์ด์ง€ ์•Š๋Š”๋‹ค. 24. 2015. 11. 12.๊ฒฝ ํ†ต์‹ ์—ฐ๋ฝ์œผ๋กœ ์ธํ•œ ๊ตญ๊ฐ€๋ณด์•ˆ๋ฒ•์œ„๋ฐ˜(ํšŒํ•ฉยทํ†ต์‹ ), ๊ตญ๊ฐ€๋ณด์•ˆ๋ฒ•์œ„๋ฐ˜(ํŽธ์˜์ œ๊ณต)์˜ ์ ์— ๊ด€ํ•˜์—ฌ ๊ฐ€. ์ฃผ์žฅ 1) ํ”ผ๊ณ ์ธ์€ โ€˜toyou11-12.docx' ํŒŒ์ผ์„ ๋ฐœ์†กํ•˜์ง€ ์•Š์•˜๊ณ , ์ˆ˜์‚ฌ๊ธฐ๊ด€์ด ์กฐ์ž‘ํ•œ ๊ฒƒ์ด๋‹ค. 2) ์ด ํŒŒ์ผ์ด (์˜๋ฌธ ID 2 ์ƒ๋žต)@โ–ฒโ–ฒโ–ฒโ–ฒ.net์—์„œ ๊ฐ™์€ ์ด๋ฉ”์ผ ์ฃผ์†Œ๋กœ ์ „์†ก๋˜์—ˆ๋‹ค๊ณ  ํ•˜๋”๋ผ๋„ ์ด ๋ฉ”์ผ์„ ๋ถํ•œ ๊ณต์ž‘์›์ด ํ•จ๊ป˜ ์‚ฌ์šฉํ•œ๋‹ค๋Š” ์ ์— ๋Œ€ํ•œ ์ฆ๋ช…์ด ์—†๋‹ค. ๋‚˜. ํŒ๋‹จ ์ด ๋ฒ•์›์ด ์ฑ„ํƒํ•˜์—ฌ ์กฐ์‚ฌํ•œ ์ฆ๊ฑฐ๋“ค์— ์˜ํ•˜์—ฌ ์ธ์ •๋˜๋Š” ๋‹ค์Œ๊ณผ ๊ฐ™์€ ์‚ฌ์ •๋“ค์„ ์ข…ํ•ฉํ•˜๋ฉด, ํ”ผ๊ณ ์ธ์ด ๋ถํ•œ์˜ 225๊ตญ ์†Œ์† ๊ณต์ž‘์›์—๊ฒŒ ํŒ์‹œ ๋ฒ”์ฃ„์‚ฌ์‹ค ๊ธฐ์žฌ ํŒŒ์ผ์„ ๋ฉ”์ผ๋กœ ๋ณด๋‚ด ํ†ต์‹ ์—ฐ๋ฝํ•˜์˜€๊ณ , ๋ถํ•œ ๋‚ด์ง€ ๊ทธ ์ง€๋ น์„ ๋ฐ›์€ ์ž์—๊ฒŒ ์ •๋ณดยท์ž๋ฃŒ๋ฅผ ์ œ๊ณตํ•˜์—ฌ ๊ทธ ํ™œ๋™์— ๋„์›€์ด ๋˜๋„๋ก ํŽธ์˜๋ฅผ ์ œ๊ณตํ•˜์˜€๋‹ค๊ณ  ์ธ์ •๋œ๋‹ค. ์ด๋Ÿฌํ•œ ํ–‰์œ„๋Š” ๊ตญ๊ฐ€์˜ ์กด๋ฆฝยท์•ˆ์ „์ด๋‚˜ ์ž์œ ๋ฏผ์ฃผ์  ๊ธฐ๋ณธ์งˆ์„œ์— ์‹ค์งˆ์  ํ•ด์•…์„ ๋ฏธ์น  ์œ„ํ—˜ํ•œ ํ–‰์œ„๋กœ ํ‰๊ฐ€๋˜๊ณ , ํ”ผ๊ณ ์ธ ์—ญ์‹œ ์ด๋Ÿฌํ•œ ์‚ฌ์ •์„ ์•Œ์•˜๋‹ค๊ณ  ๋ด„์ด ์ƒ๋‹นํ•˜๋‹ค. 1) โ€˜toyou11-12.docx' ํŒŒ์ผ์€ ํ”ผ๊ณ ์ธ์˜ ์‹ ์ฒด(์…”์ธ  ์ฃผ๋จธ๋‹ˆ)์—์„œ ๋ฐœ๊ฒฌํ•˜์—ฌ ์••์ˆ˜ํ•œ Micro SD ์นด๋“œ(์ฆ ์ œ123ํ˜ธ)์— ์ €์žฅ๋˜์–ด ์žˆ์—ˆ๋‹ค. ์ฆ ์ œ123ํ˜ธ์˜ ์••์ˆ˜๋‹น์‹œ ํ”ผ๊ณ ์ธ๊ณผ ์ฒ˜ ๊ณต์†Œ์™ธ 8์ด ๊ทธ ๊ณผ์ •์— ์ฐธ์—ฌํ•˜๊ณ  ์žˆ์—ˆ๊ณ  ๊ทธ ๋‚  ๋ด‰์ธํ•œ ๋’ค 2015. 11. 17. ์„ ๋ณ„์••์ˆ˜ํ•˜๊ธฐ ์ „๊นŒ์ง€ ๋ด‰์ธ์ด ํ•ด์ œ๋œ ์ ์ด ์—†์œผ๋ฏ€๋กœ ์ˆ˜์‚ฌ๊ด€๋“ค์ด ์••์ˆ˜ยท์ˆ˜์ƒ‰์„ ํ•˜๋ฉด์„œ ํ”ผ๊ณ ์ธ์˜ ์‹ ์ฒด์—์„œ ๋ฐœ๊ฒฌํ•œ ์ฆ ์ œ123ํ˜ธ์— ๋ชฐ๋ž˜ ํŒŒ์ผ์„ ๋„ฃ์—ˆ๋‹ค๋Š” ๊ฒƒ์€ ๋ถˆ๊ฐ€๋Šฅํ•˜๋‹ค. 2) ํ”ผ๊ณ ์ธ์€ ์ด์ „์—๋„ PC๋ฐฉ์—์„œ โ€˜(์˜๋ฌธ ID 2 ์ƒ๋žต)โ€™์ด๋ผ๋Š” ๋™์ผํ•œ ์•„์ด๋””๋กœ โ–ฝโ–ฝโ–ฝโ–ฝ.COM์— ์ ‘์†ํ•˜์—ฌ ์ด๋ฉ”์ผ์„ ์‚ฌ์šฉํ•œ ์‚ฌ์‹ค์ด ์žˆ๋‹ค. ํ”ผ๊ณ ์ธ์€ 2015. 11. 12. 17:22:10 โ–ณโ–ณ๋Œ€ํ•™๊ต ์ œ1์ข…ํ•ฉ๊ด€ ๋กœ๋น„์— ์žˆ๋Š” ๊ณต์šฉ PC์— ์œ„ ์ฆ ์ œ123ํ˜ธ๋ฅผ ์—ฐ๊ฒฐํ•˜์˜€๊ณ , 17:22:53๊ฒฝ (์˜๋ฌธ ID 2 ์ƒ๋žต)@โ–ฒโ–ฒโ–ฒโ–ฒ.net์— ๋กœ๊ทธ์ธํ•˜์—ฌ 17:23:30๊ฒฝ (์˜๋ฌธ ID 2 ์ƒ๋žต)@โ–ฒโ–ฒโ–ฒโ–ฒ.net๋กœ โ€˜toyou11-12.docx' ํŒŒ์ผ์„ ์ „์†กํ•˜์˜€๋‹ค. ํ”ผ๊ณ ์ธ์€ ์œ„ ๋ฉ”์ผ์„ ๋ณด๋‚ธ ๋‹น์ผ ๋ฐค, ์ง‘์œผ๋กœ ๋“ค์–ด์˜ค๋Š” ๊ธธ์— ์ฒดํฌ๋˜์—ˆ๊ณ  ํ”ผ๊ณ ์ธ์˜ ์…”์ธ ์—์„œ ์ฆ ์ œ123ํ˜ธ๊ฐ€ ์••์ˆ˜๋˜์—ˆ์œผ๋ฏ€๋กœ ์ œ3์ž๊ฐ€ โ€˜toyou11-12.docx' ํŒŒ์ผ์„ ๋ชฐ๋ž˜ ์ €์žฅํ•ด๋‘์—ˆ์„ ๊ฐ€๋Šฅ์„ฑ์€ ์—†๋‹ค. 3) ๋ฉ”์ผ์„ ์ „์†กํ•œ ๊ณ„์ •๊ณผ ์ˆ˜์‹ ํ•œ ๊ณ„์ •์ด ๋™์ผํ•˜๋‹ค๋Š” ๊ฒƒ์€ ๊ทธ ํŒŒ์ผ์„ ์ด๋ฉ”์ผ ์„œ๋ฒ„์— ๋‚จ๊ฒจ๋†“๋Š”๋‹ค, ๊ทธ๋ฆฌํ•˜์—ฌ ๊ทธ ๊ณณ์— ์ ‘์†ํ•˜๋Š” ์ž๊ฐ€ ์ด๋ฅผ ์—ด์–ด๋ณผ ์ˆ˜ ์žˆ๊ฒŒ ํ•œ๋‹ค๋Š” ์˜๋ฏธ๋กœ์„œ, ํ”ผ๊ณ ์ธ์€ ์œ„์—์„œ ๋ณธ ๊ฒฝ์šฐ์™€ ๋งˆ์ฐฌ๊ฐ€์ง€๋กœ ๋ถํ•œ ๊ณต์ž‘์›๊ณผ ์ด๋ฉ”์ผ์„ ํ†ตํ•˜์—ฌ ํ†ต์‹ ํ•˜๊ณ  ํŽธ์˜๋ฅผ ์ œ๊ณตํ•˜์˜€๋‹ค. โ–ฒโ–ฒโ–ฒโ–ฒ.net ์‚ฌ์ดํŠธ๋Š” ์ค‘๊ตญ์— ์„œ๋ฒ„๋ฅผ ๋‘” ๊ฒƒ์œผ๋กœ ํ”ผ๊ณ ์ธ์ด ์ด์ „์— โ–ฝโ–ฝโ–ฝโ–ฝ.COM์„ ํ†ตํ•˜์—ฌ ๋ถํ•œ ๊ณต์ž‘์›๊ณผ ์—ฐ๋ฝ์„ ํ•œ ๊ฒฝ์šฐ์™€ ๊ทธ ํ˜•ํƒœ๊ฐ€ ์œ ์‚ฌํ•˜๋‹ค. 4) ์œ„ ํŒŒ์ผ์„ ๋ณตํ˜ธํ™”ํ•œ ๋ฌธ์„œ์—๋Š” โ€˜2015๋…„ ์ดํ™”๋ณด๊ณ ๋ฅผ ๋“œ๋ฆฐ๋‹คโ€™๋Š” ์ œ๋ชฉ ํ•˜์— ์กฐ์ง(โ™ฅโ™ฅโ™ฅโ™ฅโ™ฅํšŒ)์˜ ์šด์˜ํ˜„ํ™ฉ๊ณผ ๊ทธ์— ๋Œ€ํ•œ ๋ถ„์„, ์•ž์œผ๋กœ์˜ ๊ณ„ํš ๋“ฑ์ด ๊ฐ ์ง€์—ญ๋ณ„๋กœ ๊ตฌ๋ถ„ํ•˜์—ฌ ์ƒ์„ธํžˆ ๊ธฐ์žฌ๋˜์–ด ์žˆ๊ณ  ์ด๋Š” ๋ถํ•œ์ธก์—์„œ ๊ถ๊ธˆํ•ด ํ•˜๊ฑฐ๋‚˜ ํ•„์š”๋กœ ํ•˜๋Š” ์ •๋ณด์ด๋‹ค. ๋‹ค. ์†Œ๊ฒฐ ํ”ผ๊ณ ์ธ๊ณผ ๋ณ€ํ˜ธ์ธ๋“ค์˜ ์ด ๋ถ€๋ถ„ ์ฃผ์žฅ์€ ๋ฐ›์•„๋“ค์ด์ง€ ์•Š๋Š”๋‹ค. 25. 2013. 11. 4., 2013. 12. 14., 2014. 1. 4., 2014. 2. 3., 2014. 4. 28. ๊ฐ ์ด์ ๋™์กฐ๋กœ ์ธํ•œ ๊ตญ๊ฐ€๋ณด์•ˆ๋ฒ•์œ„๋ฐ˜(์ฐฌ์–‘ยท๊ณ ๋ฌด๋“ฑ)์˜ ์ ์— ๊ด€ํ•˜์—ฌ ๊ฐ€. ์ฃผ์žฅ ํ”ผ๊ณ ์ธ์€ ๊ทธ์™€ ๊ฐ™์€ ํ–‰์œ„๋ฅผ ํ•œ ์ ์ด ์—†๋‹ค. ๋‚˜. ๊ด€๋ จ๋ฒ•๋ฆฌ ๊ตญ๊ฐ€๋ณด์•ˆ๋ฒ• ์ œ7์กฐ ์ œ1ํ•ญ์˜ ๋ฐ˜๊ตญ๊ฐ€๋‹จ์ฒด ๋“ฑ ํ™œ๋™ ์„ ์ „ยท๋™์กฐ์ฃ„ ๊ตฌ์„ฑ์š”๊ฑด์œผ๋กœ์„œ โ€˜์„ ์ „โ€™์€ ๋ถˆํŠน์ • ๋˜๋Š” ๋‹ค์ˆ˜์ธ์—๊ฒŒ ๋ฐ˜๊ตญ๊ฐ€๋‹จ์ฒด ๋“ฑ์˜ ํ™œ๋™ ๋‚ด์šฉ์ด๋‚˜ ์ทจ์ง€๋ฅผ ์ฃผ์ง€์‹œ์ผœ ์ดํ•ด ๋˜๋Š” ๊ณต๊ฐ์„ ๊ตฌํ•˜๋Š” ๊ฒƒ์„, โ€˜๋™์กฐโ€™๋Š” ๋ฐ˜๊ตญ๊ฐ€๋‹จ์ฒด ๋“ฑ์˜ ์„ ์ „ยท์„ ๋™ ๋ฐ ํ™œ๋™๊ณผ ๋™์ผํ•œ ๋‚ด์šฉ์˜ ์ฃผ์žฅ์„ ํ•˜๊ฑฐ๋‚˜ ์ด์— ํ•ฉ์น˜๋˜๋Š” ํ–‰์œ„๋ฅผ ํ•˜์—ฌ ๋ฐ˜๊ตญ๊ฐ€๋‹จ์ฒด ๋“ฑ์˜ ํ™œ๋™์— ํ˜ธ์‘ยท๊ฐ€์„ธํ•˜๋Š” ๊ฒƒ์„ ์˜๋ฏธํ•˜๋ฉฐ, ์ด๋•Œ โ€˜์„ ์ „โ€™ ๋˜๋Š” โ€˜๋™์กฐโ€™ ํ–‰์œ„๋Š” ๊ตญ๊ฐ€์˜ ์กด๋ฆฝยท์•ˆ์ „์ด๋‚˜ ์ž์œ ๋ฏผ์ฃผ์  ๊ธฐ๋ณธ์งˆ์„œ์— ์‹ค์งˆ์  ํ•ด์•…์„ ๋ผ์น  ๋ช…๋ฐฑํ•œ ์œ„ํ—˜์„ฑ์ด ์žˆ๋Š” ์ •๋„์— ์ด๋ฅด๋Ÿฌ์•ผ ํ•œ๋‹ค(๋Œ€๋ฒ•์› 2013. 2. 15. ์„ ๊ณ  2010๋„3504 ํŒ๊ฒฐ ๋“ฑ ์ฐธ์กฐ). ๋‹ค. ํŒ๋‹จ ์ด ๋ฒ•์›์ด ์ฑ„ํƒํ•˜์—ฌ ์กฐ์‚ฌํ•œ ์ฆ๊ฑฐ๋“ค์— ์˜ํ•˜์—ฌ ์•Œ ์ˆ˜ ์žˆ๋Š” ๋‹ค์Œ๊ณผ ๊ฐ™์€ ์‚ฌ์ •๋“ค์„ ์ข…ํ•ฉํ•˜๋ฉด, ํ”ผ๊ณ ์ธ์ด ๊ตญ๊ฐ€์˜ ์กด๋ฆฝยท์•ˆ์ „์ด๋‚˜ ์ž์œ ๋ฏผ์ฃผ์  ๊ธฐ๋ณธ์งˆ์„œ๋ฅผ ์œ„ํƒœ๋กญ๊ฒŒ ํ•œ๋‹ค๋Š” ์ •์„ ์•Œ๋ฉด์„œ ๋Œ€๋‚จํ˜๋ช…๋ก  ๋“ฑ ๋ถํ•œ์˜ ์„ ์ „ยทํ™œ๋™์— ํ˜ธ์‘ยท๊ฐ€์„ธํ•˜๋Š” ํ–‰์œ„๋ฅผ ํ•จ์œผ๋กœ์จ ๋ฐ˜๊ตญ๊ฐ€๋‹จ์ฒด์ธ ๋ถํ•œ์˜ ํ™œ๋™์— ๋™์กฐํ•˜์˜€์Œ์ด ๋ถ„๋ช…ํ•˜๋‹ค. ๋˜ํ•œ ์ด์™€ ๊ฐ™์€ ํ”ผ๊ณ ์ธ์˜ ํ–‰์œ„๋Š” ๊ตญ๊ฐ€์˜ ์กด๋ฆฝยท์•ˆ์ „์ด๋‚˜ ์ž์œ ๋ฏผ์ฃผ์  ๊ธฐ๋ณธ์งˆ์„œ์— ์‹ค์งˆ์  ํ•ด์•…์„ ๋ผ์น  ๋ช…๋ฐฑํ•œ ์œ„ํ—˜์„ฑ์ด ์žˆ๋‹ค. 1) ํ”ผ๊ณ ์ธ์˜ ๋Œ€ํ™”๋ฅผ ๋…น์Œํ•œ ๋…น์ทจํŒŒ์ผ๊ณผ ๋…น์ทจ๋ก์˜ ์ฆ๊ฑฐ๋Šฅ๋ ฅ์ด ์ธ์ •๋จ์€ ์•ž์„œ ๋ณธ ๋ฐ”์™€ ๊ฐ™๊ณ , ์ด ๋ฒ•์›์˜ ๊ฒ€์ฆ์— ์˜ํ•˜๋ฉด ํ”ผ๊ณ ์ธ์ด ํŒ์‹œ ๊ฐ ๋ฒ”์ฃ„์‚ฌ์‹ค๊ณผ ๊ฐ™์€ ์ผ์‹œยท์žฅ์†Œ์—์„œ ํŒ์‹œ์™€ ๊ฐ™์€ ๋ฐœ์–ธ์„ ํ•œ ์‚ฌ์‹ค์ด ์ธ์ •๋œ๋‹ค. 2) ํ”ผ๊ณ ์ธ์ด ๊ณต์†Œ์™ธ 4, ๊ณต์†Œ์™ธ 5๋ฅผ ์ƒ๋Œ€๋กœ ๋‚˜๋ˆˆ ๋Œ€ํ™”์˜ ๋‚ด์šฉ์€ ๋Œ€ํ•œ๋ฏผ๊ตญ์—์„œ์˜ ๋ณ€ํ˜์šด๋™๊ณผ ํ†ต์ผ์šด๋™์„ ํ•  ๊ฒƒ์„ ๋…ผ์˜ํ•˜๊ณ , ๋ถํ•œ์˜ ์ฃผ์ฒด์‚ฌ์ƒ, ์„ ๊ตฐ์‚ฌ์ƒ, ๊น€์ •์€ ์ •๊ถŒ, ์œ ์ผ์˜๋„์ฒด๊ณ„, ๋Œ€๋‚จํ˜๋ช…๋ก  ๋“ฑ์„ ์˜นํ˜ธยท์„ ์ „ํ•˜๋ฉฐ, ๋ถํ•œ์˜ ์ฒด์ œ์ •๋‹น์„ฑ์„ ๊ฐ•์กฐํ•˜๊ณ , ๋ถํ•œ์˜ ์‹ ๋…„์‚ฌ์˜ ๋‚ด์šฉ์— ๊ณต๊ฐยทํ˜ธ์‘ํ•˜๊ณ  ์ด๋ฅผ ์ „ํŒŒํ•˜์ž๋Š” ๊ฒƒ ๋“ฑ์ด๋‹ค. 3) ํ”ผ๊ณ ์ธ์€ ๊ทธ ๋™์•ˆ ๋ถํ•œ ๊ณต์ž‘์›๊ณผ ํ•ด์™ธ์—์„œ ํšŒํ•ฉ์„ ํ•˜๊ณ  ํ†ต์‹ ์—ฐ๋ฝ์„ ํ•˜์˜€์œผ๋ฉฐ, ๋ถํ•œ์„ ์ถ”์ข…ํ•˜๋Š” ์กฐ์ง์˜ ํ™œ๋™ํ˜„ํ™ฉ๊ณผ ์•ž์œผ๋กœ์˜ ๊ณ„ํš ๋“ฑ์„ ๋ถํ•œ ๊ณต์ž‘์›์—๊ฒŒ ๋ณด๊ณ ํ•˜๋Š” ๋“ฑ ๊นŠ์ˆ™์ด ๊ด€์—ฌํ•˜์˜€๋‹ค. ๋ผ. ์†Œ๊ฒฐ ํ”ผ๊ณ ์ธ๊ณผ ๋ณ€ํ˜ธ์ธ๋“ค์˜ ์ฃผ์žฅ์€ ์ด์œ  ์—†๋‹ค. ใ€๋ฒ•๋ น์˜ ์ ์šฉใ€‘ 1. ๋ฒ”์ฃ„์‚ฌ์‹ค์— ๋Œ€ํ•œ ํ•ด๋‹น๋ฒ•์กฐ ๊ฐ ๊ตญ๊ฐ€๋ณด์•ˆ๋ฒ• ์ œ7์กฐ ์ œ5ํ•ญ, ์ œ1ํ•ญ(์ด์ ํ‘œํ˜„๋ฌผ ์†Œ์ง€์˜ ์ฃผ38) ์ ), ๊ฐ ๊ตญ๊ฐ€๋ณด์•ˆ๋ฒ• ์ œ8์กฐ ์ œ1ํ•ญ(2015. 4. 5.๊ฒฝ ํšŒํ•ฉ์˜ ์ ์„ ์ œ์™ธํ•œ ๋‚˜๋จธ์ง€ ๊ฐ ํšŒํ•ฉ ๋ฐ ํ†ต์‹ ์˜ ์ ), ๊ตญ๊ฐ€๋ณด์•ˆ๋ฒ• ์ œ8์กฐ ์ œ1ํ•ญ, ํ˜•๋ฒ• ์ œ30์กฐ(2015. 4. 5.๊ฒฝ ํšŒํ•ฉ์˜ ์ ), ๊ฐ ๊ตญ๊ฐ€๋ณด์•ˆ๋ฒ• ์ œ9์กฐ ์ œ2ํ•ญ ๋ณธ๋ฌธ, ์ œ3์กฐ ์ œ1ํ•ญ(ํŽธ์˜์ œ๊ณต์˜ ์ ), ๊ตญ๊ฐ€๋ณด์•ˆ๋ฒ• ์ œ5์กฐ ์ œ2ํ•ญ, ํ˜•๋ฒ• ์ œ30์กฐ(๊ธˆํ’ˆ์ˆ˜์ˆ˜์˜ ์ ), ๊ฐ ๊ตญ๊ฐ€๋ณด์•ˆ๋ฒ• ์ œ7์กฐ ์ œ1ํ•ญ, ํ˜•๋ฒ• ์ œ30์กฐ(์ด์ ๋™์กฐ์˜ ์ ) 1. ๊ฒฝํ•ฉ๋ฒ”๊ฐ€์ค‘ ํ˜•๋ฒ• ์ œ37์กฐ ์ „๋‹จ, ์ œ38์กฐ ์ œ1ํ•ญ ์ œ2ํ˜ธ, ์ œ50์กฐ[ํ˜•, ์ฃ„์งˆ ๋ฐ ๋ฒ”์ •์ด ๊ฐ€์žฅ ๋ฌด๊ฑฐ์šด 2015. 4. 5.๊ฒฝ ๊ตญ๊ฐ€๋ณด์•ˆ๋ฒ•์œ„๋ฐ˜(ํšŒํ•ฉยทํ†ต์‹ ๋“ฑ)์ฃ„์— ์ •ํ•œ ํ˜•์— ๊ฒฝํ•ฉ๋ฒ” ๊ฐ€์ค‘] 1. ์ž๊ฒฉ์ •์ง€ํ˜•์˜ ๋ณ‘๊ณผ ๊ตญ๊ฐ€๋ณด์•ˆ๋ฒ• ์ œ14์กฐ 1. ์ฃผ39) ๋ชฐ์ˆ˜ ํ˜•๋ฒ• ์ œ48์กฐ ์ œ1ํ•ญ ์ œ1ํ˜ธ 1. ์ถ”์ง•์— ๋Œ€ํ•œ ํŒ๋‹จ ์ด ๋ฒ•์˜ ์ฃ„๋ฅผ ๋ฒ”ํ•˜๊ณ  ๋ฐ›์€ ๋ณด์ˆ˜๋ฅผ ๋ชฐ์ˆ˜ํ•  ์ˆ˜ ์—†์„ ๋•Œ์—๋Š” ์ถ”์ง•์„ ํ•ด์•ผ ํ•˜๋‚˜(๊ตญ๊ฐ€๋ณด์•ˆ๋ฒ• ์ œ15์กฐ ์ œ1ํ•ญ), ์ด ์‚ฌ๊ฑด์—์„œ ๊ธˆํ’ˆ์ˆ˜์ˆ˜๋Š” ํ”ผ๊ณ ์ธ์ด ์ง์ ‘ ๋ฐ›์€ ๊ฒƒ์€ ์•„๋‹ˆ๊ณ , ์‹ค์ œ ํ”ผ๊ณ ์ธ์—๊ฒŒ โ€˜๋ณด์ˆ˜โ€™๋กœ์„œ ๊ท€์†๋œ ์ด์ต์ด ์–ผ๋งˆ์ธ์ง€ ํŠน์ •ํ•  ์ˆ˜ ์—†์œผ๋ฏ€๋กœ ์ถ”์ง•์„ ์„ ๊ณ ํ•˜์ง€๋Š” ์•Š๋Š”๋‹ค. ใ€์–‘ํ˜•์˜ ์ด์œ ใ€‘ 1. ๋ฒ•๋ฅ ์ƒ ์ฒ˜๋‹จํ˜•์˜ ๋ฒ”์œ„: 15๋…„ ์ดํ•˜์˜ ์ง•์—ญ ๋ฐ ์ž๊ฒฉ์ •์ง€ 2. ์–‘ํ˜•๊ธฐ์ค€์ƒ ๊ถŒ๊ณ ํ˜•์˜ ๋ฒ”์œ„ ๋Œ€๋ฒ•์› ์–‘ํ˜•๊ธฐ์ค€์ด ์„ค์ •๋˜์–ด ์žˆ์ง€ ์•Š์Œ 3. ์„ ๊ณ ํ˜•์˜ ๊ฒฐ์ • [๋ถˆ๋ฆฌํ•œ ์ •์ƒ] ๋ถํ•œ์€ ์กฐ๊ตญ์˜ ํ‰ํ™”์  ํ†ต์ผ์„ ์œ„ํ•˜์—ฌ ๋Š์ž„์—†์ด ๊ต๋ฅ˜์™€ ํ˜‘๋ ฅ์„ ๋ชจ์ƒ‰ํ•˜์—ฌ์•ผ ํ•  ๋Œ€ํ™”์™€ ํ˜‘๋ ฅ์˜ ๋™๋ฐ˜์ž์ž„์— ํ‹€๋ฆผ์—†๋‹ค. ๊ทธ๋Ÿฌ๋‚˜ ํ•œํŽธ์œผ๋กœ ์šฐ๋ฆฌ์˜ ์ž์œ ๋ฏผ์ฃผ์  ํ—Œ๋ฒ•์งˆ์„œ์™€ ์–‘๋ฆฝํ•  ์ˆ˜ ์—†๋Š” ์ฃผ์ฒด์‚ฌ์ƒ, ์„ ๊ตฐ์‚ฌ์ƒ ๋“ฑ ์œ ์ผ์˜๋„์ฒด๊ณ„๋ฅผ ๋‚ด์„ธ์šฐ๋ฉด์„œ, ๊น€์ผ์„ฑยท๊น€์ •์ผ์— ์ด์–ด ๊น€์ •์€์— ์ด๋ฅด๊ธฐ๊นŒ์ง€ ๊ถŒ๋ ฅ์„ ์„ธ์Šตํ•˜๊ณ  ๋…์žฌ ์ •๊ถŒ์„ ์œ ์ง€ํ•˜๋ฉฐ ๋ถํ•œ ์ฃผ๋ฏผ๋“ค์˜ ์ธ๊ถŒ์„ ์œ ๋ฆฐํ•˜๊ณ  ์žˆ๊ณ , ๋˜ํ•œ ์ž‡๋”ฐ๋ฅธ ๋ฌด๋ ฅ ๋„๋ฐœ๊ณผ ์„ ์ „ยท์„ ๋™, ๊ณ ๋„์˜ ๋Œ€๋‚จ๊ณต์ž‘์„ ํ†ตํ•˜์—ฌ ๋Œ€ํ•œ๋ฏผ๊ตญ์˜ ์กด๋ฆฝ๊ณผ ์šฐ๋ฆฌ์˜ ์ž์œ ๋ฏผ์ฃผ์ฃผ์˜ ์ฒด์ œ๋ฅผ ์œ„ํ˜‘ํ•˜๋ฉด์„œ ์ ํ™”ํ†ต์ผ์˜ ๋…ธ์„ ์„ ํฌ๊ธฐํ•˜์ง€ ์•Š์•˜์Œ์„ ๊ทธ ํ–‰๋™์œผ๋กœ ๋ณด์—ฌ์ฃผ๊ณ  ์žˆ๋‹ค. ์ž์œ ๋ฏผ์ฃผ์ฃผ์˜ ์ฒด์ œ ํ•˜์—์„œ ์‚ฌ์ƒ์˜ ์ž์œ ์™€ ํ‘œํ˜„์˜ ์ž์œ , ํ†ต์‹ ์˜ ์ž์œ  ๋“ฑ ๊ธฐ๋ณธ์  ๊ถŒ๋ฆฌ๊ฐ€ ๋„๋ฆฌ ๋ณด์žฅ๋˜๊ณ  ์žˆ์œผ๋‚˜, ์ด์™€ ๊ฐ™์€ ๊ถŒ๋ฆฌ๊ฐ€ ๋ฌด์ œํ•œ์ ์œผ๋กœ ํ—ˆ์šฉ๋  ์ˆ˜ ์žˆ๋Š” ๊ฒƒ์€ ์•„๋‹ˆ๋‹ค. ๋˜ํ•œ ๋ฐ˜๊ตญ๊ฐ€๋‹จ์ฒด์ธ ๋ถํ•œ์˜ ์œ„ํ˜‘์ด ์œ„์™€ ๊ฐ™์ด ์—„์—ฐํžˆ ์ƒ์กดํ•˜๋Š” ์ด์ƒ, ๊ตญ๊ฐ€์˜ ์•ˆ์ „์„ ์œ„ํƒœ๋กญ๊ฒŒ ํ•˜๋Š” ๋ฐ˜๊ตญ๊ฐ€ํ™œ๋™์„ ๊ทœ์ œํ•จ์œผ๋กœ์จ ๊ตญ๊ฐ€์˜ ์•ˆ์ „๊ณผ ๊ตญ๋ฏผ์˜ ์ƒ์กด ๋ฐ ์ž์œ ๋ฅผ ํ™•๋ณดํ•ด์•ผ ํ•œ๋‹ค๋Š” ๊ฒƒ์€ ๋‹น์—ฐํ•˜๊ณ  ๋˜ ์šฐ๋ฆฌ์—๊ฒŒ ์ฃผ์–ด์ง„ ์ ˆ์‹คํ•œ ๊ณผ์ œ์ด๋‹ค. ๊ทธ๋Ÿผ์—๋„ ๋ถˆ๊ตฌํ•˜๊ณ  ํ”ผ๊ณ ์ธ์€ ์ด๋ฅธ๋ฐ” โ€˜์กฐ์งโ€™ ๋‚ด์ง€ โ€˜์ง€์‚ฌโ€™์˜ โ€˜์ด์ฑ…โ€™์œผ๋กœ์„œ โ€˜์›์ˆ˜๋‹˜์˜ ์œ ์ผ์˜๋„์ฒด๊ณ„โ€™, โ€˜์ด๋‚จ์˜ ์‚ฌํšŒ๋ณ€ํ˜์šด๋™โ€™๊ณผ โ€˜์กฐ๊ตญํ†ต์ผ์šด๋™โ€™์„ ์ถ”์ข…ํ•˜๊ณ  'ํ—Œ์‹ ๋ณต๋ฌดโ€˜๋ฅผ ํ•˜๊ฒ ๋‹ค๊ณ  ํ•˜๋ฉด์„œ, ์žฅ๊ธฐ๊ฐ„์— ๊ฑธ์ณ ๋ถํ•œ ๊ณต์ž‘์กฐ์ง ๊ตฌ์„ฑ์›๋“ค๊ณผ ์ˆ˜์ฐจ๋ก€ ํšŒํ•ฉํ•˜๊ณ , ๊ณต์ž‘๊ธˆ ์„ฑ๊ฒฉ์˜ ๊ธˆํ’ˆ์„ ์ˆ˜์ˆ˜ํ•˜๋Š” ํ•œํŽธ, ์ง€์†์ ์œผ๋กœ ๋ถํ•œ ์ƒ๋ถ€์„ ๊ณผ ํ†ต์‹ ยท์—ฐ๋ฝํ•˜์—ฌ ์ง€์‹œ๋ฅผ ๋ฐ›๊ฑฐ๋‚˜ ๋ณด๊ณ ํ•˜๊ณ  ์ •๋ณด๋ฅผ ์ œ๊ณตํ•˜์˜€๋‹ค. ๋˜ํ•œ ํ”ผ๊ณ ์ธ์€ ๋‹ค์ˆ˜์˜ ์ด์ ํ‘œํ˜„๋ฌผ์„ ์†Œ์ง€ํ•˜๊ณ  ์ด๋ฅผ ์ด์šฉํ•˜์—ฌ ์ฃผ๋ณ€์ธ๋“ค๊ณผ ์‚ฌ์ƒํ•™์Šต์„ ํ•˜๋Š” ๋“ฑ ๋ถํ•œ์˜ ์ •์ฑ…๊ณผ ์‚ฌ์ƒ์„ ๋™์กฐยท์„ ์ „ํ•˜์˜€๋‹ค. ์ด๋Ÿฌํ•œ ํ”ผ๊ณ ์ธ์˜ ๋ฒ”ํ–‰์€ ์ƒ๋‹นํ•œ ๊ธฐ๊ฐ„ ๋™์•ˆ ๊ณ„ํš์ , ์กฐ์ง์ , ์ง€๋Šฅ์ ์œผ๋กœ ์ด๋ฃจ์–ด์กŒ๊ณ , ๋ฐ˜๊ตญ๊ฐ€๋‹จ์ฒด์ธ ๋ถํ•œ์„ ์ด๋กญ๊ฒŒ ํ•จ๊ณผ ๋™์‹œ์— ๋Œ€ํ•œ๋ฏผ๊ตญ์˜ ์กด๋ฆฝยท์•ˆ์ „๊ณผ ์ž์œ ๋ฏผ์ฃผ์ฃผ์˜ ์งˆ์„œ์— ์‹ฌ๊ฐํ•œ ์œ„ํ—˜์„ ์ดˆ๋ž˜ํ•  ์ˆ˜ ์žˆ๋‹ค๋Š” ์ ์—์„œ ์ค‘๋Œ€ํ•œ ์‚ฌ์•ˆ์ด๊ณ , ์ฃ„์งˆ์ด ๋งค์šฐ ๋ฌด๊ฒ๋‹ค. ์ด๋Ÿฌํ•œ ํ”ผ๊ณ ์ธ์˜ ๋ฒ”ํ–‰์ด ๊ฐ๊ด€์ ์œผ๋กœ ๋“œ๋Ÿฌ๋‚ฌ์Œ์—๋„ ๋ถˆ๊ตฌํ•˜๊ณ , ํ”ผ๊ณ ์ธ์€ ์ด ๋ฒ•์ •์— ์ด๋ฅด๊ธฐ๊นŒ์ง€ ์ž์‹ ์˜ ๋ฒ”ํ–‰์„ ์กฐ๊ธˆ์ด๋ผ๋„ ์ธ์ •ํ•˜๊ณ  ๋ฐ˜์„ฑํ•˜๋Š” ์ž์„ธ๋ฅผ ๋ณด์ด์ง€ ์•„๋‹ˆํ•˜๊ณ  ์žˆ๋‹ค. ํ”ผ๊ณ ์ธ์€ ์‚ฌ์•ˆ์˜ ์‹ค์ฒด์— ๋Œ€ํ•˜์—ฌ๋Š” ๋‚ฉ๋“ํ•  ๋งŒํ•œ ํ•ด๋ช…์„ ํ•˜์ง€ ์•„๋‹ˆํ•œ ์ฑ„, ํ•œํŽธ์œผ๋กœ๋Š” ์ข…๊ต์™€ ์–‘์‹ฌ, ๋ฏผ์กฑ๊ณผ ํ†ต์ผ ๋“ฑ์„ ๋‚ด์„ธ์šฐ๋ฉด์„œ, ๋‹ค๋ฅธ ํ•œํŽธ์œผ๋กœ๋Š” ๋Œ€ํ•œ๋ฏผ๊ตญ์˜ ์ •์ฒด์„ฑ๊ณผ ๊ตญ๊ฐ€์  ๊ถŒ์œ„, ๊ทธ๋ฆฌ๊ณ  ๊ณต๋™์ฒด ๊ตฌ์„ฑ์›์œผ๋กœ์„œ์˜ ์˜๋ฌด ๋“ฑ์€ ๋ฌด์‹œํ•˜๊ณ  ์ˆ˜์‚ฌ์™€ ์‚ฌ๋ฒ•์ฒด๊ณ„์— ๋Œ€ํ•œ ๋ถˆ์‹ ๊ณผ ๋น„ํŒ๋งŒ์„ ๋“œ๋Ÿฌ๋‚ด๋ฉฐ ๋ฒ”ํ–‰์„ ์ผ์ฒด ๋ถ€์ธํ•˜๋Š” ํƒœ๋„๋กœ๋งŒ ์ผ๊ด€ํ•˜๊ณ  ์žˆ๋‹ค. ๋”ฐ๋ผ์„œ ํ”ผ๊ณ ์ธ์— ๋Œ€ํ•˜์—ฌ๋Š” ๊ทธ ์ฑ…์ž„์— ์ƒ์‘ํ•˜๋Š” ์ฒ˜๋ฒŒ์ด ํ•„์š”ํ•˜๋‹ค. [์œ ๋ฆฌํ•œ ์ •์ƒ] ํ”ผ๊ณ ์ธ์˜ ๋ฒ”ํ–‰์€ ๋น„๋ก ์šฐ๋ฆฌ ์‚ฌํšŒ์™€ ๋ฒ•์ฒด๊ณ„๊ฐ€ ํ—ˆ์šฉํ•  ์ˆ˜ ์—†๋Š” ๊ฒƒ์ด๊ธฐ๋Š” ํ•˜๋‚˜, ์–ด์จŒ๋“  ํ”ผ๊ณ ์ธ ๊ฐœ์ธ์˜ ์‹ ๋…์— ๋”ฐ๋ฅธ ์‚ฌ์ƒ๋ฒ”, ํ™•์‹ ๋ฒ”์˜ ๋ฒ”์ฃผ์— ๋“ค์–ด๊ฐ€๋Š” ๊ฒƒ์œผ๋กœ ๋ณผ ์ˆ˜ ์žˆ๋‹ค. ๊ทธ๋ฆฌ๊ณ  ์šฐ๋ฆฌ ์ž์œ ๋ฏผ์ฃผ์ฃผ์˜ ์‚ฌํšŒ๊ฐ€ ํ•œ์ธต ์„ฑ์ˆ™ยท๋ฐœ์ „ํ•˜๋ฉด์„œ ๊ฑด๊ฐ•ํ•จ์„ ์œ ์ง€ํ•˜๊ณ  ์žˆ๋Š” ํ•œ, ํŒ์‹œ์™€ ๊ฐ™์€ ํ”ผ๊ณ ์ธ์˜ ๋ฒ”ํ–‰์œผ๋กœ ์šฐ๋ฆฌ ์‚ฌํšŒยท์ฒด์ œ์— ๊ฐ€ํ•˜๋Š” ์œ„ํ•ด๋Š” ์–ด๋Š ์ •๋„ ์ž์ •๊ณผ ๊ทน๋ณต์ด ๊ฐ€๋Šฅํ•˜๋‹ค๊ณ  ํ•  ๊ฒƒ์ด๊ณ , ๊ทธ ์œ„ํ—˜์œผ๋กœ ์ธํ•˜์—ฌ ์‹ค์ œ๋กœ ์ดˆ๋ž˜๋œ ๊ฒฐ๊ณผ๊ฐ€ ์•„์ฃผ ์‹ฌ๋Œ€ํ•˜๋‹ค๊ณ ๊นŒ์ง€ ํ•˜๊ธฐ๋Š” ์–ด๋ ต๋‹ค. ํ”ผ๊ณ ์ธ์€ ๋ฒŒ๊ธˆํ˜•์„ 2ํšŒ ์„ ๊ณ ๋ฐ›์€ ์™ธ์—๋Š” ํ˜•์‚ฌ์ฒ˜๋ฒŒ ์ „๊ณผ๊ฐ€ ์—†๋‹ค. ๊ฐ€์กฑ๊ด€๊ณ„๋‚˜ ์‚ฌํšŒ์  ์œ ๋Œ€๊ด€๊ณ„๋„ ๋น„๊ต์  ์–‘ํ˜ธํ•œ ๊ฒƒ์œผ๋กœ ๋ณด์ธ๋‹ค. ๊ทธ ๋ฐ–์— ์ด ์‚ฌ๊ฑด ๋ฒ”ํ–‰์˜ ๋™๊ธฐ, ์ˆ˜๋‹จ๊ณผ ๊ฒฐ๊ณผ, ๋ฒ”ํ–‰ ํ›„์˜ ์ •ํ™ฉ, ํ”ผ๊ณ ์ธ์˜ ๋‚˜์ด, ์ง์—…, ์„ฑํ–‰, ํ™˜๊ฒฝ, ๊ฐ€์กฑ๊ด€๊ณ„ ๋“ฑ ์ด ์‚ฌ๊ฑด ๋ณ€๋ก ์— ๋‚˜ํƒ€๋‚œ ๋ชจ๋“  ์–‘ํ˜•์กฐ๊ฑด๋“ค์„ ์ข…ํ•ฉ์ ์œผ๋กœ ๊ณ ๋ คํ•˜์—ฌ ์ฃผ๋ฌธ๊ณผ ๊ฐ™์ด ํ˜•์„ ์ •ํ•œ๋‹ค. ใ€๋ฌด์ฃ„๋ถ€๋ถ„ : 2013. 11. 2. ๊ณต์†Œ์™ธ 5์™€ ์‚ฌ์ƒํ•™์Šต(์ด์ ํ‘œํ˜„๋ฌผ ์†Œ์ง€)ใ€‘ 1. ๊ณต์†Œ์‚ฌ์‹ค์˜ ์š”์ง€ ํ”ผ๊ณ ์ธ์€ 2013. 11. 2. 16:20๊ฒฝ๋ถ€ํ„ฐ ๊ฐ™์€ ๋‚  17:50๊ฒฝ๊นŒ์ง€ ์ˆ˜์›์‹œ์— ์žˆ๋Š” ์„ธ๋ฅ˜์—ญ ์ธ๊ทผ (๋ช…์นญ 3 ์ƒ๋žต) ์ปคํ”ผ์ˆ์—์„œ, ๊ณต์†Œ์™ธ 5๋ฅผ ๋งŒ๋‚˜ ํ”ผ๊ณ ์ธ์˜ ํƒœ๋ธ”๋ฆฟ PC์— ์ €์žฅ๋˜์–ด ์žˆ๋˜ โ€˜๋…ธ์ž‘-์„ ๊ตฐ์‚ฌ์ƒโ€™์ด๋ผ๋Š” ์ œ๋ชฉ์˜ 2013. 8. 25.์ž ์กฐ์„ ์ค‘์•™ํ†ต์‹  ๊ธฐ์‚ฌ์™€ โ€˜์ „๋‹น๊ณผ ์˜จ ์‚ฌํšŒ๋ฅผ ๊น€์ผ์„ฑ-๊น€์ •์ผ์ฃผ์˜ํ™” ํ•˜์žโ€™๋ผ๋Š” ์ œ๋ชฉ์˜ 2013. 9. 12.์ž ๋…ธ๋™์‹ ๋ฌธ ๊ธฐ์‚ฌ๋ฅผ ๊ณต์†Œ์™ธ 5์—๊ฒŒ ์—ด๋žŒ์‹œ์ผฐ๋Š”๋ฐ, ๊ทธ ์ฃผ์š”๋‚ด์šฉ์€ ๋‹ค์Œ๊ณผ ๊ฐ™๋‹ค. ใ€Žใ€Šโ€˜๋…ธ์ž‘-์„ ๊ตฐ์‚ฌ์ƒโ€™ ์ œํ•˜์˜ ๊ธฐ์‚ฌ ์ฃผ์š”๋‚ด์šฉใ€‹ - ์žฅ๊ตฐ๋‹˜๊ป˜์„œ ์„ ๊ตฐํ˜๋ช…๋ น๋„๋ฅผ ์‹œ์ž‘ํ•˜์‹  ์„ ๊ตฐ์ ˆ์€ ๊ฑด๊ตฐ์ ˆ๊ณผ ํ•จ๊ป˜ ์šฐ๋ฆฌ ํ˜๋ช… ๋ฌด๋ ฅ๊ฑด์„ค์‚ฌ์™€ ์กฐ๊ตญ์ฒญ์‚ฌ์— ์ค‘์š”ํ•œ ๋ฆฌ์ •ํ‘œ๋ฅผ ์ƒˆ๊ธด ๋ ฅ์‚ฌ์˜ ๋‚ ์ž…๋‹ˆ๋‹ค. - ์„ ๊ตฐ์€ ์œ„๋Œ€ํ•œ ๊น€์ผ์„ฑ๋™์ง€๊ป˜์„œ ๊ฐœ์ฒ™ํ•˜์‹œ๊ณ  ๊น€์ผ์„ฑ๋™์ง€์™€ ๊น€์ •์ผ๋™์ง€๊ป˜์„œ ๋ น๋„ํ•˜์—ฌ ์˜ค์‹  ์กฐ์„ ํ˜๋ช…์˜ ์ž๋ž‘์Šค๋Ÿฌ์šด ์ „ํ†ต์ด๋ฉฐ ์Šน๋ฆฌ์™€ ์˜๊ด‘์˜ ๊ธฐ์น˜์ž…๋‹ˆ๋‹ค. - ๊น€์ •์ผ๋™์ง€๊ป˜์„œ 1960๋…„ 8์›” 25์ผ ์กฐ์„ ์ธ๋ฏผ๊ตฐ ๊ทผ์œ„ ์„œ์šธ ๊ณต์†Œ์™ธ 29 ์ œ105๋•…ํฌ ์‚ฌ๋‹จ์— ๋ น๋„์˜ ์ž์šฑ์„ ์ƒˆ๊ธฐ์‹  ๊ฒƒ์€ ์„ ๊ตฐํ˜๋ช…์œ„์—…์˜ ๊ณ„์Šน๊ณผ ์„ ๊ตฐ๋ น๋„์˜ ์ƒˆ์ถœ๋ฐœ์„ ์„ ์–ธํ•œ ๋ ฅ์‚ฌ์  ์‚ฌ๋ณ€์ด์˜€์Šต๋‹ˆ๋‹ค. ใ€Šโ€˜์ „๋‹น๊ณผ ์˜จ ์‚ฌํšŒ๋ฅผ ๊น€์ผ์„ฑ-๊น€์ •์ผ์ฃผ์˜ํ™”ํ•˜์žโ€™ ์ œํ•˜์˜ ๊ธฐ์‚ฌ ์ฃผ์š”๋‚ด์šฉใ€‹ - ์˜จ์‚ฌํšŒ์˜ ๊น€์ผ์„ฑ-๊น€์ •์ผ์ฃผ์˜ํ™”๋Š” ์œ„๋Œ€ํ•œ ์ˆ˜๋ น๋‹˜๊ณผ ์žฅ๊ตฐ๋‹˜์˜ ํ˜๋ช…์‚ฌ์ƒ, ๊น€์ผ์„ฑ-๊น€์ •์ผ์ฃผ์˜๋ฅผ ์œ ์ผํ•œ ์ง€๋„์  ์ง€์นจ์œผ๋กœ ํ•˜์—ฌ ์šฐ๋ฆฌ ํ˜๋ช…์„ ์ „์ง„์‹œํ‚ค๋ฉฐ ๊น€์ผ์„ฑ-๊น€์ •์ผ์ฃผ์˜์— ๊ธฐ์ดˆํ•˜์—ฌ ์ฃผ์ฒด์œ„์—…์„ ์™„์„ฑํ•ด ๋‚˜๊ฐ€๋Š” ์„ฑ์Šค๋Ÿฌ์šด ์œ„์—…์ด๋‹ค. - ์œ„๋Œ€ํ•œ ์ฃผ์ฒด์‚ฌ์ƒ, ์„ ๊ตฐ์‚ฌ์ƒ์œผ๋กœ ๋ฌด์žฅํ•˜๊ธฐ ์œ„ํ•œ ์‚ฌ์—…์„ ๊ฒฐ์ •์ ์œผ๋กœ ๊ฐ•ํ™”ํ•จ์œผ๋กœ์จ ์ž์‹ ์„ ์šฐ๋ฆฌ ๋‹น์˜ ํ˜๋ช…์‚ฌ์ƒ๋ฐ–์— ๋ชจ๋ฅด๋Š” ์‚ฌ์ƒ๊ณผ ์‹ ๋…์˜ ๊ฐ•์ž๋กœ ๊ทธ ์–ด๋–ค ๊ฒฝ์šฐ์—๋„ ์˜ค์ง ๋‹น๊ณผ ์ˆ˜๋ น๋งŒ์„ ๋”ฐ๋ฅด๋Š” ์‡ ์†Œ๋ฆฌ๋‚˜๋Š” ํ˜๋ช…๊ฐ€๋กœ ํŠผํŠผํžˆ ์ค€๋น„ํ•ด ๋‚˜๊ฐ€์•ผ ํ•œ๋‹ค.ใ€ ์ด์™€ ๊ฐ™์ด ์œ„ ๊ธฐ์‚ฌ๋“ค์€ ๊น€์ผ์„ฑ์˜ ์ฃผ์ฒด์‚ฌ์ƒ๊ณผ ๊น€์ •์ผ์˜ ์„ ๊ตฐ์‚ฌ์ƒ์„ ์ฐฌ์–‘ํ•˜๋ฉด์„œ ๊น€์ผ์„ฑ๊ณผ ๊น€์ •์ผ์˜ ์—…์ ์„ ๋ฏธํ™”ํ•˜์—ฌ ์ฐฌ์–‘ํ•˜๊ณ  ์„ ์ „ํ•˜๋Š” ๋‚ด์šฉ์ด๋‹ค. ์ด๋กœ์จ ํ”ผ๊ณ ์ธ์€ ๊ตญ๊ฐ€์˜ ์กด๋ฆฝยท์•ˆ์ „์ด๋‚˜ ์ž์œ ๋ฏผ์ฃผ์  ๊ธฐ๋ณธ์งˆ์„œ๋ฅผ ์œ„ํƒœ๋กญ๊ฒŒ ํ•œ๋‹ค๋Š” ์ •์„ ์•Œ๋ฉด์„œ ๋ฐ˜๊ตญ๊ฐ€๋‹จ์ฒด๋‚˜ ๊ทธ ๊ตฌ์„ฑ์› ๋˜๋Š” ๊ทธ ์ง€๋ น์„ ๋ฐ›์€ ์ž์˜ ํ™œ๋™์„ ์ฐฌ์–‘ยท๊ณ ๋ฌดยท์„ ์ „ ๋˜๋Š” ์ด์— ๋™์กฐํ•  ๋ชฉ์ ์œผ๋กœ ์ด์ ํ‘œํ˜„๋ฌผ์„ ์†Œ์ง€ํ•˜์˜€๋‹ค. 2. ํŒ๋‹จ ์œ„ ใ€Œํ”ผ๊ณ ์ธ ๋ฐ ๋ณ€ํ˜ธ์ธ์˜ ์ฃผ์žฅ์— ๊ด€ํ•œ ํŒ๋‹จใ€์ œ14ํ•ญ์—์„œ ์‚ดํŽด๋ณธ ๋ฐ”์™€ ๊ฐ™์ด, 2013. 11. 2. ๊ตญ์ •์› ์ง์›๋“ค์ด (๋ช…์นญ 3 ์ƒ๋žต) ์ปคํ”ผ์ˆ์— ๋ชฐ๋ž˜ ๋„คํŠธ์›Œํฌ ์นด๋ฉ”๋ผ๋ฅผ ์„ค์น˜ํ•˜์—ฌ ํ”ผ๊ณ ์ธ์˜ ํ–‰๋™๊ณผ ํƒœ๋ธ”๋ฆฟ PC์˜ ํ™”๋ฉด์„ ์ดฌ์˜ํ•œ ๊ฒƒ์€ ์ˆ˜์‚ฌ์— ์žˆ์–ด์„œ์˜ ๋น„๋ก€์„ฑยท์ƒ๋‹น์„ฑ ์›์น™๊ณผ ์˜์žฅ์ฃผ์˜ ๋“ฑ์„ ์œ„๋ฐ˜ํ•œ ๊ฒƒ์ด๋ฏ€๋กœ ๊ทธ๋กœ ์ธํ•ด ์ทจ๋“ํ•œ ์˜์ƒ๋ฌผ ๋“ฑ ์ฃผ40) ์ฆ๊ฑฐ๋Š” ๊ทธ ์ฆ๊ฑฐ๋Šฅ๋ ฅ์ด ์—†๋‹ค๊ณ  ํ•  ๊ฒƒ์ด๊ณ , ๊ณต์†Œ์™ธ 18์˜ ์ด ๋ถ€๋ถ„ ์ฆ์–ธ์€ ์œ„ ์˜์ƒ๋ฌผ์— ์ดฌ์˜๋œ ๋‚ด์šฉ์„ ๊ธฐ์ดˆ๋กœ ์ง„์ˆ ํ•˜๋Š” ๊ฒƒ์ด์–ด์„œ ๋งˆ์ฐฌ๊ฐ€์ง€๋กœ ์ฆ๊ฑฐ๋Šฅ๋ ฅ์ด ์—†์œผ๋ฉฐ, ๋‹ฌ๋ฆฌ ํ”ผ๊ณ ์ธ์ด ์œ„ ์ผ์‹œยท์žฅ์†Œ์—์„œ ์œ„ ํ‘œํ˜„๋ฌผ์„ ์†Œ์ง€ํ•˜์˜€๋‹ค๋Š” ์ ์„ ์ธ์ •ํ•  ๋งŒํ•œ ์ฆ๊ฑฐ๊ฐ€ ์—†๋‹ค. ๋”ฐ๋ผ์„œ ์ด ๋ถ€๋ถ„ ๊ณต์†Œ์‚ฌ์‹ค์€ ๋ฒ”์ฃ„์˜ ์ฆ๋ช…์ด ์—†๋Š” ๊ฒฝ์šฐ์— ํ•ด๋‹นํ•˜๋ฏ€๋กœ ํ˜•์‚ฌ์†Œ์†ก๋ฒ• ์ œ325์กฐ ํ›„๋‹จ์— ์˜ํ•˜์—ฌ ๋ฌด์ฃ„๋ฅผ ์„ ๊ณ ํ•œ๋‹ค. ์ด์ƒ์˜ ์ด์œ ๋กœ ์ฃผ๋ฌธ๊ณผ ๊ฐ™์ด ํŒ๊ฒฐํ•œ๋‹ค. [๋ณ„์ง€ ์ƒ๋žต] ํŒ์‚ฌ ๋ฌธ๊ด‘์„ญ(์žฌํŒ์žฅ) ์ตœ๋ฏผ์„ ๊น€์€์†” ์ฃผ1) 1991. 9. 17. ๋Œ€ํ•œ๋ฏผ๊ตญ๊ณผ ๋ถํ•œ์ด ์œ ์—”์— ๋™์‹œ ๊ฐ€์ž…ํ•˜์˜€๊ณ , ๊ฐ™์€ ํ•ด 12. 13. ์ด๋ฅธ๋ฐ” ๋‚จ๋ถ ๊ณ ์œ„๊ธ‰ํšŒ๋‹ด์—์„œ ๋‚จ๋ถ๊ธฐ๋ณธํ•ฉ์˜์„œ๊ฐ€ ์ฑ„ํƒ๋˜์—ˆ์œผ๋ฉฐ, 2000. 6. 15. ๋‚จ๋ถ๊ณต๋™์„ ์–ธ๋ฌธ์ด ๋ฐœํ‘œ๋˜๊ณ  2007. 10. 4. ๋‚จ๋ถ๊ด€๊ณ„๋ฐœ์ „๊ณผ ํ‰ํ™”๋ฒˆ์˜์„ ์œ„ํ•œ ์„ ์–ธ๋ฌธ์ด ๋ฐœํ‘œ๋˜๋Š” ๋“ฑ ๋‚จยท๋ถํ•œ ์‚ฌ์ด์— ์ •์น˜ยท๊ฒฝ์ œยท์‚ฌํšŒยท๋ฌธํ™” ๋“ฑ ์—ฌ๋Ÿฌ ๋ถ„์•ผ์—์„œ ํ™œ๋ฐœํ•œ ๊ต๋ฅ˜์™€ ํ˜‘๋ ฅ์ด ์ด๋ฃจ์–ด์ ธ ์™”๋‹ค. ์ฃผ2) ๊ฒ€์‚ฌ๊ฐ€ ์ œ์ถœํ•œ 2016. 3. 21.์ž ์˜๊ฒฌ์„œ์— ์˜ํ•˜๋ฉด, ๊ฒ€์‚ฌ๋Š” โ€˜์Šคํ…Œ๊ฐ€๋…ธ ๊ทธ๋ผํ”ผ ๋“ฑ์œผ๋กœ ์•”ํ˜ธํ™”๋œ ๋Œ€๋ถ ๋ณด๊ณ ๋ฌธ ํŒŒ์ผ ๋“ฑ์— ๋Œ€ํ•˜์—ฌ ์•”ํ˜ธ๋ฅผ ํ‘ธ๋Š” ๊ตฌ์ฒด์ ์ธ ๊ณผ์ •์„ ์ฆ์–ธ ๋‚ด์šฉ์œผ๋กœ ํ•˜๊ณ  ์žˆ์–ด ์ด์— ํ•„์š”ํ•œ ์„ค๋น„, ๋ณตํ˜ธํ™” ๋ฐฉ๋ฒ• ๋“ฑ์„ ์–ธ๊ธ‰ํ•  ์ˆ˜๋ฐ–์— ์—†๊ณ , ์ด๋Ÿฌํ•œ ์ •๋ณด ๊ทธ ์ž์ฒด๊ฐ€ ๊ตญ๊ฐ€์•ˆ์ „๋ณด์žฅ ๋“ฑ์— ๊ด€๋ จ๋œ ์ธก๋ฉด์ด ์žˆ๋‹ค๊ณ  ํŒ๋‹จ๋œ๋‹คโ€™๋ผ๋Š” ์ทจ์ง€๋กœ ์žฌํŒ์˜ ๋น„๊ณต๊ฐœ๋ฅผ ์š”์ฒญํ•˜์˜€๋‹ค. ์ฃผ3) ๊ตญ๊ฐ€์ •๋ณด์›์ง์›๋ฒ• ์ œ17์กฐ์— ๋”ฐ๋ฅด๋ฉด, ๊ตญ์ •์› ์ง์›์€ ์ง๋ฌด์ƒ ๋น„๋ฐ€์„ ๋ˆ„์„คํ•˜์—ฌ์„œ๋Š” ์•„๋‹ˆ๋˜๊ณ , ๊ทธ ๋น„๋ฐ€์— ๊ด€ํ•œ ์‚ฌํ•ญ์„ ์ฆ์–ธํ•˜๋ ค๋ฉด ๋ฏธ๋ฆฌ ์›์žฅ์˜ ํ—ˆ๊ฐ€๋ฅผ ๋ฐ›์•„์•ผ ํ•˜๋ฉฐ, ๋ฒ•์›์€ ๊ตญ์ •์› ์ง์›์„ ์ฆ์–ธํ•˜๋„๋ก ํ•จ์— ์žˆ์–ด์„œ ๊ณต๋ฌด์ƒ ๋น„๋ฐ€๋ณดํ˜ธ ๋“ฑ์„ ์œ„ํ•˜์—ฌ ๋น„๊ณต๊ฐœ์ฆ์–ธ ๋“ฑ ์ ์ ˆํ•œ ์กฐ์น˜๋ฅผ ์ทจํ•  ์ˆ˜ ์žˆ๋„๋ก ๊ทœ์ •ํ•˜๊ณ  ์žˆ๋‹ค. ์ฃผ4) ์ดํ›„ ๋‹ค๋ฅธ ๊ฒ€์ฆ๊ธฐ์ผ์— ๊ฒ€์ฐฐ ์ˆ˜์‚ฌ๊ด€์ด ๊ฐ™์€ ๋‚ด์šฉ์˜ ์Šคํ…Œ๊ฐ€๋…ธ๊ทธ๋ผํ”ผ ๋ณตํ˜ธํ™”๋ฅผ ๊ณต๊ฐœ๋œ ์žฌํŒ์—์„œ ํ•˜์˜€๋‹ค๊ณ  ํ•ด์„œ ๋‹ฌ๋ฆฌ ๋ณผ ์ˆ˜๋„ ์—†๋‹ค. ์ฃผ5) 2016. 3. 21. ๊ณต์†Œ์™ธ 47, 2016. 4. 14. ๊ณต์†Œ์™ธ 57, 2016. 4. 28. ๊ณต์†Œ์™ธ 58, 2016. 5. 19. ๊ณต์†Œ์™ธ 46, ๊ณต์†Œ์™ธ 49, ๊ณต์†Œ์™ธ 58, 2016. 7. 27. ๊ณต์†Œ์™ธ 23, ๊ณต์†Œ์™ธ 47, ๊ณต์†Œ์™ธ 14(2016. 9. 20. ํฌํ•จ), 2016. 8. 9. ๊ณต์†Œ์™ธ 47, ๊ณต์†Œ์™ธ 48, 2016. 8. 16. ๊ณต์†Œ์™ธ 18, ๊ณต์†Œ์™ธ 49, 2016. 8. 23. ๊ณต์†Œ์™ธ 53, 2016. 8. 29. ๊ณต์†Œ์™ธ 47, ๊ณต์†Œ์™ธ 46์— ๋Œ€ํ•œ ๊ฐ ์ฆ์ธ์‹ ๋ฌธ์„ ๋งํ•œ๋‹ค. ์ฃผ6) ์ด ๊ฒฝ์šฐ ํ”ผ๊ณ ์ธ ์™ธ์— ๋ฐฉ์ฒญ์ธ ๋“ฑ์— ๋Œ€ํ•˜์—ฌ๋„ ์ฐจํ์‹œ์„ค์„ ์„ค์น˜ํ•˜๋Š” ๊ฒƒ์ด ๊ฐ€๋Šฅํ•˜๋‹ค(๋Œ€๋ฒ•์› 2015. 5. 28. ์„ ๊ณ  2014๋„18006 ํŒ๊ฒฐ ์ฐธ์กฐ). ์ฃผ7) (๋ช…์นญ 53 ์ƒ๋žต)ํšŒ๊ด€ (ํ˜ธ์ˆ˜ ์ƒ๋žต)ํ˜ธ (๋ช…์นญ 29 ์ƒ๋žต)์—ฐ๊ตฌ์†Œ ์ฃผ8) ํ˜•์‚ฌ์†Œ์†ก๋ฒ• ์ œ123์กฐ ์ œ2ํ•ญ์€ โ€˜์ œ1ํ•ญ์— ๊ทœ์ •ํ•œ(๊ณต๋ฌด์†Œ ๋“ฑ) ์ด์™ธ์˜ ํƒ€์ธ์˜ ์ฃผ๊ฑฐ, ๊ฐ„์ˆ˜์ž ์žˆ๋Š” ๊ฐ€์˜ฅ, ์„ ์ฐจ ๋“ฑโ€™์˜ ๊ฒฝ์šฐ๋ฅผ ๊ทœ์ •ํ•˜๊ณ  ์žˆ๋‹ค. ์ฃผ9) ์ œ101์กฐ(์ฒดํฌยท๊ตฌ์†์ ๋ถ€์‹ฌ์ฒญ๊ตฌ๊ถŒ์ž์˜ ์ฒดํฌยท๊ตฌ์†์˜์žฅ๋“ฑ๋ณธ ๊ต๋ถ€์ฒญ๊ตฌ๋“ฑ) ๊ตฌ์†์˜์žฅ์ด ์ฒญ๊ตฌ๋˜๊ฑฐ๋‚˜ ์ฒดํฌ ๋˜๋Š” ๊ตฌ์†๋œ ํ”ผ์˜์ž, ๊ทธ ๋ณ€ํ˜ธ์ธ, ๋ฒ•์ •๋Œ€๋ฆฌ์ธ, ๋ฐฐ์šฐ์ž, ์ง๊ณ„์นœ์กฑ, ํ˜•์ œ์ž๋งค๋‚˜ ๋™๊ฑฐ์ธ ๋˜๋Š” ๊ณ ์šฉ์ฃผ๋Š” ๊ธด๊ธ‰์ฒดํฌ์„œ, ํ˜„ํ–‰๋ฒ”์ธ์ฒดํฌ์„œ, ์ฒดํฌ์˜์žฅ, ๊ตฌ์†์˜์žฅ ๋˜๋Š” ๊ทธ ์ฒญ๊ตฌ์„œ๋ฅผ ๋ณด๊ด€ํ•˜๊ณ  ์žˆ๋Š” ๊ฒ€์‚ฌ, ์‚ฌ๋ฒ•๊ฒฝ์ฐฐ๊ด€ ๋˜๋Š” ๋ฒ•์›์‚ฌ๋ฌด๊ด€๋“ฑ์—๊ฒŒ ๊ทธ ๋“ฑ๋ณธ์˜ ๊ต๋ถ€๋ฅผ ์ฒญ๊ตฌํ•  ์ˆ˜ ์žˆ๋‹ค. ์ฃผ10) ๋ฒ•์ •์—์„œ๋Š” ๊ตญ์ •์›์—์„œ ์„ ๋ณ„์••์ˆ˜ํ•œ ๊ฒƒ์ด ์•„๋‹Œ ํ”ผ๊ณ ์ธ์œผ๋กœ๋ถ€ํ„ฐ ์••์ˆ˜ํ•˜๋ฉด์„œ ๋‹น์ผ ๋ด‰์ธํ•œ ์›๋ณธ ๊ทธ ์ž์ฒด์˜ ๋ด‰์ธ์ƒํƒœ๋ฅผ ํ™•์ธํ•˜๊ณ  ํ•ด์ œํ•˜์—ฌ ๊ทธ ์›๋ณธํŒŒ์ผ์˜ ํ•ด์‹œ๊ฐ’๊ณผ ๊ฒ€์‚ฌ๊ฐ€ ์ œ์ถœํ•œ ์••์ˆ˜๋‹น์‹œ ์‚ฐ์ถœํ•ด ๋†“์€ ํ•ด์‹œ๊ฐ’ ๋ฐ ๊ฒ€์ฆ์šฉ ์‚ฌ๋ณธ์˜ ํ•ด์‹œ๊ฐ’์„ ๊ฐ๊ฐ ๋ชจ๋‘ ๋น„๊ตํ•˜์—ฌ ํ™•์ธํ•˜์˜€๋‹ค. ์•„๋ž˜์—์„œ ๊ธฐ์žฌํ•˜์ง€ ์•Š์€ ์ฆ๊ฑฐ๋ฌผ๋“ค์€ ์ฆ๊ฑฐ๋กœ ์ œ์ถœ๋˜์ง€ ์•Š์•˜๋‹ค. ์ฃผ11) ์ฆ์ธ ๊ณต์†Œ์™ธ 15๋Š” โ€œ์ด ํ‘œํ˜„์€ ์ฆ์ธ์ด ๋‹ค์–‘ํ•œ ๋ฐฉ๋ฒ•์œผ๋กœ ์‹คํ—˜ํ–ˆ์„ ๋•Œ (์œ„ยท๋ณ€์กฐ ํ”์ ์„) ๋ฐœ๊ฒฌํ•˜์ง€ ๋ชปํ–ˆ์œผ๋‚˜ ๊ณผํ•™์ ์œผ๋กœ 0%๋Š” ์•„๋‹ˆ๊ธฐ ๋•Œ๋ฌธ์— ๊ทธ๋ ‡๊ฒŒ ๊ธฐ์žฌํ•œ ๊ฒƒ์ด๊ณ , ์ด๋Ÿฌํ•œ ๊ฒฝ์šฐ์— (ํ†ต์ƒ) ์‚ฌ์šฉํ•˜๋Š” ํ‘œํ˜„๋ฐฉ์‹์ด๋‹ค. ์ฆ์ธ์ด ์˜์ƒ๊ฐ์ • ์—…๋ฌด๋ฅผ ๋‹ด๋‹นํ•˜๋ฉด์„œ ๊ฐ์ •๊ฒฐ๊ณผ์— โ€˜์œ„ยท๋ณ€์กฐ๋˜์ง€ ์•Š์•˜๋‹คโ€™ ๋˜๋Š” โ€˜ํŽธ์ง‘๋˜์ง€ ์•Š์•˜๋‹คโ€™๋ผ๊ณ  ๋‹จ์ •์ ์œผ๋กœ ๊ธฐ์žฌํ•œ ์ ์€ ์—†๋‹คโ€๊ณ  ์ฆ์–ธํ•˜์˜€๋‹ค. ์ฃผ12) ์ด ํ‘œํ˜„์— ๋Œ€ํ•œ ์„ค๋ช…์€ ์œ„ ๊ฐ์ฃผ์—์„œ ๋ณธ ๋ฐ”์™€ ๊ฐ™๋‹ค. ์ฃผ13) ์ฆ์ธ ๊ณต์†Œ์™ธ 55๋Š” โ€œ์ด ํ‘œํ˜„์€ ์ฆ์ธ์ด ์œ„ยท๋ณ€์กฐ ๊ฐ์ •์„ ํ•  ๋•Œ ํ†ต์ƒ์ ์œผ๋กœ ๊ธฐ์žฌํ•˜๋Š” ๋ถ€๋ถ„์ด๊ณ , ๊ณผํ•™์ ์œผ๋กœ ๋ณผ ๋•Œ๋Š” ๋””์ง€ํ„ธ ๋ฐ์ดํ„ฐ์ด๋ฏ€๋กœ ๊ทธ ๊ฐ€๋Šฅ์„ฑ์„ ์™„์ „ํžˆ ๋ฐฐ์ œํ•  ์ˆ˜ ์—†๊ธฐ ๋•Œ๋ฌธ์— ํ†ต์ƒ ์ด๋Ÿฐ ํ‘œํ˜„์„ ์“ฐ๊ณ  ์žˆ๋‹ค. ์ฆ์ธ์ด ์˜์ƒ๊ฐ์ • ์—…๋ฌด๋ฅผ ๋‹ด๋‹นํ•˜๋ฉด์„œ ๊ฐ์ •๊ฒฐ๊ณผ์— โ€˜์œ„ยท๋ณ€์กฐ๋˜์ง€ ์•Š์•˜๋‹คโ€™ ๋˜๋Š” โ€˜ํŽธ์ง‘๋˜์ง€ ์•Š์•˜๋‹คโ€™๋ผ๊ณ  ๋‹จ์ •์ ์œผ๋กœ ๊ธฐ์žฌํ•œ ์ ์€ ์—†๋‹คโ€๊ณ  ์ฆ์–ธํ•˜์˜€๋‹ค. ์ฃผ14) ๊ตญ๋ฆฝ๊ณผํ•™์ˆ˜์‚ฌ์—ฐ๊ตฌ์›์˜ ๊ฐ์ •์„œ์— ์˜ํ•˜๋ฉด, ์ผ๋ฐ˜์ ์œผ๋กœ ๋™์˜์ƒ์„ ํŽธ์ง‘ํ•œ ํ›„, ๋‹ค์‹œ ๋™์˜์ƒ์œผ๋กœ ๋งŒ๋“ค๊ธฐ ์œ„ํ•ด์„œ๋Š” ์žฌ์ธ์ฝ”๋”ฉ ๊ณผ์ •์ด ํ•„์š”ํ•˜๋‹ค. ์žฌ์ธ์ฝ”๋”ฉ ๊ณผ์ •์—์„œ๋Š” ์ธ์ฝ”๋”์— ๋”ฐ๋ผ ์ฝ”๋ฑ์ด ๋ณ€ํ™”๋˜๋ฉฐ, ๋™์ผ ์ฝ”๋ฑ์„ ์‚ฌ์šฉํ•˜๋”๋ผ๋„ ๋…นํ™” ๋ฐ ์••์ถ• ์˜ต์…˜ ๋“ฑ ์ฝ”๋ฑ ์ •๋ณด๊ฐ€ ๋ณ€๊ฒฝ๋˜๊ณ , ํŒŒ์ผ ๊ตฌ์กฐ์—๋„ ๋ณ€ํ™”๊ฐ€ ๋ฐœ์ƒํ•  ์ˆ˜ ์žˆ๋‹ค. ๋˜ํ•œ ํŠน์ • ํŒŒ์ผ ๊ตฌ์กฐ์—์„œ ์€๋‹‰๋˜์–ด ์žˆ๋Š” ๋ฉ”ํƒ€๋ฐ์ดํ„ฐ๋Š” ์‚ฌ๋ผ์ง€๊ฑฐ๋‚˜, ์žฌ์ธ์ฝ”๋”ฉ์‹œ์— ์ธ์ฝ”๋”์— ์˜ํ•ด ์ƒˆ๋กœ์šด ๋ฉ”ํƒ€๋ฐ์ดํ„ฐ๊ฐ€ ๊ธฐ๋ก๋œ๋‹ค. ์ฃผ15) ๋Œ€ํ•œ๋ฏผ๊ตญ๊ณผ ์ค‘๊ตญ ๋Œ€๋ จ์˜ ์‹œ์ฐจ๋ฅผ ๊ณ ๋ คํ•˜๋ฉด ํ”ผ๊ณ ์ธ์ด ์ดฌ์˜๋œ ์‹œ๊ฐ„๊ณผ ์ผ์น˜ํ•œ๋‹ค. ์ฃผ16) ๋Œ€ํ•œ๋ฏผ๊ตญ๊ณผ ๋ฒ ํŠธ๋‚จ ํ˜ธ์น˜๋ฏผ์˜ ์‹œ์ฐจ๋ฅผ ๊ณ ๋ คํ•˜๋ฉด ํ”ผ๊ณ ์ธ์ด ์ดฌ์˜๋œ ์‹œ๊ฐ„๊ณผ ์ผ์น˜ํ•œ๋‹ค. ์ฃผ17) ์—ฌ๊ถŒ์„ ์‚ฌ์ง„์œผ๋กœ ์ฐ์–ด ์ถœ๋ ฅํ•œ ๊ฒƒ์ด๋‹ค. ์ฃผ18) ๊ทธ๋Ÿฌ๋‚˜ ๊ณต์†Œ์™ธ 39, ๊ณต์†Œ์™ธ 40๊ณผ์˜ ํšŒํ•ฉ ๊ด€๋ จ ๊ณต์†Œ์‚ฌ์‹ค์ด ๋‹ค๋ฅธ ์ฆ๊ฑฐ๋“ค์— ์˜ํ•ด ์œ ์ฃ„๋กœ ์ธ์ •๋œ๋‹ค๋Š” ์ ์€ ๋’ค์—์„œ ๋ณธ๋‹ค. ์ฃผ19) ๋ณต๊ตฌํ•œ ํŒŒ์ผ๊ณผ ๊ฒ€์ฆ์šฉ ์‚ฌ๋ณธ ํŒŒ์ผ์˜ ํ•ด์‹œ๊ฐ’์ด ๋™์ผํ•œ ๊ฒƒ์„ ํ™•์ธํ•˜์˜€๋‹ค ์ฃผ20) ์‹ค์ œ๋กœ ๊ณต์†Œ์™ธ 67์€ ์ฆ์ธ์‹ ๋ฌธ ๋ฐ ๊ฒ€์ฆ๊ธฐ์ผ์—์„œ ์œ„ ํด๋Ÿฌ์Šคํ„ฐ ๋ถ€๋ถ„์„ ๋™์ผํ•˜๊ฒŒ ๋งž์ถ˜ ๋’ค ํ•ด์‹œ๊ฐ’์„ ์‚ฐ์ถœํ•˜์˜€๊ณ , ์ด๋Š” ๊ฒ€์‚ฌ๊ฐ€ ์ œ์ถœํ•œ ์›๋ณธํŒŒ์ผ์˜ ํ•ด์‹œ๊ฐ’๊ณผ ๋™์ผํ•˜์˜€๋‹ค. ์ฃผ21) ์ด๋Š” โ€˜๊ฒ€์ฆโ€™์˜ ์ผ์ข…์œผ๋กœ ๋ณด์•„์•ผ ํ•  ๊ฒƒ์ด๋‹ค. ์ฃผ22) ์‹ค์ œ๋กœ ์ด ๋‚ ์˜ ์œ„์™€ ๊ฐ™์€ ๋ชฐ๋ž˜ ์นด๋ฉ”๋ผ ์ดฌ์˜์œผ๋กœ ์ฑ„์ฆํ•œ ๊ฒฐ๊ณผ๋Š”, ํ”ผ๊ณ ์ธ์ด ํƒœ๋ธ”๋ฆฟ PC ๋‚ด์— ์ด์ ํ‘œํ˜„๋ฌผ์„ ์†Œ์ง€ํ•˜์˜€๋‹ค๋Š” ๋ฒ”ํ–‰์ผ ๋ฟ์œผ๋กœ์„œ, ๊ทธ๊ฒƒ์ด ์œ ์ผํ•œ ์ฆ๊ฑฐ์ด์ž ํฌ์ฐฉ๋œ ๋ฒ”ํ–‰์ด๋‹ค. ์ฃผ23) ๊ฐœ์ธ์ •๋ณด ๋ณดํ˜ธ๋ฒ•์€ ์œ„์—์„œ ๋ณธ โ€˜๊ฐœ์ธ์ •๋ณด์ฒ˜๋ฆฌ์žโ€™๋Š” ๋ฌผ๋ก ์ด๊ณ  ์ œ59์กฐ์—์„œ โ€˜๊ฐœ์ธ์ •๋ณด๋ฅผ ์ฒ˜๋ฆฌํ•˜๊ฑฐ๋‚˜ ์ฒ˜๋ฆฌํ•˜์˜€๋˜ ์žโ€™์— ๋Œ€ํ•˜์—ฌ๋„ ์—…๋ฌด์ƒ ์•Œ๊ฒŒ ๋œ ๊ฐœ์ธ์ •๋ณด๋ฅผ ๋ˆ„์„คํ•˜๊ฑฐ๋‚˜ ๊ถŒํ•œ ์—†์ด ๋‹ค๋ฅธ ์‚ฌ๋žŒ์ด ์ด์šฉํ•˜๋„๋ก ์ œ๊ณตํ•˜๋Š” ํ–‰์œ„ ๋“ฑ์„ ๊ธˆ์ง€ํ•˜๊ณ  ์žˆ๋‹ค. ์ฃผ24) ๊ฐœ์ธ์ •๋ณด๋ฅผ ์‰ฝ๊ฒŒ ๊ฒ€์ƒ‰ํ•  ์ˆ˜ ์žˆ๋„๋ก ์ผ์ •ํ•œ ๊ทœ์น™์— ๋”ฐ๋ผ ์ฒด๊ณ„์ ์œผ๋กœ ๋ฐฐ์—ดํ•˜๊ฑฐ๋‚˜ ๊ตฌ์„ฑํ•œ ๊ฐœ์ธ์ •๋ณด์˜ ์ง‘ํ•ฉ๋ฌผ ์ฃผ25) ์˜์žฅ ๊ธฐ์žฌ์ƒ ์••์ˆ˜ยท์ˆ˜์ƒ‰ํ•  ๋ฌผ๊ฑด : โ€˜(์ด๋ฉ”์ผ ์ฃผ์†Œ ์ƒ๋žต)' ์ด๋ฉ”์ผ ๊ณ„์ •์— ๋Œ€ํ•˜์—ฌ 2008. 1. 1. ~ 2010. 7. 20.๊นŒ์ง€ ๋ฐ›์€ ํŽธ์ง€ํ•จ, ๋ณด๋‚ธ ํŽธ์ง€ํ•จ, ์ž„์‹œ ๋ณด๊ด€ํ•จ, ํœด์ง€ํ†ต, ์ŠคํŒธํŽธ์ง€ํ•จ ๋ฐ ๋ฉ”์ผ๋ณด๊ด€์„ ์œ„ํ•˜์—ฌ ์ž„์˜๋กœ ๋งŒ๋“ค์–ด ์‚ฌ์šฉ ์ค‘์ธ ๋ณด๊ด€ํ•จ, ๋ฉ”์ผ, ์ฃผ์†Œ๋ก, ์นดํŽ˜, ๋ธ”๋กœ๊ทธ, โ—โ—โ—โ—โ— ๋“ฑ์— ์ €์žฅ๋˜์–ด ์žˆ๋Š” ๊ตญ๊ฐ€๋ณด์•ˆ๋ฒ•์œ„๋ฐ˜ ๋“ฑ๊ณผ ๊ด€๋ จํ•œ ์ €์žฅ๋ฌผ(๋ฌธ๊ฑด, ๋™์˜์ƒ, ์‚ฌ์ง„ ๋“ฑ ๋ชจ๋‘ ํฌํ•จ) ์ผ์ฒด ์ฃผ26) ์ด๋Ÿฌํ•œ ์ทจ์ง€์—์„œ ๊ฐ•์ œ์ˆ˜์‚ฌ์˜ ์ผ์ข…์ธ ํ†ต์‹ ์ œํ•œ์กฐ์น˜์— ๊ด€ํ•˜์—ฌ ํ†ต์‹ ๋น„๋ฐ€๋ณดํ˜ธ๋ฒ• ์ œ9์กฐ ์ œ1ํ•ญ์€ ์›์น™์ ์ธ ํ†ต์‹ ์ œํ•œ์กฐ์น˜์˜ ์ฃผ์ฒด๋ฅผ โ€˜๊ฒ€์‚ฌ, ์‚ฌ๋ฒ•๊ฒฝ์ฐฐ๊ด€ ๋˜๋Š” ์ •๋ณด์ˆ˜์‚ฌ๊ธฐ๊ด€์˜ ์žฅโ€˜์œผ๋กœ ์ œํ•œํ•˜๊ณ  ์žˆ์œผ๋ฉด์„œ๋„, โ€™์ฒด์‹ ๊ด€์„œ ๊ธฐํƒ€ ๊ด€๋ จ๊ธฐ๊ด€ ๋“ฑ(ํ†ต์‹ ๊ธฐ๊ด€๋“ฑ)์— ๊ทธ ์ง‘ํ–‰์„ ์œ„ํƒํ•˜๊ฑฐ๋‚˜ ์ง‘ํ–‰์— ๊ด€ํ•œ ํ˜‘์กฐ๋ฅผ ์š”์ฒญํ•  ์ˆ˜ ์žˆ๋‹คโ€˜๊ณ  ๊ทœ์ •ํ•˜๊ณ  ์žˆ๋‹ค. ์ฃผ27) ์ฆ ์ œ126ํ˜ธ USB ๋‚ด ์Šคํ…Œ๊ฐ€๋…ธ๊ทธ๋ผํ”ผ ํŒŒ์ผ(info.docx)์„ ๋ณตํ˜ธํ™”ํ•œ ๋ถํ•œ ์ง€๋ น๋ฌธ(2011. 11.) ์ถœ๋ ฅ๋ฌผ 1๋ถ€ ์ฃผ28) ์ฆ ์ œ126ํ˜ธ USB ๋‚ด ์Šคํ…Œ๊ฐ€๋…ธ๊ทธ๋ผํ”ผ ํŒŒ์ผ(toyou.docx)์„ ๋ณตํ˜ธํ™”ํ•œ ๋Œ€๋ถ ๋ณด๊ณ ๋ฌธ ๋ฌธ๊ฑด(2011. 12. 1.์ž) ์ถœ๋ ฅ๋ฌผ 1๋ถ€ ์ฃผ29) โ€˜toyou7-7.docx' ํŒŒ์ผ์„ ๋ณตํ˜ธํ•œ ๊ฒฐ๊ณผ ์ถœ๋ ฅ๋ฌผ(2013. 7. 7.์ž ๋Œ€๋ถ๋ณด๊ณ ๋ฌธ) 1๋ถ€ ์ฃผ30) ์ฆ ์ œ123ํ˜ธ Micro SD์นด๋“œ ๋‚ด ์Šคํ…Œ๊ฐ€๋…ธ๊ทธ๋ผํ”ผ ๋ฌธ์„œํŒŒ์ผ(toyou11-12.docx)์„ ๋ณตํ˜ธํ™”ํ•œ ๋Œ€๋ถ ๋ณด๊ณ ๋ฌธ ์ถœ๋ ฅ๋ฌผ 1๋ถ€ ์ฃผ31) ๊ณต์†Œ์™ธ 40(๋˜๋Š” ๊ฐ€๋ช… โ–ทโ–ทโ–ท, โ–ทโ™คโ™ค)์€ ๋’ค์—์„œ ๋ณผ 2015. 4. 5.๊ฒฝ ํšŒํ•ฉ๊ณผ ๊ด€๊ณ„๋˜๋Š” ์‚ฌ๋žŒ์ด์ง€๋งŒ, ์—ฌ๊ธฐ์—์„œ ํ•จ๊ป˜ ๋ณธ๋‹ค. ์ฃผ32) ์ด ๋ถ€๋ถ„ ๋ฒ•๋ฆฌ๋Š” ๋’ค์—์„œ ๋ณผ ๊ฐ™์€ ์ฃ„๋ช…์˜ ๋‹ค๋ฅธ ๊ณต์†Œ์‚ฌ์‹ค์—์„œ๋„ ๊ณตํžˆ ์ ์šฉ๋œ๋‹ค. ์ฃผ33) ํ•œํŽธ 2์ธ ์ด์ƒ์ด ๊ณต๋ชจํ•˜์—ฌ ๋ฒ”์ฃ„์— ๊ณต๋™ ๊ฐ€๊ณตํ•˜๋Š” ๊ณต๋ฒ”๊ด€๊ณ„์—์„œ ๊ณต๋ชจ๋Š” ๋ฒ•๋ฅ ์ƒ ์–ด๋–ค ์ •ํ˜•์„ ์š”๊ตฌํ•˜๋Š” ๊ฒƒ์ด ์•„๋‹ˆ๊ณ  ๋ฒ”์ฃ„๋ฅผ ์‹คํ˜„ํ•˜๋ ค๋Š” ์˜์‚ฌ์˜ ๊ฒฐํ•ฉ๋งŒ ์žˆ์œผ๋ฉด ๋˜๋Š” ๊ฒƒ์œผ๋กœ์„œ, ๋น„๋ก ์ „์ฒด์˜ ๋ชจ์˜๊ณผ์ •์ด ์—†์—ˆ๋‹ค๊ณ  ํ•˜๋”๋ผ๋„ ์ˆ˜์ธ ์‚ฌ์ด์— ์ˆœ์ฐจ์ ์œผ๋กœ ๋˜๋Š” ์•”๋ฌต์ ์œผ๋กœ ์ƒํ†ตํ•˜์—ฌ ๊ทธ ์˜์‚ฌ์˜ ๊ฒฐํ•ฉ์ด ์ด๋ฃจ์–ด์ง€๋ฉด ๊ณต๋ชจ๊ด€๊ณ„๊ฐ€ ์„ฑ๋ฆฝํ•œ๋‹ค ํ•  ๊ฒƒ์ด๊ณ , ์ด๋Ÿฌํ•œ ๊ณต๋ชจ๊ฐ€ ์ด๋ฃจ์–ด์ง„ ์ด์ƒ ์‹คํ–‰ํ–‰์œ„์— ๊ด€์—ฌํ•˜์ง€ ์•„๋‹ˆํ•œ ์ž๋ผ๋„ ๋‹ค๋ฅธ ๊ณต๋ชจ์ž์˜ ํ–‰์œ„์— ๋Œ€ํ•˜์—ฌ ๊ณต๋™์ •๋ฒ”์œผ๋กœ์„œ์˜ ํ˜•์‚ฌ์ฑ…์ž„์„ ์ง„๋‹ค(๋Œ€๋ฒ•์› 2013. 11. 14. ์„ ๊ณ  2013๋„7476 ํŒ๊ฒฐ ๋“ฑ ์ฐธ์กฐ). ์ฃผ34) ๊ฐ€๋ช… โ–ทโ–ทโ–ท, โ–ทโ™คโ™ค์ด๋‹ค. ๊ทธ๊ฐ€ ๋ถํ•œ ๊ณต์ž‘์›์ด๋ผ๋Š” ์ ์— ๋Œ€ํ•œ ๊ณต์†Œ์™ธ 23์˜ ์ฆ์–ธ์„ ๋ฏฟ์„ ์ˆ˜ ์žˆ๋‹ค๋Š” ์ ์€ ์•ž์„œ ๋ณด์•˜๋‹ค. ์ฃผ35) ๊ณต์†Œ์™ธ 50์˜ ์ฆ์–ธ, ๊ฐ์ •์„œ์˜ ๊ธฐ์žฌ, ์ด ๋ฒ•์›์˜ ๊ฒ€์ฆ๊ฒฐ๊ณผ ๋“ฑ์— ์˜ํ•˜๋ฉด, ์ค‘๊ตญ ๋Œ€๋ จ๊ณผ ๋ฒ ํŠธ๋‚จ ํ˜ธ์น˜๋ฏผ์— โ˜†์ง€๋„์›์„ ์ˆ˜ํ–‰ํ•˜์—ฌ ์˜จ ์‚ฌ๋žŒ๊ณผ ๋™์ผํ•œ ์‚ฌ๋žŒ์ด๋‹ค. ์ฃผ36) ์ด์™ธ์—๋„ โ€˜๋‚ด๋…„ ๊ฒฝ๋‚จ์ง€์—ญ์ฑ…๋™์ง€์— ๋Œ€ํ•œ ๋ณธ์‚ฌ ๋ฉด๋‹ด์„ ๊ณ„ํšํ•˜๊ณ  ์žˆ์Œ. ๋น„๋ก ๋ณด์•ˆ๊ด€์ฐฐ์ค‘์ด์ง€๋งŒ, ๊ฐ€์กฑ ์—ฌํ–‰(๋ถ€์ธ๊ณผ ๋”ธ)์„ ๋นŒ๋ฏธ๋กœ ๋น„์ž๋ฅผ ๋งŒ๋“ค๊ณ , ๋ฒ ํŠธ๋‚จ์œผ๋กœ ์—ฌํ–‰ํ•ด ํ•˜๋ฃจ์ •๋„ ๊ฐ€์กฑ๊ณผ ๋–จ์–ด์ ธ ๋ณธ์‚ฌ์™€ ๋ฉด๋‹ดํ•˜๋Š” ๊ฒƒ์ž„. ์ด์ œ๋Š” ๋ฉด๋‹ดํ•ด๋„ ์ข‹์€ ์กฐ๊ฑด์ด ๋งˆ๋ จ๋๋‹ค๊ณ  ํŒ๋‹จ๋จ. ๋ฌด๋ฆฌํ•˜์ง€ ์•Š๊ฒŒ ๊ตฌ์ฒด์ƒํ™ฉ ๋ณด๋ฉด์„œ ๋ณธ์‚ฌ์˜ ์˜๊ฒฌ์„ ๋“ฃ๊ณ  ์ตœ์ข… ํŒ๋‹จํ•˜๊ฒ ์Œ. ํ•„์š”ํ•˜๋‹ค๋ฉด ์ด์ฑ…์ด ๋ณด์ด์ง€ ์•Š๊ฒŒ ์—ญํ• ํ•˜๊ฒ ์Œโ€™์ด๋ผ๋Š” ๊ธฐ์žฌ๋„ ์žˆ๋‹ค. ์ฃผ37) ์Šคํ…Œ๊ฐ€๋…ธ๊ทธ๋ผํ”ผ ํ”„๋กœ๊ทธ๋žจ์˜ ๊ตฌ์ฒด์ ์ธ ์‚ฌ์šฉ๋ฐฉ๋ฒ•, ๋น„๋ฐ€๋ฒˆํ˜ธ, ์ด๋ฉ”์ผ ์ฃผ์†Œ์˜ ๊ต์ฒด์ด์šฉ ๋“ฑ์— ๊ด€ํ•œ ์•ฝ์†์‚ฌํ•ญ, ์‹ ๋…„๋ฉ”์‹œ์ง€์˜ ์†ก๋ถ€ ์š”์ฒญ ๋“ฑ์ด ๋‹ด๊ฒจ ์žˆ๋‹ค. ๋น„๋ฐ€๋ฒˆํ˜ธ๋ฅผ โ€œA์˜ ์ƒ๋…„(1963)์œผ๋กœโ€ ์ •ํ•˜๊ธฐ๋กœ ํ•˜์˜€๋Š”๋ฐ, ์ด๋Š” ํ”ผ๊ณ ์ธ์ด SNS ๊ฐ€์ž…์ž์ •๋ณด์— 1963๋…„์ƒ์œผ๋กœ ๊ธฐ์žฌํ•œ ๊ฒƒ๊ณผ ๋ถ€ํ•ฉํ•œ๋‹ค. ์ฃผ38) ์ด์ ํ‘œํ˜„๋ฌผ์„ ์ €์žฅํ•˜์—ฌ ์†Œ์ง€ํ•œ ์‹œ๊ธฐ, ์žฅ์†Œ, ์ €์žฅ๋งค์ฒด, ์†Œ์ง€๋ฐฉ๋ฒ• ๋“ฑ์— ๋”ฐ๋ผ ๊ฒ€์‚ฌ๊ฐ€ ๊ณต์†Œ์ œ๊ธฐํ•œ ๋Œ€๋กœ ๊ฐ ๋ฌธ๊ฑด์ด๋‚˜ ๋งค์ฒด๋ณ„๋กœ 1์ฃ„๊ฐ€ ์„ฑ๋ฆฝํ•˜๋Š” ๊ฒƒ์œผ๋กœ ๋ด„์ด ์ƒ๋‹นํ•˜๋‹ค. ์ฃผ39) ๊ฒ€์‚ฌ๋Š” [๋ณ„์ง€ 1] ๋ชฐ์ˆ˜๋Œ€์ƒ ์••์ˆ˜๋ฌผ๋ชฉ๋ก ๊ธฐ์žฌ ์••์ˆ˜๋ฌผ๋“ค ์ค‘ ์ˆœ๋ฒˆ 19 ๋‚ด์ง€ 30์— ๊ด€ํ•˜์—ฌ๋„ ๋ชฐ์ˆ˜๋ฅผ ๊ตฌํ˜•ํ•˜์˜€์œผ๋‚˜, ์ด ์••์ˆ˜๋ฌผ๋“ค์€ ์ด ์‚ฌ๊ฑด์—์„œ ์œ ์ฃ„๋กœ ์ธ์ •๋˜๋Š” ๋‹นํ•ด ๋ฒ”์ฃ„ํ–‰์œ„์— ์ œ๊ณตํ•˜๊ฑฐ๋‚˜ ๊ทธ๋กœ ์ธํ•˜์—ฌ ์ทจ๋“ํ•œ ๋ฌผ๊ฑด์— ํ•ด๋‹นํ•˜์ง€ ์•„๋‹ˆํ•˜๋ฏ€๋กœ ๋ชฐ์ˆ˜ํ•˜์ง€ ์•„๋‹ˆํ•œ๋‹ค(๋Œ€๋ฒ•์› 2008. 2. 14. ์„ ๊ณ  2007๋„10034 ํŒ๊ฒฐ ์ฐธ์กฐ). ์ฃผ40) 2016๊ณ ํ•ฉ558 ์ฆ๊ฑฐ๋ชฉ๋ก ์ˆœ๋ฒˆ 484[ํ”ผ๊ณ ์ธ์ด ๊ณต์†Œ์™ธ 5์—๊ฒŒ ํƒœ๋ธ”๋ฆฟ PC๋ฅผ ํ†ตํ•ด ์žฌํ•˜๋‹ฌํ•œ ๋ถ์ง€๋ น๋ฌธยท์‚ฌ์ƒ๊ต์–‘์ž๋ฃŒ๋‚ด์šฉ(2013. 11. 2.) 1๋ถ€]์™€ ์ˆœ๋ฒˆ 485(2013. 11. 2. ํ”ผ๊ณ ์ธยท๊ณต์†Œ์™ธ 5 ์ ‘์„ ์žฅ๋ฉด ์ฑ„์ฆ CCTV ์บก์ณ ์‚ฌ์ง„ 51๋งค)

๋Œ“๊ธ€ 0๊ฐœ
๋กœ๊ทธ์ธ ํ•˜์‹œ๋ฉด, ํŒ๋ก€ ๋Œ“๊ธ€์„ ๋“ฑ๋กํ•˜์‹ค ์ˆ˜ ์žˆ์Šต๋‹ˆ๋‹ค.
๋„ค์ด๋ฒ„ ๊ฒ€์ƒ‰ ๋ธ”๋กœ๊ทธ   ์นดํŽ˜   ๋‰ด์Šค   ์›น์‚ฌ์ดํŠธ

[ ์†Œ์†ก๊ฒฝ๊ณผ ]


[ ์ฐธ์กฐ์กฐ๋ฌธ ]


[ ๋ฒ•๊ด€/๋Œ€๋ฒ•๊ด€ ํƒœ๊ทธ]


[ ๋ฒ•๋ฌด๋ฒ•์ธ ํƒœ๊ทธ]


[ ์‚ฌ๊ฑด๋ช… ํƒœ๊ทธ]


[ ์ฐธ์กฐ์กฐ๋ฌธ ํƒœ๊ทธ ]

ํ˜•์‚ฌ์†Œ์†ก๊ทœ์น™ ์ œ118์กฐ  ํ˜•์‚ฌ์†Œ์†ก๋ฒ• ์ œ254์กฐ  ํ—Œ๋ฒ• ์ œ3์กฐ  ๋‚จ๋ถ๊ด€๊ณ„ ๋ฐœ์ „์— ๊ด€ํ•œ ๋ฒ•๋ฅ  ์ œ3์กฐ  ํ—Œ๋ฒ• ์ œ4์กฐ  ํ—Œ๋ฒ• ์ œ5์กฐ  ํ—Œ๋ฒ• ์ œ27์กฐ  ํ—Œ๋ฒ• ์ œ109์กฐ  ๋ฒ•์›์กฐ์ง๋ฒ• ์ œ57์กฐ  ํ˜•์‚ฌ์†Œ์†ก๋ฒ• ์ œ165์กฐ  ํ˜•์‚ฌ์†Œ์†ก๊ทœ์น™ ์ œ84์กฐ  ๊ตญ๊ฐ€์ •๋ณด์›์ง์›๋ฒ• ์ œ17์กฐ  ํ˜•์‚ฌ์†Œ์†ก๋ฒ• ์ œ123์กฐ  ํ˜•์‚ฌ์†Œ์†ก๋ฒ• ์ œ125์กฐ  ํ˜•์‚ฌ์†Œ์†ก๊ทœ์น™ ์ œ101์กฐ  ํ˜•์‚ฌ์†Œ์†ก๋ฒ• ์ œ219์กฐ  ํ˜•์‚ฌ์†Œ์†ก๋ฒ• ์ œ120์กฐ  ํ˜•์‚ฌ์†Œ์†ก๋ฒ• ์ œ266์กฐ  ์ •๋ณดํ†ต์‹ ๋ง ์ด์šฉ์ด‰์ง„ ๋ฐ ์ •๋ณด๋ณดํ˜ธ ๋“ฑ์— ๊ด€ํ•œ ๋ฒ•๋ฅ  ์ œ48์กฐ  ํ˜•์‚ฌ์†Œ์†ก๋ฒ• ์ œ216์กฐ  ํ˜•์‚ฌ์†Œ์†ก๋ฒ• ์ œ218์กฐ  ๊ฐœ์ธ์ •๋ณด ๋ณดํ˜ธ๋ฒ• ์ œ2์กฐ  ๊ฐœ์ธ์ •๋ณด ๋ณดํ˜ธ๋ฒ• ์ œ25์กฐ  ๊ฐœ์ธ์ •๋ณด ๋ณดํ˜ธ๋ฒ• ์ œ18์กฐ  ๊ฐœ์ธ์ •๋ณด ๋ณดํ˜ธ๋ฒ• ์‹œํ–‰๋ น ์ œ2์กฐ  ํ˜•์‚ฌ์†Œ์†ก๋ฒ• ์ œ118์กฐ  ํ˜•์‚ฌ์†Œ์†ก๋ฒ• ์ œ313์กฐ  ๊ตญ๊ฐ€๋ณด์•ˆ๋ฒ• ์ œ7์กฐ  ๊ตญ๊ฐ€๋ณด์•ˆ๋ฒ• ์ œ8์กฐ  ๊ตญ๊ฐ€๋ณด์•ˆ๋ฒ• ์ œ5์กฐ  ํ˜•๋ฒ• ์ œ30์กฐ  ๊ตญ๊ฐ€๋ณด์•ˆ๋ฒ• ์ œ9์กฐ  ๊ตญ๊ฐ€๋ณด์•ˆ๋ฒ• ์ œ3์กฐ  ํ˜•๋ฒ• ์ œ37์กฐ  ํ˜•๋ฒ• ์ œ38์กฐ  ํ˜•๋ฒ• ์ œ50์กฐ  ๊ตญ๊ฐ€๋ณด์•ˆ๋ฒ• ์ œ14์กฐ  ํ˜•๋ฒ• ์ œ48์กฐ  ๊ตญ๊ฐ€๋ณด์•ˆ๋ฒ• ์ œ15์กฐ  ํ˜•์‚ฌ์†Œ์†ก๋ฒ• ์ œ325์กฐ 

[ ์œ ์‚ฌ ํŒ๋ก€ ]


[ ๊ณต์œ ํ•˜๊ธฐ ]

 

๊ฐœ์ธ์ •๋ณด์ฒ˜๋ฆฌ๋ฐฉ์นจ     ์‚ฌ์šฉ์ž ์นดํŽ˜